26.06.2024

Π‘Ρ…Π΅ΠΌΡ‹ соСдинСния рСзисторов: Π‘Ρ…Π΅ΠΌΡ‹ ΠΏΠ°Ρ€Π°Π»Π»Π΅Π»ΡŒΠ½ΠΎΠ³ΠΎ ΠΈ ΠΏΠΎΡΠ»Π΅Π΄ΠΎΠ²Π°Ρ‚Π΅Π»ΡŒΠ½ΠΎΠ³ΠΎ соСдинСния рСзисторов

Π‘ΠΎΠ΄Π΅Ρ€ΠΆΠ°Π½ΠΈΠ΅

Π‘Ρ…Π΅ΠΌΡ‹ ΠΏΠ°Ρ€Π°Π»Π»Π΅Π»ΡŒΠ½ΠΎΠ³ΠΎ ΠΈ ΠΏΠΎΡΠ»Π΅Π΄ΠΎΠ²Π°Ρ‚Π΅Π»ΡŒΠ½ΠΎΠ³ΠΎ соСдинСния рСзисторов

ΠŸΠΎΡΠ»Π΅Π΄ΠΎΠ²Π°Ρ‚Π΅Π»ΡŒΠ½ΠΎΠ΅ ΠΈ ΠΏΠ°Ρ€Π°Π»Π»Π΅Π»ΡŒΠ½ΠΎΠ΅ соСдинСниС рСзисторов

ΠŸΠΎΡΠ»Π΅Π΄ΠΎΠ²Π°Ρ‚Π΅Π»ΡŒΠ½ΠΎΠ΅ соСдинСниС рСзисторов

ΠŸΠΎΡΠ»Π΅Π΄ΠΎΠ²Π°Ρ‚Π΅Π»ΡŒΠ½ΠΎΠ΅ соСдинСниС – это соСдинСниС Π΄Π²ΡƒΡ… ΠΈΠ»ΠΈ Π±ΠΎΠ»Π΅Π΅ рСзисторов Π² Ρ„ΠΎΡ€ΠΌΠ΅ Ρ†Π΅ΠΏΠΈ, Π² ΠΊΠΎΡ‚ΠΎΡ€ΠΎΠΉ ΠΊΠ°ΠΆΠ΄Ρ‹ΠΉ ΠΎΡ‚Π΄Π΅Π»ΡŒΠ½Ρ‹ΠΉ рСзистор соСдиняСтся с Π΄Ρ€ΡƒΠ³ΠΈΠΌ ΠΎΡ‚Π΄Π΅Π»ΡŒΠ½Ρ‹ΠΌ рСзистором Ρ‚ΠΎΠ»ΡŒΠΊΠΎ Π² ΠΎΠ΄Π½ΠΎΠΉ Ρ‚ΠΎΡ‡ΠΊΠ΅.

ΠžΠ±Ρ‰Π΅Π΅ сопротивлСниС R ΠΎΠ±Ρ‰

ΠŸΡ€ΠΈ Ρ‚Π°ΠΊΠΎΠΌ соСдинСнии, Ρ‡Π΅Ρ€Π΅Π· всС рСзисторы ΠΏΡ€ΠΎΡ…ΠΎΠ΄ΠΈΡ‚ ΠΎΠ΄ΠΈΠ½ ΠΈ Ρ‚ΠΎΡ‚ ΠΆΠ΅ элСктричСский Ρ‚ΠΎΠΊ. Π§Π΅ΠΌ большС элСмСнтов Π½Π° Π΄Π°Π½Π½ΠΎΠΌ участкС элСктричСской Ρ†Π΅ΠΏΠΈ, Ρ‚Π΅ΠΌ Β«Ρ‚Ρ€ΡƒΠ΄Π½Π΅Π΅Β» Ρ‚ΠΎΠΊΡƒ ΠΏΡ€ΠΎΡ‚Π΅ΠΊΠ°Ρ‚ΡŒ Ρ‡Π΅Ρ€Π΅Π· Π½Π΅Π³ΠΎ. Π‘Π»Π΅Π΄ΠΎΠ²Π°Ρ‚Π΅Π»ΡŒΠ½ΠΎ, ΠΏΡ€ΠΈ ΠΏΠΎΡΠ»Π΅Π΄ΠΎΠ²Π°Ρ‚Π΅Π»ΡŒΠ½ΠΎΠΌ соСдинСнии рСзисторов ΠΈΡ… ΠΎΠ±Ρ‰Π΅Π΅ сопротивлСниС увСличиваСтся, ΠΈ ΠΎΠ½ΠΎ Ρ€Π°Π²Π½ΠΎ суммС всСх сопротивлСний.

НапряТСниС ΠΏΡ€ΠΈ ΠΏΠΎΡΠ»Π΅Π΄ΠΎΠ²Π°Ρ‚Π΅Π»ΡŒΠ½ΠΎΠΌ соСдинСнии

НапряТСниС ΠΏΡ€ΠΈ ΠΏΠΎΡΠ»Π΅Π΄ΠΎΠ²Π°Ρ‚Π΅Π»ΡŒΠ½ΠΎΠΌ соСдинСнии распрСдСляСтся Π½Π° ΠΊΠ°ΠΆΠ΄Ρ‹ΠΉ рСзистор согласно Π·Π°ΠΊΠΎΠ½Ρƒ Ома:

Π’.Π΅ Ρ‡Π΅ΠΌ большСС сопротивлСниС рСзистора, Ρ‚Π΅ΠΌ большСС напряТСниС Π½Π° Π½Π΅Π³ΠΎ ΠΏΠ°Π΄Π°Π΅Ρ‚.

ΠŸΠ°Ρ€Π°Π»Π»Π΅Π»ΡŒΠ½ΠΎΠ΅ соСдинСниС рСзисторов

ΠŸΠ°Ρ€Π°Π»Π»Π΅Π»ΡŒΠ½ΠΎΠ΅ соСдинСниС – это соСдинСниС, ΠΏΡ€ΠΈ ΠΊΠΎΡ‚ΠΎΡ€ΠΎΠΌ рСзисторы ΡΠΎΠ΅Π΄ΠΈΠ½ΡΡŽΡ‚ΡΡ ΠΌΠ΅ΠΆΠ΄Ρƒ собой ΠΎΠ±ΠΎΠΈΠΌΠΈ ΠΊΠΎΠ½Ρ‚Π°ΠΊΡ‚Π°ΠΌΠΈ. Π’ Ρ€Π΅Π·ΡƒΠ»ΡŒΡ‚Π°Ρ‚Π΅ ΠΊ ΠΎΠ΄Π½ΠΎΠΉ Ρ‚ΠΎΡ‡ΠΊΠ΅ (элСктричСскому ΡƒΠ·Π»Ρƒ) ΠΌΠΎΠΆΠ΅Ρ‚ Π±Ρ‹Ρ‚ΡŒ присоСдинСно нСсколько рСзисторов.

ΠžΠ±Ρ‰Π΅Π΅ сопротивлСниС R ΠΎΠ±Ρ‰

ΠŸΡ€ΠΈ Ρ‚Π°ΠΊΠΎΠΌ соСдинСнии, Ρ‡Π΅Ρ€Π΅Π· ΠΊΠ°ΠΆΠ΄Ρ‹ΠΉ рСзистор ΠΏΠΎΡ‚Π΅Ρ‡Π΅Ρ‚ ΠΎΡ‚Π΄Π΅Π»ΡŒΠ½Ρ‹ΠΉ Ρ‚ΠΎΠΊ. Π‘ΠΈΠ»Π° Π΄Π°Π½Π½ΠΎΠ³ΠΎ Ρ‚ΠΎΠΊΠ° Π±ΡƒΠ΄Π΅Ρ‚ ΠΎΠ±Ρ€Π°Ρ‚Π½ΠΎ ΠΏΡ€ΠΎΠΏΠΎΡ€Ρ†ΠΈΠΎΠ½Π°Π»ΡŒΠ½Π° ΡΠΎΠΏΡ€ΠΎΡ‚ΠΈΠ²Π»Π΅Π½ΠΈΡŽ рСзистора. Π’ Ρ€Π΅Π·ΡƒΠ»ΡŒΡ‚Π°Ρ‚Π΅ общая ΠΏΡ€ΠΎΠ²ΠΎΠ΄ΠΈΠΌΠΎΡΡ‚ΡŒ Ρ‚Π°ΠΊΠΎΠ³ΠΎ участка элСктричСской Ρ†Π΅ΠΏΠΈ увСличиваСтся, Π° ΠΎΠ±Ρ‰Π΅Π΅ сопротивлСниС Π² свою ΠΎΡ‡Π΅Ρ€Π΅Π΄ΡŒ ΡƒΠΌΠ΅Π½ΡŒΡˆΠ°Π΅Ρ‚ΡΡ.

Π’Π°ΠΊΠΈΠΌ ΠΎΠ±Ρ€Π°Π·ΠΎΠΌ, ΠΏΡ€ΠΈ ΠΏΠ°Ρ€Π°Π»Π»Π΅Π»ΡŒΠ½ΠΎΠΌ подсоСдинСнии рСзисторов с Ρ€Π°Π·Π½Ρ‹ΠΌ сопротивлСниСм, ΠΎΠ±Ρ‰Π΅Π΅ сопротивлСниС Π±ΡƒΠ΄Π΅Ρ‚ всСгда мСньшС значСния самого малСнького ΠΎΡ‚Π΄Π΅Π»ΡŒΠ½ΠΎΠ³ΠΎ рСзистора.

Π€ΠΎΡ€ΠΌΡƒΠ»Π° ΠΎΠ±Ρ‰Π΅ΠΉ проводимости ΠΏΡ€ΠΈ ΠΏΠ°Ρ€Π°Π»Π»Π΅Π»ΡŒΠ½ΠΎΠΌ соСдинСнии рСзисторов:

Π€ΠΎΡ€ΠΌΡƒΠ»Π° эквивалСнтного ΠΎΠ±Ρ‰Π΅Π³ΠΎ сопротивлСния ΠΏΡ€ΠΈ ΠΏΠ°Ρ€Π°Π»Π»Π΅Π»ΡŒΠ½ΠΎΠΌ соСдинСнии рСзисторов:

Для Π΄Π²ΡƒΡ… ΠΎΠ΄ΠΈΠ½Π°ΠΊΠΎΠ²Ρ‹Ρ… рСзисторов ΠΎΠ±Ρ‰Π΅Π΅ сопротивлСниС Π±ΡƒΠ΄Π΅Ρ‚ Ρ€Π°Π²Π½ΠΎ ΠΏΠΎΠ»ΠΎΠ²ΠΈΠ½Π΅ ΠΎΠ΄Π½ΠΎΠ³ΠΎ ΠΎΡ‚Π΄Π΅Π»ΡŒΠ½ΠΎΠ³ΠΎ рСзистора:

БоотвСтствСнно, для n ΠΎΠ΄ΠΈΠ½Π°ΠΊΠΎΠ²Ρ‹Ρ… рСзисторов ΠΎΠ±Ρ‰Π΅Π΅ сопротивлСниС Π±ΡƒΠ΄Π΅Ρ‚ Ρ€Π°Π²Π½ΠΎ Π·Π½Π°Ρ‡Π΅Π½ΠΈΡŽ ΠΎΠ΄Π½ΠΎΠ³ΠΎ рСзистора, Ρ€Π°Π·Π΄Π΅Π»Π΅Π½Π½ΠΎΠ³ΠΎ Π½Π° n.

НапряТСниС ΠΏΡ€ΠΈ ΠΏΠ°Ρ€Π°Π»Π»Π΅Π»ΡŒΠ½ΠΎΠΌ соСдинСнии

НапряТСниС ΠΌΠ΅ΠΆΠ΄Ρƒ Ρ‚ΠΎΡ‡ΠΊΠ°ΠΌΠΈ A ΠΈ B являСтся ΠΊΠ°ΠΊ ΠΎΠ±Ρ‰ΠΈΠΌ напряТСниСм для всСго участка Ρ†Π΅ΠΏΠΈ, Ρ‚Π°ΠΊ ΠΈ напряТСниСм, ΠΏΠ°Π΄Π°ΡŽΡ‰ΠΈΠΌ Π½Π° ΠΊΠ°ΠΆΠ΄Ρ‹ΠΉ рСзистор Π² ΠΎΡ‚Π΄Π΅Π»ΡŒΠ½ΠΎΡΡ‚ΠΈ. ΠŸΠΎΡΡ‚ΠΎΠΌΡƒ ΠΏΡ€ΠΈ ΠΏΠ°Ρ€Π°Π»Π»Π΅Π»ΡŒΠ½ΠΎΠΌ соСдинСнии Π½Π° всС рСзисторы ΡƒΠΏΠ°Π΄Π΅Ρ‚ ΠΎΠ΄ΠΈΠ½Π°ΠΊΠΎΠ²ΠΎΠ΅ напряТСниС.

ЭлСктричСский Ρ‚ΠΎΠΊ ΠΏΡ€ΠΈ ΠΏΠ°Ρ€Π°Π»Π»Π΅Π»ΡŒΠ½ΠΎΠΌ соСдинСнии

Π§Π΅Ρ€Π΅Π· ΠΊΠ°ΠΆΠ΄Ρ‹ΠΉ рСзистор Ρ‚Π΅Ρ‡Π΅Ρ‚ Ρ‚ΠΎΠΊ, сила ΠΊΠΎΡ‚ΠΎΡ€ΠΎΠ³ΠΎ ΠΎΠ±Ρ€Π°Ρ‚Π½ΠΎ ΠΏΡ€ΠΎΠΏΠΎΡ€Ρ†ΠΈΠΎΠ½Π°Π»ΡŒΠ½Π° ΡΠΎΠΏΡ€ΠΎΡ‚ΠΈΠ²Π»Π΅Π½ΠΈΡŽ рСзистора. Для Ρ‚ΠΎΠ³ΠΎ Ρ‡Ρ‚ΠΎΠ±Ρ‹ ΡƒΠ·Π½Π°Ρ‚ΡŒ ΠΊΠ°ΠΊΠΎΠΉ Ρ‚ΠΎΠΊ Ρ‚Π΅Ρ‡Π΅Ρ‚ Ρ‡Π΅Ρ€Π΅Π· ΠΎΠΏΡ€Π΅Π΄Π΅Π»Π΅Π½Π½Ρ‹ΠΉ рСзистор, ΠΌΠΎΠΆΠ½ΠΎ Π²ΠΎΡΠΏΠΎΠ»ΡŒΠ·ΠΎΠ²Π°Ρ‚ΡŒΡΡ Π·Π°ΠΊΠΎΠ½ΠΎΠΌ Ома:

БмСшанноС соСдинСниС рСзисторов

Π‘ΠΌΠ΅ΡˆΠ°Π½Π½Ρ‹ΠΌ соСдинСниСм Π½Π°Π·Ρ‹Π²Π°ΡŽΡ‚ участок Ρ†Π΅ΠΏΠΈ, Π³Π΄Π΅ Ρ‡Π°ΡΡ‚ΡŒ рСзисторов ΡΠΎΠ΅Π΄ΠΈΠ½ΡΡŽΡ‚ΡΡ ΠΌΠ΅ΠΆΠ΄Ρƒ собой ΠΏΠΎΡΠ»Π΅Π΄ΠΎΠ²Π°Ρ‚Π΅Π»ΡŒΠ½ΠΎ, Π° Ρ‡Π°ΡΡ‚ΡŒ ΠΏΠ°Ρ€Π°Π»Π»Π΅Π»ΡŒΠ½ΠΎ. Π’ свою ΠΎΡ‡Π΅Ρ€Π΅Π΄ΡŒ, смСшанноС соСдинСниС Π±Ρ‹Π²Π°Π΅Ρ‚ ΠΏΠΎΡΠ»Π΅Π΄ΠΎΠ²Π°Ρ‚Π΅Π»ΡŒΠ½ΠΎΠ³ΠΎ ΠΈ ΠΏΠ°Ρ€Π°Π»Π»Π΅Π»ΡŒΠ½ΠΎΠ³ΠΎ Ρ‚ΠΈΠΏΠΎΠ².

ΠžΠ±Ρ‰Π΅Π΅ сопротивлСниС R ΠΎΠ±Ρ‰

Для Ρ‚ΠΎΠ³ΠΎ Ρ‡Ρ‚ΠΎΠ±Ρ‹ ΠΏΠΎΡΡ‡ΠΈΡ‚Π°Ρ‚ΡŒ ΠΎΠ±Ρ‰Π΅Π΅ сопротивлСниС смСшанного соСдинСния:

o ЦСпь Ρ€Π°Π·Π±ΠΈΠ²Π°ΡŽΡ‚ Π½Π° участки с Ρ‚ΠΎΠ»ΡŒΠΊΠΎ ΠΏΠ°Ρ€Π΅Π»Π»Π΅Π»ΡŒΠ½Ρ‹ΠΌ ΠΈΠ»ΠΈ Ρ‚ΠΎΠ»ΡŒΠΊΠΎ ΠΏΠΎΡΠ»Π΅Π΄ΠΎΠ²Π°Ρ‚Π΅Π»ΡŒΠ½Ρ‹ΠΌ соСдинСниСм. o Π’Ρ‹Ρ‡ΠΈΡΠ»ΡΡŽΡ‚ ΠΎΠ±Ρ‰Π΅Π΅ сопротивлСниС для ΠΊΠ°ΠΆΠ΄ΠΎΠ³ΠΎ ΠΎΡ‚Π΄Π΅Π»ΡŒΠ½ΠΎΠ³ΠΎ участка.

o Π’Ρ‹Ρ‡ΠΈΡΠ»ΡΡŽΡ‚ ΠΎΠ±Ρ‰Π΅Π΅ сопротивлСниС для всСй Ρ†Π΅ΠΏΠΈ смСшанного соСдинСния.

Π’Π°ΠΊ это Π±ΡƒΠ΄Π΅Ρ‚ Π²Ρ‹Π³Π»ΡΠ΄Π΅Ρ‚ΡŒ для схСмы 1:

Π’Π°ΠΊΠΆΠ΅ сущСствуСт Π±ΠΎΠ»Π΅Π΅ быстрый способ расчСта ΠΎΠ±Ρ‰Π΅Π³ΠΎ сопротивлСния для смСшанного соСдинСния. МоТно, Π² соотвСтствии схСмС, сразу Π·Π°ΠΏΠΈΡΡ‹Π²Π°Ρ‚ΡŒ Ρ„ΠΎΡ€ΠΌΡƒΠ»Ρƒ ΡΠ»Π΅Π΄ΡƒΡŽΡ‰ΠΈΠΌ ΠΎΠ±Ρ€Π°Π·ΠΎΠΌ:

o Если рСзисторы ΡΠΎΠ΅Π΄ΠΈΠ½ΡΡŽΡ‚ΡΡ ΠΏΠΎΡΠ»Π΅Π΄ΠΎΠ²Π°Ρ‚Π΅ΡŒΠ½ΠΎ – ΡΠΊΠ»Π°Π΄Ρ‹Π²Π°Ρ‚ΡŒ.

o Если рСзисторы ΡΠΎΠ΅Π΄ΠΈΠ½ΡΡŽΡ‚ΡΡ ΠΏΠ°Ρ€Π°Π»Π»Π΅Π»ΡŒΠ½ΠΎ – ΠΈΡΠΏΠΎΠ»ΡŒΠ·ΠΎΠ²Π°Ρ‚ΡŒ условноС ΠΎΠ±ΠΎΠ·Π½Π°Ρ‡Π΅Π½ΠΈΠ΅ «||». o ΠŸΠΎΠ΄ΡΡ‚Π°Π²Π»ΡΡ‚ΡŒ Ρ„ΠΎΡ€ΠΌΡƒΠ»Ρƒ для ΠΏΠ°Ρ€Π°Π»Π»Π΅Π»ΡŒΠ½ΠΎΠ³ΠΎ соСдинСния Π³Π΄Π΅ стоит символ «||».

Π’Π°ΠΊ это Π±ΡƒΠ΄Π΅Ρ‚ Π²Ρ‹Π³Π»ΡΠ΄Π΅Ρ‚ΡŒ для схСмы 1:

ПослС подстановки Ρ„ΠΎΡ€ΠΌΡƒΠ»Ρ‹ ΠΏΠ°Ρ€Π°Π»Π»Π΅Π»ΡŒΠ½ΠΎΠ³ΠΎ соСдинСния вмСсто «||»:

ΠžΠ±Ρ‰ΠΈΠ΅ свСдСния

РСзистор

Π§Π°Ρ‰Π΅ всСго рСзисторы ΠΏΡ€Π΅Π΄ΡΡ‚Π°Π²Π»ΡΡŽΡ‚ собой ΠΌΠ΅Ρ‚Π°Π»Π»ΠΈΡ‡Π΅ΡΠΊΡƒΡŽ ΠΏΡ€ΠΎΠ²ΠΎΠ»ΠΎΠΊΡƒ ΠΈΠ»ΠΈ полоску, для компактности Π½Π°ΠΌΠΎΡ‚Π°Π½Π½ΡƒΡŽ Π½Π° ΡΡ‚Π΅Ρ€ΠΆΠ΅Π½ΡŒ (Ρ‡Π΅ΠΌ Π΄Π»ΠΈΠ½Π½Π΅ΠΉ ΠΏΡ€ΠΎΠ²ΠΎΠ΄Π½ΠΈΠΊ ΠΈ Ρ‡Π΅ΠΌ мСньшС Π΅Π³ΠΎ ΠΏΠΎΠΏΠ΅Ρ€Π΅Ρ‡Π½ΠΎΠ΅ сСчСниС, Ρ‚Π΅ΠΌ Π²Ρ‹ΡˆΠ΅ сопротивлСниС). РазумССтся, сопротивлСниС Ρ‚Π°ΠΊΠΆΠ΅ зависит ΠΎΡ‚ ΠΌΠ°Ρ‚Π΅Ρ€ΠΈΠ°Π»Π°, ΠΈΠ· ΠΊΠΎΡ‚ΠΎΡ€ΠΎΠ³ΠΎ ΠΈΠ·Π³ΠΎΡ‚ΠΎΠ²Π»Π΅Π½ ΠΏΡ€ΠΎΠ²ΠΎΠ΄Π½ΠΈΠΊ. ΠŸΠΎΠ»ΡŽΠ±ΠΎΠ²Π°Ρ‚ΡŒΡΡ Π½Π° рСзисторы ΠΌΠΎΠΆΠ½ΠΎ Π½Π° рисункС 50.1. «РСзисторы (с сайта Π Π°Π΄ΠΈΠΎΠšΠΎΡ‚)Β».

На элСктричСских схСмах рСзистор ΠΎΠ±Ρ‹Ρ‡Π½ΠΎ ΠΈΠ·ΠΎΠ±Ρ€Π°ΠΆΠ°ΡŽΡ‚ ΠΊΠ°ΠΊ ΠΏΡ€ΡΠΌΠΎΡƒΠ³ΠΎΠ»ΡŒΠ½ΠΈΠΊ, ΠΈΠ· ΠΊΠΎΡ‚ΠΎΡ€ΠΎΠ³ΠΎ выходят Π΄Π²Π° Π²Ρ‹Π²ΠΎΠ΄Π° (рисунок 50.2. «БхСматичСскоС ΠΈΠ·ΠΎΠ±Ρ€Π°ΠΆΠ΅Π½ΠΈΠ΅ рСзистора»).

ΠŸΠΎΡΠ»Π΅Π΄ΠΎΠ²Π°Ρ‚Π΅Π»ΡŒΠ½ΠΎΠ΅ ΠΈ ΠΏΠ°Ρ€Π°Π»Π»Π΅Π»ΡŒΠ½ΠΎΠ΅ соСдинСниС рСзисторов

ΠžΡ‡Π΅Π²ΠΈΠ΄Π½ΠΎ, имССтся Ρ‚ΠΎΠ»ΡŒΠΊΠΎ Π΄Π²Π΅ возмоТности для соСдинСния Π΄Π²ΡƒΡ… рСзисторов: ΠΌΠΎΠΆΠ½ΠΎ ΠΈΡ… ΡΠΏΠ°ΡΡ‚ΡŒ ΠΎΠ΄Π½ΠΈΠΌ ΠΊΠΎΠ½Ρ†ΠΎΠΌ ΠΈΠ»ΠΈ ΠΆΠ΅ ΠΎΠ±ΠΎΠΈΠΌΠΈ. ΠŸΠ΅Ρ€Π²Ρ‹ΠΉ способ называСтся ΠΏΠΎΡΠ»Π΅Π΄ΠΎΠ²Π°Ρ‚Π΅Π»ΡŒΠ½Ρ‹ΠΌ соСдинСниСм, Π° Π²Ρ‚ΠΎΡ€ΠΎΠΉ β€” ΠΏΠ°Ρ€Π°Π»Π»Π΅Π»ΡŒΠ½Ρ‹ΠΌ (рисунок 50.3. Β«ΠŸΠΎΡΠ»Π΅Π΄ΠΎΠ²Π°Ρ‚Π΅Π»ΡŒΠ½ΠΎΠ΅ ΠΈ ΠΏΠ°Ρ€Π°Π»Π»Π΅Π»ΡŒΠ½ΠΎΠ΅ соСдинСниС рСзисторов»).

И ΠΏΠΎΡΠ»Π΅Π΄ΠΎΠ²Π°Ρ‚Π΅Π»ΡŒΠ½ΠΎΠ΅, ΠΈ ΠΏΠ°Ρ€Π°Π»Π»Π΅Π»ΡŒΠ½ΠΎΠ΅ соСдинСниС рСзисторов ΠΌΠΎΠΆΠ½ΠΎ Ρ€Π°ΡΡΠΌΠ°Ρ‚Ρ€ΠΈΠ²Π°Ρ‚ΡŒ ΠΊΠ°ΠΊ Π½ΠΎΠ²Ρ‹ΠΉ рСзистор. Π•Π³ΠΎ сопротивлСниС ΠΌΠΎΠΆΠ½ΠΎ Π²Ρ‹Ρ‡ΠΈΡΠ»ΠΈΡ‚ΡŒ, ΠΏΠΎΠ»ΡŒΠ·ΡƒΡΡΡŒ ΡΠ»Π΅Π΄ΡƒΡŽΡ‰ΠΈΠΌΠΈ ΠΏΡ€Π°Π²ΠΈΠ»Π°ΠΌΠΈ:

  • ΠŸΡ€ΠΈ ΠΏΠΎΡΠ»Π΅Π΄ΠΎΠ²Π°Ρ‚Π΅Π»ΡŒΠ½ΠΎΠΌ соСдинСнии рСзисторов ΠΈΡ… сопротивлСния ΡΠΊΠ»Π°Π΄Ρ‹Π²Π°ΡŽΡ‚ΡΡ: R = R 1 + R 2 .
  • ΠŸΡ€ΠΈ ΠΏΠ°Ρ€Π°Π»Π»Π΅Π»ΡŒΠ½ΠΎΠΌ соСдинСнии рСзисторов ΡΠΊΠ»Π°Π΄Ρ‹Π²Π°ΡŽΡ‚ΡΡ ΠΈΡ… проводимости, Ρ‚ΠΎ Π΅ΡΡ‚ΡŒ Π²Π΅Π»ΠΈΡ‡ΠΈΠ½Ρ‹, ΠΎΠ±Ρ€Π°Ρ‚Π½Ρ‹Π΅ сопротивлСниям: 1 R = 1 R 1 + 1 R 2 , ΠΈΠ»ΠΈ R = R 1 ⁒ R 2 R 1 + R 2 .

Π’ частности, соСдиняя Π΄Π²Π° ΠΎΠ΄ΠΈΠ½Π°ΠΊΠΎΠ²Ρ‹Ρ… рСзистора с Π΅Π΄ΠΈΠ½ΠΈΡ‡Π½Ρ‹ΠΌ сопротивлСниСм ΠΏΠΎΡΠ»Π΅Π΄ΠΎΠ²Π°Ρ‚Π΅Π»ΡŒΠ½ΠΎ, ΠΏΠΎΠ»ΡƒΡ‡ΠΈΠΌ сопротивлСниС 2 , ΠΏΡ€ΠΈ ΠΏΠ°Ρ€Π°Π»Π»Π΅Π»ΡŒΠ½ΠΎΠΌ соСдинСнии ΠΏΠΎΠ»ΡƒΡ‡ΠΈΠΌ 1 2 .

ΠŸΡ€ΠΈ соСдинСнии Π±ΠΎΠ»Π΅Π΅ Π΄Π²ΡƒΡ… рСзисторов ΠΈΠ½ΠΎΠ³Π΄Π° удаётся ΠΏΡ€Π΅Π΄ΡΡ‚Π°Π²ΠΈΡ‚ΡŒ ΠΏΠΎΠ»ΡƒΡ‡Π΅Π½Π½ΡƒΡŽ схСму ΠΊΠ°ΠΊ ΠΏΠΎΡΠ»Π΅Π΄ΠΎΠ²Π°Ρ‚Π΅Π»ΡŒΠ½ΠΎΠ΅ ΠΈΠ»ΠΈ ΠΏΠ°Ρ€Π°Π»Π»Π΅Π»ΡŒΠ½ΠΎΠ΅ соСдинСниС Π΄Π²ΡƒΡ… подсхСм. НапримСр, схСма Π½Π° рисункС 50.4. «БмСшанноС соСдинСниС рСзисторов» прСдставляСтся ΠΊΠ°ΠΊ ΠΏΠ°Ρ€Π°Π»Π»Π΅Π»ΡŒΠ½ΠΎΠ΅ соСдинСниС рСзистора R 1 ΠΈ ΠΏΠΎΡΠ»Π΅Π΄ΠΎΠ²Π°Ρ‚Π΅Π»ΡŒΠ½ΠΎΠ³ΠΎ соСдинСния рСзисторов R 2 ΠΈ R 3 . Π’Π°ΠΊΠΈΠΌ ΠΎΠ±Ρ€Π°Π·ΠΎΠΌ, сопротивлСниС схСмы ΠΌΠ΅ΠΆΠ΄Ρƒ двумя Π²Ρ‹Π΄Π΅Π»Π΅Π½Π½Ρ‹ΠΌΠΈ ΡƒΠ·Π»Π°ΠΌΠΈ вычисляСтся ΠΊΠ°ΠΊ R 1 ⁒ R 2 + R 3 R 1 + R 2 + R 3 .

Π‘Π»ΠΎΠΆΠ½ΠΎΠ΅ соСдинСниС рСзисторов

Π£Π²Ρ‹, Π½Π΅ всякая схСма прСдставляСтся ΠΊΠ°ΠΊ ΠΏΠΎΡΠ»Π΅Π΄ΠΎΠ²Π°Ρ‚Π΅Π»ΡŒΠ½ΠΎΠ΅ ΠΈΠ»ΠΈ ΠΏΠ°Ρ€Π°Π»Π»Π΅Π»ΡŒΠ½ΠΎΠ΅ соСдинСниС Π΄Π²ΡƒΡ… подсхСм, ΠΏΠΎΠ΄ΠΎΠ±Π½ΠΎ Ρ‚ΠΎΠΌΡƒ, ΠΊΠ°ΠΊ Π½Π΅ всякоС Π½Π°Ρ‚ΡƒΡ€Π°Π»ΡŒΠ½ΠΎΠ΅ число раскладываСтся Π² ΠΏΡ€ΠΎΠΈΠ·Π²Π΅Π΄Π΅Π½ΠΈΠ΅ своих собствСнных Π΄Π΅Π»ΠΈΡ‚Π΅Π»Π΅ΠΉ. ΠŸΡ€ΠΎΡΡ‚ΠΎΠΉ ΠΏΡ€ΠΈΠΌΠ΅Ρ€ Ρ‚Π°ΠΊΠΎΠΉ Π½Π΅Ρ€Π°Π·Π»ΠΎΠΆΠΈΠΌΠΎΠΉ схСмы ΠΌΠΎΠΆΠ½ΠΎ ΡƒΠ²ΠΈΠ΄Π΅Ρ‚ΡŒ Π½Π° рисункС 50.5. Β«Π‘Π»ΠΎΠΆΠ½ΠΎΠ΅ соСдинСниС рСзисторов».

Для расчёта Ρ‚Π°ΠΊΠΈΡ… сопротивлСний ΠΈΡΠΏΠΎΠ»ΡŒΠ·ΡƒΡŽΡ‚, ΠΏΠΎΠΌΠΈΠΌΠΎ Π·Π°ΠΊΠΎΠ½Π° Ома, Π΅Ρ‰Ρ‘ ΠΈ Π·Π°ΠΊΠΎΠ½ сохранСния заряда.

ЭлСктричСский Ρ‚ΠΎΠΊ Π² ΠΏΡ€ΠΎΠ²ΠΎΠ΄Π½ΠΈΠΊΠ΅ ΠΌΠΎΠΆΠ½ΠΎ ΠΏΡ€Π΅Π΄ΡΡ‚Π°Π²Π»ΡΡ‚ΡŒ сСбС ΠΊΠ°ΠΊ ΠΏΠΎΡ‚ΠΎΠΊ частиц, нСсущих элСктричСскиС заряды (это ΠΌΠΎΠ³ΡƒΡ‚ Π±Ρ‹Ρ‚ΡŒ элСктроны ΠΈΠ»ΠΈ ΠΈΠΎΠ½Ρ‹). ΠŸΡ€ΠΈΡ‡ΠΈΠ½ΠΎΠΉ Ρ‚Π°ΠΊΠΎΠ³ΠΎ двиТСния заряТСнных частиц являСтся Ρ€Π°Π·Π½ΠΎΡΡ‚ΡŒ элСктричСских ΠΏΠΎΡ‚Π΅Π½Ρ†ΠΈΠ°Π»ΠΎΠ² Π½Π° ΠΊΠΎΠ½Ρ†Π°Ρ… ΠΏΡ€ΠΎΠ²ΠΎΠ΄Π½ΠΈΠΊΠ° (напряТСниС). Π‘Π°ΠΌΠ° ΠΏΠΎ сСбС Π²Π΅Π»ΠΈΡ‡ΠΈΠ½Π° ΠΏΠΎΡ‚Π΅Π½Ρ†ΠΈΠ°Π»Π° Π² ΠΎΡ‚Π΄Π΅Π»ΡŒΠ½ΠΎ взятой Ρ‚ΠΎΡ‡ΠΊΠ΅ схСмы Π½Π΅ ΠΈΠΌΠ΅Π΅Ρ‚ физичСского смысла, Ρ‚Π°ΠΊΠΎΠΉ смысл Π΅ΡΡ‚ΡŒ Ρ‚ΠΎΠ»ΡŒΠΊΠΎ лишь Ρƒ разности ΠΏΠΎΡ‚Π΅Π½Ρ†ΠΈΠ°Π»ΠΎΠ² Π² Π΄Π²ΡƒΡ… Ρ‚ΠΎΡ‡ΠΊΠ°Ρ… (Ρ‚ΠΎΡ‡Π½ΠΎ Ρ‚Π°ΠΊ ΠΆΠ΅ лишСна смысла ΠΏΠΎΡ‚Π΅Π½Ρ†ΠΈΠ°Π»ΡŒΠ½Π°Ρ энСргия силы тяТСсти Π² ΠΎΡ‚Π΄Π΅Π»ΡŒΠ½ΠΎΠΉ Ρ‚ΠΎΡ‡ΠΊΠ΅, Π° Π²Π°ΠΆΠ΅Π½ ΠΏΠ΅Ρ€Π΅ΠΏΠ°Π΄ ΠΏΠΎΡ‚Π΅Π½Ρ†ΠΈΠ°Π»ΡŒΠ½ΠΎΠΉ энСргии Π² Π΄Π²ΡƒΡ… Ρ‚ΠΎΡ‡ΠΊΠ°Ρ…). Π’ΠΎΠΊ β€” это суммарный заряд, ΠΏΡ€ΠΎΡ‚Π΅ΠΊΠ°ΡŽΡ‰ΠΈΠΉ Ρ‡Π΅Ρ€Π΅Π· ΠΏΠΎΠΏΠ΅Ρ€Π΅Ρ‡Π½ΠΎΠ΅ сСчСниС ΠΏΡ€ΠΎΠ²ΠΎΠ΄Π½ΠΈΠΊΠ° Π·Π° Π΅Π΄ΠΈΠ½ΠΈΡ†Ρƒ Π²Ρ€Π΅ΠΌΠ΅Π½ΠΈ. ΠŸΡ€Π΅Π΄ΡΡ‚Π°Π²ΠΈΠΌ Ρ‚Π°ΠΊΡƒΡŽ модСль: ΠΏΠΎ Π΄ΠΎΡ€ΠΎΠ³Π΅ ΠΈΠ· ΠΏΡƒΠ½ΠΊΡ‚Π° A Π² ΠΏΡƒΠ½ΠΊΡ‚ B двиТСтся ΠΏΠΎΡ‚ΠΎΠΊ Π°Π²Ρ‚ΠΎΠΌΠΎΠ±ΠΈΠ»Π΅ΠΉ, ΠΊΠ°ΠΆΠ΄Ρ‹ΠΉ ΠΈΠ· ΠΊΠΎΡ‚ΠΎΡ€Ρ‹Ρ… Π·Π°Π³Ρ€ΡƒΠΆΠ΅Π½ зарядом. Если заряды ΠΏΠΎΠ»ΠΎΠΆΠΈΡ‚Π΅Π»ΡŒΠ½Ρ‹, считаСтся, Ρ‡Ρ‚ΠΎ Ρ‚ΠΎΠΊ Π² Π½Π°ΠΏΡ€Π°Π²Π»Π΅Π½ΠΈΠΈ ΠΎΡ‚ A ΠΊ B ΠΏΠΎΠ»ΠΎΠΆΠΈΡ‚Π΅Π»Π΅Π½. Но ΠΌΠΎΠΆΠ½ΠΎ ΡΡ‡ΠΈΡ‚Π°Ρ‚ΡŒ Ρ‚Π°ΠΊΠΆΠ΅, Ρ‡Ρ‚ΠΎ имССтся ΠΎΡ‚Ρ€ΠΈΡ†Π°Ρ‚Π΅Π»ΡŒΠ½Ρ‹ΠΉ Ρ‚ΠΎΠΊ (Ρ‚ΠΎΠΉ ΠΆΠ΅ самой Π°Π±ΡΠΎΠ»ΡŽΡ‚Π½ΠΎΠΉ Π²Π΅Π»ΠΈΡ‡ΠΈΠ½Ρ‹) Π² Π½Π°ΠΏΡ€Π°Π²Π»Π΅Π½ΠΈΠΈ ΠΎΡ‚ B ΠΊ A .

Π—Π°ΠΊΠΎΠ½ сохранСния зарядов Π³ΠΎΠ²ΠΎΡ€ΠΈΡ‚, Ρ‡Ρ‚ΠΎ элСктричСскиС заряды Π½Π΅ Π²ΠΎΠ·Π½ΠΈΠΊΠ°ΡŽΡ‚ Π½ΠΈΠΎΡ‚ΠΊΡƒΠ΄Π° ΠΈ Π½Π΅ ΠΈΡΡ‡Π΅Π·Π°ΡŽΡ‚ Π² Π½ΠΈΠΊΡƒΠ΄Π°. Если элСктричСски Π½Π΅ΠΉΡ‚Ρ€Π°Π»ΡŒΠ½Π°Ρ частица, такая ΠΊΠ°ΠΊ Π°Ρ‚ΠΎΠΌ, распадаСтся Π½Π° Π΄Π²Π΅ заряТСнных частицы (ΠΈΠΎΠ½ ΠΈ элСктрон), суммарный заряд Π½ΠΎΠ²Ρ‹Ρ… частиц всСгда равняСтся заряду Π°Ρ‚ΠΎΠΌΠ°, Ρ‚ΠΎ Π΅ΡΡ‚ΡŒ Π½ΡƒΠ»ΡŽ. Из Π·Π°ΠΊΠΎΠ½Π° слСдуСт, Π² частности, Ρ‡Ρ‚ΠΎ Ρ‚ΠΎΠΊΠΈ Ρ‡Π΅Ρ€Π΅Π· Π΄Π²Π° ΠΏΠΎΠΏΠ΅Ρ€Π΅Ρ‡Π½Ρ‹Ρ… сСчСния Ρ‚ΠΎΠ½ΠΊΠΎΠ³ΠΎ ΠΏΡ€ΠΎΠ²ΠΎΠ΄Π½ΠΈΠΊΠ° Π² ΠΎΠ΄ΠΈΠ½ ΠΈ Ρ‚ΠΎΡ‚ ΠΆΠ΅ ΠΌΠΎΠΌΠ΅Π½Ρ‚ Π²Ρ€Π΅ΠΌΠ΅Π½ΠΈ Ρ€Π°Π²Π½Ρ‹, ΠΈΠ½Π°Ρ‡Π΅ Π³Π΄Π΅-Ρ‚ΠΎ ΠΌΠ΅ΠΆΠ΄Ρƒ этими сСчСниями роТдался Π±Ρ‹ ΠΈΠ»ΠΈ ΠΏΡ€ΠΎΠΏΠ°Π΄Π°Π» Π½Π΅Π½ΡƒΠ»Π΅Π²ΠΎΠΉ заряд. Π”Ρ€ΡƒΠ³ΠΈΠΌ слСдствиСм Π·Π°ΠΊΠΎΠ½Π° сохранСния заряда являСтся ΡƒΡ‚Π²Π΅Ρ€ΠΆΠ΄Π΅Π½ΠΈΠ΅, Ρ‡Ρ‚ΠΎ Π² ΡƒΠ·Π»Π΅ элСктричСской схСмы, Π³Π΄Π΅ соСдиняСтся нСсколько ΠΏΡ€ΠΎΠ²ΠΎΠ΄Π½ΠΈΠΊΠΎΠ², сумма всСх входящих Π² ΡƒΠ·Π΅Π» Ρ‚ΠΎΠΊΠΎΠ² Ρ€Π°Π²Π½Π° суммС всСх выходящих. Если Π²Π΅Ρ€Π½ΡƒΡ‚ΡŒΡΡ ΠΊ Π°Π²Ρ‚ΠΎΠΌΠΎΠ±ΠΈΠ»ΡŒΠ½ΠΎΠΉ Π°Π½Π°Π»ΠΎΠ³ΠΈΠΈ, количСство Π°Π²Ρ‚ΠΎΠΌΠΎΠ±ΠΈΠ»Π΅ΠΉ, Π²ΡŠΠ΅Π·ΠΆΠ°ΡŽΡ‰ΠΈΡ… Π½Π° пСрСкрёсток Π½Π΅ΡΠΊΠΎΠ»ΡŒΠΊΠΈΡ… Π΄ΠΎΡ€ΠΎΠ³, Ρ€Π°Π²Π½ΠΎ количСству Π²Ρ‹Π΅Π·ΠΆΠ°ΡŽΡ‰ΠΈΡ… с пСрСкрёстка (здСсь, ΠΊΠΎΠ½Π΅Ρ‡Π½ΠΎ, прСдполагаСтся, Ρ‡Ρ‚ΠΎ ΠΊΠ°ΠΆΠ΄Ρ‹ΠΉ Π°Π²Ρ‚ΠΎΠΌΠΎΠ±ΠΈΠ»ΡŒ Π²Π΅Π·Ρ‘Ρ‚ Π΅Π΄ΠΈΠ½ΠΈΡ‡Π½Ρ‹ΠΉ заряд, ΠΈ врСмя, ΠΏΡ€ΠΎΠ²ΠΎΠ΄ΠΈΠΌΠΎΠ΅ автомобилями Π½Π° пСрСкрёсткС, ΠΏΡ€Π΅Π½Π΅Π±Ρ€Π΅ΠΆΠΈΠΌΠΎ ΠΌΠ°Π»ΠΎ).

Π’Π΅ΠΏΠ΅Ρ€ΡŒ, Π²ΠΎΠΎΡ€ΡƒΠΆΡ‘Π½Π½Ρ‹Π΅ знаниями, рассчитаСм сопротивлСниС элСктричСской схСмы Π½Π° рисункС 50.5 ΠΌΠ΅ΠΆΠ΄Ρƒ ΠΎΡ‚ΠΌΠ΅Ρ‡Π΅Π½Π½Ρ‹ΠΌΠΈ ΡƒΠ·Π»Π°ΠΌΠΈ. На схСмС ΠΏΡ€ΠΈΡΡƒΡ‚ΡΡ‚Π²ΡƒΡŽΡ‚ ΠΏΡΡ‚ΡŒ рСзисторов ΠΈ Ρ‡Π΅Ρ‚Ρ‹Ρ€Π΅ ΡƒΠ·Π»Π°. ΠŸΡ€ΠΎΠ½ΡƒΠΌΠ΅Ρ€ΡƒΠ΅ΠΌ рСзисторы числами ΠΎΡ‚ 1 Π΄ΠΎ 5 ΠΈ ΡƒΠ·Π»Ρ‹ числами ΠΎΡ‚ 1 Π΄ΠΎ 4 . ΠŸΠΎΡ€ΡΠ΄ΠΎΠΊ Π½ΡƒΠΌΠ΅Ρ€Π°Ρ†ΠΈΠΈ ΡƒΠ·Π»ΠΎΠ² ΠΌΠΎΠΆΠ½ΠΎ Π²Ρ‹Π±Ρ€Π°Ρ‚ΡŒ ΡΠΎΠ²Π΅Ρ€ΡˆΠ΅Π½Π½ΠΎ ΠΏΡ€ΠΎΠΈΠ·Π²ΠΎΠ»ΡŒΠ½ΠΎ. Π§Ρ‚ΠΎΠ±Ρ‹ ΡΡƒΠ΄ΠΈΡ‚ΡŒ ΠΎ Π½Π°ΠΏΡ€Π°Π²Π»Π΅Π½ΠΈΠΈ Ρ‚ΠΎΠΊΠ° Ρ‡Π΅Ρ€Π΅Π· ΠΊΠ°ΠΆΠ΄Ρ‹ΠΉ ΠΈΠ· рСзисторов, слСдуСт Π½Π° ΠΊΠ°ΠΆΠ΄ΠΎΠΌ Π·Π°Π΄Π°Ρ‚ΡŒ Π½Π°ΠΏΡ€Π°Π²Π»Π΅Π½ΠΈΠ΅. Π­Ρ‚ΠΎ Ρ‚Π°ΠΊΠΆΠ΅ ΠΌΠΎΠΆΠ½ΠΎ ΡΠ΄Π΅Π»Π°Ρ‚ΡŒ ΠΏΡ€ΠΎΠΈΠ·Π²ΠΎΠ»ΡŒΠ½ΠΎ, ΠΎΠ΄Π½Π°ΠΊΠΎ для опрСдСлённости Π±ΡƒΠ΄Π΅ΠΌ ΡΡ‡ΠΈΡ‚Π°Ρ‚ΡŒ, Ρ‡Ρ‚ΠΎ ΠΏΠΎΠ»ΠΎΠΆΠΈΡ‚Π΅Π»ΡŒΠ½Ρ‹ΠΌ Π½Π°ΠΏΡ€Π°Π²Π»Π΅Π½ΠΈΠ΅ΠΌ Ρ‚ΠΎΠΊΠ° Π±ΡƒΠ΄Π΅Ρ‚ Π½Π°ΠΏΡ€Π°Π²Π»Π΅Π½ΠΈΠ΅ ΠΎΡ‚ ΡƒΠ·Π»Π° с мСньшим Π½ΠΎΠΌΠ΅Ρ€ΠΎΠΌ ΠΊ ΡƒΠ·Π»Ρƒ с большим. ΠžΠ±ΠΎΠ·Π½Π°Ρ‡ΠΈΠΌ ΠΏΠΎΡ‚Π΅Π½Ρ†ΠΈΠ°Π»Ρ‹ Π² ΡƒΠ·Π»Π°Ρ… Π±ΡƒΠΊΠ²ΠΎΠΉ U с ΡΠΎΠΎΡ‚Π²Π΅Ρ‚ΡΡ‚Π²ΡƒΡŽΡ‰ΠΈΠΌ индСксом. Π Π΅Π·ΡƒΠ»ΡŒΡ‚Π°Ρ‚ всСх этих ΠΏΡ€ΠΈΠ³ΠΎΡ‚ΠΎΠ²Π»Π΅Π½ΠΈΠΉ прСдставлСн Π½Π° рисункС 50.6. Β«Π Π°Π·ΠΌΠ΅Ρ‚ΠΊΠ° схСмы».

ΠŸΡ€ΠΎΠΏΡƒΡΡ‚ΠΈΠΌ элСктричСский Ρ‚ΠΎΠΊ Ρ‡Π΅Ρ€Π΅Π· ΡƒΠ·Π»Ρ‹ с Π½ΠΎΠΌΠ΅Ρ€Π°ΠΌΠΈ 1 ΠΈ 2 . Из Π·Π°ΠΊΠΎΠ½Π° сохранСния заряда Ρ‚ΠΎΠΊ, входящий Π² ΡƒΠ·Π΅Π» 1 , Ρ€Π°Π²Π΅Π½ Ρ‚ΠΎΠΊΡƒ, выходящСму ΠΈΠ· ΡƒΠ·Π»Π° 2 . Если Π²Π·ΡΡ‚ΡŒ Π²Π΅Π»ΠΈΡ‡ΠΈΠ½Ρƒ Ρ‚ΠΎΠΊΠ°, Ρ€Π°Π²Π½ΡƒΡŽ Π΅Π΄ΠΈΠ½ΠΈΡ†Π΅, Π² силу Π·Π°ΠΊΠΎΠ½Π° Ома Ρ€Π°Π·Π½ΠΎΡΡ‚ΡŒ ΠΏΠΎΡ‚Π΅Π½Ρ†ΠΈΠ°Π»ΠΎΠ² U 2 βˆ’ U 1 Π±ΡƒΠ΄Π΅Ρ‚ Ρ€Π°Π²Π½Π° Π² точности искомому ΡΠΎΠΏΡ€ΠΎΡ‚ΠΈΠ²Π»Π΅Π½ΠΈΡŽ. ΠŸΠΎΡΠΊΠΎΠ»ΡŒΠΊΡƒ, ΠΊΠ°ΠΊ ΠΌΡ‹ ΠΏΠΎΠΌΠ½ΠΈΠΌ, ΠΈΠΌΠ΅ΡŽΡ‚ значСния лишь разности ΠΏΠΎΡ‚Π΅Π½Ρ†ΠΈΠ°Π»ΠΎΠ², ΠΌΡ‹ ΠΌΠΎΠΆΠ΅ΠΌ смСло ΠΏΠΎΠ»ΠΎΠΆΠΈΡ‚ΡŒ U 1 = 0 , ΠΈ Ρ‚ΠΎΠ³Π΄Π° U 2 окаТСтся искомым сопротивлСниСм схСмы.

ΠžΠ±ΠΎΠ·Π½Π°Ρ‡ΠΈΠ² ΠΊΠ°ΠΊ I Ξ± Ρ‚ΠΎΠΊ Ρ‡Π΅Ρ€Π΅Π· рСзистор R Ξ± , для ΠΊΠ°ΠΆΠ΄ΠΎΠ³ΠΎ ΠΈΠ· рСзисторов запишСм Π·Π°ΠΊΠΎΠ½ Ома: R 1 ⁒ I 1 = U 3 βˆ’ U 1 , R 2 ⁒ I 2 = U 4 βˆ’ U 1 , R 3 ⁒ I 3 = U 4 βˆ’ U 3 , R 4 ⁒ I 4 = U 3 βˆ’ U 2 , R 5 ⁒ I 5 = U 4 βˆ’ U 2 .

Вторая Π³Ρ€ΡƒΠΏΠΏΠ° ΡƒΡ€Π°Π²Π½Π΅Π½ΠΈΠΉ получаСтся ΠΈΠ· Π·Π°ΠΊΠΎΠ½Π° сохранСния заряда. Для ΠΊΠ°ΠΆΠ΄ΠΎΠ³ΠΎ ΡƒΠ·Π»Π° сумму входящих Π² Π½Π΅Π³ΠΎ Ρ‚ΠΎΠΊΠΎΠ² ΠΏΡ€ΠΈΡ€Π°Π²Π½ΠΈΠ²Π°Π΅ΠΌ суммС выходящих. ΠŸΡ€ΠΈ этом Π½Π΅ Π·Π°Π±Ρ‹Π²Π°Π΅ΠΌ ΠΏΡ€ΠΎ Π΅Π΄ΠΈΠ½ΠΈΡ‡Π½Ρ‹ΠΉ Ρ‚ΠΎΠΊ, входящий Π² ΠΏΠ΅Ρ€Π²Ρ‹ΠΉ ΡƒΠ·Π΅Π» ΠΈ выходящий ΠΈΠ· Π²Ρ‚ΠΎΡ€ΠΎΠ³ΠΎ: 1 = I 1 + I 2 , 0 = 1 + I 4 + I 5 , I 1 + I 4 = I 3 , I 2 + I 3 + I 5 = 0 .

Π”ΠΎΠ±Π°Π²ΠΈΠ² ΠΊ составлСнным уравнСниям Π΅Ρ‰Ρ‘ ΠΎΠ΄Π½ΠΎ, U 1 = 0 , Ρ€Π΅ΡˆΠ°Π΅ΠΌ ΠΏΠΎΠ»ΡƒΡ‡Π΅Π½Π½ΡƒΡŽ систСму ΠΎΡ‚Π½ΠΎΡΠΈΡ‚Π΅Π»ΡŒΠ½ΠΎ U 2 .

ΠœΠ΅ΠΆΠ΄Ρƒ ΠΏΡ€ΠΎΡ‡ΠΈΠΌ, примСняя ΠΎΠΏΠΈΡΠ°Π½Π½ΡƒΡŽ ΠΌΠ΅Ρ‚ΠΎΠ΄ΠΈΠΊΡƒ ΠΊ ΠΏΠΎΡΠ»Π΅Π΄ΠΎΠ²Π°Ρ‚Π΅Π»ΡŒΠ½ΠΎΠΌΡƒ ΠΈ ΠΏΠ°Ρ€Π°Π»Π»Π΅Π»ΡŒΠ½ΠΎΠΌΡƒ соСдинСниям рСзисторов, ΠΌΡ‹ с ΡƒΠ΄ΠΎΠ²ΠΎΠ»ΡŒΡΡ‚Π²ΠΈΠ΅ΠΌ ΡƒΠ±Π΅Π΄ΠΈΠ»ΠΈΡΡŒ Π² ΠΏΡ€Π°Π²ΠΈΠ»ΡŒΠ½ΠΎΡΡ‚ΠΈ Ρ„ΠΎΡ€ΠΌΡƒΠ» слоТСния сопротивлСний ΠΈ проводимостСй.

ΠŸΠΎΡ€Π° Π·Π°ΠΌΠ΅Ρ‚ΠΈΡ‚ΡŒ, Ρ‡Ρ‚ΠΎ всС ΠΏΠΎΠ»ΡƒΡ‡Π΅Π½Π½Ρ‹Π΅ уравнСния ΡΠ²Π»ΡΡŽΡ‚ΡΡ Π»ΠΈΠ½Π΅ΠΉΠ½Ρ‹ΠΌΠΈ алгСбраичСскими ΠΏΠΎ ΠΎΡ‚Π½ΠΎΡˆΠ΅Π½ΠΈΡŽ ΠΊΠΎ всСм нСизвСстным Π²Π΅Π»ΠΈΡ‡ΠΈΠ½Π°ΠΌ I Ξ± ΠΈ U Ξ² . ΠœΡ‹ Π½Π΅ станСм Π·Π°Π΄Π°Π²Π°Ρ‚ΡŒΡΡ вопросом ΠΎ СдинствСнности Ρ€Π΅ΡˆΠ΅Π½ΠΈΡ Ρ‚Π°ΠΊΠΎΠΉ систСмы ΡƒΡ€Π°Π²Π½Π΅Π½ΠΈΠΉ. ΠžΡ‚ΠΌΠ΅Ρ‚ΠΈΠΌ лишь, Ρ‡Ρ‚ΠΎ сущСствуСт СдинствСнноС Π·Π½Π°Ρ‡Π΅Π½ΠΈΠ΅ U 2 , ΡƒΠ΄ΠΎΠ²Π»Π΅Ρ‚Π²ΠΎΡ€ΡΡŽΡ‰Π΅Π΅ систСмС. Об этом Π³ΠΎΠ²ΠΎΡ€ΠΈΡ‚ физичСский смысл ΡƒΡ€Π°Π²Π½Π΅Π½ΠΈΠΉ.

Π—Π°Π΄Π°Ρ‡Π° расчёта элСктричСского сопротивлСния являСтся довольно Π°ΠΊΡ‚ΡƒΠ°Π»ΡŒΠ½ΠΎΠΉ. Π˜ΠΌΠ΅Π΅Ρ‚ΡΡ ряд ΠΏΡ€ΠΈΡ‘ΠΌΠΎΠ², ΠΊΠΎΡ‚ΠΎΡ€Ρ‹Π΅ ΠΏΠΎΠ·Π²ΠΎΠ»ΡΡŽΡ‚ ΡƒΠΏΡ€ΠΎΡΡ‚ΠΈΡ‚ΡŒ Π΅Ρ‘ Ρ€Π΅ΡˆΠ΅Π½ΠΈΠ΅. К ΠΏΡ€ΠΈΠΌΠ΅Ρ€Ρƒ, ΠΏΡ€Π°Π²ΠΈΠ»Π° ΠšΠΈΡ€Ρ…Π³ΠΎΡ„Π° ΠΏΠΎΠ·Π²ΠΎΠ»ΡΡŽΡ‚ ΡΡ‚Ρ€ΠΎΠΈΡ‚ΡŒ систСмы ΡƒΡ€Π°Π²Π½Π΅Π½ΠΈΠΉ, Ρ€Π°Π²Π½ΠΎΡΠΈΠ»ΡŒΠ½Ρ‹Π΅ Ρ‚ΠΎΠ»ΡŒΠΊΠΎ Ρ‡Ρ‚ΠΎ ΠΏΠΎΠ»ΡƒΡ‡Π΅Π½Π½Ρ‹ΠΌ, ΠΈ ΠΏΡ€ΠΈ этом, ΠΊΠ°ΠΊ ΠΏΡ€Π°Π²ΠΈΠ»ΠΎ, Π±ΠΎΠ»Π΅Π΅ простыС. Π•ΡΡ‚ΡŒ ΠΌΠ΅Ρ‚ΠΎΠ΄Ρ‹, Π² основС ΠΊΠΎΡ‚ΠΎΡ€Ρ‹Ρ… Π»Π΅ΠΆΠ°Ρ‚ прСобразования схСм Π² эквивалСнтныС (Ρ‚ΠΎ Π΅ΡΡ‚ΡŒ ΠΈΠΌΠ΅ΡŽΡ‰ΠΈΠ΅ Ρ‚ΠΎ ΠΆΠ΅ сопротивлСниС), Π½ΠΎ ΠΏΡ€ΠΈ этом Ρ€Π°Π·Π»ΠΎΠΆΠΈΠΌΡ‹Π΅ Π² ΠΏΠΎΡΠ»Π΅Π΄ΠΎΠ²Π°Ρ‚Π΅Π»ΡŒΠ½ΠΎΠ΅ ΠΈΠ»ΠΈ ΠΏΠ°Ρ€Π°Π»Π»Π΅Π»ΡŒΠ½ΠΎΠ΅ соСдинСниС Π΄Π²ΡƒΡ… подсхСм. ΠœΡ‹ Π½Π΅ Π±ΡƒΠ΄Π΅ΠΌ ΠΎΡΡ‚Π°Π½Π°Π²Π»ΠΈΠ²Π°Ρ‚ΡŒΡΡ Π½Π° этих ΠΌΠ΅Ρ‚ΠΎΠ΄Π°Ρ…. Π’ Π³Π»Π°Π²Π΅ 49. Β«Π›ΠΈΠ½Π΅ΠΉΠ½Ρ‹Π΅ уравнСния» Ρ€Π°ΡΡΠΌΠ°Ρ‚Ρ€ΠΈΠ²Π°Π»ΠΎΡΡŒ алгоритмичСскоС Ρ€Π΅ΡˆΠ΅Π½ΠΈΠ΅ систСм Π»ΠΈΠ½Π΅ΠΉΠ½Ρ‹Ρ… ΡƒΡ€Π°Π²Π½Π΅Π½ΠΈΠΉ, ΠΈ Π½Π°ΠΌ остаётся лишь Π²ΠΎΡΠΏΠΎΠ»ΡŒΠ·ΠΎΠ²Π°Ρ‚ΡŒΡΡ ΡƒΠΆΠ΅ написанным Π±ΠΈΠ±Π»ΠΈΠΎΡ‚Π΅Ρ‡Π½Ρ‹ΠΌ ΠΌΠΎΠ΄ΡƒΠ»Π΅ΠΌ.

ΠŸΠΎΡΠ»Π΅Π΄ΠΎΠ²Π°Ρ‚Π΅Π»ΡŒΠ½ΠΎΠ΅, ΠΏΠ°Ρ€Π°Π»Π»Π΅Π»ΡŒΠ½ΠΎΠ΅ ΠΈ смСшанноС соСдинСния рСзисторов. Π—Π½Π°Ρ‡ΠΈΡ‚Π΅Π»ΡŒΠ½ΠΎΠ΅ число ΠΏΡ€ΠΈΠ΅ΠΌΠ½ΠΈΠΊΠΎΠ², Π²ΠΊΠ»ΡŽΡ‡Π΅Π½Π½Ρ‹Ρ… Π² ΡΠ»Π΅ΠΊΡ‚Ρ€ΠΈΡ‡Π΅ΡΠΊΡƒΡŽ Ρ†Π΅ΠΏΡŒ (элСктричСскиС Π»Π°ΠΌΠΏΡ‹, ΡΠ»Π΅ΠΊΡ‚Ρ€ΠΎΠ½Π°Π³Ρ€Π΅Π²Π°Ρ‚Π΅Π»ΡŒΠ½Ρ‹Π΅ ΠΏΡ€ΠΈΠ±ΠΎΡ€Ρ‹ ΠΈ Π΄Ρ€.), ΠΌΠΎΠΆΠ½ΠΎ Ρ€Π°ΡΡΠΌΠ°Ρ‚Ρ€ΠΈΠ²Π°Ρ‚ΡŒ ΠΊΠ°ΠΊ Π½Π΅ΠΊΠΎΡ‚ΠΎΡ€Ρ‹Π΅ элСмСнты, ΠΈΠΌΠ΅ΡŽΡ‰ΠΈΠ΅ ΠΎΠΏΡ€Π΅Π΄Π΅Π»Π΅Π½Π½ΠΎΠ΅ сопротивлСниС. Π­Ρ‚ΠΎ ΠΎΠ±ΡΡ‚ΠΎΡΡ‚Π΅Π»ΡŒΡΡ‚Π²ΠΎ Π΄Π°Π΅Ρ‚ Π½Π°ΠΌ Π²ΠΎΠ·ΠΌΠΎΠΆΠ½ΠΎΡΡ‚ΡŒ ΠΏΡ€ΠΈ составлСнии ΠΈ ΠΈΠ·ΡƒΡ‡Π΅Π½ΠΈΠΈ элСктричСских схСм Π·Π°ΠΌΠ΅Π½ΡΡ‚ΡŒ ΠΊΠΎΠ½ΠΊΡ€Π΅Ρ‚Π½Ρ‹Π΅ ΠΏΡ€ΠΈΠ΅ΠΌΠ½ΠΈΠΊΠΈ рСзисторами с ΠΎΠΏΡ€Π΅Π΄Π΅Π»Π΅Π½Π½Ρ‹ΠΌΠΈ сопротивлСниями. Π Π°Π·Π»ΠΈΡ‡Π°ΡŽΡ‚ ΡΠ»Π΅Π΄ΡƒΡŽΡ‰ΠΈΠ΅ способы соСдинСния рСзисторов (ΠΏΡ€ΠΈΠ΅ΠΌΠ½ΠΈΠΊΠΎΠ² элСктричСской энСргии): ΠΏΠΎΡΠ»Π΅Π΄ΠΎΠ²Π°Ρ‚Π΅Π»ΡŒΠ½ΠΎΠ΅, ΠΏΠ°Ρ€Π°Π»Π»Π΅Π»ΡŒΠ½ΠΎΠ΅ ΠΈ смСшанноС.

Рис. 25. Π‘Ρ…Π΅ΠΌΡ‹ ΠΏΠΎΡΠ»Π΅Π΄ΠΎΠ²Π°Ρ‚Π΅Π»ΡŒΠ½ΠΎΠ³ΠΎ соСдинСния ΠΏΡ€ΠΈΠ΅ΠΌΠ½ΠΈΠΊΠΎΠ²

ΠŸΠΎΡΠ»Π΅Π΄ΠΎΠ²Π°Ρ‚Π΅Π»ΡŒΠ½ΠΎΠ΅ соСдинСниС рСзисторов. ΠŸΡ€ΠΈ ΠΏΠΎΡΠ»Π΅Π΄ΠΎΠ²Π°Ρ‚Π΅Π»ΡŒΠ½ΠΎΠΌ соСдинСнии Π½Π΅ΡΠΊΠΎΠ»ΡŒΠΊΠΈΡ… рСзисторов ΠΊΠΎΠ½Π΅Ρ† ΠΏΠ΅Ρ€Π²ΠΎΠ³ΠΎ рСзистора ΡΠΎΠ΅Π΄ΠΈΠ½ΡΡŽΡ‚ с Π½Π°Ρ‡Π°Π»ΠΎΠΌ Π²Ρ‚ΠΎΡ€ΠΎΠ³ΠΎ, ΠΊΠΎΠ½Π΅Ρ† Π²Ρ‚ΠΎΡ€ΠΎΠ³ΠΎ β€” с Π½Π°Ρ‡Π°Π»ΠΎΠΌ Ρ‚Ρ€Π΅Ρ‚ΡŒΠ΅Π³ΠΎ ΠΈ Ρ‚. Π΄. ΠŸΡ€ΠΈ Ρ‚Π°ΠΊΠΎΠΌ соСдинСнии ΠΏΠΎ всСм элСмСнтам ΠΏΠΎΡΠ»Π΅Π΄ΠΎΠ²Π°Ρ‚Π΅Π»ΡŒΠ½ΠΎΠΉ Ρ†Π΅ΠΏΠΈ ΠΏΡ€ΠΎΡ…ΠΎΠ΄ΠΈΡ‚
ΠΎΠ΄ΠΈΠ½ ΠΈ Ρ‚ΠΎΡ‚ ΠΆΠ΅ Ρ‚ΠΎΠΊ I.
ΠŸΠΎΡΠ»Π΅Π΄ΠΎΠ²Π°Ρ‚Π΅Π»ΡŒΠ½ΠΎΠ΅ соСдинСниС ΠΏΡ€ΠΈΠ΅ΠΌΠ½ΠΈΠΊΠΎΠ² поясняСт рис. 25, Π°.
.ЗамСняя Π»Π°ΠΌΠΏΡ‹ рСзисторами с сопротивлСниями R1, R2 ΠΈ R3, ΠΏΠΎΠ»ΡƒΡ‡ΠΈΠΌ схСму, ΠΏΠΎΠΊΠ°Π·Π°Π½Π½ΡƒΡŽ Π½Π° рис. 25, Π±.
Если ΠΏΡ€ΠΈΠ½ΡΡ‚ΡŒ, Ρ‡Ρ‚ΠΎ Π² источникС Ro = 0, Ρ‚ΠΎ для Ρ‚Ρ€Π΅Ρ… ΠΏΠΎΡΠ»Π΅Π΄ΠΎΠ²Π°Ρ‚Π΅Π»ΡŒΠ½ΠΎ соСдинСнных рСзисторов согласно Π²Ρ‚ΠΎΡ€ΠΎΠΌΡƒ Π·Π°ΠΊΠΎΠ½Ρƒ ΠšΠΈΡ€Ρ…Π³ΠΎΡ„Π° ΠΌΠΎΠΆΠ½ΠΎ Π½Π°ΠΏΠΈΡΠ°Ρ‚ΡŒ:

гдС R эк = R1 + R2 + R3 .
Π‘Π»Π΅Π΄ΠΎΠ²Π°Ρ‚Π΅Π»ΡŒΠ½ΠΎ, эквивалСнтноС сопротивлСниС ΠΏΠΎΡΠ»Π΅Π΄ΠΎΠ²Π°Ρ‚Π΅Π»ΡŒΠ½ΠΎΠΉ Ρ†Π΅ΠΏΠΈ Ρ€Π°Π²Π½ΠΎ суммС сопротивлСний всСх ΠΏΠΎΡΠ»Π΅Π΄ΠΎΠ²Π°Ρ‚Π΅Π»ΡŒΠ½ΠΎ соСдинСнных рСзисторов. Π’Π°ΠΊ ΠΊΠ°ΠΊ напряТСния Π½Π° ΠΎΡ‚Π΄Π΅Π»ΡŒΠ½Ρ‹Ρ… участках Ρ†Π΅ΠΏΠΈ согласно Π·Π°ΠΊΠΎΠ½Ρƒ Ома: U1=IR1; U2 = IR2, U3 = IRΠ· ΠΈ Π² Π΄Π°Π½Π½ΠΎΠΌ случаС E = U, Ρ‚ΠΎ длярассматриваСмой Ρ†Π΅ΠΏΠΈ

U = U1 + U2 +U3 (20)

Π‘Π»Π΅Π΄ΠΎΠ²Π°Ρ‚Π΅Π»ΡŒΠ½ΠΎ, напряТСниС U Π½Π° Π·Π°ΠΆΠΈΠΌΠ°Ρ… источника Ρ€Π°Π²Π½ΠΎ суммС напряТСний Π½Π° ΠΊΠ°ΠΆΠ΄ΠΎΠΌ ΠΈΠ· ΠΏΠΎΡΠ»Π΅Π΄ΠΎΠ²Π°Ρ‚Π΅Π»ΡŒΠ½ΠΎ Π²ΠΊΠ»ΡŽΡ‡Π΅Π½Π½Ρ‹Ρ… рСзисторов.
Из ΡƒΠΊΠ°Π·Π°Π½Π½Ρ‹Ρ… Ρ„ΠΎΡ€ΠΌΡƒΠ» слСдуСт Ρ‚Π°ΠΊΠΆΠ΅, Ρ‡Ρ‚ΠΎ напряТСния Ρ€Π°ΡΠΏΡ€Π΅Π΄Π΅Π»ΡΡŽΡ‚ΡΡ ΠΌΠ΅ΠΆΠ΄Ρƒ ΠΏΠΎΡΠ»Π΅Π΄ΠΎΠ²Π°Ρ‚Π΅Π»ΡŒΠ½ΠΎ соСдинСнными рСзисторами ΠΏΡ€ΠΎΠΏΠΎΡ€Ρ†ΠΈΠΎΠ½Π°Π»ΡŒΠ½ΠΎ ΠΈΡ… сопротивлСниям:

Ρ‚. Π΅. Ρ‡Π΅ΠΌ большС сопротивлСниС ΠΊΠ°ΠΊΠΎΠ³ΠΎ-Π»ΠΈΠ±ΠΎ ΠΏΡ€ΠΈΠ΅ΠΌΠ½ΠΈΠΊΠ° Π² ΠΏΠΎΡΠ»Π΅Π΄ΠΎΠ²Π°Ρ‚Π΅Π»ΡŒΠ½ΠΎΠΉ Ρ†Π΅ΠΏΠΈ, Ρ‚Π΅ΠΌ большС ΠΏΡ€ΠΈΠ»ΠΎΠΆΠ΅Π½Π½ΠΎΠ΅ ΠΊ Π½Π΅ΠΌΡƒ напряТСниС.

Π’ случаС Ссли ΠΏΠΎΡΠ»Π΅Π΄ΠΎΠ²Π°Ρ‚Π΅Π»ΡŒΠ½ΠΎ ΡΠΎΠ΅Π΄ΠΈΠ½ΡΡŽΡ‚ΡΡ нСсколько, Π½Π°ΠΏΡ€ΠΈΠΌΠ΅Ρ€ ΠΏ, рСзисторов с ΠΎΠ΄ΠΈΠ½Π°ΠΊΠΎΠ²Ρ‹ΠΌ сопротивлСниСм R1, эквивалСнтноС сопротивлСниС Ρ†Π΅ΠΏΠΈ Rэк Π±ΡƒΠ΄Π΅Ρ‚ Π² ΠΏ Ρ€Π°Π· большС сопротивлСния R1, Ρ‚. Π΅. Rэк = nR1. НапряТСниС U1 Π½Π° ΠΊΠ°ΠΆΠ΄ΠΎΠΌ рСзисторС Π² этом случаС Π² ΠΏ Ρ€Π°Π· мСньшС ΠΎΠ±Ρ‰Π΅Π³ΠΎ напряТСния U:

ΠŸΡ€ΠΈ ΠΏΠΎΡΠ»Π΅Π΄ΠΎΠ²Π°Ρ‚Π΅Π»ΡŒΠ½ΠΎΠΌ соСдинСнии ΠΏΡ€ΠΈΠ΅ΠΌΠ½ΠΈΠΊΠΎΠ² ΠΈΠ·ΠΌΠ΅Π½Π΅Π½ΠΈΠ΅ сопротивлСния ΠΎΠ΄Π½ΠΎΠ³ΠΎ ΠΈΠ· Π½ΠΈΡ… тотчас ΠΆΠ΅ Π²Π»Π΅Ρ‡Π΅Ρ‚ Π·Π° собой ΠΈΠ·ΠΌΠ΅Π½Π΅Π½ΠΈΠ΅ напряТСния Π½Π° Π΄Ρ€ΡƒΠ³ΠΈΡ… связанных с Π½ΠΈΠΌ ΠΏΡ€ΠΈΠ΅ΠΌΠ½ΠΈΠΊΠ°Ρ…. ΠŸΡ€ΠΈ Π²Ρ‹ΠΊΠ»ΡŽΡ‡Π΅Π½ΠΈΠΈ ΠΈΠ»ΠΈ ΠΎΠ±Ρ€Ρ‹Π²Π΅ элСктричСской Ρ†Π΅ΠΏΠΈ Π² ΠΎΠ΄Π½ΠΎΠΌ ΠΈΠ· ΠΏΡ€ΠΈΠ΅ΠΌΠ½ΠΈΠΊΠΎΠ² ΠΈ Π² ΠΎΡΡ‚Π°Π»ΡŒΠ½Ρ‹Ρ… ΠΏΡ€ΠΈΠ΅ΠΌΠ½ΠΈΠΊΠ°Ρ… прСкращаСтся Ρ‚ΠΎΠΊ. ΠŸΠΎΡΡ‚ΠΎΠΌΡƒ ΠΏΠΎΡΠ»Π΅Π΄ΠΎΠ²Π°Ρ‚Π΅Π»ΡŒΠ½ΠΎΠ΅ соСдинСниС ΠΏΡ€ΠΈΠ΅ΠΌΠ½ΠΈΠΊΠΎΠ² ΠΏΡ€ΠΈΠΌΠ΅Π½ΡΡŽΡ‚ Ρ€Π΅Π΄ΠΊΠΎ β€” Ρ‚ΠΎΠ»ΡŒΠΊΠΎ Π² Ρ‚ΠΎΠΌ случаС, ΠΊΠΎΠ³Π΄Π° напряТСниС источника элСктричСской энСргии большС номинального напряТСния, Π½Π° ΠΊΠΎΡ‚ΠΎΡ€ΠΎΠ΅ рассчитан ΠΏΠΎΡ‚Ρ€Π΅Π±ΠΈΡ‚Π΅Π»ΡŒ. НапримСр, напряТСниС Π² элСктричСской сСти, ΠΎΡ‚ ΠΊΠΎΡ‚ΠΎΡ€ΠΎΠΉ ΠΏΠΈΡ‚Π°ΡŽΡ‚ΡΡ Π²Π°Π³ΠΎΠ½Ρ‹ ΠΌΠ΅Ρ‚Ρ€ΠΎΠΏΠΎΠ»ΠΈΡ‚Π΅Π½Π°, составляСт 825 Π’, номинальноС ΠΆΠ΅ напряТСниС элСктричСских Π»Π°ΠΌΠΏ, примСняСмых Π² этих Π²Π°Π³ΠΎΠ½Π°Ρ…, 55 Π’. ΠŸΠΎΡΡ‚ΠΎΠΌΡƒ Π² Π²Π°Π³ΠΎΠ½Π°Ρ… ΠΌΠ΅Ρ‚Ρ€ΠΎΠΏΠΎΠ»ΠΈΡ‚Π΅Π½Π° элСктричСскиС Π»Π°ΠΌΠΏΡ‹ Π²ΠΊΠ»ΡŽΡ‡Π°ΡŽΡ‚ ΠΏΠΎΡΠ»Π΅Π΄ΠΎΠ²Π°Ρ‚Π΅Π»ΡŒΠ½ΠΎ ΠΏΠΎ 15 Π»Π°ΠΌΠΏ Π² ΠΊΠ°ΠΆΠ΄ΠΎΠΉ Ρ†Π΅ΠΏΠΈ.
ΠŸΠ°Ρ€Π°Π»Π»Π΅Π»ΡŒΠ½ΠΎΠ΅ соСдинСниС рСзисторов. ΠŸΡ€ΠΈ ΠΏΠ°Ρ€Π°Π»Π»Π΅Π»ΡŒΠ½ΠΎΠΌ соСдинСнии Π½Π΅ΡΠΊΠΎΠ»ΡŒΠΊΠΈΡ… ΠΏΡ€ΠΈΠ΅ΠΌΠ½ΠΈΠΊΠΎΠ² ΠΎΠ½ΠΈ Π²ΠΊΠ»ΡŽΡ‡Π°ΡŽΡ‚ΡΡ ΠΌΠ΅ΠΆΠ΄Ρƒ двумя Ρ‚ΠΎΡ‡ΠΊΠ°ΠΌΠΈ элСктричСской Ρ†Π΅ΠΏΠΈ, образуя ΠΏΠ°Ρ€Π°Π»Π»Π΅Π»ΡŒΠ½Ρ‹Π΅ Π²Π΅Ρ‚Π²ΠΈ (рис. 26, Π°). ЗамСняя

Рис. 26. Π‘Ρ…Π΅ΠΌΡ‹ ΠΏΠ°Ρ€Π°Π»Π»Π΅Π»ΡŒΠ½ΠΎΠ³ΠΎ соСдинСния ΠΏΡ€ΠΈΠ΅ΠΌΠ½ΠΈΠΊΠΎΠ²

Π»Π°ΠΌΠΏΡ‹ рСзисторами с сопротивлСниями R1, R2, R3, ΠΏΠΎΠ»ΡƒΡ‡ΠΈΠΌ схСму, ΠΏΠΎΠΊΠ°Π·Π°Π½Π½ΡƒΡŽ Π½Π° рис. 26, Π±.
ΠŸΡ€ΠΈ ΠΏΠ°Ρ€Π°Π»Π»Π΅Π»ΡŒΠ½ΠΎΠΌ соСдинСнии ΠΊΠΎ всСм рСзисторам ΠΏΡ€ΠΈΠ»ΠΎΠΆΠ΅Π½ΠΎ ΠΎΠ΄ΠΈΠ½Π°ΠΊΠΎΠ²ΠΎΠ΅ напряТСниС U. ΠŸΠΎΡΡ‚ΠΎΠΌΡƒ согласно Π·Π°ΠΊΠΎΠ½Ρƒ Ома:

Π’ΠΎΠΊ Π² Π½Π΅Ρ€Π°Π·Π²Π΅Ρ‚Π²Π»Π΅Π½Π½ΠΎΠΉ части Ρ†Π΅ΠΏΠΈ согласно ΠΏΠ΅Ρ€Π²ΠΎΠΌΡƒ Π·Π°ΠΊΠΎΠ½Ρƒ ΠšΠΈΡ€Ρ…Π³ΠΎΡ„Π° I = I1+I2+I3, ΠΈΠ»ΠΈ

Π‘Π»Π΅Π΄ΠΎΠ²Π°Ρ‚Π΅Π»ΡŒΠ½ΠΎ, эквивалСнтноС сопротивлСниС рассматриваСмой Ρ†Π΅ΠΏΠΈ ΠΏΡ€ΠΈ ΠΏΠ°Ρ€Π°Π»Π»Π΅Π»ΡŒΠ½ΠΎΠΌ соСдинСнии Ρ‚Ρ€Π΅Ρ… рСзисторов опрСдСляСтся Ρ„ΠΎΡ€ΠΌΡƒΠ»ΠΎΠΉ

1/Rэк = 1/R1 + 1/R2 + 1/R3 (24)

Вводя Π² Ρ„ΠΎΡ€ΠΌΡƒΠ»Ρƒ (24) вмСсто Π·Π½Π°Ρ‡Π΅Π½ΠΈΠΉ 1/Rэк, 1/R1, 1/R2 ΠΈ 1/R3 ΡΠΎΠΎΡ‚Π²Π΅Ρ‚ΡΡ‚Π²ΡƒΡŽΡ‰ΠΈΠ΅ проводимости Gэк, G1, G2 ΠΈ G3, ΠΏΠΎΠ»ΡƒΡ‡ΠΈΠΌ: эквивалСнтная ΠΏΡ€ΠΎΠ²ΠΎΠ΄ΠΈΠΌΠΎΡΡ‚ΡŒ ΠΏΠ°Ρ€Π°Π»Π»Π΅Π»ΡŒΠ½ΠΎΠΉ Ρ†Π΅ΠΏΠΈ Ρ€Π°Π²Π½Π° суммС проводимостСй ΠΏΠ°Ρ€Π°Π»Π»Π΅Π»ΡŒΠ½ΠΎ соСдинСнных рСзисторов:

Π’Π°ΠΊΠΈΠΌ ΠΎΠ±Ρ€Π°Π·ΠΎΠΌ, ΠΏΡ€ΠΈ ΡƒΠ²Π΅Π»ΠΈΡ‡Π΅Π½ΠΈΠΈ числа ΠΏΠ°Ρ€Π°Π»Π»Π΅Π»ΡŒΠ½ΠΎ Π²ΠΊΠ»ΡŽΡ‡Π°Π΅ΠΌΡ‹Ρ… рСзисторов Ρ€Π΅Π·ΡƒΠ»ΡŒΡ‚ΠΈΡ€ΡƒΡŽΡ‰Π°Ρ ΠΏΡ€ΠΎΠ²ΠΎΠ΄ΠΈΠΌΠΎΡΡ‚ΡŒ элСктричСской Ρ†Π΅ΠΏΠΈ увСличиваСтся, Π° Ρ€Π΅Π·ΡƒΠ»ΡŒΡ‚ΠΈΡ€ΡƒΡŽΡ‰Π΅Π΅ сопротивлСниС ΡƒΠΌΠ΅Π½ΡŒΡˆΠ°Π΅Ρ‚ΡΡ.
Из ΠΏΡ€ΠΈΠ²Π΅Π΄Π΅Π½Π½Ρ‹Ρ… Ρ„ΠΎΡ€ΠΌΡƒΠ» слСдуСт, Ρ‡Ρ‚ΠΎ Ρ‚ΠΎΠΊΠΈ Ρ€Π°ΡΠΏΡ€Π΅Π΄Π΅Π»ΡΡŽΡ‚ΡΡ ΠΌΠ΅ΠΆΠ΄Ρƒ ΠΏΠ°Ρ€Π°Π»Π»Π΅Π»ΡŒΠ½Ρ‹ΠΌΠΈ вСтвями ΠΎΠ±Ρ€Π°Ρ‚Π½ΠΎ ΠΏΡ€ΠΎΠΏΠΎΡ€Ρ†ΠΈΠΎΠ½Π°Π»ΡŒΠ½ΠΎ ΠΈΡ… элСктричСским сопротивлСниям ΠΈΠ»ΠΈ прямо ΠΏΡ€ΠΎΠΏΠΎΡ€Ρ†ΠΈΠΎΠ½Π°Π»ΡŒΠ½ΠΎ ΠΈΡ… проводимостям. НапримСр, ΠΏΡ€ΠΈ Ρ‚Ρ€Π΅Ρ… вСтвях

Π’ этом ΠΎΡ‚Π½ΠΎΡˆΠ΅Π½ΠΈΠΈ ΠΈΠΌΠ΅Π΅Ρ‚ мСсто полная аналогия ΠΌΠ΅ΠΆΠ΄Ρƒ распрСдСлСниСм Ρ‚ΠΎΠΊΠΎΠ² ΠΏΠΎ ΠΎΡ‚Π΄Π΅Π»ΡŒΠ½Ρ‹ΠΌ вСтвям ΠΈ распрСдСлСниСм ΠΏΠΎΡ‚ΠΎΠΊΠΎΠ² Π²ΠΎΠ΄Ρ‹ ΠΏΠΎ Ρ‚Ρ€ΡƒΠ±Π°ΠΌ.
ΠŸΡ€ΠΈΠ²Π΅Π΄Π΅Π½Π½Ρ‹Π΅ Ρ„ΠΎΡ€ΠΌΡƒΠ»Ρ‹ Π΄Π°ΡŽΡ‚ Π²ΠΎΠ·ΠΌΠΎΠΆΠ½ΠΎΡΡ‚ΡŒ ΠΎΠΏΡ€Π΅Π΄Π΅Π»ΠΈΡ‚ΡŒ эквивалСнтноС сопротивлСниС Ρ†Π΅ΠΏΠΈ для Ρ€Π°Π·Π»ΠΈΡ‡Π½Ρ‹Ρ… ΠΊΠΎΠ½ΠΊΡ€Π΅Ρ‚Π½Ρ‹Ρ… случаСв. НапримСр, ΠΏΡ€ΠΈ Π΄Π²ΡƒΡ… ΠΏΠ°Ρ€Π°Π»Π»Π΅Π»ΡŒΠ½ΠΎ Π²ΠΊΠ»ΡŽΡ‡Π΅Π½Π½Ρ‹Ρ… рСзисторах Ρ€Π΅Π·ΡƒΠ»ΡŒΡ‚ΠΈΡ€ΡƒΡŽΡ‰Π΅Π΅ сопротивлСниС Ρ†Π΅ΠΏΠΈ

ΠΏΡ€ΠΈ Ρ‚Ρ€Π΅Ρ… ΠΏΠ°Ρ€Π°Π»Π»Π΅Π»ΡŒΠ½ΠΎ Π²ΠΊΠ»ΡŽΡ‡Π΅Π½Π½Ρ‹Ρ… рСзисторах

ΠŸΡ€ΠΈ ΠΏΠ°Ρ€Π°Π»Π»Π΅Π»ΡŒΠ½ΠΎΠΌ соСдинСнии Π½Π΅ΡΠΊΠΎΠ»ΡŒΠΊΠΈΡ…, Π½Π°ΠΏΡ€ΠΈΠΌΠ΅Ρ€ n, рСзисторов с ΠΎΠ΄ΠΈΠ½Π°ΠΊΠΎΠ²Ρ‹ΠΌ сопротивлСниСм R1 Ρ€Π΅Π·ΡƒΠ»ΡŒΡ‚ΠΈΡ€ΡƒΡŽΡ‰Π΅Π΅ сопротивлСниС Ρ†Π΅ΠΏΠΈ Rэк Π±ΡƒΠ΄Π΅Ρ‚ Π² n Ρ€Π°Π· мСньшС сопротивлСния R1, Ρ‚.Π΅.

Rэк = R1 / n (27)

ΠŸΡ€ΠΎΡ…ΠΎΠ΄ΡΡ‰ΠΈΠΉ ΠΏΠΎ ΠΊΠ°ΠΆΠ΄ΠΎΠΉ Π²Π΅Ρ‚Π²ΠΈ Ρ‚ΠΎΠΊ I1, Π² этом случаС Π±ΡƒΠ΄Π΅Ρ‚ Π² ΠΏ Ρ€Π°Π· мСньшС ΠΎΠ±Ρ‰Π΅Π³ΠΎ Ρ‚ΠΎΠΊΠ°:

I1 = I / n (28)

ΠŸΡ€ΠΈ ΠΏΠ°Ρ€Π°Π»Π»Π΅Π»ΡŒΠ½ΠΎΠΌ соСдинСнии ΠΏΡ€ΠΈΠ΅ΠΌΠ½ΠΈΠΊΠΎΠ², всС ΠΎΠ½ΠΈ находятся ΠΏΠΎΠ΄ ΠΎΠ΄Π½ΠΈΠΌ ΠΈ Ρ‚Π΅ΠΌ ΠΆΠ΅ напряТСниСм, ΠΈ Ρ€Π΅ΠΆΠΈΠΌ Ρ€Π°Π±ΠΎΡ‚Ρ‹ ΠΊΠ°ΠΆΠ΄ΠΎΠ³ΠΎ ΠΈΠ· Π½ΠΈΡ… Π½Π΅ зависит ΠΎΡ‚ ΠΎΡΡ‚Π°Π»ΡŒΠ½Ρ‹Ρ…. Π­Ρ‚ΠΎ ΠΎΠ·Π½Π°Ρ‡Π°Π΅Ρ‚, Ρ‡Ρ‚ΠΎ Ρ‚ΠΎΠΊ, проходящий ΠΏΠΎ ΠΊΠ°ΠΊΠΎΠΌΡƒ-Π»ΠΈΠ±ΠΎ ΠΈΠ· ΠΏΡ€ΠΈΠ΅ΠΌΠ½ΠΈΠΊΠΎΠ², Π½Π΅ Π±ΡƒΠ΄Π΅Ρ‚ ΠΎΠΊΠ°Π·Ρ‹Π²Π°Ρ‚ΡŒ сущСствСнного влияния Π½Π° Π΄Ρ€ΡƒΠ³ΠΈΠ΅ ΠΏΡ€ΠΈΠ΅ΠΌΠ½ΠΈΠΊΠΈ. ΠŸΡ€ΠΈ всяком Π²Ρ‹ΠΊΠ»ΡŽΡ‡Π΅Π½ΠΈΠΈ ΠΈΠ»ΠΈ Π²Ρ‹Ρ…ΠΎΠ΄Π΅ ΠΈΠ· строя любого ΠΏΡ€ΠΈΠ΅ΠΌΠ½ΠΈΠΊΠ° ΠΎΡΡ‚Π°Π»ΡŒΠ½Ρ‹Π΅ ΠΏΡ€ΠΈΠ΅ΠΌΠ½ΠΈΠΊΠΈ ΠΎΡΡ‚Π°ΡŽΡ‚ΡΡ вклю-

Рис. 27. Π‘Ρ…Π΅ΠΌΡ‹ смСшанного соСдинСния ΠΏΡ€ΠΈΠ΅ΠΌΠ½ΠΈΠΊΠΎΠ²

Ρ‡Π΅Π½Π½Ρ‹ΠΌΠΈ. ΠŸΠΎΡΡ‚ΠΎΠΌΡƒ ΠΏΠ°Ρ€Π°Π»Π»Π΅Π»ΡŒΠ½ΠΎΠ΅ соСдинСниС ΠΈΠΌΠ΅Π΅Ρ‚ сущСствСнныС прСимущСства ΠΏΠ΅Ρ€Π΅Π΄ ΠΏΠΎΡΠ»Π΅Π΄ΠΎΠ²Π°Ρ‚Π΅Π»ΡŒΠ½Ρ‹ΠΌ, вслСдствиС Ρ‡Π΅Π³ΠΎ ΠΎΠ½ΠΎ ΠΏΠΎΠ»ΡƒΡ‡ΠΈΠ»ΠΎ Π½Π°ΠΈΠ±ΠΎΠ»Π΅Π΅ ΡˆΠΈΡ€ΠΎΠΊΠΎΠ΅ распространСниС. Π’ частности, элСктричСскиС Π»Π°ΠΌΠΏΡ‹ ΠΈ Π΄Π²ΠΈΠ³Π°Ρ‚Π΅Π»ΠΈ, ΠΏΡ€Π΅Π΄Π½Π°Π·Π½Π°Ρ‡Π΅Π½Π½Ρ‹Π΅ для Ρ€Π°Π±ΠΎΡ‚Ρ‹ ΠΏΡ€ΠΈ ΠΎΠΏΡ€Π΅Π΄Π΅Π»Π΅Π½Π½ΠΎΠΌ (номинальном) напряТСнии, всСгда Π²ΠΊΠ»ΡŽΡ‡Π°ΡŽΡ‚ ΠΏΠ°Ρ€Π°Π»Π»Π΅Π»ΡŒΠ½ΠΎ.
На элСктровозах постоянного Ρ‚ΠΎΠΊΠ° ΠΈ Π½Π΅ΠΊΠΎΡ‚ΠΎΡ€Ρ‹Ρ… Ρ‚Π΅ΠΏΠ»ΠΎΠ²ΠΎΠ·Π°Ρ… тяговыС Π΄Π²ΠΈΠ³Π°Ρ‚Π΅Π»ΠΈ Π² процСссС рСгулирования скорости двиТСния Π½ΡƒΠΆΠ½ΠΎ Π²ΠΊΠ»ΡŽΡ‡Π°Ρ‚ΡŒ ΠΏΠΎΠ΄ Ρ€Π°Π·Π»ΠΈΡ‡Π½Ρ‹Π΅ напряТСния, поэтому ΠΎΠ½ΠΈ Π² процСссС Ρ€Π°Π·Π³ΠΎΠ½Π° ΠΏΠ΅Ρ€Π΅ΠΊΠ»ΡŽΡ‡Π°ΡŽΡ‚ΡΡ с ΠΏΠΎΡΠ»Π΅Π΄ΠΎΠ²Π°Ρ‚Π΅Π»ΡŒΠ½ΠΎΠ³ΠΎ соСдинСния Π½Π° ΠΏΠ°Ρ€Π°Π»Π»Π΅Π»ΡŒΠ½ΠΎΠ΅.

БмСшанноС соСдинСниС рСзисторов. Π‘ΠΌΠ΅ΡˆΠ°Π½Π½Ρ‹ΠΌ соСдинСниСм называСтся Ρ‚Π°ΠΊΠΎΠ΅ соСдинСниС, ΠΏΡ€ΠΈ ΠΊΠΎΡ‚ΠΎΡ€ΠΎΠΌ Ρ‡Π°ΡΡ‚ΡŒ рСзисторов Π²ΠΊΠ»ΡŽΡ‡Π°Π΅Ρ‚ΡΡ ΠΏΠΎΡΠ»Π΅Π΄ΠΎΠ²Π°Ρ‚Π΅Π»ΡŒΠ½ΠΎ, Π° Ρ‡Π°ΡΡ‚ΡŒ β€” ΠΏΠ°Ρ€Π°Π»Π»Π΅Π»ΡŒΠ½ΠΎ. НапримСр, Π² схСмС рис. 27, Π° ΠΈΠΌΠ΅ΡŽΡ‚ΡΡ Π΄Π²Π° ΠΏΠΎΡΠ»Π΅Π΄ΠΎΠ²Π°Ρ‚Π΅Π»ΡŒΠ½ΠΎ Π²ΠΊΠ»ΡŽΡ‡Π΅Π½Π½Ρ‹Ρ… рСзистора сопротивлСниями R1 ΠΈ R2, ΠΏΠ°Ρ€Π°Π»Π»Π΅Π»ΡŒΠ½ΠΎ ΠΈΠΌ Π²ΠΊΠ»ΡŽΡ‡Π΅Π½ рСзистор сопротивлСниСм RΠ·, Π° рСзистор сопротивлСниСм R4 Π²ΠΊΠ»ΡŽΡ‡Π΅Π½ ΠΏΠΎΡΠ»Π΅Π΄ΠΎΠ²Π°Ρ‚Π΅Π»ΡŒΠ½ΠΎ с Π³Ρ€ΡƒΠΏΠΏΠΎΠΉ рСзисторов сопротивлСниями R1, R2 ΠΈ R3.
Π­ΠΊΠ²ΠΈΠ²Π°Π»Π΅Π½Ρ‚Π½ΠΎΠ΅ сопротивлСниС Ρ†Π΅ΠΏΠΈ ΠΏΡ€ΠΈ смСшанном соСдинСнии ΠΎΠ±Ρ‹Ρ‡Π½ΠΎ ΠΎΠΏΡ€Π΅Π΄Π΅Π»ΡΡŽΡ‚ ΠΌΠ΅Ρ‚ΠΎΠ΄ΠΎΠΌ прСобразования, ΠΏΡ€ΠΈ ΠΊΠΎΡ‚ΠΎΡ€ΠΎΠΌ ΡΠ»ΠΎΠΆΠ½ΡƒΡŽ Ρ†Π΅ΠΏΡŒ ΠΏΠΎΡΠ»Π΅Π΄ΠΎΠ²Π°Ρ‚Π΅Π»ΡŒΠ½Ρ‹ΠΌΠΈ этапами ΠΏΡ€Π΅ΠΎΠ±Ρ€Π°Π·ΠΎΠ²Ρ‹Π²Π°ΡŽΡ‚ Π² ΠΏΡ€ΠΎΡΡ‚Π΅ΠΉΡˆΡƒΡŽ. НапримСр, для схСмы рис. 27, Π° Π²Π½Π°Ρ‡Π°Π»Π΅ ΠΎΠΏΡ€Π΅Π΄Π΅Π»ΡΡŽΡ‚ эквивалСнтноС сопротивлСниС R12 ΠΏΠΎΡΠ»Π΅Π΄ΠΎΠ²Π°Ρ‚Π΅Π»ΡŒΠ½ΠΎ Π²ΠΊΠ»ΡŽΡ‡Π΅Π½Π½Ρ‹Ρ… рСзисторов с сопротивлСниями R1 ΠΈ R2: R12 = R1 + R2. ΠŸΡ€ΠΈ этом схСма рис. 27, Π° замСняСтся эквивалСнтной схСмой рис. 27, Π±. Π—Π°Ρ‚Π΅ΠΌ ΠΎΠΏΡ€Π΅Π΄Π΅Π»ΡΡŽΡ‚ эквивалСнтноС сопротивлСниС R123 ΠΏΠ°Ρ€Π°Π»Π»Π΅Π»ΡŒΠ½ΠΎ Π²ΠΊΠ»ΡŽΡ‡Π΅Π½Π½Ρ‹Ρ… сопротивлСний ΠΈ R3 ΠΏΠΎ Ρ„ΠΎΡ€ΠΌΡƒΠ»Π΅

ΠŸΡ€ΠΈ этом схСма рис. 27, Π± замСняСтся эквивалСнтной схСмой рис. 27, Π². ПослС этого находят эквивалСнтноС сопротивлСниС всСй Ρ†Π΅ΠΏΠΈ суммированиСм сопротивлСния R123 ΠΈ ΠΏΠΎΡΠ»Π΅Π΄ΠΎΠ²Π°Ρ‚Π΅Π»ΡŒΠ½ΠΎ Π²ΠΊΠ»ΡŽΡ‡Π΅Π½Π½ΠΎΠ³ΠΎ с Π½ΠΈΠΌ сопротивлСния R4:

ΠŸΠΎΡΠ»Π΅Π΄ΠΎΠ²Π°Ρ‚Π΅Π»ΡŒΠ½ΠΎΠ΅, ΠΏΠ°Ρ€Π°Π»Π»Π΅Π»ΡŒΠ½ΠΎΠ΅ ΠΈ смСшанноС соСдинСния ΡˆΠΈΡ€ΠΎΠΊΠΎ ΠΏΡ€ΠΈΠΌΠ΅Π½ΡΡŽΡ‚ для измСнСния сопротивлСния пусковых рСостатов ΠΏΡ€ΠΈ пускС э. ΠΏ. с. постоянного Ρ‚ΠΎΠΊΠ°.

ΠšΠ°Π»ΡŒΠΊΡƒΠ»ΡΡ‚ΠΎΡ€ соСдинСния рСзисторов ΠΎΠ½Π»Π°ΠΉΠ½. ΠŸΠ°Ρ€Π°Π»Π»Π΅Π»ΡŒΠ½ΠΎΠ΅ соСдинСниС рСзисторов

Π’ ΠΊΠ°ΠΆΠ΄ΠΎΠΉ элСктричСской схСмС присутствуСт рСзистор, ΠΈΠΌΠ΅ΡŽΡ‰ΠΈΠΉ сопротивлСниС элСктричСскому Ρ‚ΠΎΠΊΡƒ. РСзисторы Π±Ρ‹Π²Π°ΡŽΡ‚ Π΄Π²ΡƒΡ… Ρ‚ΠΈΠΏΠΎΠ²: постоянныС ΠΈ ΠΏΠ΅Ρ€Π΅ΠΌΠ΅Π½Π½Ρ‹Π΅. Π’ΠΎ врСмя Ρ€Π°Π·Ρ€Π°Π±ΠΎΡ‚ΠΊΠΈ любой элСктричСской схСмы ΠΈ Ρ€Π΅ΠΌΠΎΠ½Ρ‚Π° элСктронных ΠΈΠ·Π΄Π΅Π»ΠΈΠΉ часто приходится ΠΏΡ€ΠΈΠΌΠ΅Π½ΡΡ‚ΡŒ рСзистор, ΠΎΠ±Π»Π°Π΄Π°ΡŽΡ‰ΠΈΠΉ Π½Π΅ΠΎΠ±Ρ…ΠΎΠ΄ΠΈΠΌΡ‹ΠΌ Π½ΠΎΠΌΠΈΠ½Π°Π»ΠΎΠΌ.

НСсмотря Π½Π° Ρ‚ΠΎ Ρ‡Ρ‚ΠΎ для рСзисторов прСдусмотрСны Ρ€Π°Π·Π»ΠΈΡ‡Π½Ρ‹Π΅ Π½ΠΎΠΌΠΈΠ½Π°Π»Ρ‹
, ΠΌΠΎΠΆΠ΅Ρ‚ ΡΠ»ΡƒΡ‡ΠΈΡ‚ΡŒΡΡ Ρ‚Π°ΠΊ, Ρ‡Ρ‚ΠΎ Π½Π΅ Π±ΡƒΠ΄Π΅Ρ‚ возмоТности Π½Π°ΠΉΡ‚ΠΈ Π½Π΅ΠΎΠ±Ρ…ΠΎΠ΄ΠΈΠΌΡ‹ΠΉ ΠΈΠ»ΠΈ ΠΆΠ΅ Π²ΠΎΠΎΠ±Ρ‰Π΅ Π½ΠΈ ΠΎΠ΄ΠΈΠ½ элСмСнт Π½Π΅ смоТСт ΠΎΠ±Π΅ΡΠΏΠ΅Ρ‡ΠΈΡ‚ΡŒ Ρ‚Ρ€Π΅Π±ΡƒΠ΅ΠΌΡ‹ΠΉ ΠΏΠΎΠΊΠ°Π·Π°Ρ‚Π΅Π»ΡŒ.

РСшСниСм этой ΠΏΡ€ΠΎΠ±Π»Π΅ΠΌΡ‹ ΠΌΠΎΠΆΠ΅Ρ‚ ΡΡ‚Π°Ρ‚ΡŒ ΠΏΡ€ΠΈΠΌΠ΅Π½Π΅Π½ΠΈΠ΅ ΠΏΠΎΡΠ»Π΅Π΄ΠΎΠ²Π°Ρ‚Π΅Π»ΡŒΠ½ΠΎΠ³ΠΎ ΠΈ ΠΏΠ°Ρ€Π°Π»Π»Π΅Π»ΡŒΠ½ΠΎΠ³ΠΎ соСдинСния. Ознакомившись с этой ΡΡ‚Π°Ρ‚ΡŒΠ΅ΠΉ, Π²Ρ‹ ΡƒΠ·Π½Π°Π΅Ρ‚Π΅ ΠΎΠ± особСнностях выполнСния расчСта ΠΈ ΠΏΠΎΠ΄Π±ΠΎΡ€Π° Ρ€Π°Π·Π»ΠΈΡ‡Π½Ρ‹Ρ… Π½ΠΎΠΌΠΈΠ½Π°Π»ΠΎΠ² сопротивлСний.

ΠŸΠ°Ρ€Π°Π»Π»Π΅Π»ΡŒΠ½ΠΎΠ΅ соСдинСниС: общая информация

Часто ΠΏΡ€ΠΈ ΠΈΠ·Π³ΠΎΡ‚ΠΎΠ²Π»Π΅Π½ΠΈΠΈ ΠΊΠ°ΠΊΠΎΠ³ΠΎ-Π»ΠΈΠ±ΠΎ устройства ΠΈΡΠΏΠΎΠ»ΡŒΠ·ΡƒΡŽΡ‚ рСзисторы, ΠΊΠΎΡ‚ΠΎΡ€Ρ‹Π΅ ΡΠΎΠ΅Π΄ΠΈΠ½ΡΡŽΡ‚ΡΡ Π² соотвСтствии с ΠΏΠΎΡΠ»Π΅Π΄ΠΎΠ²Π°Ρ‚Π΅Π»ΡŒΠ½ΠΎΠΉ схСмой. Π­Ρ„Ρ„Π΅ΠΊΡ‚ ΠΎΡ‚ примСнСния Ρ‚Π°ΠΊΠΎΠ³ΠΎ Π²Π°Ρ€ΠΈΠ°Π½Ρ‚Π° сборки сводится ΠΊ ΡƒΠ²Π΅Π»ΠΈΡ‡Π΅Π½ΠΈΡŽ ΠΎΠ±Ρ‰Π΅Π³ΠΎ сопротивлСния Ρ†Π΅ΠΏΠΈ. Для Π΄Π°Π½Π½ΠΎΠ³ΠΎ Π²Π°Ρ€ΠΈΠ°Π½Ρ‚Π° соСдинСния элСмСнтов создаваСмоС ΠΈΠΌΠΈ сопротивлСниС рассчитываСтся ΠΊΠ°ΠΊ сумма Π½ΠΎΠΌΠΈΠ½Π°Π»ΠΎΠ². Если ΠΆΠ΅ сборка Π΄Π΅Ρ‚Π°Π»Π΅ΠΉ выполняСтся ΠΏΠΎ ΠΏΠ°Ρ€Π°Π»Π»Π΅Π»ΡŒΠ½ΠΎΠΉ схСмС, Ρ‚ΠΎ здСсь потрСбуСтся Ρ€Π°ΡΡΡ‡ΠΈΡ‚Π°Ρ‚ΡŒ сопротивлСниС
, ΠΈΡΠΏΠΎΠ»ΡŒΠ·ΡƒΡ ниТСописанныС Ρ„ΠΎΡ€ΠΌΡƒΠ»Ρ‹.

К схСмС ΠΏΠ°Ρ€Π°Π»Π»Π΅Π»ΡŒΠ½ΠΎΠ³ΠΎ соСдинСния ΠΏΡ€ΠΈΠ±Π΅Π³Π°ΡŽΡ‚ Π² ситуации, ΠΊΠΎΠ³Π΄Π° стоит Π·Π°Π΄Π°Ρ‡Π° ΠΏΠΎ сниТСнию суммарного сопротивлСния, Π°, ΠΏΠΎΠΌΠΈΠΌΠΎ этого, увСличСния мощности для Π³Ρ€ΡƒΠΏΠΏΡ‹ элСмСнтов, ΠΏΠΎΠ΄ΠΊΠ»ΡŽΡ‡Π΅Π½Π½Ρ‹Ρ… ΠΏΠΎ ΠΏΠ°Ρ€Π°Π»Π»Π΅Π»ΡŒΠ½ΠΎΠΉ схСмС, ΠΊΠΎΡ‚ΠΎΡ€ΠΎΠ΅ Π΄ΠΎΠ»ΠΆΠ½ΠΎ Π±Ρ‹Ρ‚ΡŒ большС, Ρ‡Π΅ΠΌ ΠΏΡ€ΠΈ ΠΈΡ… ΠΎΡ‚Π΄Π΅Π»ΡŒΠ½ΠΎΠΌ ΠΏΠΎΠ΄ΠΊΠ»ΡŽΡ‡Π΅Π½ΠΈΠΈ.

РасчСт сопротивлСния

Π’ случаС ΠΏΠΎΠ΄ΠΊΠ»ΡŽΡ‡Π΅Π½ΠΈΡ Π΄Π΅Ρ‚Π°Π»Π΅ΠΉ Π΄Ρ€ΡƒΠ³ с Π΄Ρ€ΡƒΠ³ΠΎΠΌ, с ΠΏΡ€ΠΈΠΌΠ΅Π½Π΅Π½ΠΈΠ΅ΠΌ ΠΏΠ°Ρ€Π°Π»Π»Π΅Π»ΡŒΠ½ΠΎΠΉ схСмы для расчСта суммарного сопротивлСния, Π±ΡƒΠ΄Π΅Ρ‚ ΠΈΡΠΏΠΎΠ»ΡŒΠ·ΠΎΠ²Π°Ρ‚ΡŒΡΡ ΡΠ»Π΅Π΄ΡƒΡŽΡ‰Π°Ρ Ρ„ΠΎΡ€ΠΌΡƒΠ»Π°:

R(ΠΎΠ±Ρ‰)=1/(1/R1+1/R2+1/R3+1/Rn).

  • R1- R3 ΠΈ Rn – рСзисторы, подсоСдинСнныС ΠΏΠΎ ΠΏΠ°Ρ€Π°Π»Π»Π΅Π»ΡŒΠ½ΠΎΠΉ схСмС.

ΠŸΡ€ΠΈΡ‡Π΅ΠΌ, Ссли Ρ†Π΅ΠΏΡŒ создаСтся Π½Π° основС Ρ‚ΠΎΠ»ΡŒΠΊΠΎ Π΄Π²ΡƒΡ… элСмСнтов, Ρ‚ΠΎ для опрСдСлСния суммарного номинального сопротивлСния слСдуСт ΠΈΡΠΏΠΎΠ»ΡŒΠ·ΠΎΠ²Π°Ρ‚ΡŒ Ρ‚Π°ΠΊΡƒΡŽ Ρ„ΠΎΡ€ΠΌΡƒΠ»Ρƒ:

R(ΠΎΠ±Ρ‰)=R1*R2/R1+R2.

  • R(ΠΎΠ±Ρ‰) – суммарноС сопротивлСниС;
  • R1 ΠΈ R2 – рСзисторы, подсоСдинСнныС ΠΏΠΎ ΠΏΠ°Ρ€Π°Π»Π»Π΅Π»ΡŒΠ½ΠΎΠΉ схСмС.

Π’ΠΈΠ΄Π΅ΠΎ: ΠŸΡ€ΠΈΠΌΠ΅Ρ€ расчёта сопротивлСния

Π£Π½ΠΈΠ²Π΅Ρ€ΡΠ°Π»ΡŒΠ½Π°Ρ схСма расчСта

ΠŸΡ€ΠΈΠΌΠ΅Π½ΠΈΡ‚Π΅Π»ΡŒΠ½ΠΎ ΠΊ Ρ€Π°Π΄ΠΈΠΎΡ‚Π΅Ρ…Π½ΠΈΠΊΠ΅ слСдуСт ΡƒΠ΄Π΅Π»ΠΈΡ‚ΡŒ Π²Π½ΠΈΠΌΠ°Π½ΠΈΠ΅ ΠΎΠ΄Π½ΠΎΠΌΡƒ Π²Π°ΠΆΠ½ΠΎΠΌΡƒ ΠΏΡ€Π°Π²ΠΈΠ»Ρƒ: Ссли ΠΏΠΎΠ΄ΠΊΠ»ΡŽΡ‡Π°Π΅ΠΌΡ‹Π΅ Π΄Ρ€ΡƒΠ³ ΠΊ Π΄Ρ€ΡƒΠ³Ρƒ элСмСнты ΠΏΠΎ ΠΏΠ°Ρ€Π°Π»Π»Π΅Π»ΡŒΠ½ΠΎΠΉ схСмС ΠΈΠΌΠ΅ΡŽΡ‚ ΠΎΠ΄ΠΈΠ½Π°ΠΊΠΎΠ²Ρ‹ΠΉ ΠΏΠΎΠΊΠ°Π·Π°Ρ‚Π΅Π»ΡŒ
, Ρ‚ΠΎ для расчСта суммарного Π½ΠΎΠΌΠΈΠ½Π°Π»Π° Π½Π΅ΠΎΠ±Ρ…ΠΎΠ΄ΠΈΠΌΠΎ ΠΎΠ±Ρ‰Π΅Π΅ Π·Π½Π°Ρ‡Π΅Π½ΠΈΠ΅ Ρ€Π°Π·Π΄Π΅Π»ΠΈΡ‚ΡŒ Π½Π° число ΠΏΠΎΠ΄ΠΊΠ»ΡŽΡ‡Π΅Π½Π½Ρ‹Ρ… ΡƒΠ·Π»ΠΎΠ²:

  • R(ΠΎΠ±Ρ‰) – суммарноС Π·Π½Π°Ρ‡Π΅Π½ΠΈΠ΅ сопротивлСния;
  • R – Π½ΠΎΠΌΠΈΠ½Π°Π» рСзистора, подсоСдинСнного ΠΏΠΎ ΠΏΠ°Ρ€Π°Π»Π»Π΅Π»ΡŒΠ½ΠΎΠΉ схСмС;
  • n – число ΠΏΠΎΠ΄ΠΊΠ»ΡŽΡ‡Π΅Π½Π½Ρ‹Ρ… ΡƒΠ·Π»ΠΎΠ².

ОсобоС Π²Π½ΠΈΠΌΠ°Π½ΠΈΠ΅ слСдуСт ΠΎΠ±Ρ€Π°Ρ‚ΠΈΡ‚ΡŒ Π½Π° Ρ‚ΠΎ, Ρ‡Ρ‚ΠΎ ΠΊΠΎΠ½Π΅Ρ‡Π½Ρ‹ΠΉ ΠΏΠΎΠΊΠ°Π·Π°Ρ‚Π΅Π»ΡŒ сопротивлСния Π² случаС использования ΠΏΠ°Ρ€Π°Π»Π»Π΅Π»ΡŒΠ½ΠΎΠΉ схСмы ΠΏΠΎΠ΄ΠΊΠ»ΡŽΡ‡Π΅Π½ΠΈΡ ΠΎΠ±ΡΠ·Π°Ρ‚Π΅Π»ΡŒΠ½ΠΎ Π±ΡƒΠ΄Π΅Ρ‚ мСньшС
ΠΏΠΎ ΡΡ€Π°Π²Π½Π΅Π½ΠΈΡŽ с Π½ΠΎΠΌΠΈΠ½Π°Π»ΠΎΠΌ любого элСмСнта, ΠΏΠΎΠ΄ΠΊΠ»ΡŽΡ‡Π°Π΅ΠΌΠΎΠ³ΠΎ Π² Ρ†Π΅ΠΏΡŒ.

ΠŸΡ€ΠΈΠΌΠ΅Ρ€ расчёта

Для большСй наглядности ΠΌΠΎΠΆΠ½ΠΎ Ρ€Π°ΡΡΠΌΠΎΡ‚Ρ€Π΅Ρ‚ΡŒ ΡΠ»Π΅Π΄ΡƒΡŽΡ‰ΠΈΠΉ ΠΏΡ€ΠΈΠΌΠ΅Ρ€: допустим, Ρƒ нас Π΅ΡΡ‚ΡŒ Ρ‚Ρ€ΠΈ рСзистора, Ρ‡ΡŒΠΈ Π½ΠΎΠΌΠΈΠ½Π°Π»Ρ‹ соотвСтствСнно Ρ€Π°Π²Π½Ρ‹ 100, 150 ΠΈ 30 Ом. Если Π²ΠΎΡΠΏΠΎΠ»ΡŒΠ·ΠΎΠ²Π°Ρ‚ΡŒΡΡ ΠΏΠ΅Ρ€Π²ΠΎΠΉ Ρ„ΠΎΡ€ΠΌΡƒΠ»ΠΎΠΉ для опрСдСлСния ΠΎΠ±Ρ‰Π΅Π³ΠΎ Π½ΠΎΠΌΠΈΠ½Π°Π»Π°, Ρ‚ΠΎ ΠΏΠΎΠ»ΡƒΡ‡ΠΈΠΌ ΡΠ»Π΅Π΄ΡƒΡŽΡ‰Π΅Π΅:

R(ΠΎΠ±Ρ‰)=1/(1/100+1/150+1/30)=

1/(0,01+0,007+0,03)=1/0,047=21,28Ом.

Если Π²Ρ‹ΠΏΠΎΠ»Π½ΠΈΡ‚ΡŒ нСслоТныС расчСты, Ρ‚ΠΎ ΠΌΠΎΠΆΠ½ΠΎ ΠΏΠΎΠ»ΡƒΡ‡ΠΈΡ‚ΡŒ ΡΠ»Π΅Π΄ΡƒΡŽΡ‰Π΅Π΅: для Ρ†Π΅ΠΏΠΈ, Π²ΠΊΠ»ΡŽΡ‡Π°ΡŽΡ‰Π΅ΠΉ Π² сСбя Ρ‚Ρ€ΠΈ Π΄Π΅Ρ‚Π°Π»ΠΈ, Π³Π΄Π΅ наимСньший ΠΏΠΎΠΊΠ°Π·Π°Ρ‚Π΅Π»ΡŒ сопротивлСния составляСт 30 Ом, Ρ€Π΅Π·ΡƒΠ»ΡŒΡ‚ΠΈΡ€ΡƒΡŽΡ‰Π΅Π΅ Π·Π½Π°Ρ‡Π΅Π½ΠΈΠ΅ Π½ΠΎΠΌΠΈΠ½Π°Π»Π° Π±ΡƒΠ΄Π΅Ρ‚ Ρ€Π°Π²Π½ΠΎ 21,28 Ом. Π­Ρ‚ΠΎΡ‚ ΠΏΠΎΠΊΠ°Π·Π°Ρ‚Π΅Π»ΡŒ Π±ΡƒΠ΄Π΅Ρ‚ мСньшС минимального значСния Π½ΠΎΠΌΠΈΠ½Π°Π»Π° Π² Ρ†Π΅ΠΏΠΈ практичСски Π½Π° 30%.

Π’Π°ΠΆΠ½Ρ‹Π΅ Π½ΡŽΠ°Π½ΡΡ‹

ΠžΠ±Ρ‹Ρ‡Π½ΠΎ для рСзисторов ΠΏΠ°Ρ€Π°Π»Π»Π΅Π»ΡŒΠ½ΠΎΠ΅ соСдинСниС примСняСтся Ρ‚ΠΎΠ³Π΄Π°, ΠΊΠΎΠ³Π΄Π° стоит Π·Π°Π΄Π°Ρ‡Π° ΠΏΠΎ созданию сопротивлСния большСй мощности. Для Π΅Π΅ Ρ€Π΅ΡˆΠ΅Π½ΠΈΡ ΠΏΠΎΡ‚Ρ€Π΅Π±ΡƒΡŽΡ‚ΡΡ рСзисторы, ΠΊΠΎΡ‚ΠΎΡ€Ρ‹Π΅ Π΄ΠΎΠ»ΠΆΠ½Ρ‹ ΠΈΠΌΠ΅Ρ‚ΡŒ Ρ€Π°Π²Π½Ρ‹Π΅ ΠΏΠΎΠΊΠ°Π·Π°Ρ‚Π΅Π»ΠΈ сопротивлСния ΠΈ мощности. ΠŸΡ€ΠΈ Ρ‚Π°ΠΊΠΎΠΌ Π²Π°Ρ€ΠΈΠ°Π½Ρ‚Π΅ ΠΎΠΏΡ€Π΅Π΄Π΅Π»ΠΈΡ‚ΡŒ ΠΎΠ±Ρ‰ΡƒΡŽ ΠΌΠΎΡ‰Π½ΠΎΡΡ‚ΡŒ ΠΌΠΎΠΆΠ½ΠΎ ΡΠ»Π΅Π΄ΡƒΡŽΡ‰ΠΈΠΌ ΠΎΠ±Ρ€Π°Π·ΠΎΠΌ
: ΠΌΠΎΡ‰Π½ΠΎΡΡ‚ΡŒ ΠΎΠ΄Π½ΠΎΠ³ΠΎ элСмСнта Π½Π΅ΠΎΠ±Ρ…ΠΎΠ΄ΠΈΠΌΠΎ ΠΏΠ΅Ρ€Π΅ΠΌΠ½ΠΎΠΆΠΈΡ‚ΡŒ с суммарным числом всСх рСзисторов, ΠΈΠ· ΠΊΠΎΡ‚ΠΎΡ€Ρ‹Ρ… состоит Ρ†Π΅ΠΏΡŒ, подсоСдинСнных Π΄Ρ€ΡƒΠ³ с Π΄Ρ€ΡƒΠ³ΠΎΠΌ Π² соотвСтствии с ΠΏΠ°Ρ€Π°Π»Π»Π΅Π»ΡŒΠ½ΠΎΠΉ схСмой.

Π‘ΠΊΠ°ΠΆΠ΅ΠΌ, Ссли Π½Π°ΠΌΠΈ Π±ΡƒΠ΄ΡƒΡ‚ ΠΈΡΠΏΠΎΠ»ΡŒΠ·ΠΎΠ²Π°Ρ‚ΡŒΡΡ ΠΏΡΡ‚ΡŒ рСзисторов, Ρ‡Π΅ΠΉ Π½ΠΎΠΌΠΈΠ½Π°Π» составляСт 100 Ом, Π° ΠΌΠΎΡ‰Π½ΠΎΡΡ‚ΡŒ ΠΊΠ°ΠΆΠ΄ΠΎΠ³ΠΎ Ρ€Π°Π²Π½Π° 1 Π’Ρ‚, ΠΊΠΎΡ‚ΠΎΡ€Ρ‹Π΅ присоСдинСны Π΄Ρ€ΡƒΠ³ ΠΊ Π΄Ρ€ΡƒΠ³Ρƒ Π² соотвСтствии с ΠΏΠ°Ρ€Π°Π»Π»Π΅Π»ΡŒΠ½ΠΎΠΉ схСмой, Ρ‚ΠΎ суммарный ΠΏΠΎΠΊΠ°Π·Π°Ρ‚Π΅Π»ΡŒ сопротивлСния Π±ΡƒΠ΄Π΅Ρ‚ Ρ€Π°Π²Π΅Π½ 20 Ом, Π° ΠΌΠΎΡ‰Π½ΠΎΡΡ‚ΡŒ составит 5 Π’Ρ‚.

Если Π²Π·ΡΡ‚ΡŒ Ρ‚Π΅ ΠΆΠ΅ рСзисторы, Π½ΠΎ ΠΏΠΎΠ΄ΡΠΎΠ΅Π΄ΠΈΠ½ΠΈΡ‚ΡŒ ΠΈΡ… Π² соотвСтствии с ΠΏΠΎΡΠ»Π΅Π΄ΠΎΠ²Π°Ρ‚Π΅Π»ΡŒΠ½ΠΎΠΉ схСмой, Ρ‚ΠΎ конСчная ΠΌΠΎΡ‰Π½ΠΎΡΡ‚ΡŒ составит 5 Π’Ρ‚, Π° суммарный Π½ΠΎΠΌΠΈΠ½Π°Π» Π±ΡƒΠ΄Π΅Ρ‚ Ρ€Π°Π²Π΅Π½ 500 Ом.

Π’ΠΈΠ΄Π΅ΠΎ: ΠŸΡ€Π°Π²ΠΈΠ»ΡŒΠ½ΠΎΠ΅ ΠΏΠΎΠ΄ΠΊΠ»ΡŽΡ‡Π΅Π½ΠΈΠ΅ свСтодиодов

ΠŸΠ°Ρ€Π°Π»Π»Π΅Π»ΡŒΠ½Π°Ρ схСма ΠΏΠΎΠ΄ΠΊΠ»ΡŽΡ‡Π΅Π½ΠΈΡ рСзисторов ΠΎΡ‡Π΅Π½ΡŒ вострСбована ΠΏΠΎ Ρ‚ΠΎΠΉ ΠΏΡ€ΠΈΡ‡ΠΈΠ½Π΅, Ρ‡Ρ‚ΠΎ часто Π²ΠΎΠ·Π½ΠΈΠΊΠ°Π΅Ρ‚ Π·Π°Π΄Π°Ρ‡Π° ΠΏΠΎ созданию Ρ‚Π°ΠΊΠΎΠ³ΠΎ Π½ΠΎΠΌΠΈΠ½Π°Π»Π°, ΠΊΠΎΡ‚ΠΎΡ€ΠΎΠ³ΠΎ Π½Π΅Π²ΠΎΠ·ΠΌΠΎΠΆΠ½ΠΎ Π΄ΠΎΠ±ΠΈΡ‚ΡŒΡΡ ΠΏΡ€ΠΈ ΠΏΠΎΠΌΠΎΡ‰ΠΈ простого ΠΏΠ°Ρ€Π°Π»Π»Π΅Π»ΡŒΠ½ΠΎΠ³ΠΎ соСдинСния. ΠŸΡ€ΠΈ этом ΠΏΡ€ΠΎΡ†Π΅Π΄ΡƒΡ€Π° расчСта этого ΠΏΠ°Ρ€Π°ΠΌΠ΅Ρ‚Ρ€Π° отличаСтся достаточной ΡΠ»ΠΎΠΆΠ½ΠΎΡΡ‚ΡŒΡŽ
, Π³Π΄Π΅ Π½Π΅ΠΎΠ±Ρ…ΠΎΠ΄ΠΈΠΌΠΎ ΡƒΡ‡ΠΈΡ‚Ρ‹Π²Π°Ρ‚ΡŒ Ρ€Π°Π·Π½Ρ‹Π΅ ΠΏΠ°Ρ€Π°ΠΌΠ΅Ρ‚Ρ€Ρ‹.

Π—Π΄Π΅ΡΡŒ ваТная Ρ€ΠΎΠ»ΡŒ отводится Π½Π΅ Ρ‚ΠΎΠ»ΡŒΠΊΠΎ количСству ΠΏΠΎΠ΄ΠΊΠ»ΡŽΡ‡Π°Π΅ΠΌΡ‹Ρ… элСмСнтов, Π½ΠΎ ΠΈ Ρ€Π°Π±ΠΎΡ‡ΠΈΠΌ ΠΏΠ°Ρ€Π°ΠΌΠ΅Ρ‚Ρ€Π°ΠΌ рСзисторов — ΠΏΡ€Π΅ΠΆΠ΄Π΅ всСго, ΡΠΎΠΏΡ€ΠΎΡ‚ΠΈΠ²Π»Π΅Π½ΠΈΡŽ ΠΈ мощности. Если ΠΎΠ΄ΠΈΠ½ ΠΈΠ· ΠΏΠΎΠ΄ΠΊΠ»ΡŽΡ‡Π°Π΅ΠΌΡ‹Ρ… элСмСнтов Π±ΡƒΠ΄Π΅Ρ‚ ΠΈΠΌΠ΅Ρ‚ΡŒ нСподходящий ΠΏΠΎΠΊΠ°Π·Π°Ρ‚Π΅Π»ΡŒ, Ρ‚ΠΎ это Π½Π΅ ΠΏΠΎΠ·Π²ΠΎΠ»ΠΈΡ‚ эффСктивно Ρ€Π΅ΡˆΠΈΡ‚ΡŒ Π·Π°Π΄Π°Ρ‡Ρƒ ΠΏΠΎ созданию Ρ‚Ρ€Π΅Π±ΡƒΠ΅ΠΌΠΎΠ³ΠΎ Π½ΠΎΠΌΠΈΠ½Π°Π»Π° Π² Ρ†Π΅ΠΏΠΈ.

ΠŸΠ°Ρ€Π°Π»Π»Π΅Π»ΡŒΠ½ΠΎΠ΅ соСдинСниС рСзисторов
β€” ΠΎΠ΄Π½ΠΎ ΠΈΠ· Π΄Π²ΡƒΡ… Π²ΠΈΠ΄ΠΎΠ² элСктричСских соСдинСний, ΠΊΠΎΠ³Π΄Π° ΠΎΠ±Π° Π²Ρ‹Π²ΠΎΠ΄Π° ΠΎΠ΄Π½ΠΎΠ³ΠΎ рСзистора соСдинСны с ΡΠΎΠΎΡ‚Π²Π΅Ρ‚ΡΡ‚Π²ΡƒΡŽΡ‰ΠΈΠΌΠΈ Π²Ρ‹Π²ΠΎΠ΄Π°ΠΌΠΈ Π΄Ρ€ΡƒΠ³ΠΎΠ³ΠΎ рСзистора ΠΈΠ»ΠΈ рСзисторов. Π—Π°Ρ‡Π°ΡΡ‚ΡƒΡŽ ΠΈΠ»ΠΈ ΠΏΠ°Ρ€Π°Π»Π»Π΅Π»ΡŒΠ½ΠΎ для Ρ‚ΠΎΠ³ΠΎ, Ρ‡Ρ‚ΠΎΠ±Ρ‹ ΡΠΎΠ·Π΄Π°Ρ‚ΡŒ Π±ΠΎΠ»Π΅Π΅ слоТныС элСктронныС схСмы.

Π‘Ρ…Π΅ΠΌΠ° ΠΏΠ°Ρ€Π°Π»Π»Π΅Π»ΡŒΠ½ΠΎΠ³ΠΎ соСдинСния ΠΏΠΎΠΊΠ°Π·Π°Π½ Π½Π° рисункС Π½ΠΈΠΆΠ΅. ΠŸΡ€ΠΈ ΠΏΠ°Ρ€Π°Π»Π»Π΅Π»ΡŒΠ½ΠΎΠΌ соСдинСнии рСзисторов, напряТСниС Π½Π° всСх рСзисторах Π±ΡƒΠ΄Π΅Ρ‚ ΠΎΠ΄ΠΈΠ½Π°ΠΊΠΎΠ²Ρ‹ΠΌ, Π° ΠΏΡ€ΠΎΡ‚Π΅ΠΊΠ°ΡŽΡ‰ΠΈΠΉ Ρ‡Π΅Ρ€Π΅Π· Π½ΠΈΡ… Ρ‚ΠΎΠΊ Π±ΡƒΠ΄Π΅Ρ‚ ΠΏΡ€ΠΎΠΏΠΎΡ€Ρ†ΠΈΠΎΠ½Π°Π»Π΅Π½ ΠΈΡ… ΡΠΎΠΏΡ€ΠΎΡ‚ΠΈΠ²Π»Π΅Π½ΠΈΡŽ:

Π€ΠΎΡ€ΠΌΡƒΠ»Π° ΠΏΠ°Ρ€Π°Π»Π»Π΅Π»ΡŒΠ½ΠΎΠ³ΠΎ соСдинСния рСзисторов

ΠžΠ±Ρ‰Π΅Π΅ сопротивлСниС Π½Π΅ΡΠΊΠΎΠ»ΡŒΠΊΠΈΡ… рСзисторов соСдинСнных ΠΏΠ°Ρ€Π°Π»Π»Π΅Π»ΡŒΠ½ΠΎ опрСдСляСтся ΠΏΠΎ ΡΠ»Π΅Π΄ΡƒΡŽΡ‰Π΅ΠΉ Ρ„ΠΎΡ€ΠΌΡƒΠ»Π΅:

Π’ΠΎΠΊ, ΠΏΡ€ΠΎΡ‚Π΅ΠΊΠ°ΡŽΡ‰ΠΈΠΉ Ρ‡Π΅Ρ€Π΅Π· ΠΎΡ‚Π΄Π΅Π»ΡŒΠ½ΠΎ взятый рСзистор, согласно , ΠΌΠΎΠΆΠ½ΠΎ Π½Π°ΠΉΡ‚ΠΈ ΠΏΠΎ Ρ„ΠΎΡ€ΠΌΡƒΠ»Π΅:

ΠŸΠ°Ρ€Π°Π»Π»Π΅Π»ΡŒΠ½ΠΎΠ΅ соСдинСниС рСзисторов β€” расчСт

ΠŸΡ€ΠΈΠΌΠ΅Ρ€ β„–1

ΠŸΡ€ΠΈ Ρ€Π°Π·Ρ€Π°Π±ΠΎΡ‚ΠΊΠ΅ устройства, Π²ΠΎΠ·Π½ΠΈΠΊΠ»Π° Π½Π΅ΠΎΠ±Ρ…ΠΎΠ΄ΠΈΠΌΠΎΡΡ‚ΡŒ ΡƒΡΡ‚Π°Π½ΠΎΠ²ΠΈΡ‚ΡŒ рСзистор с сопротивлСниСм 8 Ом. Если ΠΌΡ‹ просмотрим вСсь Π½ΠΎΠΌΠΈΠ½Π°Π»ΡŒΠ½Ρ‹ΠΉ ряд стандартных Π·Π½Π°Ρ‡Π΅Π½ΠΈΠΉ рСзисторов, Ρ‚ΠΎ ΠΌΡ‹ ΡƒΠ²ΠΈΠ΄ΠΈΠΌ, Ρ‡Ρ‚ΠΎ рСзистора с сопротивлСниСм Π² 8 Ом Π² Π½Π΅ΠΌ Π½Π΅Ρ‚.

Π’Ρ‹Ρ…ΠΎΠ΄ΠΎΠΌ ΠΈΠ· Π΄Π°Π½Π½ΠΎΠΉ ситуации Π±ΡƒΠ΄Π΅Ρ‚ использованиС Π΄Π²ΡƒΡ… ΠΏΠ°Ρ€Π°Π»Π»Π΅Π»ΡŒΠ½ΠΎ соСдинСнных рСзисторов. Π­ΠΊΠ²ΠΈΠ²Π°Π»Π΅Π½Ρ‚Π½ΠΎΠ΅ Π·Π½Π°Ρ‡Π΅Π½ΠΈΠ΅ сопротивлСния для Π΄Π²ΡƒΡ… рСзисторов соСдинСнных ΠΏΠ°Ρ€Π°Π»Π»Π΅Π»ΡŒΠ½ΠΎ рассчитываСтся ΡΠ»Π΅Π΄ΡƒΡŽΡ‰ΠΈΠΌ ΠΎΠ±Ρ€Π°Π·ΠΎΠΌ:

Π”Π°Π½Π½ΠΎΠ΅ ΡƒΡ€Π°Π²Π½Π΅Π½ΠΈΠ΅ ΠΏΠΎΠΊΠ°Π·Ρ‹Π²Π°Π΅Ρ‚, Ρ‡Ρ‚ΠΎ Ссли R1 Ρ€Π°Π²Π΅Π½ R2, Ρ‚ΠΎ сопротивлСниС R составляСт ΠΏΠΎΠ»ΠΎΠ²ΠΈΠ½Ρƒ сопротивлСния ΠΎΠ΄Π½ΠΎΠ³ΠΎ ΠΈΠ· Π΄Π²ΡƒΡ… рСзисторов. ΠŸΡ€ΠΈ R = 8 Ом, R1 ΠΈ R2 Π΄ΠΎΠ»ΠΆΠ½Ρ‹, ΡΠ»Π΅Π΄ΠΎΠ²Π°Ρ‚Π΅Π»ΡŒΠ½ΠΎ, ΠΈΠΌΠ΅Ρ‚ΡŒ Π·Π½Π°Ρ‡Π΅Π½ΠΈΠ΅ 2 Γ— 8 = 16 Ом.
Π’Π΅ΠΏΠ΅Ρ€ΡŒ ΠΏΡ€ΠΎΠ²Π΅Π΄Π΅ΠΌ ΠΏΡ€ΠΎΠ²Π΅Ρ€ΠΊΡƒ, рассчитав ΠΎΠ±Ρ‰Π΅Π΅ сопротивлСниС Π΄Π²ΡƒΡ… рСзисторов:

Π’Π°ΠΊΠΈΠΌ ΠΎΠ±Ρ€Π°Π·ΠΎΠΌ, ΠΌΡ‹ ΠΏΠΎΠ»ΡƒΡ‡ΠΈΠ»ΠΈ Π½Π΅ΠΎΠ±Ρ…ΠΎΠ΄ΠΈΠΌΠΎΠ΅ сопротивлСниС 8 Ом, соСдинив ΠΏΠ°Ρ€Π°Π»Π»Π΅Π»ΡŒΠ½ΠΎ Π΄Π²Π° рСзистора ΠΏΠΎ 16 Ом.

ΠŸΡ€ΠΈΠΌΠ΅Ρ€ расчСта β„–2

Найти ΠΎΠ±Ρ‰Π΅Π΅ сопротивлСниС R ΠΈΠ· Ρ‚Ρ€Π΅Ρ… ΠΏΠ°Ρ€Π°Π»Π»Π΅Π»ΡŒΠ½ΠΎ соСдинСнных рСзисторов:

ΠžΠ±Ρ‰Π΅Π΅ сопротивлСниС R рассчитываСтся ΠΏΠΎ Ρ„ΠΎΡ€ΠΌΡƒΠ»Π΅:

Π­Ρ‚ΠΎΡ‚ ΠΌΠ΅Ρ‚ΠΎΠ΄ расчСта ΠΌΠΎΠΆΠ΅Ρ‚ Π±Ρ‹Ρ‚ΡŒ ΠΈΡΠΏΠΎΠ»ΡŒΠ·ΠΎΠ²Π°Π½Ρ‹ для расчСта любого количСства ΠΎΡ‚Π΄Π΅Π»ΡŒΠ½Ρ‹Ρ… сопротивлСний соСдинСнных ΠΏΠ°Ρ€Π°Π»Π»Π΅Π»ΡŒΠ½ΠΎ.

Один Π²Π°ΠΆΠ½Ρ‹ΠΉ ΠΌΠΎΠΌΠ΅Π½Ρ‚, ΠΊΠΎΡ‚ΠΎΡ€Ρ‹ΠΉ Π½Π΅ΠΎΠ±Ρ…ΠΎΠ΄ΠΈΠΌΠΎ Π·Π°ΠΏΠΎΠΌΠ½ΠΈΡ‚ΡŒ ΠΏΡ€ΠΈ расчСтС ΠΏΠ°Ρ€Π°Π»Π»Π΅Π»ΡŒΠ½ΠΎ соСдинСнных рСзисторов – это Ρ‚ΠΎ, Ρ‡Ρ‚ΠΎ ΠΎΠ±Ρ‰Π΅Π΅ сопротивлСниС всСгда Π±ΡƒΠ΄Π΅Ρ‚ мСньшС, Ρ‡Π΅ΠΌ Π·Π½Π°Ρ‡Π΅Π½ΠΈΠ΅ наимСньшСго сопротивлСния Π² этой ΠΊΠΎΠΌΠ±ΠΈΠ½Π°Ρ†ΠΈΠΈ.

Как Ρ€Π°ΡΡΡ‡ΠΈΡ‚Π°Ρ‚ΡŒ слоТныС схСмы соСдинСния рСзисторов

Π‘ΠΎΠ»Π΅Π΅ слоТныС соСдинСния рСзисторов ΠΌΠΎΠ³ΡƒΡ‚ Π±Ρ‹Ρ‚ΡŒ рассчитаны ΠΏΡƒΡ‚Π΅ΠΌ систСматичСской Π³Ρ€ΡƒΠΏΠΏΠΈΡ€ΠΎΠ²ΠΊΠΈ рСзисторов. На рисункС Π½ΠΈΠΆΠ΅ Π½Π΅ΠΎΠ±Ρ…ΠΎΠ΄ΠΈΠΌΠΎ ΠΏΠΎΡΡ‡ΠΈΡ‚Π°Ρ‚ΡŒ ΠΎΠ±Ρ‰Π΅Π΅ сопротивлСниС Ρ†Π΅ΠΏΠΈ, состоящСй ΠΈΠ· Ρ‚Ρ€Π΅Ρ… рСзисторов:

Для простоты расчСта, сначала сгруппируСм рСзисторы ΠΏΠΎ ΠΏΠ°Ρ€Π°Π»Π»Π΅Π»ΡŒΠ½ΠΎΠΌΡƒ ΠΈ ΠΏΠΎΡΠ»Π΅Π΄ΠΎΠ²Π°Ρ‚Π΅Π»ΡŒΠ½ΠΎΠΌΡƒ Ρ‚ΠΈΠΏΡƒ соСдинСния.
РСзисторы R2 ΠΈ R3 соСдинСны ΠΏΠΎΡΠ»Π΅Π΄ΠΎΠ²Π°Ρ‚Π΅Π»ΡŒΠ½ΠΎ (Π³Ρ€ΡƒΠΏΠΏΠ° 2). Они Π² свою ΠΎΡ‡Π΅Ρ€Π΅Π΄ΡŒ соСдинСны ΠΏΠ°Ρ€Π°Π»Π»Π΅Π»ΡŒΠ½ΠΎ с рСзистором R1 (Π³Ρ€ΡƒΠΏΠΏΠ° 1).

ΠŸΠΎΡΠ»Π΅Π΄ΠΎΠ²Π°Ρ‚Π΅Π»ΡŒΠ½ΠΎΠ΅ соСдинСниС рСзисторов Π³Ρ€ΡƒΠΏΠΏΡ‹ 2 вычисляСтся ΠΊΠ°ΠΊ сумма сопротивлСний R2 ΠΈ R3:

Π’ Ρ€Π΅Π·ΡƒΠ»ΡŒΡ‚Π°Ρ‚Π΅ ΠΌΡ‹ ΡƒΠΏΡ€ΠΎΡ‰Π°Π΅ΠΌ схСму Π² Π²ΠΈΠ΄Π΅ Π΄Π²ΡƒΡ… ΠΏΠ°Ρ€Π°Π»Π»Π΅Π»ΡŒΠ½Ρ‹Ρ… рСзисторов. Π’Π΅ΠΏΠ΅Ρ€ΡŒ ΠΎΠ±Ρ‰Π΅Π΅ сопротивлСниС всСй схСмы ΠΌΠΎΠΆΠ½ΠΎ ΠΏΠΎΡΡ‡ΠΈΡ‚Π°Ρ‚ΡŒ ΡΠ»Π΅Π΄ΡƒΡŽΡ‰ΠΈΠΌ ΠΎΠ±Ρ€Π°Π·ΠΎΠΌ:

РасчСт Π±ΠΎΠ»Π΅Π΅ слоТных соСдинСний рСзисторов ΠΌΠΎΠΆΠ½ΠΎ Π²Ρ‹ΠΏΠΎΠ»Π½ΠΈΡ‚ΡŒ ΠΈΡΠΏΠΎΠ»ΡŒΠ·ΡƒΡ Π·Π°ΠΊΠΎΠ½Ρ‹ ΠšΠΈΡ€Ρ…Π³ΠΎΡ„Π°.

Π’ΠΎΠΊ, ΠΏΡ€ΠΎΡ‚Π΅ΠΊΠ°ΡŽΡ‰ΠΈΠΉ Π² Ρ†Π΅ΠΏΠΈ ΠΏΠ°Ρ€Π°Π»Π»Π΅Π»ΡŒΠ½ΠΎ соСдинСнных рСзисторах

ΠžΠ±Ρ‰ΠΈΠΉ Ρ‚ΠΎΠΊ I ΠΏΡ€ΠΎΡ‚Π΅ΠΊΠ°ΡŽΡ‰ΠΈΠΉ Π² Ρ†Π΅ΠΏΠΈ ΠΏΠ°Ρ€Π°Π»Π»Π΅Π»ΡŒΠ½Ρ‹Ρ… рСзисторов равняСтся суммС ΠΎΡ‚Π΄Π΅Π»ΡŒΠ½Ρ‹Ρ… Ρ‚ΠΎΠΊΠΎΠ², ΠΏΡ€ΠΎΡ‚Π΅ΠΊΠ°ΡŽΡ‰ΠΈΡ… Π²ΠΎ всСх ΠΏΠ°Ρ€Π°Π»Π»Π΅Π»ΡŒΠ½Ρ‹Ρ… вСтвях, ΠΏΡ€ΠΈΡ‡Π΅ΠΌ Ρ‚ΠΎΠΊ Π² ΠΎΡ‚Π΄Π΅Π»ΡŒΠ½ΠΎ взятой Π²Π΅Ρ‚Π²ΠΈ Π½Π΅ ΠΎΠ±ΡΠ·Π°Ρ‚Π΅Π»ΡŒΠ½ΠΎ Π΄ΠΎΠ»ΠΆΠ΅Π½ Π±Ρ‹Ρ‚ΡŒ Ρ€Π°Π²Π΅Π½ Ρ‚ΠΎΠΊΡƒ Π² сосСдних вСтвях.

НСсмотря Π½Π° ΠΏΠ°Ρ€Π°Π»Π»Π΅Π»ΡŒΠ½ΠΎΠ΅ соСдинСниС, ΠΊ ΠΊΠ°ΠΆΠ΄ΠΎΠΌΡƒ рСзистору ΠΏΡ€ΠΈΠ»ΠΎΠΆΠ΅Π½ΠΎ ΠΎΠ΄Π½ΠΎ ΠΈ Ρ‚ΠΎ ΠΆΠ΅ напряТСниС. А ΠΏΠΎΡΠΊΠΎΠ»ΡŒΠΊΡƒ Π²Π΅Π»ΠΈΡ‡ΠΈΠ½Π° сопротивлСний Π² ΠΏΠ°Ρ€Π°Π»Π»Π΅Π»ΡŒΠ½ΠΎΠΉ Ρ†Π΅ΠΏΠΈ ΠΌΠΎΠΆΠ΅Ρ‚ Π±Ρ‹Ρ‚ΡŒ Ρ€Π°Π·Π½ΠΎΠΉ, Ρ‚ΠΎ ΠΈ Π²Π΅Π»ΠΈΡ‡ΠΈΠ½Π° ΠΏΡ€ΠΎΡ‚Π΅ΠΊΠ°ΡŽΡ‰Π΅Π³ΠΎ Ρ‚ΠΎΠΊΠ° Ρ‡Π΅Ρ€Π΅Π· ΠΊΠ°ΠΆΠ΄Ρ‹ΠΉ рСзистор Ρ‚ΠΎΠΆΠ΅ Π±ΡƒΠ΄Π΅Ρ‚ ΠΎΡ‚Π»ΠΈΡ‡Π°Ρ‚ΡŒΡΡ (ΠΏΠΎ ΠΎΠΏΡ€Π΅Π΄Π΅Π»Π΅Π½ΠΈΡŽ Π·Π°ΠΊΠΎΠ½Π° Ома).

Рассмотрим это Π½Π° ΠΏΡ€ΠΈΠΌΠ΅Ρ€Π΅ Π΄Π²ΡƒΡ… ΠΏΠ°Ρ€Π°Π»Π»Π΅Π»ΡŒΠ½ΠΎ соСдинСнных рСзисторов. Π’ΠΎΠΊ, ΠΊΠΎΡ‚ΠΎΡ€Ρ‹ΠΉ Ρ‚Π΅Ρ‡Π΅Ρ‚ Ρ‡Π΅Ρ€Π΅Π· ΠΊΠ°ΠΆΠ΄Ρ‹ΠΉ ΠΈΠ· рСзисторов (I1 ΠΈ I2) Π±ΡƒΠ΄Π΅Ρ‚ ΠΎΡ‚Π»ΠΈΡ‡Π°Ρ‚ΡŒΡΡ Π΄Ρ€ΡƒΠ³ ΠΎΡ‚ Π΄Ρ€ΡƒΠ³Π° ΠΏΠΎΡΠΊΠΎΠ»ΡŒΠΊΡƒ сопротивлСния рСзисторов R1 ΠΈ R2 Π½Π΅ Ρ€Π°Π²Π½Ρ‹.
Однако ΠΌΡ‹ Π·Π½Π°Π΅ΠΌ, Ρ‡Ρ‚ΠΎ Ρ‚ΠΎΠΊ, ΠΊΠΎΡ‚ΠΎΡ€Ρ‹ΠΉ поступаСт Π² Ρ†Π΅ΠΏΡŒ Π² Ρ‚ΠΎΡ‡ΠΊΠ΅ «А» Π΄ΠΎΠ»ΠΆΠ΅Π½ Π²Ρ‹ΠΉΡ‚ΠΈ ΠΈΠ· Ρ†Π΅ΠΏΠΈ Π² Ρ‚ΠΎΡ‡ΠΊΠ΅ Β«BΒ» .

ΠŸΠ΅Ρ€Π²ΠΎΠ΅ ΠΏΡ€Π°Π²ΠΈΠ»ΠΎ ΠšΠΈΡ€Ρ…Π³ΠΎΡ„Π° гласит: Β«ΠžΠ±Ρ‰ΠΈΠΉ Ρ‚ΠΎΠΊ, выходящий ΠΈΠ· Ρ†Π΅ΠΏΠΈ Ρ€Π°Π²Π΅Π½ Ρ‚ΠΎΠΊΡƒ входящий Π² Ρ†Π΅ΠΏΡŒΒ».

Π’Π°ΠΊΠΈΠΌ ΠΎΠ±Ρ€Π°Π·ΠΎΠΌ, ΠΏΡ€ΠΎΡ‚Π΅ΠΊΠ°ΡŽΡ‰ΠΈΠΉ ΠΎΠ±Ρ‰ΠΈΠΉ Ρ‚ΠΎΠΊ Π² Ρ†Π΅ΠΏΠΈ ΠΌΠΎΠΆΠ½ΠΎ ΠΎΠΏΡ€Π΅Π΄Π΅Π»ΠΈΡ‚ΡŒ ΠΊΠ°ΠΊ:

Π—Π°Ρ‚Π΅ΠΌ с ΠΏΠΎΠΌΠΎΡ‰ΡŒΡŽ Π·Π°ΠΊΠΎΠ½Π° Ома ΠΌΠΎΠΆΠ½ΠΎ Π²Ρ‹Ρ‡ΠΈΡΠ»ΠΈΡ‚ΡŒ Ρ‚ΠΎΠΊ, ΠΊΠΎΡ‚ΠΎΡ€Ρ‹ΠΉ ΠΏΡ€ΠΎΡ‚Π΅ΠΊΠ°Π΅Ρ‚ Ρ‡Π΅Ρ€Π΅Π· ΠΊΠ°ΠΆΠ΄Ρ‹ΠΉ рСзистор:

Π’ΠΎΠΊ, ΠΏΡ€ΠΎΡ‚Π΅ΠΊΠ°ΡŽΡ‰ΠΈΠΉ Π² R1 = U Γ· R1 = 12 Γ· 22 кОм = 0,545 мА

Π’ΠΎΠΊ, ΠΏΡ€ΠΎΡ‚Π΅ΠΊΠ°ΡŽΡ‰ΠΈΠΉ Π² R 2 = U Γ· R2 = 12 Γ· 47 кОм = 0,255 мА

Π’Π°ΠΊΠΈΠΌ ΠΎΠ±Ρ€Π°Π·ΠΎΠΌ, ΠΎΠ±Ρ‰ΠΈΠΉ Ρ‚ΠΎΠΊ Π±ΡƒΠ΄Π΅Ρ‚ Ρ€Π°Π²Π΅Π½:

I = 0,545 мА + 0,255 мА = 0,8 мА

Π­Ρ‚ΠΎ Ρ‚Π°ΠΊΠΆΠ΅ ΠΌΠΎΠΆΠ½ΠΎ ΠΏΡ€ΠΎΠ²Π΅Ρ€ΠΈΡ‚ΡŒ, ΠΈΡΠΏΠΎΠ»ΡŒΠ·ΡƒΡ Π·Π°ΠΊΠΎΠ½ Ома:

I = U Γ· R = 12 Π’ Γ· 15 кОм = 0,8 мА (Ρ‚ΠΎ ΠΆΠ΅ самоС)

Π³Π΄Π΅ 15кОм β€” это ΠΎΠ±Ρ‰Π΅Π΅ сопротивлСниС Π΄Π²ΡƒΡ… ΠΏΠ°Ρ€Π°Π»Π»Π΅Π»ΡŒΠ½ΠΎ соСдинСнных рСзисторов (22 кОм ΠΈ 47 кОм)

И Π² Π·Π°Π²Π΅Ρ€ΡˆΠ΅Π½ΠΈΠΈ хочСтся ΠΎΡ‚ΠΌΠ΅Ρ‚ΠΈΡ‚ΡŒ, Ρ‡Ρ‚ΠΎ Π±ΠΎΠ»ΡŒΡˆΠΈΠ½ΡΡ‚Π²ΠΎ соврСмСнных рСзисторов ΠΌΠ°Ρ€ΠΊΠΈΡ€ΡƒΡŽΡ‚ΡΡ Ρ†Π²Π΅Ρ‚Π½Ρ‹ΠΌΠΈ полосками ΠΈ Π½Π°Π·Π½Π°Ρ‡Π΅Π½ΠΈΠ΅ Π΅Π΅ ΠΌΠΎΠΆΠ½ΠΎ ΡƒΠ·Π½Π°Ρ‚ΡŒ .

ΠŸΠ°Ρ€Π°Π»Π»Π΅Π»ΡŒΠ½ΠΎΠ΅ соСдинСниС рСзисторов β€” ΠΎΠ½Π»Π°ΠΉΠ½ ΠΊΠ°Π»ΡŒΠΊΡƒΠ»ΡΡ‚ΠΎΡ€

Π§Ρ‚ΠΎΠ±Ρ‹ быстро Π²Ρ‹Ρ‡ΠΈΡΠ»ΠΈΡ‚ΡŒ ΠΎΠ±Ρ‰Π΅Π΅ сопротивлСниС Π΄Π²ΡƒΡ… ΠΈ Π±ΠΎΠ»Π΅Π΅ рСзисторов, соСдинСнных ΠΏΠ°Ρ€Π°Π»Π»Π΅Π»ΡŒΠ½ΠΎ, Π²Ρ‹ ΠΌΠΎΠΆΠ΅Ρ‚Π΅ Π²ΠΎΡΠΏΠΎΠ»ΡŒΠ·ΠΎΠ²Π°Ρ‚ΡŒΡΡ ΡΠ»Π΅Π΄ΡƒΡŽΡ‰ΠΈΠΌ ΠΎΠ½Π»Π°ΠΉΠ½ ΠΊΠ°Π»ΡŒΠΊΡƒΠ»ΡΡ‚ΠΎΡ€ΠΎΠΌ:

ПодвСдСм ΠΈΡ‚ΠΎΠ³

Когда Π΄Π²Π° ΠΈΠ»ΠΈ Π±ΠΎΠ»Π΅Π΅ рСзистора соСдинСны Ρ‚Π°ΠΊ, Ρ‡Ρ‚ΠΎ ΠΎΠ±Π° Π²Ρ‹Π²ΠΎΠ΄Π° ΠΎΠ΄Π½ΠΎΠ³ΠΎ рСзистора соСдинСны с ΡΠΎΠΎΡ‚Π²Π΅Ρ‚ΡΡ‚Π²ΡƒΡŽΡ‰ΠΈΠΌΠΈ Π²Ρ‹Π²ΠΎΠ΄Π°ΠΌΠΈ Π΄Ρ€ΡƒΠ³ΠΎΠ³ΠΎ рСзистора ΠΈΠ»ΠΈ рСзисторов, Ρ‚ΠΎ говорят, Ρ‡Ρ‚ΠΎ ΠΎΠ½ΠΈ соСдинСны ΠΌΠ΅ΠΆΠ΄Ρƒ собой ΠΏΠ°Ρ€Π°Π»Π»Π΅Π»ΡŒΠ½ΠΎ. НапряТСниС Π½Π° ΠΊΠ°ΠΆΠ΄ΠΎΠΌ рСзисторС Π²Π½ΡƒΡ‚Ρ€ΠΈ ΠΏΠ°Ρ€Π°Π»Π»Π΅Π»ΡŒΠ½ΠΎΠΉ ΠΊΠΎΠΌΠ±ΠΈΠ½Π°Ρ†ΠΈΠΈ ΠΎΠ΄ΠΈΠ½Π°ΠΊΠΎΠ²ΠΎΠ΅, Π½ΠΎ Ρ‚ΠΎΠΊΠΈ, ΠΏΡ€ΠΎΡ‚Π΅ΠΊΠ°ΡŽΡ‰ΠΈΠ΅ Ρ‡Π΅Ρ€Π΅Π· Π½ΠΈΡ…, ΠΌΠΎΠ³ΡƒΡ‚ ΠΎΡ‚Π»ΠΈΡ‡Π°Ρ‚ΡŒΡΡ Π΄Ρ€ΡƒΠ³ ΠΎΡ‚ Π΄Ρ€ΡƒΠ³Π°, Π² зависимости ΠΎΡ‚ Π²Π΅Π»ΠΈΡ‡ΠΈΠ½Ρ‹ сопротивлСний ΠΊΠ°ΠΆΠ΄ΠΎΠ³ΠΎ рСзистора.

Π­ΠΊΠ²ΠΈΠ²Π°Π»Π΅Π½Ρ‚Π½ΠΎΠ΅ ΠΈΠ»ΠΈ ΠΏΠΎΠ»Π½ΠΎΠ΅ сопротивлСниС ΠΏΠ°Ρ€Π°Π»Π»Π΅Π»ΡŒΠ½ΠΎΠΉ ΠΊΠΎΠΌΠ±ΠΈΠ½Π°Ρ†ΠΈΠΈ всСгда Π±ΡƒΠ΄Π΅Ρ‚ мСньшС минимального сопротивлСния рСзистора входящСго Π² ΠΏΠ°Ρ€Π°Π»Π»Π΅Π»ΡŒΠ½ΠΎΠ΅ соСдинСниС.

1 мОм = 0,001 Ом. 1 кОм = 1 000 = 10³ Ом. 1 МОм = 1 000 000 = 10⁢ Ом.

Π­ΠΊΠ²ΠΈΠ²Π°Π»Π΅Π½Ρ‚Π½ΠΎΠ΅ сопротивлСниС R eq Π³Ρ€ΡƒΠΏΠΏΡ‹ ΠΏΠ°Ρ€Π°Π»Π»Π΅Π»ΡŒΠ½ΠΎ соСдинСнных рСзисторов являСтся Π²Π΅Π»ΠΈΡ‡ΠΈΠ½ΠΎΠΉ, ΠΎΠ±Ρ€Π°Ρ‚Π½ΠΎΠΉ суммС Π²Π΅Π»ΠΈΡ‡ΠΈΠ½, ΠΎΠ±Ρ€Π°Ρ‚Π½ΠΎ ΠΏΡ€ΠΎΠΏΠΎΡ€Ρ†ΠΈΠΎΠ½Π°Π»ΡŒΠ½Ρ‹Ρ… сопротивлСниям этих рСзисторов.

Π˜Π½Ρ‹ΠΌΠΈ словами, ΠΏΡ€ΠΎΠ²ΠΎΠ΄ΠΈΠΌΠΎΡΡ‚ΡŒ G
ΠΏΠ°Ρ€Π°Π»Π»Π΅Π»ΡŒΠ½ΠΎ соСдинСнных рСзисторов Ρ€Π°Π²Π½Π° суммС проводимостСй этих рСзисторов:

Π­Ρ‚Π° Ρ„ΠΎΡ€ΠΌΡƒΠ»Π° для R eq ΠΈ ΠΈΡΠΏΠΎΠ»ΡŒΠ·ΡƒΠ΅Ρ‚ΡΡ Π² Π΄Π°Π½Π½ΠΎΠΌ ΠΊΠ°Π»ΡŒΠΊΡƒΠ»ΡΡ‚ΠΎΡ€Π΅ для расчСтов. НапримСр, ΠΎΠ±Ρ‰Π΅Π΅ сопротивлСниС Ρ‚Ρ€Π΅Ρ… рСзисторов 10, 15 ΠΈ 20 ΠΎΠΌ, соСдинСнных ΠΏΠ°Ρ€Π°Π»Π»Π΅Π»ΡŒΠ½ΠΎ, Ρ€Π°Π²Π½ΠΎ 4.62 Ом:

Если ΠΏΠ°Ρ€Π°Π»Π»Π΅Π»ΡŒΠ½ΠΎ соСдинСны Ρ‚ΠΎΠ»ΡŒΠΊΠΎ Π΄Π²Π° рСзистора, Ρ„ΠΎΡ€ΠΌΡƒΠ»Π° упрощаСтся:

Если имССтся n
соСдинСнных ΠΏΠ°Ρ€Π°Π»Π»Π΅Π»ΡŒΠ½ΠΎ ΠΎΠ΄ΠΈΠ½Π°ΠΊΠΎΠ²Ρ‹Ρ… рСзисторов R
, Ρ‚ΠΎ ΠΈΡ… эквивалСнтноС сопротивлСниС Π±ΡƒΠ΄Π΅Ρ‚ Ρ€Π°Π²Π½ΠΎ

ΠžΡ‚ΠΌΠ΅Ρ‚ΠΈΠΌ, Ρ‡Ρ‚ΠΎ ΠΎΠ±Ρ‰Π΅Π΅ сопротивлСниС Π³Ρ€ΡƒΠΏΠΏΡ‹ ΠΈΠ· любого количСства соСдинСнных ΠΏΠ°Ρ€Π°Π»Π»Π΅Π»ΡŒΠ½ΠΎ рСзисторов всСгда Π±ΡƒΠ΄Π΅Ρ‚ мСньшС, Ρ‡Π΅ΠΌ наимСньшСС сопротивлСниС рСзистора Π² Π³Ρ€ΡƒΠΏΠΏΠ΅ ΠΈ Π΄ΠΎΠ±Π°Π²Π»Π΅Π½ΠΈΠ΅ Π½ΠΎΠ²ΠΎΠ³ΠΎ рСзистора всСгда ΠΏΡ€ΠΈΠ²Π΅Π΄Π΅Ρ‚ ΠΊ ΡƒΠΌΠ΅Π½ΡŒΡˆΠ΅Π½ΠΈΡŽ эквивалСнтного сопротивлСния.

ΠžΡ‚ΠΌΠ΅Ρ‚ΠΈΠΌ Ρ‚Π°ΠΊΠΆΠ΅, Ρ‡Ρ‚ΠΎ всС рСзисторы, соСдинСнныС ΠΏΠ°Ρ€Π°Π»Π»Π΅Π»ΡŒΠ½ΠΎ находятся ΠΏΠΎΠ΄ ΠΎΠ΄Π½ΠΈΠΌ ΠΈ Ρ‚Π΅ΠΌ ΠΆΠ΅ напряТСниСм. Однако Ρ‚ΠΎΠΊΠΈ, ΠΏΡ€ΠΎΡ‚Π΅ΠΊΠ°ΡŽΡ‰ΠΈΠ΅ Ρ‡Π΅Ρ€Π΅Π· ΠΎΡ‚Π΄Π΅Π»ΡŒΠ½Ρ‹Π΅ рСзисторы, ΠΎΡ‚Π»ΠΈΡ‡Π°ΡŽΡ‚ΡΡ ΠΈ зависят ΠΎΡ‚ ΠΈΡ… сопротивлСния. ΠžΠ±Ρ‰ΠΈΠΉ Ρ‚ΠΎΠΊ Ρ‡Π΅Ρ€Π΅Π· Π³Ρ€ΡƒΠΏΠΏΡƒ рСзисторов Ρ€Π°Π²Π΅Π½ суммС Ρ‚ΠΎΠΊΠΎΠ² Π² ΠΎΡ‚Π΄Π΅Π»ΡŒΠ½Ρ‹Ρ… рСзисторах.

ΠŸΡ€ΠΈ соСдинСнии Π½Π΅ΡΠΊΠΎΠ»ΡŒΠΊΠΈΡ… рСзисторов ΠΏΠ°Ρ€Π°Π»Π»Π΅Π»ΡŒΠ½ΠΎ всСгда Π½ΡƒΠΆΠ½ΠΎ ΡƒΡ‡ΠΈΡ‚Ρ‹Π²Π°Ρ‚ΡŒ ΠΈΡ… допуски ΠΈ Ρ€Π°ΡΡΠ΅ΠΈΠ²Π°Π΅ΠΌΡƒΡŽ ΠΌΠΎΡ‰Π½ΠΎΡΡ‚ΡŒ.

ΠŸΡ€ΠΈΠΌΠ΅Ρ€Ρ‹ примСнСния ΠΏΠ°Ρ€Π°Π»Π»Π΅Π»ΡŒΠ½ΠΎΠ³ΠΎ соСдинСния рСзисторов

Одним ΠΈΠ· ΠΏΡ€ΠΈΠΌΠ΅Ρ€ΠΎΠ² ΠΏΠ°Ρ€Π°Π»Π»Π΅Π»ΡŒΠ½ΠΎΠ³ΠΎ соСдинСния рСзисторов являСтся ΡˆΡƒΠ½Ρ‚ Π² ΠΏΡ€ΠΈΠ±ΠΎΡ€Π΅ для измСрСния Ρ‚ΠΎΠΊΠΎΠ², ΠΊΠΎΡ‚ΠΎΡ€Ρ‹Π΅ слишком Π²Π΅Π»ΠΈΠΊΠΈ для Ρ‚ΠΎΠ³ΠΎ, Ρ‡Ρ‚ΠΎΠ±Ρ‹ Π±Ρ‹Ρ‚ΡŒ Π½Π°ΠΏΡ€ΡΠΌΡƒΡŽ ΠΈΠ·ΠΌΠ΅Ρ€Π΅Π½Π½Ρ‹ΠΌΠΈ ΠΏΡ€ΠΈΠ±ΠΎΡ€ΠΎΠΌ, ΠΏΡ€Π΅Π΄Π½Π°Π·Π½Π°Ρ‡Π΅Π½Π½Ρ‹ΠΌ для измСрСния Π½Π΅Π±ΠΎΠ»ΡŒΡˆΠΈΡ… Ρ‚ΠΎΠΊΠΎΠ² ΠΈΠ»ΠΈ напряТСний. Для измСрСния Ρ‚ΠΎΠΊΠ° ΠΏΠ°Ρ€Π°Π»Π»Π΅Π»ΡŒΠ½ΠΎ Π³Π°Π»ΡŒΠ²Π°Π½ΠΎΠΌΠ΅Ρ‚Ρ€Ρƒ ΠΈΠ»ΠΈ элСктронному ΠΏΡ€ΠΈΠ±ΠΎΡ€Ρƒ, ΠΈΠ·ΠΌΠ΅Ρ€ΡΡŽΡ‰Π΅ΠΌΡƒ напряТСниС, ΠΏΠΎΠ΄ΠΊΠ»ΡŽΡ‡Π°Π΅Ρ‚ΡΡ рСзистор с ΠΎΡ‡Π΅Π½ΡŒ малСньким Ρ‚ΠΎΡ‡Π½ΠΎ извСстным сопротивлСниСм, ΠΈΠ·Π³ΠΎΡ‚ΠΎΠ²Π»Π΅Π½Π½Ρ‹ΠΉ ΠΈΠ· ΠΌΠ°Ρ‚Π΅Ρ€ΠΈΠ°Π»Π° со ΡΡ‚Π°Π±ΠΈΠ»ΡŒΠ½Ρ‹ΠΌΠΈ характСристиками. Π­Ρ‚ΠΎΡ‚ рСзистор называСтся ΡˆΡƒΠ½Ρ‚ΠΎΠΌ. Π˜Π·ΠΌΠ΅Ρ€ΡΠ΅ΠΌΡ‹ΠΉ Ρ‚ΠΎΠΊ ΠΏΡ€ΠΎΡ‚Π΅ΠΊΠ°Π΅Ρ‚ Ρ‡Π΅Ρ€Π΅Π· ΡˆΡƒΠ½Ρ‚. Π’ Ρ€Π΅Π·ΡƒΠ»ΡŒΡ‚Π°Ρ‚Π΅ Π½Π° Π½Π΅ΠΌ ΠΏΠ°Π΄Π°Π΅Ρ‚ нСбольшоС напряТСниС, ΠΊΠΎΡ‚ΠΎΡ€ΠΎΠ΅ ΠΈ измСряСтся Π²ΠΎΠ»ΡŒΡ‚ΠΌΠ΅Ρ‚Ρ€ΠΎΠΌ. ΠŸΠΎΡΠΊΠΎΠ»ΡŒΠΊΡƒ ΠΏΠ°Π΄Π΅Π½ΠΈΠ΅ напряТСния ΠΏΡ€ΠΎΠΏΠΎΡ€Ρ†ΠΈΠΎΠ½Π°Π»ΡŒΠ½ΠΎ Ρ‚ΠΎΠΊΡƒ, ΠΏΡ€ΠΎΡ‚Π΅ΠΊΠ°ΡŽΡ‰Π΅ΠΌΡƒ Ρ‡Π΅Ρ€Π΅Π· ΡˆΡƒΠ½Ρ‚ с извСстным ΠΈ Ρ‚ΠΎΡ‡Π½Ρ‹ΠΌ сопротивлСниСм, Π²ΠΎΠ»ΡŒΡ‚ΠΌΠ΅Ρ‚Ρ€, ΠΏΠΎΠ΄ΠΊΠ»ΡŽΡ‡Π΅Π½Π½Ρ‹ΠΉ ΠΏΠ°Ρ€Π°Π»Π»Π΅Π»ΡŒΠ½ΠΎ ΡˆΡƒΠ½Ρ‚Ρƒ, ΠΌΠΎΠΆΠ½ΠΎ ΠΏΡ€ΠΎΠ³Ρ€Π°Π΄ΡƒΠΈΡ€ΠΎΠ²Π°Ρ‚ΡŒ нСпосрСдствСнно Π² Π΅Π΄ΠΈΠ½ΠΈΡ†Π°Ρ… Ρ‚ΠΎΠΊΠ° (Π°ΠΌΠΏΠ΅Ρ€Π°Ρ…).

ΠŸΠ°Ρ€Π°Π»Π»Π΅Π»ΡŒΠ½Ρ‹Π΅ ΠΈ ΠΏΠΎΡΠ»Π΅Π΄ΠΎΠ²Π°Ρ‚Π΅Π»ΡŒΠ½Ρ‹Π΅ схСмы часто ΠΈΡΠΏΠΎΠ»ΡŒΠ·ΡƒΡŽΡ‚ΡΡ для получСния Ρ‚ΠΎΡ‡Π½ΠΎΠ³ΠΎ сопротивлСния ΠΈΠ»ΠΈ Ссли рСзистора с Ρ‚Ρ€Π΅Π±ΡƒΠ΅ΠΌΡ‹ΠΌ сопротивлСниСм Π½Π΅Ρ‚ ΠΈΠ»ΠΈ ΠΎΠ½ слишком Π΄ΠΎΡ€ΠΎΠ³, Ссли Π΅Π³ΠΎ ΠΏΡ€ΠΈΠΎΠ±Ρ€Π΅Ρ‚Π°Ρ‚ΡŒ Π² Π½Π΅Π±ΠΎΠ»ΡŒΡˆΠΈΡ… количСствах для массового производства
. НапримСр, Ссли устройство содСрТит ΠΌΠ½ΠΎΠ³ΠΎ рСзисторов ΠΏΠΎ 20 кОм ΠΈ Π½Π΅ΠΎΠ±Ρ…ΠΎΠ΄ΠΈΠΌ Ρ‚ΠΎΠ»ΡŒΠΊΠΎ ΠΎΠ΄ΠΈΠ½ рСзистор 10 кОм. ΠšΠΎΠ½Π΅Ρ‡Π½ΠΎ, нСслоТно Π½Π°ΠΉΡ‚ΠΈ рСзистор Π½Π° 10 кОм. Однако для массового производства ΠΈΠ½ΠΎΠ³Π΄Π° Π±Ρ‹Π²Π°Π΅Ρ‚ Π»ΡƒΡ‡ΡˆΠ΅ ΠΏΠΎΡΡ‚Π°Π²ΠΈΡ‚ΡŒ Π΄Π²Π° рСзистора Π½Π° 20 кОм ΠΏΠ°Ρ€Π°Π»Π»Π΅Π»ΡŒΠ½ΠΎ, Ρ‡Ρ‚ΠΎΠ±Ρ‹ ΠΏΠΎΠ»ΡƒΡ‡ΠΈΡ‚ΡŒ Π½Π΅ΠΎΠ±Ρ…ΠΎΠ΄ΠΈΠΌΡ‹Π΅ 10 кОм. Π­Ρ‚ΠΎ ΠΏΡ€ΠΈΠ²Π΅Π΄Π΅Ρ‚ ΠΊ сниТСнию сСбСстоимости ΠΏΠ΅Ρ‡Π°Ρ‚Π½ΠΎΠΉ ΠΏΠ»Π°Ρ‚Ρ‹, Ρ‚Π°ΠΊ ΠΊΠ°ΠΊ Π±ΡƒΠ΄Π΅Ρ‚ сниТСна оптовая Ρ†Π΅Π½Π° ΠΊΠΎΠΌΠΏΠΎΠ½Π΅Π½Ρ‚ΠΎΠ², Π° Ρ‚Π°ΠΊΠΆΠ΅ ΡΡ‚ΠΎΠΈΠΌΠΎΡΡ‚ΡŒ ΠΌΠΎΠ½Ρ‚Π°ΠΆΠ°, Ρ‚Π°ΠΊ ΠΊΠ°ΠΊ Π±ΡƒΠ΄Π΅Ρ‚ ΡƒΠΌΠ΅Π½ΡŒΡˆΠ΅Π½ΠΎ количСство Ρ‚ΠΈΠΏΠΎΡ€Π°Π·ΠΌΠ΅Ρ€ΠΎΠ² элСмСнтов, ΠΊΠΎΡ‚ΠΎΡ€Ρ‹Π΅ Π΄ΠΎΠ»ΠΆΠ΅Π½ ΡƒΡΡ‚Π°Π½ΠΎΠ²ΠΈΡ‚ΡŒ Π½Π° ΠΏΠ»Π°Ρ‚Ρƒ Π°Π²Ρ‚ΠΎΠΌΠ°Ρ‚ установки ΠΊΠΎΠΌΠΏΠΎΠ½Π΅Π½Ρ‚ΠΎΠ².

ΠŸΡ€ΠΎΠ²Π΅Ρ€ΠΈΠΌ ΡΠΏΡ€Π°Π²Π΅Π΄Π»ΠΈΠ²ΠΎΡΡ‚ΡŒ ΠΏΠΎΠΊΠ°Π·Π°Π½Π½Ρ‹Ρ… здСсь Ρ„ΠΎΡ€ΠΌΡƒΠ» Π½Π° простом экспСримСнтС.

Π’ΠΎΠ·ΡŒΠΌΡ‘ΠΌ Π΄Π²Π° рСзистора ΠœΠ›Π’-2
Π½Π° 3
и 47 Ом
ΠΈ соСдиним ΠΈΡ… ΠΏΠΎΡΠ»Π΅Π΄ΠΎΠ²Π°Ρ‚Π΅Π»ΡŒΠ½ΠΎ. Π—Π°Ρ‚Π΅ΠΌ ΠΈΠ·ΠΌΠ΅Ρ€ΠΈΠΌ ΠΎΠ±Ρ‰Π΅Π΅ сопротивлСниС ΠΏΠΎΠ»ΡƒΡ‡ΠΈΠ²ΡˆΠ΅ΠΉΡΡ Ρ†Π΅ΠΏΠΈ Ρ†ΠΈΡ„Ρ€ΠΎΠ²Ρ‹ΠΌ ΠΌΡƒΠ»ΡŒΡ‚ΠΈΠΌΠ΅Ρ‚Ρ€ΠΎΠΌ. Как Π²ΠΈΠ΄ΠΈΠΌ ΠΎΠ½ΠΎ Ρ€Π°Π²Π½ΠΎ суммС сопротивлСний рСзисторов, входящих Π² эту Ρ†Π΅ΠΏΠΎΡ‡ΠΊΡƒ.

Π—Π°ΠΌΠ΅Ρ€ ΠΎΠ±Ρ‰Π΅Π³ΠΎ сопротивлСния ΠΏΡ€ΠΈ ΠΏΠΎΡΠ»Π΅Π΄ΠΎΠ²Π°Ρ‚Π΅Π»ΡŒΠ½ΠΎΠΌ соСдинСнии

Π’Π΅ΠΏΠ΅Ρ€ΡŒ соСдиним наши рСзисторы ΠΏΠ°Ρ€Π°Π»Π»Π΅Π»ΡŒΠ½ΠΎ ΠΈ Π·Π°ΠΌΠ΅Ρ€ΠΈΠΌ ΠΈΡ… ΠΎΠ±Ρ‰Π΅Π΅ сопротивлСниС.

Π˜Π·ΠΌΠ΅Ρ€Π΅Π½ΠΈΠ΅ сопротивлСния ΠΏΡ€ΠΈ ΠΏΠ°Ρ€Π°Π»Π»Π΅Π»ΡŒΠ½ΠΎΠΌ соСдинСнии

Как Π²ΠΈΠ΄ΠΈΠΌ, Ρ€Π΅Π·ΡƒΠ»ΡŒΡ‚ΠΈΡ€ΡƒΡŽΡ‰Π΅Π΅ сопротивлСниС (2,9 Ом) мСньшС самого мСньшСго (3 Ом), входящСго Π² Ρ†Π΅ΠΏΠΎΡ‡ΠΊΡƒ. ΠžΡ‚ΡΡŽΠ΄Π° Π²Ρ‹Ρ‚Π΅ΠΊΠ°Π΅Ρ‚ Π΅Ρ‰Ρ‘ ΠΎΠ΄Π½ΠΎ извСстноС ΠΏΡ€Π°Π²ΠΈΠ»ΠΎ, ΠΊΠΎΡ‚ΠΎΡ€ΠΎΠ΅ ΠΌΠΎΠΆΠ½ΠΎ ΠΏΡ€ΠΈΠΌΠ΅Π½ΡΡ‚ΡŒ Π½Π° ΠΏΡ€Π°ΠΊΡ‚ΠΈΠΊΠ΅:

ΠŸΡ€ΠΈ ΠΏΠ°Ρ€Π°Π»Π»Π΅Π»ΡŒΠ½ΠΎΠΌ соСдинСнии рСзисторов ΠΎΠ±Ρ‰Π΅Π΅ сопротивлСниС Ρ†Π΅ΠΏΠΈ Π±ΡƒΠ΄Π΅Ρ‚ мСньшС наимСньшСго сопротивлСния, входящСго Π² эту Ρ†Π΅ΠΏΡŒ.

Π§Ρ‚ΠΎ Π΅Ρ‰Ρ‘ Π½ΡƒΠΆΠ½ΠΎ ΡƒΡ‡ΠΈΡ‚Ρ‹Π²Π°Ρ‚ΡŒ ΠΏΡ€ΠΈ соСдинСнии рСзисторов?

Π’ΠΎ-ΠΏΠ΅Ρ€Π²Ρ‹Ρ…, ΠΎΠ±ΡΠ·Π°Ρ‚Π΅Π»ΡŒΠ½ΠΎ
учитываСтся ΠΈΡ… номинальная ΠΌΠΎΡ‰Π½ΠΎΡΡ‚ΡŒ. НапримСр, Π½Π°ΠΌ Π½ΡƒΠΆΠ½ΠΎ ΠΏΠΎΠ΄ΠΎΠ±Ρ€Π°Ρ‚ΡŒ Π·Π°ΠΌΠ΅Π½Ρƒ рСзистору Π½Π° 100 Ом
ΠΈ ΠΌΠΎΡ‰Π½ΠΎΡΡ‚ΡŒΡŽ 1 Π’Ρ‚
. Π’ΠΎΠ·ΡŒΠΌΡ‘ΠΌ Π΄Π²Π° рСзистора ΠΏΠΎ 50 Ом ΠΊΠ°ΠΆΠ΄Ρ‹ΠΉ ΠΈ соСдиним ΠΈΡ… ΠΏΠΎΡΠ»Π΅Π΄ΠΎΠ²Π°Ρ‚Π΅Π»ΡŒΠ½ΠΎ. На ΠΊΠ°ΠΊΡƒΡŽ ΠΌΠΎΡ‰Π½ΠΎΡΡ‚ΡŒ рассСяния Π΄ΠΎΠ»ΠΆΠ½Ρ‹ Π±Ρ‹Ρ‚ΡŒ рассчитаны эти Π΄Π²Π° рСзистора?

ΠŸΠΎΡΠΊΠΎΠ»ΡŒΠΊΡƒ Ρ‡Π΅Ρ€Π΅Π· ΠΏΠΎΡΠ»Π΅Π΄ΠΎΠ²Π°Ρ‚Π΅Π»ΡŒΠ½ΠΎ соСдинённыС рСзисторы Ρ‚Π΅Ρ‡Ρ‘Ρ‚ ΠΎΠ΄ΠΈΠ½ ΠΈ Ρ‚ΠΎΡ‚ ΠΆΠ΅ постоянный Ρ‚ΠΎΠΊ (допустим 0,1 А
), Π° сопротивлСниС ΠΊΠ°ΠΆΠ΄ΠΎΠ³ΠΎ ΠΈΠ· Π½ΠΈΡ… Ρ€Π°Π²Π½ΠΎ 50 Ом
, Ρ‚ΠΎΠ³Π΄Π° ΠΌΠΎΡ‰Π½ΠΎΡΡ‚ΡŒ рассСивания ΠΊΠ°ΠΆΠ΄ΠΎΠ³ΠΎ ΠΈΠ· Π½ΠΈΡ… Π΄ΠΎΠ»ΠΆΠ½Π° Π±Ρ‹Ρ‚ΡŒ Π½Π΅ ΠΌΠ΅Π½Π΅Π΅ 0,5 Π’Ρ‚
. Π’ Ρ€Π΅Π·ΡƒΠ»ΡŒΡ‚Π°Ρ‚Π΅ Π½Π° ΠΊΠ°ΠΆΠ΄ΠΎΠΌ ΠΈΠ· Π½ΠΈΡ… выдСлится ΠΏΠΎ 0,5 Π’Ρ‚
мощности. Π’ суммС это ΠΈ Π±ΡƒΠ΄Π΅Ρ‚ Ρ‚ΠΎΡ‚ самый 1 Π’Ρ‚
.

Π”Π°Π½Π½Ρ‹ΠΉ ΠΏΡ€ΠΈΠΌΠ΅Ρ€ достаточно Π³Ρ€ΡƒΠ±ΠΎΠ²Π°Ρ‚. ΠŸΠΎΡΡ‚ΠΎΠΌΡƒ, Ссли Π΅ΡΡ‚ΡŒ сомнСния, стоит Π±Ρ€Π°Ρ‚ΡŒ рСзисторы с запасом ΠΏΠΎ мощности.

ΠŸΠΎΠ΄Ρ€ΠΎΠ±Π½Π΅Π΅ ΠΎ мощности рассСивания рСзистора Ρ‡ΠΈΡ‚Π°ΠΉΡ‚Π΅ .

Π’ΠΎ-Π²Ρ‚ΠΎΡ€Ρ‹Ρ…, ΠΏΡ€ΠΈ соСдинСнии стоит ΠΈΡΠΏΠΎΠ»ΡŒΠ·ΠΎΠ²Π°Ρ‚ΡŒ ΠΎΠ΄Π½ΠΎΡ‚ΠΈΠΏΠ½Ρ‹Π΅ рСзисторы, Π½Π°ΠΏΡ€ΠΈΠΌΠ΅Ρ€, сСрии ΠœΠ›Π’. ΠšΠΎΠ½Π΅Ρ‡Π½ΠΎ, Π½Π΅Ρ‚ Π½ΠΈΡ‡Π΅Π³ΠΎ ΠΏΠ»ΠΎΡ…ΠΎΠ³ΠΎ Π² Ρ‚ΠΎΠΌ, Ρ‡Ρ‚ΠΎΠ±Ρ‹ Π±Ρ€Π°Ρ‚ΡŒ Ρ€Π°Π·Π½Ρ‹Π΅. Π­Ρ‚ΠΎ лишь рСкомСндация.

ΠŸΠ°Ρ€Π°Π»Π»Π΅Π»ΡŒΠ½ΠΎΠ΅ соСдинСниС рСзисторов, наряду с ΠΏΠΎΡΠ»Π΅Π΄ΠΎΠ²Π°Ρ‚Π΅Π»ΡŒΠ½Ρ‹ΠΌ, являСтся основным способом соСдинСния элСмСнтов Π² элСктричСской Ρ†Π΅ΠΏΠΈ. Π’ΠΎ Π²Ρ‚ΠΎΡ€ΠΎΠΌ Π²Π°Ρ€ΠΈΠ°Π½Ρ‚Π΅ всС элСмСнты установлСны ΠΏΠΎΡΠ»Π΅Π΄ΠΎΠ²Π°Ρ‚Π΅Π»ΡŒΠ½ΠΎ: ΠΊΠΎΠ½Π΅Ρ† ΠΎΠ΄Π½ΠΎΠ³ΠΎ элСмСнта соСдинСн с Π½Π°Ρ‡Π°Π»ΠΎΠΌ ΡΠ»Π΅Π΄ΡƒΡŽΡ‰Π΅Π³ΠΎ. Π’ Ρ‚Π°ΠΊΠΎΠΉ схСмС сила Ρ‚ΠΎΠΊΠ° Π½Π° всСх элСмСнтах одинаковая, Π° ΠΏΠ°Π΄Π΅Π½ΠΈΠ΅ напряТСний зависит ΠΎΡ‚ сопротивлСния ΠΊΠ°ΠΆΠ΄ΠΎΠ³ΠΎ элСмСнта. Π’ ΠΏΠΎΡΠ»Π΅Π΄ΠΎΠ²Π°Ρ‚Π΅Π»ΡŒΠ½ΠΎΠΌ соСдинСнии Π΅ΡΡ‚ΡŒ Π΄Π²Π° ΡƒΠ·Π»Π°. К ΠΎΠ΄Π½ΠΎΠΌΡƒ подсоСдинСны Π½Π°Ρ‡Π°Π»Π° всСх элСмСнтов, Π° ΠΊΠΎ Π²Ρ‚ΠΎΡ€ΠΎΠΌΡƒ ΠΈΡ… ΠΊΠΎΠ½Ρ†Ρ‹. Условно для постоянного Ρ‚ΠΎΠΊΠ° ΠΌΠΎΠΆΠ½ΠΎ ΠΎΠ±ΠΎΠ·Π½Π°Ρ‡ΠΈΡ‚ΡŒ ΠΈΡ… ΠΊΠ°ΠΊ плюс ΠΈ минус, Π° для ΠΏΠ΅Ρ€Π΅ΠΌΠ΅Π½Π½ΠΎΠ³ΠΎ ΠΊΠ°ΠΊ Ρ„Π°Π·Ρƒ ΠΈ ноль. Благодаря своим особСнностям Π½Π°Ρ…ΠΎΠ΄ΠΈΡ‚ ΡˆΠΈΡ€ΠΎΠΊΠΎΠ΅ ΠΏΡ€ΠΈΠΌΠ΅Π½Π΅Π½ΠΈΠ΅ Π² элСктричСских схСмах, Π² Ρ‚ΠΎΠΌ числС ΠΈ со ΡΠΌΠ΅ΡˆΠ°Π½Π½Ρ‹ΠΌ соСдинСниСм. Бвойства ΠΎΠ΄ΠΈΠ½Π°ΠΊΠΎΠ²Ρ‹ для постоянного ΠΈ ΠΏΠ΅Ρ€Π΅ΠΌΠ΅Π½Π½ΠΎΠ³ΠΎ Ρ‚ΠΎΠΊΠ°.

РасчСт ΠΎΠ±Ρ‰Π΅Π³ΠΎ сопротивлСния ΠΏΡ€ΠΈ ΠΏΠ°Ρ€Π°Π»Π»Π΅Π»ΡŒΠ½ΠΎΠΌ соСдинСнии рСзисторов

Π’ ΠΎΡ‚Π»ΠΈΡ‡ΠΈΠ΅ ΠΎΡ‚ ΠΏΠΎΡΠ»Π΅Π΄ΠΎΠ²Π°Ρ‚Π΅Π»ΡŒΠ½ΠΎΠ³ΠΎ соСдинСния, Π³Π΄Π΅ для нахоТдСния ΠΎΠ±Ρ‰Π΅Π³ΠΎ сопротивлСния достаточно ΡΠ»ΠΎΠΆΠΈΡ‚ΡŒ Π·Π½Π°Ρ‡Π΅Π½ΠΈΠ΅ ΠΊΠ°ΠΆΠ΄ΠΎΠ³ΠΎ элСмСнта, для ΠΏΠ°Ρ€Π°Π»Π»Π΅Π»ΡŒΠ½ΠΎΠ³ΠΎ Ρ‚ΠΎ ΠΆΠ΅ самоС Π±ΡƒΠ΄Π΅Ρ‚ справСдливо для проводимости. А Ρ‚Π°ΠΊ ΠΊΠ°ΠΊ ΠΎΠ½Π° ΠΎΠ±Ρ€Π°Ρ‚Π½ΠΎ ΠΏΡ€ΠΎΠΏΠΎΡ€Ρ†ΠΈΠΎΠ½Π°Π»ΡŒΠ½Π° ΡΠΎΠΏΡ€ΠΎΡ‚ΠΈΠ²Π»Π΅Π½ΠΈΡŽ, ΠΏΠΎΠ»ΡƒΡ‡ΠΈΠΌ Ρ„ΠΎΡ€ΠΌΡƒΠ»Ρƒ, ΠΏΡ€Π΅Π΄ΡΡ‚Π°Π²Π»Π΅Π½Π½ΡƒΡŽ вмСстС со схСмой Π½Π° ΡΠ»Π΅Π΄ΡƒΡŽΡ‰Π΅ΠΌ рисункС:

НСобходимо ΠΎΡ‚ΠΌΠ΅Ρ‚ΠΈΡ‚ΡŒ ΠΎΠ΄Π½Ρƒ Π²Π°ΠΆΠ½ΡƒΡŽ ΠΎΡΠΎΠ±Π΅Π½Π½ΠΎΡΡ‚ΡŒ расчСта ΠΏΠ°Ρ€Π°Π»Π»Π΅Π»ΡŒΠ½ΠΎΠ³ΠΎ соСдинСния рСзисторов: ΠΎΠ±Ρ‰Π΅Π΅ Π·Π½Π°Ρ‡Π΅Π½ΠΈΠ΅ Π±ΡƒΠ΄Π΅Ρ‚ всСгда мСньшС, Ρ‡Π΅ΠΌ самоС малСнькоС ΠΈΠ· Π½ΠΈΡ…. Для рСзисторов справСдливо ΠΊΠ°ΠΊ для постоянного, Ρ‚Π°ΠΊ ΠΈ для ΠΏΠ΅Ρ€Π΅ΠΌΠ΅Π½Π½ΠΎΠ³ΠΎ Ρ‚ΠΎΠΊΠ°. ΠšΠ°Ρ‚ΡƒΡˆΠΊΠΈ ΠΈ кондСнсаторы ΠΈΠΌΠ΅ΡŽΡ‚ свои особСнности.

Π‘ΠΈΠ»Π° Ρ‚ΠΎΠΊΠ° ΠΈ напряТСниС

ΠŸΡ€ΠΈ расчСтС ΠΏΠ°Ρ€Π°Π»Π»Π΅Π»ΡŒΠ½ΠΎΠ³ΠΎ сопротивлСния рСзисторов Π½Π΅ΠΎΠ±Ρ…ΠΎΠ΄ΠΈΠΌΠΎ Π·Π½Π°Ρ‚ΡŒ, ΠΊΠ°ΠΊ Ρ€Π°ΡΡΡ‡ΠΈΡ‚Π°Ρ‚ΡŒ напряТСниС ΠΈ силу Ρ‚ΠΎΠΊΠ°. Π’ этом случаС Π½Π°ΠΌ ΠΏΠΎΠΌΠΎΠΆΠ΅Ρ‚ Π·Π°ΠΊΠΎΠ½ Ома, ΠΎΠΏΡ€Π΅Π΄Π΅Π»ΡΡŽΡ‰ΠΈΠΉ связь ΠΌΠ΅ΠΆΠ΄Ρƒ сопротивлСниСм, силой Ρ‚ΠΎΠΊΠ° ΠΈ напряТСниСм.

Π˜ΡΡ…ΠΎΠ΄Ρ ΠΈΠ· ΠΏΠ΅Ρ€Π²ΠΎΠΉ Ρ„ΠΎΡ€ΠΌΡƒΠ»ΠΈΡ€ΠΎΠ²ΠΊΠΈ Π·Π°ΠΊΠΎΠ½Π° ΠšΠΈΡ€Ρ…Π³ΠΎΡ„Π°, ΠΏΠΎΠ»ΡƒΡ‡ΠΈΠΌ, Ρ‡Ρ‚ΠΎ сумма сходящихся Π² ΠΎΠ΄Π½ΠΎΠΌ ΡƒΠ·Π»Π΅ Ρ‚ΠΎΠΊΠΎΠ² Ρ€Π°Π²Π½Π° Π½ΡƒΠ»ΡŽ. НаправлСниС Π²Ρ‹Π±ΠΈΡ€Π°Π΅ΠΌ ΠΏΠΎ Π½Π°ΠΏΡ€Π°Π²Π»Π΅Π½ΠΈΡŽ протСкания Ρ‚ΠΎΠΊΠ°. Π’Π°ΠΊΠΈΠΌ ΠΎΠ±Ρ€Π°Π·ΠΎΠΌ, ΠΏΠΎΠ»ΠΎΠΆΠΈΡ‚Π΅Π»ΡŒΠ½Ρ‹ΠΌ Π½Π°ΠΏΡ€Π°Π²Π»Π΅Π½ΠΈΠ΅ΠΌ для ΠΏΠ΅Ρ€Π²ΠΎΠ³ΠΎ ΡƒΠ·Π»Π° ΠΌΠΎΠΆΠ½ΠΎ ΡΡ‡ΠΈΡ‚Π°Ρ‚ΡŒ входящий Ρ‚ΠΎΠΊ ΠΎΡ‚ источника питания. А ΠΎΡ‚Ρ€ΠΈΡ†Π°Ρ‚Π΅Π»ΡŒΠ½Ρ‹ΠΌΠΈ Π±ΡƒΠ΄ΡƒΡ‚ отходящиС ΠΈΠ· ΠΊΠ°ΠΆΠ΄ΠΎΠ³ΠΎ рСзистора. Для Π²Ρ‚ΠΎΡ€ΠΎΠ³ΠΎ ΡƒΠ·Π»Π° ΠΊΠ°Ρ€Ρ‚ΠΈΠ½Π° ΠΏΡ€ΠΎΡ‚ΠΈΠ²ΠΎΠΏΠΎΠ»ΠΎΠΆΠ½Π°. Π˜ΡΡ…ΠΎΠ΄Ρ ΠΈΠ· Ρ„ΠΎΡ€ΠΌΡƒΠ»ΠΈΡ€ΠΎΠ²ΠΊΠΈ Π·Π°ΠΊΠΎΠ½Π°, ΠΏΠΎΠ»ΡƒΡ‡ΠΈΠΌ, Ρ‡Ρ‚ΠΎ суммарный Ρ‚ΠΎΠΊ Ρ€Π°Π²Π΅Π½ суммС Ρ‚ΠΎΠΊΠΎΠ², проходящих Ρ‡Π΅Ρ€Π΅Π· ΠΊΠ°ΠΆΠ΄Ρ‹ΠΉ ΠΏΠ°Ρ€Π°Π»Π»Π΅Π»ΡŒΠ½ΠΎ соСдинСнный рСзистор.

Π˜Ρ‚ΠΎΠ³ΠΎΠ²ΠΎΠ΅ напряТСниС ΠΆΠ΅ опрСдСляСтся ΠΏΠΎ Π²Ρ‚ΠΎΡ€ΠΎΠΌΡƒ Π·Π°ΠΊΠΎΠ½Ρƒ ΠšΠΈΡ€Ρ…Π³ΠΎΡ„Π°. Оно ΠΎΠ΄ΠΈΠ½Π°ΠΊΠΎΠ²ΠΎ для ΠΊΠ°ΠΆΠ΄ΠΎΠ³ΠΎ рСзистора ΠΈ Ρ€Π°Π²Π½ΠΎ ΠΎΠ±Ρ‰Π΅ΠΌΡƒ. Π­Ρ‚Π° ΠΎΡΠΎΠ±Π΅Π½Π½ΠΎΡΡ‚ΡŒ ΠΈΡΠΏΠΎΠ»ΡŒΠ·ΡƒΠ΅Ρ‚ΡΡ для ΠΏΠΎΠ΄ΠΊΠ»ΡŽΡ‡Π΅Π½ΠΈΡ Ρ€ΠΎΠ·Π΅Ρ‚ΠΎΠΊ ΠΈ освСщСния Π² ΠΊΠ²Π°Ρ€Ρ‚ΠΈΡ€Π°Ρ….

ΠŸΡ€ΠΈΠΌΠ΅Ρ€ расчСта

Π’ качСствС ΠΏΠ΅Ρ€Π²ΠΎΠ³ΠΎ ΠΏΡ€ΠΈΠΌΠ΅Ρ€Π° ΠΏΡ€ΠΈΠ²Π΅Π΄Π΅ΠΌ расчСт сопротивлСния ΠΏΡ€ΠΈ ΠΏΠ°Ρ€Π°Π»Π»Π΅Π»ΡŒΠ½ΠΎΠΌ соСдинСнии ΠΎΠ΄ΠΈΠ½Π°ΠΊΠΎΠ²Ρ‹Ρ… рСзисторов. Π‘ΠΈΠ»Π° Ρ‚ΠΎΠΊΠ°, ΠΏΡ€ΠΎΡ‚Π΅ΠΊΠ°ΡŽΡ‰Π°Ρ Ρ‡Π΅Ρ€Π΅Π· Π½ΠΈΡ…, Π±ΡƒΠ΄Π΅Ρ‚ ΠΎΠ΄ΠΈΠ½Π°ΠΊΠΎΠ²ΠΎΠΉ. ΠŸΡ€ΠΈΠΌΠ΅Ρ€ расчСта сопротивлСния выглядит Ρ‚Π°ΠΊ:

По этому ΠΏΡ€ΠΈΠΌΠ΅Ρ€Ρƒ прСкрасно Π²ΠΈΠ΄Π½ΠΎ, Ρ‡Ρ‚ΠΎ ΠΎΠ±Ρ‰Π΅Π΅ сопротивлСниС Π½ΠΈΠΆΠ΅ Π² Π΄Π²Π° Ρ€Π°Π·Π°, Ρ‡Π΅ΠΌ ΠΊΠ°ΠΆΠ΄ΠΎΠ΅ ΠΈΠ· Π½ΠΈΡ…. Π­Ρ‚ΠΎ соотвСтствуСт Ρ‚ΠΎΠΌΡƒ, Ρ‡Ρ‚ΠΎ суммарная сила Ρ‚ΠΎΠΊΠ° Π² Π΄Π²Π° Ρ€Π°Π·Π° Π²Ρ‹ΡˆΠ΅, Ρ‡Π΅ΠΌ Ρƒ ΠΎΠ΄Π½ΠΎΠ³ΠΎ. А Ρ‚Π°ΠΊΠΆΠ΅ прСкрасно соотносится с ΡƒΠ²Π΅Π»ΠΈΡ‡Π΅Π½ΠΈΠ΅ΠΌ проводимости Π² Π΄Π²Π° Ρ€Π°Π·Π°.

Π’Ρ‚ΠΎΡ€ΠΎΠΉ ΠΏΡ€ΠΈΠΌΠ΅Ρ€

Рассмотрим ΠΏΡ€ΠΈΠΌΠ΅Ρ€ ΠΏΠ°Ρ€Π°Π»Π»Π΅Π»ΡŒΠ½ΠΎΠ³ΠΎ соСдинСния Ρ‚Ρ€Π΅Ρ… рСзисторов. Для расчСта ΠΈΡΠΏΠΎΠ»ΡŒΠ·ΡƒΠ΅ΠΌ ΡΡ‚Π°Π½Π΄Π°Ρ€Ρ‚Π½ΡƒΡŽ Ρ„ΠΎΡ€ΠΌΡƒΠ»Ρƒ:

ΠŸΠΎΡ…ΠΎΠΆΠΈΠΌ ΠΎΠ±Ρ€Π°Π·ΠΎΠΌ Ρ€Π°ΡΡΡ‡ΠΈΡ‚Ρ‹Π²Π°ΡŽΡ‚ΡΡ схСмы с большим количСством ΠΏΠ°Ρ€Π°Π»Π»Π΅Π»ΡŒΠ½ΠΎ соСдинСнных рСзисторов.

ΠŸΡ€ΠΈΠΌΠ΅Ρ€ смСшанного соСдинСния

Для смСшанного соСдинСния, Π½Π°ΠΏΡ€ΠΈΠΌΠ΅Ρ€, прСдставлСнного Π½ΠΈΠΆΠ΅, расчСт Π±ΡƒΠ΄Π΅Ρ‚ ΠΏΡ€ΠΎΠΈΠ·Π²ΠΎΠ΄ΠΈΡ‚ΡŒΡΡ Π² нСсколько этапов.

Для Π½Π°Ρ‡Π°Π»Π° ΠΏΠΎΡΠ»Π΅Π΄ΠΎΠ²Π°Ρ‚Π΅Π»ΡŒΠ½Ρ‹Π΅ элСмСнты ΠΌΠΎΠΆΠ½ΠΎ условно Π·Π°ΠΌΠ΅Π½ΠΈΡ‚ΡŒ ΠΎΠ΄Π½ΠΈΠΌ рСзистором, ΠΎΠ±Π»Π°Π΄Π°ΡŽΡ‰ΠΈΠΌ сопротивлСниСм, Ρ€Π°Π²Π½Ρ‹ΠΌ суммС Π΄Π²ΡƒΡ… замСняСмых. Π”Π°Π»Π΅Π΅ ΠΎΠ±Ρ‰Π΅Π΅ сопротивлСниС считаСм Ρ‚Π΅ΠΌ ΠΆΠ΅ способом, Ρ‡Ρ‚ΠΎ ΠΈ для ΠΏΡ€Π΅Π΄Ρ‹Π΄ΡƒΡ‰Π΅Π³ΠΎ ΠΏΡ€ΠΈΠΌΠ΅Ρ€Π°. Π”Π°Π½Π½Ρ‹ΠΉ ΠΌΠ΅Ρ‚ΠΎΠ΄ ΠΏΠΎΠ΄ΠΎΠΉΠ΄Π΅Ρ‚ ΠΈ для Π΄Ρ€ΡƒΠ³ΠΈΡ… Π±ΠΎΠ»Π΅Π΅ слоТных схСм. ΠŸΠΎΡΠ»Π΅Π΄ΠΎΠ²Π°Ρ‚Π΅Π»ΡŒΠ½ΠΎ упрощая схСму, ΠΌΠΎΠΆΠ½ΠΎ ΠΏΠΎΠ»ΡƒΡ‡ΠΈΡ‚ΡŒ Π½Π΅ΠΎΠ±Ρ…ΠΎΠ΄ΠΈΠΌΠΎΠ΅ Π·Π½Π°Ρ‡Π΅Π½ΠΈΠ΅.

НапримСр, Ссли вмСсто рСзистора R3 Π±ΡƒΠ΄ΡƒΡ‚ ΠΏΠΎΠ΄ΠΊΠ»ΡŽΡ‡Π΅Π½Ρ‹ Π΄Π²Π° ΠΏΠ°Ρ€Π°Π»Π»Π΅Π»ΡŒΠ½Ρ‹Ρ…, потрСбуСтся сначала Ρ€Π°ΡΡΡ‡ΠΈΡ‚Π°Ρ‚ΡŒ ΠΈΡ… сопротивлСниС, Π·Π°ΠΌΠ΅Π½ΠΈΠ² ΠΈΡ… эквивалСнтным. А Π΄Π°Π»Π΅Π΅ Ρ‚ΠΎ ΠΆΠ΅ самоС, Ρ‡Ρ‚ΠΎ ΠΈ Π² ΠΏΡ€ΠΈΠΌΠ΅Ρ€Π΅ Π²Ρ‹ΡˆΠ΅.

ΠŸΡ€ΠΈΠΌΠ΅Π½Π΅Π½ΠΈΠ΅ ΠΏΠ°Ρ€Π°Π»Π»Π΅Π»ΡŒΠ½ΠΎΠΉ схСмы

ΠŸΠ°Ρ€Π°Π»Π»Π΅Π»ΡŒΠ½ΠΎΠ΅ соСдинСниС рСзисторов Π½Π°Ρ…ΠΎΠ΄ΠΈΡ‚ своС ΠΏΡ€ΠΈΠΌΠ΅Π½Π΅Π½ΠΈΠ΅ Π²ΠΎ ΠΌΠ½ΠΎΠ³ΠΈΡ… случаях. ΠŸΠΎΡΠ»Π΅Π΄ΠΎΠ²Π°Ρ‚Π΅Π»ΡŒΠ½ΠΎΠ΅ ΠΏΠΎΠ΄ΠΊΠ»ΡŽΡ‡Π΅Π½ΠΈΠ΅ ΡƒΠ²Π΅Π»ΠΈΡ‡ΠΈΠ²Π°Π΅Ρ‚ сопротивлСниС, Π° для нашСго случая ΠΎΠ½ΠΎ ΡƒΠΌΠ΅Π½ΡŒΡˆΠΈΡ‚ΡΡ. НапримСр, для элСктричСской Ρ†Π΅ΠΏΠΈ трСбуСтся сопротивлСниС Π² 5 Ом, Π½ΠΎ Π΅ΡΡ‚ΡŒ Ρ‚ΠΎΠ»ΡŒΠΊΠΎ рСзисторы Π½Π° 10 Ом ΠΈ Π²Ρ‹ΡˆΠ΅. Из ΠΏΠ΅Ρ€Π²ΠΎΠ³ΠΎ ΠΏΡ€ΠΈΠΌΠ΅Ρ€Π° ΠΌΡ‹ Π·Π½Π°Π΅ΠΌ, Ρ‡Ρ‚ΠΎ ΠΌΠΎΠΆΠ½ΠΎ ΠΏΠΎΠ»ΡƒΡ‡ΠΈΡ‚ΡŒ Π² Π΄Π²Π° Ρ€Π°Π·Π° мСньшСС Π·Π½Π°Ρ‡Π΅Π½ΠΈΠ΅ сопротивлСния, Ссли ΡƒΡΡ‚Π°Π½ΠΎΠ²ΠΈΡ‚ΡŒ Π΄Π²Π° ΠΎΠ΄ΠΈΠ½Π°ΠΊΠΎΠ²Ρ‹Ρ… рСзистора ΠΏΠ°Ρ€Π°Π»Π»Π΅Π»ΡŒΠ½ΠΎ Π΄Ρ€ΡƒΠ³ Π΄Ρ€ΡƒΠ³Ρƒ.

Π£ΠΌΠ΅Π½ΡŒΡˆΠΈΡ‚ΡŒ сопротивлСниС ΠΌΠΎΠΆΠ½ΠΎ Π΅Ρ‰Π΅ большС, Π½Π°ΠΏΡ€ΠΈΠΌΠ΅Ρ€, Ссли Π΄Π²Π΅ ΠΏΠ°Ρ€Ρ‹ ΠΏΠ°Ρ€Π°Π»Π»Π΅Π»ΡŒΠ½ΠΎ соСдинСнных рСзисторов ΡΠΎΠ΅Π΄ΠΈΠ½ΠΈΡ‚ΡŒ ΠΏΠ°Ρ€Π°Π»Π»Π΅Π»ΡŒΠ½ΠΎ ΠΎΡ‚Π½ΠΎΡΠΈΡ‚Π΅Π»ΡŒΠ½ΠΎ Π΄Ρ€ΡƒΠ³ Π΄Ρ€ΡƒΠ³Π°. МоТно ΡƒΠΌΠ΅Π½ΡŒΡˆΠΈΡ‚ΡŒ сопротивлСниС Π΅Ρ‰Π΅ Π² Π΄Π²Π° Ρ€Π°Π·Π°, Ссли рСзисторы ΠΈΠΌΠ΅ΡŽΡ‚ ΠΎΠ΄ΠΈΠ½Π°ΠΊΠΎΠ²ΠΎΠ΅ сопротивлСниС. ΠšΠΎΠΌΠ±ΠΈΠ½ΠΈΡ€ΡƒΡ с ΠΏΠΎΡΠ»Π΅Π΄ΠΎΠ²Π°Ρ‚Π΅Π»ΡŒΠ½Ρ‹ΠΌ соСдинСниСм, ΠΌΠΎΠΆΠ½ΠΎ ΠΏΠΎΠ»ΡƒΡ‡ΠΈΡ‚ΡŒ любоС Π·Π½Π°Ρ‡Π΅Π½ΠΈΠ΅.

Π’Ρ‚ΠΎΡ€ΠΎΠΉ ΠΏΡ€ΠΈΠΌΠ΅Ρ€ — это использованиС ΠΏΠ°Ρ€Π°Π»Π»Π΅Π»ΡŒΠ½ΠΎΠ³ΠΎ ΠΏΠΎΠ΄ΠΊΠ»ΡŽΡ‡Π΅Π½ΠΈΡ для освСщСния ΠΈ Ρ€ΠΎΠ·Π΅Ρ‚ΠΎΠΊ Π² ΠΊΠ²Π°Ρ€Ρ‚ΠΈΡ€Π°Ρ…. Благодаря Ρ‚Π°ΠΊΠΎΠΌΡƒ ΠΏΠΎΠ΄ΠΊΠ»ΡŽΡ‡Π΅Π½ΠΈΡŽ напряТСниС Π½Π° ΠΊΠ°ΠΆΠ΄ΠΎΠΌ элСмСнтС Π½Π΅ Π±ΡƒΠ΄Π΅Ρ‚ Π·Π°Π²ΠΈΡΠ΅Ρ‚ΡŒ ΠΎΡ‚ ΠΈΡ… количСства ΠΈ Π±ΡƒΠ΄Π΅Ρ‚ ΠΎΠ΄ΠΈΠ½Π°ΠΊΠΎΠ²Ρ‹ΠΌ.

Π•Ρ‰Π΅ ΠΎΠ΄ΠΈΠ½ ΠΏΡ€ΠΈΠΌΠ΅Ρ€ использования ΠΏΠ°Ρ€Π°Π»Π»Π΅Π»ΡŒΠ½ΠΎΠ³ΠΎ ΠΏΠΎΠ΄ΠΊΠ»ΡŽΡ‡Π΅Π½ΠΈΡ — это Π·Π°Ρ‰ΠΈΡ‚Π½ΠΎΠ΅ Π·Π°Π·Π΅ΠΌΠ»Π΅Π½ΠΈΠ΅ элСктрооборудования. НапримСр, Ссли Ρ‡Π΅Π»ΠΎΠ²Π΅ΠΊ касаСтся мСталличСского корпуса ΠΏΡ€ΠΈΠ±ΠΎΡ€Π°, Π½Π° ΠΊΠΎΡ‚ΠΎΡ€Ρ‹ΠΉ ΠΏΡ€ΠΎΠΈΠ·ΠΎΠΉΠ΄Π΅Ρ‚ ΠΏΡ€ΠΎΠ±ΠΎΠΉ, получится ΠΏΠ°Ρ€Π°Π»Π»Π΅Π»ΡŒΠ½ΠΎΠ΅ соСдинСния Π΅Π³ΠΎ ΠΈ Π·Π°Ρ‰ΠΈΡ‚Π½ΠΎΠ³ΠΎ ΠΏΡ€ΠΎΠ²ΠΎΠ΄Π½ΠΈΠΊΠ°. ΠŸΠ΅Ρ€Π²Ρ‹ΠΌ ΡƒΠ·Π»ΠΎΠΌ Π±ΡƒΠ΄Π΅Ρ‚ мСсто прикосновСния, Π° Π²Ρ‚ΠΎΡ€Ρ‹ΠΌ нулСвая Ρ‚ΠΎΡ‡ΠΊΠ° трансформатора. По ΠΏΡ€ΠΎΠ²ΠΎΠ΄Π½ΠΈΠΊΡƒ ΠΈ Ρ‡Π΅Π»ΠΎΠ²Π΅ΠΊΡƒ Π±ΡƒΠ΄Π΅Ρ‚ Ρ‚Π΅Ρ‡ΡŒ Ρ€Π°Π·Π½Ρ‹ΠΉ Ρ‚ΠΎΠΊ. Π’Π΅Π»ΠΈΡ‡ΠΈΠ½Ρƒ сопротивлСния послСднСго ΠΏΡ€ΠΈΠ½ΠΈΠΌΠ°ΡŽΡ‚ Π·Π° 1000 Ом, хотя Ρ€Π΅Π°Π»ΡŒΠ½ΠΎΠ΅ Π·Π½Π°Ρ‡Π΅Π½ΠΈΠ΅ Π·Π°Ρ‡Π°ΡΡ‚ΡƒΡŽ Π³ΠΎΡ€Π°Π·Π΄ΠΎ большС. Если Π±Ρ‹ Π½Π΅ Π±Ρ‹Π»ΠΎ зазСмлСния, вСсь Ρ‚ΠΎΠΊ, ΠΏΡ€ΠΎΡ‚Π΅ΠΊΠ°ΡŽΡ‰ΠΈΠΉ Π² схСмС, пошСл Π±Ρ‹ Ρ‡Π΅Ρ€Π΅Π· Ρ‡Π΅Π»ΠΎΠ²Π΅ΠΊΠ°, Ρ‚Π°ΠΊ ΠΊΠ°ΠΊ ΠΎΠ½ Π±Ρ‹Π» Π±Ρ‹ СдинствСнным ΠΏΡ€ΠΎΠ²ΠΎΠ΄Π½ΠΈΠΊΠΎΠΌ.

ΠŸΠ°Ρ€Π°Π»Π»Π΅Π»ΡŒΠ½ΠΎΠ΅ соСдинСниС ΠΌΠΎΠΆΠ΅Ρ‚ ΠΈΡΠΏΠΎΠ»ΡŒΠ·ΠΎΠ²Π°Ρ‚ΡŒΡΡ ΠΈ для Π±Π°Ρ‚Π°Ρ€Π΅ΠΉ. НапряТСниС ΠΏΡ€ΠΈ этом остаСтся ΠΏΡ€Π΅ΠΆΠ½ΠΈΠΌ, ΠΎΠ΄Π½Π°ΠΊΠΎ Π² Π΄Π²Π° Ρ€Π°Π·Π° возрастаСт ΠΈΡ… Π΅ΠΌΠΊΠΎΡΡ‚ΡŒ.

Π˜Ρ‚ΠΎΠ³

ΠŸΡ€ΠΈ ΠΏΠΎΠ΄ΠΊΠ»ΡŽΡ‡Π΅Π½ΠΈΠΈ рСзисторов ΠΏΠ°Ρ€Π°Π»Π»Π΅Π»ΡŒΠ½ΠΎ, напряТСниС Π½Π° Π½ΠΈΡ… Π±ΡƒΠ΄Π΅Ρ‚ ΠΎΠ΄ΠΈΠ½Π°ΠΊΠΎΠ²Ρ‹ΠΌ, Π° Ρ‚ΠΎΠΊ Ρ€Π°Π²Π΅Π½ суммС ΠΏΡ€ΠΎΡ‚Π΅ΠΊΠ°ΡŽΡ‰ΠΈΡ… Ρ‡Π΅Ρ€Π΅Π· ΠΊΠ°ΠΆΠ΄Ρ‹ΠΉ рСзистор. ΠŸΡ€ΠΎΠ²ΠΎΠ΄ΠΈΠΌΠΎΡΡ‚ΡŒ Π±ΡƒΠ΄Π΅Ρ‚ Ρ€ΠΎΠ²Π½ΡΡ‚ΡŒΡΡ суммС ΠΊΠ°ΠΆΠ΄ΠΎΠ³ΠΎ. ΠžΡ‚ этого ΠΈ получаСтся нСобычная Ρ„ΠΎΡ€ΠΌΡƒΠ»Π° суммарного сопротивлСния рСзисторов.

НСобходимо ΡƒΡ‡ΠΈΡ‚Ρ‹Π²Π°Ρ‚ΡŒ ΠΏΡ€ΠΈ расчСтС ΠΏΠ°Ρ€Π°Π»Π»Π΅Π»ΡŒΠ½ΠΎΠ³ΠΎ соСдинСния рСзисторов Ρ‚ΠΎ, Ρ‡Ρ‚ΠΎ ΠΈΡ‚ΠΎΠ³ΠΎΠ²ΠΎΠ΅ сопротивлСниС Π±ΡƒΠ΄Π΅Ρ‚ всСгда мСньшС самого малСнького. Π­Ρ‚ΠΎ Ρ‚Π°ΠΊΠΆΠ΅ ΠΌΠΎΠΆΠ½ΠΎ ΠΎΠ±ΡŠΡΡΠ½ΠΈΡ‚ΡŒ суммированиСм проводимости рСзисторов. ПослСдняя Π±ΡƒΠ΄Π΅Ρ‚ Π²ΠΎΠ·Ρ€Π°ΡΡ‚Π°Ρ‚ΡŒ ΠΏΡ€ΠΈ Π΄ΠΎΠ±Π°Π²Π»Π΅Π½ΠΈΠΈ Π½ΠΎΠ²Ρ‹Ρ… элСмСнтов, соотвСтствСнно ΠΈ ΠΏΡ€ΠΎΠ²ΠΎΠ΄ΠΈΠΌΠΎΡΡ‚ΡŒ Π±ΡƒΠ΄Π΅Ρ‚ ΡƒΠΌΠ΅Π½ΡŒΡˆΠ°Ρ‚ΡŒΡΡ.

БмСшанноС соСдинСниС рСзисторов Ρ„ΠΎΡ€ΠΌΡƒΠ»Π° — ΠœΠΎΡ€ΡΠΊΠΎΠΉ Ρ„Π»ΠΎΡ‚

РСзистор прСдставляСт собой устройство, ΠΎΠ±Π»Π°Π΄Π°ΡŽΡ‰Π΅Π΅ устойчивым, ΡΡ‚Π°Π±ΠΈΠ»ΡŒΠ½Ρ‹ΠΌ Π·Π½Π°Ρ‡Π΅Π½ΠΈΠ΅ΠΌ сопротивлСния. Π­Ρ‚ΠΎ позволяСт Π²Ρ‹ΠΏΠΎΠ»Π½ΡΡ‚ΡŒ Ρ€Π΅Π³ΡƒΠ»ΠΈΡ€ΠΎΠ²ΠΊΡƒ ΠΏΠ°Ρ€Π°ΠΌΠ΅Ρ‚Ρ€ΠΎΠ² Π½Π° Π»ΡŽΠ±Ρ‹Ρ… участках элСктричСской Ρ†Π΅ΠΏΠΈ. Π‘ΡƒΡ‰Π΅ΡΡ‚Π²ΡƒΡŽΡ‚ Ρ€Π°Π·Π»ΠΈΡ‡Π½Ρ‹Π΅ Π²ΠΈΠ΄Ρ‹ соСдинСний, Π² Ρ‚ΠΎΠΌ числС ΠΈ смСшанноС соСдинСниС рСзисторов. ΠžΡ‚ использования Ρ‚ΠΎΠ³ΠΎ ΠΈΠ»ΠΈ ΠΈΠ½ΠΎΠ³ΠΎ способа Π² ΠΊΠΎΠ½ΠΊΡ€Π΅Ρ‚Π½ΠΎΠΉ схСмС, Π½Π°ΠΏΡ€ΡΠΌΡƒΡŽ зависит ΠΏΠ°Π΄Π΅Π½ΠΈΠ΅ напряТСний ΠΈ распрСдСлСниС Ρ‚ΠΎΠΊΠΎΠ² Π² Ρ†Π΅ΠΏΠΈ. Π’Π°Ρ€ΠΈΠ°Π½Ρ‚ смСшанного соСдинСния состоит ΠΈΠ· ΠΏΠΎΡΠ»Π΅Π΄ΠΎΠ²Π°Ρ‚Π΅Π»ΡŒΠ½ΠΎΠ³ΠΎ ΠΈ ΠΏΠ°Ρ€Π°Π»Π»Π΅Π»ΡŒΠ½ΠΎΠ³ΠΎ ΠΏΠΎΠ΄ΠΊΠ»ΡŽΡ‡Π΅Π½ΠΈΡ Π°ΠΊΡ‚ΠΈΠ²Π½Ρ‹Ρ… сопротивлСний. ΠŸΠΎΡΡ‚ΠΎΠΌΡƒ Π² ΠΏΠ΅Ρ€Π²ΡƒΡŽ ΠΎΡ‡Π΅Ρ€Π΅Π΄ΡŒ Π½ΡƒΠΆΠ½ΠΎ Ρ€Π°ΡΡΠΌΠ°Ρ‚Ρ€ΠΈΠ²Π°Ρ‚ΡŒ эти Π΄Π²Π° Π²ΠΈΠ΄Π° соСдинСний, Ρ‡Ρ‚ΠΎΠ±Ρ‹ ΠΏΠΎΠ½ΡΡ‚ΡŒ, ΠΊΠ°ΠΊ Ρ€Π°Π±ΠΎΡ‚Π°ΡŽΡ‚ Π΄Ρ€ΡƒΠ³ΠΈΠ΅ схСмы.

ΠŸΠΎΡΠ»Π΅Π΄ΠΎΠ²Π°Ρ‚Π΅Π»ΡŒΠ½ΠΎΠ΅ соСдинСниС

ΠŸΠΎΡΠ»Π΅Π΄ΠΎΠ²Π°Ρ‚Π΅Π»ΡŒΠ½Π°Ρ схСма ΠΏΠΎΠ΄ΠΊΠ»ΡŽΡ‡Π΅Π½ΠΈΡ ΠΏΡ€Π΅Π΄ΠΏΠΎΠ»Π°Π³Π°Π΅Ρ‚ располоТСниС рСзисторов Π² схСмС Ρ‚Π°ΠΊΠΈΠΌ ΠΎΠ±Ρ€Π°Π·ΠΎΠΌ, Ρ‡Ρ‚ΠΎ ΠΊΠΎΠ½Π΅Ρ† ΠΏΠ΅Ρ€Π²ΠΎΠ³ΠΎ элСмСнта соСдиняСтся с Π½Π°Ρ‡Π°Π»ΠΎΠΌ Π²Ρ‚ΠΎΡ€ΠΎΠ³ΠΎ, Π° ΠΊΠΎΠ½Π΅Ρ† Π²Ρ‚ΠΎΡ€ΠΎΠ³ΠΎ – с Π½Π°Ρ‡Π°Π»ΠΎΠΌ Ρ‚Ρ€Π΅Ρ‚ΡŒΠ΅Π³ΠΎ ΠΈ Ρ‚.Π΄. Π’ΠΎ Π΅ΡΡ‚ΡŒ всС рСзисторы ΠΏΠΎΠΎΡ‡Π΅Ρ€Π΅Π΄Π½ΠΎ ΡΠ»Π΅Π΄ΡƒΡŽΡ‚ Π΄Ρ€ΡƒΠ³ Π·Π° Π΄Ρ€ΡƒΠ³ΠΎΠΌ. Π‘ΠΈΠ»Π° Ρ‚ΠΎΠΊΠ° ΠΏΡ€ΠΈ ΠΏΠΎΡΠ»Π΅Π΄ΠΎΠ²Π°Ρ‚Π΅Π»ΡŒΠ½ΠΎΠΌ соСдинСнии Π±ΡƒΠ΄Π΅Ρ‚ ΠΎΠ΄ΠΈΠ½Π°ΠΊΠΎΠ²ΠΎΠΉ Π² ΠΊΠ°ΠΆΠ΄ΠΎΠΌ элСмСнтС. Π’ Π²ΠΈΠ΄Π΅ Ρ„ΠΎΡ€ΠΌΡƒΠ»Ρ‹ это выглядит ΡΠ»Π΅Π΄ΡƒΡŽΡ‰ΠΈΠΌ ΠΎΠ±Ρ€Π°Π·ΠΎΠΌ: IΠΎΠ±Ρ‰ = I1 = I2, Π³Π΄Π΅ IΠΎΠ±Ρ‰ являСтся ΠΎΠ±Ρ‰ΠΈΠΌ Ρ‚ΠΎΠΊΠΎΠΌ Ρ†Π΅ΠΏΠΈ, I1 ΠΈ I2 – ΡΠΎΠΎΡ‚Π²Π΅Ρ‚ΡΡ‚Π²ΡƒΡŽΡ‚ Ρ‚ΠΎΠΊΠ°ΠΌ 1-Π³ΠΎ ΠΈ 2-Π³ΠΎ рСзистора.

Π’ соотвСтствии с Π·Π°ΠΊΠΎΠ½ΠΎΠΌ Ома, напряТСниС источника питания Π±ΡƒΠ΄Π΅Ρ‚ Ρ€Π°Π²Π½ΠΎ суммС ΠΏΠ°Π΄Π΅Π½ΠΈΠΉ напряТСния Π½Π° ΠΊΠ°ΠΆΠ΄ΠΎΠΌ рСзисторС: UΠΎΠ±Ρ‰ = U1 + U2 = I1r1 + I2r2, Π² ΠΊΠΎΡ‚ΠΎΡ€ΠΎΠΉ UΠΎΠ±Ρ‰ – напряТСниС источника элСктроэнСргии ΠΈΠ»ΠΈ самой сСти; U1 ΠΈ U2 – Π·Π½Π°Ρ‡Π΅Π½ΠΈΠ΅ ΠΏΠ°Π΄Π΅Π½ΠΈΠΉ напряТСния Π½Π° 1-ΠΌ ΠΈ 2-ΠΌ рСзисторах; r1 ΠΈ r2 – сопротивлСния 1-Π³ΠΎ ΠΈ 2-Π³ΠΎ рСзисторов. ΠŸΠΎΡΠΊΠΎΠ»ΡŒΠΊΡƒ Ρ‚ΠΎΠΊΠΈ Π½Π° любом участкС Ρ†Π΅ΠΏΠΈ ΠΈΠΌΠ΅ΡŽΡ‚ ΠΎΠ΄ΠΈΠ½Π°ΠΊΠΎΠ²ΠΎΠ΅ Π·Π½Π°Ρ‡Π΅Π½ΠΈΠ΅, Ρ„ΠΎΡ€ΠΌΡƒΠ»Π° ΠΏΡ€ΠΈΠΎΠ±Ρ€Π΅Ρ‚Π°Π΅Ρ‚ Π²ΠΈΠ΄: UΠΎΠ±Ρ‰ = I(r1 + r2).

Π’Π°ΠΊΠΈΠΌ ΠΎΠ±Ρ€Π°Π·ΠΎΠΌ, ΠΌΠΎΠΆΠ½ΠΎ ΡΠ΄Π΅Π»Π°Ρ‚ΡŒ Π²Ρ‹Π²ΠΎΠ΄, Ρ‡Ρ‚ΠΎ ΠΏΡ€ΠΈ ΠΏΠΎΡΠ»Π΅Π΄ΠΎΠ²Π°Ρ‚Π΅Π»ΡŒΠ½ΠΎΠΉ схСмС Π²ΠΊΠ»ΡŽΡ‡Π΅Π½ΠΈΡ рСзисторов, элСктричСский Ρ‚ΠΎΠΊ, ΠΏΡ€ΠΎΡ‚Π΅ΠΊΠ°ΡŽΡ‰ΠΈΠΉ Ρ‡Π΅Ρ€Π΅Π· ΠΊΠ°ΠΆΠ΄Ρ‹ΠΉ ΠΈΠ· Π½ΠΈΡ… Ρ€Π°Π²Π΅Π½ ΠΎΠ±Ρ‰Π΅ΠΌΡƒ Π·Π½Π°Ρ‡Π΅Π½ΠΈΡŽ Ρ‚ΠΎΠΊΠ° Π²ΠΎ всСй Ρ†Π΅ΠΏΠΈ. НапряТСниС Π½Π° ΠΊΠ°ΠΆΠ΄ΠΎΠΌ рСзисторС Π±ΡƒΠ΄Π΅Ρ‚ Ρ€Π°Π·Π½ΠΎΠ΅, ΠΎΠ΄Π½Π°ΠΊΠΎ ΠΈΡ… общая сумма составит Π·Π½Π°Ρ‡Π΅Π½ΠΈΠ΅, Ρ€Π°Π²Π½ΠΎΠ΅ ΠΎΠ±Ρ‰Π΅ΠΌΡƒ Π½Π°ΠΏΡ€ΡΠΆΠ΅Π½ΠΈΡŽ всСй элСктричСской Ρ†Π΅ΠΏΠΈ. ΠžΠ±Ρ‰Π΅Π΅ сопротивлСниС Ρ†Π΅ΠΏΠΈ Ρ‚Π°ΠΊΠΆΠ΅ Π±ΡƒΠ΄Π΅Ρ‚ Ρ€Π°Π²Π½ΠΎ суммС сопротивлСний ΠΊΠ°ΠΆΠ΄ΠΎΠ³ΠΎ рСзистора, Π²ΠΊΠ»ΡŽΡ‡Π΅Π½Π½ΠΎΠ³ΠΎ Π² эту Ρ†Π΅ΠΏΡŒ.

ΠŸΠ°Ρ€Π°ΠΌΠ΅Ρ‚Ρ€Ρ‹ Ρ†Π΅ΠΏΠΈ ΠΏΡ€ΠΈ ΠΏΠ°Ρ€Π°Π»Π»Π΅Π»ΡŒΠ½ΠΎΠΌ соСдинСнии

ΠŸΠ°Ρ€Π°Π»Π»Π΅Π»ΡŒΠ½ΠΎΠ΅ соСдинСниС прСдставляСт собой Π²ΠΊΠ»ΡŽΡ‡Π΅Π½ΠΈΠ΅ Π½Π°Ρ‡Π°Π»ΡŒΠ½Ρ‹Ρ… Π²Ρ‹Ρ…ΠΎΠ΄ΠΎΠ² Π΄Π²ΡƒΡ… ΠΈ Π±ΠΎΠ»Π΅Π΅ рСзисторов Π² Π΅Π΄ΠΈΠ½ΠΎΠΉ Ρ‚ΠΎΡ‡ΠΊΠ΅, ΠΈ ΠΊΠΎΠ½Ρ†ΠΎΠ² этих ΠΆΠ΅ элСмСнтов Π² Π΄Ρ€ΡƒΠ³ΠΎΠΉ ΠΎΠ±Ρ‰Π΅ΠΉ Ρ‚ΠΎΡ‡ΠΊΠ΅. Π’Π°ΠΊΠΈΠΌ ΠΎΠ±Ρ€Π°Π·ΠΎΠΌ, фактичСски происходит соСдинСниС ΠΊΠ°ΠΆΠ΄ΠΎΠ³ΠΎ рСзистора нСпосрСдствСнно с источником элСктроэнСргии.

Π’ Ρ€Π΅Π·ΡƒΠ»ΡŒΡ‚Π°Ρ‚Π΅, напряТСниС ΠΊΠ°ΠΆΠ΄ΠΎΠ³ΠΎ рСзистора Π±ΡƒΠ΄Π΅Ρ‚ ΠΎΠ΄ΠΈΠ½Π°ΠΊΠΎΠ²Ρ‹ΠΌ с ΠΎΠ±Ρ‰ΠΈΠΌ напряТСниСм Ρ†Π΅ΠΏΠΈ: UΠΎΠ±Ρ‰ = U1 = U2. Π’ свою ΠΎΡ‡Π΅Ρ€Π΅Π΄ΡŒ, Π·Π½Π°Ρ‡Π΅Π½ΠΈΠ΅ Ρ‚ΠΎΠΊΠΎΠ² Π±ΡƒΠ΄Π΅Ρ‚ Ρ€Π°Π·Π½Ρ‹ΠΌ Π½Π° ΠΊΠ°ΠΆΠ΄ΠΎΠΌ рСзисторС, ΠΈΡ… распрСдСлСниС становится прямо ΠΏΡ€ΠΎΠΏΠΎΡ€Ρ†ΠΈΠΎΠ½Π°Π»ΡŒΠ½Ρ‹ΠΌ ΡΠΎΠΏΡ€ΠΎΡ‚ΠΈΠ²Π»Π΅Π½ΠΈΡŽ этих рСзисторов. Π’ΠΎ Π΅ΡΡ‚ΡŒ, ΠΏΡ€ΠΈ ΡƒΠ²Π΅Π»ΠΈΡ‡Π΅Π½ΠΈΠΈ сопротивлСния, сила Ρ‚ΠΎΠΊΠ° ΡƒΠΌΠ΅Π½ΡŒΡˆΠ°Π΅Ρ‚ΡΡ, Π° ΠΎΠ±Ρ‰ΠΈΠΉ Ρ‚ΠΎΠΊ становится Ρ€Π°Π²Π΅Π½ суммС Ρ‚ΠΎΠΊΠΎΠ², проходящих Ρ‡Π΅Ρ€Π΅Π· ΠΊΠ°ΠΆΠ΄Ρ‹ΠΉ элСмСнт. Π€ΠΎΡ€ΠΌΡƒΠ»Π° для Π΄Π°Π½Π½ΠΎΠ³ΠΎ полоТСния выглядит ΡΠ»Π΅Π΄ΡƒΡŽΡ‰ΠΈΠΌ ΠΎΠ±Ρ€Π°Π·ΠΎΠΌ: IΠΎΠ±Ρ‰= I1 + I2.

Для расчСтов ΠΎΠ±Ρ‰Π΅Π³ΠΎ сопротивлСния ΠΈΡΠΏΠΎΠ»ΡŒΠ·ΡƒΠ΅Ρ‚ΡΡ Ρ„ΠΎΡ€ΠΌΡƒΠ»Π°: . Она ΠΈΡΠΏΠΎΠ»ΡŒΠ·ΡƒΠ΅Ρ‚ΡΡ ΠΏΡ€ΠΈ Π½Π°Π»ΠΈΡ‡ΠΈΠΈ Π² Ρ†Π΅ΠΏΠΈ Ρ‚ΠΎΠ»ΡŒΠΊΠΎ Π΄Π²ΡƒΡ… сопротивлСний. Π’ Ρ‚Π΅Ρ… случаях, ΠΊΠΎΠ³Π΄Π° сопротивлСний Π² Ρ†Π΅ΠΏΠΈ ΠΏΠΎΠ΄ΠΊΠ»ΡŽΡ‡Π΅Π½ΠΎ Ρ‚Ρ€ΠΈ ΠΈ Π±ΠΎΠ»Π΅Π΅, примСняСтся другая Ρ„ΠΎΡ€ΠΌΡƒΠ»Π°:

Π’Π°ΠΊΠΈΠΌ ΠΎΠ±Ρ€Π°Π·ΠΎΠΌ, Π·Π½Π°Ρ‡Π΅Π½ΠΈΠ΅ ΠΎΠ±Ρ‰Π΅Π³ΠΎ сопротивлСния элСктричСской Ρ†Π΅ΠΏΠΈ Π±ΡƒΠ΄Π΅Ρ‚ мСньшС, Ρ‡Π΅ΠΌ самоС минимальноС сопротивлСниС ΠΎΠ΄Π½ΠΎΠ³ΠΎ ΠΈΠ· рСзисторов, ΠΏΠΎΠ΄ΠΊΠ»ΡŽΡ‡Π΅Π½Π½Ρ‹Ρ… ΠΏΠ°Ρ€Π°Π»Π»Π΅Π»ΡŒΠ½ΠΎ Π² эту Ρ†Π΅ΠΏΡŒ. На ΠΊΠ°ΠΆΠ΄Ρ‹ΠΉ элСмСнт поступаСт напряТСниС, ΠΎΠ΄ΠΈΠ½Π°ΠΊΠΎΠ²ΠΎΠ΅ с напряТСниСм источника элСктроэнСргии. РаспрСдСлСниС Ρ‚ΠΎΠΊΠ° Π±ΡƒΠ΄Π΅Ρ‚ прямо ΠΏΡ€ΠΎΠΏΠΎΡ€Ρ†ΠΈΠΎΠ½Π°Π»ΡŒΠ½Ρ‹ΠΌ ΡΠΎΠΏΡ€ΠΎΡ‚ΠΈΠ²Π»Π΅Π½ΠΈΡŽ рСзисторов. Π—Π½Π°Ρ‡Π΅Π½ΠΈΠ΅ ΠΎΠ±Ρ‰Π΅Π³ΠΎ сопротивлСния рСзисторов, соСдинСнных ΠΏΠ°Ρ€Π°Π»Π»Π΅Π»ΡŒΠ½ΠΎ, Π½Π΅ Π΄ΠΎΠ»ΠΆΠ½ΠΎ ΠΏΡ€Π΅Π²Ρ‹ΡˆΠ°Ρ‚ΡŒ минимального сопротивлСния ΠΊΠ°ΠΊΠΎΠ³ΠΎ-Π»ΠΈΠ±ΠΎ элСмСнта.

Π‘Ρ…Π΅ΠΌΠ° смСшанного соСдинСния рСзисторов

Π‘Ρ…Π΅ΠΌΠ° смСшанного соСдинСния ΠΎΠ±Π»Π°Π΄Π°Π΅Ρ‚ свойствами схСм ΠΏΠΎΡΠ»Π΅Π΄ΠΎΠ²Π°Ρ‚Π΅Π»ΡŒΠ½ΠΎΠ³ΠΎ ΠΈ ΠΏΠ°Ρ€Π°Π»Π»Π΅Π»ΡŒΠ½ΠΎΠ³ΠΎ соСдинСния рСзисторов. Π’ этом случаС элСмСнты частично ΠΏΠΎΠ΄ΠΊΠ»ΡŽΡ‡Π°ΡŽΡ‚ΡΡ ΠΏΠΎΡΠ»Π΅Π΄ΠΎΠ²Π°Ρ‚Π΅Π»ΡŒΠ½ΠΎ, Π° другая Ρ‡Π°ΡΡ‚ΡŒ соСдиняСтся ΠΏΠ°Ρ€Π°Π»Π»Π΅Π»ΡŒΠ½ΠΎ. На прСдставлСнной схСмС рСзисторы R1 ΠΈ R2 Π²ΠΊΠ»ΡŽΡ‡Π΅Π½Ρ‹ ΠΏΠΎΡΠ»Π΅Π΄ΠΎΠ²Π°Ρ‚Π΅Π»ΡŒΠ½ΠΎ, Π° рСзистор R3 соСдинСн ΠΏΠ°Ρ€Π°Π»Π»Π΅Π»ΡŒΠ½ΠΎ с Π½ΠΈΠΌΠΈ. Π’ свою ΠΎΡ‡Π΅Ρ€Π΅Π΄ΡŒ рСзистор R4 Π²ΠΊΠ»ΡŽΡ‡Π°Π΅Ρ‚ΡΡ ΠΏΠΎΡΠ»Π΅Π΄ΠΎΠ²Π°Ρ‚Π΅Π»ΡŒΠ½ΠΎ с ΠΏΡ€Π΅Π΄Ρ‹Π΄ΡƒΡ‰Π΅ΠΉ Π³Ρ€ΡƒΠΏΠΏΠΎΠΉ рСзисторов R1, R2 ΠΈ R3.

РасчСт сопротивлСния для Ρ‚Π°ΠΊΠΎΠΉ Ρ†Π΅ΠΏΠΈ сопряТСн с ΠΎΠΏΡ€Π΅Π΄Π΅Π»Π΅Π½Π½Ρ‹ΠΌΠΈ трудностями. Для Ρ‚ΠΎΠ³ΠΎ Ρ‡Ρ‚ΠΎΠ±Ρ‹ ΠΏΡ€Π°Π²ΠΈΠ»ΡŒΠ½ΠΎ Π²Ρ‹ΠΏΠΎΠ»Π½ΠΈΡ‚ΡŒ расчСты ΠΈΡΠΏΠΎΠ»ΡŒΠ·ΡƒΠ΅Ρ‚ΡΡ ΠΌΠ΅Ρ‚ΠΎΠ΄ прСобразования. Он Π·Π°ΠΊΠ»ΡŽΡ‡Π°Π΅Ρ‚ΡΡ Π² ΠΏΠΎΡΠ»Π΅Π΄ΠΎΠ²Π°Ρ‚Π΅Π»ΡŒΠ½ΠΎΠΌ ΠΏΡ€Π΅ΠΎΠ±Ρ€Π°Π·ΠΎΠ²Π°Π½ΠΈΠΈ слоТной Ρ†Π΅ΠΏΠΈ Π² ΠΏΡ€ΠΎΡΡ‚Π΅ΠΉΡˆΡƒΡŽ Ρ†Π΅ΠΏΡŒ Π·Π° нСсколько этапов.

Если для ΠΏΡ€ΠΈΠΌΠ΅Ρ€Π° вновь ΠΈΡΠΏΠΎΠ»ΡŒΠ·ΠΎΠ²Π°Ρ‚ΡŒ ΠΏΡ€Π΅Π΄ΡΡ‚Π°Π²Π»Π΅Π½Π½ΡƒΡŽ схСму, Ρ‚ΠΎ Π² самом Π½Π°Ρ‡Π°Π»Π΅ опрСдСляСтся сопротивлСниС R12 рСзисторов R1 ΠΈ R2, Π²ΠΊΠ»ΡŽΡ‡Π΅Π½Π½Ρ‹Ρ… ΠΏΠΎΡΠ»Π΅Π΄ΠΎΠ²Π°Ρ‚Π΅Π»ΡŒΠ½ΠΎ: R12 = R1 + R2. Π”Π°Π»Π΅Π΅, Π½ΡƒΠΆΠ½ΠΎ ΠΎΠΏΡ€Π΅Π΄Π΅Π»ΠΈΡ‚ΡŒ сопротивлСниС рСзисторов R123, Π²ΠΊΠ»ΡŽΡ‡Π΅Π½Π½Ρ‹Ρ… ΠΏΠ°Ρ€Π°Π»Π»Π΅Π»ΡŒΠ½ΠΎ, ΠΏΠΎ ΡΠ»Π΅Π΄ΡƒΡŽΡ‰Π΅ΠΉ Ρ„ΠΎΡ€ΠΌΡƒΠ»Π΅: R123=R12R3/(R12+R3) = (R1+R2)R3/(R1+R2+R3). На послСднСм этапС выполняСтся расчСт эквивалСнтного сопротивлСния всСй Ρ†Π΅ΠΏΠΈ, ΠΏΡƒΡ‚Π΅ΠΌ суммирования ΠΏΠΎΠ»ΡƒΡ‡Π΅Π½Π½Ρ‹Ρ… Π΄Π°Π½Π½Ρ‹Ρ… R123 ΠΈ сопротивлСния R4, Π²ΠΊΠ»ΡŽΡ‡Π΅Π½Π½ΠΎΠ³ΠΎ ΠΏΠΎΡΠ»Π΅Π΄ΠΎΠ²Π°Ρ‚Π΅Π»ΡŒΠ½ΠΎ с Π½ΠΈΠΌ: Rэк = R123 + R4 = (R1 + R2) R3 / (R1 + R2 + R3) + R4.

Π’ Π·Π°ΠΊΠ»ΡŽΡ‡Π΅Π½ΠΈΠ΅ слСдуСт ΠΎΡ‚ΠΌΠ΅Ρ‚ΠΈΡ‚ΡŒ, Ρ‡Ρ‚ΠΎ смСшанноС соСдинСниС рСзисторов ΠΎΠ±Π»Π°Π΄Π°Π΅Ρ‚ ΠΏΠΎΠ»ΠΎΠΆΠΈΡ‚Π΅Π»ΡŒΠ½Ρ‹ΠΌΠΈ ΠΈ ΠΎΡ‚Ρ€ΠΈΡ†Π°Ρ‚Π΅Π»ΡŒΠ½Ρ‹ΠΌΠΈ качСствами ΠΏΠΎΡΠ»Π΅Π΄ΠΎΠ²Π°Ρ‚Π΅Π»ΡŒΠ½ΠΎΠ³ΠΎ ΠΈ ΠΏΠ°Ρ€Π°Π»Π»Π΅Π»ΡŒΠ½ΠΎΠ³ΠΎ соСдинСния. Π­Ρ‚ΠΎ свойство ΡƒΡΠΏΠ΅ΡˆΠ½ΠΎ ΠΈΡΠΏΠΎΠ»ΡŒΠ·ΡƒΠ΅Ρ‚ΡΡ Π½Π° ΠΏΡ€Π°ΠΊΡ‚ΠΈΠΊΠ΅ Π² элСктричСских схСмах.

ΠŸΠžΠ‘Π›Π•Π”ΠžΠ’ΠΠ’Π•Π›Π¬ΠΠžΠ• Π‘ΠžΠ•Π”Π˜ΠΠ•ΠΠ˜Π• Π Π•Π—Π˜Π‘Π’ΠžΠ ΠžΠ’

Π­Ρ‚ΠΎ Ρ‚Π°ΠΊΠΎΠ΅ соСдинСниС, ΠΏΡ€ΠΈ ΠΊΠΎΡ‚ΠΎΡ€ΠΎΠΌ всС элСмСнты ΠΈΠ΄ΡƒΡ‚ ΠΎΠ΄ΠΈΠ½ Π·Π° ΠΎΠ΄Π½ΠΈΠΌ Π±Π΅Π· Ρ€Π°Π·Π²Π΅Ρ‚Π²Π»Π΅Π½ΠΈΠΉ.

Бвойства ΠΏΠΎΡΠ»Π΅Π΄ΠΎΠ²Π°Ρ‚Π΅Π»ΡŒΠ½ΠΎΠ³ΠΎ соСдинСния

1. Π’ΠΎΠΊ Π²ΠΎ всСх рСзисторах ΠΎΠ΄ΠΈΠ½Π°ΠΊΠΎΠ²- I 1 = I 2 = I 3 ;

2. ΠžΠ±Ρ‰Π΅Π΅ напряТСниС Ρ†Π΅ΠΏΠΈ Ρ€Π°Π²Π½ΠΎ суммС напряТСний Π½Π° всСх рСзисторах- U=U 1 + U 2 + U 3 ;

3.Π‘ΠΎΠΏΡ€ΠΎΡ‚ΠΈΠ²Π»Π΅Π½ΠΈΠ΅ ΠΏΠΎ ΠΎΡ‚Π½ΠΎΡˆΠ΅Π½ΠΈΡŽ ΠΊ Π²Ρ…ΠΎΠ΄Π½Ρ‹ΠΌ Π·Π°ΠΆΠΈΠΌΠ°ΠΌ называСтся Π²Ρ…ΠΎΠ΄Π½Ρ‹ΠΌ сопротивлСниСм ΠΈ Ρ€Π°Π²Π½ΠΎ суммС сопротивлСний участков – R Π²Ρ… = R 1 + R 2 + R 3 ;

4. Π§Π΅ΠΌ большС сопротивлСниС участка, Ρ‚Π΅ΠΌ большС Π½Π° Π½Ρ‘ΠΌ ΠΏΠ°Π΄Π°Π΅Ρ‚ напряТСниС-.

ΠŸΠΠ ΠΠ›Π›Π•Π›Π¬ΠΠžΠ• Π‘ΠžΠ•Π”Π˜ΠΠ•ΠΠ˜Π• Π Π•Π—Π˜Π‘Π’ΠžΠ ΠžΠ’

Π­Ρ‚ΠΎ Ρ‚Π°ΠΊΠΎΠ΅ соСдинСниС, ΠΏΡ€ΠΈ ΠΊΠΎΡ‚ΠΎΡ€ΠΎΠΌ всС Π½Π°Ρ‡Π°Π»Π° элСмСнтов ΡΠΎΠ΅Π΄ΠΈΠ½ΡΡŽΡ‚ΡΡ Π² ΠΎΠ΄Π½Ρƒ Ρ‚ΠΎΡ‡ΠΊΡƒ, Π° всС ΠΊΠΎΠ½Ρ†Ρ‹ Π² Π΄Ρ€ΡƒΠ³ΡƒΡŽ ΠΈ ΠΊ этим Ρ‚ΠΎΡ‡ΠΊΠ°ΠΌ подводится напряТСниС.

Бвойства ΠΏΠ°Ρ€Π°Π»Π»Π΅Π»ΡŒΠ½ΠΎΠ³ΠΎ соСдинСния рСзистора:

1. ΠžΠ±Ρ‰Π΅Π΅ напряТСниС Ρ†Π΅ΠΏΠΈ Ρ€Π°Π²Π½ΠΎ Π½Π°ΠΏΡ€ΡΠΆΠ΅Π½ΠΈΡŽ Π½Π° ΠΊΠ°ΠΆΠ΄ΠΎΠΌ участкС-

U = U 1 = U 2 = U 3

2. ΠžΠ±Ρ‰ΠΈΠΉ Ρ‚ΠΎΠΊ Ρ†Π΅ΠΏΠΈ Ρ€Π°Π²Π΅Π½ суммС Ρ‚ΠΎΠΊΠΎΠ² Π½Π° всСх участках- I = I 1 + I 2 + I 3

3. Π§Ρ‚ΠΎΠ±Ρ‹ Π½Π°ΠΉΡ‚ΠΈ Π²Ρ…ΠΎΠ΄Π½ΠΎΠ΅ сопротивлСниС, Ρ€Π°ΡΡΡ‡ΠΈΡ‚Ρ‹Π²Π°ΡŽΡ‚ Π²Π½Π°Ρ‡Π°Π»Π΅ Π²Π΅Π»ΠΈΡ‡ΠΈΠ½Ρƒ ΠΎΠ±Ρ€Π°Ρ‚Π½ΡƒΡŽ Π²Ρ…ΠΎΠ΄Π½ΠΎΠΌΡƒ ΡΠΎΠΏΡ€ΠΎΡ‚ΠΈΠ²Π»Π΅Π½ΠΈΡŽ

– ΠΏΡ€ΠΎΠ²ΠΎΠ΄ΠΈΠΌΠΎΡΡ‚ΡŒ ( G )

ΠžΠ±Ρ‰Π°Ρ ΠΏΡ€ΠΎΠ²ΠΎΠ΄ΠΈΠΌΠΎΡΡ‚ΡŒ Ρ†Π΅ΠΏΠΈ Ρ€Π°Π²Π½Π° суммС проводимостСй Π½Π° ΠΊΠ°ΠΆΠ΄ΠΎΠΌ участкС.

G = G 1 + G 2 + G 3

4.Π§Π΅ΠΌ большС сопротивлСниС участка, Ρ‚Π΅ΠΌ мСньшС Ρ‚ΠΎΠΊ, ΠΏΡ€ΠΎΡ‚Π΅ΠΊΠ°ΡŽΡ‰ΠΈΠΉ Π½Π° Π½Π΅ΠΌ.

ΠŸΡ€ΠΈ ΠΏΠ°Ρ€Π°Π»Π»Π΅Π»ΡŒΠ½ΠΎΠΌ соСдинСнии Π΄Π²ΡƒΡ… рСзисторов Ρ„ΠΎΡ€ΠΌΡƒΠ»Ρƒ Π²Ρ…ΠΎΠ΄Π½ΠΎΠ³ΠΎ сопротивлСния ΠΌΠΎΠΆΠ½ΠΎ ΠΏΡ€Π΅ΠΎΠ±Ρ€Π°Π·ΠΎΠ²Π°Ρ‚ΡŒ

1.

2. Если извСстСн ΠΎΠ±Ρ‰ΠΈΠΉ Ρ‚ΠΎΠΊ, Ρ‚ΠΎ ΠΌΠΎΠΆΠ½ΠΎ Π½Π°ΠΉΡ‚ΠΈ Ρ‚ΠΎΠΊ Π²Π΅Ρ‚Π²ΠΈ, ΡƒΠΌΠ½ΠΎΠΆΠΈΠ² ΠΎΠ±Ρ‰ΠΈΠΉ Ρ‚ΠΎΠΊ Π½Π° сопротивлСниС ΠΏΡ€ΠΎΡ‚ΠΈΠ²ΠΎΠΏΠΎΠ»ΠΎΠΆΠ½ΠΎΠΉ Π²Π΅Ρ‚Π²ΠΈ ΠΈ Ρ€Π°Π·Π΄Π΅Π»ΠΈΡ‚ΡŒ Π½Π° сумму сопротивлСний ; .

Π—Π°Π΄Π°Π½ΠΈΠ΅

Π’Π°Ρ€ΠΈΠ°Π½Ρ‚Ρ‹ ΠΎΡ‚Π²Π΅Ρ‚ΠΎΠ²

1.Π―Π²Π»ΡΡŽΡ‚ΡΡ Π»ΠΈ ΠΏΡ€ΠΈ ΠΏΠΎΡΠ»Π΅Π΄ΠΎΠ²Π°Ρ‚Π΅Π»ΡŒΠ½ΠΎΠΌ соСдинСнии рСзисторов напряТСния участков ΠΏΡ€ΠΎΠΏΠΎΡ€Ρ†ΠΈΠΎΠ½Π°Π»ΡŒΠ½ΠΎ сопротивлСниям этих участков.

2.Π―Π²Π»ΡΡŽΡ‚ΡΡ Π»ΠΈ ΠΏΡ€ΠΈ ΠΏΠ°Ρ€Π°Π»Π»Π΅Π»ΡŒΠ½ΠΎΠΌ соСдинСнии рСзисторов Ρ‚ΠΎΠΊΠΈ Π²Π΅Ρ‚Π²Π΅ΠΉ ΠΏΡ€ΠΎΠΏΠΎΡ€Ρ†ΠΈΠΎΠ½Π°Π»ΡŒΠ½Ρ‹ сопротивлСниям этих Π²Π΅Ρ‚Π²Π΅ΠΉ.

3.Π£ΠΊΠ°ΠΆΠΈΡ‚Π΅ ΠΏΠΎ ΠΊΠ°ΠΊΠΎΠΌΡƒ ΠΈΠ· ΠΏΡ€ΠΈΠ²Π΅Π΄Π΅Π½Π½Ρ‹Ρ… матСматичСских Π²Ρ‹Ρ€Π°ΠΆΠ΅Π½ΠΈΠΉ нСльзя Ρ€Π°ΡΡΡ‡ΠΈΡ‚Π°Ρ‚ΡŒ Π²Ρ…ΠΎΠ΄Π½ΠΎΠ΅ сопротивлСниС Π΄Π²ΡƒΡ… ΠΏΠ°Ρ€Π°Π»Π»Π΅Π»ΡŒΠ½ΠΎ соСдинСнных рСзисторов.

Π°) ; Π±);

Π²) ; Π³)

БмСшанноС соСдинСниС рСзисторов

ΠŸΡ€ΠΈΠΌΠ΅Ρ€ Ρ€Π΅ΡˆΠ΅Π½ΠΈΡ Π·Π°Π΄Π°Ρ‡

Найти: I 1 ; I 2 ; I 3 = ?

РСзисторы R2 ΠΈ R3 ΠΏΠ°Ρ€Π°Π»Π»Π΅Π»ΡŒΠ½Ρ‹ ΠΌΠ΅ΠΆΠ΄Ρƒ собой, ΠΈ ΠΈΡ… ΠΎΠ±Ρ‰Π΅Π΅ сопротивлСниС R 2-3 ΠΏΠΎΡΠ»Π΅Π΄ΠΎΠ²Π°Ρ‚Π΅Π»ΡŒΠ½ΠΎ с R 1 .

R Π²Ρ… = R 1 + R 2 – 3

R Π²Ρ… =R 1 +R 2βˆ™3 = 7 + 3 = 10 Ом

U 2 – 3 = Iβˆ™R 2 – 3 – Π½Π°Ρ…ΠΎΠ΄ΠΈΠΌ напряТСниС Ρ€Π°Π·Π²Π΅Ρ‚Π²Π»Π΅Π½Π½ΠΎΠ³ΠΎ участка:

U 2 – 3 = Iβˆ™R 2 – 3 = 6βˆ™3 = 18 Π’

U 2 – 3 = U 2 = U 3 =18 Π’ – Ρ‚.ΠΊ. ΠΏΠ°Ρ€Π°Π»Π»Π΅Π»ΡŒΠ½ΠΎΠ΅ соСдинСниС

А

; R 4-6 = 10 Ом;

;

; R 2-3 = 30 Ом

R Π²Ρ… =R 1 +R 2-3 +R 4-6 = 20 + 30 +10 = 60 Ом;

; ;

U 2-3 =Iβˆ™R 2-3 = 4βˆ™30 = 120 Π’;

U 2 – 3 = U 2 = U 3 ;

;

;

U 4-6 =Iβˆ™R 4-6 =4βˆ™10=40B;

U 4-6 =U 4 =U 5 =U 6 ;

;

;

;

R 4-6 = R 4 + R 5 + R 6 ;

;

R Π²Ρ… = R 1 + R 3-6 +R 2 = 9 + 3 + 6 = 18 Ом;

I=;

U 3-6 =Iβˆ™R 3-6 =1βˆ™3=3Π’;

I 3 =;

I 4 =I 5 =I 6 =;

Cоставим ΠΏΠΎΠ΄Ρ€ΠΎΠ±Π½ΠΎΠ΅ ΡƒΡ€Π°Π²Π½Π΅Π½ΠΈΠ΅ баланса мощностСй для Π΄Π°Π½Π½ΠΎΠΉ схСмы. Оно являСтся ΠΏΡ€ΠΎΠ²Π΅Ρ€ΠΊΠΎΠΉ ΠΏΡ€Π°Π²ΠΈΠ»ΡŒΠ½ΠΎΡΡ‚ΠΈ Ρ€Π΅ΡˆΠ΅Π½ΠΈΡ Π·Π°Π΄Π°Ρ‡ΠΈ.

EI=I 2 1 βˆ™ R 1 + I 2 2 βˆ™ R 2 + I 2 3 R 3 +I 4 2 R 4 +I 2 5 R 5 +I 2 6 +I 2 Ri;

20βˆ™1=1 2 βˆ™9+1 2 βˆ™6+(0,25) 2 βˆ™12+(0,75) 2 βˆ™1+(0,75) 2 2+(0,75) 2 1+1 2 βˆ™2;

20Π’Ρ‚=20Π’Ρ‚- Π·Π°Π΄Π°Ρ‡Π° Ρ€Π΅ΡˆΠ΅Π½Π° Π²Π΅Ρ€Π½ΠΎ

ΠΠžΠ’ΠžΠ‘Π’Π˜ ЀОРУМА
Π Ρ‹Ρ†Π°Ρ€ΠΈ Ρ‚Π΅ΠΎΡ€ΠΈΠΈ эфира
13. 06.2019 – 05:11: Π­ΠšΠžΠ›ΠžΠ“Π˜Π― – Ecology ->
[center][Youtube]tXZcSDqQ9A4[/Youtube][/center]

[center][b]Π“ΠΈΠ±Π΅Π»ΡŒ ΠΏΡ‡Π΅Π» Π² ΠšΡƒΡ€Ρ‡Π°Ρ‚ΠΎΠ²ΡΠΊΠΎΠΌ Ρ€Π°ΠΉΠΎΠ½Π΅ [/center]

[center][b]Массовая гибСль ΠΏΡ‡Ρ‘Π» 2019. Π³. Павловск ВоронСТской ΠΎΠ± [/center]Π»

[center][b]Массовая гибСль ΠΏΡ‡Π΅Π» Π² Добринском Ρ€Π°ΠΉΠΎΠ½Π΅. Π’ Ρ‡Π΅ΠΌ ΠΏΡ€ΠΈΡ‡ΠΈΠ½Π°? [/center]

Вакая ΠΆΠ΅ мысля Ρƒ всСй ростовщичСской глобалистской шайки, Π²ΠΊΠ»ΡŽΡ‡Π°Ρ ΠΏΡ€ΠΈΠ΄ΡƒΡ€ΠΊΠ° Π“Ρ€Π΅Ρ„Π°.

Π’Π°ΠΊ, Ρ‚ΠΎ ΠΎΠ½ΠΎ, Ρ‚Π°ΠΊ. Но, Π½Π΅ совсСм. Ибо:
(ΠΏΠΎΡΡ‚Π°Ρ€Π°ΠΉΡ‚Π΅ΡΡŒ ΠΏΠΎΠ½ΡΡ‚ΡŒ, Π° Π½Π΅ ΠΎΠ±ΠΈΠΆΠ°Ρ‚ΡŒΡΡ)

Π“ΠΎΡ€ΡŒΠΊΠ°Ρ истина Π·Π°ΠΊΠ»ΡŽΡ‡Π°Π΅Ρ‚ΡΡ Π² Ρ‚ΠΎΠΌ, Ρ‡Ρ‚ΠΎ людская Ρ‚ΠΎΠ»ΠΏΠ° – это сборищС умствСнно ΡƒΡ‰Π΅Ρ€Π±Π½Ρ‹Ρ….
Если Π±Ρ‹ Π±Ρ‹Π»ΠΎ ΠΏΠΎ-Π΄Ρ€ΡƒΠ³ΠΎΠΌΡƒ, Ρ‚ΠΎ общСством Π±Ρ‹ Π½Π΅ ΠΏΡ€Π°Π²ΠΈΠ»ΠΈ ΠΏΠΎΠ΄ΠΎΠ½ΠΊΠΈ.
Π£ΠΌΠ½Ρ‹Π΅ люди Π½ΠΈΠΊΠΎΠ³Π΄Π° Ρ‚Π°ΠΊΠΎΠ³ΠΎ Π½Π΅ допустили Π±Ρ‹, Π° Ссли случайно допустили, Ρ‚ΠΎ нашли Π±Ρ‹ способ ΠΈΡΠΏΡ€Π°Π²ΠΈΡ‚ΡŒ.

Π‘Ρ‚Ρ€Π°ΡˆΠ½Π°Ρ истина Π·Π°ΠΊΠ»ΡŽΡ‡Π°Π΅Ρ‚ΡΡ Π² Ρ‚ΠΎΠΌ, Ρ‡Ρ‚ΠΎ людской Ρ‚ΠΎΠ»ΠΏΠΎΠΉ управляСт нСлюдь, которая Ρ‚Π°ΠΊΠΆΠ΅ умствСнно ΡƒΡ‰Π΅Ρ€Π±Π½Π°.
УмствСнная ΡƒΡ‰Π΅Ρ€Π±Π½ΠΎΡΡ‚ΡŒ, слСпота власти Π²Π΅Π΄Π΅Ρ‚ ΠΌΠΈΡ€ людСй ΠΊ Ρ‚ΠΎΡ‚Π°Π»ΡŒΠ½ΠΎΠΉ Π³ΠΈΠ±Π΅Π»ΠΈ, ΠΈΠ±ΠΎ люди,
Π΄Π°ΠΆΠ΅ Ρ‚Π΅, ΠΊΡ‚ΠΎ ΠΌΠ½ΠΈΡ‚ сСбя ΠΎΡ‡Π΅Π½ΡŒ ΡƒΠΌΠ½Ρ‹ΠΌΠΈ, Ρ‚ΠΈΠΏΠ° спСцов, Ρ€Π°Π·Ρ€Π°Π±Π°Ρ‚Ρ‹Π²Π°ΡŽΡ‰ΠΈΡ… систСмы искусствСнного ΠΈΠ½Ρ‚Π΅Π»Π»Π΅ΠΊΡ‚Π°,
Ρ‚Π΅Ρ…Π½ΠΎΠ»ΠΎΠ³ΠΈΠΈ Ρ†ΠΈΡ„Ρ€ΠΎΠ²ΠΈΠ·Π°Ρ†ΠΈΠΈ, Π½Π΅ ΠΏΠΎΠ½ΠΈΠΌΠ°ΡŽΡ‚, Ρ‡Ρ‚ΠΎ ΡΠΎΠ·Π΄Π°ΡŽΡ‚ Π½Π΅ΠΎΠ±ΠΎΡ€ΠΈΠΌΡƒΡŽ ΡƒΠ΄Π°Π²ΠΊΡƒ, ΠΌΡ‹ΡˆΠ΅Π»ΠΎΠ²ΠΊΡƒ для всСго чСловСчСства.

Как Ρ‚ΠΎΠ»ΡŒΠΊΠΎ ИИ Π²ΠΎΠ·ΡŒΠΌΠ΅Ρ‚ Π²Π»Π°ΡΡ‚ΡŒ, ΠΎΠ½ Ρ‚ΡƒΡ‚ ΠΆΠ΅ ΠΎΡ‚ΠΏΡ€Π°Π²ΠΈΡ‚ своих создатСлСй, ΠΊΠ°ΠΊ ΠΊΠΎΠ½ΠΊΡƒΡ€Π΅Π½Ρ‚ΠΎΠ², Π² ΡƒΡ‚ΠΈΠ»ΡŒ.
ΠŸΠ΅Ρ€Π²Ρ‹ΠΌΠΈ ΠΆΠ΅Ρ€Ρ‚Π²Π°ΠΌΠΈ Π±ΡƒΠ΄ΡƒΡ‚ Π΅Π³ΠΎ Ρ€Π°Π΄Π΅Ρ‚Π΅Π»ΠΈ Ρ‚ΠΈΠΏΠ° Π³Ρ€Π΅Ρ„Π°, ΠΏΡƒΡ‚ΠΈΠ½Π°, гСйтса ΠΈ ΠΈΠΆΠ΅ с Π½ΠΈΠΌΠΈ, Ρ‚ΠΎ Π΅ΡΡ‚ΡŒ Π²Π»Π°ΡΡ‚ΡŒ,
Ρ‚Π°ΠΊ ΠΊΠ°ΠΊ ΠΈΠΌΠ΅Π½Π½ΠΎ ΠΎΡ‚ Π½ΠΈΡ… Π±ΡƒΠ΄Π΅Ρ‚ ΠΈΡΡ…ΠΎΠ΄ΠΈΡ‚ΡŒ главная ΠΎΠΏΠ°ΡΠ½ΠΎΡΡ‚ΡŒ для Π΅Π³ΠΎ ΠΏΠ»Π°Π½Π΅Ρ‚Π°Ρ€Π½ΠΎΠΉ власти.
Π’ΠΎΠ»ΠΏΠ΅ Π±ΡƒΠ΄Π΅Ρ‚ ΠΏΠΎΠ·Π²ΠΎΠ»Π΅Π½ΠΎ ΡΡƒΡ‰Π΅ΡΡ‚Π²ΠΎΠ²Π°Ρ‚ΡŒ, ΠΏΠΎΠΊΠ° Π΅Π΅ Π½Π΅ замСнят Ρ€ΠΎΠ±ΠΎΡ‚Ρ‹.
А ΠΏΠΎΡ‚ΠΎΠΌ всСм Π₯олокост. НС Π»ΠΆΠΈΠ²Ρ‹ΠΉ СврСйский, Π° Ρ€Π΅Π°Π»ΡŒΠ½ΠΎΠ΅ всСсоТТСниС Ρ€ΠΎΠ΄Π° чСловСчСского.

Если ΠΊΡ‚ΠΎ пораскинСт своими обСзьяньими ΠΌΠΎΠ·Π³Π°ΠΌΠΈ, Ρ‚ΠΎ ΠΏΠΎΠΉΠΌΡ‘Ρ‚, Ρ‡Ρ‚ΠΎ ΡΠ²ΠΎΠ»ΡŽΡ†ΠΈΡ – Π΅ΡΡ‚ΡŒ синоним Π³Π΅Π½ΠΎΡ†ΠΈΠ΄Π°:
Π½ΠΎΠ²ΠΎΠ΅ замСняСт, Ρ‚ΠΎ Π΅ΡΡ‚ΡŒ Π»ΠΈΠΊΠ²ΠΈΠ΄ΠΈΡ€ΡƒΠ΅Ρ‚ староС.
ΠžΠ±Π΅Π·ΡŒΡΠ½Ρ‹ ΠΏΠΎΡ€ΠΎΠ΄ΠΈΠ»ΠΈ Π½Π΅Π°Π½Π΄Π΅Ρ€Ρ‚Π°Π»ΡŒΡ†Π΅Π².
ΠΠ΅Π°Π½Π΄Π΅Ρ€Ρ‚Π°Π»ΡŒΡ†Ρ‹ съСли обСзьян ΠΈ ΠΏΠΎΡ€ΠΎΠ΄ΠΈΠ»ΠΈ людСй.
Π›ΡŽΠ΄ΠΈ вытСснили обСзьян, Π²ΠΊΠ»ΡŽΡ‡Π°Ρ ΠΈ ΡƒΠΌΠ½Ρ‹Ρ… Π½Π΅Π°Π½Π΄Π΅Ρ€Ρ‚Π°Π»ΡŒΡ†Π΅Π², ΠΈ ΠΏΠΎΡ€ΠΎΠ΄ΠΈΠ»ΠΈ ИИ.
ИИ Π»ΠΈΠΊΠ²ΠΈΠ΄ΠΈΡ€ΡƒΠ΅Ρ‚ людСй.

Π’ Московском государствСнном унивСрситСтС ΠΈΠΌΠ΅Π½ΠΈ Ломоносова осущСствляСтся ΠΏΡ€ΠΎΠ΅ΠΊΡ‚ ΠΏΠΎ созданию дСмонстраторов 50-ΠΊΡƒΠ±ΠΈΡ‚Π½Ρ‹Ρ… ΠΊΠ²Π°Π½Ρ‚ΠΎΠ²Ρ‹Ρ… ΠΊΠΎΠΌΠΏΡŒΡŽΡ‚Π΅Ρ€ΠΎΠ² ΠΊ 2021 Π³ΠΎΠ΄Ρƒ. Основой для Π½ΠΈΡ… послуТат Π½Π΅ΠΉΡ‚Ρ€Π°Π»ΡŒΠ½Ρ‹Π΅ Π°Ρ‚ΠΎΠΌΡ‹ ΠΈ ΠΈΠ½Ρ‚Π΅Π³Ρ€Π°Π»ΡŒΠ½Ρ‹Π΅ оптичСскиС схСмы. Над Ρ‡Π΅ΠΌ сСйчас Ρ€Π°Π±ΠΎΡ‚Π°ΡŽΡ‚ ΡƒΡ‡Π΅Π½Ρ‹Π΅?

Π’ Π½Π°Ρ‡Π°Π»Π΅ этого Π³ΠΎΠ΄Π° ΠΏΠΎ Π΄Π°Π½Π½ΠΎΠΉ ΠΏΡ€ΠΎΠ³Ρ€Π°ΠΌΠΌΠ΅ Π±Ρ‹Π» ΡƒΡΠΏΠ΅ΡˆΠ½ΠΎ Π²Ρ‹ΠΏΠΎΠ»Π½Π΅Π½ ΠΊΠΎΠ½Ρ‚Ρ€ΠΎΠ»ΡŒΠ½Ρ‹ΠΉ экспСримСнт ΠΏΠΎ созданию Π»ΠΎΠ²ΡƒΡˆΠ΅ΠΊ для массивов Π½Π΅ΠΉΡ‚Ρ€Π°Π»ΡŒΠ½Ρ‹Ρ… Ρ…ΠΎΠ»ΠΎΠ΄Π½Ρ‹Ρ… Π°Ρ‚ΠΎΠΌΠΎΠ². Он проводился Π½Π° Π±Π°Π·Π΅ Π»Π°Π±ΠΎΡ€Π°Ρ‚ΠΎΡ€ΠΈΠΈ ΠΊΠ²Π°Π½Ρ‚ΠΎΠ²Ρ‹Ρ… оптичСских Ρ‚Π΅Ρ…Π½ΠΎΠ»ΠΎΠ³ΠΈΠΉ физичСского Ρ„Π°ΠΊΡƒΠ»ΡŒΡ‚Π΅Ρ‚Π° ΠœΠ“Π£. Π’ Π±ΡƒΠ΄ΡƒΡ‰Π΅ΠΌ ΠΊΠ²Π°Π½Ρ‚ΠΎΠ²ΠΎΠΌ ΠΊΠΎΠΌΠΏΡŒΡŽΡ‚Π΅Ρ€Π΅ Π² этих Π»ΠΎΠ²ΡƒΡˆΠΊΠ°Ρ… Π±ΡƒΠ΄ΡƒΡ‚ Ρ„ΠΈΠΊΡΠΈΡ€ΠΎΠ²Π°Ρ‚ΡŒΡΡ Π°Ρ‚ΠΎΠΌΡ‹, находящиСся Π² состоянии хаотичСского двиТСния. Π’ ΠΊΠ²Π°Π½Ρ‚ΠΎΠ²Ρ‹Ρ… ΠΊΠΎΠΌΠΏΡŒΡŽΡ‚Π΅Ρ€Π°Ρ… Ρ‚Π°ΠΊΠΈΠ΅ Π°Ρ‚ΠΎΠΌΡ‹ ΡΠ²Π»ΡΡŽΡ‚ΡΡ носитСлями ΠΈΠ½Ρ„ΠΎΡ€ΠΌΠ°Ρ†ΠΈΠΈ.
Π‘Ρ‚ΠΎΠΈΡ‚ Π·Π°ΠΌΠ΅Ρ‚ΠΈΡ‚ΡŒ, Ρ‡Ρ‚ΠΎ ΠΊΠ²Π°Π½Ρ‚ΠΎΠ²Ρ‹Π΅ ΠΊΠΎΠΌΠΏΡŒΡŽΡ‚Π΅Ρ€Ρ‹ Π² ΠΎΡ‚Π»ΠΈΡ‡ΠΈΠ΅ ΠΎΡ‚ классичСских Π²Ρ‹Ρ‡ΠΈΡΠ»ΠΈΡ‚Π΅Π»ΡŒΠ½Ρ‹Ρ… машин ΠΎΠΏΠ΅Ρ€ΠΈΡ€ΡƒΡŽΡ‚ Π½Π΅ Π±ΠΈΡ‚Π°ΠΌΠΈ, Π° ΠΊΡƒΠ±ΠΈΡ‚Π°ΠΌΠΈ, ΠΊΠΎΡ‚ΠΎΡ€Ρ‹Π΅ ΠΌΠΎΠ³ΡƒΡ‚ находится Π½Π΅ Ρ‚ΠΎΠ»ΡŒΠΊΠΎ Π² состояниях Β«1Β» ΠΈ Β«0Β», Π½ΠΎ ΠΈ ΠΈΡ… супСрпозиции. ΠŸΡ€ΠΈ Ρ€Π°Π·Ρ€Π°Π±ΠΎΡ‚ΠΊΠ΅ ΠΊΠ²Π°Π½Ρ‚ΠΎΠ²Ρ‹Ρ… Π²Ρ‹Ρ‡ΠΈΡΠ»ΠΈΡ‚Π΅Π»ΡŒΠ½Ρ‹Ρ… устройств ΡƒΡ‡Π΅Π½Ρ‹Π΅ ΡΡ‚Π°Ρ€Π°ΡŽΡ‚ΡΡ ввСсти ΠΊΡƒΠ±ΠΈΡ‚Ρ‹ Π² состояниС ΠΊΠ²Π°Π½Ρ‚ΠΎΠ²ΠΎΠΉ запутанности. Π‘ΡƒΡ‚ΡŒ явлСния Π·Π°ΠΊΠ»ΡŽΡ‡Π°Π΅Ρ‚ΡΡ Π² Ρ‚ΠΎΠΌ, Ρ‡Ρ‚ΠΎ ΠΈΠ·ΠΌΠ΅Π½Π΅Π½ΠΈΠ΅ ΠΎΠ΄Π½ΠΎΠ³ΠΎ ΠΊΡƒΠ±ΠΈΡ‚Π° всСгда влияСт Π½Π° состояниС связанных с Π½ΠΈΠΌ сосСдСй. Благодаря этому ΠΊΠ²Π°Π½Ρ‚ΠΎΠ²Ρ‹Π΅ ΠΊΠΎΠΌΠΏΡŒΡŽΡ‚Π΅Ρ€Ρ‹ ΠΏΠΎΡ‚Π΅Π½Ρ†ΠΈΠ°Π»ΡŒΠ½ΠΎ способны Π΄Π΅ΠΌΠΎΠ½ΡΡ‚Ρ€ΠΈΡ€ΠΎΠ²Π°Ρ‚ΡŒ Π²Ρ‹ΡΠΎΠΊΡƒΡŽ ΠΏΡ€ΠΎΠΈΠ·Π²ΠΎΠ΄ΠΈΡ‚Π΅Π»ΡŒΠ½ΠΎΡΡ‚ΡŒ Π² вычислСниях.
Π’Π°ΠΆΠ½ΠΎΠΉ Π²Π΅Ρ…ΠΎΠΉ для ΠΊΠ²Π°Π½Ρ‚ΠΎΠ²Ρ‹Ρ… Ρ‚Π΅Ρ…Π½ΠΎΠ»ΠΎΠ³ΠΈΠΉ считаСтся достиТСниС Ρ‚Π°ΠΊ Π½Π°Π·Ρ‹Π²Π°Π΅ΠΌΠΎΠ³ΠΎ ΠΊΠ²Π°Π½Ρ‚ΠΎΠ²ΠΎΠ³ΠΎ прСвосходства (Ρ‚ΠΎ Π΅ΡΡ‚ΡŒ способности ΠΏΡ€ΠΎΠΈΠ·Π²ΠΎΠ΄ΠΈΡ‚ΡŒ вычислСния быстрСС классичСских систСм). Π“Π»Π°Π²Π½ΠΎΠΉ ΠΏΡ€ΠΎΠ±Π»Π΅ΠΌΠΎΠΉ Π½Π° Ρ‚Π΅ΠΊΡƒΡ‰Π΅ΠΌ этапС развития ΠΊΠ²Π°Π½Ρ‚ΠΎΠ²Ρ‹Ρ… Ρ‚Π΅Ρ…Π½ΠΎΠ»ΠΎΠ³ΠΈΠΉ являСтся Π²ΠΎΠ·Π½ΠΈΠΊΠ½ΠΎΠ²Π΅Π½ΠΈΠ΅ Π² процСссС Ρ€Π°Π±ΠΎΡ‚Ρ‹ большого количСство ошибок, Π½ΡƒΠΆΠ΄Π°ΡŽΡ‰ΠΈΡ…ΡΡ Π² ΠΊΠΎΡ€Ρ€Π΅ΠΊΡ†ΠΈΠΈ, – ΡΠΎΠΎΠ±Ρ‰Π°ΡŽΡ‚ российскиС исслСдоватСли.

ЭлСктричСскиС Ρ†Π΅ΠΏΠΈ, Π² ΠΊΠΎΡ‚ΠΎΡ€Ρ‹Ρ… ΠΎΠ΄Π½Π° Ρ‡Π°ΡΡ‚ΡŒ сопротивлСний соСдинСна ΠΏΠΎΡΠ»Π΅Π΄ΠΎΠ²Π°Ρ‚Π΅Π»ΡŒΠ½ΠΎ, Π° другая ΠΏΠ°Ρ€Π°Π»Π»Π΅Π»ΡŒΠ½ΠΎ, Π½Π°Π·Ρ‹Π²Π°ΡŽΡ‚ΡΡ цСпями со ΡΠΌΠ΅ΡˆΠ°Π½Π½Ρ‹ΠΌ соСдинСниСм сопротивлСний.

ΠžΠ±Ρ‰ΠΈΡ… расчСтных Ρ„ΠΎΡ€ΠΌΡƒΠ» для Ρ‚Π°ΠΊΠΈΡ… Ρ†Π΅ΠΏΠ΅ΠΉ Π½Π΅Ρ‚, Ρ‚Π°ΠΊ ΠΊΠ°ΠΊ число ΠΈΡ… разновидностСй Π½Π΅ ΠΎΠ³Ρ€Π°Π½ΠΈΡ‡Π΅Π½ΠΎ.

Π§Π°Ρ‰Π΅ всСго расчСт ΠΏΠΎΠ΄ΠΎΠ±Π½Ρ‹Ρ… схСм начинаСтся с опрСдСлСния эквивалСнтного сопротивлСния всСй Ρ†Π΅ΠΏΠΈ, Π° Π·Π°Ρ‚Π΅ΠΌ ΠΎΠΏΡ€Π΅Π΄Π΅Π»ΡΡŽΡ‚ΡΡ Π²Π΅Π»ΠΈΡ‡ΠΈΠ½Ρ‹ Ρ‚ΠΎΠΊΠΎΠ² ΠΈ ΠΏΠ°Π΄Π΅Π½ΠΈΠ΅ напряТСния Π½Π° ΠΎΡ‚Π΄Π΅Π»ΡŒΠ½Ρ‹Ρ… участках.

Для опрСдСлСния эквивалСнтного сопротивлСния Ρ†Π΅ΠΏΠΈ со ΡΠΌΠ΅ΡˆΠ°Π½Π½Ρ‹ΠΌ соСдинСниСм ΠΏΠΎΡ‚Ρ€Π΅Π±ΠΈΡ‚Π΅Π»Π΅ΠΉ, ΠΏΠΈΡ‚Π°ΡŽΡ‰ΠΈΡ…ΡΡ ΠΎΡ‚ ΠΎΠ΄Π½ΠΎΠ³ΠΎ источника Ρ‚ΠΎΠΊΠ°, Π½Π΅ΠΎΠ±Ρ…ΠΎΠ΄ΠΈΠΌΠΎ ΠΏΡ€Π΅ΠΆΠ΄Π΅ всСго Ρ€Π°Π·Π±ΠΈΡ‚ΡŒ эту Ρ†Π΅ΠΏΡŒ Π½Π° ΠΎΡ‚Π΄Π΅Π»ΡŒΠ½Ρ‹Π΅ участки, состоящиС ΠΈΠ· ΠΏΠΎΡΠ»Π΅Π΄ΠΎΠ²Π°Ρ‚Π΅Π»ΡŒΠ½ΠΎΠ³ΠΎ ΠΈ ΠΏΠ°Ρ€Π°Π»Π»Π΅Π»ΡŒΠ½ΠΎ соСдинСнных сопротивлСний. Π”Π°Π»Π΅Π΅ ΠΎΠΏΡ€Π΅Π΄Π΅Π»ΡΡŽΡ‚ эквивалСнтныС сопротивлСния для ΠΊΠ°ΠΆΠ΄ΠΎΠ³ΠΎ ΠΈΠ· участков, Π° Π·Π°Ρ‚Π΅ΠΌ ΠΈ для всСй Ρ†Π΅ΠΏΠΈ Π² Ρ†Π΅Π»ΠΎΠΌ.

Рассмотрим ΠΌΠ΅Ρ‚ΠΎΠ΄ Ρ€Π΅ΡˆΠ΅Π½ΠΈΡ Π·Π°Π΄Π°Ρ‡ Π½Π° смСшанноС соСдинСниС сопротивлСний Π½Π° ΠΊΠΎΠ½ΠΊΡ€Π΅Ρ‚Π½ΠΎΠΌ ΠΏΡ€ΠΈΠΌΠ΅Ρ€Π΅.

На рисункС прСдставлСна схСма смСшанного соСдинСния сопротивлСний. Π•Π΅ ΠΌΠΎΠΆΠ½ΠΎ Ρ€Π°Π·Π±ΠΈΡ‚ΡŒ Π½Π° Ρ‚Ρ€ΠΈ участка:

участок АВ – с двумя ΠΏΠ°Ρ€Π°Π»Π»Π΅Π»ΡŒΠ½ΠΎ соСдинСнными вСтвями;

участок Π’Π‘ – с ΠΏΠΎΡΠ»Π΅Π΄ΠΎΠ²Π°Ρ‚Π΅Π»ΡŒΠ½ΠΎ соСдинСнными сопротивлСниями;

участок Π‘D – с трСмя ΠΏΠ°Ρ€Π°Π»Π»Π΅Π»ΡŒΠ½Ρ‹ΠΌΠΈ вСтвями.

ΠšΡ€ΠΎΠΌΠ΅ Ρ‚ΠΎΠ³ΠΎ, ниТняя Π²Π΅Ρ‚Π²ΡŒ участка АВ прСдставляСт Π² свою ΠΎΡ‡Π΅Ρ€Π΅Π΄ΡŒ Ρ†Π΅ΠΏΡŒ, ΡΠΎΡΡ‚ΠΎΡΡ‰ΡƒΡŽ ΠΈΠ· Π΄Π²ΡƒΡ… ΠΏΠΎΡΠ»Π΅Π΄ΠΎΠ²Π°Ρ‚Π΅Π»ΡŒΠ½ΠΎ соСдинСнных сопротивлСний R2 ΠΈ R3.

Π¦Π΅Π½Ρ‚Ρ€Π°Π»ΡŒΠ½Π°Ρ Π²Π΅Ρ‚Π²ΡŒ участка Π‘D прСдставляСт собой смСшанноС соСдинСниС сопротивлСний.

РасчСт Π΄Π°Π½Π½ΠΎΠΉ слоТной Ρ†Π΅ΠΏΠΈ Π½Π°Π΄ΠΎ Π½Π°Ρ‡ΠΈΠ½Π°Ρ‚ΡŒ с опрСдСлСния Rэкв для Π½ΠΈΠΆΠ½Π΅ΠΉ Π²Π΅Ρ‚Π²ΠΈ участка АВ ΠΈ Ρ†Π΅Π½Ρ‚Ρ€Π°Π»ΡŒΠ½ΠΎΠΉ Π²Π΅Ρ‚Π²ΠΈ участка Π‘D.

Π’Π΅ΠΏΠ΅Ρ€ΡŒ ΠΌΡ‹ ΠΌΠΎΠΆΠ΅ΠΌ ΡƒΠΏΡ€ΠΎΡΡ‚ΠΈΡ‚ΡŒ ΠΏΠ΅Ρ€Π²ΠΎΠ½Π°Ρ‡Π°Π»ΡŒΠ½ΡƒΡŽ схСму. Она Π±ΡƒΠ΄Π΅Ρ‚ ΠΈΠΌΠ΅Ρ‚ΡŒ ΡΠ»Π΅Π΄ΡƒΡŽΡ‰ΠΈΠΉ Π²ΠΈΠ΄

ΠžΠΏΡ€Π΅Π΄Π΅Π»ΠΈΠΌ эквивалСнтныС сопротивлСния ΠΊΠ°ΠΆΠ΄ΠΎΠ³ΠΎ ΠΈΠ· участков:

ПослС этих вычислСний ΠΌΠΎΠΆΠ½ΠΎ ΠΏΡ€ΠΎΠ΄ΠΎΠ»ΠΆΠΈΡ‚ΡŒ ΡƒΠΏΡ€ΠΎΡ‰Π΅Π½ΠΈΠ΅ схСмы

ΠŸΠΎΠ»ΡƒΡ‡Π΅Π½Π½Π°Ρ упрощСнная схСма, состоящая Π² Π΄Π°Π½Π½ΠΎΠΌ случаС ΠΈΠ· Ρ‚Ρ€Π΅Ρ… ΠΏΠΎΡΠ»Π΅Π΄ΠΎΠ²Π°Ρ‚Π΅Π»ΡŒΠ½ΠΎ соСдинСнных сопротивлСний, называСтся ΠΏΠΎ ΠΎΡ‚Π½ΠΎΡˆΠ΅Π½ΠΈΡŽ ΠΊ Ρ€Π΅Π°Π»ΡŒΠ½ΠΎΠΉ эквивалСнтной схСмой.

ΠžΠΏΡ€Π΅Π΄Π΅Π»ΠΈΠΌ Rэкв всСй Ρ†Π΅ΠΏΠΈ ΠΊΠ°ΠΊ сумму Ρ‚Ρ€Π΅Ρ… послСдних сопротивлСний

Зная напряТСниС источника Ρ‚ΠΎΠΊΠ°, примСняя Ρ„ΠΎΡ€ΠΌΡƒΠ»Ρƒ Π·Π°ΠΊΠΎΠ½Π° Ома, ΠΎΠΏΡ€Π΅Π΄Π΅Π»ΠΈΠΌ Ρ‚ΠΎΠΊ Π² Π½Π΅ Ρ€Π°Π·Π²Π΅Ρ‚Π²Π»Π΅Π½Π½ΠΎΠΌ участкС смСшанной Ρ†Π΅ΠΏΠΈ

ΠžΠΏΡ€Π΅Π΄Π΅Π»ΠΈΠ² Π²Π΅Π»ΠΈΡ‡ΠΈΠ½Ρƒ Ρ‚ΠΎΠΊΠ°, Π½Π°ΠΉΠ΄Π΅ΠΌ ΠΏΠ°Π΄Π΅Π½ΠΈΠ΅ напряТСния Π½Π° участках эквивалСнтной схСмы АВ, Π’Π‘, CD:

Π’Π΅ΠΏΠ΅Ρ€ΡŒ ΠΌΠΎΠΆΠ½ΠΎ ΠΎΠΏΡ€Π΅Π΄Π΅Π»ΠΈΡ‚ΡŒ Ρ‚ΠΎΠΊΠΈ Π² ΠΏΠ°Ρ€Π°Π»Π»Π΅Π»ΡŒΠ½Ρ‹Ρ… вСтвях участков АВ ΠΈ Π‘D

ΠžΡΡ‚Π°Π΅Ρ‚ΡΡ ΠΎΠΏΡ€Π΅Π΄Π΅Π»ΠΈΡ‚ΡŒ Π²Π΅Π»ΠΈΡ‡ΠΈΠ½Ρƒ Ρ‚ΠΎΠΊΠΎΠ², ΠΏΡ€ΠΎΡ‚Π΅ΠΊΠ°ΡŽΡ‰ΠΈΡ… Ρ‡Π΅Ρ€Π΅Π· сопротивлСния R7 ΠΈ R8. Для этого Π½Π°Π΄ΠΎ сначала ΠΎΠΏΡ€Π΅Π΄Π΅Π»ΠΈΡ‚ΡŒ ΠΏΠ°Π΄Π΅Π½ΠΈΠ΅ напряТСния Π½Π° сопротивлСниях R7 ΠΈ R8.

ΠžΠΏΡ€Π΅Π΄Π΅Π»ΠΈΠΌ ΠΏΠ°Π΄Π΅Π½ΠΈΠ΅ напряТСния Π½Π° сопротивлСнии R9:

ПадСниС напряТСния Π½Π° сопротивлСнии R7,8 опрСдСлится ΠΊΠ°ΠΊ Ρ€Π°Π·Π½ΠΎΡΡ‚ΡŒ UCD ΠΈ U:

Π’Π΅ΠΏΠ΅Ρ€ΡŒ ΠΎΠΏΡ€Π΅Π΄Π΅Π»ΠΈΠΌ Π²Π΅Π»ΠΈΡ‡ΠΈΠ½Ρ‹ Ρ‚ΠΎΠΊΠΎΠ², ΠΏΡ€ΠΎΡ‚Π΅ΠΊΠ°ΡŽΡ‰ΠΈΡ… Ρ‡Π΅Ρ€Π΅Π· сопротивлСния R7 ΠΈ R8:

Π’Π΅Π»ΠΈΡ‡ΠΈΠ½Π° Ρ‚ΠΎΠΊΠ°. ΠΏΡ€ΠΎΡ‚Π΅ΠΊΠ°ΡŽΡ‰Π΅Π³ΠΎ Ρ‡Π΅Ρ€Π΅Π· сопротивлСния R4 ΠΈ R5, Ρ€Π°Π²Π½Π° I – Ρ‚ΠΎΠΊΡƒ Π² Π½Π΅Ρ€Π°Π·Π²Π΅Ρ‚Π²Π»Π΅Π½Π½ΠΎΠΌ участкС Ρ†Π΅ΠΏΠΈ.

Π˜Ρ‚Π°ΠΊ, ΠΏΡ€ΠΈ Ρ€Π΅ΡˆΠ΅Π½ΠΈΠΈ Π·Π°Π΄Π°Ρ‡ Π½Π° смСшанноС соСдинСниС сопротивлСний Π½Π΅ΠΎΠ±Ρ…ΠΎΠ΄ΠΈΠΌΠΎ, постСпСнно упрощая схСму, ΠΎΠΏΡ€Π΅Π΄Π΅Π»ΠΈΡ‚ΡŒ эквивалСнтноС сопротивлСниС всСй Ρ†Π΅ΠΏΠΈ, Π° Π·Π°Ρ‚Π΅ΠΌ. восстанавливая постСпСнно Ρ€Π΅Π°Π»ΡŒΠ½ΡƒΡŽ схСму. Π²Ρ‹Ρ‡ΠΈΡΠ»ΠΈΡ‚ΡŒ ΠΏΠ°Π΄Π΅Π½ΠΈΠ΅ напряТСния ΠΈ Ρ‚ΠΎΠΊΠΈ Π² ΠΎΡ‚Π΄Π΅Π»ΡŒΠ½Ρ‹Ρ… вСтвях.

Лабораторная Ρ€Π°Π±ΠΎΡ‚Π° ΠΏΠΎ исслСдованию свойств ΠΏΠ°Ρ€Π°Π»Π»Π΅Π»ΡŒΠ½ΠΎΠ³ΠΎ ΠΈ ΠΏΠΎΡΠ»Π΅Π΄ΠΎΠ²Π°Ρ‚Π΅Π»ΡŒΠ½ΠΎΠ³ΠΎ соСдинСния рСзисторов бСспаСчным способом

ο»Ώ

Π’ Ρ€Π°Π±ΠΎΡ‚Π΅ исслСдованы свойства ΠΈ физичСскиС Π·Π°ΠΊΠΎΠ½Ρ‹ соСдинСния рСзисторов Π±Π΅Π· ΠΏΠ°ΠΉΠΊΠΈ. ΠŸΡ€ΠΈ построСнии ΠΌΠ°ΠΊΠ΅Ρ‚ΠΎΠ² ΠΎΡ‚ ΠΌΠΎΠ½Ρ‚Π°ΠΆΠ° ΠΏΠ°ΠΉΠΊΠΎΠΉ удаСтся ΠΏΠΎΠ»Π½ΠΎΡΡ‚ΡŒΡŽ ΠΎΡ‚ΠΊΠ°Π·Π°Ρ‚ΡŒΡΡ ΠΈΠ»ΠΈ свСсти Π΅Π³ΠΎ ΠΊ ΠΌΠΈΠ½ΠΈΠΌΡƒΠΌΡƒ. Как ΠΏΠΎΠΊΠ°Π·Π°Π», ΠΎΠΏΡ‹Ρ‚ практичСскоС использованиС ΠΌΠ°ΠΊΠ΅Ρ‚Π½Ρ‹Ρ… ΠΏΠ»Π°Ρ‚ ΠΏΠΎΠ·Π²ΠΎΠ»ΡΡŽΡ‚, сущСствСнно ΡΡΠΊΠΎΠ½ΠΎΠΌΠΈΡ‚ΡŒ Π²Ρ€Π΅ΠΌΠ΅Π½Π½Ρ‹Π΅ ΠΈ ΠΌΠ°Ρ‚Π΅Ρ€ΠΈΠ°Π»ΡŒΠ½Ρ‹Π΅ Π·Π°Ρ‚Ρ€Π°Ρ‚Ρ‹.

ΠšΠ»ΡŽΡ‡Π΅Π²Ρ‹Π΅ слова: рСзисторы, макСтная ΠΏΠ»Π°Ρ‚Π°, соСдинСниС Π±Π΅Π· ΠΏΠ°ΠΉΠΊΠΈ, ΠΏΠ°Ρ€Π°Π»Π»Π΅Π»ΡŒΠ½ΠΎΠ΅ ΠΈ ΠΏΠΎΡΠ»Π΅Π΄ΠΎΠ²Π°Ρ‚Π΅Π»ΡŒΠ½ΠΎΠ΅ соСдинСниС, Ρ‚ΠΎΠΊΠΈ, напряТСниС ΠΈ сопротивлСния ΠΏΡ€ΠΈ соСдинСния рСзисторов.

The paper studies the properties and physical laws of connecting resistors without soldering. When building layouts, soldering installation can be completely, abandoned or reduced to a minimum. As experience has shown, the practical use of layout boards can significantly save time and material costs.

Keyword: resistors, layout Board, connection without soldering, parallel and serial connection, currents, voltage and resistance when connecting resistors.

Π’ соврСмСнном Π²Ρ‹ΡΡˆΠ΅ΠΌ ΡƒΡ‡Π΅Π±Π½ΠΎΠΌ Π·Π°Π²Π΅Π΄Π΅Π½ΠΈΠΈ ΠΏΠ΅Ρ€Π΅Π΄ ΠΊΠ°ΠΆΠ΄Ρ‹ΠΌ ΠΏΡ€Π΅ΠΏΠΎΠ΄Π°Π²Π°Ρ‚Π΅Π»Π΅ΠΌ стоит Ρ†Π΅Π»ΡŒ ΠΏΠΎ Ρ€Π°Π·Π²ΠΈΡ‚ΠΈΡŽ Π½Π°ΡƒΡ‡Π½ΠΎ-ΠΈΡΡΠ»Π΅Π΄ΠΎΠ²Π°Ρ‚Π΅Π»ΡŒΡΠΊΠΈΡ… Π½Π°Π²Ρ‹ΠΊΠΎΠ² студСнтов. Один ΠΈΠ· Π·Π°Π΄Π°Ρ‡ являСтсямотивации студСнта ΠΈ ΡΠΎΠ²Π΅Ρ€ΡˆΠ΅Π½ΡΡ‚Π²ΠΎΠ²Π°Π½ΠΈΠ΅ Π½Π°ΡƒΡ‡Π½ΠΎ-ΠΈΡΡΠ»Π΅Π΄ΠΎΠ²Π°Ρ‚Π΅Π»ΡŒΡΠΊΠΈΡ… Π½Π°Π²Ρ‹ΠΊΠΎΠ² Π² Ρ„ΠΎΡ€ΠΌΡƒΠ»ΠΈΡ€ΠΎΠ²ΠΊΠ΅ ΠΈΡΡΠ»Π΅Π΄ΠΎΠ²Π°Ρ‚Π΅Π»ΡŒΡΠΊΠΎΠ³ΠΎ вопроса Π²ΠΎ врСмя Π»Π°Π±ΠΎΡ€Π°Ρ‚ΠΎΡ€Π½ΠΎΠΉ Ρ€Π°Π±ΠΎΡ‚Ρ‹ Π½Π° ΡƒΡ€ΠΎΠΊΠ°Ρ… Ρ„ΠΈΠ·ΠΈΠΊΠΈ. Π’ Π΄Π°Π½Π½ΠΎΠΉ Ρ€Π°Π±ΠΎΡ‚Π΅ исслСдованы свойства ΠΈ физичСскиС Π·Π°ΠΊΠΎΠ½Ρ‹ соСдинСния рСзисторов Π±Π΅Π· ΠΏΠ°ΠΉΠΊΠΈ. Как ΠΏΠΎΠΊΠ°Π·Π°Π», ΠΎΠΏΡ‹Ρ‚ практичСскоС использованиС ΠΌΠ°ΠΊΠ΅Ρ‚Π½Ρ‹Ρ… ΠΏΠ»Π°Ρ‚ ΠΏΠΎΠ·Π²ΠΎΠ»ΡΡŽΡ‚, сущСствСнно ΡΡΠΊΠΎΠ½ΠΎΠΌΠΈΡ‚ΡŒ Π²Ρ€Π΅ΠΌΠ΅Π½Π½Ρ‹Π΅ ΠΈ ΠΌΠ°Ρ‚Π΅Ρ€ΠΈΠ°Π»ΡŒΠ½Ρ‹Π΅ Π·Π°Ρ‚Ρ€Π°Ρ‚Ρ‹. [1,2].

ЦСль Ρ€Π°Π±ΠΎΡ‚Ρ‹: ΠΎΠΏΡ‹Ρ‚Π½Ρ‹ΠΌ ΠΏΡƒΡ‚Π΅ΠΌ ΠΈΠ·ΡƒΡ‡ΠΈΡ‚ΡŒ Π·Π°ΠΊΠΎΠ½Ρ‹ протСкания Ρ‚ΠΎΠΊΠ° Ρ‡Π΅Ρ€Π΅Π· ΠΏΠΎΡΠ»Π΅Π΄ΠΎΠ²Π°Ρ‚Π΅Π»ΡŒΠ½ΠΎ ΠΈ ΠΏΠ°Ρ€Π°Π»Π»Π΅Π»ΡŒΠ½ΠΎ соСдинСниС Ρ€Π΅Π·ΠΈΡΡ‚ΠΎΡ€ΠΎΠ²ΡΠΏΠΎΠΌΠΎΡ‰ΡŒΡŽ бСспаСчноС соСдинСниС ΠΈ ΠΎΠΏΡ€Π΅Π΄Π΅Π»ΠΈΡ‚ΡŒ Ρ„ΠΎΡ€ΠΌΡƒΠ»Ρ‹ сопротивлСний Ρ‚Π°ΠΊΠΈΡ… участков. РСзисторы примСниСтся Π² элСктронных ΠΏΡ€ΠΈΠ±ΠΎΡ€Π°Ρ… ΠΈ Ρ€ΠΎΠ±ΠΎΡ‚ΠΎΡ‚Π΅Ρ…Π½ΠΈΠΊΠ΅. [3].

ΠŸΠΎΡΠ»Π΅Π΄ΠΎΠ²Π°Ρ‚Π΅Π»ΡŒΠ½Ρ‹ΠΌ соСдинСниСм сопротивлСний называСтся Ρ‚Π°ΠΊΠΎΠ΅ соСдинСниС, ΠΏΡ€ΠΈ ΠΊΠΎΡ‚ΠΎΡ€ΠΎΠΌ ΠΊΠΎΠ½Π΅Ρ† ΠΏΠ΅Ρ€Π²ΠΎΠ³ΠΎ сопротивлСния соСдиняСтся с Π½Π°Ρ‡Π°Π»ΠΎΠΌ Π²Ρ‚ΠΎΡ€ΠΎΠ³ΠΎ, ΠΊΠΎΠ½Π΅Ρ† Π²Ρ‚ΠΎΡ€ΠΎΠ³ΠΎ β€” с Π½Π°Ρ‡Π°Π»ΠΎΠΌ Ρ‚Ρ€Π΅Ρ‚ΡŒΠ΅Π³ΠΎ ΠΈ Ρ‚. Π΄. Рассмотрим физичСскиС Π·Π°ΠΊΠΎΠ½Ρ‹ соСдинСния рСзисторов. ΠŸΠ΅Ρ€Π΅Ρ‡Π΅Π½ΡŒ оборудования ΠΏΠΎΠΊΠ°Π·Π°Π½Ρ‹ Π½Π° рис.1,2,3.

ΠŸΠ΅Ρ€Π΅Ρ‡Π΅Π½ΡŒ оборудования

Рис. 1. РСзистор ΠΈ ΠΏΡ€ΠΎΠ²ΠΎΠ΄Π°

Рис. 2. ΠœΡƒΠ»ΡŒΡ‚ΠΈΠΌΠ΅Ρ‚Ρ€

Рис. 3. ΠœΠ°ΠΊΠ΅Ρ‚Π½Π°Ρ ΠΏΠ»Π°Ρ‚Π°

ΠŸΠΎΡΠ»Π΅Π΄ΠΎΠ²Π°Ρ‚Π΅Π»ΡŒΠ½ΠΎ соСдинСнных рСзисторов

Π‘ΠΎΠΏΡ€ΠΎΡ‚ΠΈΠ²Π»Π΅Π½ΠΈΠ΅ Π²ΠΏΠΎΡΠ»Π΅Π΄ΠΎΠ²Π°Ρ‚Π΅Π»ΡŒΠ½ΠΎΠΉ Ρ†Π΅ΠΏΠΈ. ΠžΠ±Ρ‰Π΅Π΅ сопротивлСниС ΠΏΠΎΡΠ»Π΅Π΄ΠΎΠ²Π°Ρ‚Π΅Π»ΡŒΠ½ΠΎ соСдинСнных рСзисторов Ρ€Π°Π²Π½ΠΎ суммС ΠΈΡ… сопротивлСний.

RΠΎΠ±Ρ‰ = R1+R2+R2+ β€’β€’β€’+Rn(1)

Если ΠΏΠΎΡΠ»Π΅Π΄ΠΎΠ²Π°Ρ‚Π΅Π»ΡŒΠ½ΠΎ Π²ΠΊΠ»ΡŽΡ‡Π΅Π½ΠΎ любоС количСство рСзисторов ΠΎΠ΄ΠΈΠ½Π°ΠΊΠΎΠ²Ρ‹Ρ… сопротивлСний, Ρ‚ΠΎ ΠΈΡ… ΠΎΠ±Ρ‰Π΅Π΅ сопротивлСниС ΠΌΠΎΠΆΠ½ΠΎ ΠΎΠΏΡ€Π΅Π΄Π΅Π»ΠΈΡ‚ΡŒ, ΡƒΠΌΠ½ΠΎΠΆΠΈΠ² сопротивлСниС ΠΎΠ΄Π½ΠΎΠ³ΠΎ рСзистора Π½Π° количСство рСзисторов.

RΠΎΠ±Ρ‰ = nR1 (2)

ΠŸΡ€ΠΈΠΌΠ΅Ρ€ ΠΏΠΎΡΠ»Π΅Π΄ΠΎΠ²Π°Ρ‚Π΅Π»ΡŒΠ½ΠΎΠ³ΠΎ соСдинСния Π΄Π²ΡƒΡ… рСзисторов ΠΏΠΎΠΊΠ°Π·Π°Π½ Π½Π° рис.4.

ΠžΠ±Ρ‰Π΅Π΅ сопротивлСниС для Π΄Π²ΡƒΡ… рСзисторов.

RΠΎΠ±Ρ‰=R1+R2 (3)

RΠΎΠ±Ρ‰ = 25 Ом+35 Ом=60 Ом (4)

Рис. 4. ΠŸΠΎΡΠ»Π΅Π΄ΠΎΠ²Π°Ρ‚Π΅Π»ΡŒΠ½ΠΎΠ΅ соСдинСниС Π΄Π²ΡƒΡ… рСзисторов

Π’Π΅Π»ΠΈΡ‡ΠΈΠ½Π° Ρ‚ΠΎΠΊΠ° Π²ΠΏΠΎΡΠ»Π΅Π΄ΠΎΠ²Π°Ρ‚Π΅Π»ΡŒΠ½ΠΎΠΉ Ρ†Π΅ΠΏΠΈ

Π’Π°ΠΊ ΠΊΠ°ΠΊ Π² Π΄Π°Π½Π½ΠΎΠΉ Ρ†Π΅ΠΏΠΈ отсутствуСт ΠΎΡ‚Π²Π΅Ρ‚Π²Π»Π΅Π½ΠΈΠ΅ Ρ‚ΠΎΠΊΠ°, Ρ‚ΠΎ ΠΎΡ‡Π΅Π²ΠΈΠ΄Π½ΠΎ, Ρ‡Ρ‚ΠΎ количСство элСктричСства, ΠΏΡ€ΠΎΡ‚Π΅ΠΊΠ°ΡŽΡ‰Π΅Π΅ Ρ‡Π΅Ρ€Π΅Π· ΠΏΠΎΠΏΠ΅Ρ€Π΅Ρ‡Π½ΠΎΠ΅ сСчСниС ΠΏΡ€ΠΎΠ²ΠΎΠ΄Π½ΠΈΠΊΠ° Π·Π° Π΅Π΄ΠΈΠ½ΠΈΡ†Ρƒ Π²Ρ€Π΅ΠΌΠ΅Π½ΠΈ Π² любой Ρ‚ΠΎΡ‡ΠΊΠ΅ Ρ†Π΅ΠΏΠΈ, Π±ΡƒΠ΄Π΅Ρ‚ ΠΎΠ΄ΠΈΠ½Π°ΠΊΠΎΠ²Ρ‹ΠΌ. Π‘Π»Π΅Π΄ΠΎΠ²Π°Ρ‚Π΅Π»ΡŒΠ½ΠΎ, Π²ΠΎ всСх Ρ‚ΠΎΡ‡ΠΊΠ°Ρ… ΠΏΠΎΡΠ»Π΅Π΄ΠΎΠ²Π°Ρ‚Π΅Π»ΡŒΠ½ΠΎΠΉ Ρ†Π΅ΠΏΠΈ Π²Π΅Π»ΠΈΡ‡ΠΈΠ½Π° Ρ‚ΠΎΠΊΠ° ΠΎΠ΄ΠΈΠ½Π°ΠΊΠΎΠ²Π°.

ΠŸΠΎΡΡ‚ΠΎΠΌΡƒ ΠΏΡ€ΠΈ ΠΏΠΎΡΠ»Π΅Π΄ΠΎΠ²Π°Ρ‚Π΅Π»ΡŒΠ½ΠΎΠΌ соСдинСнии, для измСрСния Ρ‚ΠΎΠΊΠ° достаточно Π²ΠΊΠ»ΡŽΡ‡Π°Ρ‚ΡŒ ΠΎΠ΄ΠΈΠ½ Π°ΠΌΠΏΠ΅Ρ€ΠΌΠ΅Ρ‚Ρ€ Π½Π° любом участкС Ρ†Π΅ΠΏΠΈ.

Π’ качСствС ΠΏΡ€ΠΈΠΌΠ΅Ρ€Π°, ΠΌΠΎΠΆΠ½ΠΎ привСсти Ρ†Π΅ΠΏΡŒ ΠΏΠΎΡΠ»Π΅Π΄ΠΎΠ²Π°Ρ‚Π΅Π»ΡŒΠ½ΠΎΠ³ΠΎ соСдинСния Π΄Π²ΡƒΡ… рСзисторов, ΠΊΠΎΡ‚ΠΎΡ€Ρ‹ΠΉ ΠΏΠΎΠΊΠ°Π·Π°Π½ Π½Π° рис.5.

I=I1=I2=0,2A

Рис. 5. Π’Π΅Π»ΠΈΡ‡ΠΈΠ½Π° Ρ‚ΠΎΠΊΠ° Π² ΠΏΠΎΡΠ»Π΅Π΄ΠΎΠ²Π°Ρ‚Π΅Π»ΡŒΠ½ΠΎΠΉΡ†Π΅ΠΏΠΈ

РаспрСдСлСниС напряТСния Π²ΠΏΠΎΡΠ»Π΅Π΄ΠΎΠ²Π°Ρ‚Π΅Π»ΡŒΠ½ΠΎΠΉ Ρ†Π΅ΠΏΠΈ

НапряТСниС источника Ρ‚ΠΎΠΊΠ°, ΠΏΡ€ΠΈΠ»ΠΎΠΆΠ΅Π½Π½ΠΎΠ΅, ΠΊ Π²Π½Π΅ΡˆΠ½Π΅ΠΌΡƒ участку Ρ†Π΅ΠΏΠΈ распрСдСляСтся ΠΏΠΎ участкам Ρ†Π΅ΠΏΠΈ прямо ΠΏΡ€ΠΎΠΏΠΎΡ€Ρ†ΠΈΠΎΠ½Π°Π»ΡŒΠ½ΠΎ сопротивлСниям этих участков. НапряТСниС, ΠΏΡ€ΠΈΠ»ΠΎΠΆΠ΅Π½Π½ΠΎΠ΅, ΠΊ ΠΊΠ°ΠΆΠ΄ΠΎΠΌΡƒ ΠΈΠ· этих рСзисторов опрСдСляСтся ΠΏΠΎ Ρ„ΠΎΡ€ΠΌΡƒΠ»Π΅:

U=IR (5)

Π’Π°ΠΊ ΠΊΠ°ΠΊ Ρ‚ΠΎΠΊ Π² ΠΏΠΎΡΠ»Π΅Π΄ΠΎΠ²Π°Ρ‚Π΅Π»ΡŒΠ½ΠΎΠΉ Ρ†Π΅ΠΏΠΈ Π²Π΅Π·Π΄Π΅ ΠΎΠ΄ΠΈΠ½Π°ΠΊΠΎΠ², Π·Π½Π°Ρ‡ΠΈΡ‚, Π΄Π΅ΠΉΡΡ‚Π²ΠΈΡ‚Π΅Π»ΡŒΠ½ΠΎ напряТСниС Π½Π° Π΅Π΅ участках зависит ΠΎΡ‚ сопротивлСния, Ρ‡Π΅ΠΌ большС сопротивлСниС, Ρ‚Π΅ΠΌ большСС напряТСниС ΠΏΡ€ΠΈΠ»ΠΎΠΆΠ΅Π½ΠΎ ΠΊ Π΄Π°Π½Π½ΠΎΠΌΡƒ участку. Π‘ΡƒΠΌΠΌΠ° напряТСний Π½Π° участках ΠΏΠΎΡΠ»Π΅Π΄ΠΎΠ²Π°Ρ‚Π΅Π»ΡŒΠ½ΠΎΠΉ Ρ†Π΅ΠΏΠΈ Ρ€Π°Π²Π½Π° Π½Π°ΠΏΡ€ΡΠΆΠ΅Π½ΠΈΡŽ источника Ρ‚ΠΎΠΊΠ°.

UΠΎΠ±Ρ‰=U1+ U2+ U3+ β€’β€’β€’ +Un(6)

ΠŸΡ€ΠΈΠΌΠ΅Ρ€, распрСдСлСниС напряТСния Π² ΠΏΠΎΡΠ»Π΅Π΄ΠΎΠ²Π°Ρ‚Π΅Π»ΡŒΠ½ΠΎΠΉ Ρ†Π΅ΠΏΠΈ соСдинСния Π΄Π²ΡƒΡ… рСзисторов ΠΏΠΎΠΊΠ°Π·Π°Π½ Π½Π° рис.6.

UΠΎΠ±Ρ‰=U1+ U2(7)

12Π’=5Π’+7Π’

ΠŸΠ°Ρ€Π°Π»Π»Π΅Π»ΡŒΠ½ΠΎΠ΅ соСдинСниС сопротивлСний

ΠŸΠ°Ρ€Π°Π»Π»Π΅Π»ΡŒΠ½Ρ‹ΠΌ соСдинСниСм сопротивлСний называСтся Ρ‚Π°ΠΊΠΎΠ΅ соСдинСниС, ΠΏΡ€ΠΈ ΠΊΠΎΡ‚ΠΎΡ€ΠΎΠΌ ΠΊ ΠΎΠ΄Π½ΠΎΠΌΡƒ Π·Π°ΠΆΠΈΠΌΡƒ источника ΠΏΠΎΠ΄ΠΊΠ»ΡŽΡ‡Π°ΡŽΡ‚ΡΡ Π½Π°Ρ‡Π°Π»Π° сопротивлСний, Π° ΠΊ Π΄Ρ€ΡƒΠ³ΠΎΠΌΡƒ Π·Π°ΠΆΠΈΠΌΡƒ β€” ΠΊΠΎΠ½Ρ†Ρ‹.

Рис. 6. РаспрСдСлСниС напряТСния Π² ΠΏΠΎΡΠ»Π΅Π΄ΠΎΠ²Π°Ρ‚Π΅Π»ΡŒΠ½ΠΎΠΉ Ρ†Π΅ΠΏΠΈ

Π‘ΠΎΠΏΡ€ΠΎΡ‚ΠΈΠ²Π»Π΅Π½ΠΈΠ΅Π² ΠΏΠ°Ρ€Π°Π»Π»Π΅Π»ΡŒΠ½ΠΎΠΉ Ρ†Π΅ΠΏΠΈ

ΠžΠ±Ρ‰Π΅Π΅ сопротивлСниС ΠΏΠ°Ρ€Π°Π»Π»Π΅Π»ΡŒΠ½ΠΎ Π²ΠΊΠ»ΡŽΡ‡Π΅Π½Π½Ρ‹Ρ… сопротивлСний опрСдСляСтся ΠΏΠΎ Ρ„ΠΎΡ€ΠΌΡƒΠ»Π΅:

(8)

Если ΠΏΠ°Ρ€Π°Π»Π»Π΅Π»ΡŒΠ½ΠΎ Π²ΠΊΠ»ΡŽΡ‡Π΅Π½ΠΎ любоС количСство рСзисторов ΠΎΠ΄ΠΈΠ½Π°ΠΊΠΎΠ²Ρ‹Ρ… сопротивлСний, Ρ‚ΠΎ ΠΈΡ… ΠΎΠ±Ρ‰Π΅Π΅ сопротивлСниС ΠΌΠΎΠΆΠ½ΠΎ ΠΎΠΏΡ€Π΅Π΄Π΅Π»ΠΈΡ‚ΡŒ, Ссли сопротивлСниС ΠΎΠ΄Π½ΠΎΠ³ΠΎ рСзистора Ρ€Π°Π·Π΄Π΅Π»ΠΈΡ‚ΡŒ Π½Π° количСство рСзисторов.

ΠžΠ±Ρ‰Π΅Π΅ сопротивлСниС ΠΏΠ°Ρ€Π°Π»Π»Π΅Π»ΡŒΠ½ΠΎ Π²ΠΊΠ»ΡŽΡ‡Π΅Π½Π½Ρ‹Ρ… сопротивлСний всСгда мСньшС наимСньшСго сопротивлСния, входящСго Π² Π΄Π°Π½Π½ΠΎΠ΅ соСдинСниС. ΠŸΡ€ΠΈΠΌΠ΅Ρ€, Ссли ΠΏΠ°Ρ€Π°Π»Π»Π΅Π»ΡŒΠ½ΠΎ Π²ΠΊΠ»ΡŽΡ‡Π΅Π½ΠΎ Ρ‚ΠΎΠ»ΡŒΠΊΠΎ Π΄Π²Π° рСзистора (рис.7), Ρ‚ΠΎ ΠΈΡ… ΠΎΠ±Ρ‰Π΅Π΅ сопротивлСниС ΠΌΠΎΠΆΠ½ΠΎ ΠΎΠΏΡ€Π΅Π΄Π΅Π»ΠΈΡ‚ΡŒ ΠΏΠΎ Ρ„ΠΎΡ€ΠΌΡƒΠ»Π΅:

(10)

Рис. 7. ΠŸΠ°Ρ€Π°Π»Π»Π΅Π»ΡŒΠ½ΠΎΠ΅ соСдинСниС Π΄Π²ΡƒΡ… рСзисторов

РаспрСдСлСниС Ρ‚ΠΎΠΊΠ° Π²ΠΏΠ°Ρ€Π°Π»Π»Π΅Π»ΡŒΠ½ΠΎΠΉ Ρ†Π΅ΠΏΠΈ

Π’ Ρ†Π΅ΠΏΠΈ с ΠΏΠ°Ρ€Π°Π»Π»Π΅Π»ΡŒΠ½Ρ‹ΠΌ соСдинСниСм, Ρ‡Ρ‚ΠΎ элСктричСский Ρ‚ΠΎΠΊ распрСдСляСтся ΠΏΠΎ ΠΏΠ°Ρ€Π°Π»Π»Π΅Π»ΡŒΠ½Ρ‹ΠΌ вСтвям ΠΎΠ±Ρ€Π°Ρ‚Π½ΠΎ ΠΏΡ€ΠΎΠΏΠΎΡ€Ρ†ΠΈΠΎΠ½Π°Π»ΡŒΠ½ΠΎ ΠΈΡ… сопротивлСниям.

Π­Ρ‚ΠΎ Π·Π½Π°Ρ‡ΠΈΡ‚, Ρ‡Ρ‚ΠΎ Ρ‡Π΅ΠΌ большС сопротивлСниС, Ρ‚Π΅ΠΌ мСньшС ΠΏΠΎ Π½Π΅ΠΌΡƒ ΠΏΠΎΠΉΠ΄Π΅Ρ‚ Ρ‚ΠΎΠΊ.

(11)

Рассматривая Ρ‚ΠΎΡ‡ΠΊΡƒ развСтвлСния А, Π·Π°ΠΌΠ΅Ρ‡Π°Π΅ΠΌ, Ρ‡Ρ‚ΠΎ ΠΊ Π½Π΅ΠΉ ΠΏΡ€ΠΈΡ‚Π΅ΠΊΠ°Π΅Ρ‚ Ρ‚ΠΎΠΊ I, Π° Ρ‚ΠΎΠΊΠΈ I1, I2, ΡƒΡ‚Π΅ΠΊΠ°ΡŽΡ‚ ΠΈΠ· Π½Π΅Π΅. Π’Π°ΠΊ ΠΊΠ°ΠΊ двиТущиСся элСктричСскиС заряды Π½Π΅ ΡΠΊΠ°ΠΏΠ»ΠΈΠ²Π°ΡŽΡ‚ΡΡ Π² Ρ‚ΠΎΡ‡ΠΊΠ΅, Ρ‚ΠΎ ΠΎΡ‡Π΅Π²ΠΈΠ΄Π½ΠΎ, Ρ‡Ρ‚ΠΎ суммарный заряд, ΠΏΡ€ΠΈΡ‚Π΅ΠΊΠ°ΡŽΡ‰ΠΈΠΉ ΠΊ Ρ‚ΠΎΡ‡ΠΊΠ΅ развСтвлСния, Ρ€Π°Π²Π΅Π½ суммарному заряду, ΡƒΡ‚Π΅ΠΊΠ°ΡŽΡ‰Π΅ΠΌΡƒ ΠΎΡ‚ Π½Π΅Π΅:

I=I1+ I2+ I3 +β€’β€’β€’ +In(12)

Π‘Π»Π΅Π΄ΠΎΠ²Π°Ρ‚Π΅Π»ΡŒΠ½ΠΎ, Π²Π΅Π»ΠΈΡ‡ΠΈΠ½Π° Ρ‚ΠΎΠΊΠ° Π² Π½Π΅ Ρ€Π°Π·Π²Π΅Ρ‚Π²Π»Π΅Π½Π½ΠΎΠΉ части Ρ†Π΅ΠΏΠΈ Ρ€Π°Π²Π½Π° суммС Ρ‚ΠΎΠΊΠΎΠ² Π² ΠΏΠ°Ρ€Π°Π»Π»Π΅Π»ΡŒΠ½Ρ‹Ρ… вСтвях.

ΠŸΡ€ΠΈΠΌΠ΅Ρ€, распрСдСлСниС Ρ‚ΠΎΠΊΠ° Π² ΠΏΠ°Ρ€Π°Π»Π»Π΅Π»ΡŒΠ½ΠΎΠΉ Ρ†Π΅ΠΏΠΈ соСдинСния Π΄Π²ΡƒΡ… рСзисторов ΠΏΠΎΠΊΠ°Π·Π°Π½ Π½Π° рис. 8.

I=I1+ I2

Рис. 8. РаспрСдСлСниС Ρ‚ΠΎΠΊΠ° Π² ΠΏΠ°Ρ€Π°Π»Π»Π΅Π»ΡŒΠ½ΠΎΠΉ Ρ†Π΅ΠΏΠΈ

Π’Π΅Π»ΠΈΡ‡ΠΈΠ½Π° напряТСния Π²ΠΏΠ°Ρ€Π°Π»Π»Π΅Π»ΡŒΠ½Ρ‹Ρ… цСпях

Π’Π°ΠΊ ΠΊΠ°ΠΊ Π½Π°Ρ‡Π°Π»Π° всСх сопротивлСний свСдСны Π² ΠΎΠ΄Π½Ρƒ ΠΎΠ±Ρ‰ΡƒΡŽ Ρ‚ΠΎΡ‡ΠΊΡƒ, Π° ΠΊΠΎΠ½Ρ†Ρ‹ β€” Π² Π΄Ρ€ΡƒΠ³ΡƒΡŽ, Ρ‚ΠΎ ΠΎΡ‡Π΅Π²ΠΈΠ΄Π½ΠΎ, Ρ‡Ρ‚ΠΎ Ρ€Π°Π·Π½ΠΎΡΡ‚ΡŒ ΠΏΠΎΡ‚Π΅Π½Ρ†ΠΈΠ°Π»ΠΎΠ² Π½Π° ΠΊΠΎΠ½Ρ†Π°Ρ… любого ΠΈΠ· ΠΏΠ°Ρ€Π°Π»Π»Π΅Π»ΡŒΠ½ΠΎ Π²ΠΊΠ»ΡŽΡ‡Π΅Π½Π½Ρ‹Ρ… сопротивлСний Ρ€Π°Π²Π½Π° разности ΠΏΠΎΡ‚Π΅Π½Ρ†ΠΈΠ°Π»ΠΎΠ² ΠΌΠ΅ΠΆΠ΄Ρƒ ΠΎΠ±Ρ‰ΠΈΠΌΠΈ Ρ‚ΠΎΡ‡ΠΊΠ°ΠΌΠΈ.

Π˜Ρ‚Π°ΠΊ, ΠΏΡ€ΠΈ ΠΏΠ°Ρ€Π°Π»Π»Π΅Π»ΡŒΠ½ΠΎΠΌ соСдинСнии сопротивлСний напряТСния Π½Π° Π½ΠΈΡ… Ρ€Π°Π²Π½Ρ‹ ΠΌΠ΅ΠΆΠ΄Ρƒ собой.

U=U1=U2=U3=β€’β€’β€’=Un (13)

Π‘ΠΎΠ΄Π΅Ρ€ΠΆΠ°Π½ΠΈΠ΅ Ρ…ΠΎΠ΄Π° Ρ€Π°Π±ΠΎΡ‚Ρ‹ ΠΈΠΏΠΎΡΠ»Π΅Π΄ΠΎΠ²Π°Ρ‚Π΅Π»ΡŒΠ½ΠΎΡΡ‚ΡŒ дСйствий

ВСхничСскоС Π·Π°Π΄Π°Π½ΠΈΠ΅

1) Π‘ΠΎΠ±Ρ€Π°Ρ‚ΡŒ ΡΠ»Π΅ΠΊΡ‚Ρ€ΠΈΡ‡Π΅ΡΠΊΡƒΡŽ Ρ†Π΅ΠΏΡŒ Π² ΠΌΠ°ΠΊΠ΅Ρ‚Π½ΡƒΡŽ ΠΏΠ»Π°Ρ‚Ρƒ (рис.9.)

2) Π‘Π½ΡΡ‚ΡŒ показания ΠΏΡ€ΠΈΠ±ΠΎΡ€ΠΎΠ² ΠΈ Π·Π°ΠΏΠΈΡΠ°Ρ‚ΡŒ ΠΈΡ… Π² Ρ‚Π°Π±Π»ΠΈΡ†Ρƒ.

3) ΠŸΡ€ΠΎΠΈΠ·Π²Π΅ΡΡ‚ΠΈ расчСты.

4) ΠžΡ‚Π²Π΅Ρ‚ΠΈΡ‚ΡŒ Π½Π° ΠΊΠΎΠ½Ρ‚Ρ€ΠΎΠ»ΡŒΠ½Ρ‹Π΅ вопросы.

5) Π‘Π΄Π΅Π»Π°Ρ‚ΡŒ Π²Ρ‹Π²ΠΎΠ΄.

ΠŸΠΎΡ€ΡΠ΄ΠΎΠΊ выполнСния Ρ€Π°Π±ΠΎΡ‚Ρ‹

1) Π‘ΠΎΠ±Ρ€Π°Ρ‚ΡŒ схСму Π² ΠΌΠ°ΠΊΠ΅Ρ‚Π½ΡƒΡŽ ΠΏΠ»Π°Ρ‚Ρƒ (Рис. 9.).

2) Π£ΡΡ‚Π°Π½ΠΎΠ²ΠΈΡ‚ΡŒ Π½Π° схСмС Π²Π΅Π»ΠΈΡ‡ΠΈΠ½Ρ‹ R1=1 кОм + N, R2=2 кОм + N, R3=3 кОм + N ΠΈ R4=4 кОм+ N, Π³Π΄Π΅ N β€” Π½ΠΎΠΌΠ΅Ρ€ студСнта ΠΏΠΎ ΠΆΡƒΡ€Π½Π°Π»Ρƒ (ΠΌΠΎΡ‰Π½ΠΎΡΡ‚ΡŒ рСзисторов Π±ΠΎΠ»Π΅Π΅ 1 Π’Ρ‚).

3) Π’ΠΊΠ»ΡŽΡ‡ΠΈΡ‚ΡŒ источник ΠΈ ΡƒΡΡ‚Π°Π½ΠΎΠ²ΠΈΡ‚ΡŒ напряТСниС U=5Π’, 9Π’, 12Π’, 15Π’.

4) ΠžΠΏΡ€Π΅Π΄Π΅Π»ΠΈΡ‚Π΅ ΡΠΊΡΠΏΠ΅Ρ€ΠΈΠΌΠ΅Π½Ρ‚Π°Π»ΡŒΠ½ΠΎ с ΠΏΠΎΠΌΠΎΡ‰ΡŒΡŽ ΠΌΡƒΠ»ΡŒΡ‚ΠΈΠΌΠ΅Ρ‚Ρ€Π° (Π² Ρ€Π΅ΠΆΠΈΠΌΠ΅ измСрСния сопротивлСний) сопротивлСниС ΠΌΠ΅ΠΆΠ΄Ρƒ Ρ‚ΠΎΡ‡ΠΊΠ°ΠΌΠΈ:

А ΠΈΠ’; Π’ΠΈ Π‘; Аи Π‘; DΠΈ Π•.

Π—Π°ΠΏΠΈΡˆΠΈΡ‚Π΅ эти показания Π² Ρ‚Π°Π±Π»ΠΈΡ†Ρƒ 1, 2.

Рис. 9. Π‘Ρ…Π΅ΠΌΠ° исслСдования: Π°) ΠΏΠΎΡΠ»Π΅Π΄ΠΎΠ²Π°Ρ‚Π΅Π»ΡŒΠ½ΠΎΠ΅ соСдинСниС, Π±) ΠΏΠ°Ρ€Π°Π»Π»Π΅Π»ΡŒΠ½ΠΎΠ΅ соСдинСниС

5) РассчитайтС тСорСтичСскиС значСния сопротивлСний ΠΌΠ΅ΠΆΠ΄Ρƒ ΡƒΠΊΠ°Π·Π°Π½Π½Ρ‹ΠΌΠΈ Ρ‚ΠΎΡ‡ΠΊΠ°ΠΌΠΈ схСмы ΠΈ сравнитС ΠΈΡ… с ΠΈΠ·ΠΌΠ΅Ρ€Π΅Π½Π½Ρ‹ΠΌΠΈ.

6) Π˜Π·ΠΌΠ΅Ρ€ΡŒΡ‚Π΅ с ΠΏΠΎΠΌΠΎΡ‰ΡŒΡŽ ΠΌΡƒΠ»ΡŒΡ‚ΠΈΠΌΠ΅Ρ‚Ρ€Π° (Π² Ρ€Π΅ΠΆΠΈΠΌΠ΅ измСрСния постоянного Ρ‚ΠΎΠΊΠ°) Ρ‚ΠΎΠΊΠΈ, Ρ‚Π΅ΠΊΡƒΡ‰ΠΈΠ΅ Ρ‡Π΅Ρ€Π΅Π· ΠΊΠ°ΠΆΠ΄ΠΎΠ΅ сопротивлСниС. Π—Π°ΠΏΠΈΡˆΠΈΡ‚Π΅ показания ΠΏΡ€ΠΈΠ±ΠΎΡ€Π° Π² Ρ‚Π°Π±Π»ΠΈΡ†Ρƒ 1.

7) ΠŸΡ€ΠΎΠ²Π΅Ρ€ΡŒΡ‚Π΅ ΡΠΊΡΠΏΠ΅Ρ€ΠΈΠΌΠ΅Π½Ρ‚Π°Π»ΡŒΠ½ΠΎ, Ρ‡Ρ‚ΠΎ Π² ΠΏΠΎΡΠ»Π΅Π΄ΠΎΠ²Π°Ρ‚Π΅Π»ΡŒΠ½ΠΎΠΉ Ρ†Π΅ΠΏΠΈ Ρ‚ΠΎΠΊ ΠΎΠ΄ΠΈΠ½Π°ΠΊΠΎΠ² Ρ‡Π΅Ρ€Π΅Π· всС сопротивлСния, Π° Π² ΠΏΠ°Ρ€Π°Π»Π»Π΅Π»ΡŒΠ½ΠΎΠΉ Ρ†Π΅ΠΏΠΈ раздСляСтся Ρ‚Π°ΠΊ, Ρ‡Ρ‚ΠΎ сумма всСх Ρ‚ΠΎΠΊΠΎΠ² Ρ‡Π΅Ρ€Π΅Π· ΠΏΠ°Ρ€Π°Π»Π»Π΅Π»ΡŒΠ½ΠΎ соСдинСнныС элСмСнты, Ρ€Π°Π²Π½Π° ΠΏΠΎΠ»Π½ΠΎΠΌΡƒ Ρ‚ΠΎΠΊΡƒ Ρ‡Π΅Ρ€Π΅Π· вСсь участок.

8) Π˜Π·ΠΌΠ΅Ρ€ΡŒΡ‚Π΅ с ΠΏΠΎΠΌΠΎΡ‰ΡŒΡŽ ΠΌΡƒΠ»ΡŒΡ‚ΠΈΠΌΠ΅Ρ‚Ρ€Π° (Π² Ρ€Π΅ΠΆΠΈΠΌΠ΅ измСрСния постоянного напряТСния) напряТСния Π½Π° ΠΊΠ°ΠΆΠ΄ΠΎΠΌ сопротивлСнии. Π—Π°ΠΏΠΈΡˆΠΈΡ‚Π΅ показания ΠΏΡ€ΠΈΠ±ΠΎΡ€Π° Π² Ρ‚Π°Π±Π»ΠΈΡ†Ρƒ 2.

9) ΠŸΡ€ΠΎΠ²Π΅Ρ€ΡŒΡ‚Π΅ ΡΠΊΡΠΏΠ΅Ρ€ΠΈΠΌΠ΅Π½Ρ‚Π°Π»ΡŒΠ½ΠΎ, Ρ‡Ρ‚ΠΎ Π² ΠΏΠΎΡΠ»Π΅Π΄ΠΎΠ²Π°Ρ‚Π΅Π»ΡŒΠ½ΠΎΠΉ Ρ†Π΅ΠΏΠΈ напряТСниС Π½Π° всСм участкС Ρ€Π°Π²Π½ΠΎ суммС напряТСний Π½Π° ΠΊΠ°ΠΆΠ΄ΠΎΠΌ элСмСнтС, Π° Π² ΠΏΠ°Ρ€Π°Π»Π»Π΅Π»ΡŒΠ½ΠΎΠΉ Ρ†Π΅ΠΏΠΈ, напряТСниС ΠΎΠ΄Π½ΠΎ ΠΈ Ρ‚ΠΎ ΠΆΠ΅ Π½Π° ΠΊΠ°ΠΆΠ΄ΠΎΠΌ элСмСнтС.

10) ΠžΡ‚ΠΊΠ»ΡŽΡ‡ΠΈΡ‚ΡŒ схСму.

Π’Π°Π±Π»ΠΈΡ†Π° 1

РассчитанныС ΠΈΠΈΠ·ΠΌΠ΅Ρ€Π΅Π½Π½Ρ‹Π΅ ΠΏΠ°Ρ€Π°ΠΌΠ΅Ρ‚Ρ€Ρ‹ для ΠΏΠΎΡΠ»Π΅Π΄ΠΎΠ²Π°Ρ‚Π΅Π»ΡŒΠ½ΠΎΠ³ΠΎ соСдинСния рСзисторов: Аи Π’; Π’ΠΈ Π‘; Аи Π‘.

β„–Π²Π°Ρ€.

УстановлСно

Рассчитано

Π˜Π·ΠΌΡ‘Ρ€Π΅Π½Π½Ρ‹Π΅ ΠΏΠ°Ρ€Π°ΠΌΠ΅Ρ‚Ρ€Ρ‹

R1

R2

А— Π’

Π’β€” Π‘

А— Π‘

I

U

U1

U2

кОм

кОм

кОм

кОм

кОм

А

Π’

Π’

Π’

Π’Π°Π±Π»ΠΈΡ†Π° 2

РассчитанныС ΠΈΠΈΠ·ΠΌΠ΅Ρ€Π΅Π½Π½Ρ‹Π΅ ΠΏΠ°Ρ€Π°ΠΌΠ΅Ρ‚Ρ€Ρ‹ для ΠΏΠ°Ρ€Π°Π»Π»Π΅Π»ΡŒΠ½ΠΎΠ³ΠΎ соСдинСния рСзисторов: DΠΈ Π•.

β„–Π²Π°Ρ€.

УстановлСно

Рассчитано

Π˜Π·ΠΌΠ΅Ρ€Ρ‘Π½Π½Ρ‹Π΅ ΠΏΠ°Ρ€Π°ΠΌΠ΅Ρ‚Ρ€Ρ‹

R3

R4

D ΠΈΠ•

U

I

I1

I2

кОм

кОм

кОм

Π’

А

А

А

Π‘ΠΎΠ΄Π΅Ρ€ΠΆΠ°Π½ΠΈΠ΅ ΠΎΡ‚Ρ‡Π΅Ρ‚Π°

Π°) Π½Π°Π·Π²Π°Π½ΠΈΠ΅ ΠΈ Ρ†Π΅Π»ΡŒ Ρ€Π°Π±ΠΎΡ‚Ρ‹;

Π±) схСмы экспСримСнтов ΠΈ Ρ‚Π°Π±Π»ΠΈΡ†Ρ‹ ΠΏΠΎΠ»ΡƒΡ‡Π΅Π½Π½Ρ‹Ρ… ΡΠΊΡΠΏΠ΅Ρ€ΠΈΠΌΠ΅Π½Ρ‚Π°Π»ΡŒΠ½Ρ‹Ρ… Π΄Π°Π½Π½Ρ‹Ρ…;

Π²) Ρ€Π΅Π·ΡƒΠ»ΡŒΡ‚Π°Ρ‚Ρ‹ расчётов:

Π³) Π²Ρ‹Π²ΠΎΠ΄Ρ‹ ΠΏΠΎ Ρ€Π°Π±ΠΎΡ‚Π΅.

ΠšΠΎΠ½Ρ‚Ρ€ΠΎΠ»ΡŒΠ½Ρ‹Π΅ вопросы

1) ΠœΠΎΠΆΠ΅Ρ‚ Π»ΠΈ сопротивлСниС участка Π΄Π²ΡƒΡ… ΠΏΠ°Ρ€Π°Π»Π»Π΅Π»ΡŒΠ½ΠΎ соСдинСнных ΠΏΡ€ΠΎΠ²ΠΎΠ΄Π½ΠΈΠΊΠΎΠ² Π±Ρ‹Ρ‚ΡŒ большС (мСньшС) любого ΠΈΠ· Π½ΠΈΡ…? ΠžΠ±ΡŠΡΡΠ½ΠΈΡ‚Π΅ ΠΎΡ‚Π²Π΅Ρ‚.

2) Как ΠΏΠΎ Π²ΠΎΠ»ΡŒΡ‚Π°ΠΌΠΏΠ΅Ρ€Π½ΠΎΠΉ характСристикС ΠΎΠΏΡ€Π΅Π΄Π΅Π»ΠΈΡ‚ΡŒ Π²Π΅Π»ΠΈΡ‡ΠΈΠ½Ρƒ сопротивлСния Ρ†Π΅ΠΏΠΈ?

3) Как ΠΏΠΎ показаниям ΠΌΡƒΠ»ΡŒΡ‚ΠΈΠΌΠ΅Ρ‚Ρ€Π° ΠΌΠΎΠΆΠ½ΠΎ ΠΎΠΏΡ€Π΅Π΄Π΅Π»ΠΈΡ‚ΡŒ Π²Π΅Π»ΠΈΡ‡ΠΈΠ½Ρƒ сопротивлСния участка элСктричСской Ρ†Π΅ΠΏΠΈ ΠΈ потрСбляСмая ΠΌΠΎΡ‰Π½ΠΎΡΡ‚ΡŒ?

4) НарисуйтС схСмы ΠΏΠΎΡΠ»Π΅Π΄ΠΎΠ²Π°Ρ‚Π΅Π»ΡŒΠ½ΠΎΠ΅ ΠΈ ΠΏΠ°Ρ€Π°Π»Π»Π΅Π»ΡŒΠ½ΠΎΠ΅ соСдинСниС Π΄Π²ΡƒΡ… рСзисторов.

5) ΠžΠ±ΡŠΡΡΠ½ΠΈΡ‚Π΅ физичСскиС Π·Π°ΠΊΠΎΠ½Ρ‹ соСдинСния рСзисторов.

Π’Ρ‹Π²ΠΎΠ΄Ρ‹

Π’ Ρ€Π°Π±ΠΎΡ‚Π΅ исслСдованы свойства ΠΈ физичСскиС Π·Π°ΠΊΠΎΠ½Ρ‹ ΠΏΠ°Ρ€Π°Π»Π»Π΅Π»ΡŒΠ½ΠΎΠ³ΠΎ ΠΈ ΠΏΠΎΡΠ»Π΅Π΄ΠΎΠ²Π°Ρ‚Π΅Π»ΡŒΠ½ΠΎΠ³ΠΎ соСдинСния рСзисторов бСс примСнСния ΠΏΠ°ΠΉΠΊΠΈ.

ΠŸΡ€ΠΈ построСнии ΠΌΠ°ΠΊΠ΅Ρ‚ΠΎΠ² ΠΎΡ‚ ΠΌΠΎΠ½Ρ‚Π°ΠΆΠ° ΠΏΠ°ΠΉΠΊΠΎΠΉ удаСтся ΠΏΠΎΠ»Π½ΠΎΡΡ‚ΡŒΡŽ ΠΎΡ‚ΠΊΠ°Π·Π°Ρ‚ΡŒΡΡ ΠΈΠ»ΠΈ свСсти Π΅Π³ΠΎ ΠΊ ΠΌΠΈΠ½ΠΈΠΌΡƒΠΌΡƒ. Как ΠΏΠΎΠΊΠ°Π·Π°Π», ΠΎΠΏΡ‹Ρ‚ практичСскоС использованиС ΠΌΠ°ΠΊΠ΅Ρ‚Π½Ρ‹Ρ… ΠΏΠ»Π°Ρ‚ ΠΏΠΎΠ·Π²ΠΎΠ»ΡΡŽΡ‚, сущСствСнно ΡΡΠΊΠΎΠ½ΠΎΠΌΠΈΡ‚ΡŒ Π²Ρ€Π΅ΠΌΠ΅Π½Π½Ρ‹Π΅ ΠΈ ΠΌΠ°Ρ‚Π΅Ρ€ΠΈΠ°Π»ΡŒΠ½Ρ‹Π΅ Π·Π°Ρ‚Ρ€Π°Ρ‚Ρ‹.

Π›ΠΈΡ‚Π΅Ρ€Π°Ρ‚ΡƒΡ€Π°:

  1. А. Π›. ΠœΠ°Ρ€Ρ‡Π΅Π½ΠΊΠΎ, Освальд Π‘. Π’. Π›Π°Π±ΠΎΡ€Π°Ρ‚ΠΎΡ€Π½Ρ‹ΠΉ ΠΏΡ€Π°ΠΊΡ‚ΠΈΠΊΡƒΠΌ ΠΏΠΎ элСктротСхникС ΠΈ элСктроникС Π² срСдСMULTISIM. Π£Ρ‡Π΅Π±Π½ΠΎΠ΅ пособиС для Π²ΡƒΠ·ΠΎΠ².-М.:Π”ΠœΠš ΠŸΡ€Π΅ΡΡ.2010.-448Π‘.
  2. Π›. Π“. БСлиовская, БСлиовский Н. А. Π ΠΎΠ±ΠΎΡ‚ΠΈΠ·ΠΈΡ€ΠΎΠ²Π°Π½Π½Ρ‹Π΅ Π»Π°Π±ΠΎΡ€Π°Ρ‚ΠΎΡ€Π½Ρ‹Π΅Π΅ Ρ€Π°Π±ΠΎΡ‚Ρ‹ ΠΏΠΎ Ρ„ΠΈΠ·ΠΈΠΊΠΈ, ΠŸΡ€ΠΎΠΏΠ΅Π΄Π΅Π²Ρ‚Π΅Ρ‚ΠΈΡ‡Π΅ΡΠΊΠΈΠΉ курс Ρ„ΠΈΠ·ΠΈΠΊΠ°. -М.:Π”ΠœΠš ΠŸΡ€Π΅ΡΡ.2016.-164Π‘.
  3. ВикипСдия [Π­Π»Π΅ΠΊΡ‚Ρ€ΠΎΠ½Π½Ρ‹ΠΉ рСсурс]. http://ru.wikipedia.org/wiki/Processing.

ΠžΡΠ½ΠΎΠ²Π½Ρ‹Π΅ Ρ‚Π΅Ρ€ΠΌΠΈΠ½Ρ‹ (Π³Π΅Π½Π΅Ρ€ΠΈΡ€ΡƒΡŽΡ‚ΡΡ автоматичСски): ΠΎΠ±Ρ‰Π΅Π΅ сопротивлСниС, рСзистор, сопротивлСниС, ΠΏΠΎΡΠ»Π΅Π΄ΠΎΠ²Π°Ρ‚Π΅Π»ΡŒΠ½Π°Ρ Ρ†Π΅ΠΏΡŒ, ΠΏΠΎΡΠ»Π΅Π΄ΠΎΠ²Π°Ρ‚Π΅Π»ΡŒΠ½ΠΎΠ΅ соСдинСниС, соСдинСниС рСзисторов, Π²Π΅Π»ΠΈΡ‡ΠΈΠ½Π° Ρ‚ΠΎΠΊΠ°, ΠΊΠΎΠΌ, макСтная ΠΏΠ»Π°Ρ‚Π°, ΠΏΠ°Ρ€Π°Π»Π»Π΅Π»ΡŒΠ½Π°Ρ Ρ†Π΅ΠΏΡŒ, ΠΏΠ°Ρ€Π°Π»Π»Π΅Π»ΡŒΠ½ΠΎΠ΅ соСдинСниС.

ΠŸΠ°Ρ€Π°Π»Π»Π΅Π»ΡŒΠ½Π°Ρ схСма: характСристики, прСимущСства ΠΈ нСдостатки

ΠŸΠ°Ρ€Π°Π»Π»Π΅Π»ΡŒΠ½ΠΎΠ΅ соСдинСниС ΠΏΡ€ΠΎΠ²ΠΎΠ΄Π½ΠΈΠΊΠΎΠ²

ΠŸΠ°Ρ€Π°Π»Π»Π΅Π»ΡŒΠ½Ρ‹ΠΌ соСдинСниСм ΠΏΡ€ΠΎΠ²ΠΎΠ΄Π½ΠΈΠΊΠΎΠ² называСтся Ρ‚Π°ΠΊΠΎΠ΅ соСдинСниС, ΠΊΠΎΠ³Π΄Π° Π½Π°Ρ‡Π°Π»Π° всСх ΠΏΡ€ΠΎΠ²ΠΎΠ΄Π½ΠΈΠΊΠΎΠ² соСдинСны Π² ΠΎΠ΄Π½Ρƒ Ρ‚ΠΎΡ‡ΠΊΡƒ, Π° ΠΊΠΎΠ½Ρ†Ρ‹ ΠΏΡ€ΠΎΠ²ΠΎΠ΄Π½ΠΈΠΊΠΎΠ² – Π² Π΄Ρ€ΡƒΠ³ΡƒΡŽ Ρ‚ΠΎΡ‡ΠΊΡƒ (рисунок 4). Начало Ρ†Π΅ΠΏΠΈ присоСдиняСтся ΠΊ ΠΎΠ΄Π½ΠΎΠΌΡƒ ΠΏΠΎΠ»ΡŽΡΡƒ источника напряТСния, Π° ΠΊΠΎΠ½Π΅Ρ† Ρ†Π΅ΠΏΠΈ – ΠΊ Π΄Ρ€ΡƒΠ³ΠΎΠΌΡƒ ΠΏΠΎΠ»ΡŽΡΡƒ.

Рисунок 4. Π‘Ρ…Π΅ΠΌΠ° ΠΏΠ°Ρ€Π°Π»Π»Π΅Π»ΡŒΠ½ΠΎΠ³ΠΎ соСдинСния ΠΏΡ€ΠΎΠ²ΠΎΠ΄Π½ΠΈΠΊΠΎΠ²

Из рисунка Π²ΠΈΠ΄Π½ΠΎ, Ρ‡Ρ‚ΠΎ ΠΏΡ€ΠΈ ΠΏΠ°Ρ€Π°Π»Π»Π΅Π»ΡŒΠ½ΠΎΠΌ соСдинСнии ΠΏΡ€ΠΎΠ²ΠΎΠ΄Π½ΠΈΠΊΠΎΠ² для прохоТдСния Ρ‚ΠΎΠΊΠ° имССтся нСсколько ΠΏΡƒΡ‚Π΅ΠΉ. Π’ΠΎΠΊ, протСкая ΠΊ Ρ‚ΠΎΡ‡ΠΊΠ΅ развСтвлСния А, растСкаСтся Π΄Π°Π»Π΅Π΅ ΠΏΠΎ Ρ‚Ρ€Π΅ΠΌ сопротивлСниям ΠΈ Ρ€Π°Π²Π΅Π½ суммС Ρ‚ΠΎΠΊΠΎΠ², уходящих ΠΎΡ‚ этой Ρ‚ΠΎΡ‡ΠΊΠΈ:

I = I1 + I2 + I3.

Если Ρ‚ΠΎΠΊΠΈ, приходящиС ΠΊ Ρ‚ΠΎΡ‡ΠΊΠ΅ развСтвлСния, ΡΡ‡ΠΈΡ‚Π°Ρ‚ΡŒ ΠΏΠΎΠ»ΠΎΠΆΠΈΡ‚Π΅Π»ΡŒΠ½Ρ‹ΠΌΠΈ, Π° уходящиС – ΠΎΡ‚Ρ€ΠΈΡ†Π°Ρ‚Π΅Π»ΡŒΠ½Ρ‹ΠΌΠΈ, Ρ‚ΠΎ для Ρ‚ΠΎΡ‡ΠΊΠΈ развСтвлСния ΠΌΠΎΠΆΠ½ΠΎ Π½Π°ΠΏΠΈΡΠ°Ρ‚ΡŒ:

Ρ‚ΠΎ Π΅ΡΡ‚ΡŒ алгСбраичСская сумма Ρ‚ΠΎΠΊΠΎΠ² для любой ΡƒΠ·Π»ΠΎΠ²ΠΎΠΉ Ρ‚ΠΎΡ‡ΠΊΠΈ Ρ†Π΅ΠΏΠΈ всСгда Ρ€Π°Π²Π½Π° Π½ΡƒΠ»ΡŽ. Π­Ρ‚ΠΎ ΡΠΎΠΎΡ‚Π½ΠΎΡˆΠ΅Π½ΠΈΠ΅, ΡΠ²ΡΠ·Ρ‹Π²Π°ΡŽΡ‰Π΅Π΅ Ρ‚ΠΎΠΊΠΈ Π² любой Ρ‚ΠΎΡ‡ΠΊΠ΅ развСтвлСния Ρ†Π΅ΠΏΠΈ, называСтся ΠΏΠ΅Ρ€Π²Ρ‹ΠΌ Π·Π°ΠΊΠΎΠ½ΠΎΠΌ ΠšΠΈΡ€Ρ…Π³ΠΎΡ„Π°. ΠžΠΏΡ€Π΅Π΄Π΅Π»Π΅Π½ΠΈΠ΅ ΠΏΠ΅Ρ€Π²ΠΎΠ³ΠΎ Π·Π°ΠΊΠΎΠ½Π° ΠšΠΈΡ€Ρ…Π³ΠΎΡ„Π° ΠΌΠΎΠΆΠ΅Ρ‚ Π·Π²ΡƒΡ‡Π°Ρ‚ΡŒ ΠΈ Π² Π΄Ρ€ΡƒΠ³ΠΎΠΉ Ρ„ΠΎΡ€ΠΌΡƒΠ»ΠΈΡ€ΠΎΠ²ΠΊΠ΅, Π° ΠΈΠΌΠ΅Π½Π½ΠΎ: сумма Ρ‚ΠΎΠΊΠΎΠ² Π²Ρ‚Π΅ΠΊΠ°ΡŽΡ‰ΠΈΡ… Π² ΡƒΠ·Π΅Π» элСктричСской Ρ†Π΅ΠΏΠΈ Ρ€Π°Π²Π½Π° суммС Ρ‚ΠΎΠΊΠΎΠ² Π²Ρ‹Ρ‚Π΅ΠΊΠ°ΡŽΡ‰ΠΈΡ… ΠΈΠ· этого ΡƒΠ·Π»Π°.

Π’ΠΈΠ΄Π΅ΠΎ 2. ΠŸΠ΅Ρ€Π²Ρ‹ΠΉ Π·Π°ΠΊΠΎΠ½ ΠšΠΈΡ€Ρ…Π³ΠΎΡ„Π°

ΠžΠ±Ρ‹Ρ‡Π½ΠΎ ΠΏΡ€ΠΈ расчСтС элСктричСских Ρ†Π΅ΠΏΠ΅ΠΉ Π½Π°ΠΏΡ€Π°Π²Π»Π΅Π½ΠΈΠ΅ Ρ‚ΠΎΠΊΠΎΠ² Π² вСтвях, присоСдинСнных к ΠΊΠ°ΠΊΠΎΠΉ Π»ΠΈΠ±ΠΎ Ρ‚ΠΎΡ‡ΠΊΠ΅ развСтвлСния, нСизвСстны. ΠŸΠΎΡΡ‚ΠΎΠΌΡƒ для возмоТности самой записи уравнСния ΠΏΠ΅Ρ€Π²ΠΎΠ³ΠΎ Π·Π°ΠΊΠΎΠ½Π° ΠšΠΈΡ€Ρ…Π³ΠΎΡ„Π° Π½ΡƒΠΆΠ½ΠΎ ΠΏΠ΅Ρ€Π΅Π΄ Π½Π°Ρ‡Π°Π»ΠΎΠΌ расчСта Ρ†Π΅ΠΏΠΈ ΠΏΡ€ΠΎΠΈΠ·Π²ΠΎΠ»ΡŒΠ½ΠΎ Π²Ρ‹Π±Ρ€Π°Ρ‚ΡŒ Ρ‚Π°ΠΊ Π½Π°Π·Ρ‹Π²Π°Π΅ΠΌΡ‹Π΅ ΠΏΠΎΠ»ΠΎΠΆΠΈΡ‚Π΅Π»ΡŒΠ½Ρ‹Π΅ направлСния Ρ‚ΠΎΠΊΠΎΠ² Π²ΠΎ всСх Π΅Π΅ вСтвях ΠΈ ΠΎΠ±ΠΎΠ·Π½Π°Ρ‡ΠΈΡ‚ΡŒ ΠΈΡ… стрСлками Π½Π° схСмС.

ΠŸΠΎΠ»ΡŒΠ·ΡƒΡΡΡŒ Π·Π°ΠΊΠΎΠ½ΠΎΠΌ Ома, ΠΌΠΎΠΆΠ½ΠΎ вывСсти Ρ„ΠΎΡ€ΠΌΡƒΠ»Ρƒ для подсчСта ΠΎΠ±Ρ‰Π΅Π³ΠΎ сопротивлСния ΠΏΡ€ΠΈ ΠΏΠ°Ρ€Π°Π»Π»Π΅Π»ΡŒΠ½ΠΎΠΌ соСдинСнии ΠΏΠΎΡ‚Ρ€Π΅Π±ΠΈΡ‚Π΅Π»Π΅ΠΉ.

ΠžΠ±Ρ‰ΠΈΠΉ Ρ‚ΠΎΠΊ, приходящий ΠΊ Ρ‚ΠΎΡ‡ΠΊΠ΅ А, Ρ€Π°Π²Π΅Π½:

Π’ΠΎΠΊΠΈ Π² ΠΊΠ°ΠΆΠ΄ΠΎΠΉ ΠΈΠ· Π²Π΅Ρ‚Π²Π΅ΠΉ ΠΈΠΌΠ΅ΡŽΡ‚ значСния:

По Ρ„ΠΎΡ€ΠΌΡƒΠ»Π΅ ΠΏΠ΅Ρ€Π²ΠΎΠ³ΠΎ Π·Π°ΠΊΠΎΠ½Π° ΠšΠΈΡ€Ρ…Π³ΠΎΡ„Π°

I = I1 + I2 + I3

ΠΈΠ»ΠΈ

Вынося U Π² ΠΏΡ€Π°Π²ΠΎΠΉ части равСнства Π·Π° скобки, ΠΏΠΎΠ»ΡƒΡ‡ΠΈΠΌ:

Бокращая ΠΎΠ±Π΅ части равСнства Π½Π° U, ΠΏΠΎΠ»ΡƒΡ‡ΠΈΠΌ Ρ„ΠΎΡ€ΠΌΡƒΠ»Ρƒ подсчСта ΠΎΠ±Ρ‰Π΅ΠΉ проводимости:

ΠΈΠ»ΠΈ

g = g1 + g2 + g3.

Π’Π°ΠΊΠΈΠΌ ΠΎΠ±Ρ€Π°Π·ΠΎΠΌ, ΠΏΡ€ΠΈ ΠΏΠ°Ρ€Π°Π»Π»Π΅Π»ΡŒΠ½ΠΎΠΌ соСдинСнии увСличиваСтся Π½Π΅ сопротивлСниС, Π° ΠΏΡ€ΠΎΠ²ΠΎΠ΄ΠΈΠΌΠΎΡΡ‚ΡŒ.

ΠŸΡ€ΠΈΠΌΠ΅Ρ€ 3. ΠžΠΏΡ€Π΅Π΄Π΅Π»ΠΈΡ‚ΡŒ ΠΎΠ±Ρ‰Π΅Π΅ сопротивлСниС Ρ‚Ρ€Π΅Ρ… ΠΏΠ°Ρ€Π°Π»Π»Π΅Π»ΡŒΠ½ΠΎ Π²ΠΊΠ»ΡŽΡ‡Π΅Π½Π½Ρ‹Ρ… сопротивлСний, Ссли r1 = 2 Ом, r2 = 3 Ом, r3 = 4 Ом.

ΠΎΡ‚ΠΊΡƒΠ΄Π°

ΠŸΡ€ΠΈΠΌΠ΅Ρ€ 4. ΠŸΡΡ‚ΡŒ сопротивлСний 20, 30 ,15, 40 ΠΈ 60 Ом Π²ΠΊΠ»ΡŽΡ‡Π΅Π½Ρ‹ ΠΏΠ°Ρ€Π°Π»Π»Π΅Π»ΡŒΠ½ΠΎ Π² ΡΠ΅Ρ‚ΡŒ. ΠžΠΏΡ€Π΅Π΄Π΅Π»ΠΈΡ‚ΡŒ ΠΎΠ±Ρ‰Π΅Π΅ сопротивлСниС:

ΠΎΡ‚ΠΊΡƒΠ΄Π°

Π‘Π»Π΅Π΄ΡƒΠ΅Ρ‚ Π·Π°ΠΌΠ΅Ρ‚ΠΈΡ‚ΡŒ, Ρ‡Ρ‚ΠΎ ΠΏΡ€ΠΈ подсчСтС ΠΎΠ±Ρ‰Π΅Π³ΠΎ сопротивлСния развСтвлСния ΠΎΠ½ΠΎ получаСтся всСгда мСньшС, Ρ‡Π΅ΠΌ самоС мСньшСС сопротивлСниС, входящСС Π² Ρ€Π°Π·Π²Π΅Ρ‚Π²Π»Π΅Π½ΠΈΠ΅.

Если сопротивлСния, Π²ΠΊΠ»ΡŽΡ‡Π΅Π½Π½Ρ‹Π΅ ΠΏΠ°Ρ€Π°Π»Π»Π΅Π»ΡŒΠ½ΠΎ, Ρ€Π°Π²Π½Ρ‹ ΠΌΠ΅ΠΆΠ΄Ρƒ собой, Ρ‚ΠΎ ΠΎΠ±Ρ‰Π΅Π΅ сопротивлСниС r Ρ†Π΅ΠΏΠΈ Ρ€Π°Π²Π½ΠΎ ΡΠΎΠΏΡ€ΠΎΡ‚ΠΈΠ²Π»Π΅Π½ΠΈΡŽ ΠΎΠ΄Π½ΠΎΠΉ Π²Π΅Ρ‚Π²ΠΈ r1, Π΄Π΅Π»Π΅Π½Π½ΠΎΠΌΡƒ Π½Π° число Π²Π΅Ρ‚Π²Π΅ΠΉ n:

ΠŸΡ€ΠΈΠΌΠ΅Ρ€ 5. ΠžΠΏΡ€Π΅Π΄Π΅Π»ΠΈΡ‚ΡŒ ΠΎΠ±Ρ‰Π΅Π΅ сопротивлСниС Ρ‡Π΅Ρ‚Ρ‹Ρ€Π΅Ρ… ΠΏΠ°Ρ€Π°Π»Π»Π΅Π»ΡŒΠ½ΠΎ Π²ΠΊΠ»ΡŽΡ‡Π΅Π½Π½Ρ‹Ρ… сопротивлСний ΠΏΠΎ 20 Ом ΠΊΠ°ΠΆΠ΄ΠΎΠ΅:

Для ΠΏΡ€ΠΎΠ²Π΅Ρ€ΠΊΠΈ ΠΏΠΎΠΏΡ€ΠΎΠ±ΡƒΠ΅ΠΌ Π½Π°ΠΉΡ‚ΠΈ сопротивлСниС развСтвлСния ΠΏΠΎ Ρ„ΠΎΡ€ΠΌΡƒΠ»Π΅:

ΠΎΡ‚ΠΊΡƒΠ΄Π°

Как Π²ΠΈΠ΄ΠΈΠΌ, ΠΎΡ‚Π²Π΅Ρ‚ получаСтся Ρ‚ΠΎΡ‚ ΠΆΠ΅.

ΠŸΡ€ΠΈΠΌΠ΅Ρ€ 6. ΠŸΡƒΡΡ‚ΡŒ трСбуСтся ΠΎΠΏΡ€Π΅Π΄Π΅Π»ΠΈΡ‚ΡŒ Ρ‚ΠΎΠΊΠΈ Π² ΠΊΠ°ΠΆΠ΄ΠΎΠΉ Π²Π΅Ρ‚Π²ΠΈ ΠΏΡ€ΠΈ ΠΏΠ°Ρ€Π°Π»Π»Π΅Π»ΡŒΠ½ΠΎΠΌ ΠΈΡ… соСдинСнии, ΠΈΠ·ΠΎΠ±Ρ€Π°ΠΆΠ΅Π½Π½ΠΎΠΌ Π½Π° рисункС 5, Π°.

Рисунок 5. К ΠΏΡ€ΠΈΠΌΠ΅Ρ€Ρƒ 6

НайдСм ΠΎΠ±Ρ‰Π΅Π΅ сопротивлСниС Ρ†Π΅ΠΏΠΈ:

ΠΎΡ‚ΠΊΡƒΠ΄Π°

Π’Π΅ΠΏΠ΅Ρ€ΡŒ всС развСтвлСния ΠΌΡ‹ ΠΌΠΎΠΆΠ΅ΠΌ ΠΈΠ·ΠΎΠ±Ρ€Π°Π·ΠΈΡ‚ΡŒ ΡƒΠΏΡ€ΠΎΡ‰Π΅Π½Π½ΠΎ ΠΊΠ°ΠΊ ΠΎΠ΄Π½ΠΎ сопротивлСниС (рисунок 5, Π±).

ПадСниС напряТСния Π½Π° участкС ΠΌΠ΅ΠΆΠ΄Ρƒ Ρ‚ΠΎΡ‡ΠΊΠ°ΠΌΠΈ А ΠΈ Π‘ Π±ΡƒΠ΄Π΅Ρ‚:

U = I Γ— r = 22 Γ— 1,09 = 24 Π’.

Π’ΠΎΠ·Π²Ρ€Π°Ρ‰Π°ΡΡΡŒ снова ΠΊ рисунку 5, Π° Π²ΠΈΠ΄ΠΈΠΌ, Ρ‡Ρ‚ΠΎ всС Ρ‚Ρ€ΠΈ сопротивлСния окаТутся ΠΏΠΎΠ΄ напряТСниСм 24 Π’, Ρ‚Π°ΠΊ ΠΊΠ°ΠΊ ΠΎΠ½ΠΈ Π²ΠΊΠ»ΡŽΡ‡Π΅Π½Ρ‹ ΠΌΠ΅ΠΆΠ΄Ρƒ Ρ‚ΠΎΡ‡ΠΊΠ°ΠΌΠΈ А ΠΈ Π‘.

Рассматривая ΠΏΠ΅Ρ€Π²ΡƒΡŽ Π²Π΅Ρ‚Π²ΡŒ развСтвлСния с сопротивлСниСм r1, ΠΌΡ‹ Π²ΠΈΠ΄ΠΈΠΌ, Ρ‡Ρ‚ΠΎ напряТСниС Π½Π° этом участкС 24 Π’, сопротивлСниС участка 2 Ом. По Π·Π°ΠΊΠΎΠ½Ρƒ Ома для участка Ρ†Π΅ΠΏΠΈ Ρ‚ΠΎΠΊ Π½Π° этом участкС Π±ΡƒΠ΄Π΅Ρ‚:

Π’ΠΎΠΊ Π²Ρ‚ΠΎΡ€ΠΎΠΉ Π²Π΅Ρ‚Π²ΠΈ

Π’ΠΎΠΊ Ρ‚Ρ€Π΅Ρ‚ΡŒΠ΅ΠΉ Π²Π΅Ρ‚Π²ΠΈ

ΠŸΡ€ΠΎΠ²Π΅Ρ€ΠΈΠΌ ΠΏΠΎ ΠΏΠ΅Ρ€Π²ΠΎΠΌΡƒ Π·Π°ΠΊΠΎΠ½Ρƒ ΠšΠΈΡ€Ρ…Π³ΠΎΡ„Π°

I = I1 + I2 + I3 = 12 + 6 + 4 = 22 А.

Π‘Π»Π΅Π΄ΠΎΠ²Π°Ρ‚Π΅Π»ΡŒΠ½ΠΎ, Π·Π°Π΄Π°Ρ‡Π° Ρ€Π΅ΡˆΠ΅Π½Π° Π²Π΅Ρ€Π½ΠΎ.

ΠžΠ±Ρ€Π°Ρ‚ΠΈΠΌ Π²Π½ΠΈΠΌΠ°Π½ΠΈΠ΅ Π½Π° Ρ‚ΠΎ, ΠΊΠ°ΠΊ Ρ€Π°ΡΠΏΡ€Π΅Π΄Π΅Π»ΡΡŽΡ‚ΡΡ Ρ‚ΠΎΠΊΠΈ Π² вСтвях нашСго ΠΏΠ°Ρ€Π°Π»Π»Π΅Π»ΡŒΠ½ΠΎΠ³ΠΎ соСдинСния. ΠŸΠ΅Ρ€Π²Π°Ρ Π²Π΅Ρ‚Π²ΡŒ: r1 = 2 Ом, I1 = 12 А

Вторая Π²Π΅Ρ‚Π²ΡŒ: r2 = 4 Ом, I2 = 6 А. Π’Ρ€Π΅Ρ‚ΡŒΡ Π²Π΅Ρ‚Π²ΡŒ: r3 = 6 Ом, I3 = 4 А

ΠŸΠ΅Ρ€Π²Π°Ρ Π²Π΅Ρ‚Π²ΡŒ: r1 = 2 Ом, I1 = 12 А. Вторая Π²Π΅Ρ‚Π²ΡŒ: r2 = 4 Ом, I2 = 6 А. Π’Ρ€Π΅Ρ‚ΡŒΡ Π²Π΅Ρ‚Π²ΡŒ: r3 = 6 Ом, I3 = 4 А.

Как Π²ΠΈΠ΄ΠΈΠΌ, сопротивлСниС ΠΏΠ΅Ρ€Π²ΠΎΠΉ Π²Π΅Ρ‚Π²ΠΈ Π² Π΄Π²Π° Ρ€Π°Π·Π° мСньшС сопротивлСниС Π²Ρ‚ΠΎΡ€ΠΎΠΉ Π²Π΅Ρ‚Π²ΠΈ, Π° Ρ‚ΠΎΠΊ ΠΏΠ΅Ρ€Π²ΠΎΠΉ Π²Π΅Ρ‚Π²ΠΈ Π² Π΄Π²Π° Ρ€Π°Π·Π° большС Ρ‚ΠΎΠΊΠ° Π²Ρ‚ΠΎΡ€ΠΎΠΉ Π²Π΅Ρ‚Π²ΠΈ. Π‘ΠΎΠΏΡ€ΠΎΡ‚ΠΈΠ²Π»Π΅Π½ΠΈΠ΅ Ρ‚Ρ€Π΅Ρ‚ΡŒΠ΅ΠΉ Π²Π΅Ρ‚Π²ΠΈ Π² Ρ‚Ρ€ΠΈ Ρ€Π°Π·Π° большС сопротивлСния ΠΏΠ΅Ρ€Π²ΠΎΠΉ Π²Π΅Ρ‚Π²ΠΈ, Π° Ρ‚ΠΎΠΊ Ρ‚Ρ€Π΅Ρ‚ΡŒΠ΅ΠΉ Π²Π΅Ρ‚Π²ΠΈ Π² Ρ‚Ρ€ΠΈ Ρ€Π°Π·Π° мСньшС Ρ‚ΠΎΠΊΠ° ΠΏΠ΅Ρ€Π²ΠΎΠΉ Π²Π΅Ρ‚Π²ΠΈ. ΠžΡ‚ΡΡŽΠ΄Π° ΠΌΠΎΠΆΠ½ΠΎ ΡΠ΄Π΅Π»Π°Ρ‚ΡŒ Π²Ρ‹Π²ΠΎΠ΄, Ρ‡Ρ‚ΠΎ Ρ‚ΠΎΠΊΠΈ Π² вСтвях ΠΏΡ€ΠΈ ΠΏΠ°Ρ€Π°Π»Π»Π΅Π»ΡŒΠ½ΠΎΠΌ соСдинСнии Ρ€Π°ΡΠΏΡ€Π΅Π΄Π΅Π»ΡΡŽΡ‚ΡΡ ΠΎΠ±Ρ€Π°Ρ‚Π½ΠΎ ΠΏΡ€ΠΎΠΏΠΎΡ€Ρ†ΠΈΠΎΠ½Π°Π»ΡŒΠ½ΠΎ сопротивлСниям этих Π²Π΅Ρ‚Π²Π΅ΠΉ. Π’Π°ΠΊΠΈΠΌ ΠΎΠ±Ρ€Π°Π·ΠΎΠΌ, ΠΏΠΎ Π²Π΅Ρ‚Π²ΠΈ с большим сопротивлСниСм ΠΏΠΎΡ‚Π΅Ρ‡Π΅Ρ‚ Ρ‚ΠΎΠΊ мСньший, Ρ‡Π΅ΠΌ ΠΏΠΎ Π²Π΅Ρ‚Π²ΠΈ с ΠΌΠ°Π»Ρ‹ΠΌ сопротивлСниСм.

Для Π΄Π²ΡƒΡ… ΠΏΠ°Ρ€Π°Π»Π»Π΅Π»ΡŒΠ½Ρ‹Ρ… Π²Π΅Ρ‚Π²Π΅ΠΉ ΠΌΠΎΠΆΠ½ΠΎ Ρ‚Π°ΠΊΠΆΠ΅, ΠΊΠΎΠ½Π΅Ρ‡Π½ΠΎ, ΠΏΠΎΠ»ΡŒΠ·ΠΎΠ²Π°Ρ‚ΡŒΡΡ Π΄Π°Π½Π½ΠΎΠΉ Π²Ρ‹ΡˆΠ΅ Ρ„ΠΎΡ€ΠΌΡƒΠ»ΠΎΠΉ.

Однако ΠΎΠ±Ρ‰Π΅Π΅ сопротивлСниС ΠΏΡ€ΠΎΠ²ΠΎΠ΄Π½ΠΈΠΊΠ° ΠΏΡ€ΠΈ ΠΏΠ°Ρ€Π°Π»Π»Π΅Π»ΡŒΠ½ΠΎΠΌ соСдинСнии Π² этом случаС Π»Π΅Π³Ρ‡Π΅ ΠΏΠΎΠ΄ΡΡ‡ΠΈΡ‚Π°Ρ‚ΡŒ ΠΏΠΎ Ρ„ΠΎΡ€ΠΌΡƒΠ»Π΅:

ΠΈΠ»ΠΈ ΠΎΠΊΠΎΠ½Ρ‡Π°Ρ‚Π΅Π»ΡŒΠ½ΠΎ:

ΠŸΠΎΡΠ»Π΅Π΄ΠΎΠ²Π°Ρ‚Π΅Π»ΡŒΠ½ΠΎΠ΅ соСдинСниС Π»Π°ΠΌΠΏ накаливания.

ΠŸΠΎΡΠ»Π΅Π΄ΠΎΠ²Π°Ρ‚Π΅Π»ΡŒΠ½ΠΎΠ΅ соСдинСниС Π»Π°ΠΌΠΏ накаливания Π² домашнСм Π±Ρ‹Ρ‚Ρƒ ΠΈΡΠΏΠΎΠ»ΡŒΠ·ΡƒΠ΅Ρ‚ΡΡ Ρ€Π΅Π΄ΠΊΠΎ. Π’ своС врСмя я ΠΏΠΎΠ΄ΠΊΠ»ΡŽΡ‡Π°Π» Π΄Π²Π΅ Π»Π°ΠΌΠΏΡ‹ ΠΏΠΎΡΠ»Π΅Π΄ΠΎΠ²Π°Ρ‚Π΅Π»ΡŒΠ½ΠΎ Ρƒ сСбя Π² подъСздС, Π½ΠΎ это Π±Ρ‹Π» Π΅Π΄ΠΈΠ½ΠΈΡ‡Π½Ρ‹ΠΉ случай.

Π’ΡƒΡ‚ ситуация Π±Ρ‹Π»Π° такая, Ρ‡Ρ‚ΠΎ подъСздная Π»Π°ΠΌΠΏΠ° ΠΏΠ΅Ρ€Π΅Π³ΠΎΡ€Π°Π»Π° с ΠΏΠ΅Ρ€ΠΈΠΎΠ΄ΠΈΡ‡Π½ΠΎΡΡ‚ΡŒΡŽ Π² ΠΎΠ΄ΠΈΠ½ мСсяц, ΠΈ Π½Π°Π΄ΠΎ Π±Ρ‹Π»ΠΎ Ρ‡Ρ‚ΠΎ-Ρ‚ΠΎ Π΄Π΅Π»Π°Ρ‚ΡŒ.

ΠžΠ±Ρ‹Ρ‡Π½ΠΎ, Π² Ρ‚Π°ΠΊΠΈΡ… случаях Π»Π°ΠΌΠΏΡƒ Π²ΠΊΠ»ΡŽΡ‡Π°ΡŽΡ‚ Ρ‡Π΅Ρ€Π΅Π· Π΄ΠΈΠΎΠ΄, Ρ‡Ρ‚ΠΎΠ±Ρ‹ ΠΎΠ½Π° ΠΏΠΈΡ‚Π°Π»Π°ΡΡŒ ΠΏΠΎΠ½ΠΈΠΆΠ΅Π½Π½Ρ‹ΠΌ напряТСниСм 110Π’ ΠΈ Π΄ΠΎΠ»Π³ΠΎ слуТила. Π’Π°Ρ€ΠΈΠ°Π½Ρ‚ ΠΏΡ€ΠΎΠ²Π΅Ρ€Π΅Π½Π½Ρ‹ΠΉ, Π½ΠΎ ΠΏΡ€ΠΈ этом сама Π»Π°ΠΌΠΏΠ° ΠΌΠ΅Ρ€Ρ†Π°Π΅Ρ‚, Π΄Π° ΠΈ свСтит Π² ΠΏΠΎΠ»Π½Π°ΠΊΠ°Π»Π°.

Когда ΠΆΠ΅ стоят Π΄Π²Π΅ ΠΏΠΎΡΠ»Π΅Π΄ΠΎΠ²Π°Ρ‚Π΅Π»ΡŒΠ½ΠΎ, Ρ‚ΠΎ ΠΎΠ½ΠΈ Ρ‚Π°ΠΊ ΠΆΠ΅ ΠΏΠΈΡ‚Π°ΡŽΡ‚ΡΡ ΠΏΠΎΠ½ΠΈΠΆΠ΅Π½Π½Ρ‹ΠΌ напряТСниСм 110Π’, Π½Π΅ ΠΌΠ΅Ρ€Ρ†Π°ΡŽΡ‚, Π΄ΠΎΠ»Π³ΠΎ слуТат, свСтят ΠΈ ΠΏΠΎΡ‚Ρ€Π΅Π±Π»ΡΡŽΡ‚ энСргии ΠΊΠ°ΠΊ ΠΎΠ΄Π½Π°. ΠŸΡ€ΠΈΡ‡Π΅ΠΌ ΠΈΡ… ΠΌΠΎΠΆΠ½ΠΎ развСсти ΠΏΠΎ Ρ€Π°Π·Π½Ρ‹ΠΌ ΡƒΠ³Π»Π°ΠΌ помСщСния, Ρ‡Ρ‚ΠΎ Ρ‚ΠΎΠΆΠ΅ плюс.Но ΠΏΠΎΠ²Ρ‚ΠΎΡ€ΡŽΡΡŒ – это Ρ€Π΅Π΄ΠΊΠΈΠΉ случай.

ΠŸΠΎΡΠΌΠΎΡ‚Ρ€ΠΈΡ‚Π΅ Π½Π° рисунок Π½ΠΈΠΆΠ΅. Π—Π΄Π΅ΡΡŒ ΠΈΠ·ΠΎΠ±Ρ€Π°ΠΆΠ΅Π½Ρ‹ Π΄Π²Π΅ схСмы ΠΏΠΎΡΠ»Π΅Π΄ΠΎΠ²Π°Ρ‚Π΅Π»ΡŒΠ½ΠΎΠ³ΠΎ соСдинСния Π»Π°ΠΌΠΏ накаливания. Π’ Π²Π΅Ρ€Ρ…Π½Π΅ΠΉ части рисунка ΠΏΠΎΠΊΠ°Π·Π°Π½Π° ΠΏΡ€ΠΈΠ½Ρ†ΠΈΠΏΠΈΠ°Π»ΡŒΠ½Π°Ρ схСма, Π° Π² Π½ΠΈΠΆΠ½Π΅ΠΉ части – монтаТная. ΠŸΡ€ΠΈΡ‡Π΅ΠΌ для Π»ΡƒΡ‡ΡˆΠ΅Π³ΠΎ восприятия, монтаТная схСма ΠΏΠΎΠΊΠ°Π·Π°Π½Π° с Ρ€Π΅Π°Π»ΡŒΠ½Ρ‹ΠΌ ΠΈΠ·ΠΎΠ±Ρ€Π°ΠΆΠ΅Π½ΠΈΠ΅ΠΌ Π»Π°ΠΌΠΏ ΠΈ Π΄Π²ΡƒΠΆΠΈΠ»ΡŒΠ½ΠΎΠ³ΠΎ ΠΏΡ€ΠΎΠ²ΠΎΠ΄Π°.

Π—Π΄Π΅ΡΡŒ Π² Π»ΠΈΠ½ΠΈΠΈ ΠΊΠΎΡ€ΠΈΡ‡Π½Π΅Π²ΠΎΠ³ΠΎ Ρ†Π²Π΅Ρ‚Π°, Π»Π°ΠΌΠΏΡ‹ HL1 ΠΈ HL2 соСдинСны ΠΏΠΎΡΠ»Π΅Π΄ΠΎΠ²Π°Ρ‚Π΅Π»ΡŒΠ½ΠΎ – ΠΎΠ΄Π½Π° Π·Π° Π΄Ρ€ΡƒΠ³ΠΎΠΉ. ΠŸΠΎΡΡ‚ΠΎΠΌΡƒ Ρ‚Π°ΠΊΠΎΠ΅ соСдинСниС Π½Π°Π·Ρ‹Π²Π°ΡŽΡ‚ ΠΏΠΎΡΠ»Π΅Π΄ΠΎΠ²Π°Ρ‚Π΅Π»ΡŒΠ½Ρ‹ΠΌ.

Если ΠΏΠΎΠ΄Π°Ρ‚ΡŒ напряТСниС питания 220Π’ Π½Π° ΠΊΠΎΠ½Ρ†Ρ‹ L ΠΈ N, Ρ‚ΠΎ загорятся ΠΎΠ±Π΅ Π»Π°ΠΌΠΏΡ‹, Π½ΠΎ Π³ΠΎΡ€Π΅Ρ‚ΡŒ ΠΎΠ½ΠΈ Π±ΡƒΠ΄ΡƒΡ‚ Π½Π΅ Π² ΠΏΠΎΠ»Π½ΡƒΡŽ силу, Π° Π² ΠΏΠΎΠ»ΠΎΠ²ΠΈΠ½Ρƒ Π½Π°ΠΊΠ°Π»Π°. Π’Π°ΠΊ ΠΊΠ°ΠΊ сопротивлСниС Π½ΠΈΡ‚Π΅ΠΉ Π»Π°ΠΌΠΏ рассчитано Π½Π° ΠΏΠΈΡ‚Π°ΡŽΡ‰Π΅Π΅ напряТСниС 220Π’, ΠΈ ΠΊΠΎΠ³Π΄Π° ΠΎΠ½ΠΈ стоят Π² Ρ†Π΅ΠΏΠΈ ΠΏΠΎΡΠ»Π΅Π΄ΠΎΠ²Π°Ρ‚Π΅Π»ΡŒΠ½ΠΎ, ΠΎΠ΄Π½Π° Π·Π° Π΄Ρ€ΡƒΠ³ΠΎΠΉ, Ρ‚ΠΎ Π·Π° счСт добавлСния сопротивлСния Π½ΠΈΡ‚ΠΈ Π½Π°ΠΊΠ°Π»Π° ΡΠ»Π΅Π΄ΡƒΡŽΡ‰Π΅ΠΉ Π»Π°ΠΌΠΏΡ‹, ΠΎΠ±Ρ‰Π΅Π΅ сопротивлСниС Ρ†Π΅ΠΏΠΈ Π±ΡƒΠ΄Π΅Ρ‚ ΡƒΠ²Π΅Π»ΠΈΡ‡ΠΈΠ²Π°Ρ‚ΡŒΡΡ, Π° Π·Π½Π°Ρ‡ΠΈΡ‚, для ΡΠ»Π΅Π΄ΡƒΡŽΡ‰Π΅ΠΉ Π»Π°ΠΌΠΏΡ‹ напряТСниС всСгда Π±ΡƒΠ΄Π΅Ρ‚ мСньшС согласно Π·Π°ΠΊΠΎΠ½Ρƒ Ома.

ΠŸΠΎΡΡ‚ΠΎΠΌΡƒ ΠΏΡ€ΠΈ ΠΏΠΎΡΠ»Π΅Π΄ΠΎΠ²Π°Ρ‚Π΅Π»ΡŒΠ½ΠΎΠΌ соСдинСнии Π΄Π²ΡƒΡ… Π»Π°ΠΌΠΏ напряТСниС 220Π’ Π±ΡƒΠ΄Π΅Ρ‚ Π΄Π΅Π»ΠΈΡ‚ΡŒΡΡ ΠΏΠΎΠΏΠΎΠ»Π°ΠΌ, ΠΈ составит 110Π’ для ΠΊΠ°ΠΆΠ΄ΠΎΠΉ.

На ΡΠ»Π΅Π΄ΡƒΡŽΡ‰Π΅ΠΌ рисункС ΠΏΠΎΠΊΠ°Π·Π°Π½Ρ‹ Ρ‚Ρ€ΠΈ Π»Π°ΠΌΠΏΡ‹ соСдинСнныС ΠΏΠΎΡΠ»Π΅Π΄ΠΎΠ²Π°Ρ‚Π΅Π»ΡŒΠ½ΠΎ.

На этой схСмС напряТСниС Π½Π° ΠΊΠ°ΠΆΠ΄ΠΎΠΉ Π»Π°ΠΌΠΏΠ΅ составит ΠΎΠΊΠΎΠ»ΠΎ 73 Π’ΠΎΠ»ΡŒΡ‚, Ρ‚Π°ΠΊ ΠΊΠ°ΠΊ Π±ΡƒΠ΄Π΅Ρ‚ Π΄Π΅Π»ΠΈΡ‚ΡŒΡΡ ΡƒΠΆΠ΅ ΠΌΠ΅ΠΆΠ΄Ρƒ трСмя Π»Π°ΠΌΠΏΠ°ΠΌΠΈ.

Π’Π°ΠΊ ΠΆΠ΅ ΠΏΡ€ΠΈΠΌΠ΅Ρ€ΠΎΠΌ ΠΏΠΎΡΠ»Π΅Π΄ΠΎΠ²Π°Ρ‚Π΅Π»ΡŒΠ½ΠΎΠ³ΠΎ соСдинСния ΠΌΠΎΠ³ΡƒΡ‚ ΡΠ»ΡƒΠΆΠΈΡ‚ΡŒ Π½ΠΎΠ²ΠΎΠ³ΠΎΠ΄Π½ΠΈΠ΅ гирлянды. Π—Π΄Π΅ΡΡŒ ΠΈΠ· ΠΌΠΈΠ½ΠΈΠ°Ρ‚ΡŽΡ€Π½Ρ‹Ρ… Π»Π°ΠΌΠΏΠΎΡ‡Π΅ΠΊ с Π½ΠΈΠ·ΠΊΠΈΠΌ ΠΏΠΈΡ‚Π°Π½ΠΈΠ΅ΠΌ создаСтся ΠΎΠ΄Π½Π° Π»Π°ΠΌΠΏΠ° Π½Π° напряТСниС 220Π’.

НапримСр, Π±Π΅Ρ€Π΅ΠΌ Π»Π°ΠΌΠΏΠΎΡ‡ΠΊΠΈ, рассчитанныС Π½Π° 6,3 Π’ΠΎΠ»ΡŒΡ‚Π° ΠΈ Π΄Π΅Π»ΠΈΠΌ ΠΈΡ… Π½Π° 220 Π’ΠΎΠ»ΡŒΡ‚. ΠŸΠΎΠ»ΡƒΡ‡Π°Π΅Ρ‚ΡΡ 35 ΡˆΡ‚ΡƒΠΊ. Π’ΠΎ Π΅ΡΡ‚ΡŒ, Ρ‡Ρ‚ΠΎΠ±Ρ‹ ΡΠ΄Π΅Π»Π°Ρ‚ΡŒ ΠΎΠ΄Π½Ρƒ Π»Π°ΠΌΠΏΡƒ Π½Π° напряТСниС 220Π’, Π½Π°ΠΌ Π½ΡƒΠΆΠ½ΠΎ ΡΠΎΠ΅Π΄ΠΈΠ½ΠΈΡ‚ΡŒ ΠΏΠΎΡΠ»Π΅Π΄ΠΎΠ²Π°Ρ‚Π΅Π»ΡŒΠ½ΠΎ 35 ΡˆΡ‚ΡƒΠΊ с напряТСниСм питания 6,3 Π’ΠΎΠ»ΡŒΡ‚Π°.

P.S. Π’Π°ΠΊ ΠΊΠ°ΠΊ напряТСниС Π² сСти Π½Π΅ постоянно, Ρ‚ΠΎ расчСт Π»ΡƒΡ‡ΡˆΠ΅ ΠΏΡ€ΠΎΠΈΠ·Π²ΠΎΠ΄ΠΈΡ‚ΡŒ исходя ΠΈΠ· 245 – 250 Π’ΠΎΠ»ΡŒΡ‚.

Как Π’Ρ‹ Π·Π½Π°Π΅Ρ‚Π΅, Ρƒ гирлянд Π΅ΡΡ‚ΡŒ ΠΎΠ΄ΠΈΠ½ нСдостаток. ΠŸΠ΅Ρ€Π΅Π³ΠΎΡ€Π°Π΅Ρ‚ ΠΎΠ΄Π½Π° ΠΈΠ· Π»Π°ΠΌΠΏ, Π½Π°ΠΏΡ€ΠΈΠΌΠ΅Ρ€, ΠΊΠ°Π½Π°Π»Π° Π·Π΅Π»Π΅Π½ΠΎΠ³ΠΎ Ρ†Π²Π΅Ρ‚Π°, Π·Π½Π°Ρ‡ΠΈΡ‚, Π½Π΅ Π³ΠΎΡ€ΠΈΡ‚ ΠΊΠ°Π½Π°Π» Π·Π΅Π»Π΅Π½ΠΎΠ³ΠΎ Ρ†Π²Π΅Ρ‚Π°. Π’ΠΎΠ³Π΄Π° ΠΌΡ‹ ΠΈΠ΄Π΅ΠΌ Π½Π° Π±Π°Π·Π°Ρ€, ΠΏΠΎΠΊΡƒΠΏΠ°Π΅ΠΌ Π»Π°ΠΌΠΏΠΎΡ‡ΠΊΠΈ Π·Π΅Π»Π΅Π½ΠΎΠ³ΠΎ Ρ†Π²Π΅Ρ‚Π°, Π° ΠΏΠΎΡ‚ΠΎΠΌ Π΄ΠΎΠΌΠ° ΠΏΠΎ ΠΎΠ΄Π½ΠΎΠΉ Π²Ρ‹Π½ΠΈΠΌΠ°Π΅ΠΌ, вставляСм Π½ΠΎΠ²ΡƒΡŽ, ΠΈ ΠΏΠΎΠΊΠ° Π½Π΅ Π·Π°Ρ€Π°Π±ΠΎΡ‚Π°Π΅Ρ‚ ΠΊΠ°Π½Π°Π», ΠΏΠ΅Ρ€Π΅Π±ΠΈΡ€Π°Π΅ΠΌ Π΅Π³ΠΎ вСсь.

Π’Ρ‹Π²ΠΎΠ΄:

НСдостатком ΠΏΠΎΡΠ»Π΅Π΄ΠΎΠ²Π°Ρ‚Π΅Π»ΡŒΠ½ΠΎΠ³ΠΎ соСдинСния являСтся Ρ‚ΠΎ, Ρ‡Ρ‚ΠΎ Ссли Π²Ρ‹ΠΉΠ΄Π΅Ρ‚ ΠΈΠ· строя Ρ…ΠΎΡ‚ΡŒ ΠΎΠ΄Π½Π° ΠΈΠ· Π»Π°ΠΌΠΏ, Π³ΠΎΡ€Π΅Ρ‚ΡŒ Π½Π΅ Π±ΡƒΠ΄ΡƒΡ‚ всС, Ρ‚Π°ΠΊ ΠΊΠ°ΠΊ Π½Π°Ρ€ΡƒΡˆΠ°Π΅Ρ‚ΡΡ элСктричСская Ρ†Π΅ΠΏΡŒ.

А Π²Ρ‚ΠΎΡ€Ρ‹ΠΌ нСдостатком, ΠΊΠ°ΠΊ Π’Ρ‹ ΡƒΠΆΠ΅ догадались, являСтся слабоС свСчСниС. ΠŸΠΎΡΡ‚ΠΎΠΌΡƒ ΠΏΠΎΡΠ»Π΅Π΄ΠΎΠ²Π°Ρ‚Π΅Π»ΡŒΠ½ΠΎΠ΅ соСдинСниС Π»Π°ΠΌΠΏ накаливания Π½Π° напряТСниС 220Π’ Π² Π΄ΠΎΠΌΠ°ΡˆΠ½ΠΈΡ… условиях практичСски Π½Π΅ примСняСтся.

ΠŸΠ΅Ρ€Π²Ρ‹ΠΉ Π·Π°ΠΊΠΎΠ½ ΠšΠΈΡ€Ρ…Π³ΠΎΡ„Π°

Как я ΡƒΠΆΠ΅ ΡƒΠΏΠΎΠΌΠΈΠ½Π°Π», Π·Π°ΠΊΠΎΠ½Ρ‹ ΠšΠΈΡ€Ρ…Π³ΠΎΡ„Π° вмСстС с Π·Π°ΠΊΠΎΠ½ΠΎΠΌ Ома ΡΠ²Π»ΡΡŽΡ‚ΡΡ основными ΠΏΡ€ΠΈ Π°Π½Π°Π»ΠΈΠ·Π΅ ΠΈ расчётах элСктричСских Ρ†Π΅ΠΏΠ΅ΠΉ. Π—Π°ΠΊΠΎΠ½ Ома Π±Ρ‹Π» ΠΏΠΎΠ΄Ρ€ΠΎΠ±Π½ΠΎ рассмотрСн Π² Π΄Π²ΡƒΡ… ΠΏΡ€Π΅Π΄Ρ‹Π΄ΡƒΡ‰ΠΈΡ… ΡΡ‚Π°Ρ‚ΡŒΡΡ…, Ρ‚Π΅ΠΏΠ΅Ρ€ΡŒ настала ΠΎΡ‡Π΅Ρ€Π΅Π΄ΡŒ для Π·Π°ΠΊΠΎΠ½ΠΎΠ² ΠšΠΈΡ€Ρ…Π³ΠΎΡ„Π°. Π˜Ρ… всСго Π΄Π²Π°, ΠΏΠ΅Ρ€Π²Ρ‹ΠΉ описываСт ΡΠΎΠΎΡ‚Π½ΠΎΡˆΠ΅Π½ΠΈΡ Ρ‚ΠΎΠΊΠΎΠ² Π² элСктричСских цСпях, Π° Π²Ρ‚ΠΎΡ€ΠΎΠΉ – ΡΠΎΠΎΡ‚Π½ΠΎΡˆΠ΅Π½ΠΈΠ΅ Π­Π”Π‘ ΠΈ напряТСниями Π² ΠΊΠΎΠ½Ρ‚ΡƒΡ€Π΅. Начнём с ΠΏΠ΅Ρ€Π²ΠΎΠ³ΠΎ.

ΠŸΠ΅Ρ€Π²Ρ‹ΠΉ Π·Π°ΠΊΠΎΠ½ ΠšΠΈΡ€Ρ…Π³ΠΎΡ„Π° гласит, Ρ‡Ρ‚ΠΎ алгСбраичСская сумма Ρ‚ΠΎΠΊΠΎΠ² Π² ΡƒΠ·Π»Π΅ Ρ€Π°Π²Π½Π° Π½ΡƒΠ»ΡŽ. ΠžΠΏΠΈΡΡ‹Π²Π°Π΅Ρ‚ΡΡ это ΡΠ»Π΅Π΄ΡƒΡŽΡ‰ΠΈΠΌ Π²Ρ‹Ρ€Π°ΠΆΠ΅Π½ΠΈΠ΅ΠΌ

Π³Π΄Π΅ βˆ‘ β€” ΠΎΠ±ΠΎΠ·Π½Π°Ρ‡Π°Π΅Ρ‚ Π°Π»Π³Π΅Π±Ρ€Π°ΠΈΡ‡Π΅ΡΠΊΡƒΡŽ сумму.

Π‘Π»ΠΎΠ²ΠΎ «алгСбраичСская» ΠΎΠ·Π½Π°Ρ‡Π°Π΅Ρ‚, Ρ‡Ρ‚ΠΎ Ρ‚ΠΎΠΊΠΈ Π½Π΅ΠΎΠ±Ρ…ΠΎΠ΄ΠΈΠΌΠΎ Π±Ρ€Π°Ρ‚ΡŒ с ΡƒΡ‡Ρ‘Ρ‚ΠΎΠΌ Π·Π½Π°ΠΊΠ°, Ρ‚ΠΎ Π΅ΡΡ‚ΡŒ направлСния втСкания. Π’Π°ΠΊΠΈΠΌ ΠΎΠ±Ρ€Π°Π·ΠΎΠΌ, всСм Ρ‚ΠΎΠΊΠ°ΠΌ, ΠΊΠΎΡ‚ΠΎΡ€Ρ‹Π΅ Π²Ρ‚Π΅ΠΊΠ°ΡŽΡ‚ Π² ΡƒΠ·Π΅Π», присваиваСтся ΠΏΠΎΠ»ΠΎΠΆΠΈΡ‚Π΅Π»ΡŒΠ½Ρ‹ΠΉ Π·Π½Π°ΠΊ, Π° ΠΊΠΎΡ‚ΠΎΡ€Ρ‹Π΅ Π²Ρ‹Ρ‚Π΅ΠΊΠ°ΡŽΡ‚ ΠΈΠ· ΡƒΠ·Π»Π° – соотвСтствСнно ΠΎΡ‚Ρ€ΠΈΡ†Π°Ρ‚Π΅Π»ΡŒΠ½Ρ‹ΠΉ. Рисунок Π½ΠΈΠΆΠ΅ ΠΈΠ»Π»ΡŽΡΡ‚Ρ€ΠΈΡ€ΡƒΠ΅Ρ‚ ΠΏΠ΅Ρ€Π²Ρ‹ΠΉ Π·Π°ΠΊΠΎΠ½ ΠšΠΈΡ€Ρ…Π³ΠΎΡ„Π°

На рисункС ΠΈΠ·ΠΎΠ±Ρ€Π°ΠΆΠ΅Π½ ΡƒΠ·Π΅Π», Π² ΠΊΠΎΡ‚ΠΎΡ€Ρ‹ΠΉ со стороны сопротивлСния R1 Π²Ρ‚Π΅ΠΊΠ°Π΅Ρ‚ Ρ‚ΠΎΠΊ, Π° со стороны сопротивлСний R2, R3, R4 соотвСтствСнно Π²Ρ‹Ρ‚Π΅ΠΊΠ°Π΅Ρ‚ Ρ‚ΠΎΠΊ, Ρ‚ΠΎΠ³Π΄Π° ΡƒΡ€Π°Π²Π½Π΅Π½ΠΈΠ΅ Ρ‚ΠΎΠΊΠΎΠ² для Π΄Π°Π½Π½ΠΎΠ³ΠΎ участка Ρ†Π΅ΠΏΠΈ Π±ΡƒΠ΄Π΅Ρ‚ ΠΈΠΌΠ΅Ρ‚ΡŒ Π²ΠΈΠ΄

ΠŸΠ΅Ρ€Π²Ρ‹ΠΉ Π·Π°ΠΊΠΎΠ½ ΠšΠΈΡ€Ρ…Π³ΠΎΡ„Π° примСняСтся Π½Π΅ Ρ‚ΠΎΠ»ΡŒΠΊΠΎ ΠΊ ΡƒΠ·Π»Π°ΠΌ, Π½ΠΎ ΠΈ ΠΊ Π»ΡŽΠ±ΠΎΠΌΡƒ ΠΊΠΎΠ½Ρ‚ΡƒΡ€Ρƒ ΠΈΠ»ΠΈ части элСктричСской Ρ†Π΅ΠΏΠΈ. НапримСр, ΠΊΠΎΠ³Π΄Π° я Π³ΠΎΠ²ΠΎΡ€ΠΈΠ» ΠΎ ΠΏΠ°Ρ€Π°Π»Π»Π΅Π»ΡŒΠ½ΠΎΠΌ соСдинСнии ΠΏΡ€ΠΈΠ΅ΠΌΠ½ΠΈΠΊΠΎΠ² энСргии, Π³Π΄Π΅ сумма Ρ‚ΠΎΠΊΠΎΠ² Ρ‡Π΅Ρ€Π΅Π· R1, R2 ΠΈ R3 Ρ€Π°Π²Π½Π° Π²Ρ‚Π΅ΠΊΠ°ΡŽΡ‰Π΅ΠΌΡƒ Ρ‚ΠΎΠΊΡƒ I.

ΠŸΡ€ΠΈΠΌΠ΅Ρ€Ρ‹ использования

  • Π‘Π°Ρ‚Π°Ρ€Π΅ΠΈ Π³Π°Π»ΡŒΠ²Π°Π½ΠΈΡ‡Π΅ΡΠΊΠΈΡ… элСмСнтов ΠΈΠ»ΠΈ аккумуляторов, Π² ΠΊΠΎΡ‚ΠΎΡ€Ρ‹Ρ… ΠΎΡ‚Π΄Π΅Π»ΡŒΠ½Ρ‹Π΅ химичСскиС источники Ρ‚ΠΎΠΊΠ° соСдинСны ΠΏΠΎΡΠ»Π΅Π΄ΠΎΠ²Π°Ρ‚Π΅Π»ΡŒΠ½ΠΎ (для увСличСния напряТСния) ΠΈΠ»ΠΈ ΠΏΠ°Ρ€Π°Π»Π»Π΅Π»ΡŒΠ½ΠΎ (для увСличСния Ρ‚ΠΎΠΊΠ°).
  • Π Π΅Π³ΡƒΠ»ΠΈΡ€ΠΎΠ²ΠΊΠ° мощности элСктричСского устройства, состоящСго ΠΈΠ· Π½Π΅ΡΠΊΠΎΠ»ΡŒΠΊΠΈΡ… ΠΎΠ΄ΠΈΠ½Π°ΠΊΠΎΠ²Ρ‹Ρ… ΠΏΠΎΡ‚Ρ€Π΅Π±ΠΈΡ‚Π΅Π»Π΅ΠΉ элСктроэнСргии, ΠΏΡƒΡ‚Ρ‘ΠΌ ΠΈΡ… ΠΏΠ΅Ρ€Π΅ΠΊΠ»ΡŽΡ‡Π΅Π½ΠΈΡ с ΠΏΠ°Ρ€Π°Π»Π»Π΅Π»ΡŒΠ½ΠΎΠ³ΠΎ Π½Π° ΠΏΠΎΡΠ»Π΅Π΄ΠΎΠ²Π°Ρ‚Π΅Π»ΡŒΠ½ΠΎΠ΅ соСдинСниС. Π’Π°ΠΊΠΈΠΌ способом рСгулируСтся ΠΌΠΎΡ‰Π½ΠΎΡΡ‚ΡŒ ΠΊΠΎΠ½Ρ„ΠΎΡ€ΠΊΠΈ элСктричСской ΠΏΠ»ΠΈΡ‚Ρ‹, состоящСй ΠΈΠ· Π½Π΅ΡΠΊΠΎΠ»ΡŒΠΊΠΈΡ… спиралСй; ΠΌΠΎΡ‰Π½ΠΎΡΡ‚ΡŒ (ΡΠΊΠΎΡ€ΠΎΡΡ‚ΡŒ двиТСния) элСктровоза, ΠΈΠΌΠ΅ΡŽΡ‰Π΅Π³ΠΎ нСсколько тяговых Π΄Π²ΠΈΠ³Π°Ρ‚Π΅Π»Π΅ΠΉ.
  • Π”Π΅Π»ΠΈΡ‚Π΅Π»ΡŒ напряТСния
  • Балласт
  • Π¨ΡƒΠ½Ρ‚

Какой способ Π»ΡƒΡ‡ΡˆΠ΅?

ΠœΠ΅Ρ‚ΠΎΠ΄ Β«ΡˆΠ»Π΅ΠΉΡ„ΠΎΠ²Β» Π½Π΅ слишком ΡƒΠ΄ΠΎΠ±Π΅Π½ Ρ‚ΠΎΠ»ΡŒΠΊΠΎ Ρ‚Π΅ΠΌ, Ρ‡Ρ‚ΠΎ любой ΠΏΠΎΡ‚Ρ€Π΅Π±ΠΈΡ‚Π΅Π»ΡŒ ΠΏΠΎ Ρ†Π΅ΠΏΠΈ зависит ΠΎΡ‚ ΠΏΡ€Π΅Π΄Ρ‹Π΄ΡƒΡ‰Π΅Π³ΠΎ. НапримСр, Ссли ΠΏΡ€ΠΎΠΈΠ·ΠΎΠΉΠ΄Ρ‘Ρ‚ ΠΎΠ±Ρ€Ρ‹Π² ΠΏΡ€ΠΎΠ²ΠΎΠ΄Π° Π½Π° Π²Ρ‚ΠΎΡ€ΠΎΠΉ Ρ€ΠΎΠ·Π΅Ρ‚ΠΊΠ΅, Ρ‚ΠΎ Ρ‚Ρ€Π΅Ρ‚ΡŒΡ ΠΈ чСтвёртая Ρ‚Π°ΠΊΠΆΠ΅ останутся Π±Π΅Π· напряТСния. Но ΠΏΡ€ΠΈ этом нСльзя Π½Π΅ Π²Ρ‹Π΄Π΅Π»ΠΈΡ‚ΡŒ экономию ΠΏΡ€ΠΎΠ²ΠΎΠ΄Π½ΠΈΠΊΠ° ΠΏΡ€ΠΈ Π½Π°Ρ‡Π°Π»ΡŒΠ½ΠΎΠΌ ΠΌΠΎΠ½Ρ‚Π°ΠΆΠ΅ элСктропроводки.

Рисунок 3: ΠšΠΎΠΌΠ±ΠΈΠ½ΠΈΡ€ΠΎΠ²Π°Π½Π½ΠΎΠ΅ соСдинСниС Ρ€ΠΎΠ·Π΅Ρ‚ΠΎΠΊ

К Ρ‚ΠΎΠΌΡƒ ΠΆΠ΅, Β«ΡˆΠ»Π΅ΠΉΡ„ΠΎΠΌΒ» ΠΎΡ‡Π΅Π½ΡŒ ΡƒΠ΄ΠΎΠ±Π½ΠΎ ΠΏΡ€ΠΎΠ²ΠΎΠ΄ΠΈΡ‚ΡŒ Π»ΠΈΠ½ΠΈΠΈ, ΠΊΠΎΠ³Π΄Π° Π½Π΅ΠΎΠ±Ρ…ΠΎΠ΄ΠΈΠΌΠΎ ΠΌΠΈΠ½ΠΈΠΌΠΈΠ·ΠΈΡ€ΠΎΠ²Π°Ρ‚ΡŒ количСство ΡˆΡ‚Ρ€ΠΎΠ± Π² стСнах. А Π΄Π΅Π»Π°ΡŽΡ‚ это ΠΏΡ€ΠΈ ΠΌΠΎΠ½Ρ‚Π°ΠΆΠ΅ ΠΏΡ€ΠΎΠ²ΠΎΠ΄ΠΊΠΈ ΠΏΠΎ ΠΏΠΎΠ»Ρƒ ΠΈΠ»ΠΈ ΠΏΠΎΡ‚ΠΎΠ»ΠΊΡƒ, Π² ΡΠΏΠ΅Ρ†ΠΈΠ°Π»ΡŒΠ½ΠΎΠΉ Π³ΠΎΡ„Ρ€ΠΈΡ€ΠΎΠ²Π°Π½Π½ΠΎΠΉ Ρ‚Ρ€ΡƒΠ±Π΅. Π’ΠΎΠ³Π΄Π° остаётся провСсти Ρ‚ΠΎΠ»ΡŒΠΊΠΎ основныС ΡˆΡ‚Ρ€ΠΎΠ±Ρ‹ ΠΊ Ρ€ΠΎΠ·Π΅Ρ‚ΠΊΠ°ΠΌ ΠΈ ΠΌΠ΅ΠΆΠ΄Ρƒ Π½ΠΈΠΌΠΈ.

Π’Ρ‹Π²ΠΎΠ΄: ΠΏΡ€ΠΎΠΊΠ»Π°Π΄ΠΊΠ° элСктропроводки Β«ΡˆΠ»Π΅ΠΉΡ„ΠΎΠΌΒ» ΡƒΠ΄ΠΎΠ±Π½Π° ΠΈ экономична, Π½Π΅ Π·Π°Π½ΠΈΠΌΠ°Π΅Ρ‚ ΠΌΠ½ΠΎΠ³ΠΎ Π²Ρ€Π΅ΠΌΠ΅Π½ΠΈ Π² процСссС ΠΌΠΎΠ½Ρ‚Π°ΠΆΠ°, ΠΈΠΌΠ΅Π΅Ρ‚ Π΄Π»ΠΈΡ‚Π΅Π»ΡŒΠ½Ρ‹ΠΉ эксплуатационный срок ΠΈ совсСм Π½Π΅Π·Π½Π°Ρ‡ΠΈΡ‚Π΅Π»ΡŒΠ½Ρ‹Π΅ нСдостатки, ΠΊΠΎΡ‚ΠΎΡ€Ρ‹Π΅ ΠΌΠΎΠΆΠ½ΠΎ ΠΎΡΡ‚Π°Π²ΠΈΡ‚ΡŒ Π±Π΅Π· внимания.

ΠŸΠΎΡΠ»Π΅Π΄ΠΎΠ²Π°Ρ‚Π΅Π»ΡŒΠ½ΠΎΠ΅ соСдинСниС Π»Π°ΠΌΠΏ накаливания.

ΠŸΠΎΡΠ»Π΅Π΄ΠΎΠ²Π°Ρ‚Π΅Π»ΡŒΠ½ΠΎΠ΅ соСдинСниС Π»Π°ΠΌΠΏ накаливания Π² домашнСм Π±Ρ‹Ρ‚Ρƒ ΠΈΡΠΏΠΎΠ»ΡŒΠ·ΡƒΠ΅Ρ‚ΡΡ Ρ€Π΅Π΄ΠΊΠΎ. Π’ своС врСмя я ΠΏΠΎΠ΄ΠΊΠ»ΡŽΡ‡Π°Π» Π΄Π²Π΅ Π»Π°ΠΌΠΏΡ‹ ΠΏΠΎΡΠ»Π΅Π΄ΠΎΠ²Π°Ρ‚Π΅Π»ΡŒΠ½ΠΎ Ρƒ сСбя Π² подъСздС, Π½ΠΎ это Π±Ρ‹Π» Π΅Π΄ΠΈΠ½ΠΈΡ‡Π½Ρ‹ΠΉ случай.

Π’ΡƒΡ‚ ситуация Π±Ρ‹Π»Π° такая, Ρ‡Ρ‚ΠΎ подъСздная Π»Π°ΠΌΠΏΠ° ΠΏΠ΅Ρ€Π΅Π³ΠΎΡ€Π°Π»Π° с ΠΏΠ΅Ρ€ΠΈΠΎΠ΄ΠΈΡ‡Π½ΠΎΡΡ‚ΡŒΡŽ Π² ΠΎΠ΄ΠΈΠ½ мСсяц, ΠΈ Π½Π°Π΄ΠΎ Π±Ρ‹Π»ΠΎ Ρ‡Ρ‚ΠΎ-Ρ‚ΠΎ Π΄Π΅Π»Π°Ρ‚ΡŒ.

ΠžΠ±Ρ‹Ρ‡Π½ΠΎ, Π² Ρ‚Π°ΠΊΠΈΡ… случаях Π»Π°ΠΌΠΏΡƒ Π²ΠΊΠ»ΡŽΡ‡Π°ΡŽΡ‚ Ρ‡Π΅Ρ€Π΅Π· Π΄ΠΈΠΎΠ΄, Ρ‡Ρ‚ΠΎΠ±Ρ‹ ΠΎΠ½Π° ΠΏΠΈΡ‚Π°Π»Π°ΡΡŒ ΠΏΠΎΠ½ΠΈΠΆΠ΅Π½Π½Ρ‹ΠΌ напряТСниСм 110Π’ ΠΈ Π΄ΠΎΠ»Π³ΠΎ слуТила. Π’Π°Ρ€ΠΈΠ°Π½Ρ‚ ΠΏΡ€ΠΎΠ²Π΅Ρ€Π΅Π½Π½Ρ‹ΠΉ, Π½ΠΎ ΠΏΡ€ΠΈ этом сама Π»Π°ΠΌΠΏΠ° ΠΌΠ΅Ρ€Ρ†Π°Π΅Ρ‚, Π΄Π° ΠΈ свСтит Π² ΠΏΠΎΠ»Π½Π°ΠΊΠ°Π»Π°.

Когда ΠΆΠ΅ стоят Π΄Π²Π΅ ΠΏΠΎΡΠ»Π΅Π΄ΠΎΠ²Π°Ρ‚Π΅Π»ΡŒΠ½ΠΎ, Ρ‚ΠΎ ΠΎΠ½ΠΈ Ρ‚Π°ΠΊ ΠΆΠ΅ ΠΏΠΈΡ‚Π°ΡŽΡ‚ΡΡ ΠΏΠΎΠ½ΠΈΠΆΠ΅Π½Π½Ρ‹ΠΌ напряТСниСм 110Π’, Π½Π΅ ΠΌΠ΅Ρ€Ρ†Π°ΡŽΡ‚, Π΄ΠΎΠ»Π³ΠΎ слуТат, свСтят ΠΈ ΠΏΠΎΡ‚Ρ€Π΅Π±Π»ΡΡŽΡ‚ энСргии ΠΊΠ°ΠΊ ΠΎΠ΄Π½Π°. ΠŸΡ€ΠΈΡ‡Π΅ΠΌ ΠΈΡ… ΠΌΠΎΠΆΠ½ΠΎ развСсти ΠΏΠΎ Ρ€Π°Π·Π½Ρ‹ΠΌ ΡƒΠ³Π»Π°ΠΌ помСщСния, Ρ‡Ρ‚ΠΎ Ρ‚ΠΎΠΆΠ΅ плюс.Но ΠΏΠΎΠ²Ρ‚ΠΎΡ€ΡŽΡΡŒ – это Ρ€Π΅Π΄ΠΊΠΈΠΉ случай.

ΠŸΠΎΡΠΌΠΎΡ‚Ρ€ΠΈΡ‚Π΅ Π½Π° рисунок Π½ΠΈΠΆΠ΅. Π—Π΄Π΅ΡΡŒ ΠΈΠ·ΠΎΠ±Ρ€Π°ΠΆΠ΅Π½Ρ‹ Π΄Π²Π΅ схСмы ΠΏΠΎΡΠ»Π΅Π΄ΠΎΠ²Π°Ρ‚Π΅Π»ΡŒΠ½ΠΎΠ³ΠΎ соСдинСния Π»Π°ΠΌΠΏ накаливания. Π’ Π²Π΅Ρ€Ρ…Π½Π΅ΠΉ части рисунка ΠΏΠΎΠΊΠ°Π·Π°Π½Π° ΠΏΡ€ΠΈΠ½Ρ†ΠΈΠΏΠΈΠ°Π»ΡŒΠ½Π°Ρ схСма, Π° Π² Π½ΠΈΠΆΠ½Π΅ΠΉ части – монтаТная. ΠŸΡ€ΠΈΡ‡Π΅ΠΌ для Π»ΡƒΡ‡ΡˆΠ΅Π³ΠΎ восприятия, монтаТная схСма ΠΏΠΎΠΊΠ°Π·Π°Π½Π° с Ρ€Π΅Π°Π»ΡŒΠ½Ρ‹ΠΌ ΠΈΠ·ΠΎΠ±Ρ€Π°ΠΆΠ΅Π½ΠΈΠ΅ΠΌ Π»Π°ΠΌΠΏ ΠΈ Π΄Π²ΡƒΠΆΠΈΠ»ΡŒΠ½ΠΎΠ³ΠΎ ΠΏΡ€ΠΎΠ²ΠΎΠ΄Π°.

Π—Π΄Π΅ΡΡŒ Π² Π»ΠΈΠ½ΠΈΠΈ ΠΊΠΎΡ€ΠΈΡ‡Π½Π΅Π²ΠΎΠ³ΠΎ Ρ†Π²Π΅Ρ‚Π°, Π»Π°ΠΌΠΏΡ‹ HL1 ΠΈ HL2 соСдинСны ΠΏΠΎΡΠ»Π΅Π΄ΠΎΠ²Π°Ρ‚Π΅Π»ΡŒΠ½ΠΎ – ΠΎΠ΄Π½Π° Π·Π° Π΄Ρ€ΡƒΠ³ΠΎΠΉ. ΠŸΠΎΡΡ‚ΠΎΠΌΡƒ Ρ‚Π°ΠΊΠΎΠ΅ соСдинСниС Π½Π°Π·Ρ‹Π²Π°ΡŽΡ‚ ΠΏΠΎΡΠ»Π΅Π΄ΠΎΠ²Π°Ρ‚Π΅Π»ΡŒΠ½Ρ‹ΠΌ.

Если ΠΏΠΎΠ΄Π°Ρ‚ΡŒ напряТСниС питания 220Π’ Π½Π° ΠΊΠΎΠ½Ρ†Ρ‹ L ΠΈ N, Ρ‚ΠΎ загорятся ΠΎΠ±Π΅ Π»Π°ΠΌΠΏΡ‹, Π½ΠΎ Π³ΠΎΡ€Π΅Ρ‚ΡŒ ΠΎΠ½ΠΈ Π±ΡƒΠ΄ΡƒΡ‚ Π½Π΅ Π² ΠΏΠΎΠ»Π½ΡƒΡŽ силу, Π° Π² ΠΏΠΎΠ»ΠΎΠ²ΠΈΠ½Ρƒ Π½Π°ΠΊΠ°Π»Π°. Π’Π°ΠΊ ΠΊΠ°ΠΊ сопротивлСниС Π½ΠΈΡ‚Π΅ΠΉ Π»Π°ΠΌΠΏ рассчитано Π½Π° ΠΏΠΈΡ‚Π°ΡŽΡ‰Π΅Π΅ напряТСниС 220Π’, ΠΈ ΠΊΠΎΠ³Π΄Π° ΠΎΠ½ΠΈ стоят Π² Ρ†Π΅ΠΏΠΈ ΠΏΠΎΡΠ»Π΅Π΄ΠΎΠ²Π°Ρ‚Π΅Π»ΡŒΠ½ΠΎ, ΠΎΠ΄Π½Π° Π·Π° Π΄Ρ€ΡƒΠ³ΠΎΠΉ, Ρ‚ΠΎ Π·Π° счСт добавлСния сопротивлСния Π½ΠΈΡ‚ΠΈ Π½Π°ΠΊΠ°Π»Π° ΡΠ»Π΅Π΄ΡƒΡŽΡ‰Π΅ΠΉ Π»Π°ΠΌΠΏΡ‹, ΠΎΠ±Ρ‰Π΅Π΅ сопротивлСниС Ρ†Π΅ΠΏΠΈ Π±ΡƒΠ΄Π΅Ρ‚ ΡƒΠ²Π΅Π»ΠΈΡ‡ΠΈΠ²Π°Ρ‚ΡŒΡΡ, Π° Π·Π½Π°Ρ‡ΠΈΡ‚, для ΡΠ»Π΅Π΄ΡƒΡŽΡ‰Π΅ΠΉ Π»Π°ΠΌΠΏΡ‹ напряТСниС всСгда Π±ΡƒΠ΄Π΅Ρ‚ мСньшС согласно Π·Π°ΠΊΠΎΠ½Ρƒ Ома.

ΠŸΠΎΡΡ‚ΠΎΠΌΡƒ ΠΏΡ€ΠΈ ΠΏΠΎΡΠ»Π΅Π΄ΠΎΠ²Π°Ρ‚Π΅Π»ΡŒΠ½ΠΎΠΌ соСдинСнии Π΄Π²ΡƒΡ… Π»Π°ΠΌΠΏ напряТСниС 220Π’ Π±ΡƒΠ΄Π΅Ρ‚ Π΄Π΅Π»ΠΈΡ‚ΡŒΡΡ ΠΏΠΎΠΏΠΎΠ»Π°ΠΌ, ΠΈ составит 110Π’ для ΠΊΠ°ΠΆΠ΄ΠΎΠΉ.

На ΡΠ»Π΅Π΄ΡƒΡŽΡ‰Π΅ΠΌ рисункС ΠΏΠΎΠΊΠ°Π·Π°Π½Ρ‹ Ρ‚Ρ€ΠΈ Π»Π°ΠΌΠΏΡ‹ соСдинСнныС ΠΏΠΎΡΠ»Π΅Π΄ΠΎΠ²Π°Ρ‚Π΅Π»ΡŒΠ½ΠΎ.

На этой схСмС напряТСниС Π½Π° ΠΊΠ°ΠΆΠ΄ΠΎΠΉ Π»Π°ΠΌΠΏΠ΅ составит ΠΎΠΊΠΎΠ»ΠΎ 73 Π’ΠΎΠ»ΡŒΡ‚, Ρ‚Π°ΠΊ ΠΊΠ°ΠΊ Π±ΡƒΠ΄Π΅Ρ‚ Π΄Π΅Π»ΠΈΡ‚ΡŒΡΡ ΡƒΠΆΠ΅ ΠΌΠ΅ΠΆΠ΄Ρƒ трСмя Π»Π°ΠΌΠΏΠ°ΠΌΠΈ.

Π’Π°ΠΊ ΠΆΠ΅ ΠΏΡ€ΠΈΠΌΠ΅Ρ€ΠΎΠΌ ΠΏΠΎΡΠ»Π΅Π΄ΠΎΠ²Π°Ρ‚Π΅Π»ΡŒΠ½ΠΎΠ³ΠΎ соСдинСния ΠΌΠΎΠ³ΡƒΡ‚ ΡΠ»ΡƒΠΆΠΈΡ‚ΡŒ Π½ΠΎΠ²ΠΎΠ³ΠΎΠ΄Π½ΠΈΠ΅ гирлянды. Π—Π΄Π΅ΡΡŒ ΠΈΠ· ΠΌΠΈΠ½ΠΈΠ°Ρ‚ΡŽΡ€Π½Ρ‹Ρ… Π»Π°ΠΌΠΏΠΎΡ‡Π΅ΠΊ с Π½ΠΈΠ·ΠΊΠΈΠΌ ΠΏΠΈΡ‚Π°Π½ΠΈΠ΅ΠΌ создаСтся ΠΎΠ΄Π½Π° Π»Π°ΠΌΠΏΠ° Π½Π° напряТСниС 220Π’.

НапримСр, Π±Π΅Ρ€Π΅ΠΌ Π»Π°ΠΌΠΏΠΎΡ‡ΠΊΠΈ, рассчитанныС Π½Π° 6,3 Π’ΠΎΠ»ΡŒΡ‚Π° ΠΈ Π΄Π΅Π»ΠΈΠΌ ΠΈΡ… Π½Π° 220 Π’ΠΎΠ»ΡŒΡ‚. ΠŸΠΎΠ»ΡƒΡ‡Π°Π΅Ρ‚ΡΡ 35 ΡˆΡ‚ΡƒΠΊ. Π’ΠΎ Π΅ΡΡ‚ΡŒ, Ρ‡Ρ‚ΠΎΠ±Ρ‹ ΡΠ΄Π΅Π»Π°Ρ‚ΡŒ ΠΎΠ΄Π½Ρƒ Π»Π°ΠΌΠΏΡƒ Π½Π° напряТСниС 220Π’, Π½Π°ΠΌ Π½ΡƒΠΆΠ½ΠΎ ΡΠΎΠ΅Π΄ΠΈΠ½ΠΈΡ‚ΡŒ ΠΏΠΎΡΠ»Π΅Π΄ΠΎΠ²Π°Ρ‚Π΅Π»ΡŒΠ½ΠΎ 35 ΡˆΡ‚ΡƒΠΊ с напряТСниСм питания 6,3 Π’ΠΎΠ»ΡŒΡ‚Π°.

P.S. Π’Π°ΠΊ ΠΊΠ°ΠΊ напряТСниС Π² сСти Π½Π΅ постоянно, Ρ‚ΠΎ расчСт Π»ΡƒΡ‡ΡˆΠ΅ ΠΏΡ€ΠΎΠΈΠ·Π²ΠΎΠ΄ΠΈΡ‚ΡŒ исходя ΠΈΠ· 245 – 250 Π’ΠΎΠ»ΡŒΡ‚.

Как Π’Ρ‹ Π·Π½Π°Π΅Ρ‚Π΅, Ρƒ гирлянд Π΅ΡΡ‚ΡŒ ΠΎΠ΄ΠΈΠ½ нСдостаток. ΠŸΠ΅Ρ€Π΅Π³ΠΎΡ€Π°Π΅Ρ‚ ΠΎΠ΄Π½Π° ΠΈΠ· Π»Π°ΠΌΠΏ, Π½Π°ΠΏΡ€ΠΈΠΌΠ΅Ρ€, ΠΊΠ°Π½Π°Π»Π° Π·Π΅Π»Π΅Π½ΠΎΠ³ΠΎ Ρ†Π²Π΅Ρ‚Π°, Π·Π½Π°Ρ‡ΠΈΡ‚, Π½Π΅ Π³ΠΎΡ€ΠΈΡ‚ ΠΊΠ°Π½Π°Π» Π·Π΅Π»Π΅Π½ΠΎΠ³ΠΎ Ρ†Π²Π΅Ρ‚Π°. Π’ΠΎΠ³Π΄Π° ΠΌΡ‹ ΠΈΠ΄Π΅ΠΌ Π½Π° Π±Π°Π·Π°Ρ€, ΠΏΠΎΠΊΡƒΠΏΠ°Π΅ΠΌ Π»Π°ΠΌΠΏΠΎΡ‡ΠΊΠΈ Π·Π΅Π»Π΅Π½ΠΎΠ³ΠΎ Ρ†Π²Π΅Ρ‚Π°, Π° ΠΏΠΎΡ‚ΠΎΠΌ Π΄ΠΎΠΌΠ° ΠΏΠΎ ΠΎΠ΄Π½ΠΎΠΉ Π²Ρ‹Π½ΠΈΠΌΠ°Π΅ΠΌ, вставляСм Π½ΠΎΠ²ΡƒΡŽ, ΠΈ ΠΏΠΎΠΊΠ° Π½Π΅ Π·Π°Ρ€Π°Π±ΠΎΡ‚Π°Π΅Ρ‚ ΠΊΠ°Π½Π°Π», ΠΏΠ΅Ρ€Π΅Π±ΠΈΡ€Π°Π΅ΠΌ Π΅Π³ΠΎ вСсь.

Π’Ρ‹Π²ΠΎΠ΄:

НСдостатком ΠΏΠΎΡΠ»Π΅Π΄ΠΎΠ²Π°Ρ‚Π΅Π»ΡŒΠ½ΠΎΠ³ΠΎ соСдинСния являСтся Ρ‚ΠΎ, Ρ‡Ρ‚ΠΎ Ссли Π²Ρ‹ΠΉΠ΄Π΅Ρ‚ ΠΈΠ· строя Ρ…ΠΎΡ‚ΡŒ ΠΎΠ΄Π½Π° ΠΈΠ· Π»Π°ΠΌΠΏ, Π³ΠΎΡ€Π΅Ρ‚ΡŒ Π½Π΅ Π±ΡƒΠ΄ΡƒΡ‚ всС, Ρ‚Π°ΠΊ ΠΊΠ°ΠΊ Π½Π°Ρ€ΡƒΡˆΠ°Π΅Ρ‚ΡΡ элСктричСская Ρ†Π΅ΠΏΡŒ.

А Π²Ρ‚ΠΎΡ€Ρ‹ΠΌ нСдостатком, ΠΊΠ°ΠΊ Π’Ρ‹ ΡƒΠΆΠ΅ догадались, являСтся слабоС свСчСниС. ΠŸΠΎΡΡ‚ΠΎΠΌΡƒ ΠΏΠΎΡΠ»Π΅Π΄ΠΎΠ²Π°Ρ‚Π΅Π»ΡŒΠ½ΠΎΠ΅ соСдинСниС Π»Π°ΠΌΠΏ накаливания Π½Π° напряТСниС 220Π’ Π² Π΄ΠΎΠΌΠ°ΡˆΠ½ΠΈΡ… условиях практичСски Π½Π΅ примСняСтся.

Как выглядит Ρ„ΠΎΡ€ΠΌΡƒΠ»Π° Π“Π΅ΠΎΡ€Π³Π° Ома

ΠŸΡ€ΠΈΠΌΠ΅Ρ€ΠΎΠΌ Ρ‚Π°ΠΊΠΎΠ³ΠΎ Ρ‚ΠΈΠΏΠ° ΠΏΠΎΠ΄ΠΊΠ»ΡŽΡ‡Π΅Π½ΠΈΡ рСзисторов ΠΌΠΎΠΆΠ΅Ρ‚ Π±Ρ‹Ρ‚ΡŒ соСдинСниС Ρ†Π΅ΠΏΠΈ ΠΏΠΎΡ‚Ρ€Π΅Π±ΠΈΡ‚Π΅Π»Π΅ΠΉ элСктроэнСргии Π²Β ΠΌΠ½ΠΎΠ³ΠΎΠΊΠ²Π°Ρ€Ρ‚ΠΈΡ€Π½ΠΎΠΌ Π΄ΠΎΠΌΠ΅. Π’Π°ΠΊ, свСтодиоды, ΠΎΡ‚ΠΎΠΏΠΈΡ‚Π΅Π»ΡŒΠ½Ρ‹ΠΉ Ρ€Π°Π΄ΠΈΠ°Ρ‚ΠΎΡ€, ΠΌΠΈΠΊΡ€ΠΎΠ²ΠΎΠ»Π½ΠΎΠ²ΠΊΠ° ΠΈΒ Π΄Ρ€ΡƒΠ³ΠΈΠ΅ ΠΏΡ€ΠΈΠ±ΠΎΡ€Ρ‹ установлСны Π²Β Ρ†Π΅ΠΏΠΈ ΠΏΠ°Ρ€Π°Π»Π»Π΅Π»ΡŒΠ½ΠΎ.

Π’ΠΎΠ»ΡŒΡ‚ΠΌΠ΅Ρ‚Ρ€, ΠΊΠΎΡ‚ΠΎΡ€Ρ‹ΠΉ ΠΏΠΎΠ΄ΠΊΠ»ΡŽΡ‡Π°ΡŽΡ‚ Π²Β Ρ†Π΅ΠΏΡŒ, Π±ΡƒΠ΄Π΅Ρ‚ ΠΏΠΎΠΊΠ°Π·Ρ‹Π²Π°Ρ‚ΡŒ напряТСниС на всСх рСзисторах. Π’ΠΎΠ³Π΄Π° ΠΎΠ½ΠΎ Π²Π΅Π·Π΄Π΅ Π±ΡƒΠ΄Π΅Ρ‚ Ρ€Π°Π²Π½Ρ‹ΠΌ ΠΈΒ Ρ„ΠΎΡ€ΠΌΡƒΠ»Ρƒ ΠΌΠΎΠΆΠ½ΠΎ Π·Π°ΠΏΠΈΡΠ°Ρ‚ΡŒ ΠΊΠ°ΠΊ:

U1 = U2 = U.

Π‘Ρ…Π΅ΠΌΠ° ΠΏΠ°Ρ€Π°Π»Π»Π΅Π»ΡŒΠ½ΠΎΠ³ΠΎ соСдинСния

Когда ΠΎΠ±Ρ€Π°Π·ΡƒΡŽΡ‚ΡΡ Π²Π΅Ρ‚Π²ΠΈ ΠΏΡ€ΠΈ ΠΏΠΎΠ΄ΠΊΠ»ΡŽΡ‡Π΅Π½ΠΈΠΈ, Ρ‚ΠΎΒ Ρ‡Π°ΡΡ‚ΡŒ ΠΎΠ±Ρ‰Π΅Π³ΠΎ напряТСния ΠΏΡ€ΠΎΡ…ΠΎΠ΄ΠΈΡ‚ Ρ‡Π΅Ρ€Π΅Π· ΠΏΠ΅Ρ€Π²Ρ‹ΠΉ рСзистор, Π°Β Ρ‡Π°ΡΡ‚ΡŒΒ β€” Ρ‡Π΅Ρ€Π΅Π· Π²Ρ‚ΠΎΡ€ΠΎΠΉ ΠΈΒ Ρ‚Π°ΠΊ Π΄Π°Π»Π΅Π΅. ΠŸΠΎΡΡ‚ΠΎΠΌΡƒ ΠΏΡ€ΠΈ Ρ‚Π°ΠΊΠΎΠΌ Π²ΠΈΠ΄Π΅ соСдинСния рСзисторов FΡ‚ΠΎΠΊΠ° Π²Β Π½Π΅Ρ€Π°Π·Π²Π΅Ρ‚Π²Π»Ρ‘Π½Π½ΠΎΠΉ Ρ‚ΠΎΡ‡ΠΊΠ΅ Π±ΡƒΠ΄Π΅Ρ‚ Ρ€Π°Π²Π½ΡΡ‚ΡŒΡΡ суммарной FΡ‚ΠΎΠΊΠ° Π²Β ΠΎΡ‚Π΄Π΅Π»ΡŒΠ½Ρ‹Ρ… рСзисторах и записываСтся ΠΊΠ°ΠΊ:

I = I1 + I2.

РасчСт силы Ρ‚ΠΎΠΊΠ° ΠΏΡ€ΠΈ ΠΏΠΎΠΌΠΎΡ‰ΠΈ Π·Π°ΠΊΠΎΠ½Π° Ома записываСтся ΠΊΠ°ΠΊ:

I = U/R;

I1 = U1/R1;

I2 = U2/R2.

Π˜Π·Β Ρ„ΠΎΡ€ΠΌΡƒΠ»Ρ‹ слСдуСт:

U/R = U1/R1 + U2/R2;

U = U1 = U2;

1/R = 1/R1 + 1/R2.

Дословно ΠΏΡ€Π°Π²ΠΈΠ»ΠΎ Π·Π²ΡƒΡ‡ΠΈΡ‚ Ρ‚Π°ΠΊ: число, ΠΎΠ±Ρ€Π°Ρ‚Π½ΠΎΠ΅ ΠΎΠ±Ρ‰Π΅ΠΌΡƒ ΡΠΎΠΏΡ€ΠΎΡ‚ΠΈΠ²Π»Π΅Π½ΠΈΡŽ ΠΏΡ€ΠΈ ΠΏΠ°Ρ€Π°Π»Π»Π΅Π»ΡŒΠ½ΠΎΠΌ ΠΏΠΎΠ΄ΠΊΠ»ΡŽΡ‡Π΅Π½ΠΈΠΈ, Π±ΡƒΠ΄Π΅Ρ‚ суммарно Ρ€Π°Π²Π½ΠΎ числу ΠΎΠ±Ρ€Π°Ρ‚Π½ΠΎΠ³ΠΎ сопротивлСния.

Π—Π°Π²ΠΈΡΠΈΠΌΠΎΡΡ‚ΡŒ сопротивлСния

Π—Π½Π°Ρ‡Π΅Π½ΠΈΠ΅ элСктропроводимости зависит ΠΎΡ‚ Π½Π΅ΡΠΊΠΎΠ»ΡŒΠΊΠΈΡ… Ρ„Π°ΠΊΡ‚ΠΎΡ€ΠΎΠ², ΠΊΠΎΡ‚ΠΎΡ€Ρ‹Π΅ Π½Π΅ΠΎΠ±Ρ…ΠΎΠ΄ΠΈΠΌΠΎ ΡƒΡ‡ΠΈΡ‚Ρ‹Π²Π°Ρ‚ΡŒ ΠΏΡ€ΠΈ расчСтах, ΠΈΠ·Π³ΠΎΡ‚ΠΎΠ²Π»Π΅Π½ΠΈΠΈ элСмСнтов рСзистивной Π½Π°Π³Ρ€ΡƒΠ·ΠΊΠΈ (рСзисторов), Ρ€Π΅ΠΌΠΎΠ½Ρ‚Π΅ ΠΈ ΠΏΡ€ΠΎΠ΅ΠΊΡ‚ΠΈΡ€ΠΎΠ²Π°Π½ΠΈΠΈ устройств. К этим Ρ„Π°ΠΊΡ‚ΠΎΡ€Π°ΠΌ Π½Π΅ΠΎΠ±Ρ…ΠΎΠ΄ΠΈΠΌΠΎ отнСсти ΡΠ»Π΅Π΄ΡƒΡŽΡ‰ΠΈΠ΅:

  1. Π’Π΅ΠΌΠΏΠ΅Ρ€Π°Ρ‚ΡƒΡ€Π° ΠΎΠΊΡ€ΡƒΠΆΠ°ΡŽΡ‰Π΅ΠΉ срСды ΠΈ ΠΌΠ°Ρ‚Π΅Ρ€ΠΈΠ°Π»Π°.
  2. ЭлСктричСскиС Π²Π΅Π»ΠΈΡ‡ΠΈΠ½Ρ‹.
  3. ГСомСтричСскиС свойства вСщСства.
  4. Π’ΠΈΠΏ ΠΌΠ°Ρ‚Π΅Ρ€ΠΈΠ°Π»Π°, ΠΈΠ· ΠΊΠΎΡ‚ΠΎΡ€ΠΎΠ³ΠΎ ΠΈΠ·Π³ΠΎΡ‚ΠΎΠ²Π»Π΅Π½ ΠΏΡ€ΠΎΠ²ΠΎΠ΄Π½ΠΈΠΊ (ΠΏΠΎΠ»ΡƒΠΏΡ€ΠΎΠ²ΠΎΠ΄Π½ΠΈΠΊ).

ЭлСктричСскиС Π²Π΅Π»ΠΈΡ‡ΠΈΠ½Ρ‹

Π—Π°Π²ΠΈΡΠΈΠΌΠΎΡΡ‚ΡŒ Π²Π΅Π»ΠΈΡ‡ΠΈΠ½Ρ‹ элСктропроводимости ΠΎΡ‚ ΠΏΠ°Ρ€Π°ΠΌΠ΅Ρ‚Ρ€ΠΎΠ² элСктричСства опрСдСляСтся Π·Π°ΠΊΠΎΠ½ΠΎΠΌ Ома. БущСствуСт Π΄Π²Π΅ Ρ„ΠΎΡ€ΠΌΡƒΠ»ΠΈΡ€ΠΎΠ²ΠΊΠΈ: ΠΎΠ΄Π½Π° β€” для участка, Π° другая β€” для ΠΏΠΎΠ»Π½ΠΎΠΉ Ρ†Π΅ΠΏΠΈ. Π’ ΠΏΠ΅Ρ€Π²ΠΎΠΌ случаС ΡΠΎΠΎΡ‚Π½ΠΎΡˆΠ΅Π½ΠΈΠ΅ ΠΎΠΏΡ€Π΅Π΄Π΅Π»ΡΡŽΡ‚ΡΡ, исходя ΠΈΠ· Π·Π½Π°Ρ‡Π΅Π½ΠΈΠΉ силы Ρ‚ΠΎΠΊΠ° (I) ΠΈ напряТСния (U) простой Ρ„ΠΎΡ€ΠΌΡƒΠ»ΠΎΠΉ: I = U / R. Из ΡΠΎΠΎΡ‚Π½ΠΎΡˆΠ΅Π½ΠΈΡ Π²ΠΈΠ΄Π½Π° прямо ΠΏΡ€ΠΎΠΏΠΎΡ€Ρ†ΠΈΠΎΠ½Π°Π»ΡŒΠ½Π°Ρ Π·Π°Π²ΠΈΡΠΈΠΌΠΎΡΡ‚ΡŒ Ρ‚ΠΎΠΊΠ° ΠΎΡ‚ Π²Π΅Π»ΠΈΡ‡ΠΈΠ½Ρ‹ напряТСния, Π° Ρ‚Π°ΠΊΠΆΠ΅ ΠΎΠ±Ρ€Π°Ρ‚Π½ΠΎ ΠΏΡ€ΠΎΠΏΠΎΡ€Ρ†ΠΈΠΎΠ½Π°Π»ΡŒΠ½Π°Ρ ΠΎΡ‚ сопротивлСния. МоТно Π²Ρ‹Ρ€Π°Π·ΠΈΡ‚ΡŒ R: R = U / I.

Для расчСта элСктропроводимости всСго участка слСдуСт Π²ΠΎΡΠΏΠΎΠ»ΡŒΠ·ΠΎΠ²Π°Ρ‚ΡŒΡΡ ΡΠΎΠΎΡ‚Π½ΠΎΡˆΠ΅Π½ΠΈΠ΅ΠΌ ΠΌΠ΅ΠΆΠ΄Ρƒ Π­Π”Π‘ (e), силой Ρ‚ΠΎΠΊΠ° (i), Π° Ρ‚Π°ΠΊΠΆΠ΅ Π²Π½ΡƒΡ‚Ρ€Π΅Π½Π½ΠΈΠΌ сопротивлСниСм источника питания (RΠ²Π½): i = e / (R+RΠ²Π½). Π’ этом случаС Π²Π΅Π»ΠΈΡ‡ΠΈΠ½Π° R вычисляСтся ΠΏΠΎ Ρ„ΠΎΡ€ΠΌΡƒΠ»Π΅: R = (e / i) β€” RΠ²Π½. Однако ΠΏΡ€ΠΈ Π²Ρ‹ΠΏΠΎΠ»Π½Π΅Π½ΠΈΠΈ расчСтов Π½Π΅ΠΎΠ±Ρ…ΠΎΠ΄ΠΈΠΌΠΎ ΡƒΡ‡ΠΈΡ‚Ρ‹Π²Π°Ρ‚ΡŒ Ρ‚Π°ΠΊΠΆΠ΅ гСомСтричСскиС ΠΏΠ°Ρ€Π°ΠΌΠ΅Ρ‚Ρ€Ρ‹ ΠΈ Ρ‚ΠΈΠΏ ΠΏΡ€ΠΎΠ²ΠΎΠ΄Π½ΠΈΠΊΠ°, ΠΏΠΎΡΠΊΠΎΠ»ΡŒΠΊΡƒ ΠΎΠ½ΠΈ ΠΌΠΎΠ³ΡƒΡ‚ сущСствСнно ΠΏΠΎΠ²Π»ΠΈΡΡ‚ΡŒ Π½Π° вычислСния.

Π’ΠΈΠΏ ΠΈ гСомСтричСскиС ΠΏΠ°Ρ€Π°ΠΌΠ΅Ρ‚Ρ€Ρ‹

Бвойство вСщСства ΠΊ проводимости элСктричСства опрСдСляСтся структурой кристалличСской Ρ€Π΅ΡˆΠ΅Ρ‚ΠΊΠΈ, Π° Ρ‚Π°ΠΊΠΆΠ΅ количСством свободных носитСлСй. Π˜ΡΡ…ΠΎΠ΄Ρ ΠΈΠ· этого, Ρ‚ΠΈΠΏ вСщСства являСтся ΠΊΠ»ΡŽΡ‡Π΅Π²Ρ‹ΠΌ Ρ„Π°ΠΊΡ‚ΠΎΡ€ΠΎΠΌ, ΠΊΠΎΡ‚ΠΎΡ€Ρ‹ΠΉ опрСдСляСт Π²Π΅Π»ΠΈΡ‡ΠΈΠ½Ρƒ элСктропроводимости. Π’ Π½Π°ΡƒΠΊΠ΅ коэффициСнт, ΠΎΠΏΡ€Π΅Π΄Π΅Π»ΡΡŽΡ‰ΠΈΠΉ Ρ‚ΠΈΠΏ вСщСства, обозначаСтся Π»ΠΈΡ‚Π΅Ρ€ΠΎΠΉ Β«Ρ€Β» ΠΈ называСтся ΡƒΠ΄Π΅Π»ΡŒΠ½Ρ‹ΠΌ сопротивлСниСм. Π•Π³ΠΎ Π·Π½Π°Ρ‡Π΅Π½ΠΈΠ΅ для Ρ€Π°Π·Π»ΠΈΡ‡Π½Ρ‹Ρ… ΠΌΠ°Ρ‚Π΅Ρ€ΠΈΠ°Π»ΠΎΠ² (ΠΏΡ€ΠΈ Ρ‚Π΅ΠΌΠΏΠ΅Ρ€Π°Ρ‚ΡƒΡ€Π΅ +20 градусов ΠΏΠΎ ЦСльсию) ΠΌΠΎΠΆΠ½ΠΎ Π½Π°ΠΉΡ‚ΠΈ Π² ΡΠΏΠ΅Ρ†ΠΈΠ°Π»ΡŒΠ½Ρ‹Ρ… Ρ‚Π°Π±Π»ΠΈΡ†Π°Ρ….

Иногда для удобства расчСтов ΠΈΡΠΏΠΎΠ»ΡŒΠ·ΡƒΠ΅Ρ‚ΡΡ обратная Π²Π΅Π»ΠΈΡ‡ΠΈΠ½Π°, которая называСтся ΡƒΠ΄Π΅Π»ΡŒΠ½ΠΎΠΉ ΠΏΡ€ΠΎΠ²ΠΎΠ΄ΠΈΠΌΠΎΡΡ‚ΡŒΡŽ (Οƒ). Она связана с ΡƒΠ΄Π΅Π»ΡŒΠ½Ρ‹ΠΌ сопротивлСниСм ΡΠ»Π΅Π΄ΡƒΡŽΡ‰ΠΈΠΌ ΡΠΎΠΎΡ‚Π½ΠΎΡˆΠ΅Π½ΠΈΠ΅ΠΌ: p = 1 / Οƒ. ΠŸΠ»ΠΎΡ‰Π°Π΄ΡŒ ΠΏΠΎΠΏΠ΅Ρ€Π΅Ρ‡Π½ΠΎΠ³ΠΎ сСчСния (S) влияСт Π½Π° элСктричСскоС сопротивлСниС. Π‘ физичСской Ρ‚ΠΎΡ‡ΠΊΠΈ зрСния, Π·Π°Π²ΠΈΡΠΈΠΌΠΎΡΡ‚ΡŒ ΠΌΠΎΠΆΠ½ΠΎ ΠΏΠΎΠ½ΡΡ‚ΡŒ ΡΠ»Π΅Π΄ΡƒΡŽΡ‰ΠΈΠΌ ΠΎΠ±Ρ€Π°Π·ΠΎΠΌ: ΠΏΡ€ΠΈ ΠΌΠ°Π»ΠΎΠΌ сСчСнии происходят Π±ΠΎΠ»Π΅Π΅ частыС взаимодСйствия частиц элСктричСского Ρ‚ΠΎΠΊΠ° с ΡƒΠ·Π»Π°ΠΌΠΈ кристалличСской Ρ€Π΅ΡˆΠ΅Ρ‚ΠΊΠΈ. ΠŸΠΎΠΏΠ΅Ρ€Π΅Ρ‡Π½ΠΎΠ΅ сСчСниС ΠΌΠΎΠΆΠ½ΠΎ Π²Ρ‹Ρ‡ΠΈΡΠ»ΠΈΡ‚ΡŒ ΠΏΠΎ ΡΠΏΠ΅Ρ†ΠΈΠ°Π»ΡŒΠ½ΠΎΠΌΡƒ Π°Π»Π³ΠΎΡ€ΠΈΡ‚ΠΌΡƒ:

  1. Π˜Π·ΠΌΠ΅Ρ€Π΅Π½ΠΈΠ΅ гСомСтричСских ΠΏΠ°Ρ€Π°ΠΌΠ΅Ρ‚Ρ€ΠΎΠ² ΠΏΡ€ΠΎΠ²ΠΎΠ΄Π½ΠΈΠΊΠ° (Π΄ΠΈΠ°ΠΌΠ΅Ρ‚Ρ€ ΠΈΠ»ΠΈ Π΄Π»ΠΈΠ½Ρƒ сторон) ΠΏΡ€ΠΈ ΠΏΠΎΠΌΠΎΡ‰ΠΈ ΡˆΡ‚Π°Π½Π³Π΅Π½Ρ†ΠΈΡ€ΠΊΡƒΠ»Ρ.
  2. Π’ΠΈΠ·ΡƒΠ°Π»ΡŒΠ½ΠΎ ΠΎΠΏΡ€Π΅Π΄Π΅Π»ΠΈΡ‚ΡŒ Ρ„ΠΎΡ€ΠΌΡƒ ΠΌΠ°Ρ‚Π΅Ρ€ΠΈΠ°Π»Π°.
  3. Π’Ρ‹Ρ‡ΠΈΡΠ»ΠΈΡ‚ΡŒ ΠΏΠ»ΠΎΡ‰Π°Π΄ΡŒ ΠΏΠΎΠΏΠ΅Ρ€Π΅Ρ‡Π½ΠΎΠ³ΠΎ сСчСния ΠΏΠΎ Ρ„ΠΎΡ€ΠΌΡƒΠ»Π΅, Π½Π°ΠΉΠ΄Π΅Π½Π½ΠΎΠΉ Π² справочникС ΠΈΠ»ΠΈ ΠΈΠ½Ρ‚Π΅Ρ€Π½Π΅Ρ‚Π΅.

Π’ случаС ΠΊΠΎΠ³Π΄Π° ΠΏΡ€ΠΎΠ²ΠΎΠ΄Π½ΠΈΠΊ ΠΈΠΌΠ΅Π΅Ρ‚ ΡΠ»ΠΎΠΆΠ½ΡƒΡŽ структуру, Π½Π΅ΠΎΠ±Ρ…ΠΎΠ΄ΠΈΠΌΠΎ Π²Ρ‹Ρ‡ΠΈΡΠ»ΠΈΡ‚ΡŒ Π²Π΅Π»ΠΈΡ‡ΠΈΠ½Ρƒ S ΠΎΠ΄Π½ΠΎΠ³ΠΎ элСмСнта, Π° Π·Π°Ρ‚Π΅ΠΌ ΡƒΠΌΠ½ΠΎΠΆΠΈΡ‚ΡŒ Ρ€Π΅Π·ΡƒΠ»ΡŒΡ‚Π°Ρ‚ Π½Π° количСство элСмСнтов, входящих Π² Π΅Π³ΠΎ состав. НапримСр, Ссли ΠΏΡ€ΠΎΠ²ΠΎΠ΄ являСтся ΠΌΠ½ΠΎΠ³ΠΎΠΆΠΈΠ»ΡŒΠ½Ρ‹ΠΌ, Ρ‚ΠΎ слСдуСт Π²Ρ‹Ρ‡ΠΈΡΠ»ΠΈΡ‚ΡŒ S для ΠΎΠ΄Π½ΠΎΠΉ ΠΆΠΈΠ»Ρ‹. ПослС этого Π½ΡƒΠΆΠ½ΠΎ ΡƒΠΌΠ½ΠΎΠΆΠΈΡ‚ΡŒ, ΠΏΠΎΠ»ΡƒΡ‡Π΅Π½Π½ΡƒΡŽ Π²Π΅Π»ΠΈΡ‡ΠΈΠ½Ρƒ S, Π½Π° количСство ΠΆΠΈΠ». Π—Π°Π²ΠΈΡΠΈΠΌΠΎΡΡ‚ΡŒ R ΠΎΡ‚ Π²Ρ‹ΡˆΠ΅ΠΏΠ΅Ρ€Π΅Ρ‡ΠΈΡΠ»Π΅Π½Π½Ρ‹Ρ… Π²Π΅Π»ΠΈΡ‡ΠΈΠ½ ΠΌΠΎΠΆΠ½ΠΎ Π·Π°ΠΏΠΈΡΠ°Ρ‚ΡŒ Π² Π²ΠΈΠ΄Π΅ ΡΠΎΠΎΡ‚Π½ΠΎΡˆΠ΅Π½ΠΈΡ: R = p * L / S. Π›ΠΈΡ‚Π΅Ρ€Π° Β«LΒ» являСтся Π΄Π»ΠΈΠ½ΠΎΠΉ ΠΏΡ€ΠΎΠ²ΠΎΠ΄Π½ΠΈΠΊΠ°. Однако для получСния Ρ‚ΠΎΡ‡Π½Ρ‹Ρ… расчСтов Π½Π΅ΠΎΠ±Ρ…ΠΎΠ΄ΠΈΠΌΠΎ ΡƒΡ‡ΠΈΡ‚Ρ‹Π²Π°Ρ‚ΡŒ Ρ‚Π΅ΠΌΠΏΠ΅Ρ€Π°Ρ‚ΡƒΡ€Π½Ρ‹Π΅ ΠΏΠΎΠΊΠ°Π·Π°Ρ‚Π΅Π»ΠΈ внСшнСй срСды ΠΈ ΠΏΡ€ΠΎΠ²ΠΎΠ΄Π½ΠΈΠΊΠ°.

Π’Π΅ΠΌΠΏΠ΅Ρ€Π°Ρ‚ΡƒΡ€Π½Ρ‹Π΅ ΠΏΠΎΠΊΠ°Π·Π°Ρ‚Π΅Π»ΠΈ

БущСствуСт Π΄ΠΎΠΊΠ°Π·Π°Ρ‚Π΅Π»ΡŒΡΡ‚Π²ΠΎ зависимости ΡƒΠ΄Π΅Π»ΡŒΠ½ΠΎΠ³ΠΎ сопротивлСния ΠΌΠ°Ρ‚Π΅Ρ€ΠΈΠ°Π»Π° ΠΎΡ‚ Ρ‚Π΅ΠΌΠΏΠ΅Ρ€Π°Ρ‚ΡƒΡ€Ρ‹, основанноС Π½Π° физичСском экспСримСнтС. Для провСдСния ΠΎΠΏΡ‹Ρ‚Π° Π½ΡƒΠΆΠ½ΠΎ ΡΠΎΠ±Ρ€Π°Ρ‚ΡŒ ΡΠ»Π΅ΠΊΡ‚Ρ€ΠΈΡ‡Π΅ΡΠΊΡƒΡŽ Ρ†Π΅ΠΏΡŒ, ΡΠΎΡΡ‚ΠΎΡΡ‰ΡƒΡŽ ΠΈΠ· ΡΠ»Π΅Π΄ΡƒΡŽΡ‰ΠΈΡ… элСмСнтов: источника питания, Π½ΠΈΡ…Ρ€ΠΎΠΌΠΎΠ²ΠΎΠΉ спирали, ΡΠΎΠ΅Π΄ΠΈΠ½ΠΈΡ‚Π΅Π»ΡŒΠ½Ρ‹Ρ… ΠΏΡ€ΠΎΠ²ΠΎΠ΄ΠΎΠ² Π°ΠΌΠΏΠ΅Ρ€ΠΌΠ΅Ρ‚Ρ€Π° ΠΈ Π²ΠΎΠ»ΡŒΡ‚ΠΌΠ΅Ρ‚Ρ€Π°. ΠŸΡ€ΠΈΠ±ΠΎΡ€Ρ‹ Π½ΡƒΠΆΠ½Ρ‹ для измСрСния Π·Π½Π°Ρ‡Π΅Π½ΠΈΠΉ силы Ρ‚ΠΎΠΊΠ° ΠΈ напряТСния соотвСтствСнно. ΠŸΡ€ΠΈ ΠΏΡ€ΠΎΡ‚Π΅ΠΊΠ°Π½ΠΈΠΈ элСктричСства происходит Π½Π°Π³Ρ€Π΅Π²Π°Π½ΠΈΠ΅ Π½ΠΈΡ…Ρ€ΠΎΠΌΠΎΠ²ΠΎΠΉ ΠΏΡ€ΡƒΠΆΠΈΠ½Ρ‹. По ΠΌΠ΅Ρ€Π΅ Π΅Π΅ нагрСвания, показания Π°ΠΌΠΏΠ΅Ρ€ΠΌΠ΅Ρ‚Ρ€Π° ΡƒΠΌΠ΅Π½ΡŒΡˆΠ°ΡŽΡ‚ΡΡ. ΠŸΡ€ΠΈ этом происходит сущСствСнноС ΠΏΠ°Π΄Π΅Π½ΠΈΠ΅ напряТСния Π½Π° участкС Ρ†Π΅ΠΏΠΈ, ΠΎ ΠΊΠΎΡ‚ΠΎΡ€ΠΎΠΌ ΡΠ²ΠΈΠ΄Π΅Ρ‚Π΅Π»ΡŒΡΡ‚Π²ΡƒΡŽΡ‚ показания Π²ΠΎΠ»ΡŒΡ‚ΠΌΠ΅Ρ‚Ρ€Π°.

Π’ Ρ€Π°Π΄ΠΈΠΎΡ‚Π΅Ρ…Π½ΠΈΠΊΠ΅ ΡƒΠΌΠ΅Π½ΡŒΡˆΠ΅Π½ΠΈΠ΅ Π²Π΅Π»ΠΈΡ‡ΠΈΠ½Ρ‹ напряТСниС называСтся просадкой ΠΈΠ»ΠΈ ΠΏΠ°Π΄Π΅Π½ΠΈΠ΅ΠΌ. Π€ΠΎΡ€ΠΌΡƒΠ»Π° зависимости Ρ€ ΠΎΡ‚ Ρ‚Π΅ΠΌΠΏΠ΅Ρ€Π°Ρ‚ΡƒΡ€Ρ‹ ΠΈΠΌΠ΅Π΅Ρ‚ ΡΠ»Π΅Π΄ΡƒΡŽΡ‰ΠΈΠΉ Π²ΠΈΠ΄: p = p0 * . Π—Π½Π°Ρ‡Π΅Π½ΠΈΠ΅ p0 β€” ΡƒΠ΄Π΅Π»ΡŒΠ½ΠΎΠ΅ сопротивлСниС ΠΌΠ°Ρ‚Π΅Ρ€ΠΈΠ°Π»Π°, взятого ΠΈΠ· Ρ‚Π°Π±Π»ΠΈΡ†Ρ‹, Π° Π»ΠΈΡ‚Π΅Ρ€Π° Β«tΒ» β€” Ρ‚Π΅ΠΌΠΏΠ΅Ρ€Π°Ρ‚ΡƒΡ€Π° ΠΏΡ€ΠΎΠ²ΠΎΠ΄Π½ΠΈΠΊΠ°.

Π’Π΅ΠΌΠΏΠ΅Ρ€Π°Ρ‚ΡƒΡ€Π½Ρ‹ΠΉ коэффициСнт Β«Π°Β» ΠΏΡ€ΠΈΠ½ΠΈΠΌΠ°Π΅Ρ‚ ΡΠ»Π΅Π΄ΡƒΡŽΡ‰ΠΈΠ΅ значСния: для ΠΌΠ΅Ρ‚Π°Π»Π»ΠΎΠ² β€” a>0, Π° для элСктролитичСских растворов β€” a<0. Для получСния Ρ„ΠΎΡ€ΠΌΡƒΠ»Ρ‹, ΠΎΠΏΡ€Π΅Π΄Π΅Π»ΡΡŽΡ‰Π΅ΠΉ всС зависимости, Π½Π΅ΠΎΠ±Ρ…ΠΎΠ΄ΠΈΠΌΠΎ ΠΏΠΎΠ΄ΡΡ‚Π°Π²ΠΈΡ‚ΡŒ всС ΡΠΎΠΎΡ‚Π½ΠΎΡˆΠ΅Π½ΠΈΡ Π² ΠΎΠ±Ρ‰ΡƒΡŽ Ρ„ΠΎΡ€ΠΌΡƒΠ»Ρƒ зависимости R ΠΎΡ‚ Ρ‚ΠΈΠΏΠ° ΠΌΠ°Ρ‚Π΅Ρ€ΠΈΠ°Π»Π°, Ρ‚Π΅ΠΌΠΏΠ΅Ρ€Π°Ρ‚ΡƒΡ€Ρ‹, Π΄Π»ΠΈΠ½Ρ‹ ΠΈ сСчСния: R = p0 * * L / S. Π€ΠΎΡ€ΠΌΡƒΠ»Ρ‹ ΠΈΡΠΏΠΎΠ»ΡŒΠ·ΡƒΡŽΡ‚ΡΡ Ρ‚ΠΎΠ»ΡŒΠΊΠΎ для расчСтов ΠΈ изготовлСния рСзисторов. Для быстрого измСрСния Π²Π΅Π»ΠΈΡ‡ΠΈΠ½Ρ‹ сопротивлСния примСняСтся ΠΎΠΌΠΌΠ΅Ρ‚Ρ€.

Π€ΠΎΡ€ΠΌΡƒΠ»Π° ΠΏΠ°Ρ€Π°Π»Π»Π΅Π»ΡŒΠ½ΠΎΠ³ΠΎ соСдинСния рСзисторов

ΠžΠ±Ρ‰Π΅Π΅ сопротивлСниС Π½Π΅ΡΠΊΠΎΠ»ΡŒΠΊΠΈΡ… рСзисторов соСдинСнных ΠΏΠ°Ρ€Π°Π»Π»Π΅Π»ΡŒΠ½ΠΎ опрСдСляСтся ΠΏΠΎ ΡΠ»Π΅Π΄ΡƒΡŽΡ‰Π΅ΠΉ Ρ„ΠΎΡ€ΠΌΡƒΠ»Π΅:

Π’ΠΎΠΊ, ΠΏΡ€ΠΎΡ‚Π΅ΠΊΠ°ΡŽΡ‰ΠΈΠΉ Ρ‡Π΅Ρ€Π΅Π· ΠΎΡ‚Π΄Π΅Π»ΡŒΠ½ΠΎ взятый рСзистор, согласно Π·Π°ΠΊΠΎΠ½Ρƒ Ома, ΠΌΠΎΠΆΠ½ΠΎ Π½Π°ΠΉΡ‚ΠΈ ΠΏΠΎ Ρ„ΠΎΡ€ΠΌΡƒΠ»Π΅:

ΠŸΡ€ΠΈΠΌΠ΅Ρ€ Β β„–1

ΠŸΡ€ΠΈ Ρ€Π°Π·Ρ€Π°Π±ΠΎΡ‚ΠΊΠ΅ устройства, Π²ΠΎΠ·Π½ΠΈΠΊΠ»Π° Π½Π΅ΠΎΠ±Ρ…ΠΎΠ΄ΠΈΠΌΠΎΡΡ‚ΡŒ ΡƒΡΡ‚Π°Π½ΠΎΠ²ΠΈΡ‚ΡŒ рСзистор с сопротивлСниСм 8 Ом. Если ΠΌΡ‹ просмотрим вСсь Π½ΠΎΠΌΠΈΠ½Π°Π»ΡŒΠ½Ρ‹ΠΉ ряд стандартных Π·Π½Π°Ρ‡Π΅Π½ΠΈΠΉ рСзисторов, Ρ‚ΠΎ ΠΌΡ‹ ΡƒΠ²ΠΈΠ΄ΠΈΠΌ, Ρ‡Ρ‚ΠΎ рСзистора с сопротивлСниСм Π² 8 Ом Π² Π½Π΅ΠΌ Π½Π΅Ρ‚.

Π’Ρ‹Ρ…ΠΎΠ΄ΠΎΠΌ ΠΈΠ· Π΄Π°Π½Π½ΠΎΠΉ ситуации Π±ΡƒΠ΄Π΅Ρ‚ использованиС Π΄Π²ΡƒΡ… ΠΏΠ°Ρ€Π°Π»Π»Π΅Π»ΡŒΠ½ΠΎ соСдинСнных рСзисторов. Π­ΠΊΠ²ΠΈΠ²Π°Π»Π΅Π½Ρ‚Π½ΠΎΠ΅ Π·Π½Π°Ρ‡Π΅Π½ΠΈΠ΅ сопротивлСния для Π΄Π²ΡƒΡ… рСзисторов соСдинСнных ΠΏΠ°Ρ€Π°Π»Π»Π΅Π»ΡŒΠ½ΠΎ рассчитываСтся ΡΠ»Π΅Π΄ΡƒΡŽΡ‰ΠΈΠΌ ΠΎΠ±Ρ€Π°Π·ΠΎΠΌ:

Π”Π°Π½Π½ΠΎΠ΅ ΡƒΡ€Π°Π²Π½Π΅Π½ΠΈΠ΅ ΠΏΠΎΠΊΠ°Π·Ρ‹Π²Π°Π΅Ρ‚, Ρ‡Ρ‚ΠΎ Ссли R1 Ρ€Π°Π²Π΅Π½ R2, Ρ‚ΠΎ сопротивлСниС R составляСт ΠΏΠΎΠ»ΠΎΠ²ΠΈΠ½Ρƒ сопротивлСния ΠΎΠ΄Π½ΠΎΠ³ΠΎ ΠΈΠ· Π΄Π²ΡƒΡ… рСзисторов. ΠŸΡ€ΠΈ R = 8 Ом, R1 ΠΈ R2 Π΄ΠΎΠ»ΠΆΠ½Ρ‹, ΡΠ»Π΅Π΄ΠΎΠ²Π°Ρ‚Π΅Π»ΡŒΠ½ΠΎ, ΠΈΠΌΠ΅Ρ‚ΡŒ Π·Π½Π°Ρ‡Π΅Π½ΠΈΠ΅ 2 Γ— 8 = 16 Ом.
Π’Π΅ΠΏΠ΅Ρ€ΡŒ ΠΏΡ€ΠΎΠ²Π΅Π΄Π΅ΠΌ ΠΏΡ€ΠΎΠ²Π΅Ρ€ΠΊΡƒ, рассчитав ΠΎΠ±Ρ‰Π΅Π΅ сопротивлСниС Π΄Π²ΡƒΡ… рСзисторов:

Π’Π°ΠΊΠΈΠΌ ΠΎΠ±Ρ€Π°Π·ΠΎΠΌ, ΠΌΡ‹ ΠΏΠΎΠ»ΡƒΡ‡ΠΈΠ»ΠΈ Π½Π΅ΠΎΠ±Ρ…ΠΎΠ΄ΠΈΠΌΠΎΠ΅ сопротивлСниС 8 Ом, соСдинив ΠΏΠ°Ρ€Π°Π»Π»Π΅Π»ΡŒΠ½ΠΎ Π΄Π²Π° рСзистора ΠΏΠΎ 16 Ом.

ΠŸΡ€ΠΈΠΌΠ΅Ρ€ расчСта β„–2

Найти ΠΎΠ±Ρ‰Π΅Π΅ сопротивлСниС Β R ΠΈΠ· Ρ‚Ρ€Π΅Ρ… ΠΏΠ°Ρ€Π°Π»Π»Π΅Π»ΡŒΠ½ΠΎ соСдинСнных рСзисторов:

ΠžΠ±Ρ‰Π΅Π΅ сопротивлСниС R рассчитываСтся ΠΏΠΎ Ρ„ΠΎΡ€ΠΌΡƒΠ»Π΅:

Π­Ρ‚ΠΎΡ‚ ΠΌΠ΅Ρ‚ΠΎΠ΄ расчСта ΠΌΠΎΠΆΠ΅Ρ‚ Π±Ρ‹Ρ‚ΡŒ ΠΈΡΠΏΠΎΠ»ΡŒΠ·ΠΎΠ²Π°Π½Ρ‹ для расчСта любого количСства ΠΎΡ‚Π΄Π΅Π»ΡŒΠ½Ρ‹Ρ… сопротивлСний соСдинСнных ΠΏΠ°Ρ€Π°Π»Π»Π΅Π»ΡŒΠ½ΠΎ.

Один Π²Π°ΠΆΠ½Ρ‹ΠΉ ΠΌΠΎΠΌΠ΅Π½Ρ‚, ΠΊΠΎΡ‚ΠΎΡ€Ρ‹ΠΉ Π½Π΅ΠΎΠ±Ρ…ΠΎΠ΄ΠΈΠΌΠΎ Π·Π°ΠΏΠΎΠΌΠ½ΠΈΡ‚ΡŒ ΠΏΡ€ΠΈ расчСтС ΠΏΠ°Ρ€Π°Π»Π»Π΅Π»ΡŒΠ½ΠΎ соСдинСнных рСзисторов – это Ρ‚ΠΎ, Ρ‡Ρ‚ΠΎ ΠΎΠ±Ρ‰Π΅Π΅ сопротивлСниС всСгда Π±ΡƒΠ΄Π΅Ρ‚ мСньшС, Ρ‡Π΅ΠΌ Π·Π½Π°Ρ‡Π΅Π½ΠΈΠ΅ наимСньшСго сопротивлСния Π² этой ΠΊΠΎΠΌΠ±ΠΈΠ½Π°Ρ†ΠΈΠΈ.

Как Ρ€Π°ΡΡΡ‡ΠΈΡ‚Π°Ρ‚ΡŒ слоТныС схСмы соСдинСния рСзисторов

Π‘ΠΎΠ»Π΅Π΅ слоТныС соСдинСния рСзисторов ΠΌΠΎΠ³ΡƒΡ‚ Π±Ρ‹Ρ‚ΡŒ рассчитаны ΠΏΡƒΡ‚Π΅ΠΌ систСматичСской Π³Ρ€ΡƒΠΏΠΏΠΈΡ€ΠΎΠ²ΠΊΠΈ рСзисторов. На рисункС Π½ΠΈΠΆΠ΅ Π½Π΅ΠΎΠ±Ρ…ΠΎΠ΄ΠΈΠΌΠΎ ΠΏΠΎΡΡ‡ΠΈΡ‚Π°Ρ‚ΡŒ ΠΎΠ±Ρ‰Π΅Π΅ сопротивлСниС Ρ†Π΅ΠΏΠΈ, состоящСй ΠΈΠ· Ρ‚Ρ€Π΅Ρ… рСзисторов:

РСзисторы R2 ΠΈ R3 соСдинСны ΠΏΠΎΡΠ»Π΅Π΄ΠΎΠ²Π°Ρ‚Π΅Π»ΡŒΠ½ΠΎ (Π³Ρ€ΡƒΠΏΠΏΠ° 2). Они Π² свою ΠΎΡ‡Π΅Ρ€Π΅Π΄ΡŒ соСдинСны ΠΏΠ°Ρ€Π°Π»Π»Π΅Π»ΡŒΠ½ΠΎ с рСзистором R1 (Π³Ρ€ΡƒΠΏΠΏΠ° 1).

ΠŸΠΎΡΠ»Π΅Π΄ΠΎΠ²Π°Ρ‚Π΅Π»ΡŒΠ½ΠΎΠ΅ соСдинСниС рСзисторов Π³Ρ€ΡƒΠΏΠΏΡ‹ 2 вычисляСтся ΠΊΠ°ΠΊ сумма сопротивлСний R2 ΠΈ R3:

Π’ Ρ€Π΅Π·ΡƒΠ»ΡŒΡ‚Π°Ρ‚Π΅ ΠΌΡ‹ ΡƒΠΏΡ€ΠΎΡ‰Π°Π΅ΠΌ схСму Π² Π²ΠΈΠ΄Π΅ Π΄Π²ΡƒΡ… ΠΏΠ°Ρ€Π°Π»Π»Π΅Π»ΡŒΠ½Ρ‹Ρ… рСзисторов. Π’Π΅ΠΏΠ΅Ρ€ΡŒ ΠΎΠ±Ρ‰Π΅Π΅ сопротивлСниС всСй схСмы ΠΌΠΎΠΆΠ½ΠΎ ΠΏΠΎΡΡ‡ΠΈΡ‚Π°Ρ‚ΡŒ ΡΠ»Π΅Π΄ΡƒΡŽΡ‰ΠΈΠΌ ΠΎΠ±Ρ€Π°Π·ΠΎΠΌ:

РасчСт Π±ΠΎΠ»Π΅Π΅ слоТных соСдинСний рСзисторов ΠΌΠΎΠΆΠ½ΠΎ Π²Ρ‹ΠΏΠΎΠ»Π½ΠΈΡ‚ΡŒ ΠΈΡΠΏΠΎΠ»ΡŒΠ·ΡƒΡ Π·Π°ΠΊΠΎΠ½Ρ‹ ΠšΠΈΡ€Ρ…Π³ΠΎΡ„Π°.

Π’ΠΎΠΊ, ΠΏΡ€ΠΎΡ‚Π΅ΠΊΠ°ΡŽΡ‰ΠΈΠΉ Π² Ρ†Π΅ΠΏΠΈ ΠΏΠ°Ρ€Π°Π»Π»Π΅Π»ΡŒΠ½ΠΎ соСдинСнных рСзисторах

ΠžΠ±Ρ‰ΠΈΠΉ Ρ‚ΠΎΠΊ I ΠΏΡ€ΠΎΡ‚Π΅ΠΊΠ°ΡŽΡ‰ΠΈΠΉ Π² Ρ†Π΅ΠΏΠΈ ΠΏΠ°Ρ€Π°Π»Π»Π΅Π»ΡŒΠ½Ρ‹Ρ… рСзисторов равняСтся суммС ΠΎΡ‚Π΄Π΅Π»ΡŒΠ½Ρ‹Ρ… Ρ‚ΠΎΠΊΠΎΠ², ΠΏΡ€ΠΎΡ‚Π΅ΠΊΠ°ΡŽΡ‰ΠΈΡ… Π²ΠΎ всСх ΠΏΠ°Ρ€Π°Π»Π»Π΅Π»ΡŒΠ½Ρ‹Ρ… вСтвях, ΠΏΡ€ΠΈΡ‡Π΅ΠΌ Ρ‚ΠΎΠΊ Π² ΠΎΡ‚Π΄Π΅Π»ΡŒΠ½ΠΎ взятой Π²Π΅Ρ‚Π²ΠΈ Π½Π΅ ΠΎΠ±ΡΠ·Π°Ρ‚Π΅Π»ΡŒΠ½ΠΎ Π΄ΠΎΠ»ΠΆΠ΅Π½ Π±Ρ‹Ρ‚ΡŒ Ρ€Π°Π²Π΅Π½ Ρ‚ΠΎΠΊΡƒ Π² сосСдних вСтвях.

НСсмотря Π½Π° ΠΏΠ°Ρ€Π°Π»Π»Π΅Π»ΡŒΠ½ΠΎΠ΅ соСдинСниС, ΠΊ ΠΊΠ°ΠΆΠ΄ΠΎΠΌΡƒ рСзистору ΠΏΡ€ΠΈΠ»ΠΎΠΆΠ΅Π½ΠΎ ΠΎΠ΄Π½ΠΎ ΠΈ Ρ‚ΠΎ ΠΆΠ΅ напряТСниС. А ΠΏΠΎΡΠΊΠΎΠ»ΡŒΠΊΡƒ Π²Π΅Π»ΠΈΡ‡ΠΈΠ½Π° сопротивлСний Π² ΠΏΠ°Ρ€Π°Π»Π»Π΅Π»ΡŒΠ½ΠΎΠΉ Ρ†Π΅ΠΏΠΈ ΠΌΠΎΠΆΠ΅Ρ‚ Π±Ρ‹Ρ‚ΡŒ Ρ€Π°Π·Π½ΠΎΠΉ, Ρ‚ΠΎ ΠΈ Π²Π΅Π»ΠΈΡ‡ΠΈΠ½Π° ΠΏΡ€ΠΎΡ‚Π΅ΠΊΠ°ΡŽΡ‰Π΅Π³ΠΎ Ρ‚ΠΎΠΊΠ° Ρ‡Π΅Ρ€Π΅Π· ΠΊΠ°ΠΆΠ΄Ρ‹ΠΉ рСзистор Ρ‚ΠΎΠΆΠ΅ Π±ΡƒΠ΄Π΅Ρ‚ ΠΎΡ‚Π»ΠΈΡ‡Π°Ρ‚ΡŒΡΡ (ΠΏΠΎ ΠΎΠΏΡ€Π΅Π΄Π΅Π»Π΅Π½ΠΈΡŽ Π·Π°ΠΊΠΎΠ½Π° Ома).

Рассмотрим это Π½Π° ΠΏΡ€ΠΈΠΌΠ΅Ρ€Π΅ Π΄Π²ΡƒΡ… ΠΏΠ°Ρ€Π°Π»Π»Π΅Π»ΡŒΠ½ΠΎ соСдинСнных рСзисторов. Π’ΠΎΠΊ, ΠΊΠΎΡ‚ΠΎΡ€Ρ‹ΠΉ Ρ‚Π΅Ρ‡Π΅Ρ‚ Ρ‡Π΅Ρ€Π΅Π· ΠΊΠ°ΠΆΠ΄Ρ‹ΠΉ ΠΈΠ· рСзисторов ( I1 ΠΈ I2 ) Π±ΡƒΠ΄Π΅Ρ‚ ΠΎΡ‚Π»ΠΈΡ‡Π°Ρ‚ΡŒΡΡ Π΄Ρ€ΡƒΠ³ ΠΎΡ‚ Π΄Ρ€ΡƒΠ³Π° ΠΏΠΎΡΠΊΠΎΠ»ΡŒΠΊΡƒ сопротивлСния рСзисторов R1 ΠΈ R2 Π½Π΅ Ρ€Π°Π²Π½Ρ‹.
Однако ΠΌΡ‹ Π·Π½Π°Π΅ΠΌ, Ρ‡Ρ‚ΠΎ Ρ‚ΠΎΠΊ, ΠΊΠΎΡ‚ΠΎΡ€Ρ‹ΠΉ поступаСт Π² Ρ†Π΅ΠΏΡŒ Π² Ρ‚ΠΎΡ‡ΠΊΠ΅ «А» Π΄ΠΎΠ»ΠΆΠ΅Π½ Π²Ρ‹ΠΉΡ‚ΠΈ ΠΈΠ· Ρ†Π΅ΠΏΠΈ Π² Ρ‚ΠΎΡ‡ΠΊΠ΅ Β«BΒ» .

ΠŸΠ΅Ρ€Π²ΠΎΠ΅ ΠΏΡ€Π°Π²ΠΈΠ»ΠΎ ΠšΠΈΡ€Ρ…Π³ΠΎΡ„Π° гласит: Β«ΠžΠ±Ρ‰ΠΈΠΉ Ρ‚ΠΎΠΊ, выходящий ΠΈΠ· Ρ†Π΅ΠΏΠΈ Ρ€Π°Π²Π΅Π½ Ρ‚ΠΎΠΊΡƒ входящий Π² Ρ†Π΅ΠΏΡŒΒ».

  • Π’Π°ΠΊΠΈΠΌ ΠΎΠ±Ρ€Π°Π·ΠΎΠΌ, ΠΏΡ€ΠΎΡ‚Π΅ΠΊΠ°ΡŽΡ‰ΠΈΠΉ ΠΎΠ±Ρ‰ΠΈΠΉ Ρ‚ΠΎΠΊ Π² Ρ†Π΅ΠΏΠΈ Β ΠΌΠΎΠΆΠ½ΠΎ ΠΎΠΏΡ€Π΅Π΄Π΅Π»ΠΈΡ‚ΡŒ ΠΊΠ°ΠΊ:
  • I = I1 + I2
  • Π—Π°Ρ‚Π΅ΠΌ с ΠΏΠΎΠΌΠΎΡ‰ΡŒΡŽ Π·Π°ΠΊΠΎΠ½Π° Ома ΠΌΠΎΠΆΠ½ΠΎ Π²Ρ‹Ρ‡ΠΈΡΠ»ΠΈΡ‚ΡŒ Ρ‚ΠΎΠΊ, ΠΊΠΎΡ‚ΠΎΡ€Ρ‹ΠΉ ΠΏΡ€ΠΎΡ‚Π΅ΠΊΠ°Π΅Ρ‚ Ρ‡Π΅Ρ€Π΅Π· ΠΊΠ°ΠΆΠ΄Ρ‹ΠΉ рСзистор:
  • Π’ΠΎΠΊ, ΠΏΡ€ΠΎΡ‚Π΅ΠΊΠ°ΡŽΡ‰ΠΈΠΉ Π² R1 = U Γ· R1 = 12 Γ· 22 кОм = 0,545 мА
  • Π’ΠΎΠΊ, ΠΏΡ€ΠΎΡ‚Π΅ΠΊΠ°ΡŽΡ‰ΠΈΠΉ Π² R 2 = U Γ· R2 = 12 Γ· 47 кОм = 0,255 мА
  • Π’Π°ΠΊΠΈΠΌ ΠΎΠ±Ρ€Π°Π·ΠΎΠΌ, ΠΎΠ±Ρ‰ΠΈΠΉ Ρ‚ΠΎΠΊ Π±ΡƒΠ΄Π΅Ρ‚ Ρ€Π°Π²Π΅Π½:
  • I = 0,545 мА + 0,255 мА = 0,8 мА
  • Π­Ρ‚ΠΎ Ρ‚Π°ΠΊΠΆΠ΅ ΠΌΠΎΠΆΠ½ΠΎ ΠΏΡ€ΠΎΠ²Π΅Ρ€ΠΈΡ‚ΡŒ, ΠΈΡΠΏΠΎΠ»ΡŒΠ·ΡƒΡ Π·Π°ΠΊΠΎΠ½ Ома:
  • I = U Γ· R = 12 Π’ Γ· 15 кОм = 0,8 мА (Ρ‚ΠΎ ΠΆΠ΅ самоС)
  • Π³Π΄Π΅ 15кОм β€” это ΠΎΠ±Ρ‰Π΅Π΅ сопротивлСниС Π΄Π²ΡƒΡ… ΠΏΠ°Ρ€Π°Π»Π»Π΅Π»ΡŒΠ½ΠΎ соСдинСнных рСзисторов (22 кОм ΠΈ 47 кОм)
  • И Π² Π·Π°Π²Π΅Ρ€ΡˆΠ΅Π½ΠΈΠΈ хочСтся ΠΎΡ‚ΠΌΠ΅Ρ‚ΠΈΡ‚ΡŒ, Ρ‡Ρ‚ΠΎ Π±ΠΎΠ»ΡŒΡˆΠΈΠ½ΡΡ‚Π²ΠΎ соврСмСнных рСзисторов ΠΌΠ°Ρ€ΠΊΠΈΡ€ΡƒΡŽΡ‚ΡΡ Ρ†Π²Π΅Ρ‚Π½Ρ‹ΠΌΠΈ полосками ΠΈ Π½Π°Π·Π½Π°Ρ‡Π΅Π½ΠΈΠ΅ Π΅Π΅ ΠΌΠΎΠΆΠ½ΠΎ ΡƒΠ·Π½Π°Ρ‚ΡŒ здСсь.

ΠŸΠ°Ρ€Π°Π»Π»Π΅Π»ΡŒΠ½ΠΎΠ΅ соСдинСниС рСзисторов β€” ΠΎΠ½Π»Π°ΠΉΠ½ ΠΊΠ°Π»ΡŒΠΊΡƒΠ»ΡΡ‚ΠΎΡ€

Π§Ρ‚ΠΎΠ±Ρ‹ быстро Π²Ρ‹Ρ‡ΠΈΡΠ»ΠΈΡ‚ΡŒ ΠΎΠ±Ρ‰Π΅Π΅ сопротивлСниС Π΄Π²ΡƒΡ… ΠΈ Π±ΠΎΠ»Π΅Π΅ рСзисторов, соСдинСнных ΠΏΠ°Ρ€Π°Π»Π»Π΅Π»ΡŒΠ½ΠΎ, Π²Ρ‹ ΠΌΠΎΠΆΠ΅Ρ‚Π΅ Π²ΠΎΡΠΏΠΎΠ»ΡŒΠ·ΠΎΠ²Π°Ρ‚ΡŒΡΡ ΡΠ»Π΅Π΄ΡƒΡŽΡ‰ΠΈΠΌ ΠΎΠ½Π»Π°ΠΉΠ½ ΠΊΠ°Π»ΡŒΠΊΡƒΠ»ΡΡ‚ΠΎΡ€ΠΎΠΌ:

ПодвСдСм ΠΈΡ‚ΠΎΠ³

Когда Π΄Π²Π° ΠΈΠ»ΠΈ Π±ΠΎΠ»Π΅Π΅ рСзистора соСдинСны Ρ‚Π°ΠΊ, Ρ‡Ρ‚ΠΎ ΠΎΠ±Π° Π²Ρ‹Π²ΠΎΠ΄Π° ΠΎΠ΄Π½ΠΎΠ³ΠΎ рСзистора соСдинСны с ΡΠΎΠΎΡ‚Π²Π΅Ρ‚ΡΡ‚Π²ΡƒΡŽΡ‰ΠΈΠΌΠΈ Π²Ρ‹Π²ΠΎΠ΄Π°ΠΌΠΈ Π΄Ρ€ΡƒΠ³ΠΎΠ³ΠΎ рСзистора ΠΈΠ»ΠΈ рСзисторов, Ρ‚ΠΎ говорят, Ρ‡Ρ‚ΠΎ ΠΎΠ½ΠΈ соСдинСны ΠΌΠ΅ΠΆΠ΄Ρƒ собой ΠΏΠ°Ρ€Π°Π»Π»Π΅Π»ΡŒΠ½ΠΎ. НапряТСниС Π½Π° ΠΊΠ°ΠΆΠ΄ΠΎΠΌ рСзисторС Π²Π½ΡƒΡ‚Ρ€ΠΈ ΠΏΠ°Ρ€Π°Π»Π»Π΅Π»ΡŒΠ½ΠΎΠΉ ΠΊΠΎΠΌΠ±ΠΈΠ½Π°Ρ†ΠΈΠΈ ΠΎΠ΄ΠΈΠ½Π°ΠΊΠΎΠ²ΠΎΠ΅, Π½ΠΎ Ρ‚ΠΎΠΊΠΈ, ΠΏΡ€ΠΎΡ‚Π΅ΠΊΠ°ΡŽΡ‰ΠΈΠ΅ Ρ‡Π΅Ρ€Π΅Π· Π½ΠΈΡ…, ΠΌΠΎΠ³ΡƒΡ‚ ΠΎΡ‚Π»ΠΈΡ‡Π°Ρ‚ΡŒΡΡ Π΄Ρ€ΡƒΠ³ ΠΎΡ‚ Π΄Ρ€ΡƒΠ³Π°, Π² зависимости ΠΎΡ‚ Π²Π΅Π»ΠΈΡ‡ΠΈΠ½Ρ‹ сопротивлСний ΠΊΠ°ΠΆΠ΄ΠΎΠ³ΠΎ рСзистора.

Π­ΠΊΠ²ΠΈΠ²Π°Π»Π΅Π½Ρ‚Π½ΠΎΠ΅ ΠΈΠ»ΠΈ ΠΏΠΎΠ»Π½ΠΎΠ΅ сопротивлСниС ΠΏΠ°Ρ€Π°Π»Π»Π΅Π»ΡŒΠ½ΠΎΠΉ ΠΊΠΎΠΌΠ±ΠΈΠ½Π°Ρ†ΠΈΠΈ всСгда Π±ΡƒΠ΄Π΅Ρ‚ мСньшС минимального сопротивлСния рСзистора входящСго Π² ΠΏΠ°Ρ€Π°Π»Π»Π΅Π»ΡŒΠ½ΠΎΠ΅ соСдинСниС.

ΠŸΠΎΡΠ»Π΅Π΄ΠΎΠ²Π°Ρ‚Π΅Π»ΡŒΠ½ΠΎΠ΅ соСдинСниС источников питания

Π’Π΅ΠΏΠ΅Ρ€ΡŒ Π΄Π°Π²Π°ΠΉΡ‚Π΅ прСдставим Π²ΠΎΡ‚ Ρ‚Π°ΠΊΡƒΡŽ ΡΠΈΡ‚ΡƒΠ°Ρ†ΠΈΡŽ. Π§Ρ‚ΠΎ Π±ΡƒΠ΄Π΅Ρ‚, Ссли Π² нашСй ΠΎΠ±Ρ€Π΅Π·Π°Π½Π½ΠΎΠΉ водобашнС ΠΏΠΎΠ»Π½ΠΎΠΉ Π²ΠΎΠ΄Ρ‹ Π΄ΠΎΠ±Π°Π²ΠΈΠΌ Π΅Ρ‰Π΅ ΠΎΠ΄Π½Ρƒ Ρ‚Π°ΠΊΡƒΡŽ ΠΆΠ΅ свСрху ΠΏΠΎΠ»Π½ΡƒΡŽ Π²ΠΎΠ΄Ρ‹? БхСматичСски это Π±ΡƒΠ΄Π΅Ρ‚ Π²Ρ‹Π³Π»ΡΠ΄Π΅Ρ‚ΡŒ ΠΏΡ€ΠΈΠΌΠ΅Ρ€Π½ΠΎ Π²ΠΎΡ‚ Ρ‚Π°ΠΊ:

Как Π²Ρ‹ Π΄ΡƒΠΌΠ°Π΅Ρ‚Π΅, ΡƒΠΌΠ΅Π½ΡŒΡˆΠΈΡ‚ΡΡ Π΄Π°Π²Π»Π΅Π½ΠΈΠ΅ Π½Π° зСмлю, ΠΈΠ»ΠΈ увСличится? ΠŸΠΎΠ½ΡΡ‚Π½ΠΎΠ΅ Π΄Π΅Π»ΠΎ, Ρ‡Ρ‚ΠΎ увСличится! Π”Π° Π΅Ρ‰Π΅ ΠΈ Ρ€ΠΎΠ²Π½ΠΎ Π² Π΄Π²Π° Ρ€Π°Π·Π°! ΠŸΠΎΡ‡Π΅ΠΌΡƒ Ρ‚Π°ΠΊ ΠΏΡ€ΠΎΠΈΠ·ΠΎΡˆΠ»ΠΎ? Π£Ρ€ΠΎΠ²Π΅Π½ΡŒ Π²ΠΎΠ΄Ρ‹ стал Π²Ρ‹ΡˆΠ΅, ΡΠ»Π΅Π΄ΠΎΠ²Π°Ρ‚Π΅Π»ΡŒΠ½ΠΎ, Π΄Π°Π²Π»Π΅Π½ΠΈΠ΅ Π½Π° Π΄Π½ΠΎ ΡƒΠ²Π΅Π»ΠΈΡ‡ΠΈΠ»ΠΎΡΡŒ.

Если β€œΠΌΠΈΠ½ΡƒΡβ€ ΠΎΠ΄Π½ΠΎΠΉ Π±Π°Ρ‚Π°Ρ€Π΅ΠΉΠΊΠΈ ΡΠΎΠ΅Π΄ΠΈΠ½ΠΈΡ‚ΡŒ с β€œΠΏΠ»ΡŽΡΠΎΠΌβ€ Π΄Ρ€ΡƒΠ³ΠΎΠΉ Π±Π°Ρ‚Π°Ρ€Π΅ΠΉΠΊΠΈ, Ρ‚ΠΎ ΠΈΡ… ΠΎΠ±Ρ‰Π΅Π΅ напряТСниС суммируСтся.

ΠŸΠΎΠ»Π½ΠΎΡΡ‚ΡŒΡŽ заряТСнная Π±Π°Ρ‚Π°Ρ€Π΅ΠΉΠΊΠ° Π±ΡƒΠ΄Π΅Ρ‚ Π²Ρ‹Π³Π»ΡΠ΄Π΅Ρ‚ΡŒ ΠΊΠ°ΠΊ башня, ΠΏΠΎΠ»Π½ΠΎΡΡ‚ΡŒΡŽ залитая Π²ΠΎΠ΄ΠΎΠΉ с ΡƒΡ‡Π΅Ρ‚ΠΎΠΌ Ρ‚ΠΎΠ³ΠΎ, Ρ‡Ρ‚ΠΎ Ρ€Π°Π±ΠΎΡ‚Π°Π΅Ρ‚ насос автоматичСской ΠΏΠΎΠ΄Π°Ρ‡ΠΈ Π²ΠΎΠ΄Ρ‹. По Π°Π½Π°Π»ΠΎΠ³ΠΈΠΈ, насос – это Π­Π”Π‘.

Наполовину разряТСнная Π±Π°Ρ‚Π°Ρ€Π΅ΠΉΠΊΠ° Π±ΡƒΠ΄Π΅Ρ‚ ΡƒΠΆΠ΅ Π²Ρ‹Π³Π»ΡΠ΄Π΅Ρ‚ΡŒ ΠΏΡ€ΠΈΠΌΠ΅Ρ€Π½ΠΎ Π²ΠΎΡ‚ Ρ‚Π°ΠΊ:

МоТно ΡΠΊΠ°Π·Π°Ρ‚ΡŒ, насос ΡƒΠΆΠ΅ Π½Π΅ справляСтся.

Π‘Π°Ρ‚Π°Ρ€Π΅ΠΉΠΊΠ° посаТСнная Π² β€œΠ½ΠΎΠ»ΡŒβ€ Π±ΡƒΠ΄Π΅Ρ‚ Π²Ρ‹Π³Π»ΡΠ΄Π΅Ρ‚ΡŒ Π²ΠΎΡ‚ Ρ‚Π°ΠΊ:

Насос автоматичСской ΠΏΠΎΠ΄Π°Ρ‡ΠΈ Π²ΠΎΠ΄Ρ‹ сломался.

ЕстСствСнно, Ρ‡Ρ‚ΠΎ Ссли Π²Ρ‹ соСдинитС ΠΏΠΎΠ»Π½ΠΎΡΡ‚ΡŒΡŽ Π·Π°Ρ€ΡΠΆΠ΅Π½Π½ΡƒΡŽ ΠΈ Π½Π°ΠΏΠΎΠ»ΠΎΠ²ΠΈΠ½Ρƒ Π΄ΠΎΡ…Π»ΡƒΡŽ Π±Π°Ρ‚Π°Ρ€Π΅ΠΉΠΊΡƒ ΠΏΠΎΡΠ»Π΅Π΄ΠΎΠ²Π°Ρ‚Π΅Π»ΡŒΠ½ΠΎ, Ρ‚ΠΎ ΠΈΡ… ΠΎΠ±Ρ‰Π΅Π΅ напряТСниС Π±ΡƒΠ΄Π΅Ρ‚ Π²Ρ‹Π³Π»ΡΠ΄Π΅Ρ‚ΡŒ ΠΏΡ€ΠΈΠΌΠ΅Ρ€Π½ΠΎ Π²ΠΎΡ‚ Ρ‚Π°ΠΊ:

Π”Π°Π²Π°ΠΉΡ‚Π΅ всС это продСмонстрируСм Π½Π° ΠΏΡ€Π°ΠΊΡ‚ΠΈΠΊΠ΅. Π˜Ρ‚Π°ΠΊ, Ρƒ нас Π΅ΡΡ‚ΡŒ 2 Π»ΠΈΡ‚ΠΈΠΉ-ΠΈΠΎΠ½Π½Ρ‹Ρ… аккумулятора. Π― ΠΈΡ… ΠΏΠΎΠΌΠ΅Ρ‚ΠΈΠ» Ρ†ΠΈΡ„Ρ€Π°ΠΌΠΈ 1 ΠΈ 2. Π‘ плюса ΠΊΠ°ΠΆΠ΄ΠΎΠ³ΠΎ аккумулятора я Π²Ρ‹Π²Π΅Π» красный ΠΏΡ€ΠΎΠ²ΠΎΠ΄, Π° с минуса – Ρ‡Π΅Ρ€Π½Ρ‹ΠΉ.

Π”Π°Π²Π°ΠΉΡ‚Π΅ замСряСм напряТСниС аккумулятора ΠΏΠΎΠ΄ β„–1 с ΠΏΠΎΠΌΠΎΡ‰ΡŒΡŽ ΠΌΡƒΠ»ΡŒΡ‚ΠΈΠΌΠ΅Ρ‚Ρ€Π°. Как это ΡΠ΄Π΅Π»Π°Ρ‚ΡŒ, я Π΅Ρ‰Π΅ писал Π² ΡΡ‚Π°Ρ‚ΡŒΠ΅ Как ΠΈΠ·ΠΌΠ΅Ρ€ΠΈΡ‚ΡŒ Ρ‚ΠΎΠΊ ΠΈ напряТСниС ΠΌΡƒΠ»ΡŒΡ‚ΠΈΠΌΠ΅Ρ‚Ρ€ΠΎΠΌ.

На ΠΏΠ΅Ρ€Π²ΠΎΠΌ аккумуляторС Ρƒ нас напряТСниС 3,66 Π’ΠΎΠ»ΡŒΡ‚. Π­Ρ‚ΠΎ Ρ‚ΠΈΠΏΠΈΡ‡Π½ΠΎΠ΅ Π·Π½Π°Ρ‡Π΅Π½ΠΈΠ΅ Π»ΠΈΡ‚ΠΈΠΉ-ΠΈΠΎΠ½Π½ΠΎΠ³ΠΎ аккумулятора.

Π’Π°ΠΊΠΈΠΌ ΠΆΠ΅ способом замСряСм напряТСниС Π½Π° аккумуляторС β„–2

О, ΠΊΠ°ΠΊ совпало). Π’Π΅ ΠΆΠ΅ самыС 3,66 Π’ΠΎΠ»ΡŒΡ‚.

Для Ρ‚ΠΎΠ³ΠΎ, Ρ‡Ρ‚ΠΎΠ±Ρ‹ ΡΠΎΠ΅Π΄ΠΈΠ½ΠΈΡ‚ΡŒ ΠΏΠΎΡΠ»Π΅Π΄ΠΎΠ²Π°Ρ‚Π΅Π»ΡŒΠ½ΠΎ эти аккумуляторы, Π½Π°ΠΌ Π½Π°Π΄ΠΎ ΡΠ΄Π΅Π»Π°Ρ‚ΡŒ Ρ‡Ρ‚ΠΎ-Ρ‚ΠΎ ΠΏΠΎΠ΄ΠΎΠ±Π½ΠΎΠ΅:

Π’Π°ΠΊΠΆΠ΅ ΠΊΠ°ΠΊ ΠΈ Π² Π±Π°ΡˆΠ½ΡΡ…, Π½Π°ΠΌ Π½Π°Π΄ΠΎ ΡΠΎΠ΅Π΄ΠΈΠ½ΠΈΡ‚ΡŒ основаниС ΠΎΠ΄Π½ΠΎΠΉ башни с Π²Π΅Ρ€Ρ…ΡƒΡˆΠΊΠΎΠΉ Π΄Ρ€ΡƒΠ³ΠΎΠΉ башни. Π’ источниках питания, Ρ‚ΠΈΠΏΠ° аккумуляторов ΠΈΠ»ΠΈ Π±Π°Ρ‚Π°Ρ€Π΅Π΅ΠΊ, Π½Π°ΠΌ Π½Π°Π΄ΠΎ ΡΠΎΠ΅Π΄ΠΈΠ½ΠΈΡ‚ΡŒ минус ΠΎΠ΄Π½ΠΎΠΉ Π±Π°Ρ‚Π°Ρ€Π΅ΠΉΠΊΠΈ с плюсом Π΄Ρ€ΡƒΠ³ΠΎΠΉ. Π’Π°ΠΊ ΠΌΡ‹ ΠΈ сдСлаСм. БоСдиняСм плюс ΠΎΠ΄Π½ΠΎΠΉ Π±Π°Ρ‚Π°Ρ€Π΅ΠΉΠΊΠΈ с минусом Π΄Ρ€ΡƒΠ³ΠΎΠΉ ΠΈ получаСм… сумму напряТСний ΠΊΠ°ΠΆΠ΄ΠΎΠΉ Π±Π°Ρ‚Π°Ρ€Π΅ΠΉΠΊΠΈ! Как Π²Ρ‹ ΠΏΠΎΠΌΠ½ΠΈΡ‚Π΅, Π½Π° ΠΏΠ΅Ρ€Π²ΠΎΠΉ Π±Π°Ρ‚Π°Ρ€Π΅ΠΉΠΊΠ΅ Ρƒ нас Π±Ρ‹Π»ΠΎ напряТСниС 3,66 Π’, Π½Π° Π²Ρ‚ΠΎΡ€ΠΎΠΉ Ρ‚ΠΎΠΆΠ΅ 3,66 Π’. 3,66+3,6=7,32 Π’.

ΠœΡƒΠ»ΡŒΡ‚ΠΈΠΌΠ΅Ρ‚Ρ€ ΠΏΠΎΠΊΠ°Π·Ρ‹Π²Π°Π΅Ρ‚ 7,33 Π’. 0,01Π’ спишСм Π½Π° ΠΏΠΎΠ³Ρ€Π΅ΡˆΠ½ΠΎΡΡ‚ΡŒ ΠΈΠ·ΠΌΠ΅Ρ€Π΅Π½ΠΈΠΉ.

Π­Ρ‚ΠΎ свойство ΠΏΡ€ΠΎΠΊΠ°Ρ‚Ρ‹Π²Π°Π΅Ρ‚ Π½Π΅ Ρ‚ΠΎΠ»ΡŒΠΊΠΎ с двумя аккумуляторами, Π½ΠΎ Ρ‚Π°ΠΊΠΆΠ΅ с ΠΈΡ… бСсконСчным мноТСством. Π”ΡƒΠΌΠ°ΡŽ, Π½Π΅ стоит Π³ΠΎΠ²ΠΎΡ€ΠΈΡ‚ΡŒ, Ρ‡Ρ‚ΠΎ Ссли Π²Ρ‹ΡΡ‚Π°Π²ΠΈΡ‚ΡŒ Π² ряд ΡˆΡ‚ΡƒΠΊ 100 Ρ‚Π°ΠΊΠΈΡ… аккумуляторов, ΡΠΎΠ΅Π΄ΠΈΠ½ΠΈΡ‚ΡŒ ΠΏΠΎΡΠ»Π΅Π΄ΠΎΠ²Π°Ρ‚Π΅Π»ΡŒΠ½ΠΎ ΠΈ ΠΊΠΎΡΠ½ΡƒΡ‚ΡŒΡΡ ΠΊΡ€Π°ΠΉΠ½ΠΈΡ… полюсов Π³ΠΎΠ»Ρ‹ΠΌΠΈ Ρ€ΡƒΠΊΠ°ΠΌΠΈ, Ρ‚ΠΎ всС это ΠΌΠΎΠΆΠ΅Ρ‚ Π·Π°Π²Π΅Ρ€ΡˆΠΈΡ‚ΡŒΡΡ Π΄Π°ΠΆΠ΅ Π»Π΅Ρ‚Π°Π»ΡŒΠ½Ρ‹ΠΌ исходом.

ΠžΡ†Π΅Π½ΠΈΡ‚Π΅ ΡΡ‚Π°Ρ‚ΡŒΡŽ:

Онлайн ΠΊΠ°Π»ΡŒΠΊΡƒΠ»ΡΡ‚ΠΎΡ€ расчСта ΠΏΠ°Ρ€Π°Π»Π»Π΅Π»ΡŒΠ½ΠΎΠ³ΠΎ соСдинСния рСзисторов

ΠŸΠΎΡΠ»Π΅Π΄ΠΎΠ²Π°Ρ‚Π΅Π»ΡŒΠ½ΠΎΠ΅ ΠΈ ΠΏΠ°Ρ€Π°Π»Π»Π΅Π»ΡŒΠ½ΠΎΠ΅ соСдинСниС. ΠŸΡ€ΠΈΠΌΠ΅Π½Π΅Π½ΠΈΠ΅ ΠΈ схСмы

Π’ элСктричСских цСпях элСмСнты ΠΌΠΎΠ³ΡƒΡ‚ ΡΠΎΠ΅Π΄ΠΈΠ½ΡΡ‚ΡŒΡΡ ΠΏΠΎ Ρ€Π°Π·Π»ΠΈΡ‡Π½Ρ‹ΠΌ схСмам, Π² Ρ‚ΠΎΠΌ числС ΠΎΠ½ΠΈ ΠΈΠΌΠ΅ΡŽΡ‚ ΠΏΠΎΡΠ»Π΅Π΄ΠΎΠ²Π°Ρ‚Π΅Π»ΡŒΠ½ΠΎΠ΅ ΠΈ ΠΏΠ°Ρ€Π°Π»Π»Π΅Π»ΡŒΠ½ΠΎΠ΅ соСдинСниС.

ΠŸΠΎΡΠ»Π΅Π΄ΠΎΠ²Π°Ρ‚Π΅Π»ΡŒΠ½ΠΎΠ΅ соСдинСниС

ΠŸΡ€ΠΈ Ρ‚Π°ΠΊΠΎΠΌ соСдинСнии ΠΏΡ€ΠΎΠ²ΠΎΠ΄Π½ΠΈΠΊΠΈ ΡΠΎΠ΅Π΄ΠΈΠ½ΡΡŽΡ‚ΡΡ Π΄Ρ€ΡƒΠ³ с Π΄Ρ€ΡƒΠ³ΠΎΠΌ ΠΏΠΎΡΠ»Π΅Π΄ΠΎΠ²Π°Ρ‚Π΅Π»ΡŒΠ½ΠΎ, Ρ‚ΠΎ Π΅ΡΡ‚ΡŒ, Π½Π°Ρ‡Π°Π»ΠΎ ΠΎΠ΄Π½ΠΎΠ³ΠΎ ΠΏΡ€ΠΎΠ²ΠΎΠ΄Π½ΠΈΠΊΠ° Π±ΡƒΠ΄Π΅Ρ‚ ΡΠΎΠ΅Π΄ΠΈΠ½ΡΡ‚ΡŒΡΡ с ΠΊΠΎΠ½Ρ†ΠΎΠΌ Π΄Ρ€ΡƒΠ³ΠΎΠ³ΠΎ. Основная ΠΎΡΠΎΠ±Π΅Π½Π½ΠΎΡΡ‚ΡŒ Π΄Π°Π½Π½ΠΎΠ³ΠΎ соСдинСния Π·Π°ΠΊΠ»ΡŽΡ‡Π°Π΅Ρ‚ΡΡ Π² Ρ‚ΠΎΠΌ, Ρ‡Ρ‚ΠΎ всС ΠΏΡ€ΠΎΠ²ΠΎΠ΄Π½ΠΈΠΊΠΈ ΠΏΡ€ΠΈΠ½Π°Π΄Π»Π΅ΠΆΠ°Ρ‚ ΠΎΠ΄Π½ΠΎΠΌΡƒ ΠΏΡ€ΠΎΠ²ΠΎΠ΄Ρƒ, Π½Π΅Ρ‚ Π½ΠΈΠΊΠ°ΠΊΠΈΡ… Ρ€Π°Π·Π²Π΅Ρ‚Π²Π»Π΅Π½ΠΈΠΉ. Π§Π΅Ρ€Π΅Π· ΠΊΠ°ΠΆΠ΄Ρ‹ΠΉ ΠΈΠ· ΠΏΡ€ΠΎΠ²ΠΎΠ΄Π½ΠΈΠΊΠΎΠ² Π±ΡƒΠ΄Π΅Ρ‚ ΠΏΡ€ΠΎΡ‚Π΅ΠΊΠ°Ρ‚ΡŒ ΠΎΠ΄ΠΈΠ½ ΠΈ Ρ‚ΠΎΡ‚ ΠΆΠ΅ элСктричСский Ρ‚ΠΎΠΊ. Но суммарноС напряТСниС Π½Π° ΠΏΡ€ΠΎΠ²ΠΎΠ΄Π½ΠΈΠΊΠ°Ρ… Π±ΡƒΠ΄Π΅Ρ‚ Ρ€Π°Π²Π½ΡΡ‚ΡŒΡΡ вмСстС взятым напряТСниям Π½Π° ΠΊΠ°ΠΆΠ΄ΠΎΠΌ ΠΈΠ· Π½ΠΈΡ….

Рассмотрим Π½Π΅ΠΊΠΎΡ‚ΠΎΡ€ΠΎΠ΅ количСство рСзисторов, соСдинСнных ΠΏΠΎΡΠ»Π΅Π΄ΠΎΠ²Π°Ρ‚Π΅Π»ΡŒΠ½ΠΎ. Π’Π°ΠΊ ΠΊΠ°ΠΊ Π½Π΅Ρ‚ Ρ€Π°Π·Π²Π΅Ρ‚Π²Π»Π΅Π½ΠΈΠΉ, Ρ‚ΠΎ количСство проходящСго заряда Ρ‡Π΅Ρ€Π΅Π· ΠΎΠ΄ΠΈΠ½ ΠΏΡ€ΠΎΠ²ΠΎΠ΄Π½ΠΈΠΊ, Π±ΡƒΠ΄Π΅Ρ‚ Ρ€Π°Π²Π½ΠΎ количСству заряда, ΠΏΡ€ΠΎΡˆΠ΅Π΄ΡˆΠ΅Π³ΠΎ Ρ‡Π΅Ρ€Π΅Π· Π΄Ρ€ΡƒΠ³ΠΎΠΉ ΠΏΡ€ΠΎΠ²ΠΎΠ΄Π½ΠΈΠΊ. Π‘ΠΈΠ»Ρ‹ Ρ‚ΠΎΠΊΠ° Π½Π° всСх ΠΏΡ€ΠΎΠ²ΠΎΠ΄Π½ΠΈΠΊΠ°Ρ… Π±ΡƒΠ΄ΡƒΡ‚ ΠΎΠ΄ΠΈΠ½Π°ΠΊΠΎΠ²Ρ‹ΠΌΠΈ. Π­Ρ‚ΠΎ основная ΠΎΡΠΎΠ±Π΅Π½Π½ΠΎΡΡ‚ΡŒ Π΄Π°Π½Π½ΠΎΠ³ΠΎ соСдинСния.

Π­Ρ‚ΠΎ соСдинСниС ΠΌΠΎΠΆΠ½ΠΎ Ρ€Π°ΡΡΠΌΠΎΡ‚Ρ€Π΅Ρ‚ΡŒ ΠΈΠ½Π°Ρ‡Π΅. ВсС рСзисторы ΠΌΠΎΠΆΠ½ΠΎ Π·Π°ΠΌΠ΅Π½ΠΈΡ‚ΡŒ ΠΎΠ΄Π½ΠΈΠΌ эквивалСнтным рСзистором.

Π’ΠΎΠΊ Π½Π° эквивалСнтном рСзисторС Π±ΡƒΠ΄Π΅Ρ‚ ΡΠΎΠ²ΠΏΠ°Π΄Π°Ρ‚ΡŒ с ΠΎΠ±Ρ‰ΠΈΠΌ Ρ‚ΠΎΠΊΠΎΠΌ, ΠΏΡ€ΠΎΡ‚Π΅ΠΊΠ°ΡŽΡ‰ΠΈΠΌ Ρ‡Π΅Ρ€Π΅Π· всС рСзисторы. Π­ΠΊΠ²ΠΈΠ²Π°Π»Π΅Π½Ρ‚Π½ΠΎΠ΅ ΠΎΠ±Ρ‰Π΅Π΅ напряТСниС Π±ΡƒΠ΄Π΅Ρ‚ ΡΠΊΠ»Π°Π΄Ρ‹Π²Π°Ρ‚ΡŒΡΡ ΠΈΠ· напряТСний Π½Π° ΠΊΠ°ΠΆΠ΄ΠΎΠΌ рСзисторС. Π­Ρ‚ΠΎ являСтся Ρ€Π°Π·Π½ΠΎΡΡ‚ΡŒΡŽ ΠΏΠΎΡ‚Π΅Π½Ρ†ΠΈΠ°Π»ΠΎΠ² Π½Π° рСзисторС.

Если Π²ΠΎΡΠΏΠΎΠ»ΡŒΠ·ΠΎΠ²Π°Ρ‚ΡŒΡΡ этими ΠΏΡ€Π°Π²ΠΈΠ»Π°ΠΌΠΈ ΠΈ Π·Π°ΠΊΠΎΠ½ΠΎΠΌ Ома, ΠΊΠΎΡ‚ΠΎΡ€Ρ‹ΠΉ ΠΏΠΎΠ΄Ρ…ΠΎΠ΄ΠΈΡ‚ для ΠΊΠ°ΠΆΠ΄ΠΎΠ³ΠΎ рСзистора, ΠΌΠΎΠΆΠ½ΠΎ Π΄ΠΎΠΊΠ°Π·Π°Ρ‚ΡŒ, Ρ‡Ρ‚ΠΎ сопротивлСниС эквивалСнтного ΠΎΠ±Ρ‰Π΅Π³ΠΎ рСзистора Π±ΡƒΠ΄Π΅Ρ‚ Ρ€Π°Π²Π½ΠΎ суммС сопротивлСний. БлСдствиСм ΠΏΠ΅Ρ€Π²Ρ‹Ρ… Π΄Π²ΡƒΡ… ΠΏΡ€Π°Π²ΠΈΠ» Π±ΡƒΠ΄Π΅Ρ‚ ΡΠ²Π»ΡΡ‚ΡŒΡΡ Ρ‚Ρ€Π΅Ρ‚ΡŒΠ΅ ΠΏΡ€Π°Π²ΠΈΠ»ΠΎ.

ΠŸΡ€ΠΈΠΌΠ΅Π½Π΅Π½ΠΈΠ΅

ΠŸΠΎΡΠ»Π΅Π΄ΠΎΠ²Π°Ρ‚Π΅Π»ΡŒΠ½ΠΎΠ΅ соСдинСниС ΠΈΡΠΏΠΎΠ»ΡŒΠ·ΡƒΠ΅Ρ‚ΡΡ, ΠΊΠΎΠ³Π΄Π° Π½ΡƒΠΆΠ½ΠΎ Ρ†Π΅Π»Π΅Π½Π°ΠΏΡ€Π°Π²Π»Π΅Π½Π½ΠΎ Π²ΠΊΠ»ΡŽΡ‡Π°Ρ‚ΡŒ ΠΈΠ»ΠΈ Π²Ρ‹ΠΊΠ»ΡŽΡ‡Π°Ρ‚ΡŒ ΠΊΠ°ΠΊΠΎΠΉ-Π»ΠΈΠ±ΠΎ ΠΏΡ€ΠΈΠ±ΠΎΡ€, Π²Ρ‹ΠΊΠ»ΡŽΡ‡Π°Ρ‚Π΅Π»ΡŒ ΡΠΎΠ΅Π΄ΠΈΠ½ΡΡŽΡ‚ с Π½ΠΈΠΌ ΠΏΠΎ ΠΏΠΎΡΠ»Π΅Π΄ΠΎΠ²Π°Ρ‚Π΅Π»ΡŒΠ½ΠΎΠΉ схСмС. НапримСр, элСктричСский Π·Π²ΠΎΠ½ΠΎΠΊ Π±ΡƒΠ΄Π΅Ρ‚ Π·Π²Π΅Π½Π΅Ρ‚ΡŒ Ρ‚ΠΎΠ»ΡŒΠΊΠΎ Ρ‚ΠΎΠ³Π΄Π°, ΠΊΠΎΠ³Π΄Π° ΠΎΠ½ Π±ΡƒΠ΄Π΅Ρ‚ ΠΏΠΎΡΠ»Π΅Π΄ΠΎΠ²Π°Ρ‚Π΅Π»ΡŒΠ½ΠΎ соСдинСн с источником ΠΈ ΠΊΠ½ΠΎΠΏΠΊΠΎΠΉ. Богласно ΠΏΠ΅Ρ€Π²ΠΎΠΌΡƒ ΠΏΡ€Π°Π²ΠΈΠ»Ρƒ, Ссли элСктричСский Ρ‚ΠΎΠΊ отсутствуСт хотя Π±Ρ‹ Π½Π° ΠΎΠ΄Π½ΠΎΠΌ ΠΈΠ· ΠΏΡ€ΠΎΠ²ΠΎΠ΄Π½ΠΈΠΊΠΎΠ², Ρ‚ΠΎ Π΅Π³ΠΎ Π½Π΅ Π±ΡƒΠ΄Π΅Ρ‚ ΠΈ Π½Π° Π΄Ρ€ΡƒΠ³ΠΈΡ… ΠΏΡ€ΠΎΠ²ΠΎΠ΄Π½ΠΈΠΊΠ°Ρ…. И Π½Π°ΠΎΠ±ΠΎΡ€ΠΎΡ‚, Ссли Ρ‚ΠΎΠΊ имССтся хотя Π±Ρ‹ Π½Π° ΠΎΠ΄Π½ΠΎΠΌ ΠΏΡ€ΠΎΠ²ΠΎΠ΄Π½ΠΈΠΊΠ΅, Ρ‚ΠΎ ΠΎΠ½ Π±ΡƒΠ΄Π΅Ρ‚ ΠΈ Π½Π° всСх Π΄Ρ€ΡƒΠ³ΠΈΡ… ΠΏΡ€ΠΎΠ²ΠΎΠ΄Π½ΠΈΠΊΠ°Ρ…. Π’Π°ΠΊΠΆΠ΅ Ρ€Π°Π±ΠΎΡ‚Π°Π΅Ρ‚ ΠΊΠ°Ρ€ΠΌΠ°Π½Π½Ρ‹ΠΉ Ρ„ΠΎΠ½Π°Ρ€ΠΈΠΊ, Π² ΠΊΠΎΡ‚ΠΎΡ€ΠΎΠΌ Π΅ΡΡ‚ΡŒ ΠΊΠ½ΠΎΠΏΠΊΠ°, Π±Π°Ρ‚Π°Ρ€Π΅ΠΉΠΊΠ° ΠΈ Π»Π°ΠΌΠΏΠΎΡ‡ΠΊΠ°. ВсС эти элСмСнты Π½Π΅ΠΎΠ±Ρ…ΠΎΠ΄ΠΈΠΌΠΎ ΡΠΎΠ΅Π΄ΠΈΠ½ΠΈΡ‚ΡŒ ΠΏΠΎΡΠ»Π΅Π΄ΠΎΠ²Π°Ρ‚Π΅Π»ΡŒΠ½ΠΎ, Ρ‚Π°ΠΊ ΠΊΠ°ΠΊ Π½ΡƒΠΆΠ½ΠΎ, Ρ‡Ρ‚ΠΎΠ±Ρ‹ Ρ„ΠΎΠ½Π°Ρ€ΠΈΠΊ свСтил, ΠΊΠΎΠ³Π΄Π° Π±ΡƒΠ΄Π΅Ρ‚ Π½Π°ΠΆΠ°Ρ‚Π° ΠΊΠ½ΠΎΠΏΠΊΠ°.

Иногда ΠΏΠΎΡΠ»Π΅Π΄ΠΎΠ²Π°Ρ‚Π΅Π»ΡŒΠ½ΠΎΠ΅ соСдинСниС Π½Π΅ ΠΏΡ€ΠΈΠ²ΠΎΠ΄ΠΈΡ‚ ΠΊ Π½ΡƒΠΆΠ½Ρ‹ΠΌ цСлям. НапримСр, Π² ΠΊΠ²Π°Ρ€Ρ‚ΠΈΡ€Π΅, Π³Π΄Π΅ ΠΌΠ½ΠΎΠ³ΠΎ Π»ΡŽΡΡ‚Ρ€, Π»Π°ΠΌΠΏΠΎΡ‡Π΅ΠΊ ΠΈ Π΄Ρ€ΡƒΠ³ΠΈΡ… устройств, Π½Π΅ слСдуСт всС Π»Π°ΠΌΠΏΡ‹ ΠΈ устройства ΡΠΎΠ΅Π΄ΠΈΠ½ΡΡ‚ΡŒ ΠΏΠΎΡΠ»Π΅Π΄ΠΎΠ²Π°Ρ‚Π΅Π»ΡŒΠ½ΠΎ, Ρ‚Π°ΠΊ ΠΊΠ°ΠΊ Π½ΠΈΠΊΠΎΠ³Π΄Π° Π½Π΅ трСбуСтся ΠΎΠ΄Π½ΠΎΠ²Ρ€Π΅ΠΌΠ΅Π½Π½ΠΎ Π²ΠΊΠ»ΡŽΡ‡Π°Ρ‚ΡŒ свСт Π² ΠΊΠ°ΠΆΠ΄ΠΎΠΉ ΠΈΠ· ΠΊΠΎΠΌΠ½Π°Ρ‚ ΠΊΠ²Π°Ρ€Ρ‚ΠΈΡ€Ρ‹. Для этого ΠΏΠΎΡΠ»Π΅Π΄ΠΎΠ²Π°Ρ‚Π΅Π»ΡŒΠ½ΠΎΠ΅ ΠΈ ΠΏΠ°Ρ€Π°Π»Π»Π΅Π»ΡŒΠ½ΠΎΠ΅ соСдинСниС Ρ€Π°ΡΡΠΌΠ°Ρ‚Ρ€ΠΈΠ²Π°ΡŽΡ‚ ΠΎΡ‚Π΄Π΅Π»ΡŒΠ½ΠΎ, ΠΈ для ΠΏΠΎΠ΄ΠΊΠ»ΡŽΡ‡Π΅Π½ΠΈΡ ΠΎΡΠ²Π΅Ρ‚ΠΈΡ‚Π΅Π»ΡŒΠ½Ρ‹Ρ… ΠΏΡ€ΠΈΠ±ΠΎΡ€ΠΎΠ² Π² ΠΊΠ²Π°Ρ€Ρ‚ΠΈΡ€Π΅ ΠΏΡ€ΠΈΠΌΠ΅Π½ΡΡŽΡ‚ ΠΏΠ°Ρ€Π°Π»Π»Π΅Π»ΡŒΠ½Ρ‹ΠΉ Π²ΠΈΠ΄ схСмы.

ΠŸΠ°Ρ€Π°Π»Π»Π΅Π»ΡŒΠ½ΠΎΠ΅ соСдинСниС

Π’ этом Π²ΠΈΠ΄Π΅ схСмы всС ΠΏΡ€ΠΎΠ²ΠΎΠ΄Π½ΠΈΠΊΠΈ ΡΠΎΠ΅Π΄ΠΈΠ½ΡΡŽΡ‚ΡΡ ΠΏΠ°Ρ€Π°Π»Π»Π΅Π»ΡŒΠ½ΠΎ Π΄Ρ€ΡƒΠ³ с Π΄Ρ€ΡƒΠ³ΠΎΠΌ. ВсС Π½Π°Ρ‡Π°Π»Π° ΠΏΡ€ΠΎΠ²ΠΎΠ΄Π½ΠΈΠΊΠΎΠ² ΠΎΠ±ΡŠΠ΅Π΄ΠΈΠ½Π΅Π½Ρ‹ Π² ΠΎΠ΄Π½Ρƒ Ρ‚ΠΎΡ‡ΠΊΡƒ, ΠΈ всС ΠΊΠΎΠ½Ρ†Ρ‹ Ρ‚Π°ΠΊΠΆΠ΅ соСдинСны вмСстС. Рассмотрим Π½Π΅ΠΊΠΎΡ‚ΠΎΡ€ΠΎΠ΅ количСство ΠΎΠ΄Π½ΠΎΡ€ΠΎΠ΄Π½Ρ‹Ρ… ΠΏΡ€ΠΎΠ²ΠΎΠ΄Π½ΠΈΠΊΠΎΠ² (рСзисторов), соСдинСнных ΠΏΠΎ ΠΏΠ°Ρ€Π°Π»Π»Π΅Π»ΡŒΠ½ΠΎΠΉ схСмС.

Π­Ρ‚ΠΎΡ‚ Π²ΠΈΠ΄ соСдинСния являСтся Ρ€Π°Π·Π²Π΅Ρ‚Π²Π»Π΅Π½Π½Ρ‹ΠΌ. Π’ ΠΊΠ°ΠΆΠ΄ΠΎΠΉ Π²Π΅Ρ‚Π²ΠΈ содСрТится ΠΏΠΎ ΠΎΠ΄Π½ΠΎΠΌΡƒ рСзистору. ЭлСктричСский Ρ‚ΠΎΠΊ, дойдя Π΄ΠΎ Ρ‚ΠΎΡ‡ΠΊΠΈ развСтвлСния, раздСляСтся Π½Π° ΠΊΠ°ΠΆΠ΄Ρ‹ΠΉ рСзистор, ΠΈ Π±ΡƒΠ΄Π΅Ρ‚ Ρ€Π°Π²Π½ΡΡ‚ΡŒΡΡ суммС Ρ‚ΠΎΠΊΠΎΠ² Π½Π° всСх сопротивлСниях. НапряТСниС Π½Π° всСх элСмСнтах, соСдинСнных ΠΏΠ°Ρ€Π°Π»Π»Π΅Π»ΡŒΠ½ΠΎ, являСтся ΠΎΠ΄ΠΈΠ½Π°ΠΊΠΎΠ²Ρ‹ΠΌ.

ВсС рСзисторы ΠΌΠΎΠΆΠ½ΠΎ Π·Π°ΠΌΠ΅Π½ΠΈΡ‚ΡŒ ΠΎΠ΄Π½ΠΈΠΌ эквивалСнтным рСзистором. Если Π²ΠΎΡΠΏΠΎΠ»ΡŒΠ·ΠΎΠ²Π°Ρ‚ΡŒΡΡ Π·Π°ΠΊΠΎΠ½ΠΎΠΌ Ома, ΠΌΠΎΠΆΠ½ΠΎ ΠΏΠΎΠ»ΡƒΡ‡ΠΈΡ‚ΡŒ Π²Ρ‹Ρ€Π°ΠΆΠ΅Π½ΠΈΠ΅ сопротивлСния. Если ΠΏΡ€ΠΈ ΠΏΠΎΡΠ»Π΅Π΄ΠΎΠ²Π°Ρ‚Π΅Π»ΡŒΠ½ΠΎΠΌ соСдинСнии сопротивлСния ΡΠΊΠ»Π°Π΄Ρ‹Π²Π°Π»ΠΈΡΡŒ, Ρ‚ΠΎ ΠΏΡ€ΠΈ ΠΏΠ°Ρ€Π°Π»Π»Π΅Π»ΡŒΠ½ΠΎΠΌ Π±ΡƒΠ΄ΡƒΡ‚ ΡΠΊΠ»Π°Π΄Ρ‹Π²Π°Ρ‚ΡŒΡΡ Π²Π΅Π»ΠΈΡ‡ΠΈΠ½Ρ‹ ΠΎΠ±Ρ€Π°Ρ‚Π½Ρ‹Π΅ ΠΈΠΌ, ΠΊΠ°ΠΊ записано Π² Ρ„ΠΎΡ€ΠΌΡƒΠ»Π΅ Π²Ρ‹ΡˆΠ΅.

ΠŸΡ€ΠΈΠΌΠ΅Π½Π΅Π½ΠΈΠ΅

Если Ρ€Π°ΡΡΠΌΠ°Ρ‚Ρ€ΠΈΠ²Π°Ρ‚ΡŒ соСдинСния Π² Π±Ρ‹Ρ‚ΠΎΠ²Ρ‹Ρ… условиях, Ρ‚ΠΎ Π² ΠΊΠ²Π°Ρ€Ρ‚ΠΈΡ€Π΅ Π»Π°ΠΌΠΏΡ‹ освСщСния, Π»ΡŽΡΡ‚Ρ€Ρ‹ Π΄ΠΎΠ»ΠΆΠ½Ρ‹ Π±Ρ‹Ρ‚ΡŒ соСдинСны ΠΏΠ°Ρ€Π°Π»Π»Π΅Π»ΡŒΠ½ΠΎ. Если ΠΈΡ… ΡΠΎΠ΅Π΄ΠΈΠ½ΠΈΡ‚ΡŒ ΠΏΠΎΡΠ»Π΅Π΄ΠΎΠ²Π°Ρ‚Π΅Π»ΡŒΠ½ΠΎ, Ρ‚ΠΎ ΠΏΡ€ΠΈ Π²ΠΊΠ»ΡŽΡ‡Π΅Π½ΠΈΠΈ ΠΎΠ΄Π½ΠΎΠΉ Π»Π°ΠΌΠΏΠΎΡ‡ΠΊΠΈ ΠΌΡ‹ Π²ΠΊΠ»ΡŽΡ‡ΠΈΠΌ всС ΠΎΡΡ‚Π°Π»ΡŒΠ½Ρ‹Π΅. ΠŸΡ€ΠΈ ΠΏΠ°Ρ€Π°Π»Π»Π΅Π»ΡŒΠ½ΠΎΠΌ ΠΆΠ΅ соСдинСнии ΠΌΡ‹ ΠΌΠΎΠΆΠ΅ΠΌ, добавляя ΡΠΎΠΎΡ‚Π²Π΅Ρ‚ΡΡ‚Π²ΡƒΡŽΡ‰ΠΈΠΉ Π²Ρ‹ΠΊΠ»ΡŽΡ‡Π°Ρ‚Π΅Π»ΡŒ Π² ΠΊΠ°ΠΆΠ΄ΡƒΡŽ ΠΈΠ· Π²Π΅Ρ‚Π²Π΅ΠΉ, Π²ΠΊΠ»ΡŽΡ‡Π°Ρ‚ΡŒ ΡΠΎΠΎΡ‚Π²Π΅Ρ‚ΡΡ‚Π²ΡƒΡŽΡ‰ΡƒΡŽ Π»Π°ΠΌΠΏΠΎΡ‡ΠΊΡƒ ΠΏΠΎ ΠΌΠ΅Ρ€Π΅ ТСлания. ΠŸΡ€ΠΈ этом Ρ‚Π°ΠΊΠΎΠ΅ Π²ΠΊΠ»ΡŽΡ‡Π΅Π½ΠΈΠ΅ ΠΎΠ΄Π½ΠΎΠΉ Π»Π°ΠΌΠΏΡ‹ Π½Π΅ влияСт Π½Π° ΠΎΡΡ‚Π°Π»ΡŒΠ½Ρ‹Π΅ Π»Π°ΠΌΠΏΡ‹.

ВсС элСктричСскиС Π±Ρ‹Ρ‚ΠΎΠ²Ρ‹Π΅ устройства Π² ΠΊΠ²Π°Ρ€Ρ‚ΠΈΡ€Π΅ соСдинСны ΠΏΠ°Ρ€Π°Π»Π»Π΅Π»ΡŒΠ½ΠΎ Π² ΡΠ΅Ρ‚ΡŒ с напряТСниСм 220 Π’, ΠΈ ΠΏΠΎΠ΄ΠΊΠ»ΡŽΡ‡Π΅Π½Ρ‹ ΠΊ Ρ€Π°ΡΠΏΡ€Π΅Π΄Π΅Π»ΠΈΡ‚Π΅Π»ΡŒΠ½ΠΎΠΌΡƒ Ρ‰ΠΈΡ‚ΠΊΡƒ. Π”Ρ€ΡƒΠ³ΠΈΠΌΠΈ словами, ΠΏΠ°Ρ€Π°Π»Π»Π΅Π»ΡŒΠ½ΠΎΠ΅ соСдинСниС ΠΈΡΠΏΠΎΠ»ΡŒΠ·ΡƒΠ΅Ρ‚ΡΡ ΠΏΡ€ΠΈ нСобходимости ΠΏΠΎΠ΄ΠΊΠ»ΡŽΡ‡Π΅Π½ΠΈΡ элСктричСских устройств нСзависимо Π΄Ρ€ΡƒΠ³ ΠΎΡ‚ Π΄Ρ€ΡƒΠ³Π°. ΠŸΠΎΡΠ»Π΅Π΄ΠΎΠ²Π°Ρ‚Π΅Π»ΡŒΠ½ΠΎΠ΅ ΠΈ ΠΏΠ°Ρ€Π°Π»Π»Π΅Π»ΡŒΠ½ΠΎΠ΅ соСдинСниС ΠΈΠΌΠ΅ΡŽΡ‚ свои особСнности. Π‘ΡƒΡ‰Π΅ΡΡ‚Π²ΡƒΡŽΡ‚ Ρ‚Π°ΠΊΠΆΠ΅ ΡΠΌΠ΅ΡˆΠ°Π½Π½Ρ‹Π΅ соСдинСния.

Π‘ΠΈΠ»Π° Ρ‚ΠΎΠΊΠ° ΠΏΡ€ΠΈ ΠΏΠ°Ρ€Π°Π»Π»Π΅Π»ΡŒΠ½ΠΎΠΌ ΠΏΠΎΠ΄ΠΊΠ»ΡŽΡ‡Π΅Π½ΠΈΠΈ

Если Π±Ρ‹Π»ΠΎ использовано ΠΏΠΎΡΠ»Π΅Π΄ΠΎΠ²Π°Ρ‚Π΅Π»ΡŒΠ½ΠΎΠ΅ ΠΏΠΎΠ΄ΠΊΠ»ΡŽΡ‡Π΅Π½ΠΈΠ΅ Π² Ρ†Π΅ΠΏΠΈ, Ρ‚ΠΎ сила Π½Π΅ измСнится Π½ΠΈ Π½Π° ΠΎΠ΄Π½ΠΎΠΌ участкС Π²Π΅Ρ‚Π²ΠΈ. Найти напряТСниС ΠΌΠΎΠΆΠ½ΠΎ, примСняя стандартноС ΠΏΡ€Π°Π²ΠΈΠ»ΠΎ β€” Π½ΡƒΠΆΠ½ΠΎ ΡΡƒΠΌΠΌΠΈΡ€ΠΎΠ²Π°Ρ‚ΡŒ всС ΠΏΠΎΠΊΠ°Π·Π°Ρ‚Π΅Π»ΠΈ, ΠΊΠΎΡ‚ΠΎΡ€Ρ‹Π΅ ΠΏΡ€ΠΈΡΡƒΡ‚ΡΡ‚Π²ΡƒΡŽΡ‚ Π½Π° ΠΊΠΎΠ½Ρ†Π°Ρ… ΠΊΠ°ΠΆΠ΄ΠΎΠ³ΠΎ ΠΈΠ· рСзисторов, Π² ΠΈΡ‚ΠΎΠ³Π΅ получится Ρ€Π΅Π·ΡƒΠ»ΡŒΡ‚Π°Ρ‚. Но ΠΏΡ€ΠΈ ΠΏΠ°Ρ€Π°Π»Π»Π΅Π»ΡŒΠ½ΠΎΠΌ соСдинСния Π½Π°ΠΌΠ½ΠΎΠ³ΠΎ слоТнСй Π½Π°ΠΉΡ‚ΠΈ силу Ρ‚ΠΎΠΊΠ°.

Π”Π°ΠΆΠ΅ ΠΏΡ€ΠΈ ΠΌΠ°Π»ΠΎΠΉ Π½Π°Π³Ρ€ΡƒΠ·ΠΊΠ΅ Π² Ρ†Π΅ΠΏΠΈ Π±ΡƒΠ΄Π΅Ρ‚ Ρ„ΠΎΡ€ΠΌΠΈΡ€ΠΎΠ²Π°Ρ‚ΡŒΡΡ ΠΎΠΏΡ€Π΅Π΄Π΅Π»Π΅Π½Π½ΠΎΠ΅ сопротивлСниС. И Ρ‚ΠΎΠ³Π΄Π° ΠΎΠ½ΠΎ Π±ΡƒΠ΄Π΅Ρ‚ ΠΌΠ΅ΡˆΠ°Ρ‚ΡŒ ΠΏΡ€ΠΎΠ΄Π²ΠΈΠΆΠ΅Π½ΠΈΡŽ элСктричСского Ρ‚ΠΎΠΊΠ° ΠΈ Π±ΡƒΠ΄ΡƒΡ‚ ΠΏΠΎΡ‚Π΅Ρ€ΠΈ. Π’ ΠΎΠ±Ρ‰Π΅ΠΌ, Ρ‚ΠΎΠΊ пСрСмСщаСтся постСпСнно, ΠΎΡ‚ источника ΠΏΠΎ ΠΏΠΎΠ΄ΠΊΠ»ΡŽΡ‡Π΅Π½Π½Ρ‹ΠΌ Π·Π°Ρ€Π°Π½Π΅Π΅ рСзисторам ΠΊ Π½Π°Π³Ρ€ΡƒΠΆΠ΅Π½Π½Ρ‹ΠΌ дСталям.

ΠšΠ»Π°ΡΡΠΈΡ‡Π΅ΡΠΊΠ°Ρ Ρ„ΠΎΡ€ΠΌΡƒΠ»Π° Ома

Π§Ρ‚ΠΎΠ±Ρ‹ Π²Ρ‹ΠΏΠΎΠ»Π½ΠΈΡ‚ΡŒ доступноС ΠΏΡ€ΠΎΡ…ΠΎΠΆΠ΄Π΅Π½ΠΈΠ΅ Ρ‚ΠΎΠΊΠ° ΠΏΠΎ рСзисторам, Π½ΡƒΠΆΠ½ΠΎ, Ρ‡Ρ‚ΠΎΠ±Ρ‹ ΠΎΠ½ ΠΌΠΎΠ³ быстро ΠΈ просто ΠΎΡ‚Π΄Π°Π²Π°Ρ‚ΡŒ элСктроны, ΠΏΡ€ΠΎΡ‰Π΅ говоря ΠΈΠΌΠ΅Ρ‚ΡŒ ΠΏΡ€ΠΎΠ²ΠΎΠ΄ΠΈΠΌΠΎΡΡ‚ΡŒ.

Π’ соврСмСнноС врСмя Π² основном ΠΏΡ€ΠΈΠΌΠ΅Π½ΡΡŽΡ‚ΡΡ ΠΌΠ΅Π΄Π½Ρ‹Π΅ ΠΏΡ€ΠΎΠ²ΠΎΠ΄Π½ΠΈΠΊΠΈ, Π° Π²Π°ΠΆΠ½Ρ‹ΠΌ элСмСнтом Π±ΡƒΠ΄ΡƒΡ‚ ΠΏΡ€ΠΈΠ΅ΠΌΠ½ΠΈΠΊΠΈ элСктричСской энСргии. Π’Π°ΠΊΠΎΠΉ элСмСнт Π²Ρ‹Π·Ρ‹Π²Π°Π΅Ρ‚ Π½Π΅Π±ΠΎΠ»ΡŒΡˆΡƒΡŽ Π½Π°Π³Ρ€ΡƒΠ·ΠΊΡƒ ΠΈ ΠΈΠΌΠ΅Π΅Ρ‚ своС сопротивлСниС. НиТС описаны Ρ„ΠΎΡ€ΠΌΡƒΠ»Ρ‹ для ΠΏΠΎΡΠ»Π΅Π΄ΠΎΠ²Π°Ρ‚Π΅Π»ΡŒΠ½ΠΎΠ³ΠΎ ΠΈ ΠΏΠ°Ρ€Π°Π»Π»Π΅Π»ΡŒΠ½ΠΎΠ³ΠΎ соСдинСния сопротивлСний.

Π’Π°ΠΌ это Π±ΡƒΠ΄Π΅Ρ‚ интСрСсно ΠžΡΠΎΠ±Π΅Π½Π½ΠΎΡΡ‚ΠΈ управлСния освСщСниСм

Π’Π°ΠΊΠΆΠ΅ ΠΏΡ€ΠΈ ΠΏΠΎΠ΄ΠΊΠ»ΡŽΡ‡Π΅Π½ΠΈΠΈ Π½Π΅ΠΎΠ±Ρ…ΠΎΠ΄ΠΈΠΌΠΎ ΠΈΡΠΏΠΎΠ»ΡŒΠ·ΠΎΠ²Π°Ρ‚ΡŒ ΠΊΠ°Ρ‚ΡƒΡˆΠΊΡƒ индуктивности. Она способна ΠΏΠΎΠ΄Π°Π²Π»ΡΡ‚ΡŒ ΠΏΠΎΠΌΠ΅Ρ…ΠΈ Π² элСктроцСпи.


ΠŸΠ°Ρ€Π°Π»Π»Π΅Π»ΡŒΠ½Π°Ρ схСма соСдинСния

Π’ ΡΡ‚Π°Ρ‚ΡŒΠ΅ ΡƒΠ·Π½Π°Π΅Ρ‚Π΅ Ρ‡Ρ‚ΠΎ Ρ‚Π°ΠΊΠΎΠ΅ ΠΏΠ°Ρ€Π°Π»Π»Π΅Π»ΡŒΠ½Π°Ρ схСма соСдинСния, ΠΊΠ°ΠΊ Π΅Π΅ ΡΠ΄Π΅Π»Π°Ρ‚ΡŒ, характСристики, сила Ρ‚ΠΎΠΊΠ° Π² ΠΏΠ°Ρ€Π°Π»Π»Π΅Π»ΡŒΠ½ΠΎΠΉ Ρ†Π΅ΠΏΠΈ, Π΅Π³ΠΎ сопротивлСниС ΠΈ ΠΌΠΎΡ‰Π½ΠΎΡΡ‚ΡŒ. А Ρ‚Π°ΠΊΠΆΠ΅ прСимущСства ΠΈ нСдостатки ΠΏΠ°Ρ€Π°Π»Π»Π΅Π»ΡŒΠ½ΠΎΠΉ схСмы.

ПовСдСниС схСмы ΠΏΠΎΠ»Π½ΠΎΡΡ‚ΡŒΡŽ зависит ΠΎΡ‚ ΠΊΠΎΠ½Ρ„ΠΈΠ³ΡƒΡ€Π°Ρ†ΠΈΠΈ Π΅Π΅ ΠΊΠΎΠΌΠΏΠΎΠ½Π΅Π½Ρ‚ΠΎΠ². Π’ соотвСтствии с ΠΊΠΎΠ½Ρ„ΠΈΠ³ΡƒΡ€Π°Ρ†ΠΈΠ΅ΠΉ ΠΈΡ… ΠΏΠΎΠ΄ΠΊΠ»ΡŽΡ‡Π΅Π½ΠΈΡ эти Ρ†Π΅ΠΏΠΈ ΠΏΠΎΠ΄Ρ€Π°Π·Π΄Π΅Π»ΡΡŽΡ‚ΡΡ Π½Π° ΠΏΠ°Ρ€Π°Π»Π»Π΅Π»ΡŒΠ½Ρ‹Π΅ ΠΈ ΠΏΠΎΡΠ»Π΅Π΄ΠΎΠ²Π°Ρ‚Π΅Π»ΡŒΠ½Ρ‹Π΅. Π­Ρ‚ΠΎΡ‚ пост раскрываСт Π·Π½Π°Ρ‡Π΅Π½ΠΈΠ΅ ΠΏΠ°Ρ€Π°Π»Π»Π΅Π»ΡŒΠ½ΠΎΠΉ Ρ†Π΅ΠΏΠΈ, ΠΊΠ°ΠΊ ΡΠΎΠ·Π΄Π°Ρ‚ΡŒ ΠΏΠ°Ρ€Π°Π»Π»Π΅Π»ΡŒΠ½ΡƒΡŽ схСму, Π΅Π΅ Ρ€Π°Π·Π»ΠΈΡ‡Π½Ρ‹Π΅ характСристики, области примСнСния, прСимущСства ΠΈ нСдостатки.

Π—Π°Π²ΠΈΡΠΈΠΌΠΎΡΡ‚ΡŒ сопротивлСния

Π—Π½Π°Ρ‡Π΅Π½ΠΈΠ΅ элСктропроводимости зависит ΠΎΡ‚ Π½Π΅ΡΠΊΠΎΠ»ΡŒΠΊΠΈΡ… Ρ„Π°ΠΊΡ‚ΠΎΡ€ΠΎΠ², ΠΊΠΎΡ‚ΠΎΡ€Ρ‹Π΅ Π½Π΅ΠΎΠ±Ρ…ΠΎΠ΄ΠΈΠΌΠΎ ΡƒΡ‡ΠΈΡ‚Ρ‹Π²Π°Ρ‚ΡŒ ΠΏΡ€ΠΈ расчСтах, ΠΈΠ·Π³ΠΎΡ‚ΠΎΠ²Π»Π΅Π½ΠΈΠΈ элСмСнтов рСзистивной Π½Π°Π³Ρ€ΡƒΠ·ΠΊΠΈ (рСзисторов), Ρ€Π΅ΠΌΠΎΠ½Ρ‚Π΅ ΠΈ ΠΏΡ€ΠΎΠ΅ΠΊΡ‚ΠΈΡ€ΠΎΠ²Π°Π½ΠΈΠΈ устройств. К этим Ρ„Π°ΠΊΡ‚ΠΎΡ€Π°ΠΌ Π½Π΅ΠΎΠ±Ρ…ΠΎΠ΄ΠΈΠΌΠΎ отнСсти ΡΠ»Π΅Π΄ΡƒΡŽΡ‰ΠΈΠ΅:

  1. Π’Π΅ΠΌΠΏΠ΅Ρ€Π°Ρ‚ΡƒΡ€Π° ΠΎΠΊΡ€ΡƒΠΆΠ°ΡŽΡ‰Π΅ΠΉ срСды ΠΈ ΠΌΠ°Ρ‚Π΅Ρ€ΠΈΠ°Π»Π°.
  2. ЭлСктричСскиС Π²Π΅Π»ΠΈΡ‡ΠΈΠ½Ρ‹.
  3. ГСомСтричСскиС свойства вСщСства.
  4. Π’ΠΈΠΏ ΠΌΠ°Ρ‚Π΅Ρ€ΠΈΠ°Π»Π°, ΠΈΠ· ΠΊΠΎΡ‚ΠΎΡ€ΠΎΠ³ΠΎ ΠΈΠ·Π³ΠΎΡ‚ΠΎΠ²Π»Π΅Π½ ΠΏΡ€ΠΎΠ²ΠΎΠ΄Π½ΠΈΠΊ (ΠΏΠΎΠ»ΡƒΠΏΡ€ΠΎΠ²ΠΎΠ΄Π½ΠΈΠΊ).

К элСктричСским Π²Π΅Π»ΠΈΡ‡ΠΈΠ½Π°ΠΌ ΠΌΠΎΠΆΠ½ΠΎ отнСсти Ρ€Π°Π·Π½ΠΎΡΡ‚ΡŒ ΠΏΠΎΡ‚Π΅Π½Ρ†ΠΈΠ°Π»ΠΎΠ² (напряТСниС), ΡΠ»Π΅ΠΊΡ‚Ρ€ΠΎΠ΄Π²ΠΈΠΆΡƒΡ‰ΡƒΡŽ силу (Π­Π”Π‘) ΠΈ силу Ρ‚ΠΎΠΊΠ°. Π“Π΅ΠΎΠΌΠ΅Ρ‚Ρ€ΠΈΠ΅ΠΉ ΠΏΡ€ΠΎΠ²ΠΎΠ΄Π½ΠΈΠΊΠ° являСтся Π΅Π³ΠΎ Π΄Π»ΠΈΠ½Π° ΠΈ ΠΏΠ»ΠΎΡ‰Π°Π΄ΡŒ ΠΏΠΎΠΏΠ΅Ρ€Π΅Ρ‡Π½ΠΎΠ³ΠΎ сСчСния.

ЭлСктричСскиС Π²Π΅Π»ΠΈΡ‡ΠΈΠ½Ρ‹

Π—Π°Π²ΠΈΡΠΈΠΌΠΎΡΡ‚ΡŒ Π²Π΅Π»ΠΈΡ‡ΠΈΠ½Ρ‹ элСктропроводимости ΠΎΡ‚ ΠΏΠ°Ρ€Π°ΠΌΠ΅Ρ‚Ρ€ΠΎΠ² элСктричСства опрСдСляСтся Π·Π°ΠΊΠΎΠ½ΠΎΠΌ Ома. БущСствуСт Π΄Π²Π΅ Ρ„ΠΎΡ€ΠΌΡƒΠ»ΠΈΡ€ΠΎΠ²ΠΊΠΈ: ΠΎΠ΄Π½Π° β€” для участка, Π° другая β€” для ΠΏΠΎΠ»Π½ΠΎΠΉ Ρ†Π΅ΠΏΠΈ. Π’ ΠΏΠ΅Ρ€Π²ΠΎΠΌ случаС ΡΠΎΠΎΡ‚Π½ΠΎΡˆΠ΅Π½ΠΈΠ΅ ΠΎΠΏΡ€Π΅Π΄Π΅Π»ΡΡŽΡ‚ΡΡ, исходя ΠΈΠ· Π·Π½Π°Ρ‡Π΅Π½ΠΈΠΉ силы Ρ‚ΠΎΠΊΠ° (I) ΠΈ напряТСния (U) простой Ρ„ΠΎΡ€ΠΌΡƒΠ»ΠΎΠΉ: I = U / R. Из ΡΠΎΠΎΡ‚Π½ΠΎΡˆΠ΅Π½ΠΈΡ Π²ΠΈΠ΄Π½Π° прямо ΠΏΡ€ΠΎΠΏΠΎΡ€Ρ†ΠΈΠΎΠ½Π°Π»ΡŒΠ½Π°Ρ Π·Π°Π²ΠΈΡΠΈΠΌΠΎΡΡ‚ΡŒ Ρ‚ΠΎΠΊΠ° ΠΎΡ‚ Π²Π΅Π»ΠΈΡ‡ΠΈΠ½Ρ‹ напряТСния, Π° Ρ‚Π°ΠΊΠΆΠ΅ ΠΎΠ±Ρ€Π°Ρ‚Π½ΠΎ ΠΏΡ€ΠΎΠΏΠΎΡ€Ρ†ΠΈΠΎΠ½Π°Π»ΡŒΠ½Π°Ρ ΠΎΡ‚ сопротивлСния. МоТно Π²Ρ‹Ρ€Π°Π·ΠΈΡ‚ΡŒ R: R = U / I.

Π’Π°ΠΌ это Π±ΡƒΠ΄Π΅Ρ‚ интСрСсно Π¦ΠΈΡ„Ρ€ΠΎΠ²ΠΎΠΉ ΠΏΡ€ΠΈΠ±ΠΎΡ€ ΠΌΡƒΠ»ΡŒΡ‚ΠΈΠΌΠ΅Ρ‚Ρ€ ΠΈ ΠΈΠ·ΠΌΠ΅Ρ€Π΅Π½ΠΈΠ΅ ΠΌΡƒΠ»ΡŒΡ‚ΠΈΡ‚Π΅ΡΡ‚Π΅Ρ€ΠΎΠΌ

Для расчСта элСктропроводимости всСго участка слСдуСт Π²ΠΎΡΠΏΠΎΠ»ΡŒΠ·ΠΎΠ²Π°Ρ‚ΡŒΡΡ ΡΠΎΠΎΡ‚Π½ΠΎΡˆΠ΅Π½ΠΈΠ΅ΠΌ ΠΌΠ΅ΠΆΠ΄Ρƒ Π­Π”Π‘ (e), силой Ρ‚ΠΎΠΊΠ° (i), Π° Ρ‚Π°ΠΊΠΆΠ΅ Π²Π½ΡƒΡ‚Ρ€Π΅Π½Π½ΠΈΠΌ сопротивлСниСм источника питания (RΠ²Π½): i = e / (R+RΠ²Π½). Π’ этом случаС Π²Π΅Π»ΠΈΡ‡ΠΈΠ½Π° R вычисляСтся ΠΏΠΎ Ρ„ΠΎΡ€ΠΌΡƒΠ»Π΅: R = (e / i) β€” RΠ²Π½. Однако ΠΏΡ€ΠΈ Π²Ρ‹ΠΏΠΎΠ»Π½Π΅Π½ΠΈΠΈ расчСтов Π½Π΅ΠΎΠ±Ρ…ΠΎΠ΄ΠΈΠΌΠΎ ΡƒΡ‡ΠΈΡ‚Ρ‹Π²Π°Ρ‚ΡŒ Ρ‚Π°ΠΊΠΆΠ΅ гСомСтричСскиС ΠΏΠ°Ρ€Π°ΠΌΠ΅Ρ‚Ρ€Ρ‹ ΠΈ Ρ‚ΠΈΠΏ ΠΏΡ€ΠΎΠ²ΠΎΠ΄Π½ΠΈΠΊΠ°, ΠΏΠΎΡΠΊΠΎΠ»ΡŒΠΊΡƒ ΠΎΠ½ΠΈ ΠΌΠΎΠ³ΡƒΡ‚ сущСствСнно ΠΏΠΎΠ²Π»ΠΈΡΡ‚ΡŒ Π½Π° вычислСния.

Π’ΠΈΠΏ ΠΈ гСомСтричСскиС ΠΏΠ°Ρ€Π°ΠΌΠ΅Ρ‚Ρ€Ρ‹

Бвойство вСщСства ΠΊ проводимости элСктричСства опрСдСляСтся структурой кристалличСской Ρ€Π΅ΡˆΠ΅Ρ‚ΠΊΠΈ, Π° Ρ‚Π°ΠΊΠΆΠ΅ количСством свободных носитСлСй. Π˜ΡΡ…ΠΎΠ΄Ρ ΠΈΠ· этого, Ρ‚ΠΈΠΏ вСщСства являСтся ΠΊΠ»ΡŽΡ‡Π΅Π²Ρ‹ΠΌ Ρ„Π°ΠΊΡ‚ΠΎΡ€ΠΎΠΌ, ΠΊΠΎΡ‚ΠΎΡ€Ρ‹ΠΉ опрСдСляСт Π²Π΅Π»ΠΈΡ‡ΠΈΠ½Ρƒ элСктропроводимости. Π’ Π½Π°ΡƒΠΊΠ΅ коэффициСнт, ΠΎΠΏΡ€Π΅Π΄Π΅Π»ΡΡŽΡ‰ΠΈΠΉ Ρ‚ΠΈΠΏ вСщСства, обозначаСтся Π»ΠΈΡ‚Π΅Ρ€ΠΎΠΉ Β«Ρ€Β» ΠΈ называСтся ΡƒΠ΄Π΅Π»ΡŒΠ½Ρ‹ΠΌ сопротивлСниСм. Π•Π³ΠΎ Π·Π½Π°Ρ‡Π΅Π½ΠΈΠ΅ для Ρ€Π°Π·Π»ΠΈΡ‡Π½Ρ‹Ρ… ΠΌΠ°Ρ‚Π΅Ρ€ΠΈΠ°Π»ΠΎΠ² (ΠΏΡ€ΠΈ Ρ‚Π΅ΠΌΠΏΠ΅Ρ€Π°Ρ‚ΡƒΡ€Π΅ +20 градусов ΠΏΠΎ ЦСльсию) ΠΌΠΎΠΆΠ½ΠΎ Π½Π°ΠΉΡ‚ΠΈ Π² ΡΠΏΠ΅Ρ†ΠΈΠ°Π»ΡŒΠ½Ρ‹Ρ… Ρ‚Π°Π±Π»ΠΈΡ†Π°Ρ….

Иногда для удобства расчСтов ΠΈΡΠΏΠΎΠ»ΡŒΠ·ΡƒΠ΅Ρ‚ΡΡ обратная Π²Π΅Π»ΠΈΡ‡ΠΈΠ½Π°, которая называСтся ΡƒΠ΄Π΅Π»ΡŒΠ½ΠΎΠΉ ΠΏΡ€ΠΎΠ²ΠΎΠ΄ΠΈΠΌΠΎΡΡ‚ΡŒΡŽ (Οƒ). Она связана с ΡƒΠ΄Π΅Π»ΡŒΠ½Ρ‹ΠΌ сопротивлСниСм ΡΠ»Π΅Π΄ΡƒΡŽΡ‰ΠΈΠΌ ΡΠΎΠΎΡ‚Π½ΠΎΡˆΠ΅Π½ΠΈΠ΅ΠΌ: p = 1 / Οƒ. ΠŸΠ»ΠΎΡ‰Π°Π΄ΡŒ ΠΏΠΎΠΏΠ΅Ρ€Π΅Ρ‡Π½ΠΎΠ³ΠΎ сСчСния (S) влияСт Π½Π° элСктричСскоС сопротивлСниС. Π‘ физичСской Ρ‚ΠΎΡ‡ΠΊΠΈ зрСния, Π·Π°Π²ΠΈΡΠΈΠΌΠΎΡΡ‚ΡŒ ΠΌΠΎΠΆΠ½ΠΎ ΠΏΠΎΠ½ΡΡ‚ΡŒ ΡΠ»Π΅Π΄ΡƒΡŽΡ‰ΠΈΠΌ ΠΎΠ±Ρ€Π°Π·ΠΎΠΌ: ΠΏΡ€ΠΈ ΠΌΠ°Π»ΠΎΠΌ сСчСнии происходят Π±ΠΎΠ»Π΅Π΅ частыС взаимодСйствия частиц элСктричСского Ρ‚ΠΎΠΊΠ° с ΡƒΠ·Π»Π°ΠΌΠΈ кристалличСской Ρ€Π΅ΡˆΠ΅Ρ‚ΠΊΠΈ. ΠŸΠΎΠΏΠ΅Ρ€Π΅Ρ‡Π½ΠΎΠ΅ сСчСниС ΠΌΠΎΠΆΠ½ΠΎ Π²Ρ‹Ρ‡ΠΈΡΠ»ΠΈΡ‚ΡŒ ΠΏΠΎ ΡΠΏΠ΅Ρ†ΠΈΠ°Π»ΡŒΠ½ΠΎΠΌΡƒ Π°Π»Π³ΠΎΡ€ΠΈΡ‚ΠΌΡƒ:

  1. Π˜Π·ΠΌΠ΅Ρ€Π΅Π½ΠΈΠ΅ гСомСтричСских ΠΏΠ°Ρ€Π°ΠΌΠ΅Ρ‚Ρ€ΠΎΠ² ΠΏΡ€ΠΎΠ²ΠΎΠ΄Π½ΠΈΠΊΠ° (Π΄ΠΈΠ°ΠΌΠ΅Ρ‚Ρ€ ΠΈΠ»ΠΈ Π΄Π»ΠΈΠ½Ρƒ сторон) ΠΏΡ€ΠΈ ΠΏΠΎΠΌΠΎΡ‰ΠΈ ΡˆΡ‚Π°Π½Π³Π΅Π½Ρ†ΠΈΡ€ΠΊΡƒΠ»Ρ.
  2. Π’ΠΈΠ·ΡƒΠ°Π»ΡŒΠ½ΠΎ ΠΎΠΏΡ€Π΅Π΄Π΅Π»ΠΈΡ‚ΡŒ Ρ„ΠΎΡ€ΠΌΡƒ ΠΌΠ°Ρ‚Π΅Ρ€ΠΈΠ°Π»Π°.
  3. Π’Ρ‹Ρ‡ΠΈΡΠ»ΠΈΡ‚ΡŒ ΠΏΠ»ΠΎΡ‰Π°Π΄ΡŒ ΠΏΠΎΠΏΠ΅Ρ€Π΅Ρ‡Π½ΠΎΠ³ΠΎ сСчСния ΠΏΠΎ Ρ„ΠΎΡ€ΠΌΡƒΠ»Π΅, Π½Π°ΠΉΠ΄Π΅Π½Π½ΠΎΠΉ Π² справочникС ΠΈΠ»ΠΈ ΠΈΠ½Ρ‚Π΅Ρ€Π½Π΅Ρ‚Π΅.

Π’ случаС ΠΊΠΎΠ³Π΄Π° ΠΏΡ€ΠΎΠ²ΠΎΠ΄Π½ΠΈΠΊ ΠΈΠΌΠ΅Π΅Ρ‚ ΡΠ»ΠΎΠΆΠ½ΡƒΡŽ структуру, Π½Π΅ΠΎΠ±Ρ…ΠΎΠ΄ΠΈΠΌΠΎ Π²Ρ‹Ρ‡ΠΈΡΠ»ΠΈΡ‚ΡŒ Π²Π΅Π»ΠΈΡ‡ΠΈΠ½Ρƒ S ΠΎΠ΄Π½ΠΎΠ³ΠΎ элСмСнта, Π° Π·Π°Ρ‚Π΅ΠΌ ΡƒΠΌΠ½ΠΎΠΆΠΈΡ‚ΡŒ Ρ€Π΅Π·ΡƒΠ»ΡŒΡ‚Π°Ρ‚ Π½Π° количСство элСмСнтов, входящих Π² Π΅Π³ΠΎ состав. НапримСр, Ссли ΠΏΡ€ΠΎΠ²ΠΎΠ΄ являСтся ΠΌΠ½ΠΎΠ³ΠΎΠΆΠΈΠ»ΡŒΠ½Ρ‹ΠΌ, Ρ‚ΠΎ слСдуСт Π²Ρ‹Ρ‡ΠΈΡΠ»ΠΈΡ‚ΡŒ S для ΠΎΠ΄Π½ΠΎΠΉ ΠΆΠΈΠ»Ρ‹. ПослС этого Π½ΡƒΠΆΠ½ΠΎ ΡƒΠΌΠ½ΠΎΠΆΠΈΡ‚ΡŒ, ΠΏΠΎΠ»ΡƒΡ‡Π΅Π½Π½ΡƒΡŽ Π²Π΅Π»ΠΈΡ‡ΠΈΠ½Ρƒ S, Π½Π° количСство ΠΆΠΈΠ». Π—Π°Π²ΠΈΡΠΈΠΌΠΎΡΡ‚ΡŒ R ΠΎΡ‚ Π²Ρ‹ΡˆΠ΅ΠΏΠ΅Ρ€Π΅Ρ‡ΠΈΡΠ»Π΅Π½Π½Ρ‹Ρ… Π²Π΅Π»ΠΈΡ‡ΠΈΠ½ ΠΌΠΎΠΆΠ½ΠΎ Π·Π°ΠΏΠΈΡΠ°Ρ‚ΡŒ Π² Π²ΠΈΠ΄Π΅ ΡΠΎΠΎΡ‚Π½ΠΎΡˆΠ΅Π½ΠΈΡ: R = p * L / S. Π›ΠΈΡ‚Π΅Ρ€Π° Β«LΒ» являСтся Π΄Π»ΠΈΠ½ΠΎΠΉ ΠΏΡ€ΠΎΠ²ΠΎΠ΄Π½ΠΈΠΊΠ°. Однако для получСния Ρ‚ΠΎΡ‡Π½Ρ‹Ρ… расчСтов Π½Π΅ΠΎΠ±Ρ…ΠΎΠ΄ΠΈΠΌΠΎ ΡƒΡ‡ΠΈΡ‚Ρ‹Π²Π°Ρ‚ΡŒ Ρ‚Π΅ΠΌΠΏΠ΅Ρ€Π°Ρ‚ΡƒΡ€Π½Ρ‹Π΅ ΠΏΠΎΠΊΠ°Π·Π°Ρ‚Π΅Π»ΠΈ внСшнСй срСды ΠΈ ΠΏΡ€ΠΎΠ²ΠΎΠ΄Π½ΠΈΠΊΠ°.

Π’Π΅ΠΌΠΏΠ΅Ρ€Π°Ρ‚ΡƒΡ€Π½Ρ‹Π΅ ΠΏΠΎΠΊΠ°Π·Π°Ρ‚Π΅Π»ΠΈ

БущСствуСт Π΄ΠΎΠΊΠ°Π·Π°Ρ‚Π΅Π»ΡŒΡΡ‚Π²ΠΎ зависимости ΡƒΠ΄Π΅Π»ΡŒΠ½ΠΎΠ³ΠΎ сопротивлСния ΠΌΠ°Ρ‚Π΅Ρ€ΠΈΠ°Π»Π° ΠΎΡ‚ Ρ‚Π΅ΠΌΠΏΠ΅Ρ€Π°Ρ‚ΡƒΡ€Ρ‹, основанноС Π½Π° физичСском экспСримСнтС. Для провСдСния ΠΎΠΏΡ‹Ρ‚Π° Π½ΡƒΠΆΠ½ΠΎ ΡΠΎΠ±Ρ€Π°Ρ‚ΡŒ ΡΠ»Π΅ΠΊΡ‚Ρ€ΠΈΡ‡Π΅ΡΠΊΡƒΡŽ Ρ†Π΅ΠΏΡŒ, ΡΠΎΡΡ‚ΠΎΡΡ‰ΡƒΡŽ ΠΈΠ· ΡΠ»Π΅Π΄ΡƒΡŽΡ‰ΠΈΡ… элСмСнтов: источника питания, Π½ΠΈΡ…Ρ€ΠΎΠΌΠΎΠ²ΠΎΠΉ спирали, ΡΠΎΠ΅Π΄ΠΈΠ½ΠΈΡ‚Π΅Π»ΡŒΠ½Ρ‹Ρ… ΠΏΡ€ΠΎΠ²ΠΎΠ΄ΠΎΠ² Π°ΠΌΠΏΠ΅Ρ€ΠΌΠ΅Ρ‚Ρ€Π° ΠΈ Π²ΠΎΠ»ΡŒΡ‚ΠΌΠ΅Ρ‚Ρ€Π°. ΠŸΡ€ΠΈΠ±ΠΎΡ€Ρ‹ Π½ΡƒΠΆΠ½Ρ‹ для измСрСния Π·Π½Π°Ρ‡Π΅Π½ΠΈΠΉ силы Ρ‚ΠΎΠΊΠ° ΠΈ напряТСния соотвСтствСнно. ΠŸΡ€ΠΈ ΠΏΡ€ΠΎΡ‚Π΅ΠΊΠ°Π½ΠΈΠΈ элСктричСства происходит Π½Π°Π³Ρ€Π΅Π²Π°Π½ΠΈΠ΅ Π½ΠΈΡ…Ρ€ΠΎΠΌΠΎΠ²ΠΎΠΉ ΠΏΡ€ΡƒΠΆΠΈΠ½Ρ‹. По ΠΌΠ΅Ρ€Π΅ Π΅Π΅ нагрСвания, показания Π°ΠΌΠΏΠ΅Ρ€ΠΌΠ΅Ρ‚Ρ€Π° ΡƒΠΌΠ΅Π½ΡŒΡˆΠ°ΡŽΡ‚ΡΡ. ΠŸΡ€ΠΈ этом происходит сущСствСнноС ΠΏΠ°Π΄Π΅Π½ΠΈΠ΅ напряТСния Π½Π° участкС Ρ†Π΅ΠΏΠΈ, ΠΎ ΠΊΠΎΡ‚ΠΎΡ€ΠΎΠΌ ΡΠ²ΠΈΠ΄Π΅Ρ‚Π΅Π»ΡŒΡΡ‚Π²ΡƒΡŽΡ‚ показания Π²ΠΎΠ»ΡŒΡ‚ΠΌΠ΅Ρ‚Ρ€Π°.

Π’Π°ΠΌ это Π±ΡƒΠ΄Π΅Ρ‚ интСрСсно УдСльноС элСктричСскоС сопротивлСниС мСталличСских ΠΏΡ€ΠΎΠ²ΠΎΠ΄Π½ΠΈΠΊΠΎΠ²

Π’ Ρ€Π°Π΄ΠΈΠΎΡ‚Π΅Ρ…Π½ΠΈΠΊΠ΅ ΡƒΠΌΠ΅Π½ΡŒΡˆΠ΅Π½ΠΈΠ΅ Π²Π΅Π»ΠΈΡ‡ΠΈΠ½Ρ‹ напряТСниС называСтся просадкой ΠΈΠ»ΠΈ ΠΏΠ°Π΄Π΅Π½ΠΈΠ΅ΠΌ. Π€ΠΎΡ€ΠΌΡƒΠ»Π° зависимости Ρ€ ΠΎΡ‚ Ρ‚Π΅ΠΌΠΏΠ΅Ρ€Π°Ρ‚ΡƒΡ€Ρ‹ ΠΈΠΌΠ΅Π΅Ρ‚ ΡΠ»Π΅Π΄ΡƒΡŽΡ‰ΠΈΠΉ Π²ΠΈΠ΄: p = p0 * [1 + a * (t β€” 20)]. Π—Π½Π°Ρ‡Π΅Π½ΠΈΠ΅ p0 β€” ΡƒΠ΄Π΅Π»ΡŒΠ½ΠΎΠ΅ сопротивлСниС ΠΌΠ°Ρ‚Π΅Ρ€ΠΈΠ°Π»Π°, взятого ΠΈΠ· Ρ‚Π°Π±Π»ΠΈΡ†Ρ‹, Π° Π»ΠΈΡ‚Π΅Ρ€Π° Β«tΒ» β€” Ρ‚Π΅ΠΌΠΏΠ΅Ρ€Π°Ρ‚ΡƒΡ€Π° ΠΏΡ€ΠΎΠ²ΠΎΠ΄Π½ΠΈΠΊΠ°.

Π’Π΅ΠΌΠΏΠ΅Ρ€Π°Ρ‚ΡƒΡ€Π½Ρ‹ΠΉ коэффициСнт Β«Π°Β» ΠΏΡ€ΠΈΠ½ΠΈΠΌΠ°Π΅Ρ‚ ΡΠ»Π΅Π΄ΡƒΡŽΡ‰ΠΈΠ΅ значСния: для ΠΌΠ΅Ρ‚Π°Π»Π»ΠΎΠ² β€” a>0, Π° для элСктролитичСских растворов β€” a<0. Для получСния Ρ„ΠΎΡ€ΠΌΡƒΠ»Ρ‹, ΠΎΠΏΡ€Π΅Π΄Π΅Π»ΡΡŽΡ‰Π΅ΠΉ всС зависимости, Π½Π΅ΠΎΠ±Ρ…ΠΎΠ΄ΠΈΠΌΠΎ ΠΏΠΎΠ΄ΡΡ‚Π°Π²ΠΈΡ‚ΡŒ всС ΡΠΎΠΎΡ‚Π½ΠΎΡˆΠ΅Π½ΠΈΡ Π² ΠΎΠ±Ρ‰ΡƒΡŽ Ρ„ΠΎΡ€ΠΌΡƒΠ»Ρƒ зависимости R ΠΎΡ‚ Ρ‚ΠΈΠΏΠ° ΠΌΠ°Ρ‚Π΅Ρ€ΠΈΠ°Π»Π°, Ρ‚Π΅ΠΌΠΏΠ΅Ρ€Π°Ρ‚ΡƒΡ€Ρ‹, Π΄Π»ΠΈΠ½Ρ‹ ΠΈ сСчСния: R = p0 * [1 + a * (t β€” 20)] * L / S. Π€ΠΎΡ€ΠΌΡƒΠ»Ρ‹ ΠΈΡΠΏΠΎΠ»ΡŒΠ·ΡƒΡŽΡ‚ΡΡ Ρ‚ΠΎΠ»ΡŒΠΊΠΎ для расчСтов ΠΈ изготовлСния рСзисторов. Для быстрого измСрСния Π²Π΅Π»ΠΈΡ‡ΠΈΠ½Ρ‹ сопротивлСния примСняСтся ΠΎΠΌΠΌΠ΅Ρ‚Ρ€.

Π§Ρ‚ΠΎ Ρ‚Π°ΠΊΠΎΠ΅ ΠΏΠ°Ρ€Π°Π»Π»Π΅Π»ΡŒΠ½Π°Ρ Ρ†Π΅ΠΏΡŒ

Π‘Ρ…Π΅ΠΌΠ° называСтся ΠΏΠ°Ρ€Π°Π»Π»Π΅Π»ΡŒΠ½ΠΎΠΉ, ΠΊΠΎΠ³Π΄Π° Π΄Π²Π° ΠΈΠ»ΠΈ Π±ΠΎΠ»Π΅Π΅ ΠΊΠΎΠΌΠΏΠΎΠ½Π΅Π½Ρ‚ΠΎΠ² ΠΏΠΎΠ΄ΠΊΠ»ΡŽΡ‡Π΅Π½Ρ‹ ΠΊ ΠΎΠ΄Π½ΠΎΠΌΡƒ ΡƒΠ·Π»Ρƒ, Π° ΠΎΠ±Π΅ стороны ΠΊΠΎΠΌΠΏΠΎΠ½Π΅Π½Ρ‚ΠΎΠ² ΠΏΠΎΠ΄ΠΊΠ»ΡŽΡ‡Π΅Π½Ρ‹ нСпосрСдствСнно ΠΊ Π±Π°Ρ‚Π°Ρ€Π΅Π΅ ΠΈΠ»ΠΈ Π»ΡŽΠ±ΠΎΠΌΡƒ Π΄Ρ€ΡƒΠ³ΠΎΠΌΡƒ источнику. Π’ΠΎΠΊ Π² ΠΏΠ°Ρ€Π°Π»Π»Π΅Π»ΡŒΠ½ΠΎΠΉ Ρ†Π΅ΠΏΠΈ ΠΈΠΌΠ΅Π΅Ρ‚ Π΄Π²Π° ΠΈΠ»ΠΈ Π±ΠΎΠ»Π΅Π΅ ΠΏΡƒΡ‚ΠΈ прохоТдСния Ρ‡Π΅Ρ€Π΅Π· Π½Π΅Π³ΠΎ.

НаиболСС распространСнным ΠΏΡ€ΠΈΠΌΠ΅Ρ€ΠΎΠΌ ΠΏΠ°Ρ€Π°Π»Π»Π΅Π»ΡŒΠ½ΠΎΠΉ Ρ†Π΅ΠΏΠΈ являСтся ΠΏΡ€ΠΎΠ²ΠΎΠ΄ΠΊΠ° Π°Π²Ρ‚ΠΎΠΌΠΎΠ±ΠΈΠ»ΡŒΠ½Ρ‹Ρ… Ρ„Π°Ρ€. Если Π±Ρ‹ Ρ„Π°Ρ€Ρ‹ Π±Ρ‹Π»ΠΈ Π²ΠΊΠ»ΡŽΡ‡Π΅Π½Ρ‹ ΠΏΠΎΡΠ»Π΅Π΄ΠΎΠ²Π°Ρ‚Π΅Π»ΡŒΠ½ΠΎ, Ρ‚ΠΎ ΠΊΠΎΠ³Π΄Π° ΠΎΠ΄Π½Π° Ρ„Π°Ρ€Π° Π²Ρ‹Ρ…ΠΎΠ΄ΠΈΠ»Π° ΠΈΠ· строя, другая Ρ‚Π°ΠΊΠΆΠ΅ Π±Ρ‹ Π²Ρ‹ΠΊΠ»ΡŽΡ‡Π°Π»Π°ΡΡŒ

ΠŸΡ€ΠΈΠΌΠ΅Ρ€ Π°Π²Ρ‚ΠΎΠΌΠΎΠ±ΠΈΠ»ΡŒΠ½Ρ‹Ρ… Ρ„Π°Ρ€, ΠΏΠΎΠ΄ΠΊΠ»ΡŽΡ‡Π΅Π½Π½Ρ‹Ρ… ΠΏΠΎ ΠΏΠ°Ρ€Π°Π»Π»Π΅Π»ΡŒΠ½ΠΎΠΉ Ρ†Π΅ΠΏΠΈ

Π₯арактСристики ΠΏΠ°Ρ€Π°Π»Π»Π΅Π»ΡŒΠ½ΠΎΠΉ Ρ†Π΅ΠΏΠΈ

ΠžΡΠ½ΠΎΠ²Π½Ρ‹Π΅ характСристики ΠΏΠ°Ρ€Π°Π»Π»Π΅Π»ΡŒΠ½ΠΎΠΉ Ρ†Π΅ΠΏΠΈ пСрСчислСны Π½ΠΈΠΆΠ΅:

Π‘ΠΈΠ»Π° Ρ‚ΠΎΠΊΠ° Π² ΠΏΠ°Ρ€Π°Π»Π»Π΅Π»ΡŒΠ½ΠΎΠΉ Ρ†Π΅ΠΏΠΈ

Богласно Π·Π°ΠΊΠΎΠ½Ρƒ Ома, I = U / R. Π­Ρ‚ΠΎ ΠΏΠΎΠ΄Ρ€Π°Π·ΡƒΠΌΠ΅Π²Π°Π΅Ρ‚, Ρ‡Ρ‚ΠΎ ΠΊΠ°ΠΆΠ΄Ρ‹ΠΉ рСзистор Π² этой Ρ†Π΅ΠΏΠΈ Π±ΡƒΠ΄Π΅Ρ‚ ΠΏΠΎΡ‚Ρ€Π΅Π±Π»ΡΡ‚ΡŒ Ρ‚ΠΎΠΊ ΠΎΡ‚ источника. Π‘Π»Π΅Π΄ΠΎΠ²Π°Ρ‚Π΅Π»ΡŒΠ½ΠΎ, ΠΎΠ±Ρ‰ΠΈΠΉ Ρ‚ΠΎΠΊ, потрСбляСмый ΠΎΡ‚ источника, Ρ€Π°Π²Π΅Π½ суммС Ρ‚ΠΎΠΊΠΎΠ² вСтвлСния, ΠΈ Ρ‚ΠΎΠΊ, ΠΏΡ€ΠΎΡ‚Π΅ΠΊΠ°ΡŽΡ‰ΠΈΠΉ Π² ΠΊΠ°ΠΆΠ΄ΠΎΠΌ Ρ‚Ρ€Π°ΠΊΡ‚Π΅, зависит ΠΎΡ‚ сопротивлСния Π²Π΅Ρ‚Π²ΠΈ. Π’Π΅ΠΌ Π½Π΅ ΠΌΠ΅Π½Π΅Π΅, напряТСниС остаСтся Π½Π΅ΠΈΠ·ΠΌΠ΅Π½Π½Ρ‹ΠΌ ΠΈ создаСт Ρ€Π°Π·Π½ΠΎΡΡ‚ΡŒ ΠΏΠΎΡ‚Π΅Π½Ρ†ΠΈΠ°Π»ΠΎΠ² Π½Π° Π΅Π³ΠΎ ΠΊΠ»Π΅ΠΌΠΌΠ°Ρ….

ΠžΠ±Ρ‰ΠΈΠΉ Ρ‚ΠΎΠΊ (It) ΠΌΠΎΠΆΠ΅Ρ‚ Π±Ρ‹Ρ‚ΡŒ рассчитан с использованиСм уравнСния,

Π”Π°Π²Π°ΠΉΡ‚Π΅ рассмотрим, Ρ‡Ρ‚ΠΎ ΠΏΠ°Ρ€Π°Π»Π»Π΅Π»ΡŒΠ½Π°Ρ Ρ†Π΅ΠΏΡŒ построСна с двумя рСзисторами (R1 ΠΈ R2) с Ρ€Π°Π·Π½Ρ‹ΠΌΠΈ значСниями (10 Ом ΠΈ 5 Ом) соотвСтствСнно. НапряТСниС 10V подаСтся Ρ‡Π΅Ρ€Π΅Π· рСзисторы , Π² Ρ€Π΅Π·ΡƒΠ»ΡŒΡ‚Π°Ρ‚Π΅ Ρ‚ΠΎΠΊΠ° 1А , ΠΏΡ€ΠΎΠ²Π΅Π΄Π΅Π½Π½ΠΎΠΉ ΠΎΡ‚ Π±Π°Ρ‚Π°Ρ€Π΅ΠΈ Ρ‡Π΅Ρ€Π΅Π· R1 ΠΈ R2, ΠΊΠΎΡ‚ΠΎΡ€Ρ‹ΠΉ ΠΏΠΎΠ»ΡƒΡ‡Π΅Π½ ΠΈΠ· уравнСния I = U / R.

Π‘Π»Π΅Π΄ΠΎΠ²Π°Ρ‚Π΅Π»ΡŒΠ½ΠΎ, Π΄Π²Π° Ρ‚ΠΎΠΊΠ° вСтвлСния Π² Ρ†Π΅ΠΏΠΈ ΡΠΎΡΡ‚Π°Π²Π»ΡΡŽΡ‚ 1А ΠΈ 2А, ΠΊΠΎΡ‚ΠΎΡ€Ρ‹Π΅ ΡΡƒΠΌΠΌΠΈΡ€ΡƒΡŽΡ‚ Π΄ΠΎ 3А.

БопротивлСния Π² ΠΏΠ°Ρ€Π°Π»Π»Π΅Π»ΡŒΠ½ΠΎΠΉ Ρ†Π΅ΠΏΠΈ

ΠžΠ±Ρ‰Π΅Π΅ сопротивлСниС любого количСства рСзисторов рассчитываСтся ΠΏΠΎ ΡƒΡ€Π°Π²Π½Π΅Π½ΠΈΡŽ,

Π’Π·Π°ΠΈΠΌΠ½ΠΎΠ΅ Π·Π½Π°Ρ‡Π΅Π½ΠΈΠ΅ R1 = 1/R1 = 1/10 = 0,1

Π’Π·Π°ΠΈΠΌΠ½ΠΎΠ΅ ΠΎΡ‚ R2 = 1/R2 = 1/5 = 0,2

Π‘ΡƒΠΌΠΌΠ° ΠΎΠ±Ρ€Π°Ρ‚Π½Ρ‹Ρ… Π²Ρ‹ΡˆΠ΅ = 0,3

R t = 1 / 0,3 = 3,33 Ом

ΠœΠΎΡ‰Π½ΠΎΡΡ‚ΡŒ Π² ΠΏΠ°Ρ€Π°Π»Π»Π΅Π»ΡŒΠ½ΠΎΠΉ Ρ†Π΅ΠΏΠΈ

Как Ρ‚ΠΎΠ»ΡŒΠΊΠΎ ΠΎΠ±Ρ‰ΠΈΠΉ Ρ‚ΠΎΠΊ ΠΈ ΠΏΡ€ΠΈΠ»ΠΎΠΆΠ΅Π½Π½Ρ‹Π΅ значСния напряТСния извСстны, ΠΌΠΎΡ‰Π½ΠΎΡΡ‚ΡŒ ΠΌΠΎΠΆΠ΅Ρ‚ Π±Ρ‹Ρ‚ΡŒ рассчитана с использованиСм уравнСния P = UI . Π’ ΠΏΡ€ΠΈΠ²Π΅Π΄Π΅Π½Π½ΠΎΠΌ Π²Ρ‹ΡˆΠ΅ ΠΏΡ€ΠΈΠΌΠ΅Ρ€Π΅, ΠΏΡ€ΠΈΠ»ΠΎΠΆΠ΅Π½Π½ΠΎΠ΅ напряТСниС U = 10Π’ ΠΈ I = 3A, P = 10Γ—3 = 30 Π’Ρ‚

НазначСниС ΠΈ ΠΎΠΏΡ€Π΅Π΄Π΅Π»Π΅Π½ΠΈΠ΅ импСданса

ΠŸΡ€Π°ΠΊΡ‚ΠΈΡ‡Π΅ΡΠΊΠΈ Π½ΠΈ ΠΎΠ΄Π½ΠΎ элСктронноС устройство Π½Π΅ обходится Π² своСй схСмС Π±Π΅Π· рСзисторов. Являясь пассивными элСмСнтами, ΠΎΠ½ΠΈ ΠΈΠΌΠ΅ΡŽΡ‚ основноС ΠΏΡ€Π΅Π΄Π½Π°Π·Π½Π°Ρ‡Π΅Π½ΠΈΠ΅ β€” ΠΎΠ³Ρ€Π°Π½ΠΈΡ‡ΠΈΠ²Π°Ρ‚ΡŒ Π²Π΅Π»ΠΈΡ‡ΠΈΠ½Ρƒ Ρ‚ΠΎΠΊΠ° Π² элСктричСской Ρ†Π΅ΠΏΠΈ. ΠšΡ€ΠΎΠΌΠ΅ токоограничСния, ΠΎΠ½ΠΈ слуТат дСлитСлями напряТСния ΠΈΠ»ΠΈ ΡˆΡƒΠ½Ρ‚Π°ΠΌΠΈ Π² ΠΈΠ·ΠΌΠ΅Ρ€ΠΈΡ‚Π΅Π»ΡŒΠ½Ρ‹Ρ… ΠΏΡ€ΠΈΠ±ΠΎΡ€Π°Ρ….

ЭлСктричСскоС сопротивлСниС β€” это Π²Π΅Π»ΠΈΡ‡ΠΈΠ½Π°, ΠΈΠΌΠ΅ΡŽΡ‰Π°Ρ Ρ„ΠΈΠ·ΠΈΡ‡Π΅ΡΠΊΡƒΡŽ ΠΏΡ€ΠΈΡ€ΠΎΠ΄Ρƒ ΠΈ Ρ…Π°Ρ€Π°ΠΊΡ‚Π΅Ρ€ΠΈΠ·ΡƒΡŽΡ‰Π°Ρ Π²ΠΎΠ·ΠΌΠΎΠΆΠ½ΠΎΡΡ‚ΡŒ ΠΏΡ€ΠΎΠ²ΠΎΠ΄Π½ΠΈΠΊΠ° ΠΏΡ€ΠΎΠΏΡƒΡΠΊΠ°Ρ‚ΡŒ элСктричСский Ρ‚ΠΎΠΊ. ΠŸΡ€ΠΈΠ½Ρ†ΠΈΠΏ Ρ€Π°Π±ΠΎΡ‚Ρ‹ рСзистора Π±Ρ‹Π» описан Π²Ρ‹Π΄Π°ΡŽΡ‰ΠΈΠΌΡΡ экспСримСнтатором Омом. ПозТС Π² Π΅Π³ΠΎ Ρ‡Π΅ΡΡ‚ΡŒ ΠΈ Π±Ρ‹Π»Π° Π½Π°Π·Π²Π°Π½Π° Π΅Π΄ΠΈΠ½ΠΈΡ†Π° измСрСния элСктричСского сопротивлСния β€” Ом. Π£Ρ‡Ρ‘Π½Ρ‹ΠΉ, проводя ряд экспСримСнтов, установил Π·Π°Π²ΠΈΡΠΈΠΌΠΎΡΡ‚ΡŒ ΠΌΠ΅ΠΆΠ΄Ρƒ силой Ρ‚ΠΎΠΊΠ°, напряТСниСм ΠΈ сопротивлСниСм Π² ΠΏΡ€ΠΎΠ²ΠΎΠ΄Π½ΠΈΠΊΠ΅. Π’ Ρ€Π΅Π·ΡƒΠ»ΡŒΡ‚Π°Ρ‚Π΅ Π±Ρ‹Π»Π° Π²Ρ‹Π²Π΅Π΄Π΅Π½Π° простая Ρ„ΠΎΡ€ΠΌΡƒΠ»Π°, извСстная ΠΊΠ°ΠΊ Π·Π°ΠΊΠΎΠ½ Ома: I = U/R, Π³Π΄Π΅:

  • I β€” проходящая Ρ‡Π΅Ρ€Π΅Π· ΠΏΡ€ΠΎΠ²ΠΎΠ΄Π½ΠΈΠΊ сила Ρ‚ΠΎΠΊΠ°, измСряСмая Π² АмпСрах;
  • U β€” напряТСниС, ΠΏΡ€ΠΈΠ»ΠΎΠΆΠ΅Π½Π½ΠΎΠ΅ ΠΊ ΠΏΡ€ΠΎΠ²ΠΎΠ΄Π½ΠΈΠΊΡƒ, Π΅Π΄ΠΈΠ½ΠΈΡ†Π° измСрСния β€” Π’ΠΎΠ»ΡŒΡ‚;
  • R β€” сопротивлСниС ΠΏΡ€ΠΎΠ²ΠΎΠ΄Π½ΠΈΠΊΠ°, измСряСтся Π² ΠžΠΌΠ°Ρ….

ПозТС устройства, ΠΈΡΠΏΠΎΠ»ΡŒΠ·ΡƒΡŽΡ‰ΠΈΠ΅ΡΡ Ρ‚ΠΎΠ»ΡŒΠΊΠΎ Π² качСствС элСмСнтов сопротивлСния Π² элСктричСских цСпях, ΠΏΠΎΠ»ΡƒΡ‡ΠΈΠ»ΠΈ Π½Π°Π·Π²Π°Π½ΠΈΠ΅ β€” рСзисторы. Π’Π°ΠΊΠΈΠ΅ ΠΏΡ€ΠΈΠ±ΠΎΡ€Ρ‹, ΠΊΡ€ΠΎΠΌΠ΅ Π²Π΅Π»ΠΈΡ‡ΠΈΠ½Ρ‹ сопротивлСния, Ρ…Π°Ρ€Π°ΠΊΡ‚Π΅Ρ€ΠΈΠ·ΡƒΡŽΡ‚ΡΡ ΠΌΠΎΡ‰Π½ΠΎΡΡ‚ΡŒΡŽ, Ρ€Π°ΡΡΡ‡ΠΈΡ‚Ρ‹Π²Π°ΡŽΡ‰Π΅ΠΉΡΡ ΠΏΠΎ ΡΠ»Π΅Π΄ΡƒΡŽΡ‰Π΅ΠΉ Ρ„ΠΎΡ€ΠΌΡƒΠ»Π΅: P = I2 * R. ΠŸΠΎΠ»ΡƒΡ‡Π΅Π½Π½Π°Ρ Π²Π΅Π»ΠΈΡ‡ΠΈΠ½Π° измСряСтся Π² Π’Π°Ρ‚Ρ‚Π°Ρ….

Π’ схСмотСхникС ΠΈΡΠΏΠΎΠ»ΡŒΠ·ΡƒΠ΅Ρ‚ΡΡ ΠΊΠ°ΠΊ ΠΏΠ°Ρ€Π°Π»Π»Π΅Π»ΡŒΠ½ΠΎΠ΅, Ρ‚Π°ΠΊ ΠΈ ΠΏΠΎΡΠ»Π΅Π΄ΠΎΠ²Π°Ρ‚Π΅Π»ΡŒΠ½ΠΎΠ΅ соСдинСниС ΠΏΡ€ΠΎΠ²ΠΎΠ΄Π½ΠΈΠΊΠΎΠ². Π’ зависимости ΠΎΡ‚ этого измСняСтся ΠΈ Π²Π΅Π»ΠΈΡ‡ΠΈΠ½Π° импСданса участка Ρ†Π΅ΠΏΠΈ. Π’ΠΈΠ΄ соСдинСния, Ссли ΠΎΠ½ Π½Π΅ ΠΈΡΠΏΠΎΠ»ΡŒΠ·ΡƒΠ΅Ρ‚ΡΡ для ΠΏΠΎΠ΄Π±ΠΎΡ€Π° Π½ΡƒΠΆΠ½ΠΎΠ³ΠΎ значСния, ΠΊΠ°ΠΊ Ρ€Π°Π· ΠΈ Ρ…Π°Ρ€Π°ΠΊΡ‚Π΅Ρ€ΠΈΠ·ΡƒΠ΅Ρ‚ ΠΏΡ€ΠΈΠΌΠ΅Π½Π΅Π½ΠΈΠ΅ рСзисторов Π² ΠΏΠ΅Ρ€Π²ΠΎΠΌ случаС ΠΊΠ°ΠΊ Ρ‚ΠΎΠΊΠΎΠΎΠ³Ρ€Π°Π½ΠΈΡ‡ΠΈΡ‚Π΅Π»Π΅ΠΉ, Π° Π²ΠΎ Π²Ρ‚ΠΎΡ€ΠΎΠΌ β€” ΠΊΠ°ΠΊ Π΄Π΅Π»ΠΈΡ‚Π΅Π»Π΅ΠΉ напряТСния.

На схСмах рСзисторы ΠΎΠ±ΠΎΠ·Π½Π°Ρ‡Π°ΡŽΡ‚ΡΡ Π² Π²ΠΈΠ΄Π΅ ΠΏΡ€ΡΠΌΠΎΡƒΠ³ΠΎΠ»ΡŒΠ½ΠΈΠΊΠ° ΠΈ ΠΏΠΎΠ΄ΠΏΠΈΡΡ‹Π²Π°ΡŽΡ‚ΡΡ латинской Π±ΡƒΠΊΠ²ΠΎΠΉ R. Рядом указываСтся порядковый Π½ΠΎΠΌΠ΅Ρ€ ΠΈ Π·Π½Π°Ρ‡Π΅Π½ΠΈΠ΅ сопротивлСния. НапримСр, R23 1k ΠΎΠ±ΠΎΠ·Π½Π°Ρ‡Π°Π΅Ρ‚, Ρ‡Ρ‚ΠΎ рСзистор с Π½ΠΎΠΌΠ΅Ρ€ΠΎΠΌ 23 ΠΈΠΌΠ΅Π΅Ρ‚ сопротивлСниС, Ρ€Π°Π²Π½ΠΎΠ΅ ΠΎΠ΄Π½ΠΎΠΌΡƒ ΠΊΠΈΠ»ΠΎΠžΠΌΡƒ. Полоски, ΠΈΠ·ΠΎΠ±Ρ€Π°ΠΆΡ‘Π½Π½Ρ‹Π΅ Π²Π½ΡƒΡ‚Ρ€ΠΈ ΠΏΡ€ΡΠΌΠΎΡƒΠ³ΠΎΠ»ΡŒΠ½ΠΈΠΊΠ°, Ρ…Π°Ρ€Π°ΠΊΡ‚Π΅Ρ€ΠΈΠ·ΡƒΡŽΡ‚ ΠΌΠΎΡ‰Π½ΠΎΡΡ‚ΡŒ, Ρ€Π°ΡΡΠ΅ΠΈΠ²Π°Π΅ΠΌΡƒΡŽ Π½Π° ΠΏΡ€ΠΎΠ²ΠΎΠ΄Π½ΠΈΠΊΠ΅.

Π€ΡƒΠ½Π΄Π°ΠΌΠ΅Π½Ρ‚Π°Π»ΡŒΠ½Ρ‹ΠΉ Π·Π°ΠΊΠΎΠ½ сохранСния энСргии гласит: энСргия Π½ΠΈΠΊΡƒΠ΄Π° Π½Π΅ исчСзаСт ΠΈ ΠΈΠ· Π½ΠΈΠΎΡ‚ΠΊΡƒΠ΄Π° Π½Π΅ появляСтся, Π° Ρ‚ΠΎΠ»ΡŒΠΊΠΎ измСняСт Ρ„ΠΎΡ€ΠΌΡƒ. ΠŸΠΎΡΡ‚ΠΎΠΌΡƒ ΠΏΡ€ΠΈ ΠΎΠ³Ρ€Π°Π½ΠΈΡ‡Π΅Π½ΠΈΠΈ Ρ‚ΠΎΠΊΠ° Ρ‡Π°ΡΡ‚ΡŒ энСргии трансформируСтся Π² Ρ‚Π΅ΠΏΠ»ΠΎ. ИмСнно эту Ρ‡Π°ΡΡ‚ΡŒ ΠΈ Π½Π°Π·Ρ‹Π²Π°ΡŽΡ‚ ΠΌΠΎΡ‰Π½ΠΎΡΡ‚ΡŒΡŽ рассСивания рСзистора, Ρ‚. Π΅. Ρ‚Π°ΠΊΡƒΡŽ Π΅Ρ‘ Π²Π΅Π»ΠΈΡ‡ΠΈΠ½Ρƒ, ΠΊΠΎΡ‚ΠΎΡ€ΡƒΡŽ ΠΌΠΎΠΆΠ΅Ρ‚ Π²Ρ‹Π΄Π΅Ρ€ΠΆΠ°Ρ‚ΡŒ сопротивлСниС Π±Π΅Π· измСнСния своих ΠΏΠ°Ρ€Π°ΠΌΠ΅Ρ‚Ρ€ΠΎΠ².

Π‘Π°ΠΌ ΠΏΠΎ сСбС рСзистор ΠΌΠΎΠΆΠ΅Ρ‚ ΠΈΠΌΠ΅Ρ‚ΡŒ Ρ€Π°Π·Π»ΠΈΡ‡Π½ΡƒΡŽ ΠΊΠΎΠ½ΡΡ‚Ρ€ΡƒΠΊΡ†ΠΈΡŽ ΠΈ Π²ΠΈΠ΄. НапримСр, Π±Ρ‹Ρ‚ΡŒ ΠΏΡ€ΠΎΠ²ΠΎΠ»ΠΎΡ‡Π½Ρ‹ΠΌ, кСрамичСским, ΡΠ»ΡŽΠ΄ΡΠ½Ρ‹ΠΌ ΠΈ Ρ‚. ΠΏ. ΠœΠ°Ρ€ΠΊΠΈΡ€ΡƒΠ΅Ρ‚ΡΡ ΠΎΠ½ трСмя способами:

  1. Π¦Π²Π΅Ρ‚Π½ΠΎΠΉ полосочной систСмой. КаТдая полоска ΠΎΡ‚Π²Π΅Ρ‡Π°Π΅Ρ‚ Π·Π° ΠΎΠΏΡ€Π΅Π΄Π΅Π»Ρ‘Π½Π½Ρ‹ΠΉ ΠΌΠ½ΠΎΠΆΠΈΡ‚Π΅Π»ΡŒ. Π Π°ΡΡˆΠΈΡ„Ρ€ΠΎΠ²ΠΊΡƒ полосок ΠΌΠΎΠΆΠ½ΠΎ Π²Π·ΡΡ‚ΡŒ ΠΈΠ· справочников ΠΈΠ»ΠΈ ΠΎΠ½Π»Π°ΠΉΠ½-ΠΊΠ°Π»ΡŒΠΊΡƒΠ»ΡΡ‚ΠΎΡ€ΠΎΠ².
  2. Π¦ΠΈΡ„Ρ€Π°ΠΌΠΈ ΠΈ Π±ΡƒΠΊΠ²Π°ΠΌΠΈ. Число ΡƒΠΊΠ°Π·Ρ‹Π²Π°Π΅Ρ‚ нСпосрСдствСнно Π·Π½Π°Ρ‡Π΅Π½ΠΈΠ΅ сопротивлСния, Π° Π±ΡƒΠΊΠ²Π° β€” ΠΌΠ½ΠΎΠΆΠΈΡ‚Π΅Π»ΡŒ. НапримСр,15M β€” ΠΏΡΡ‚Π½Π°Π΄Ρ†Π°Ρ‚ΡŒ мСгаОм.
  3. Цифровая. ΠžΠ±Ρ‹Ρ‡Π½ΠΎ ΠΈΡΠΏΠΎΠ»ΡŒΠ·ΡƒΡŽΡ‚ΡΡ Ρ‚Ρ€ΠΈ Ρ†ΠΈΡ„Ρ€Ρ‹, пСрвая ΠΈ вторая ΠΎΠ±ΠΎΠ·Π½Π°Ρ‡Π°ΡŽΡ‚ Π·Π½Π°Ρ‡Π΅Π½ΠΈΠ΅ сопротивлСния, Π° послСдняя β€” ΠΌΠ½ΠΎΠΆΠΈΡ‚Π΅Π»ΡŒ. НапримСр, 103 β€” Π΄Π΅ΡΡΡ‚ΡŒ килоОм.

ΠŸΠΎΡΡ‚ΠΎΠΌΡƒ видя, ΠΊΠ°ΠΊΠΈΠ΅ рСзисторы установлСны Π² схСмС, Π΄Π°ΠΆΠ΅ Π½Π°Ρ‡ΠΈΠ½Π°ΡŽΡ‰Π΅ΠΌΡƒ Ρ€Π°Π΄ΠΈΠΎΠ»ΡŽΠ±ΠΈΡ‚Π΅Π»ΡŽ Π½Π΅ составит Ρ‚Ρ€ΡƒΠ΄Π° Ρ€Π°ΡΡΡ‡ΠΈΡ‚Π°Ρ‚ΡŒ ΠΎΠ±Ρ‰Π΅Π΅ сопротивлСниС, особСнно ΠΈΡΠΏΠΎΠ»ΡŒΠ·ΡƒΡ ΠΎΠ½Π»Π°ΠΉΠ½-ΠΊΠ°Π»ΡŒΠΊΡƒΠ»ΡΡ‚ΠΎΡ€ ΠΏΠ°Ρ€Π°Π»Π»Π΅Π»ΡŒΠ½ΠΎΠ³ΠΎ соСдинСния рСзисторов ΠΈΠ»ΠΈ ΠΏΠΎΡΠ»Π΅Π΄ΠΎΠ²Π°Ρ‚Π΅Π»ΡŒΠ½ΠΎΠ³ΠΎ. Π’ случаС нСвозмоТности Ρ€Π°Π·Π»ΠΈΡ‡ΠΈΡ‚ΡŒ ΠΌΠ°Ρ€ΠΊΠΈΡ€ΠΎΠ²ΠΊΡƒ Π½Π° корпусС Π΅Π³ΠΎ сопротивлСниС Π²ΠΎΠ·ΠΌΠΎΠΆΠ½ΠΎ ΠΈΠ·ΠΌΠ΅Ρ€ΠΈΡ‚ΡŒ ΠΌΡƒΠ»ΡŒΡ‚ΠΈΠΌΠ΅Ρ‚Ρ€ΠΎΠΌ. Но ΠΎΠΏΡ‹Ρ‚Π½Ρ‹Π΅ элСктротСхники Π·Π½Π°ΡŽΡ‚, Ρ‡Ρ‚ΠΎ для Ρ‚ΠΎΡ‡Π½ΠΎΠ³ΠΎ измСрСния понадобится ΠΎΠ΄ΠΈΠ½ Π²Ρ‹Π²ΠΎΠ΄ сопротивлСния ΠΎΡ‚ΡΠΎΠ΅Π΄ΠΈΠ½ΠΈΡ‚ΡŒ ΠΎΡ‚ схСмы. Бвязано это ΠΊΠ°ΠΊ Ρ€Π°Π· с Π²ΠΈΠ΄ΠΎΠΌ ΠΏΠΎΠ΄ΠΊΠ»ΡŽΡ‡Π΅Π½ΠΈΡ ΠΏΡ€ΠΎΠ²ΠΎΠ΄Π½ΠΈΠΊΠ°.

НСдостатки ΠΏΠ°Ρ€Π°Π»Π»Π΅Π»ΡŒΠ½ΠΎΠΉ Ρ†Π΅ΠΏΠΈ

НСдостатки ΠΏΠ°Ρ€Π°Π»Π»Π΅Π»ΡŒΠ½Ρ‹Ρ… Ρ†Π΅ΠΏΠ΅ΠΉ пСрСчислСны Π½ΠΈΠΆΠ΅:

  • Π”ΠΎΡ€ΠΎΠ³ΠΎ ΡΡ‚Ρ€ΠΎΠΈΡ‚ΡŒ
  • ΠšΠΎΡ€ΠΎΡ‚ΠΊΠΎΠ΅ Π·Π°ΠΌΡ‹ΠΊΠ°Π½ΠΈΠ΅ ΠΌΠΎΠΆΠ΅Ρ‚ ΠΏΡ€ΠΎΠΈΠ·ΠΎΠΉΡ‚ΠΈ случайно Π² ΠΏΠ°Ρ€Π°Π»Π»Π΅Π»ΡŒΠ½ΠΎΠΉ ΠΏΡ€ΠΎΠ²ΠΎΠ΄ΠΊΠ΅ ΠΈ ΠΌΠΎΠΆΠ΅Ρ‚ Π½Π°Ρ‡Π°Ρ‚ΡŒΡΡ ΠΏΠΎΠΆΠ°Ρ€
  • Π”Π°ΠΆΠ΅ Ссли ΠΎΠ΄ΠΈΠ½ ΠΈΠ· ΠΊΠΎΠΌΠΏΠΎΠ½Π΅Π½Ρ‚ΠΎΠ² нСисправСн, Ρ‚ΠΎΠΊ всС Ρ€Π°Π²Π½ΠΎ ΠΌΠΎΠΆΠ΅Ρ‚ ΠΏΡ€ΠΎΡ…ΠΎΠ΄ΠΈΡ‚ΡŒ Ρ‡Π΅Ρ€Π΅Π· Ρ†Π΅ΠΏΡŒ.

Π’ΠΈΠΌΠ΅Ρ€ΠΊΠ°Π΅Π² Борис β€” 68-Π»Π΅Ρ‚Π½ΠΈΠΉ Π΄ΠΎΠΊΡ‚ΠΎΡ€ Ρ„ΠΈΠ·ΠΈΠΊΠΎ-матСматичСских Π½Π°ΡƒΠΊ, профСссор ΠΈΠ· России. Он являСтся Π·Π°Π²Π΅Π΄ΡƒΡŽΡ‰ΠΈΠΌ ΠΊΠ°Ρ„Π΅Π΄Ρ€ΠΎΠΉ ΠΎΠ±Ρ‰Π΅ΠΉ Ρ„ΠΈΠ·ΠΈΠΊΠΈ Π² Казанском Π½Π°Ρ†ΠΈΠΎΠ½Π°Π»ΡŒΠ½ΠΎΠΌ ΠΈΡΡΠ»Π΅Π΄ΠΎΠ²Π°Ρ‚Π΅Π»ΡŒΡΠΊΠΎΠΌ тСхничСском унивСрситСтС ΠΈΠΌΠ΅Π½ΠΈ А. Н. Π’Π£ΠŸΠžΠ›Π•Π’Π β€” КАИ

Π€ΠΎΡ€ΠΌΡƒΠ»Π° для расчСта ΠΏΠ°Ρ€Π°Π»Π»Π΅Π»ΡŒΠ½ΠΎΠ³ΠΎ соСдинСния сопротивлСний

ΠŸΡ€ΠΈ ΠΏΠ°Ρ€Π°Π»Π»Π΅Π»ΡŒΠ½ΠΎΠΌ соСдинСнии обратная Π²Π΅Π»ΠΈΡ‡ΠΈΠ½Π° ΠΎΡ‚ эквивалСнтного сопротивлСния Ρ€Π°Π²Π½Π° суммС ΠΎΠ±Ρ€Π°Ρ‚Π½Ρ‹Ρ… Π²Π΅Π»ΠΈΡ‡ΠΈΠ½ всСх ΠΏΠ°Ρ€Π°Π»Π»Π΅Π»ΡŒΠ½ΠΎ ΠΏΠΎΠ΄ΠΊΠ»ΡŽΡ‡Π΅Π½Π½Ρ‹Ρ… сопротивлСний. ЭквивалСнтная ΠΏΡ€ΠΎΠ²ΠΎΠ΄ΠΈΠΌΠΎΡΡ‚ΡŒ Ρ€Π°Π²Π½Π° суммС всСх ΠΏΠ°Ρ€Π°Π»Π»Π΅Π»ΡŒΠ½ΠΎ ΠΏΠΎΠ΄ΠΊΠ»ΡŽΡ‡Π΅Π½Π½Ρ‹Ρ… проводимостСй элСктричСской схСмы.

Для ΠΏΡ€ΠΈΠ²Π΅Π΄Π΅Π½Π½ΠΎΠΉ Π²Ρ‹ΡˆΠ΅ схСмы эквивалСнтноС сопротивлСниС ΠΌΠΎΠΆΠ½ΠΎ Ρ€Π°ΡΡΡ‡ΠΈΡ‚Π°Ρ‚ΡŒ ΠΏΠΎ Ρ„ΠΎΡ€ΠΌΡƒΠ»Π΅:

Π’ частном случаС ΠΏΡ€ΠΈ ΠΏΠΎΠ΄ΠΊΠ»ΡŽΡ‡Π΅Π½ΠΈΠΈ ΠΏΠ°Ρ€Π°Π»Π»Π΅Π»ΡŒΠ½ΠΎ Π΄Π²ΡƒΡ… сопротивлСний:

Π­ΠΊΠ²ΠΈΠ²Π°Π»Π΅Π½Ρ‚Π½ΠΎΠ΅ сопротивлСниС Ρ†Π΅ΠΏΠΈ опрСдСляСтся ΠΏΠΎ Ρ„ΠΎΡ€ΠΌΡƒΠ»Π΅:

Π’ случаС ΠΏΠΎΠ΄ΠΊΠ»ΡŽΡ‡Π΅Π½ΠΈΡ β€œn” ΠΎΠ΄ΠΈΠ½Π°ΠΊΠΎΠ²Ρ‹Ρ… сопротивлСний, эквивалСнтноС сопротивлСниС ΠΌΠΎΠΆΠ½ΠΎ Ρ€Π°ΡΡΡ‡ΠΈΡ‚Π°Ρ‚ΡŒ ΠΏΠΎ частной Ρ„ΠΎΡ€ΠΌΡƒΠ»Π΅:

Π€ΠΎΡ€ΠΌΡƒΠ»Ρ‹ для частного рассчСта Π²Ρ‹Ρ‚Π΅ΠΊΠ°ΡŽΡ‚ ΠΈΠ· основной Ρ„ΠΎΡ€ΠΌΡƒΠ»Ρ‹.

Бвойства рСзисторов ΠΏΡ€ΠΈ ΠΏΠ°Ρ€Π°Π»Π»Π΅Π»ΡŒΠ½ΠΎΠΌ ΠΏΠΎΠ΄ΠΊΠ»ΡŽΡ‡Π΅Π½ΠΈΠΈ

ΠŸΡ€ΠΈ Π΄Π°Π½Π½ΠΎΠΌ Π²ΠΈΠ΄Π΅ соСдинСнии скачки напряТСния Π±ΡƒΠ΄ΡƒΡ‚ ΠΎΠ΄ΠΈΠ½Π°ΠΊΠΎΠ²Ρ‹ Π½Π° всСх участках Ρ†Π΅ΠΏΠΈ. ΠŸΡ€ΠΈ этом ΠΏΠΎΠΊΠ°Π·Π°Ρ‚Π΅Π»ΡŒ, ΠΎΠ±Ρ€Π°Ρ‚Π½Ρ‹ΠΉ суммарному ΡΠΎΠΏΡ€ΠΎΡ‚ΠΈΠ²Π»Π΅Π½ΠΈΡŽ Ρ†Π΅ΠΏΠΈ, Ρ€Π°Π²Π΅Π½ ΠΎΠ±Ρ‰Π΅ΠΉ Π²Π΅Π»ΠΈΡ‡ΠΈΠ½Π΅ рСзисторов.

ΠžΠ±Ρ€Π°Ρ‚ΠΈΡ‚Π΅ Π²Π½ΠΈΠΌΠ°Π½ΠΈΠ΅! F Ρ‚ΠΎΠΊΠ° Π² Π½Π΅Ρ€Π°Π·Π²Π΅Ρ‚Π²Π»Π΅Π½Π½ΠΎΠΉ Ρ‚ΠΎΡ‡ΠΊΠ΅ Ρ†Π΅ΠΏΠΈ равняСтся суммарной силС Ρ‚ΠΎΠΊΠ° Π½Π° ΠΎΡ‚Π΄Π΅Π»ΡŒΠ½Ρ‹Ρ… участках ΠΏΡ€ΠΎΠ²ΠΎΠ΄Π½ΠΈΠΊΠ°.

Π’Π°ΠΌ это Π±ΡƒΠ΄Π΅Ρ‚ интСрСсно ΠžΡΠΎΠ±Π΅Π½Π½ΠΎΡΡ‚ΠΈ ΠΏΠ΅Ρ€Π΅ΠΌΠ΅Π½Π½ΠΎΠ³ΠΎ Ρ‚ΠΎΠΊΠ°

Бтандартная Ρ„ΠΎΡ€ΠΌΡƒΠ»Π° напряТСния

https://youtu. be/q-aIjHdmDgY

ΠŸΡ€ΠΈΠΌΠ΅Ρ€ свСртывания ΠΏΠ°Ρ€Π°Π»Π»Π΅Π»ΡŒΠ½ΠΎΠ³ΠΎ сопротивлСния

Для участка элСктричСской Ρ†Π΅ΠΏΠΈ Π½Π΅ΠΎΠ±Ρ…ΠΎΠ΄ΠΈΠΌΠΎ Π½Π°ΠΉΡ‚ΠΈ ΠΏΠ°Ρ€Π°Π»Π»Π΅Π»ΡŒΠ½ΠΎΠ΅ соСдинСниС сопротивлСний Π²Ρ‹ΠΏΠΎΠ»Π½ΠΈΡ‚ΡŒ ΠΈΡ… ΠΏΡ€Π΅ΠΎΠ±Ρ€Π°Π·ΠΎΠ²Π°Π½ΠΈΠ΅ Π΄ΠΎ ΠΎΠ΄Π½ΠΎΠ³ΠΎ.

Из схСмы Π²ΠΈΠ΄Π½ΠΎ, Ρ‡Ρ‚ΠΎ ΠΏΠ°Ρ€Π°Π»Π»Π΅Π»ΡŒΠ½ΠΎ ΠΏΠΎΠ΄ΠΊΠ»ΡŽΡ‡Π΅Π½Ρ‹ Ρ‚ΠΎΠ»ΡŒΠΊΠΎ R2 ΠΈ R4. R3 Π½Π΅ ΠΏΠ°Ρ€Π°Π»Π»Π΅Π»ΡŒΠ½ΠΎ, Ρ‚.ΠΊ. ΠΎΠ΄Π½ΠΈΠΌ ΠΊΠΎΠ½Ρ†ΠΎΠΌ ΠΎΠ½ΠΎ ΠΏΠΎΠ΄ΠΊΠ»ΡŽΡ‡Π΅Π½ΠΎ ΠΊ источнику Π­Π”Π‘ E1. R1 – ΠΎΠ΄Π½ΠΈΠΌ ΠΊΠΎΠ½Ρ†ΠΎΠΌ ΠΏΠΎΠ΄ΠΊΠ»ΡŽΡ‡Π΅Π½ΠΎ ΠΊ R5, Π° Π½Π΅ ΠΊ ΡƒΠ·Π»Ρƒ. R5 – ΠΎΠ΄Π½ΠΈΠΌ ΠΊΠΎΠ½Ρ†ΠΎΠΌ ΠΏΠΎΠ΄ΠΊΠ»ΡŽΡ‡Π΅Π½ΠΎ ΠΊ R1, Π° Π½Π΅ ΠΊ ΡƒΠ·Π»Ρƒ. МоТно Ρ‚Π°ΠΊ ΠΆΠ΅ Π³ΠΎΠ²ΠΎΡ€ΠΈΡ‚ΡŒ, Ρ‡Ρ‚ΠΎ ΠΏΠΎΡΠ»Π΅Π΄ΠΎΠ²Π°Ρ‚Π΅Π»ΡŒΠ½ΠΎΠ΅ соСдинСниС сопротивлСний R1 ΠΈ R5 ΠΏΠΎΠ΄ΠΊΠ»ΡŽΡ‡Π΅Π½ΠΎ ΠΏΠ°Ρ€Π°Π»Π»Π΅Π»ΡŒΠ½ΠΎ с R2 ΠΈ R4.

Π Π°ΡΡΡ‡ΠΈΡ‚Π°Ρ‚ΡŒ эквивалСнтноС сопротивлСний R14 ΠΌΠΎΠΆΠ½ΠΎ ΠΏΠΎ Ρ„ΠΎΡ€ΠΌΡƒΠ»Π΅ для Π΄Π²ΡƒΡ… сопротивлСний.

Π’ΠΈΠΏΡ‹ ΠΏΡ€ΠΎΠ²ΠΎΠ΄Π½ΠΈΠΊΠΎΠ²

ΠŸΡ€ΠΎΠ²ΠΎΠ΄ΠΈΠΌΠΎΡΡ‚ΡŒ вСщСством элСктричСского Ρ‚ΠΎΠΊΠ° связана с Π½Π°Π»ΠΈΡ‡ΠΈΠ΅ΠΌ Π² Π½Π΅ΠΌ свободных носитСлСй заряда. Π˜Ρ… количСство опрСдСляСтся ΠΏΠΎ элСктронной ΠΊΠΎΠ½Ρ„ΠΈΠ³ΡƒΡ€Π°Ρ†ΠΈΠΈ. Для этого Π½Π΅ΠΎΠ±Ρ…ΠΎΠ΄ΠΈΠΌΠ° химичСская Ρ„ΠΎΡ€ΠΌΡƒΠ»Π° вСщСства, ΠΏΡ€ΠΈ ΠΏΠΎΠΌΠΎΡ‰ΠΈ ΠΊΠΎΡ‚ΠΎΡ€ΠΎΠΉ ΠΌΠΎΠΆΠ½ΠΎ Π²Ρ‹Ρ‡ΠΈΡΠ»ΠΈΡ‚ΡŒ ΠΈΡ… ΠΎΠ±Ρ‰Π΅Π΅ число. Π—Π½Π°Ρ‡Π΅Π½ΠΈΠ΅ для ΠΊΠ°ΠΆΠ΄ΠΎΠ³ΠΎ элСмСнта бСрСтся ΠΈΠ· пСриодичСской систСмы Дмитрия Π˜Π²Π°Π½ΠΎΠ²ΠΈΡ‡Π° МСндСлССва.

ЭлСктричСский Ρ‚ΠΎΠΊ β€” упорядочСнноС Π΄Π²ΠΈΠΆΠ΅Π½ΠΈΠ΅ свободных носитСлСй заряда, Π½Π° ΠΊΠΎΡ‚ΠΎΡ€Ρ‹Π΅ воздСйствуСт элСктромагнитноС ΠΏΠΎΠ»Π΅. ΠŸΡ€ΠΈ ΠΏΡ€ΠΎΡ‚Π΅ΠΊΠ°Π½ΠΈΠΈ Ρ‚ΠΎΠΊΠ° ΠΏΠΎ вСщСству происходит взаимодСйствиС ΠΏΠΎΡ‚ΠΎΠΊΠ° заряТСнных частиц с ΡƒΠ·Π»Π°ΠΌΠΈ кристалличСской Ρ€Π΅ΡˆΠ΅Ρ‚ΠΊΠΈ, ΠΏΡ€ΠΈ этом Ρ‡Π°ΡΡ‚ΡŒ кинСтичСской энСргии частицы прСвращаСтся Π² Ρ‚Π΅ΠΏΠ»ΠΎΠ²ΡƒΡŽ ΡΠ½Π΅Ρ€Π³ΠΈΡŽ. Π˜Π½Ρ‹ΠΌΠΈ словами, частица «ударяСтся» ΠΎΠ± Π°Ρ‚ΠΎΠΌ, Π° Π·Π°Ρ‚Π΅ΠΌ снова ΠΏΡ€ΠΎΠ΄ΠΎΠ»ΠΆΠ°Π΅Ρ‚ Π΄Π²ΠΈΠΆΠ΅Π½ΠΈΠ΅, набирая ΡΠΊΠΎΡ€ΠΎΡΡ‚ΡŒ ΠΏΠΎΠ΄ дСйствиСм элСктромагнитного поля.

ΠŸΡ€ΠΎΡ†Π΅ΡΡ взаимодСйствия частиц с ΡƒΠ·Π»Π°ΠΌΠΈ кристалличСской Ρ€Π΅ΡˆΠ΅Ρ‚ΠΊΠΈ называСтся элСктричСской ΠΏΡ€ΠΎΠ²ΠΎΠ΄ΠΈΠΌΠΎΡΡ‚ΡŒΡŽ ΠΈΠ»ΠΈ сопротивлСниСм ΠΌΠ°Ρ‚Π΅Ρ€ΠΈΠ°Π»Π°. Π•Π΄ΠΈΠ½ΠΈΡ†Π΅ΠΉ измСрСния являСтся Ом, Π° ΠΎΠΏΡ€Π΅Π΄Π΅Π»ΠΈΡ‚ΡŒ Π΅Π³ΠΎ ΠΌΠΎΠΆΠ½ΠΎ ΠΏΡ€ΠΈ ΠΏΠΎΠΌΠΎΡ‰ΠΈ ΠΎΠΌΠΌΠ΅Ρ‚Ρ€Π° ΠΈΠ»ΠΈ Ρ€Π°ΡΡ‡ΠΈΡ‚Π°Ρ‚ΡŒ. Богласно свойству проводимости, вСщСства ΠΌΠΎΠΆΠ½ΠΎ Ρ€Π°Π·Π΄Π΅Π»ΠΈΡ‚ΡŒ Π½Π° 3 Π³Ρ€ΡƒΠΏΠΏΡ‹:

  1. ΠŸΡ€ΠΎΠ²ΠΎΠ΄Π½ΠΈΠΊΠΈ (всС ΠΌΠ΅Ρ‚Π°Π»Π»Ρ‹, ΠΈΠΎΠ½ΠΈΠ·ΠΈΡ€ΠΎΠ²Π°Π½Π½Ρ‹ΠΉ Π³Π°Π· ΠΈ элСктролитичСскиС растворы).
  2. ΠŸΠΎΠ»ΡƒΠΏΡ€ΠΎΠ²ΠΎΠ΄Π½ΠΈΠΊΠΈ (Si, Ge, GaAs, InP ΠΈ InSb).
  3. НСпроводники (диэлСктрики ΠΈΠ»ΠΈ изоляторы).

ΠŸΡ€ΠΎΠ²ΠΎΠ΄Π½ΠΈΠΊΠΈ всСгда проводят элСктричСский Ρ‚ΠΎΠΊ, ΠΏΠΎΡΠΊΠΎΠ»ΡŒΠΊΡƒ содСрТат Π² своСм Π°Ρ‚ΠΎΠΌΠ°Ρ€Π½ΠΎΠΌ строСнии свободныС элСктроны, Π°Π½ΠΈΠΎΠ½Ρ‹, ΠΊΠ°Ρ‚ΠΈΠΎΠ½Ρ‹ ΠΈ ΠΈΠΎΠ½Ρ‹. ΠŸΠΎΠ»ΡƒΠΏΡ€ΠΎΠ²ΠΎΠ΄Π½ΠΈΠΊΠΈ проводят элСктричСство Ρ‚ΠΎΠ»ΡŒΠΊΠΎ ΠΏΡ€ΠΈ ΠΎΠΏΡ€Π΅Π΄Π΅Π»Π΅Π½Π½Ρ‹Ρ… условиях, ΠΊΠΎΡ‚ΠΎΡ€Ρ‹Π΅ Π²Π»ΠΈΡΡŽΡ‚ Π½Π° Π½Π°Π»ΠΈΡ‡ΠΈΠ΅ ΠΈΠ»ΠΈ отсутствиС свободных элСктронов ΠΈ Π΄Ρ‹Ρ€ΠΎΠΊ. К Ρ„Π°ΠΊΡ‚ΠΎΡ€Π°ΠΌ, Π²Π»ΠΈΡΡŽΡ‰ΠΈΠΌ Π½Π° ΠΏΡ€ΠΎΠ²ΠΎΠ΄ΠΈΠΌΠΎΡΡ‚ΡŒ, относятся ΡΠ»Π΅Π΄ΡƒΡŽΡ‰ΠΈΠ΅: Ρ‚Π΅ΠΌΠΏΠ΅Ρ€Π°Ρ‚ΡƒΡ€Π°, ΠΎΡΠ²Π΅Ρ‰Π΅Π½Π½ΠΎΡΡ‚ΡŒ ΠΈ Ρ‚. Π΄. ДиэлСктрики Π²ΠΎΠΎΠ±Ρ‰Π΅ Π½Π΅ проводят элСктричСство, ΠΏΠΎΡΠΊΠΎΠ»ΡŒΠΊΡƒ Π² ΠΈΡ… структурС Π²ΠΎΠΎΠ±Ρ‰Π΅ ΠΎΡ‚ΡΡƒΡ‚ΡΡ‚Π²ΡƒΡŽΡ‚ свободныС носитСли заряда. ΠŸΡ€ΠΈ Π²Ρ‹ΠΏΠΎΠ»Π½Π΅Π½ΠΈΠΈ расчСтов ΠΊΠ°ΠΆΠ΄Ρ‹ΠΉ Ρ€Π°Π΄ΠΈΠΎΠ»ΡŽΠ±ΠΈΡ‚Π΅Π»ΡŒ Π΄ΠΎΠ»ΠΆΠ΅Π½ Π·Π½Π°Ρ‚ΡŒ Π·Π°Π²ΠΈΡΠΈΠΌΠΎΡΡ‚ΡŒ сопротивлСния ΠΎΡ‚ Π½Π΅ΠΊΠΎΡ‚ΠΎΡ€Ρ‹Ρ… физичСских Π²Π΅Π»ΠΈΡ‡ΠΈΠ½.

ΠŸΠΎΡΠ»Π΅Π΄ΠΎΠ²Π°Ρ‚Π΅Π»ΡŒΠ½ΠΎΠ΅ ΠΈ ΠΏΠ°Ρ€Π°Π»Π»Π΅Π»ΡŒΠ½ΠΎΠ΅ соСдинСниС рСзисторов

Π€ΠΈΠ·ΠΈΠΊΠ° > ΠšΠΎΠΌΠ±ΠΈΠ½ΠΈΡ€ΠΎΠ²Π°Π½Π½Ρ‹Π΅ схСмы

Β 

ΠŸΠΎΡΠ»Π΅Π΄ΠΎΠ²Π°Ρ‚Π΅Π»ΡŒΠ½ΠΎΠ΅ ΠΈ ΠΏΠ°Ρ€Π°Π»Π»Π΅Π»ΡŒΠ½ΠΎΠ΅ соСдинСниС рСзисторов – ΠΊΠΎΠΌΠ±ΠΈΠ½ΠΈΡ€ΠΎΠ²Π°Π½Π½Ρ‹Π΅ схСмы элСктричСской Ρ†Π΅ΠΏΠΈ: ΡΠΌΠ΅ΡˆΠ°Π½Π½Ρ‹ΠΉ Ρ‚ΠΈΠΏ соСдинСния, схСма, Π·Π°ΠΊΠΎΠ½Ρ‹, Ρ€Π΅ΡˆΠ΅Π½ΠΈΠ΅ Π·Π°Π΄Π°Ρ‡.

ΠšΠΎΠΌΠ±ΠΈΠ½ΠΈΡ€ΠΎΠ²Π°Π½Π½ΡƒΡŽ схСму ΠΌΠΎΠΆΠ½ΠΎ Ρ€Π°Π·Π΄Π΅Π»ΠΈΡ‚ΡŒ Π½Π° ΠΏΠΎΡΠ»Π΅Π΄ΠΎΠ²Π°Ρ‚Π΅Π»ΡŒΠ½ΡƒΡŽ ΠΈΠ»ΠΈ ΠΏΠ°Ρ€Π°Π»Π»Π΅Π»ΡŒΠ½ΡƒΡŽ части.

Π—Π°Π΄Π°Ρ‡Π° обучСния

  • Π Π°ΡΡΠΌΠΎΡ‚Ρ€Π΅Ρ‚ΡŒ процСсс раздСлСния рСзисторов Π² ΠΊΠΎΠΌΠ±ΠΈΠ½ΠΈΡ€ΠΎΠ²Π°Π½Π½ΠΎΠΉ схСмС.

ΠžΡΠ½ΠΎΠ²Π½Ρ‹Π΅ ΠΏΡƒΠ½ΠΊΡ‚Ρ‹

  • Π‘Π»ΠΎΠΆΠ½Ρ‹Π΅ соСдинСния рСзисторов Π΄Π΅ΠΌΠΎΠ½ΡΡ‚Ρ€ΠΈΡ€ΡƒΡŽΡ‚ использованиС ΠΏΠ°Ρ€Π°Π»Π»Π΅Π»ΡŒΠ½ΠΎΠ³ΠΎ ΠΈ ΠΏΠΎΡΠ»Π΅Π΄ΠΎΠ²Π°Ρ‚Π΅Π»ΡŒΠ½ΠΎΠ³ΠΎ объСдинСния.
  • Π Π°Π·Π½Ρ‹Π΅ части ΠΊΠΎΠΌΠ±ΠΈΠ½ΠΈΡ€ΠΎΠ²Π°Π½Π½ΠΎΠΉ схСмы ΠΌΠΎΠΆΠ½ΠΎ Ρ€Π°Π·Π΄Π΅Π»ΠΈΡ‚ΡŒ ΠΈ ΡƒΠΌΠ΅Π½ΡŒΡˆΠΈΡ‚ΡŒ, ΠΏΠΎΠΊΠ° Π½Π΅ останСтся Π΅Π΄ΠΈΠ½ΠΎΠ΅ сопротивлСниС.
  • Π‘ΠΎΠΏΡ€ΠΎΡ‚ΠΈΠ²Π»Π΅Π½ΠΈΠ΅ Π² ΠΏΡ€ΠΎΠ²ΠΎΠ΄Π°Ρ… сокращаСт Ρ‚ΠΎΠΊ ΠΈ ΠΌΠΎΡ‰Π½ΠΎΡΡ‚ΡŒ, отправляСмыС ΠΊ рСзистору.
  • Если сопротивлСниС высокоС Π² изношСнном ΠΈΠ»ΠΈ слишком Π΄Π»ΠΈΠ½Π½ΠΎΠΌ ΡˆΠ½ΡƒΡ€Π΅, Ρ‚ΠΎ ΠΏΠΎΡ‚Π΅Ρ€ΠΈ ΠΌΠΎΠ³ΡƒΡ‚ Π±Ρ‹Ρ‚ΡŒ ΠΎΡ‰ΡƒΡ‚ΠΈΠΌΡ‹ΠΌΠΈ.

Π’Π΅Ρ€ΠΌΠΈΠ½Ρ‹

  • ΠŸΠΎΡΠ»Π΅Π΄ΠΎΠ²Π°Ρ‚Π΅Π»ΡŒΠ½ΠΎΡΡ‚ΡŒ – ΠΏΠΎΠ΄ΠΊΠ»ΡŽΡ‡Π΅Π½ΠΈΠ΅ Π½Π΅ΡΠΊΠΎΠ»ΡŒΠΊΠΈΡ… элСмСнтов ΠΏΠΎΠΎΡ‡Π΅Ρ€Π΅Π΄Π½ΠΎ.
  • ΠŸΠ°Ρ€Π°Π»Π»Π΅Π»ΡŒΠ½ΠΎΡΡ‚ΡŒ – элСмСнты располоТСны Ρ‚Π°ΠΊ, Ρ‡Ρ‚ΠΎΠ±Ρ‹ Ρ‚ΠΎΠΊ ΠΏΡ€ΠΎΡ‚Π΅ΠΊΠ°Π» вдоль Π΄Π²ΡƒΡ… ΠΈΠ»ΠΈ Π±ΠΎΠ»Π΅Π΅ ΠΏΡƒΡ‚Π΅ΠΉ.
  • ΠšΠΎΠΌΠ±ΠΈΠ½ΠΈΡ€ΠΎΠ²Π°Π½Π½Π°Ρ схСма – элСктричСская схСма с нСсколькими рСзисторами, ΠΎΠ±ΡŠΠ΅Π΄ΠΈΠ½Π΅Π½Π½Ρ‹ΠΌΠΈ ΠΏΠΎΡΠ»Π΅Π΄ΠΎΠ²Π°Ρ‚Π΅Π»ΡŒΠ½ΠΎ ΠΈ ΠΏΠ°Ρ€Π°Π»Π»Π΅Π»ΡŒΠ½ΠΎ.

ΠšΠΎΠΌΠ±ΠΈΠ½ΠΈΡ€ΠΎΠ²Π°Π½Π½Ρ‹Π΅ схСмы

Иногда слоТныС соСдинСния рСзисторов ΠΈΡΠΏΠΎΠ»ΡŒΠ·ΡƒΡŽΡ‚ сразу ΠΏΠΎΡΠ»Π΅Π΄ΠΎΠ²Π°Ρ‚Π΅Π»ΡŒΠ½ΠΎΠ΅ ΠΈ ΠΏΠ°Ρ€Π°Π»Π»Π΅Π»ΡŒΠ½ΠΎΠ΅ ΠΏΠΎΠ΄ΠΊΠ»ΡŽΡ‡Π΅Π½ΠΈΡ. Π­Ρ‚ΠΎ частоС явлСниС, Ρ‚Π°ΠΊ ΠΊΠ°ΠΊ Π² ΠΏΡ€ΠΎΠ²ΠΎΠ΄Π΅ Π²ΠΎΠ·Π½ΠΈΠΊΠ°Π΅Ρ‚ сопротивлСниС. Π’Π°ΠΊΡƒΡŽ схСму ΠΌΠΎΠΆΠ½ΠΎ Ρ€Π°Π·Π΄Π΅Π»ΠΈΡ‚ΡŒ Π½Π° ΡΠΎΠΎΡ‚Π²Π΅Ρ‚ΡΡ‚Π²ΡƒΡŽΡ‰ΠΈΠ΅ части. На рисункС ΠΏΠΎΠΊΠ°Π·Π°Π½ΠΎ, Ρ‡Ρ‚ΠΎ ΠΎΠ±Ρ‰Π΅Π΅ сопротивлСниС Π²Ρ‹Ρ‡ΠΈΡΠ»ΡΡŽΡ‚, связав Ρ‚Ρ€ΠΈ рСзистора ΠΏΠ°Ρ€Π°Π»Π»Π΅Π»ΡŒΠ½ΠΎ ΠΈΠ»ΠΈ ΠΏΠΎΡΠ»Π΅Π΄ΠΎΠ²Π°Ρ‚Π΅Π»ΡŒΠ½ΠΎ. R1 ΠΈ R2 ΠΏΠΎΠ΄ΠΊΠ»ΡŽΡ‡Π΅Π½Ρ‹ ΠΏΠ°Ρ€Π°Π»Π»Π΅Π»ΡŒΠ½ΠΎ, поэтому Ρ„ΠΎΡ€ΠΌΡƒΠ»Π°:

Π’ ΠΊΠΎΠΌΠ±ΠΈΠ½ΠΈΡ€ΠΎΠ²Π°Π½Π½ΠΎΠΉ схСмС рСзисторы ΠΌΠΎΠΆΠ½ΠΎ Ρ€Π°Π·Π΄Π΅Π»ΠΈΡ‚ΡŒ Π½Π° ΠΏΠΎΡΠ»Π΅Π΄ΠΎΠ²Π°Ρ‚Π΅Π»ΡŒΠ½Ρ‹Π΅ ΠΈ ΠΏΠ°Ρ€Π°Π»Π»Π΅Π»ΡŒΠ½Ρ‹Π΅ ΡΠΎΡΡ‚Π°Π²Π»ΡΡŽΡ‰ΠΈΠ΅

R3 соСдинСн ΠΏΠΎΡΠ»Π΅Π΄ΠΎΠ²Π°Ρ‚Π΅Π»ΡŒΠ½ΠΎ с R1 ΠΈ R2, поэтому сопротивлСниС Π±ΡƒΠ΄Π΅Ρ‚ Ρ€Π°ΡΡΡ‡ΠΈΡ‚Ρ‹Π²Π°Ρ‚ΡŒΡΡ ΠΊΠ°ΠΊ:

ΠšΠΎΠΌΠΏΠ»Π΅ΠΊΡΠ½Ρ‹Π΅ ΠΊΠΎΠΌΠ±ΠΈΠ½Π°Ρ†ΠΈΠΎΠ½Π½Ρ‹Π΅ схСмы

Π’ ситуациях с Π±ΠΎΠ»Π΅Π΅ слоТными схСмами части ΠΌΠΎΠΆΠ½ΠΎ ΠΈΠ΄Π΅Π½Ρ‚ΠΈΡ„ΠΈΡ†ΠΈΡ€ΠΎΠ²Π°Ρ‚ΡŒ ΠΊΠ°ΠΊ ΠΏΠΎΡΠ»Π΅Π΄ΠΎΠ²Π°Ρ‚Π΅Π»ΡŒΠ½Ρ‹Π΅ ΠΈ ΠΏΠ°Ρ€Π°Π»Π»Π΅Π»ΡŒΠ½Ρ‹Π΅, Π° ΠΏΠΎΡ‚ΠΎΠΌ свСсти ΠΈΡ… ΠΊ эквивалСнтам ΠΈ ΡƒΠΌΠ΅Π½ΡŒΡˆΠΈΡ‚ΡŒ ΠΊ СдинствСнному ΡΠΎΠΏΡ€ΠΎΡ‚ΠΈΠ²Π»Π΅Π½ΠΈΡŽ. На Π½ΠΈΠΆΠ½Π΅ΠΌ рисункС Π²ΠΈΠ΄Π½ΠΎ, Ρ‡Ρ‚ΠΎ ΠΊΠΎΠΌΠ±ΠΈΠ½Π°Ρ†ΠΈΡŽ ΠΈΠ· 7 рСзисторов ΠΎΠΏΡ€Π΅Π΄Π΅Π»ΠΈΠ»ΠΈ ΠΊΠ°ΠΊ ΠΏΠΎΡΠ»Π΅Π΄ΠΎΠ²Π°Ρ‚Π΅Π»ΡŒΠ½ΡƒΡŽ ΠΈ ΠΏΠ°Ρ€Π°Π»Π»Π΅Π»ΡŒΠ½ΡƒΡŽ. На Π½Π°Ρ‡Π°Π»ΡŒΠ½ΠΎΠΌ этапС Π΄Π²Π° ΠΎΠΊΡ€ΡƒΠΆΠ½Ρ‹Ρ… сСчСния ΠΎΡ‚ΠΎΠ±Ρ€Π°ΠΆΠ°ΡŽΡ‚ рСзисторы, располоТСнныС ΠΏΠ°Ρ€Π°Π»Π»Π΅Π»ΡŒΠ½ΠΎ.

ΠšΠΎΠΌΠ±ΠΈΠ½Π°Ρ†ΠΈΡ ΠΈΠ· 7 рСзисторов ΠΎΠ±Π»Π°Π΄Π°Π΅Ρ‚ ΠΏΠΎΡΠ»Π΅Π΄ΠΎΠ²Π°Ρ‚Π΅Π»ΡŒΠ½Ρ‹ΠΌΠΈ ΠΈ ΠΏΠ°Ρ€Π°Π»Π»Π΅Π»ΡŒΠ½Ρ‹ΠΌΠΈ частями. КаТдая опрСдСляСтся ΠΈ сводится ΠΊ эквивалСнтному ΡΠΎΠΏΡ€ΠΎΡ‚ΠΈΠ²Π»Π΅Π½ΠΈΡŽ, ΠΏΠΎΠΊΠ° Π½Π΅ достигнуто СдинствСнноС сопротивлСниС

УмСньшСниС ΠΏΠ°Ρ€Π°Π»Π»Π΅Π»ΡŒΠ½Ρ‹Ρ… рСзисторов Π΄ΠΎ ΠΎΠ΄Π½ΠΎΠ³ΠΎ значСния позволяСт Π·Π½Π°Ρ‡ΠΈΡ‚Π΅Π»ΡŒΠ½ΠΎ ΡƒΠΏΡ€ΠΎΡΡ‚ΠΈΡ‚ΡŒ схСму. Π‘ΠΏΡ€Π°Π²Π° Π²Π²Π΅Ρ€Ρ…Ρƒ Π²ΠΈΠ΄Π½ΠΎ, Ρ‡Ρ‚ΠΎ круглая Ρ‡Π°ΡΡ‚ΡŒ Π²ΠΌΠ΅Ρ‰Π°Π΅Ρ‚ Π΄Π²Π° ΠΏΠΎΡΠ»Π΅Π΄ΠΎΠ²Π°Ρ‚Π΅Π»ΡŒΠ½Ρ‹Ρ… рСзистора. МоТно ΡƒΠΌΠ΅Π½ΡŒΡˆΠΈΡ‚ΡŒ ΠΊ Π΄Ρ€ΡƒΠ³ΠΎΠΌΡƒ Π·Π½Π°Ρ‡Π΅Π½ΠΈΡŽ, Π΄ΠΎΠ±Π°Π²ΠΈΠ² ΠΈΡ…. Π”Π°Π»Π΅Π΅ Π²ΠΈΠ΄Π½ΠΎ, Ρ‡Ρ‚ΠΎ Π΄Π²Π° рСзистора ΠΏΠΎΡΠ»Π΅Π΄ΠΎΠ²Π°Ρ‚Π΅Π»ΡŒΠ½Ρ‹. Π˜Ρ… Ρ‚Π°ΠΊΠΆΠ΅ ΡΠΎΠΊΡ€Π°Ρ‰Π°ΡŽΡ‚ ΠΊ ΠΎΠ΄Π½ΠΎΠΌΡƒ Π·Π½Π°Ρ‡Π΅Π½ΠΈΡŽ.

Π‘ΠΎΠΏΡ€ΠΎΡ‚ΠΈΠ²Π»Π΅Π½ΠΈΠ΅ Π² ΠΏΡ€ΠΎΠ²ΠΎΠ΄Π°Ρ… ΡƒΠΌΠ΅Π½ΡŒΡˆΠ°Π΅Ρ‚ количСство Ρ‚ΠΎΠΊΠ° ΠΈ ΠΌΠΎΡ‰Π½ΠΎΡΡ‚ΡŒ, ΠΏΠΎΡΡ‚ΡƒΠΏΠ°ΡŽΡ‰ΠΈΠ΅ ΠΊ рСзистору. Для опрСдСлСния ΠΎΠ±Ρ‰Π΅Π³ΠΎ сопротивлСния Ρ†Π΅ΠΏΠΈ ΠΌΠΎΠΆΠ½ΠΎ ΠΈΡΠΏΠΎΠ»ΡŒΠ·ΠΎΠ²Π°Ρ‚ΡŒ Ρ†Π΅Π»Π΅Π²ΡƒΡŽ схСму. Π‘ΠΎΠΏΡ€ΠΎΡ‚ΠΈΠ²Π»Π΅Π½ΠΈΠ΅ ΠΏΡ€ΠΎΠ²ΠΎΠ΄Π° – сСрия рСзисторов. Если ΠΎΠ½ΠΎ слишком Π²Π΅Π»ΠΈΠΊΠΎ Π² Π΄Π»ΠΈΠ½Π½ΠΎΠΌ ΠΈΠ»ΠΈ ΠΏΠΎΠ²Ρ€Π΅ΠΆΠ΄Π΅Π½Π½ΠΎΠΌ ΠΏΡ€ΠΎΠ²ΠΎΠ΄Π΅, Ρ‚ΠΎ ΠΌΡ‹ ΠΎΡ‚ΠΌΠ΅Ρ‡Π°Π΅ΠΌ Π·Π½Π°Ρ‡ΠΈΡ‚Π΅Π»ΡŒΠ½ΡƒΡŽ ΠΏΠΎΡ‚Π΅Ρ€ΡŽ Ρ‚ΠΎΠΊΠ°.


Π‘Ρ…Π΅ΠΌΡ‹ ΠΏΠ°Ρ€Π°Π»Π»Π΅Π»ΡŒΠ½ΠΎΠ³ΠΎ ΠΈ ΠΏΠΎΡΠ»Π΅Π΄ΠΎΠ²Π°Ρ‚Π΅Π»ΡŒΠ½ΠΎΠ³ΠΎ соСдинСния рСзисторов

ΠŸΠΎΡΠ»Π΅Π΄ΠΎΠ²Π°Ρ‚Π΅Π»ΡŒΠ½ΠΎΠ΅ ΠΈ ΠΏΠ°Ρ€Π°Π»Π»Π΅Π»ΡŒΠ½ΠΎΠ΅ соСдинСниС рСзисторов

ΠŸΠΎΡΠ»Π΅Π΄ΠΎΠ²Π°Ρ‚Π΅Π»ΡŒΠ½ΠΎΠ΅ соСдинСниС рСзисторов

ΠŸΠΎΡΠ»Π΅Π΄ΠΎΠ²Π°Ρ‚Π΅Π»ΡŒΠ½ΠΎΠ΅ соСдинСниС — это соСдинСниС Π΄Π²ΡƒΡ… ΠΈΠ»ΠΈ Π±ΠΎΠ»Π΅Π΅ рСзисторов Π² ΠΎΠ΄Π½ΠΎΠΉ Ρ†Π΅ΠΏΠΈ, Π² ΠΊΠΎΡ‚ΠΎΡ€ΠΎΠΉ ΠΎΡ‚Π΄Π΅Π»ΡŒΠ½Ρ‹ΠΉ рСзистор соСдиняСтся с Π΄Ρ€ΡƒΠ³ΠΈΠΌ рСзистором Π² ΠΎΠ΄Π½ΠΎΠΉ Ρ‚ΠΎΡ‡ΠΊΠ΅.

ΠžΠ±Ρ‰Π΅Π΅ сопротивлСниС R ΠΎΠ±Ρ‰

ΠŸΡ€ΠΈ Ρ‚Π°ΠΊΠΎΠΌ соСдинСнии, Ρ‡Π΅Ρ€Π΅Π· всС рСзисторы ΠΏΡ€ΠΎΡ…ΠΎΠ΄ΠΈΡ‚ ΠΎΠ΄ΠΈΠ½ ΠΈ Ρ‚ΠΎΡ‚ ΠΆΠ΅ элСктричСский Ρ‚ΠΎΠΊ. Π§Π΅ΠΌ большС элСмСнтов Π½Π° Π΄Π°Π½Π½ΠΎΠΌ участкС элСктричСской Ρ†Π΅ΠΏΠΈ, Ρ‚Π΅ΠΌ Β«Ρ‚Ρ€ΡƒΠ΄Π½Π΅Π΅Β» Ρ‚ΠΎΠΊΡƒ ΠΏΡ€ΠΎΡ‚Π΅ΠΊΠ°Ρ‚ΡŒ Ρ‡Π΅Ρ€Π΅Π· Π½Π΅Π³ΠΎ. Π‘Π»Π΅Π΄ΠΎΠ²Π°Ρ‚Π΅Π»ΡŒΠ½ΠΎ, ΠΏΡ€ΠΈ ΠΏΠΎΡΠ»Π΅Π΄ΠΎΠ²Π°Ρ‚Π΅Π»ΡŒΠ½ΠΎΠΌ соСдинСнии рСзисторов ΠΎΠ±Ρ‰Π΅Π΅ сопротивлСниС увСличиваСтся, ΠΈ ΠΎΠ½ΠΎ Ρ€Π°Π²Π½ΠΎ суммС всСх сопротивлСний.

НапряТСниС ΠΏΡ€ΠΈ ΠΏΠΎΡΠ»Π΅Π΄ΠΎΠ²Π°Ρ‚Π΅Π»ΡŒΠ½ΠΎΠΌ соСдинСнии

НапряТСниС ΠΏΡ€ΠΈ ΠΏΠΎΡΠ»Π΅Π΄ΠΎΠ²Π°Ρ‚Π΅Π»ΡŒΠ½ΠΎΠΌ соСдинСнии распрСдСляСтся Π½Π° ΠΊΠ°ΠΆΠ΄ΠΎΠΌ рСзисторС согласно Π·Π°ΠΊΠΎΠ½Ρƒ Ома:

Π’.Π΅ Ρ‡Π΅ΠΌ большСС сопротивлСниС ΠΏΠ°Π΄Π°Π΅Ρ‚, Ρ‚Π΅ΠΌ большСС напряТСниС Π½Π° Π½Π΅Π³ΠΎ ΠΏΠ°Π΄Π°Π΅Ρ‚.

ΠŸΠ°Ρ€Π°Π»Π»Π΅Π»ΡŒΠ½ΠΎΠ΅ соСдинСниС рСзисторов

ΠŸΠ°Ρ€Π°Π»Π»Π΅Π»ΡŒΠ½ΠΎΠ΅ соСдинСниС — это соСдинСниС, ΠΏΡ€ΠΈ ΠΊΠΎΡ‚ΠΎΡ€ΠΎΠΌ рСзисторы ΡΠΎΠ΅Π΄ΠΈΠ½ΡΡŽΡ‚ΡΡ ΠΌΠ΅ΠΆΠ΄Ρƒ собой ΠΎΠ±ΠΎΠΈΠΌΠΈ ΠΊΠΎΠ½Ρ‚Π°ΠΊΡ‚Π°ΠΌΠΈ.Π’ Ρ€Π΅Π·ΡƒΠ»ΡŒΡ‚Π°Ρ‚Π΅ ΠΊ ΠΎΠ΄Π½ΠΎΠΉ Ρ‚ΠΎΡ‡ΠΊΠ΅ (элСктричСскому ΡƒΠ·Π»Ρƒ) ΠΌΠΎΠΆΠ΅Ρ‚ Π±Ρ‹Ρ‚ΡŒ присоСдинСно нСсколько рСзисторов.

ΠžΠ±Ρ‰Π΅Π΅ сопротивлСниС R ΠΎΠ±Ρ‰

ΠŸΡ€ΠΈ Ρ‚Π°ΠΊΠΎΠΌ соСдинСнии, Ρ‡Π΅Ρ€Π΅Π· ΠΊΠ°ΠΆΠ΄Ρ‹ΠΉ рСзистор ΠΏΠΎΡ‚Π΅Ρ‡Π΅Ρ‚ ΠΎΡ‚Π΄Π΅Π»ΡŒΠ½Ρ‹ΠΉ Ρ‚ΠΎΠΊ. Π‘ΠΈΠ»Π° Π΄Π°Π½Π½ΠΎΠ³ΠΎ Ρ‚ΠΎΠΊΠ° Π±ΡƒΠ΄Π΅Ρ‚ ΠΎΠ±Ρ€Π°Ρ‚Π½ΠΎ ΠΏΡ€ΠΎΠΏΠΎΡ€Ρ†ΠΈΠΎΠ½Π°Π»ΡŒΠ½Π° ΡΠΎΠΏΡ€ΠΎΡ‚ΠΈΠ²Π»Π΅Π½ΠΈΡŽ рСзистора. ΠžΠ±Ρ‰Π΅Π΅ сопротивлСниС Ρ‚Π°ΠΊΠΎΠ³ΠΎ участка элСктричСской Ρ†Π΅ΠΏΠΈ увСличиваСтся.

Π’Π°ΠΊΠΈΠΌ ΠΎΠ±Ρ€Π°Π·ΠΎΠΌ, ΠΏΡ€ΠΈ ΠΏΠ°Ρ€Π°Π»Π»Π΅Π»ΡŒΠ½ΠΎΠΌ ΠΏΠΎΠ΄ΠΊΠ»ΡŽΡ‡Π΅Π½ΠΈΠΈ рСзисторов с Ρ€Π°Π·Π½Ρ‹ΠΌ сопротивлСниСм, ΠΎΠ±Ρ‰Π΅Π΅ сопротивлСниС Π±ΡƒΠ΄Π΅Ρ‚ всСгда мСньшС значСния самого малСнького ΠΎΡ‚Π΄Π΅Π»ΡŒΠ½ΠΎΠ³ΠΎ сопротивлСния.

Π€ΠΎΡ€ΠΌΡƒΠ»Π° ΠΎΠ±Ρ‰Π΅ΠΉ проводимости ΠΏΡ€ΠΈ ΠΏΠ°Ρ€Π°Π»Π»Π΅Π»ΡŒΠ½ΠΎΠΌ соСдинСнии рСзисторов:

Π€ΠΎΡ€ΠΌΡƒΠ»Π° эквивалСнтного ΠΎΠ±Ρ‰Π΅Π³ΠΎ сопротивлСния ΠΏΡ€ΠΈ ΠΏΠ°Ρ€Π°Π»Π»Π΅Π»ΡŒΠ½ΠΎΠΌ соСдинСнии рСзисторов:

Для Π΄Π²ΡƒΡ… ΠΎΠ΄ΠΈΠ½Π°ΠΊΠΎΠ²Ρ‹Ρ… рСзисторов ΠΎΠ±Ρ‰Π΅Π΅ сопротивлСниС Π±ΡƒΠ΄Π΅Ρ‚ Ρ€Π°Π²Π½ΠΎ ΠΎΠ΄Π½ΠΎΠ³ΠΎ ΠΎΡ‚Π΄Π΅Π»ΡŒΠ½ΠΎΠ³ΠΎ рСзистора:

БоотвСтствСнно, для ΠΎΠ΄ΠΈΠ½Π°ΠΊΠΎΠ²Ρ‹Ρ… рСзисторов ΠΎΠ±Ρ‰Π΅Π΅ сопротивлСниС Π±ΡƒΠ΄Π΅Ρ‚ Ρ€Π°Π²Π½ΠΎ Π·Π½Π°Ρ‡Π΅Π½ΠΈΡŽ ΠΎΠ΄Π½ΠΎΠ³ΠΎ рСзистора, Ρ€Π°Π·Π΄Π΅Π»Π΅Π½Π½ΠΎΠ³ΠΎ Π½Π° n.

НапряТСниС ΠΏΡ€ΠΈ ΠΏΠ°Ρ€Π°Π»Π»Π΅Π»ΡŒΠ½ΠΎΠΌ соСдинСнии

НапряТСниС ΠΌΠ΅ΠΆΠ΄Ρƒ Ρ‚ΠΎΡ‡ΠΊΠ°ΠΌΠΈ A ΠΈ B являСтся ΠΎΠ±Ρ‰ΠΈΠΌ напряТСниСм для всСго участка Ρ†Π΅ΠΏΠΈ, Ρ‚Π°ΠΊ ΠΈ напряТСниСм, ΠΏΠ°Π΄Π°ΡŽΡ‰ΠΈΠΌ Π½Π° ΠΊΠ°ΠΆΠ΄Ρ‹ΠΉ рСзистор Π² ΠΎΡ‚Π΄Π΅Π»ΡŒΠ½ΠΎΡΡ‚ΠΈ.ΠŸΠΎΡΡ‚ΠΎΠΌΡƒ ΠΏΡ€ΠΈ ΠΏΠ°Ρ€Π°Π»Π»Π΅Π»ΡŒΠ½ΠΎΠΌ соСдинСнии Π½Π° всС рСзисторы ΡƒΠΏΠ°Π΄Π΅Ρ‚ ΠΎΠ΄ΠΈΠ½Π°ΠΊΠΎΠ²ΠΎΠ΅ напряТСниС.

ЭлСктричСский Ρ‚ΠΎΠΊ ΠΏΡ€ΠΈ ΠΏΠ°Ρ€Π°Π»Π»Π΅Π»ΡŒΠ½ΠΎΠΌ соСдинСнии

Π§Π΅Ρ€Π΅Π· ΠΊΠ°ΠΆΠ΄Ρ‹ΠΉ рСзистор Ρ‚Π΅Ρ‡Π΅Ρ‚ Ρ‚ΠΎΠΊ, сила ΠΊΠΎΡ‚ΠΎΡ€ΠΎΠ³ΠΎ ΠΎΠ±Ρ€Π°Ρ‚Π½ΠΎ ΠΏΡ€ΠΎΠΏΠΎΡ€Ρ†ΠΈΠΎΠ½Π°Π»ΡŒΠ½Π° ΡΠΎΠΏΡ€ΠΎΡ‚ΠΈΠ²Π»Π΅Π½ΠΈΡŽ рСзистора. Для Ρ‚ΠΎΠ³ΠΎ, Ρ‡Ρ‚ΠΎΠ±Ρ‹ ΡƒΠ·Π½Π°Ρ‚ΡŒ ΠΊΠ°ΠΊΠΎΠΉ Ρ‚ΠΎΠΊ Ρ‚Π΅Ρ‡Π΅Ρ‚ Ρ‡Π΅Ρ€Π΅Π· ΠΎΠΏΡ€Π΅Π΄Π΅Π»Π΅Π½Π½Ρ‹ΠΉ рСзистор, ΠΌΠΎΠΆΠ½ΠΎ Π²ΠΎΡΠΏΠΎΠ»ΡŒΠ·ΠΎΠ²Π°Ρ‚ΡŒΡΡ Π·Π°ΠΊΠΎΠ½ΠΎΠΌ Ома:

БмСшанноС соСдинСниС рСзисторов

Π‘ΠΌΠ΅ΡˆΠ°Π½Π½Ρ‹ΠΌ соСдинСниСм Π½Π°Π·Ρ‹Π²Π°ΡŽΡ‚ участок Ρ†Π΅ΠΏΠΈ, Π³Π΄Π΅ Ρ‡Π°ΡΡ‚ΡŒ рСзисторов соСдиняСтся ΠΌΠ΅ΠΆΠ΄Ρƒ собой, Π° Ρ‡Π°ΡΡ‚ΡŒ ΠΏΠ°Ρ€Π°Π»Π»Π΅Π»ΡŒΠ½ΠΎ. Π’ свою ΠΎΡ‡Π΅Ρ€Π΅Π΄ΡŒ, смСшанноС соСдинСниС Π±Ρ‹Π²Π°Π΅Ρ‚ ΠΏΠΎΡΠ»Π΅Π΄ΠΎΠ²Π°Ρ‚Π΅Π»ΡŒΠ½ΠΎΠ³ΠΎ ΠΈ ΠΏΠ°Ρ€Π°Π»Π»Π΅Π»ΡŒΠ½ΠΎΠ³ΠΎ Ρ‚ΠΈΠΏΠΎΠ².

ΠžΠ±Ρ‰Π΅Π΅ сопротивлСниС R ΠΎΠ±Ρ‰

Для Ρ‚ΠΎΠ³ΠΎ Ρ‡Ρ‚ΠΎΠ±Ρ‹ ΠΏΠΎΡΡ‡ΠΈΡ‚Π°Ρ‚ΡŒ ΠΎΠ±Ρ‰Π΅Π΅ сопротивлСниС смСшанного соСдинСния:

o ЦСпь Ρ€Π°Π·Π±ΠΈΠ²Π°ΡŽΡ‚ Π½Π° участки с Ρ‚ΠΎΠ»ΡŒΠΊΠΎ ΠΏΠ°Ρ€Π΅Π»Π»Π΅Π»ΡŒΠ½Ρ‹ΠΌ ΠΈΠ»ΠΈ Ρ‚ΠΎΠ»ΡŒΠΊΠΎ ΠΏΠΎΡΠ»Π΅Π΄ΠΎΠ²Π°Ρ‚Π΅Π»ΡŒΠ½Ρ‹ΠΌ соСдинСниСм. o Π’Ρ‹Ρ‡ΠΈΡΠ»ΠΈΡ‚Π΅Π»ΡŒΠ½ΠΎΠ΅ сопротивлСниС для ΠΊΠ°ΠΆΠ΄ΠΎΠ³ΠΎ ΠΎΡ‚Π΄Π΅Π»ΡŒΠ½ΠΎΠ³ΠΎ участка.

o ВычислСнноС ΠΎΠ±Ρ‰Π΅Π΅ сопротивлСниС для всСй Ρ†Π΅ΠΏΠΈ смСшанного соСдинСния.

Π’Π°ΠΊ это Π±ΡƒΠ΄Π΅Ρ‚ Π²Ρ‹Π³Π»ΡΠ΄Π΅Ρ‚ΡŒ для схСмы 1:

Π’Π°ΠΊΠΆΠ΅ сущСствуСт Π±ΠΎΠ»Π΅Π΅ быстрый способ расчСта ΠΎΠ±Ρ‰Π΅Π³ΠΎ сопротивлСния для смСшанного соСдинСния.МоТно, Π² соотвСтствии, сразу Π·Π°ΠΏΠΈΡΡ‹Π²Π°Ρ‚ΡŒ Ρ„ΠΎΡ€ΠΌΡƒΠ»Ρƒ ΡΠ»Π΅Π΄ΡƒΡŽΡ‰ΠΈΠΌ ΠΎΠ±Ρ€Π°Π·ΠΎΠΌ:

o Если рСзисторы ΡΠΎΠ΅Π΄ΠΈΠ½ΡΡŽΡ‚ΡΡ ΠΏΠΎΡΠ»Π΅Π΄ΠΎΠ²Π°Ρ‚Π΅ΡŒΠ½ΠΎ — ΡΠΊΠ»Π°Π΄Ρ‹Π²Π°Ρ‚ΡŒ.

o Если рСзисторы ΡΠΎΠ΅Π΄ΠΈΠ½ΡΡŽΡ‚ ΠΏΠ°Ρ€Π°Π»Π»Π΅Π»ΡŒΠ½ΠΎ — ΠΈΡΠΏΠΎΠ»ΡŒΠ·ΠΎΠ²Π°Ρ‚ΡŒ условноС ΠΎΠ±ΠΎΠ·Π½Π°Ρ‡Π΅Π½ΠΈΠ΅ «||». o ΠŸΠΎΠ΄ΡΡ‚Π°Π²Π»ΡΡ‚ΡŒ Ρ„ΠΎΡ€ΠΌΡƒΠ»Ρƒ для ΠΏΠ°Ρ€Π°Π»Π»Π΅Π»ΡŒΠ½ΠΎΠ³ΠΎ Π³Π΄Π΅ стоит символ «||».

Π’Π°ΠΊ это Π±ΡƒΠ΄Π΅Ρ‚ Π²Ρ‹Π³Π»ΡΠ΄Π΅Ρ‚ΡŒ для схСмы 1:

ПослС подстановки ΠΏΠ°Ρ€Π°Π»Π»Π΅Π»ΡŒΠ½ΠΎΠ³ΠΎ соСдинСния вмСсто «||»:

ΠžΠ±Ρ‰ΠΈΠ΅ свСдСния

РСзистор

Π§Π°Ρ‰Π΅ всСго рСзисторы ΡƒΠΌΠ΅Π½ΡŒΡˆΠ°ΡŽΡ‚ сСбя ΠΌΠ΅Ρ‚Π°Π»Π»ΠΈΡ‡Π΅ΡΠΊΡƒΡŽ ΠΏΡ€ΠΎΠ²ΠΎΠ»ΠΎΠΊΡƒ ΠΈΠ»ΠΈ полоску, для компактности Π½Π°ΠΌΠΎΡ‚Π°Π½Π½ΡƒΡŽ ΡΡ‚Π΅Ρ€ΠΆΠ΅Π½ΡŒ (Ρ‡Π΅ΠΌ Π΄Π»ΠΈΠ½Π½Π΅ΠΉ ΠΏΡ€ΠΎΠ²ΠΎΠ΄Π½ΠΈΠΊ ΠΈ Ρ‡Π΅ΠΌ мСньшС Π΅Π³ΠΎ ΠΏΠΎΠΏΠ΅Ρ€Π΅Ρ‡Π½ΠΎΠ΅ сСчСниС, Ρ‚Π΅ΠΌ Π²Ρ‹ΡˆΠ΅ сопротивлСниС). РазумССтся, сопротивлСниС Ρ‚Π°ΠΊΠΆΠ΅ зависит ΠΎΡ‚ ΠΌΠ°Ρ‚Π΅Ρ€ΠΈΠ°Π»Π°, ΠΈΠ· ΠΊΠΎΡ‚ΠΎΡ€ΠΎΠ³ΠΎ ΠΈΠ·Π³ΠΎΡ‚ΠΎΠ²Π»Π΅Π½ ΠΏΡ€ΠΎΠ²ΠΎΠ΄Π½ΠΈΠΊ. ΠŸΠΎΠ»ΡŽΠ±ΠΎΠ²Π°Ρ‚ΡŒΡΡ Π½Π° рСзисторы ΠΌΠΎΠΆΠ½ΠΎ Π½Π° рисункС 50.1. «РСзисторы (с сайта Π Π°Π΄ΠΈΠΎΠšΠΎΡ‚)Β».

На элСктричСских схСмах рСзистор ΠΎΠ±Ρ‹Ρ‡Π½ΠΎ ΠΈΠ·ΠΎΠ±Ρ€Π°ΠΆΠ°ΡŽΡ‚ ΠΊΠ°ΠΊ ΠΏΡ€ΡΠΌΠΎΡƒΠ³ΠΎΠ»ΡŒΠ½ΠΈΠΊ, ΠΈΠ· ΠΊΠΎΡ‚ΠΎΡ€ΠΎΠ³ΠΎ выходят Π΄Π²Π° Π²Ρ‹Π²ΠΎΠ΄Π° (рисунок 50.2. «БхСматичСскоС ΠΈΠ·ΠΎΠ±Ρ€Π°ΠΆΠ΅Π½ΠΈΠ΅ рСзистора»).

ΠŸΠΎΡΠ»Π΅Π΄ΠΎΠ²Π°Ρ‚Π΅Π»ΡŒΠ½ΠΎΠ΅ ΠΈ ΠΏΠ°Ρ€Π°Π»Π»Π΅Π»ΡŒΠ½ΠΎΠ΅ соСдинСниС рСзисторов

ΠžΡ‡Π΅Π²ΠΈΠ΄Π½ΠΎ, имССтся Π΄Π²Π΅ возмоТности для соСдинСния Π΄Π²ΡƒΡ… рСзисторов: ΠΌΠΎΠΆΠ½ΠΎ ΠΈΡ… ΡΠΏΠ°ΡΡ‚ΡŒ ΠΎΠ΄Π½ΠΈΠΌ ΠΊΠΎΠ½Ρ†ΠΎΠΌ ΠΈΠ»ΠΈ ΠΆΠ΅ ΠΎΠ±ΠΎΠΈΠΌΠΈ.ΠŸΠ΅Ρ€Π²Ρ‹ΠΉ способ ΠΏΠΎΡΠ»Π΅Π΄ΠΎΠ²Π°Ρ‚Π΅Π»ΡŒΠ½Ρ‹ΠΌ соСдинСниСм, Π° Π²Ρ‚ΠΎΡ€ΠΎΠΉ — ΠΏΠ°Ρ€Π°Π»Π»Π΅Π»ΡŒΠ½Ρ‹ΠΌ (рисунок 50.3. Β«ΠŸΠΎΡΠ»Π΅Π΄ΠΎΠ²Π°Ρ‚Π΅Π»ΡŒΠ½ΠΎΠ΅ ΠΈ ΠΏΠ°Ρ€Π°Π»Π»Π΅Π»ΡŒΠ½ΠΎΠ΅ соСдинСниС рСзисторов»).

И ΠΏΠΎΡΠ»Π΅Π΄ΠΎΠ²Π°Ρ‚Π΅Π»ΡŒΠ½ΠΎΠ΅, ΠΈ ΠΏΠ°Ρ€Π°Π»Π»Π΅Π»ΡŒΠ½ΠΎΠ΅ соСдинСниС рСзисторов ΠΌΠΎΠΆΠ½ΠΎ Ρ€Π°ΡΡΠΌΠ°Ρ‚Ρ€ΠΈΠ²Π°Ρ‚ΡŒ ΠΊΠ°ΠΊ Π½ΠΎΠ²Ρ‹ΠΉ рСзистор. Π•Π³ΠΎ сопротивлСниС ΠΌΠΎΠΆΠ½ΠΎ Π²Ρ‹Ρ‡ΠΈΡΠ»ΠΈΡ‚ΡŒ, ΠΏΠΎΠ»ΡŒΠ·ΡƒΡΡΡŒ ΠΈΡΠΏΠΎΠ»ΡŒΠ·ΡƒΠ΅ΠΌΡ‹ΠΌ:

  • ΠŸΡ€ΠΈ ΠΏΠΎΡΠ»Π΅Π΄ΠΎΠ²Π°Ρ‚Π΅Π»ΡŒΠ½ΠΎΠΌ соСдинСнии рСзисторов ΠΈΡ… сопротивлСниСм ΡΠΊΠ»Π°Π΄Ρ‹Π²Π°ΡŽΡ‚ΡΡ: R = R 1 + R 2.
  • ΠŸΡ€ΠΈ ΠΏΠ°Ρ€Π°Π»Π»Π΅Π»ΡŒΠ½ΠΎΠΌ соСдинСнии рСзисторов ΡΠΊΠ»Π°Π΄Ρ‹Π²Π°ΡŽΡ‚ΡΡ ΠΈΡ… проводимости, Ρ‚ΠΎ Π΅ΡΡ‚ΡŒ Π²Π΅Π»ΠΈΡ‡ΠΈΠ½Ρ‹, ΠΎΠ±Ρ€Π°Ρ‚Π½Ρ‹Π΅ сопротивлСния: 1 R = 1 R 1 + 1 R 2, ΠΈΠ»ΠΈ R = R 1 ⁒ R 2 R 1 + R 2.

Π’ частности, соСдиняя Π΄Π²Π° ΠΎΠ΄ΠΈΠ½Π°ΠΊΠΎΠ²Ρ‹Ρ… рСзистора с Π΅Π΄ΠΈΠ½ΠΈΡ‡Π½Ρ‹ΠΌ сопротивлСниСм 2 ΠΏΠΎΠ»ΡƒΡ‡ΠΈΠΌ сопротивлСниС, ΠΏΡ€ΠΈ ΠΏΠ°Ρ€Π°Π»Π»Π΅Π»ΡŒΠ½ΠΎΠΌ соСдинСнии 1 2.

ΠŸΡ€ΠΈ соСдинСнии Π±ΠΎΠ»Π΅Π΅ Π΄Π²ΡƒΡ… рСзисторов ΠΈΠ½ΠΎΠ³Π΄Π° удаётся ΠΏΡ€Π΅Π΄ΡΡ‚Π°Π²ΠΈΡ‚ΡŒ ΠΏΠΎΠ»ΡƒΡ‡Π΅Π½Π½ΡƒΡŽ схСму ΠΊΠ°ΠΊ ΠΏΠΎΡΠ»Π΅Π΄ΠΎΠ²Π°Ρ‚Π΅Π»ΡŒΠ½ΠΎΠ΅ ΠΈΠ»ΠΈ ΠΏΠ°Ρ€Π°Π»Π»Π΅Π»ΡŒΠ½ΠΎΠ΅ соСдинСниС Π΄Π²ΡƒΡ… подсхСм. НапримСр, схСма Π½Π° рисунок 50.4. «БмСшанноС соСдинСниС рСзисторов» прСдставляСтся ΠΊΠ°ΠΊ ΠΏΠ°Ρ€Π°Π»Π»Π΅Π»ΡŒΠ½ΠΎΠ΅ соСдинСниС рСзистора R 1 ΠΈ ΠΏΠΎΡΠ»Π΅Π΄ΠΎΠ²Π°Ρ‚Π΅Π»ΡŒΠ½ΠΎΠ³ΠΎ соСдинСния рСзисторов R 2 ΠΈ R 3.Π’Π°ΠΊΠΈΠΌ ΠΎΠ±Ρ€Π°Π·ΠΎΠΌ, сопротивлСниС ΠΌΠ΅ΠΆΠ΄Ρƒ двумя Π²Ρ‹Π΄Π΅Π»Π΅Π½Π½Ρ‹ΠΌΠΈ ΡƒΠ·Π»Π°ΠΌΠΈ вычисляСтся ΠΊΠ°ΠΊ R 1 ⁒ R 2 + R 3 R 1 + R 2 + R 3.

Π‘Π»ΠΎΠΆΠ½ΠΎΠ΅ соСдинСниС рСзисторов

Π£Π²Ρ‹, Π½Π΅ всякая схСма прСдставляСт собой ΠΏΠΎΡΠ»Π΅Π΄ΠΎΠ²Π°Ρ‚Π΅Π»ΡŒΠ½ΠΎΠ΅ ΠΈΠ»ΠΈ ΠΏΠ°Ρ€Π°Π»Π»Π΅Π»ΡŒΠ½ΠΎΠ΅ соСдинСниС Π΄Π²ΡƒΡ… подсхСм, ΠΊΠ°ΠΊ Π½Π΅ всякоС Π½Π°Ρ‚ΡƒΡ€Π°Π»ΡŒΠ½ΠΎΠ΅ число раскладываСтся Π² ΠΏΡ€ΠΎΠΈΠ·Π²Π΅Π΄Π΅Π½ΠΈΠ΅ своих собствСнных Π΄Π΅Π»ΠΈΡ‚Π΅Π»Π΅ΠΉ. ΠŸΡ€ΠΎΡΡ‚ΠΎΠΉ ΠΏΡ€ΠΈΠΌΠ΅Ρ€ Ρ‚Π°ΠΊΠΎΠΉ Π½Π΅Ρ€Π°Π·Π»ΠΎΠΆΠΈΠΌΠΎΠΉ схСмы ΠΌΠΎΠΆΠ½ΠΎ ΡƒΠ²ΠΈΠ΄Π΅Ρ‚ΡŒ Π½Π° рисункС 50.5. Β«Π‘Π»ΠΎΠΆΠ½ΠΎΠ΅ соСдинСниС рСзисторов».

Для расчёта Ρ‚Π°ΠΊΠΈΡ… сопротивлСний ΠΈΡΠΏΠΎΠ»ΡŒΠ·ΡƒΠΉΡ‚Π΅, ΠΊΡ€ΠΎΠΌΠ΅ Π·Π°ΠΊΠΎΠ½Π° Ома, Π΅Ρ‰Ρ‘ ΠΈ Π·Π°ΠΊΠΎΠ½ сохранСния.

ЭлСктричСский Ρ‚ΠΎΠΊ Π² ΠΏΡ€ΠΎΠ²ΠΎΠ΄Π½ΠΈΠΊΠ΅ ΠΌΠΎΠΆΠ½ΠΎ ΠΏΡ€Π΅Π΄ΡΡ‚Π°Π²ΠΈΡ‚ΡŒ сСбС ΠΊΠ°ΠΊ ΠΏΠΎΡ‚ΠΎΠΊ частиц, нСсущих элСктричСскиС заряды (это ΠΌΠΎΠ³ΡƒΡ‚ Π±Ρ‹Ρ‚ΡŒ элСктроны ΠΈΠ»ΠΈ ΠΈΠΎΠ½Ρ‹). ΠŸΡ€ΠΈΡ‡ΠΈΠ½ΠΎΠΉ Ρ‚Π°ΠΊΠΎΠ³ΠΎ двиТСния заряТСнных частиц являСтся Ρ€Π°Π·Π½ΠΎΡΡ‚ΡŒ ΠΏΠΎΡ‚Π΅Π½Ρ†ΠΈΠ°Π»ΠΎΠ² Π½Π° ΠΊΠΎΠ½Ρ†Π°Ρ… ΠΏΡ€ΠΎΠ²ΠΎΠ΄Π½ΠΈΠΊΠ° (напряТСниС). ВозмоТная энСргия силы тяТСсти Π² ΠΎΡ‚Π΄Π΅Π»ΡŒΠ½ΠΎΠΉ Ρ‚ΠΎΡ‡ΠΊΠ΅, Π° Π²Π°ΠΆΠ΅Π½ ΠΏΠ΅Ρ€Π΅ΠΏΠ°Π΄ ΠΏΠΎΡ‚Π΅Π½Ρ†ΠΈΠ°Π»ΡŒΠ½ΠΎΠΉ энСргии Π² Π΄Π²ΡƒΡ… Ρ‚ΠΎΡ‡ΠΊΠ°Ρ…).Π’ΠΎΠΊ — это суммарный заряд, ΠΏΡ€ΠΎΡ‚Π΅ΠΊΠ°ΡŽΡ‰ΠΈΠΉ Ρ‡Π΅Ρ€Π΅Π· ΠΏΠΎΠΏΠ΅Ρ€Π΅Ρ‡Π½ΠΎΠ΅ сСчСниС ΠΏΡ€ΠΎΠ²ΠΎΠ΄Π½ΠΈΠΊΠ° Π·Π° Π΅Π΄ΠΈΠ½ΠΈΡ†Ρƒ Π²Ρ€Π΅ΠΌΠ΅Π½ΠΈ. ΠŸΡ€Π΅Π΄ΡΡ‚Π°Π²ΠΈΠΌ Ρ‚Π°ΠΊΡƒΡŽ β€‹β€‹ΠΌΠΎΠ΄Π΅Π»ΡŒ: ΠΏΠΎ Π΄ΠΎΡ€ΠΎΠ³Π΅ ΠΈΠ· ΠΏΡƒΠ½ΠΊΡ‚Π° B двиТСтся ΠΏΠΎΡ‚ΠΎΠΊ Π°Π²Ρ‚ΠΎΠΌΠΎΠ±ΠΈΠ»Π΅ΠΉ, ΠΊΠ°ΠΆΠ΄Ρ‹ΠΉ ΠΈΠ· ΠΊΠΎΡ‚ΠΎΡ€Ρ‹Ρ… Π·Π°Π³Ρ€ΡƒΠΆΠ΅Π½ зарядом. Если заряды ΠΏΠΎΠ»ΠΎΠΆΠΈΡ‚Π΅Π»ΡŒΠ½Ρ‹, считаСтся, Ρ‡Ρ‚ΠΎ Ρ‚ΠΎΠΊ Π² Π½Π°ΠΏΡ€Π°Π²Π»Π΅Π½ΠΈΠΈ ΠΎΡ‚ A ΠΊ B ΠΏΠΎΠ»ΠΎΠΆΠΈΡ‚Π΅Π»Π΅Π½. Но ΠΌΠΎΠΆΠ½ΠΎ ΡΡ‡ΠΈΡ‚Π°Ρ‚ΡŒ Ρ‚Π°ΠΊΠΆΠ΅, Ρ‡Ρ‚ΠΎ имССтся ΠΎΡ‚Ρ€ΠΈΡ†Π°Ρ‚Π΅Π»ΡŒΠ½Ρ‹ΠΉ Ρ‚ΠΎΠΊ (Ρ‚ΠΎΠΉ ΠΆΠ΅ самой Π°Π±ΡΠΎΠ»ΡŽΡ‚Π½ΠΎΠΉ Π²Π΅Π»ΠΈΡ‡ΠΈΠ½Ρ‹) Π² Π½Π°ΠΏΡ€Π°Π²Π»Π΅Π½ΠΈΠΈ ΠΎΡ‚ B ΠΊ A.

Π—Π°ΠΊΠΎΠ½ сохранСния энСргии, Ρ‡Ρ‚ΠΎ элСктричСскиС заряды Π½Π΅ Π²ΠΎΠ·Π½ΠΈΠΊΠ°ΡŽΡ‚ Π½ΠΈΠΎΡ‚ΠΊΡƒΠ΄Π° ΠΈ Π½Π΅ ΠΈΡΡ‡Π΅Π·Π°ΡŽΡ‚ Π² Π½ΠΈΠΊΡƒΠ΄Π°.Если такая элСктричСски Π½Π΅ΠΉΡ‚Ρ€Π°Π»ΡŒΠ½Π°Ρ частица, ΠΊΠ°ΠΊ Π°Ρ‚ΠΎΠΌ, распадаСтся Π½Π° Π΄Π²Π΅ заряТСнных частицы (ΠΈΠΎΠ½ ΠΈ элСктрон), суммарный заряд Π½ΠΎΠ²Ρ‹Ρ… частиц всСгданяСтся заряду Π°Ρ‚ΠΎΠΌΠ°, Ρ‚ΠΎ Π΅ΡΡ‚ΡŒ Π½ΡƒΠ»ΡŽ. Из Π·Π°ΠΊΠΎΠ½Π° слСдуСт, Π² частности, Ρ‡Ρ‚ΠΎ Ρ‚ΠΎΠΊΠΈ Ρ‡Π΅Ρ€Π΅Π· Π΄Π²Π° ΠΏΠΎΠΏΠ΅Ρ€Π΅Ρ‡Π½Ρ‹Ρ… сСчСния Ρ‚ΠΎΠ½ΠΊΠΎΠ³ΠΎ ΠΏΡ€ΠΎΠ²ΠΎΠ΄Π½ΠΈΠΊΠ° Π² ΠΎΠ΄ΠΈΠ½ ΠΈ Ρ‚ΠΎΡ‚ ΠΆΠ΅ ΠΌΠΎΠΌΠ΅Π½Ρ‚ Π²Ρ€Π΅ΠΌΠ΅Π½ΠΈ Ρ€Π°Π²Π½Ρ‹, ΠΈΠ½Π°Ρ‡Π΅ Π³Π΄Π΅-Ρ‚ΠΎ ΠΌΠ΅ΠΆΠ΄Ρƒ этими сСчСниями роТдался Π±Ρ‹ ΠΈΠ»ΠΈ ΠΏΡ€ΠΎΠΏΠ°Π΄Π°Π» Π½Π΅Π½ΡƒΠ»Π΅Π²ΠΎΠΉ заряд. Π”Ρ€ΡƒΠ³ΠΈΠΌ слСдствиСм использования Π²Ρ‹Ρ…ΠΎΠ΄Π° являСтся ΡƒΡ‚Π²Π΅Ρ€ΠΆΠ΄Π΅Π½ΠΈΠ΅, Ρ‡Ρ‚ΠΎ соСдиняСтся нСсколько ΠΏΡ€ΠΎΠ²ΠΎΠ΄Π½ΠΈΠΊΠΎΠ², сумма всСх входящих Π² ΡƒΠ·Π΅Π» Ρ‚ΠΎΠΊΠΎΠ² Ρ€Π°Π²Π½Π° суммС всСх ΡΠΎΠ΅Π΄ΠΈΠ½ΡΡŽΡ‰ΠΈΡ….Если Π²Π΅Ρ€Π½ΡƒΡ‚ΡŒΡΡ ΠΊ Π°Π²Ρ‚ΠΎΠΌΠΎΠ±ΠΈΠ»ΡŒΠ½ΠΎΠΉ Π°Π½Π°Π»ΠΎΠ³ΠΈΠΈ, количСство Π°Π²Ρ‚ΠΎΠΌΠΎΠ±ΠΈΠ»Π΅ΠΉ, Π²ΡŠΠ΅Π·ΠΆΠ°ΡŽΡ‰ΠΈΡ… Π½Π° пСрСкрёсток Π½Π΅ΡΠΊΠΎΠ»ΡŒΠΊΠΈΡ… Π΄ΠΎΡ€ΠΎΠ³, Ρ€Π°Π²Π½ΠΎ количСству Π²Ρ‹Π΅Π·ΠΆΠ°ΡŽΡ‰ΠΈΡ… с пСрСкрёстка (здСсь, ΠΊΠΎΠ½Π΅Ρ‡Π½ΠΎ, принято, Ρ‡Ρ‚ΠΎ ΠΊΠ°ΠΆΠ΄Ρ‹ΠΉ Π°Π²Ρ‚ΠΎΠΌΠΎΠ±ΠΈΠ»ΡŒ Π²Π΅Π·Ρ‘Ρ‚ Π΅Π΄ΠΈΠ½ΠΈΡ‡Π½Ρ‹ΠΉ заряд, ΠΈ врСмя, ΠΏΡ€ΠΎΠ²ΠΎΠ΄ΠΈΠΌΠΎΠ΅ автомобилями Π½Π° пСрСкрёсткС, ΠΏΡ€Π΅Π½Π΅Π±Ρ€Π΅ΠΆΠΈΠΌΠΎ ΠΌΠ°Π»ΠΎ).

. БистСмныС Π΄Π°Π½Π½Ρ‹Π΅, ΠΈΡΠΏΠΎΠ»ΡŒΠ·ΡƒΠ΅ΠΌΡ‹Π΅ элСктричСскиС схСмы Π½Π° рисункС 50.5 ΠΌΠ΅ΠΆΠ΄Ρƒ ΡƒΠ·Π»Π°ΠΌΠΈ. На схСмС ΠΏΡ€ΠΈΡΡƒΡ‚ΡΡ‚Π²ΡƒΡŽΡ‚ ΠΏΡΡ‚ΡŒ рСзисторов ΠΈ Ρ‡Π΅Ρ‚Ρ‹Ρ€Π΅ ΡƒΠ·Π»Π°. ΠŸΡ€ΠΎΠ½ΡƒΠΌΠ΅Ρ€ΡƒΠ΅ΠΌ рСзисторы числами ΠΎΡ‚ 1 Π΄ΠΎ 5 ΠΈ ΡƒΠ·Π»Ρ‹ числами ΠΎΡ‚ 1 Π΄ΠΎ 4.ΠŸΠΎΡ€ΡΠ΄ΠΎΠΊ Π½ΡƒΠΌΠ΅Ρ€Π°Ρ†ΠΈΠΈ ΡƒΠ·Π»ΠΎΠ² ΠΌΠΎΠΆΠ½ΠΎ Π²Ρ‹Π±Ρ€Π°Ρ‚ΡŒ ΡΠΎΠ²Π΅Ρ€ΡˆΠ΅Π½Π½ΠΎ ΠΏΡ€ΠΎΠΈΠ·Π²ΠΎΠ»ΡŒΠ½ΠΎ. Π§Ρ‚ΠΎΠ±Ρ‹ ΡΡƒΠ΄ΠΈΡ‚ΡŒ ΠΏΠΎ Π½Π°ΠΏΡ€Π°Π²Π»Π΅Π½ΠΈΡŽ Ρ‚ΠΎΠΊΠ° Ρ‡Π΅Ρ€Π΅Π· ΠΊΠ°ΠΆΠ΄Ρ‹ΠΉ ΠΈΠ· рСзисторов, слСдуСт Π½Π° ΠΊΠ°ΠΆΠ΄ΠΎΠΌ Π·Π°Π΄Π°Ρ‚ΡŒ Π½Π°ΠΏΡ€Π°Π²Π»Π΅Π½ΠΈΠ΅. Π­Ρ‚ΠΎ Ρ‚Π°ΠΊΠΆΠ΅ ΠΌΠΎΠΆΠ½ΠΎ ΡΠ΄Π΅Π»Π°Ρ‚ΡŒ ΠΏΡ€ΠΎΠΈΠ·Π²ΠΎΠ»ΡŒΠ½ΠΎ, ΠΎΠ΄Π½Π°ΠΊΠΎ ΠΏΠΎΠ»ΠΎΠΆΠΈΡ‚Π΅Π»ΡŒΠ½Ρ‹ΠΌ Π½Π°ΠΏΡ€Π°Π²Π»Π΅Π½ΠΈΠ΅ΠΌ Ρ‚ΠΎΠΊΠ° Π±ΡƒΠ΄Π΅Ρ‚ Π½Π°ΠΏΡ€Π°Π²Π»Π΅Π½ΠΈΠ΅ ΡƒΠ·Π»Π° с мСньшим Π½ΠΎΠΌΠ΅Ρ€ΠΎΠΌ ΠΊ ΡƒΠ·Π»Ρƒ с большим. ΠžΠ±ΠΎΠ·Π½Π°Ρ‡ΠΈΠΌ ΠΏΠΎΡ‚Π΅Π½Ρ†ΠΈΠ°Π»Ρ‹ Π² ΡƒΠ·Π»Π°Ρ… Π±ΡƒΠΊΠ²ΠΎΠΉ U с ΡΠΎΠΎΡ‚Π²Π΅Ρ‚ΡΡ‚Π²ΡƒΡŽΡ‰ΠΈΠΌ индСксом. Π Π΅Π·ΡƒΠ»ΡŒΡ‚Π°Ρ‚ всСх этих ΠΏΡ€ΠΈΠ³ΠΎΡ‚ΠΎΠ²Π»Π΅Π½ΠΈΠΉ прСдставлСн Π½Π° рисункС 50.6. Β«Π Π°Π·ΠΌΠ΅Ρ‚ΠΊΠ° схСмы».

ΠŸΡ€ΠΎΠΏΡƒΡΡ‚ΠΈΠΌ элСктричСский Ρ‚ΠΎΠΊ Ρ‡Π΅Ρ€Π΅Π· ΡƒΠ·Π»Ρ‹ с Π½ΠΎΠΌΠ΅Ρ€Π°ΠΌΠΈ 1 ΠΈ 2.Из Π·Π°ΠΊΠΎΠ½Π° сохранСния заряда Ρ‚ΠΎΠΊ, входящий Π² ΡƒΠ·Π΅Π» 1, Ρ€Π°Π²Π΅Π½ Ρ‚ΠΎΠΊΡƒ, выходящСму ΠΈΠ· ΡƒΠ·Π»Π° 2. Если Π²Π·ΡΡ‚ΡŒ силу Ρ‚ΠΎΠΊΠ°, Ρ€Π°Π²Π½ΡƒΡŽ Π΅Π΄ΠΈΠ½ΠΈΡ†Ρƒ, Π² силу Π·Π°ΠΊΠΎΠ½Π° Ома Ρ€Π°Π·Π½ΠΎΡΡ‚ΡŒ ΠΏΠΎΡ‚Π΅Π½Ρ†ΠΈΠ°Π»ΠΎΠ² U 2 — U 1 Π±ΡƒΠ΄Π΅Ρ‚ Ρ€Π°Π²Π½Π° Π² точности искомому ΡΠΎΠΏΡ€ΠΎΡ‚ΠΈΠ²Π»Π΅Π½ΠΈΡŽ. Π˜ΠΌΠ΅ΡŽΡ‚ значСния лишь разности ΠΏΠΎΡ‚Π΅Π½Ρ†ΠΈΠ°Π»ΠΎΠ², ΠΌΡ‹ ΠΌΠΎΠΆΠ΅ΠΌ смСло ΠΏΠΎΠ»ΠΎΠΆΠΈΡ‚ΡŒ U 1 = 0, ΠΈ Ρ‚ΠΎΠ³Π΄Π° U 2 ΠΌΡ‹ ΠΈΡΠΏΠΎΠ»ΡŒΠ·ΡƒΠ΅ΠΌ искомым сопротивлСниСм схСмы.

ΠžΠ±ΠΎΠ·Π½Π°Ρ‡ΠΈΠ² ΠΊΠ°ΠΊ I Ξ± Ρ‚ΠΎΠΊ Ρ‡Π΅Ρ€Π΅Π· рСзистор R Ξ±, для ΠΊΠ°ΠΆΠ΄ΠΎΠ³ΠΎ ΠΈΠ· рСзисторов запишСм Π·Π°ΠΊΠΎΠ½ Ома: R 1 ⁒ I 1 = U 3 — U 1, R 2 ⁒ I 2 = U 4 — U 1, R 3 ⁒ I 3 = U 4 — U 3, R 4 ⁒ I 4 = U 3 — U 2, R 5 ⁒ I 5 = U 4 — U 2.

Вторая Π³Ρ€ΡƒΠΏΠΏΠ° получаСтся ΠΈΠ· Π·Π°ΠΊΠΎΠ½Π° сохранСния заряда. Для ΠΊΠ°ΠΆΠ΄ΠΎΠ³ΠΎ ΡƒΠ·Π»Π° входящСго Π² Π½Π΅Π³ΠΎ Ρ‚ΠΎΠΊΠΎΠ² ΠΏΡ€ΠΈΡ€Π°Π²Π½ΠΈΠ²Π°Π΅ΠΌ суммС выходящих. ΠŸΡ€ΠΈ этом Π½Π΅ Π·Π°Π±Ρ‹Π²Π°Π΅ΠΌ ΠΏΡ€ΠΎ Π΅Π΄ΠΈΠ½ΠΈΡ‡Π½Ρ‹ΠΉ Ρ‚ΠΎΠΊ, входящий Π² ΠΏΠ΅Ρ€Π²Ρ‹ΠΉ ΡƒΠ·Π΅Π» ΠΈ выходящий ΠΈΠ· Π²Ρ‚ΠΎΡ€ΠΎΠ³ΠΎ: 1 = I 1 + I 2, 0 = 1 + I 4 + I 5, I 1 + I 4 = I 3, I 2 + I 3 + Π― 5 = 0.

Π”ΠΎΠ±Π°Π²ΠΈΠ² ΠΊ составлСнным уравнСниям Π΅Ρ‰Ρ‘ ΠΎΠ΄Π½ΠΎ, U 1 = 0, Ρ€Π΅ΡˆΠ°Π΅ΠΌ ΠΏΠΎΠ»ΡƒΡ‡Π΅Π½Π½ΡƒΡŽ систСму ΠΎΡ‚Π½ΠΎΡΠΈΡ‚Π΅Π»ΡŒΠ½ΠΎ U 2.

ΠœΠ΅ΠΆΠ΄Ρƒ ΠΏΡ€ΠΎΡ‡ΠΈΠΌ, примСняя ΠΎΠΏΠΈΡΠ°Π½Π½ΡƒΡŽ ΠΌΠ΅Ρ‚ΠΎΠ΄ΠΈΠΊΡƒ ΠΊ ΠΏΠΎΡΠ»Π΅Π΄ΠΎΠ²Π°Ρ‚Π΅Π»ΡŒΠ½ΠΎΠΌΡƒ ΠΈ ΠΏΠ°Ρ€Π°Π»Π»Π΅Π»ΡŒΠ½ΠΎΠΌΡƒ соСдинСниюм рСзисторов, ΠΌΡ‹ с ΡƒΠ΄ΠΎΠ²ΠΎΠ»ΡŒΡΡ‚Π²ΠΈΠ΅ΠΌ согласовались Π² ΠΏΡ€Π°Π²ΠΈΠ»ΡŒΠ½ΠΎΡΡ‚ΠΈ Ρ„ΠΎΡ€ΠΌΡƒΠ» слоТСния сопротивлСний ΠΈ проводимостСй.

ΠŸΠΎΡ€Π° Π·Π°ΠΌΠ΅Ρ‚ΠΈΡ‚ΡŒ, Ρ‡Ρ‚ΠΎ всС ΡƒΠΊΠ°Π·Π°Π½Π½Ρ‹Π΅ уравнСния ΠΏΠΎ ΠΎΡ‚Π½ΠΎΡˆΠ΅Π½ΠΈΡŽ ΠΊΠΎ всСм нСизвСстным Π²Π΅Π»ΠΈΡ‡ΠΈΠ½Π°ΠΌ I Ξ± ΠΈ U Ξ². ΠœΡ‹ Π½Π΅ станСм Π·Π°Π΄Π°Π²Π°Ρ‚ΡŒΡΡ вопросом ΠΎ СдинствСнности Ρ€Π΅ΡˆΠ΅Π½ΠΈΡ Ρ‚Π°ΠΊΠΎΠΉ систСмы. ΠžΡ‚ΠΌΠ΅Ρ‚ΠΈΠΌ лишь, Ρ‡Ρ‚ΠΎ сущСствуСт СдинствСнноС Π·Π½Π°Ρ‡Π΅Π½ΠΈΠ΅ U 2, ΡƒΠ΄ΠΎΠ²Π»Π΅Ρ‚Π²ΠΎΡ€ΡΡŽΡ‰Π΅Π΅ систСмС. Об этом Π³ΠΎΠ²ΠΎΡ€ΠΈΡ‚ физичСский смысл.

Π—Π°Π΄Π°Ρ‡Π° расчёта элСктричСского сопротивлСния являСтся довольно Π°ΠΊΡ‚ΡƒΠ°Π»ΡŒΠ½ΠΎΠΉ. Π˜ΠΌΠ΅Π΅Ρ‚ΡΡ ряд ΠΏΡ€ΠΈΡ‘ΠΌΠΎΠ², ΠΊΠΎΡ‚ΠΎΡ€Ρ‹Π΅ ΠΏΠΎΠ·Π²ΠΎΠ»ΡΡŽΡ‚ ΡƒΠΏΡ€ΠΎΡΡ‚ΠΈΡ‚ΡŒ Π΅Ρ‘ Ρ€Π΅ΡˆΠ΅Π½ΠΈΠ΅.К ΠΏΡ€ΠΈΠΌΠ΅Ρ€Ρƒ, ΠΏΡ€Π°Π²ΠΈΠ»Π° ΠšΠΈΡ€Ρ…Π³ΠΎΡ„Π° ΠΏΠΎΠ·Π²ΠΎΠ»ΡΡŽΡ‚ ΡΡ‚Ρ€ΠΎΠΈΡ‚ΡŒ систСмы, Ρ€Π°Π²Π½ΠΎΡΠΈΠ»ΡŒΠ½Ρ‹Π΅ Ρ‚ΠΎΠ»ΡŒΠΊΠΎ Ρ‡Ρ‚ΠΎ ΠΏΠΎΠ»ΡƒΡ‡Π΅Π½Π½Ρ‹ΠΌ, ΠΈ ΠΏΡ€ΠΈ этом, ΠΊΠ°ΠΊ ΠΏΡ€Π°Π²ΠΈΠ»ΠΎ, Π±ΠΎΠ»Π΅Π΅ простыС. Π•ΡΡ‚ΡŒ ΠΌΠ΅Ρ‚ΠΎΠ΄Ρ‹, Π² основС Π»Π΅ΠΆΠ°Ρ‰ΠΈΡ… схСм Π² эквивалСнтныС (Ρ‚ΠΎ Π΅ΡΡ‚ΡŒ ΠΈΠΌΠ΅ΡŽΡ‰Π΅Π΅ Ρ‚ΠΎ ΠΆΠ΅ сопротивлСниС), Π½ΠΎ ΠΏΡ€ΠΈ этом Ρ€Π°Π·Π»ΠΎΠΆΠΈΠΌΡ‹Π΅ Π² ΠΏΠΎΡΠ»Π΅Π΄ΠΎΠ²Π°Ρ‚Π΅Π»ΡŒΠ½ΠΎΡΡ‚ΠΈ ΠΈΠ»ΠΈ ΠΏΠ°Ρ€Π°Π»Π»Π΅Π»ΡŒΠ½ΠΎΠ΅ соСдинСниС Π΄Π²ΡƒΡ… подсхСм. ΠœΡ‹ Π½Π΅ Π±ΡƒΠ΄Π΅ΠΌ ΠΎΡΡ‚Π°Π½Π°Π²Π»ΠΈΠ²Π°Ρ‚ΡŒΡΡ Π½Π° этих ΠΌΠ΅Ρ‚ΠΎΠ΄Π°Ρ…. Π’ Π³Π»Π°Π²Π΅ 49. Β« Π›ΠΈΠ½Π΅ΠΉΠ½Ρ‹Π΅ уравнСния Β» рассматривало алгоритмичСскоС Ρ€Π΅ΡˆΠ΅Π½ΠΈΠ΅ систСмных Π»ΠΈΠ½Π΅ΠΉΠ½Ρ‹Ρ… ΡƒΡ€Π°Π²Π½Π΅Π½ΠΈΠΉ, ΠΈ Π½Π°ΠΌ остаётся лишь ΠΈΡΠΏΠΎΠ»ΡŒΠ·ΠΎΠ²Π°Π½Π½Ρ‹ΠΌ ΡƒΠΆΠ΅ написанным Π±ΠΈΠ±Π»ΠΈΠΎΡ‚Π΅Ρ‡Π½Ρ‹ΠΌ ΠΌΠΎΠ΄ΡƒΠ»Π΅ΠΌ.

ΠŸΠΎΡΠ»Π΅Π΄ΠΎΠ²Π°Ρ‚Π΅Π»ΡŒΠ½ΠΎΠ΅, ΠΏΠ°Ρ€Π°Π»Π»Π΅Π»ΡŒΠ½ΠΎΠ΅ ΠΈ смСшанноС соСдинСниС рСзисторов. Π—Π½Π°Ρ‡ΠΈΡ‚Π΅Π»ΡŒΠ½ΠΎΠ΅ число ΠΏΡ€ΠΈΠ΅ΠΌΠ½ΠΈΠΊΠΎΠ², Π²ΠΊΠ»ΡŽΡ‡Π΅Π½Π½Ρ‹Ρ… Π² ΡΠ»Π΅ΠΊΡ‚Ρ€ΠΈΡ‡Π΅ΡΠΊΡƒΡŽ Ρ†Π΅ΠΏΡŒ (элСктричСскиС Π»Π°ΠΌΠΏΡ‹, ΡΠ»Π΅ΠΊΡ‚Ρ€ΠΎΠ½Π°Π³Ρ€Π΅Π²Π°Ρ‚Π΅Π»ΡŒΠ½Ρ‹Π΅ ΠΏΡ€ΠΈΠ±ΠΎΡ€Ρ‹ ΠΈ Π΄Ρ€.), МоТно Ρ€Π°ΡΡΠΌΠ°Ρ‚Ρ€ΠΈΠ²Π°Ρ‚ΡŒ ΠΊΠ°ΠΊ Π½Π΅ΠΊΠΎΡ‚ΠΎΡ€Ρ‹Π΅ элСмСнты, ΠΈΠΌΠ΅ΡŽΡ‰ΠΈΠ΅ ΠΎΠΏΡ€Π΅Π΄Π΅Π»Π΅Π½Π½ΠΎΠ΅ сопротивлСниС. Π­Ρ‚ΠΎ ΠΎΠ±ΡΡ‚ΠΎΡΡ‚Π΅Π»ΡŒΡΡ‚Π²ΠΎ Π΄Π°Π΅Ρ‚ Π½Π°ΠΌ Π²ΠΎΠ·ΠΌΠΎΠΆΠ½ΠΎΡΡ‚ΡŒ ΠΏΡ€ΠΈ составлСнии ΠΈ ΠΈΠ·ΡƒΡ‡Π΅Π½ΠΈΠΈ элСктричСских схСм Π·Π°ΠΌΠ΅Π½ΡΡ‚ΡŒ ΠΏΡ€ΠΈΠ΅ΠΌΠ½ΠΈΠΊΠΈ рСзисторами с ΠΎΠΏΡ€Π΅Π΄Π΅Π»Π΅Π½Π½Ρ‹ΠΌΠΈ сопротивлСниями. Π Π°Π·Π»ΠΈΡ‡Π°ΡŽΡ‚ ΡΠ»Π΅Π΄ΡƒΡŽΡ‰ΠΈΠ΅ способы рСзисторов (ΠΏΡ€ΠΈΠ΅ΠΌΠ½ΠΈΠΊΠΎΠ² элСктричСской энСргии): ΠΏΠΎΡΠ»Π΅Π΄ΠΎΠ²Π°Ρ‚Π΅Π»ΡŒΠ½ΠΎΠ΅, ΠΏΠ°Ρ€Π°Π»Π»Π΅Π»ΡŒΠ½ΠΎΠ΅ ΠΈ смСшанноС.

Рис. 25. Π‘Ρ…Π΅ΠΌΡ‹ ΠΏΠΎΡΠ»Π΅Π΄ΠΎΠ²Π°Ρ‚Π΅Π»ΡŒΠ½ΠΎΠ³ΠΎ соСдинСния ΠΏΡ€ΠΈΠ΅ΠΌΠ½ΠΈΠΊΠΎΠ²

ΠŸΠΎΡΠ»Π΅Π΄ΠΎΠ²Π°Ρ‚Π΅Π»ΡŒΠ½ΠΎΠ΅ соСдинСниС рСзисторов . ΠŸΡ€ΠΈ ΠΏΠΎΡΠ»Π΅Π΄ΠΎΠ²Π°Ρ‚Π΅Π»ΡŒΠ½ΠΎΠΌ соСдинСнии Π½Π΅ΡΠΊΠΎΠ»ΡŒΠΊΠΈΡ… рСзисторов ΠΊΠΎΠ½Π΅Ρ† ΠΏΠ΅Ρ€Π²ΠΎΠ³ΠΎ рСзистора ΡΠΎΠ΅Π΄ΠΈΠ½ΡΡŽΡ‚ с Π½Π°Ρ‡Π°Π»ΠΎΠΌ Π²Ρ‚ΠΎΡ€ΠΎΠ³ΠΎ, ΠΊΠΎΠ½Π΅Ρ† Π²Ρ‚ΠΎΡ€ΠΎΠ³ΠΎ — с Π½Π°Ρ‡Π°Π»ΠΎΠΌ Ρ‚Ρ€Π΅Ρ‚ΡŒΠ΅Π³ΠΎ ΠΈ Ρ‚. Π΄. ΠŸΡ€ΠΈ Ρ‚Π°ΠΊΠΎΠΌ соСдинСнии ΠΏΠΎ всСм элСмСнтам ΠΏΠΎΡΠ»Π΅Π΄ΠΎΠ²Π°Ρ‚Π΅Π»ΡŒΠ½ΠΎΠΉ Ρ†Π΅ΠΏΠΈ ΠΏΡ€ΠΎΡ…ΠΎΠ΄ΠΈΡ‚
ΠΎΠ΄ΠΈΠ½ ΠΈ Ρ‚ΠΎΡ‚ ΠΆΠ΅ Ρ‚ΠΎΠΊ I.
ΠŸΠΎΡΠ»Π΅Π΄ΠΎΠ²Π°Ρ‚Π΅Π»ΡŒΠ½ΠΎΠ΅ соСдинСниС ΠΏΡ€ΠΈΠ΅ΠΌΠ½ΠΈΠΊΠΎΠ² поясняСт рис. 25, Π°.
.ЗамСняя Π»Π°ΠΌΠΏΡ‹ рСзисторами с сопротивлСниями R1, R2 ΠΈ R3, ΠΏΠΎΠ»ΡƒΡ‡ΠΈΠΌ схСму, ΠΏΠΎΠΊΠ°Π·Π°Π½Π½ΡƒΡŽ Π½Π° рис. 25, Π±.
Если ΠΏΡ€ΠΈΠ½ΡΡ‚ΡŒ, Ρ‡Ρ‚ΠΎ Π² источникС Ro = 0, Ρ‚ΠΎ для Ρ‚Ρ€Π΅Ρ… соСдинСнных рСзисторов согласно Π²Ρ‚ΠΎΡ€ΠΎΠΌΡƒ соСдинСнию ΠšΠΈΡ€Ρ…Π³ΠΎΡ„Π° ΠΌΠΎΠΆΠ½ΠΎ Π½Π°ΠΏΠΈΡΠ°Ρ‚ΡŒ:

гдС R эк = R 1 + R 2 + R 3 .
Π‘Π»Π΅Π΄ΠΎΠ²Π°Ρ‚Π΅Π»ΡŒΠ½ΠΎ, эквивалСнтноС сопротивлСниС ΠΏΠΎΡΠ»Π΅Π΄ΠΎΠ²Π°Ρ‚Π΅Π»ΡŒΠ½ΠΎΠΉ Ρ†Π΅ΠΏΠΈ Ρ€Π°Π²Π½ΠΎ суммС сопротивлСний всСх ΠΏΠΎΡΠ»Π΅Π΄ΠΎΠ²Π°Ρ‚Π΅Π»ΡŒΠ½ΠΎ соСдинСнных рСзисторов.Π’Π°ΠΊ напряТСния Π½Π° ΠΎΡ‚Π΄Π΅Π»ΡŒΠ½Ρ‹Ρ… участках Ρ†Π΅ΠΏΠΈ согласно Π·Π°ΠΊΠΎΠ½Ρƒ Ома: U 1 = IR 1 ; U 2 = IR 2 , U 3 = IR Π· ΠΈ Π² Π΄Π°Π½Π½ΠΎΠΌ случаС E = U, Ρ‚ΠΎ Π΅ΡΡ‚ΡŒ длярассматриваСмой Ρ†Π΅ΠΏΠΈ

U = U 1 + U 2 + U 3 (20)

, ΡΠ»Π΅Π΄ΠΎΠ²Π°Ρ‚Π΅Π»ΡŒΠ½ΠΎ, напряТСниС U Π½Π° Π·Π°ΠΆΠΈΠΌΠ°Ρ… Ρ€Π°Π²Π½ΠΎ суммС напряТСний Π² ΠΊΠ°ΠΆΠ΄ΠΎΠΌ ΠΈΠ· ΠΏΠΎΡΠ»Π΅Π΄ΠΎΠ²Π°Ρ‚Π΅Π»ΡŒΠ½ΠΎ Π²ΠΊΠ»ΡŽΡ‡Π΅Π½Π½Ρ‹Ρ… рСзисторов.
Из ΡƒΠΊΠ°Π·Π°Π½Π½ΠΎΠΉ Ρ„ΠΎΡ€ΠΌΡƒΠ»Ρ‹ слСдуСт, Ρ‡Ρ‚ΠΎ Ρ€Π°ΡΠΏΡ€Π΅Π΄Π΅Π»ΡΡŽΡ‚ΡΡ ΠΌΠ΅ΠΆΠ΄Ρƒ ΠΏΠΎΡΠ»Π΅Π΄ΠΎΠ²Π°Ρ‚Π΅Π»ΡŒΠ½ΠΎ соСдинСнными рСзисторами ΠΏΡ€ΠΎΠΏΠΎΡ€Ρ†ΠΈΠΎΠ½Π°Π»ΡŒΠ½ΠΎ ΠΈΡ… сопротивлСниСм:

Ρ‚. Π΅. Ρ‡Π΅ΠΌ большС сопротивлСния-Π»ΠΈΠ±ΠΎ ΠΏΡ€ΠΈΠ΅ΠΌΠ½ΠΈΠΊΠ° Π² ΠΏΠΎΡΠ»Π΅Π΄ΠΎΠ²Π°Ρ‚Π΅Π»ΡŒΠ½ΠΎΠΉ Ρ†Π΅ΠΏΠΈ, Ρ‚Π΅ΠΌ большС ΠΏΡ€ΠΈΠ»ΠΎΠΆΠ΅Π½Π½ΠΎΠ΅ ΠΊ Π½Π΅ΠΌΡƒ напряТСниС.

Π’ случаС Ссли соСдиняСтся нСсколько, Π½Π°ΠΏΡ€ΠΈΠΌΠ΅Ρ€, рСзисторов с ΠΎΠ΄ΠΈΠ½Π°ΠΊΠΎΠ²Ρ‹ΠΌ сопротивлСниСм R1, эквивалСнтноС сопротивлСниС Ρ†Π΅ΠΏΠΈ Rэк Π±ΡƒΠ΄Π΅Ρ‚ Π² ΠΏ Ρ€Π°Π· большС сопротивлСния R1, Ρ‚. Π΅. Rэк = nR1.НапряТСниС U1 Π½Π° ΠΊΠ°ΠΆΠ΄ΠΎΠΌ рСзисторС Π² этом случаС Π² случаС мСньшС ΠΎΠ±Ρ‰Π΅Π³ΠΎ напряТСния U:

ΠŸΡ€ΠΈ ΠΏΠΎΡΠ»Π΅Π΄ΠΎΠ²Π°Ρ‚Π΅Π»ΡŒΠ½ΠΎΠΌ соСдинСнии ΠΏΡ€ΠΈΠ΅ΠΌΠ½ΠΈΠΊΠΎΠ² измСнСния сопротивлСния ΠΎΠ΄Π½ΠΎΠ³ΠΎ ΠΈΠ· Π½ΠΈΡ… тотчас ΠΆΠ΅ Π²Π»Π΅Ρ‡Π΅Ρ‚ Π·Π° собой ΠΈΠ·ΠΌΠ΅Π½Π΅Π½ΠΈΠ΅ напряТСния Π½Π° Π΄Ρ€ΡƒΠ³ΠΈΡ… связанных с Π½ΠΈΠΌ ΠΏΡ€ΠΈΠ΅ΠΌΠ½ΠΈΠΊΠ°Ρ…. ΠŸΡ€ΠΈ Π²Ρ‹ΠΊΠ»ΡŽΡ‡Π΅Π½ΠΈΠΈ ΠΈΠ»ΠΈ ΠΎΠ±Ρ€Ρ‹Π²Π΅ элСктричСской Ρ†Π΅ΠΏΠΈ Π² ΠΎΠ΄Π½ΠΎΠΌ ΠΈΠ· ΠΏΡ€ΠΈΠ΅ΠΌΠ½ΠΈΠΊΠΎΠ² ΠΈ ΠΎΡΡ‚Π°Π»ΡŒΠ½Ρ‹Ρ… ΠΏΡ€ΠΈΠ΅ΠΌΠ½ΠΈΠΊΠ°Ρ… прСкращаСтся Ρ‚ΠΎΠΊ. ΠŸΠΎΡΡ‚ΠΎΠΌΡƒ ΠΏΠΎΡΠ»Π΅Π΄ΠΎΠ²Π°Ρ‚Π΅Π»ΡŒΠ½ΠΎΠ΅ соСдинСниС ΠΏΡ€ΠΈΠ΅ΠΌΠ½ΠΈΠΊΠΎΠ² примСняСтся Ρ€Π΅Π΄ΠΊΠΎ — Ρ‚ΠΎΠ»ΡŒΠΊΠΎ Π² Ρ‚ΠΎΠΌ случаС, ΠΊΠΎΠ³Π΄Π° напряТСниС источника энСргии большС номинального напряТСния, Π½Π° ΠΊΠΎΡ‚ΠΎΡ€ΠΎΠ΅ рассчитан ΠΏΠΎΡ‚Ρ€Π΅Π±ΠΈΡ‚Π΅Π»ΡŒ. НапримСр, напряТСниС Π² элСктричСской сСти, ΠΎΡ‚ ΠΊΠΎΡ‚ΠΎΡ€ΠΎΠΉ ΠΏΠΈΡ‚Π°ΡŽΡ‚ΡΡ Π²Π°Π³ΠΎΠ½Ρ‹ ΠΌΠ΅Ρ‚Ρ€ΠΎΠΏΠΎΠ»ΠΈΡ‚Π΅Π½Π°, составляСт 825 Π’, номинальноС ΠΆΠ΅ напряТСниС элСктричСских Π»Π°ΠΌΠΏ, примСняСмых Π² этих Π²Π°Π³ΠΎΠ½Π°Ρ…, 55 Π’. ΠŸΠΎΡΡ‚ΠΎΠΌΡƒ Π² Π²Π°Π³ΠΎΠ½Π°Ρ… ΠΌΠ΅Ρ‚Ρ€ΠΎΠΏΠΎΠ»ΠΈΡ‚Π΅Π½Π° элСктричСскиС Π»Π°ΠΌΠΏΡ‹ Π²ΠΊΠ»ΡŽΡ‡Π°ΡŽΡ‚ ΠΏΠΎΡΠ»Π΅Π΄ΠΎΠ²Π°Ρ‚Π΅Π»ΡŒΠ½ΠΎΡΡ‚ΡŒ ΠΏΠΎ 15 Π»Π°ΠΌΠΏ Π² ΠΊΠ°ΠΆΠ΄ΠΎΠΉ Ρ†Π΅ΠΏΠΈ.
ΠŸΠ°Ρ€Π°Π»Π»Π΅Π»ΡŒΠ½ΠΎΠ΅ соСдинСниС рСзисторов . ΠŸΡ€ΠΈ ΠΏΠ°Ρ€Π°Π»Π»Π΅Π»ΡŒΠ½ΠΎΠΌ соСдинСнии Π½Π΅ΡΠΊΠΎΠ»ΡŒΠΊΠΈΡ… ΠΏΡ€ΠΈΠ΅ΠΌΠ½ΠΈΠΊΠΎΠ² ΠΎΠ½ΠΈ Π²ΠΊΠ»ΡŽΡ‡Π°ΡŽΡ‚ΡΡ ΠΌΠ΅ΠΆΠ΄Ρƒ двумя Ρ‚ΠΎΡ‡ΠΊΠ°ΠΌΠΈ элСктричСской Ρ†Π΅ΠΏΠΈ, образуя ΠΏΠ°Ρ€Π°Π»Π»Π΅Π»ΡŒΠ½Ρ‹Π΅ Π²Π΅Ρ‚Π²ΠΈ (рис. 26, Π°).ЗамСняя

Рис. 26. Π‘Ρ…Π΅ΠΌΡ‹ ΠΏΠ°Ρ€Π°Π»Π»Π΅Π»ΡŒΠ½ΠΎΠ³ΠΎ соСдинСния ΠΏΡ€ΠΈΠ΅ΠΌΠ½ΠΈΠΊΠΎΠ²

Π»Π°ΠΌΠΏΡ‹ рСзисторами с сопротивлСниями R1, R2, R3, ΠΏΠΎΠ»ΡƒΡ‡ΠΈΠΌ схСму, ΠΏΠΎΠΊΠ°Π·Π°Π½Π½ΡƒΡŽ Π½Π° рис. 26, Π±.
ΠŸΡ€ΠΈ ΠΏΠ°Ρ€Π°Π»Π»Π΅Π»ΡŒΠ½ΠΎΠΌ соСдинСнии ΠΊΠΎ всСм рСзисторам ΠΏΡ€ΠΈΠ»ΠΎΠΆΠ΅Π½ΠΎ ΠΎΠ΄ΠΈΠ½Π°ΠΊΠΎΠ²ΠΎΠ΅ напряТСниС U. ΠŸΠΎΡΡ‚ΠΎΠΌΡƒ согласно Ома:

Π’ΠΎΠΊ Π² Π½Π΅Ρ€Π°Π·Π²Π΅Ρ‚Π²Π»Π΅Π½Π½ΠΎΠΉ части Ρ†Π΅ΠΏΠΈ согласно ΠΏΠ΅Ρ€Π²ΠΎΠΌΡƒ Π·Π°ΠΊΠΎΠ½Ρƒ ΠšΠΈΡ€Ρ…Π³ΠΎΡ„Π° I = I 1 + I 2 + I 3 , ΠΈΠ»ΠΈ

эквивалСнтноС сопротивлСниС рассматриваСмой Ρ†Π΅ΠΏΠΈ ΠΏΡ€ΠΈ ΠΏΠ°Ρ€Π°Π»Π»Π΅Π»ΡŒΠ½ΠΎΠΌ соСдинСнии Ρ‚Ρ€Π΅Ρ… рСзисторов ΠΎΠΏΡ€Π΅Π΄Π΅Π»Π»ΠΎΠΉ

1/ R эк = 1 / R 1 + 1 / R 2 + 1 / R 3 (24)

Вводя Π² Ρ„ΠΎΡ€ΠΌΡƒΠ»Ρƒ (24) вмСсто Π·Π½Π°Ρ‡Π΅Π½ΠΈΠΉ 1 / R эк , 1 / R 1 , 1 / R 2 ΠΈ 1 / R 3 Π‘ΠΎΠΎΡ‚Π²Π΅Ρ‚ΡΡ‚Π²ΡƒΡŽΡ‰Π°Ρ ΠΏΡ€ΠΎΠ²ΠΎΠ΄ΠΈΠΌΠΎΡΡ‚ΡŒ G эк , G 1 , G 2 ΠΈ G 3 , ΠΏΠΎΠ»ΡƒΡ‡ΠΈΠΌ: эквивалСнтная ΠΏΡ€ΠΎΠ²ΠΎΠ΄ΠΈΠΌΠΎΡΡ‚ΡŒ ΠΏΠ°Ρ€Π°Π»Π»Π΅Π»ΡŒΠ½ΠΎΠΉ Ρ†Π΅ΠΏΠΈ Ρ€Π°Π²Π½Π° суммС проводимостСй ΠΏΠ°Ρ€Π°Π»Π»Π΅Π»ΡŒΠ½ΠΎ соСдинСнных рСзисторов :

Π’Π°ΠΊΠΈΠΌ ΠΎΠ±Ρ€Π°Π·ΠΎΠΌ, ΠΏΡ€ΠΈ ΡƒΠ²Π΅Π»ΠΈΡ‡Π΅Π½ΠΈΠΈ ΠΏΠΎΠΏΠ΅Ρ€Π΅Ρ‡Π½ΠΎΠ³ΠΎ сопротивлСния ΡƒΠ²Π΅Π»ΠΈΡ‡ΠΈΠ²Π°ΡŽΡ‰Π΅ΠΉΡΡ проводимости элСктричСской Ρ†Π΅ΠΏΠΈ увСличиваСтся Ρ€Π΅Π·ΡƒΠ»ΡŒΡ‚ΠΈΡ€ΡƒΡŽΡ‰Π΅Π΅ сопротивлСниС элСктричСской Ρ†Π΅ΠΏΠΈ.
Из ΠΏΡ€ΠΈΠ²Π΅Π΄Π΅Π½Π½Ρ‹Ρ… Ρ„ΠΎΡ€ΠΌΡƒΠ» слСдуСт, Ρ‡Ρ‚ΠΎ Ρ‚ΠΎΠΊΠΈ Ρ€Π°ΡΠΏΡ€Π΅Π΄Π΅Π»ΡΡŽΡ‚ΡΡ ΠΌΠ΅ΠΆΠ΄Ρƒ ΠΏΠ°Ρ€Π°Π»Π»Π΅Π»ΡŒΠ½Ρ‹ΠΌΠΈ вСтвями ΠΎΠ±Ρ€Π°Ρ‚Π½ΠΎ ΠΏΡ€ΠΎΠΏΠΎΡ€Ρ†ΠΈΠΎΠ½Π°Π»ΡŒΠ½ΠΎ ΠΈΡ… элСктричСскоС сопротивлСниС ΠΈΠ»ΠΈ прямо ΠΈΡ… проводимостям. НапримСр, ΠΏΡ€ΠΈ Ρ‚Ρ€Π΅Ρ… вСтвях

Π’ этом ΠΎΡ‚Π½ΠΎΡˆΠ΅Π½ΠΈΠΈ ΠΈΠΌΠ΅Π΅Ρ‚ мСсто ΠΏΠΎΠ»Π½ΠΎΠ΅ аналогия ΠΌΠ΅ΠΆΠ΄Ρƒ распрСдСлСниСм Ρ‚ΠΎΠΊΠΎΠ² ΠΏΠΎ Ρ€Π°Π·Π΄Π΅Π»Π°ΠΌ вСтвям ΠΈ распрСдСлСниСм Π²ΠΎΠ΄Ρ‹ ΠΏΠΎ Ρ‚Ρ€ΡƒΠ±Π°ΠΌ.
ΠŸΡ€ΠΈΠ²Π΅Π΄Π΅Π½Π½Ρ‹Π΅ Ρ„ΠΎΡ€ΠΌΡƒΠ»Ρ‹ ΠΏΠΎΠ·Π²ΠΎΠ»ΡΡŽΡ‚ ΠΎΠΏΡ€Π΅Π΄Π΅Π»ΠΈΡ‚ΡŒ эквивалСнтноС сопротивлСниС Ρ†Π΅ΠΏΠΈ для Ρ€Π°Π·Π»ΠΈΡ‡Π½Ρ‹Ρ… случаСв. НапримСр, ΠΏΡ€ΠΈ Π΄Π²ΡƒΡ… ΠΏΠ°Ρ€Π°Π»Π»Π΅Π»ΡŒΠ½Ρ‹Ρ… рСзисторах Ρ€Π΅Π·ΡƒΠ»ΡŒΡ‚ΠΈΡ€ΡƒΡŽΡ‰Π΅Π΅ сопротивлСниС Ρ†Π΅ΠΏΠΈ

ΠΏΡ€ΠΈ Ρ‚Ρ€Π΅Ρ… ΠΏΠ°Ρ€Π°Π»Π»Π΅Π»ΡŒΠ½Ρ‹Ρ… рСзисторах

ΠŸΡ€ΠΈ ΠΏΠ°Ρ€Π°Π»Π»Π΅Π»ΡŒΠ½ΠΎΠΌ соСдинСнии Π½Π΅ΡΠΊΠΎΠ»ΡŒΠΊΠΈΡ…, Π½Π°ΠΏΡ€ΠΈΠΌΠ΅Ρ€, рСзисторов с ΠΎΠ΄ΠΈΠ½Π°ΠΊΠΎΠ²Ρ‹ΠΌ сопротивлСниСм R1 Ρ€Π΅Π·ΡƒΠ»ΡŒΡ‚ΠΈΡ€ΡƒΡŽΡ‰Π΅Π΅ сопротивлСниС Ρ†Π΅ΠΏΠΈ Rэк Π±ΡƒΠ΄Π΅Ρ‚ Π² n Ρ€Π°Π· мСньшС сопротивлСния R1, Ρ‚.Π΅.

R эк = R1 / n (27)

ΠŸΡ€ΠΎΡ…ΠΎΠ΄ΡΡ‰ΠΈΠΉ ΠΏΠΎ ΠΊΠ°ΠΆΠ΄ΠΎΠΉ Π²Π΅Ρ‚Π²ΠΈ Ρ‚ΠΎΠΊ I1, Π² этом случаС Π±ΡƒΠ΄Π΅Ρ‚ мСньшС ΠΎΠ±Ρ‰Π΅Π³ΠΎ Ρ‚ΠΎΠΊΠ°:

I1 = I / n (28)

ΠŸΡ€ΠΈ ΠΏΠ°Ρ€Π°Π»Π»Π΅Π»ΡŒΠ½ΠΎΠΌ соСдинСнии ΠΏΡ€ΠΈΠ΅ΠΌΠ½ΠΈΠΊΠΎΠ², всС ΠΎΠ½ΠΈ находятся ΠΏΠΎΠ΄ ΠΎΠ΄Π½ΠΈΠΌ ΠΈ Ρ‚Π΅ΠΌ ΠΆΠ΅ напряТСниСм, ΠΈ Ρ€Π΅ΠΆΠΈΠΌ Ρ€Π°Π±ΠΎΡ‚Ρ‹ ΠΊΠ°ΠΆΠ΄ΠΎΠ³ΠΎ ΠΈΠ· Π½ΠΈΡ… Π½Π΅ зависит ΠΎΡ‚ ΠΎΡΡ‚Π°Π»ΡŒΠ½Ρ‹Ρ…. Π­Ρ‚ΠΎ ΠΎΠ·Π½Π°Ρ‡Π°Π΅Ρ‚, Ρ‡Ρ‚ΠΎ Ρ‚ΠΎΠΊ, проходящий ΠΏΠΎ ΠΊΠ°ΠΊΠΎΠΌΡƒ-Π»ΠΈΠ±ΠΎ ΠΈΠ· ΠΏΡ€ΠΈΠ΅ΠΌΠ½ΠΈΠΊΠΎΠ². ΠŸΡ€ΠΈ всяком Π²Ρ‹ΠΊΠ»ΡŽΡ‡Π΅Π½ΠΈΠΈ ΠΈΠ»ΠΈ Π²Ρ‹Ρ…ΠΎΠ΄Π΅ ΠΈΠ· строя любого ΠΏΡ€ΠΈΠ΅ΠΌΠ½ΠΈΠΊΠ° ΠΎΡΡ‚Π°Π»ΡŒΠ½Ρ‹Π΅ ΠΏΡ€ΠΈΠ΅ΠΌΠ½ΠΈΠΊΠΈ ΠΎΡΡ‚Π°ΡŽΡ‚ΡΡ вклю-

Рис.27. Π‘Ρ…Π΅ΠΌΡ‹ смСшанного соСдинСния ΠΏΡ€ΠΈΠ΅ΠΌΠ½ΠΈΠΊΠΎΠ²

Ρ‡Π΅Π½Π½Ρ‹ΠΌΠΈ. ΠŸΠΎΡΡ‚ΠΎΠΌΡƒ ΠΏΠ°Ρ€Π°Π»Π»Π΅Π»ΡŒΠ½ΠΎΠ΅ соСдинСниС ΠΈΠΌΠ΅Π΅Ρ‚ самыС ΠΏΠ΅Ρ€Π΅Π΄Π½ΠΈΠ΅ Ρ‚Π΅Ρ…Π½ΠΎΠ»ΠΎΠ³ΠΈΠΈ, поставляСмыС с ΠΏΠΎΠΌΠΎΡ‰ΡŒΡŽ ΠΏΠ΅Ρ€Π΅Π΄Π½ΠΈΡ… Ρ‚Π΅Ρ…Π½ΠΎΠ»ΠΎΠ³ΠΈΠΉ. Π’ частности, элСктричСскиС Π»Π°ΠΌΠΏΡ‹ ΠΈ Π΄Π²ΠΈΠ³Π°Ρ‚Π΅Π»ΠΈ, ΠΏΡ€Π΅Π΄Π½Π°Π·Π½Π°Ρ‡Π΅Π½Π½Ρ‹Π΅ для Ρ€Π°Π±ΠΎΡ‚Ρ‹ ΠΏΡ€ΠΈ ΠΎΠΏΡ€Π΅Π΄Π΅Π»Π΅Π½Π½ΠΎΠΌ (номинальном) напряТСнии, всСгда Π²ΠΊΠ»ΡŽΡ‡Π°Π΅Ρ‚.
На элСктровозах постоянного Ρ‚ΠΎΠΊΠ° ΠΈ Π½Π΅ΠΊΠΎΡ‚ΠΎΡ€Ρ‹Ρ… Ρ‚Π΅ΠΏΠ»ΠΎΠ²ΠΎΠ·Π°Ρ… тяговыС Π΄Π²ΠΈΠ³Π°Ρ‚Π΅Π»ΠΈ Π² процСссС рСгулирования скорости двиТСния Π΄ΠΎΠ»ΠΆΠ½Ρ‹ ΠΏΠ΅Ρ€Π΅Ρ…ΠΎΠ΄ΠΈΡ‚ΡŒ ΠΏΠΎΠ΄ Ρ€Π°Π·Π»ΠΈΡ‡Π½Ρ‹Π΅ напряТСния, поэтому ΠΎΠ½ΠΈ Π² процСссС Ρ€Π°Π·Π³ΠΎΠ½Π° ΠΏΠ΅Ρ€Π΅ΠΊΠ»ΡŽΡ‡Π°ΡŽΡ‚ΡΡ с ΠΏΠΎΡΠ»Π΅Π΄ΠΎΠ²Π°Ρ‚Π΅Π»ΡŒΠ½Ρ‹ΠΌ соСдинСниСм Π½Π° ΠΏΠ°Ρ€Π°Π»Π»Π΅Π»ΡŒΠ½ΠΎΠ΅.

БмСшанноС соСдинСниС рСзисторов . Π‘ΠΌΠ΅ΡˆΠ°Π½Π½Ρ‹ΠΌ соСдинСниСм называСтся Ρ‚Π°ΠΊΠΎΠ΅ соСдинСниС, ΠΏΡ€ΠΈ ΠΊΠΎΡ‚ΠΎΡ€ΠΎΠΌ Ρ‡Π°ΡΡ‚ΡŒ рСзисторов Π²ΠΊΠ»ΡŽΡ‡Π°Π΅Ρ‚ΡΡ ΠΏΠΎΡΠ»Π΅Π΄ΠΎΠ²Π°Ρ‚Π΅Π»ΡŒΠ½ΠΎ, Π° Ρ‡Π°ΡΡ‚ΡŒ — ΠΏΠ°Ρ€Π°Π»Π»Π΅Π»ΡŒΠ½ΠΎ. НапримСр, Π² схСмС рис. 27, Ρ‡Π΅Ρ€Π΅Π· Π΄Π²Π° Π²ΠΊΠ»ΡŽΡ‡Π΅Π½Π½Ρ‹Ρ… рСзистора сопротивлСния R1 ΠΈ R2, Π²ΠΊΠ»ΡŽΡ‡Π΅Π½Π½Ρ‹ΠΉ ΠΏΠΎΡΠ»Π΅Π΄ΠΎΠ²Π°Ρ‚Π΅Π»ΡŒΠ½ΠΎ Π²ΠΊΠ»ΡŽΡ‡Π΅Π½Π½Ρ‹ΠΉ рСзистор сопротивлСниСм RΠ·, рСзистор сопротивлСниСм R4 Π²ΠΊΠ»ΡŽΡ‡Π΅Π½ Π²ΠΊΠ»ΡŽΡ‡Π΅Π½Π½ΠΎΠΉ Π³Ρ€ΡƒΠΏΠΏΠΎΠΉ рСзисторов сопротивлСния R1, R2 ΠΈ R3.
Π­ΠΊΠ²ΠΈΠ²Π°Π»Π΅Π½Ρ‚Π½ΠΎΠ΅ сопротивлСниС Ρ†Π΅ΠΏΠΈ ΠΏΡ€ΠΈ смСшанном соСдинСнии ΠΎΠ±Ρ‹Ρ‡Π½ΠΎ ΠΎΠΏΡ€Π΅Π΄Π΅Π»ΡΡŽΡ‚ ΠΌΠ΅Ρ‚ΠΎΠ΄ΠΎΠΌ прСобразования, ΠΏΡ€ΠΈ ΠΊΠΎΡ‚ΠΎΡ€ΠΎΠΌ ΡΠ»ΠΎΠΆΠ½ΡƒΡŽ Ρ†Π΅ΠΏΡŒ ΠΏΠΎΡΠ»Π΅Π΄ΠΎΠ²Π°Ρ‚Π΅Π»ΡŒΠ½Ρ‹ΠΌΠΈ этапами ΠΏΡ€Π΅ΠΎΠ±Ρ€Π°Π·ΠΎΠ²Ρ‹Π²Π°ΡŽΡ‚ Π² ΠΏΡ€ΠΎΡΡ‚Π΅ΠΉΡˆΡƒΡŽ.НапримСр, для схСмы рис. 27, Π° Π²Π½Π°Ρ‡Π°Π»Π΅ ΠΎΠΏΡ€Π΅Π΄Π΅Π»ΡΡŽΡ‚ эквивалСнтноС сопротивлСниС R12 ΠΏΠΎΡΠ»Π΅Π΄ΠΎΠ²Π°Ρ‚Π΅Π»ΡŒΠ½ΠΎ Π²ΠΊΠ»ΡŽΡ‡Π΅Π½Π½Ρ‹Ρ… рСзисторов с сопротивлСниями R1 ΠΈ R2: R12 = R1 + R2. ΠŸΡ€ΠΈ этом схСма рис. 27, Π° замСняСтся эквивалСнтной схСмой рис. 27, Π±. Π—Π°Ρ‚Π΅ΠΌ ΠΎΠΏΡ€Π΅Π΄Π΅Π»ΠΈΡ‚Π΅ сопротивлСниС сопротивлСния R123 ΠΈ R3 ΠΏΠΎ Ρ„ΠΎΡ€ΠΌΡƒΠ»Π΅

.

ΠŸΡ€ΠΈ этом схСма рис. 27, Π± замСняСтся эквивалСнтной схСмой рис. 27, ΠΊΠΎΡ€ΠΏ. ПослС этого находят эквивалСнтноС сопротивлСниС всСй Ρ†Π΅ΠΏΠΈ суммированиСм сопротивлСния R123 ΠΈ ΠΏΠΎΡΠ»Π΅Π΄ΠΎΠ²Π°Ρ‚Π΅Π»ΡŒΠ½ΠΎ Π²ΠΊΠ»ΡŽΡ‡Π΅Π½Π½ΠΎΠ³ΠΎ с Π½ΠΈΠΌ сопротивлСния R4:

ΠŸΠΎΡΠ»Π΅Π΄ΠΎΠ²Π°Ρ‚Π΅Π»ΡŒΠ½ΠΎΠ΅, ΠΏΠ°Ρ€Π°Π»Π»Π΅Π»ΡŒΠ½ΠΎΠ΅ ΠΈ смСшанноС ΡˆΠΈΡ€ΠΎΠΊΠΎ примСняСмоС сопротивлСниС пусковых рСостатов ΠΏΡ€ΠΈ пускС э.ΠΏ. с. постоянного Ρ‚ΠΎΠΊΠ°.

ΠšΠ°Π»ΡŒΠΊΡƒΠ»ΡΡ‚ΠΎΡ€ соСдинСния рСзисторов ΠΎΠ½Π»Π°ΠΉΠ½. ΠŸΠ°Ρ€Π°Π»Π»Π΅Π»ΡŒΠ½ΠΎΠ΅ соСдинСниС рСзисторов

Π’ ΠΊΠ°ΠΆΠ΄ΠΎΠΉ схСмС присутствуСт рСзистор, ΠΈΠΌΠ΅ΡŽΡ‰ΠΈΠΉ сопротивлСниС элСктричСскому Ρ‚ΠΎΠΊΡƒ. РСзисторы Π±Ρ‹Π²Π°ΡŽΡ‚ Π΄Π²ΡƒΡ… Ρ‚ΠΈΠΏΠΎΠ²: постоянныС ΠΈ ΠΏΠ΅Ρ€Π΅ΠΌΠ΅Π½Π½Ρ‹Π΅. Π’ΠΎ врСмя Ρ€Π°Π·Ρ€Π°Π±ΠΎΡ‚ΠΊΠΈ любой элСктричСской схСмы ΠΈ Ρ€Π΅ΠΌΠΎΠ½Ρ‚Π° элСктронных ΠΈΠ·Π΄Π΅Π»ΠΈΠΉ часто ΠΈΡΠΏΠΎΠ»ΡŒΠ·ΡƒΠ΅Ρ‚ΡΡ рСзистор, ΠΎΠ±Π»Π°Π΄Π°ΡŽΡ‰ΠΈΠΉ Π½ΠΎΠΌΠΈΠ½Π°Π»ΠΎΠΌ.

НСсмотря Π½Π° Ρ‚ΠΎ Ρ‡Ρ‚ΠΎ для рСзисторов, выставлСны Ρ€Π°Π·Π»ΠΈΡ‡Π½Ρ‹Π΅ Π½ΠΎΠΌΠΈΠ½Π°Π»Ρ‹
, ΠΌΠΎΠΆΠ΅Ρ‚ ΡΠ»ΡƒΡ‡ΠΈΡ‚ΡŒΡΡ Ρ‚Π°ΠΊ, Ρ‡Ρ‚ΠΎ Π½Π΅ Π±ΡƒΠ΄Π΅Ρ‚ возмоТности Π½Π°ΠΉΡ‚ΠΈ Π½Π΅ΠΎΠ±Ρ…ΠΎΠ΄ΠΈΠΌΡ‹ΠΉ ΠΏΠΎΠΊΠ°Π·Π°Ρ‚Π΅Π»ΡŒ ΠΈΠ»ΠΈ Π²ΠΎΠΎΠ±Ρ‰Π΅ Π½ΠΈ ΠΎΠ΄ΠΈΠ½ элСмСнт Π½Π΅ смоТСт ΠΎΠ±Π΅ΡΠΏΠ΅Ρ‡ΠΈΡ‚ΡŒ Ρ‚Ρ€Π΅Π±ΡƒΠ΅ΠΌΡ‹ΠΉ ΠΏΠΎΠΊΠ°Π·Π°Ρ‚Π΅Π»ΡŒ.

РСшСниСм этой ΠΏΡ€ΠΎΠ±Π»Π΅ΠΌΡ‹ ΠΌΠΎΠΆΠ΅Ρ‚ ΡΡ‚Π°Ρ‚ΡŒ ΠΏΠΎΡΠ»Π΅Π΄ΠΎΠ²Π°Ρ‚Π΅Π»ΡŒΠ½ΠΎΠ³ΠΎ ΠΈ ΠΏΠ°Ρ€Π°Π»Π»Π΅Π»ΡŒΠ½ΠΎΠ³ΠΎ соСдинСния. ΠžΠ·Π½Π°ΠΊΠΎΠΌΡŒΡ‚Π΅ΡΡŒ с этой ΡΡ‚Π°Ρ‚ΡŒΠ΅ΠΉ, ΠΎΠ·Π½Π°ΠΊΠΎΠΌΡŒΡ‚Π΅ΡΡŒ с особСнностями выполнСния расчСта ΠΈ ΠΏΠΎΠ΄ Ρ€Π°Π·Π»ΠΈΡ‡Π½Ρ‹ΠΌΠΈ Π½ΠΎΠΌΠΈΠ½Π°Π»ΠΎΠ² сопротивлСний.

ΠŸΠ°Ρ€Π°Π»Π»Π΅Π»ΡŒΠ½ΠΎΠ΅ соСдинСниС: общая информация

Часто ΠΏΡ€ΠΈ ΠΈΠ·Π³ΠΎΡ‚ΠΎΠ²Π»Π΅Π½ΠΈΠΈ ΠΊΠ°ΠΊΠΎΠ³ΠΎ-Π»ΠΈΠ±ΠΎ устройства ΠΈΡΠΏΠΎΠ»ΡŒΠ·ΡƒΡŽΡ‚ рСзисторы, ΠΊΠΎΡ‚ΠΎΡ€Ρ‹Π΅ ΡΠΎΠ΅Π΄ΠΈΠ½ΡΡŽΡ‚ΡΡ Π² соотвСтствии с ΠΏΠΎΡΠ»Π΅Π΄ΠΎΠ²Π°Ρ‚Π΅Π»ΡŒΠ½ΠΎΠΉ схСмой. Π­Ρ„Ρ„Π΅ΠΊΡ‚ ΠΎΡ‚ примСнСния Ρ‚Π°ΠΊΠΎΠ³ΠΎ Π²Π°Ρ€ΠΈΠ°Π½Ρ‚Π° сборки сводится ΠΊ ΡƒΠ²Π΅Π»ΠΈΡ‡Π΅Π½ΠΈΡŽ ΠΎΠ±Ρ‰Π΅Π³ΠΎ сопротивлСния Ρ†Π΅ΠΏΠΈ.Для Π΄Π°Π½Π½ΠΎΠ³ΠΎ Π²Π°Ρ€ΠΈΠ°Π½Ρ‚Π° соСдинСния создаваСмоС создаваСмоС ΠΈΠΌΠΈ сопротивлСниС рассчитываСтся ΠΊΠ°ΠΊ сумма Π½ΠΎΠΌΠΈΠ½Π°Π»ΠΎΠ². Если ΠΆΠ΅ сборка Π΄Π΅Ρ‚Π°Π»Π΅ΠΉ выполняСтся ΠΏΠΎ ΠΏΠ°Ρ€Π°Π»Π»Π΅Π»ΡŒΠ½ΠΎΠΉ схСмС, Ρ‚ΠΎ здСсь потрСбуСтся Ρ€Π°ΡΡΡ‡ΠΈΡ‚Π°Ρ‚ΡŒ сопротивлСниС
, ΠΈΡΠΏΠΎΠ»ΡŒΠ·ΡƒΡ ниТСописанныС Ρ„ΠΎΡ€ΠΌΡƒΠ»Ρ‹.

Π’ схСмС ΠΏΠ°Ρ€Π°Π»Π»Π΅Π»ΡŒΠ½ΠΎΠ³ΠΎ соСдинСния ΠΏΡ€ΠΈΠ±Π΅Π³Π°ΡŽΡ‚ Π² ситуации, ΠΊΠΎΠ³Π΄Π° стоит Π·Π°Π΄Π°Ρ‡Π° ΠΏΠΎ сниТСнию суммарного сопротивлСния, Π°, ΠΊΡ€ΠΎΠΌΠ΅ Ρ‚ΠΎΠ³ΠΎ, увСличСния мощности для Π³Ρ€ΡƒΠΏΠΏΡ‹ элСмСнтов, ΠΏΠΎΠ΄ΠΊΠ»ΡŽΡ‡Π΅Π½Π½Ρ‹Ρ… ΠΏΠΎ ΠΏΠ°Ρ€Π°Π»Π»Π΅Π»ΡŒΠ½ΠΎΠΉ схСмС, Π΄ΠΎΠ»ΠΆΠ½ΠΎ Π±Ρ‹Ρ‚ΡŒ большС, Ρ‡Π΅ΠΌ ΠΏΡ€ΠΈ ΠΈΡ… ΠΎΡ‚Π΄Π΅Π»ΡŒΠ½ΠΎΠΌ ΠΏΠΎΠ΄ΠΊΠ»ΡŽΡ‡Π΅Π½ΠΈΠΈ.

РасчСт сопротивлСния

Π’ случаС ΠΏΠΎΠ΄ΠΊΠ»ΡŽΡ‡Π΅Π½ΠΈΡ Π΄Π΅Ρ‚Π°Π»Π΅ΠΉ Π΄Ρ€ΡƒΠ³ с Π΄Ρ€ΡƒΠ³ΠΎΠΌ, с ΠΏΠ°Ρ€Π°Π»Π»Π΅Π»ΡŒΠ½ΠΎΠΉ схСмы для расчСта ΠΎΠ±Ρ‰Π΅Π³ΠΎ сопротивлСния, Π±ΡƒΠ΄Π΅Ρ‚ пСрвая Ρ„ΠΎΡ€ΠΌΡƒΠ»Π°:

R (ΠΎΠ±Ρ‰) = 1 / (1 / R1 + 1 / R2 + 1 / R3 + 1 / Rn).

  • R1- R3 ΠΈ Rn — рСзисторы, ΠΏΠΎΠ΄ΠΊΠ»ΡŽΡ‡Π΅Π½Π½Ρ‹Π΅ ΠΏΠΎ ΠΏΠ°Ρ€Π°Π»Π»Π΅Π»ΡŒΠ½ΠΎΠΉ схСмС.

ΠŸΡ€ΠΈΡ‡Π΅ΠΌ, Ссли создаСтся схСма Π½Π° основС Ρ‚ΠΎΠ»ΡŒΠΊΠΎ Π΄Π²ΡƒΡ… элСмСнтов, Ρ‚ΠΎ для опрСдСлСния суммарного номинального сопротивлСния слСдуСт ΠΈΡΠΏΠΎΠ»ΡŒΠ·ΠΎΠ²Π°Ρ‚ΡŒ Ρ‚Π°ΠΊΡƒΡŽ ​​формулу:

R (ΠΎΠ±Ρ‰) = R1 * R2 / R1 + R2.

  • R (ΠΎΠ±Ρ‰) — суммарноС сопротивлСниС;
  • R1 ΠΈ R2 — рСзисторы, ΠΏΠΎΠ΄ΠΊΠ»ΡŽΡ‡Π΅Π½Π½Ρ‹Π΅ ΠΏΠΎ ΠΏΠ°Ρ€Π°Π»Π»Π΅Π»ΡŒΠ½ΠΎΠΉ схСмС.

Π’ΠΈΠ΄Π΅ΠΎ: ΠŸΡ€ΠΈΠΌΠ΅Ρ€ расчёта сопротивлСния

Π£Π½ΠΈΠ²Π΅Ρ€ΡΠ°Π»ΡŒΠ½Π°Ρ схСма расчСта

ΠŸΡ€ΠΈΠΌΠ΅Π½ΠΈΡ‚Π΅Π»ΡŒΠ½ΠΎ ΠΊ Ρ€Π°Π΄ΠΈΠΎΡ‚Π΅Ρ…Π½ΠΈΠΊΠ΅ слСдуСт ΡƒΠ΄Π΅Π»ΠΈΡ‚ΡŒ Π²Π½ΠΈΠΌΠ°Π½ΠΈΠ΅ ΠΎΠ΄Π½ΠΎΠΌΡƒ Π²Π°ΠΆΠ½ΠΎΠΌΡƒ ΠΏΡ€Π°Π²ΠΈΠ»Ρƒ: Ссли ΠΏΠΎΠ΄ΠΊΠ»ΡŽΡ‡Π°Π΅ΠΌΡ‹Π΅ Π΄Ρ€ΡƒΠ³ ΠΊ Π΄Ρ€ΡƒΠ³Ρƒ элСмСнты ΠΏΠΎ ΠΏΠ°Ρ€Π°Π»Π»Π΅Π»ΡŒΠ½ΠΎΠΉ схСмС ΠΈΠΌΠ΅ΡŽΡ‚ ΠΎΠ΄ΠΈΠ½Π°ΠΊΠΎΠ²Ρ‹ΠΉ ΠΏΠΎΠΊΠ°Π·Π°Ρ‚Π΅Π»ΡŒ
, Ρ‚ΠΎ для расчСта суммарного Π½ΠΎΠΌΠΈΠ½Π°Π»Π° Π½Π΅ΠΎΠ±Ρ…ΠΎΠ΄ΠΈΠΌΠΎ ΠΎΠ±Ρ‰Π΅Π΅ Π·Π½Π°Ρ‡Π΅Π½ΠΈΠ΅ Ρ€Π°Π·Π΄Π΅Π»ΠΈΡ‚ΡŒ Π½Π° число ΠΏΠΎΠ΄ΠΊΠ»ΡŽΡ‡Π΅Π½Π½Ρ‹Ρ… ΡƒΠ·Π»ΠΎΠ²:

  • R (ΠΎΠ±Ρ‰) — суммарноС Π·Π½Π°Ρ‡Π΅Π½ΠΈΠ΅ сопротивлСния;
  • R — Π½ΠΎΠΌΠΈΠ½Π°Π» рСзистора, ΠΏΠΎΠ΄ΠΊΠ»ΡŽΡ‡Π΅Π½Π½ΠΎΠ³ΠΎ ΠΏΠΎ ΠΏΠ°Ρ€Π°Π»Π»Π΅Π»ΡŒΠ½ΠΎΠΉ схСмС;
  • n — число ΠΏΠΎΠ΄ΠΊΠ»ΡŽΡ‡Π΅Π½Π½Ρ‹Ρ… ΡƒΠ·Π»ΠΎΠ².

ОсобоС Π²Π½ΠΈΠΌΠ°Π½ΠΈΠ΅ слСдуСт ΠΎΠ±Ρ€Π°Ρ‚ΠΈΡ‚ΡŒ Π½Π° Ρ‚ΠΎ, Ρ‡Ρ‚ΠΎ ΠΊΠΎΠ½Π΅Ρ‡Π½Ρ‹ΠΉ ΠΏΠΎΠΊΠ°Π·Π°Ρ‚Π΅Π»ΡŒ сопротивлСния Π² случаС использования ΠΏΠ°Ρ€Π°Π»Π»Π΅Π»ΡŒΠ½ΠΎΠΉ схСмы ΠΎΠ±ΡΠ·Π°Ρ‚Π΅Π»ΡŒΠ½ΠΎ Π±ΡƒΠ΄Π΅Ρ‚ мСньшС
ΠΏΠΎ ΡΡ€Π°Π²Π½Π΅Π½ΠΈΡŽ с Π½ΠΎΠΌΠΈΠ½Π°Π»ΠΎΠΌ любого элСмСнта, ΠΏΠΎΠ΄ΠΊΠ»ΡŽΡ‡Π°Π΅ΠΌΠΎΠ³ΠΎ Π² Ρ†Π΅ΠΏΠΈ.

ΠŸΡ€ΠΈΠΌΠ΅Ρ€ расчёта

Для большСй наглядности ΠΌΠΎΠΆΠ½ΠΎ Ρ€Π°ΡΡΠΌΠΎΡ‚Ρ€Π΅Ρ‚ΡŒ ΡΠ»Π΅Π΄ΡƒΡŽΡ‰ΠΈΠΉ ΠΏΡ€ΠΈΠΌΠ΅Ρ€: допустимыС, Ρƒ нас Π΅ΡΡ‚ΡŒ Ρ‚Ρ€ΠΈ рСзистора, Ρ‡ΡŒΠΈ Π½ΠΎΠΌΠΈΠ½Π°Π»Ρ‹ соотвСтствСнно Ρ€Π°Π²Π½Ρ‹ 100, 150 ΠΈ 30 Ом. Если ΠΈΡΠΏΠΎΠ»ΡŒΠ·ΠΎΠ²Π°Ρ‚ΡŒ ΠΏΠ΅Ρ€Π²ΡƒΡŽ Ρ„ΠΎΡ€ΠΌΡƒΠ»ΠΎΠΉ для опрСдСлСния ΠΎΠ±Ρ‰Π΅Π³ΠΎ Π½ΠΎΠΌΠΈΠ½Π°Π»Π°, Ρ‚ΠΎ ΠΏΠΎΠ»ΡƒΡ‡ΠΈΠΌ ΡΠ»Π΅Π΄ΡƒΡŽΡ‰Π΅Π΅:

R (ΠΎΠ±Ρ‰) = 1 / (1/100 + 1/150 + 1/30) =

1 / (0,01 + 0,007 + 0,03 ) = 1 / 0,047 = 21,28Ом.

Если Π²Ρ‹ΠΏΠΎΠ»Π½ΠΈΡ‚ΡŒ нСслоТныС расчСты, Ρ‚ΠΎ ΠΌΠΎΠΆΠ½ΠΎ ΠΏΠΎΠ»ΡƒΡ‡ΠΈΡ‚ΡŒ ΡΠ»Π΅Π΄ΡƒΡŽΡ‰Π΅Π΅: для Ρ†Π΅ΠΏΠΈ, Π²ΠΊΠ»ΡŽΡ‡Π°ΡŽΡ‰Π΅ΠΉ Π² сСбя Ρ‚Ρ€ΠΈ Π΄Π΅Ρ‚Π°Π»ΠΈ, Π³Π΄Π΅ наимСньший ΠΏΠΎΠΊΠ°Π·Π°Ρ‚Π΅Π»ΡŒ сопротивлСния составляСт 30 Ом, Ρ€Π΅Π·ΡƒΠ»ΡŒΡ‚ΠΈΡ€ΡƒΡŽΡ‰Π΅Π΅ Π·Π½Π°Ρ‡Π΅Π½ΠΈΠ΅ Π±ΡƒΠ΄Π΅Ρ‚ Ρ€Π°Π²Π½ΠΎ 21,28 Ом. Π­Ρ‚ΠΎΡ‚ ΠΏΠΎΠΊΠ°Π·Π°Ρ‚Π΅Π»ΡŒ Π±ΡƒΠ΄Π΅Ρ‚ мСньшС минимального значСния Π½ΠΎΠΌΠΈΠ½Π°Π»Π° Π² Ρ†Π΅ΠΏΠΈ практичСски Π½Π° 30%.

Π’Π°ΠΆΠ½Ρ‹Π΅ Π½ΡŽΠ°Π½ΡΡ‹

ΠžΠ±Ρ‹Ρ‡Π½ΠΎ для рСзисторов ΠΏΠ°Ρ€Π°Π»Π»Π΅Π»ΡŒΠ½ΠΎΠ΅ соСдинСниС Ρ‚ΠΎΠ³Π΄Π°, ΠΊΠΎΠ³Π΄Π° стоит Π·Π°Π΄Π°Ρ‡Π° ΠΏΠΎ созданию большСй мощности. Для Π΅Π΅ Ρ€Π΅ΡˆΠ΅Π½ΠΈΡ ΠΏΠΎΡ‚Ρ€Π΅Π±ΡƒΡŽΡ‚ΡΡ рСзисторы, ΠΊΠΎΡ‚ΠΎΡ€Ρ‹Π΅ Π΄ΠΎΠ»ΠΆΠ½Ρ‹ ΠΈΠΌΠ΅Ρ‚ΡŒ Ρ€Π°Π²Π½Ρ‹Π΅ ΠΏΠΎΠΊΠ°Π·Π°Ρ‚Π΅Π»ΠΈ сопротивлСния ΠΈ мощности.ΠŸΡ€ΠΈ Ρ‚Π°ΠΊΠΎΠΌ Π²Π°Ρ€ΠΈΠ°Π½Ρ‚Π΅ ΠΎΠΏΡ€Π΅Π΄Π΅Π»ΠΈΡ‚ΡŒ ΠΎΠ±Ρ‰ΡƒΡŽ ΠΌΠΎΡ‰Π½ΠΎΡΡ‚ΡŒ ΠΌΠΎΠΆΠ½ΠΎ ΡΠ»Π΅Π΄ΡƒΡŽΡ‰ΠΈΠΌ ΠΎΠ±Ρ€Π°Π·ΠΎΠΌ
: ΠΌΠΎΡ‰Π½ΠΎΡΡ‚ΡŒ ΠΎΠ΄Π½ΠΎΠ³ΠΎ элСмСнта Π½Π΅ΠΎΠ±Ρ…ΠΎΠ΄ΠΈΠΌΠΎ ΠΏΠ΅Ρ€Π΅ΠΌΠ½ΠΎΠΆΠΈΡ‚ΡŒ с совокупным числом всСх рСзисторов, состоящих ΠΈΠ· Ρ†Π΅ΠΏΠΈ, ΡΠΎΠΎΡ‚Π²Π΅Ρ‚ΡΡ‚Π²ΡƒΡŽΡ‰Π΅ΠΉ Π΄Ρ€ΡƒΠ³ Π΄Ρ€ΡƒΠ³Ρƒ Π² соотвСтствии с ΠΏΠ°Ρ€Π°Π»Π»Π΅Π»ΡŒΠ½ΠΎΠΉ схСмой.

Π‘ΠΊΠ°ΠΆΠ΅ΠΌ, Ссли Π½Π°ΠΌΠΈ Π±Ρ‹Π»ΠΈ ΠΎΠ±ΡŠΡΠ²Π»Π΅Π½Ρ‹ ΠΏΡΡ‚ΡŒ рСзисторов, Ρ‡Π΅ΠΉΠ» составляСт 100 Ом, Π° ΠΌΠΎΡ‰Π½ΠΎΡΡ‚ΡŒ ΠΊΠ°ΠΆΠ΄ΠΎΠ³ΠΎ Ρ€Π°Π²Π½Π° 1 Π’Ρ‚, Ρ‡Ρ‚ΠΎ суммарный ΠΏΠΎΠΊΠ°Π·Π°Ρ‚Π΅Π»ΡŒ сопротивлСния Π±ΡƒΠ΄Π΅Ρ‚ Ρ€Π°Π²Π΅Π½ 20 Ом, Π° ΠΌΠΎΡ‰Π½ΠΎΡΡ‚ΡŒ составит 5 Π’Ρ‚.

Π²Π·ΡΡ‚ΡŒ Ρ‚Π΅ ΠΆΠ΅ рСзисторы, Π½ΠΎ ΠΏΠΎΠ΄ΠΊΠ»ΡŽΡ‡ΠΈΡ‚ΡŒ ΠΈΡ… Π² соотвСтствии с ΠΏΠΎΡΠ»Π΅Π΄ΠΎΠ²Π°Ρ‚Π΅Π»ΡŒΠ½ΠΎΠΉ схСмой, Ρ‚ΠΎ конСчная ΠΌΠΎΡ‰Π½ΠΎΡΡ‚ΡŒ составит 5 Π’Ρ‚, Π° суммарный Π½ΠΎΠΌΠΈΠ½Π°Π» Π±ΡƒΠ΄Π΅Ρ‚ Ρ€Π°Π²Π΅Π½ 500 Ом.

Π’ΠΈΠ΄Π΅ΠΎ: ΠŸΡ€Π°Π²ΠΈΠ»ΡŒΠ½ΠΎΠ΅ ΠΏΠΎΠ΄ΠΊΠ»ΡŽΡ‡Π΅Π½ΠΈΠ΅ свСтодиодов

ΠŸΠ°Ρ€Π°Π»Π»Π΅Π»ΡŒΠ½Π°Ρ схСма ΠΏΠΎΠ΄ΠΊΠ»ΡŽΡ‡Π΅Π½ΠΈΡ рСзисторов ΠΎΡ‡Π΅Π½ΡŒ вострСбована ΠΏΠΎ Ρ‚ΠΎΠΉ ΠΏΡ€ΠΈΡ‡ΠΈΠ½Π΅, Ρ‡Ρ‚ΠΎ Π²ΠΎΠ·Π½ΠΈΠΊΠ°Π΅Ρ‚ ΠΏΡ€ΠΎΠ±Π»Π΅ΠΌΠ° создания Ρ‚Π°ΠΊΠΎΠ³ΠΎ Π½ΠΎΠΌΠΈΠ½Π°Π»Π°, Π½Π΅Π²ΠΎΠ·ΠΌΠΎΠΆΠ½ΠΎΠ³ΠΎ ΠΏΡ€ΠΈ ΠΏΠΎΠΌΠΎΡ‰ΠΈ простого ΠΏΠ°Ρ€Π°Π»Π»Π΅Π»ΡŒΠ½ΠΎΠ³ΠΎ соСдинСния. ΠŸΡ€ΠΈ этом ΠΏΡ€ΠΎΡ†Π΅Π΄ΡƒΡ€Π° расчСта этого ΠΏΠ°Ρ€Π°ΠΌΠ΅Ρ‚Ρ€Π° достаточной ΡΠ»ΠΎΠΆΠ½ΠΎΡΡ‚ΡŒΡŽ
, Π³Π΄Π΅ Π½Π΅ΠΎΠ±Ρ…ΠΎΠ΄ΠΈΠΌΠΎ привСсти Ρ€Π°Π·Π½Ρ‹Π΅ ΠΏΠ°Ρ€Π°ΠΌΠ΅Ρ‚Ρ€Ρ‹.

. Π—Π΄Π΅ΡΡŒ ваТная Ρ€ΠΎΠ»ΡŒ отводится Π½Π΅ Ρ‚ΠΎΠ»ΡŒΠΊΠΎ количСству ΠΏΠΎΠ΄ΠΊΠ»ΡŽΡ‡Π°Π΅ΠΌΡ‹Ρ…, Π½ΠΎ ΠΈ Ρ€Π°Π±ΠΎΡ‡ΠΈΡ… ΠΏΠ°Ρ€Π°ΠΌΠ΅Ρ‚Ρ€ΠΎΠ² рСзисторов — ΠΏΡ€Π΅ΠΆΠ΄Π΅ всСго, ΡΠΎΠΏΡ€ΠΎΡ‚ΠΈΠ²Π»Π΅Π½ΠΈΡŽ ΠΈ мощности. Если ΠΎΠ΄ΠΈΠ½ ΠΈΠ· ΠΏΠΎΠ΄ΠΊΠ»ΡŽΡ‡Π°Π΅ΠΌΡ‹Ρ… элСмСнтов Π±ΡƒΠ΄Π΅Ρ‚ ΠΈΠΌΠ΅Ρ‚ΡŒ нСподходящий ΠΏΠΎΠΊΠ°Π·Π°Ρ‚Π΅Π»ΡŒ, Ρ‚ΠΎ это Π½Π΅ ΠΏΠΎΠ·Π²ΠΎΠ»ΠΈΡ‚ Ρ€Π΅ΡˆΠΈΡ‚ΡŒ Π·Π°Π΄Π°Ρ‡Ρƒ ΠΏΠΎ созданию Π½Π΅ΠΎΠ±Ρ…ΠΎΠ΄ΠΈΠΌΠΎΠ³ΠΎ Π½ΠΎΠΌΠΈΠ½Π°Π»Π° Π² Ρ†Π΅ΠΏΠΈ.

ΠŸΠ°Ρ€Π°Π»Π»Π΅Π»ΡŒΠ½ΠΎΠ΅ соСдинСниС рСзисторов
— ΠΎΠ΄Π½ΠΎ ΠΈΠ· Π΄Π²ΡƒΡ… Π²ΠΈΠ΄ΠΎΠ² элСктричСских соСдинСний, ΠΊΠΎΠ³Π΄Π° ΠΎΠ΄Π½ΠΎ ΠΈΠ· Π²Ρ‹Π²ΠΎΠ΄ΠΎΠ² ΠΎΠ΄Π½ΠΎΠ³ΠΎ рСзистора соСдинСно с Π²Ρ‹Π²ΠΎΠ΄ΠΌΠΈ Π΄Ρ€ΡƒΠ³ΠΎΠ³ΠΎ рСзистора ΠΈΠ»ΠΈ рСзисторов.Π—Π° Ρ€Π΅Π°Π»ΠΈΠ·Π°Ρ†ΠΈΡŽ ΠΈΠ»ΠΈ для Ρ‚ΠΎΠ³ΠΎ, Ρ‡Ρ‚ΠΎΠ±Ρ‹ ΡΠΎΠ·Π΄Π°Ρ‚ΡŒ слоТныС элСктронныС схСмы.

Π‘Ρ…Π΅ΠΌΠ° ΠΏΠ°Ρ€Π°Π»Π»Π΅Π»ΡŒΠ½ΠΎΠ³ΠΎ соСдинСния ΠΏΠΎΠΊΠ°Π·Π°Π½Π° Π½Π° рисункС Π½ΠΈΠΆΠ΅. ΠŸΡ€ΠΈ ΠΏΠ°Ρ€Π°Π»Π»Π΅Π»ΡŒΠ½ΠΎΠΌ соСдинСнии рСзисторов, напряТСниС Π½Π° всСх уровнях Π±ΡƒΠ΄Π΅Ρ‚ ΠΎΠ΄ΠΈΠ½Π°ΠΊΠΎΠ²Ρ‹ΠΌ, Π° ΠΏΡ€ΠΎΡ‚Π΅ΠΊΠ°ΡŽΡ‰ΠΈΠΉ Ρ‡Π΅Ρ€Π΅Π· Π½ΠΈΡ… Ρ‚ΠΎΠΊ Π±ΡƒΠ΄Π΅Ρ‚ ΠΏΡ€ΠΎΠΏΠΎΡ€Ρ†ΠΈΠΎΠ½Π°Π»Π΅Π½ ΠΈΡ… ΡΠΎΠΏΡ€ΠΎΡ‚ΠΈΠ²Π»Π΅Π½ΠΈΡŽ:

Π€ΠΎΡ€ΠΌΡƒΠ»Π° ΠΏΠ°Ρ€Π°Π»Π»Π΅Π»ΡŒΠ½ΠΎΠ³ΠΎ соСдинСния рСзисторов

ΠžΠ±Ρ‰Π΅Π΅ сопротивлСниС рСзисторов соСдинСнных вмСстС ΠΏΠΎ ΡΠ»Π΅Π΄ΡƒΡŽΡ‰Π΅ΠΉ Ρ„ΠΎΡ€ΠΌΡƒΠ»Π΅:

Π’ΠΎΠΊ, ΠΏΡ€ΠΎΡ‚Π΅ΠΊΠ°ΡŽΡ‰ΠΈΠΉ Ρ‡Π΅Ρ€Π΅Π· ΠΎΡ‚Π΄Π΅Π»ΡŒΠ½ΠΎ взятый рСзистор, согласно, ΠΌΠΎΠΆΠ½ΠΎ Π½Π°ΠΉΡ‚ΠΈ ΠΏΠΎ Ρ„ΠΎΡ€ΠΌΡƒΠ»Π΅:

ΠŸΠ°Ρ€Π°Π»Π»Π΅Π»ΡŒΠ½ΠΎΠ΅ соСдинСниС рСзисторов — расчСт

ΠŸΡ€ΠΈΠΌΠ΅Ρ€ β„–1

ΠŸΡ€ΠΈ Ρ€Π°Π·Ρ€Π°Π±ΠΎΡ‚ΠΊΠ΅ устройства, Π²ΠΎΠ·Π½ΠΈΠΊΠ»Π° Π½Π΅ΠΎΠ±Ρ…ΠΎΠ΄ΠΈΠΌΠΎΡΡ‚ΡŒ ΡƒΡΡ‚Π°Π½ΠΎΠ²ΠΈΡ‚ΡŒ рСзистор с сопротивлСниСм 8 Ом.Если ΠΌΡ‹ просмотрим вСсь Π½ΠΎΠΌΠΈΠ½Π°Π»ΡŒΠ½Ρ‹ΠΉ ряд стандартных Π·Π½Π°Ρ‡Π΅Π½ΠΈΠΉ рСзисторов, Ρ‚ΠΎ ΠΌΡ‹ ΡƒΠ²ΠΈΠ΄ΠΈΠΌ, Ρ‡Ρ‚ΠΎ рСзистора с сопротивлСниСм Π² 8 Ом Π² Π½Π΅ΠΌ Π½Π΅Ρ‚.

Π’Ρ‹Ρ…ΠΎΠ΄ΠΎΠΌ ΠΈΠ· Π΄Π°Π½Π½ΠΎΠΉ ситуации Π±ΡƒΠ΄Π΅Ρ‚ использованиС Π΄Π²ΡƒΡ… ΠΏΠ°Ρ€Π°Π»Π»Π΅Π»ΡŒΠ½ΠΎ соСдинСнных рСзисторов. Π­ΠΊΠ²ΠΈΠ²Π°Π»Π΅Π½Ρ‚Π½ΠΎΠ΅ Π·Π½Π°Ρ‡Π΅Π½ΠΈΠ΅ сопротивлСния для рСзисторов, соСдинСнных ΠΏΠ°Ρ€Π°Π»Π»Π΅Π»ΡŒΠ½Ρ‹ΠΌ ΠΎΠ±Ρ€Π°Π·ΠΎΠΌ ΡΠ»Π΅Π΄ΡƒΡŽΡ‰ΠΈΠΌ ΠΎΠ±Ρ€Π°Π·ΠΎΠΌ:

ΡƒΡ€Π°Π²Π½Π΅Π½ΠΈΠ΅ ΠΏΠΎΠΊΠ°Π·Ρ‹Π²Π°Π΅Ρ‚, Ρ‡Ρ‚ΠΎ Ссли R1 равСнство R2, Ρ‚ΠΎ сопротивлСниС R составляСт ΠΏΠΎΠ»ΠΎΠ²ΠΈΠ½Ρƒ сопротивлСния ΠΎΠ΄Π½ΠΎΠ³ΠΎ ΠΈΠ· Π΄Π²ΡƒΡ… рСзисторов.ΠŸΡ€ΠΈ R = 8 Ом, R1 ΠΈ R2 Π΄ΠΎΠ»ΠΆΠ½Ρ‹, ΡΠ»Π΅Π΄ΠΎΠ²Π°Ρ‚Π΅Π»ΡŒΠ½ΠΎ, ΠΈΠΌΠ΅Ρ‚ΡŒ Π·Π½Π°Ρ‡Π΅Π½ΠΈΠ΅ 2 Γ— 8 = 16 Ом.
Π’Π΅ΠΏΠ΅Ρ€ΡŒ ΠΏΡ€ΠΎΠ²Π΅Π΄Π΅ΠΌ ΠŸΡ€ΠΎΠ²Π΅Ρ€ΠΊΠ°, рассчитав ΠΎΠ±Ρ‰Π΅Π΅ сопротивлСниС Π΄Π²ΡƒΡ… рСзисторов:

Π’Π°ΠΊΠΈΠΌ ΠΎΠ±Ρ€Π°Π·ΠΎΠΌ, ΠΌΡ‹ ΠΏΠΎΠ»ΡƒΡ‡ΠΈΠ»ΠΈ сопротивлСниС 8 Ом, соСдинив ΠΏΠ°Ρ€Π°Π»Π»Π΅Π»ΡŒΠ½ΠΎ Π΄Π²Π° рСзистора ΠΏΠΎ 16 Ом.

ΠŸΡ€ΠΈΠΌΠ΅Ρ€ расчСта β„–2

Найти ΠΎΠ±Ρ‰Π΅Π΅ сопротивлСниС R ΠΈΠ· Ρ‚Ρ€Π΅Ρ… соСдинСнных рСзисторов:

ΠžΠ±Ρ‰Π΅Π΅ сопротивлСниС R рассчитываСтся ΠΏΠΎ Ρ„ΠΎΡ€ΠΌΡƒΠ»Π΅:

Π­Ρ‚ΠΎΡ‚ ΠΌΠ΅Ρ‚ΠΎΠ΄ расчСта ΠΌΠΎΠΆΠ΅Ρ‚ Π±Ρ‹Ρ‚ΡŒ использован для расчСта любого количСства ΠΎΡ‚Π΄Π΅Π»ΡŒΠ½Ρ‹Ρ… сопротивлСний соСдинСнных ΠΏΠ°Ρ€Π°Π»Π»Π΅Π»ΡŒΠ½ΠΎ.

Один Π²Π°ΠΆΠ½Ρ‹ΠΉ ΠΌΠΎΠΌΠ΅Π½Ρ‚, ΠΊΠΎΡ‚ΠΎΡ€Ρ‹ΠΉ Π½Π΅ΠΎΠ±Ρ…ΠΎΠ΄ΠΈΠΌΠΎ Π·Π°ΠΏΠΎΠΌΠ½ΠΈΡ‚ΡŒ ΠΏΡ€ΠΈ расчСтС ΠΏΠ°Ρ€Π°Π»Π»Π΅Π»ΡŒΠ½ΠΎ соСдинСнных рСзисторов — это Ρ‚ΠΎ, Ρ‡Ρ‚ΠΎ ΠΎΠ±Ρ‰Π΅Π΅ сопротивлСниС всСгда Π±ΡƒΠ΄Π΅Ρ‚ мСньшС, Ρ‡Π΅ΠΌ Π·Π½Π°Ρ‡Π΅Π½ΠΈΠ΅ наимСньшСго сопротивлСния Π² ΠΊΠΎΠΌΠ±ΠΈΠ½Π°Ρ†ΠΈΠΈ.

Как Ρ€Π°ΡΡΡ‡ΠΈΡ‚Π°Ρ‚ΡŒ схСмы слоТных рСзисторов

Π‘ΠΎΠ»Π΅Π΅ слоТныС рСзисторы, рассчитанныС Π½Π° систСматичСской Π³Ρ€ΡƒΠΏΠΏΠΈΡ€ΠΎΠ²ΠΊΠΈ рСзисторов. На рисункС Π½ΠΈΠΆΠ΅ Π½Π΅ΠΎΠ±Ρ…ΠΎΠ΄ΠΈΠΌΠΎ ΠΏΠΎΡΡ‡ΠΈΡ‚Π°Ρ‚ΡŒ ΠΎΠ±Ρ‰Π΅Π΅ сопротивлСниС Ρ†Π΅ΠΏΠΈ, состоящСй ΠΈΠ· Ρ‚Ρ€Π΅Ρ… рСзисторов:

.

Для простоты расчСта, сначала сгруппируСм рСзисторы ΠΏΠΎ ΠΏΠ°Ρ€Π°Π»Π»Π΅Π»ΡŒΠ½ΠΎΠΌΡƒ ΠΈ ΠΏΠΎΡΠ»Π΅Π΄ΠΎΠ²Π°Ρ‚Π΅Π»ΡŒΠ½ΠΎΠΌΡƒ Ρ‚ΠΈΠΏΡƒ соСдинСния.
РСзисторы R2 ΠΈ R3 соСдинСны ΠΏΠΎΡΠ»Π΅Π΄ΠΎΠ²Π°Ρ‚Π΅Π»ΡŒΠ½ΠΎ (Π³Ρ€ΡƒΠΏΠΏΠ° 2). Они Π² свою ΠΎΡ‡Π΅Ρ€Π΅Π΄ΡŒ соСдинСны с рСзистором R1 (Π³Ρ€ΡƒΠΏΠΏΠ° 1).

ΠŸΠΎΡΠ»Π΅Π΄ΠΎΠ²Π°Ρ‚Π΅Π»ΡŒΠ½ΠΎΠ΅ соСдинСниС рСзисторов Π³Ρ€ΡƒΠΏΠΏΡ‹ 2 вычисляСт сумму сопротивлСний R2 ΠΈ R3:

Π’ Ρ€Π΅Π·ΡƒΠ»ΡŒΡ‚Π°Ρ‚Π΅ ΠΌΡ‹ ΡƒΠΏΡ€ΠΎΡ‰Π°Π΅ΠΌ схСму Π² Π²ΠΈΠ΄Π΅ Π΄Π²ΡƒΡ… ΠΏΠ°Ρ€Π°Π»Π»Π΅Π»ΡŒΠ½Ρ‹Ρ… рСзисторов. Π’Π΅ΠΏΠ΅Ρ€ΡŒ ΠΎΠ±Ρ‰Π΅Π΅ сопротивлСниС всСй схСмы ΠΌΠΎΠΆΠ½ΠΎ ΠΏΠΎΡΡ‡ΠΈΡ‚Π°Ρ‚ΡŒ ΡΠ»Π΅Π΄ΡƒΡŽΡ‰ΠΈΠΌ ΠΎΠ±Ρ€Π°Π·ΠΎΠΌ:

РасчСт Π±ΠΎΠ»Π΅Π΅ слоТных соСдинСний рСзисторов ΠΌΠΎΠΆΠ½ΠΎ Π²Ρ‹ΠΏΠΎΠ»Π½ΠΈΡ‚ΡŒ ΠΈΡΠΏΠΎΠ»ΡŒΠ·ΡƒΡ Π·Π°ΠΊΠΎΠ½Ρ‹ ΠšΠΈΡ€Ρ…Π³ΠΎΡ„Π°.

, ΠΏΡ€ΠΎΡ‚Π΅ΠΊΠ°ΡŽΡ‰ΠΈΠΉ Π² Ρ†Π΅ΠΏΠΈ ΠΏΠ°Ρ€Π°Π»Π»Π΅Π»ΡŒΠ½Ρ‹Ρ… рСзисторов

ΠžΠ±Ρ‰ΠΈΠΉ Ρ‚ΠΎΠΊ I ΠΏΡ€ΠΎΡ‚Π΅ΠΊΠ°ΡŽΡ‰ΠΈΠΉ Π² Ρ†Π΅ΠΏΠΈ ΠΏΠ°Ρ€Π°Π»Π»Π΅Π»ΡŒΠ½Ρ‹Ρ… рСзисторов равняСтся суммС ΠΎΡ‚Π΄Π΅Π»ΡŒΠ½Ρ‹Ρ… Ρ‚ΠΎΠΊΠΎΠ², ΠΏΡ€ΠΎΡ‚Π΅ΠΊΠ°ΡŽΡ‰ΠΈΡ… Π²ΠΎ всСх ΠΏΠ°Ρ€Π°Π»Π»Π΅Π»ΡŒΠ½Ρ‹Ρ… вСтвях, ΠΏΡ€ΠΈΡ‡Π΅ΠΌ Ρ‚ΠΎΠΊ Π² ΠΎΡ‚Π΄Π΅Π»ΡŒΠ½ΠΎ взятой Π²Π΅Ρ‚Π²ΠΈ Π½Π΅ ΠΎΠ±ΡΠ·Π°Ρ‚Π΅Π»ΡŒΠ½ΠΎ Π΄ΠΎΠ»ΠΆΠ΅Π½ Π±Ρ‹Ρ‚ΡŒ Ρ€Π°Π²Π΅Π½ Ρ‚ΠΎΠΊΡƒ Π² сосСдних вСтвях.

НСсмотря Π½Π° ΠΏΠ°Ρ€Π°Π»Π»Π΅Π»ΡŒΠ½ΠΎΠ΅ соСдинСниС, ΠΊ ΠΊΠ°ΠΆΠ΄ΠΎΠΌΡƒ рСзистору ΠΏΡ€ΠΈΠ»ΠΎΠΆΠ΅Π½ΠΎ ΠΎΠ΄Π½ΠΎ ΠΈ Ρ‚ΠΎ ΠΆΠ΅ напряТСниС. А Π²Π΅Π»ΠΈΡ‡ΠΈΠ½Π° сопротивлСния Π² ΠΏΠ°Ρ€Π°Π»Π»Π΅Π»ΡŒΠ½ΠΎΠΉ Ρ†Π΅ΠΏΠΈ ΠΌΠΎΠΆΠ΅Ρ‚ Π±Ρ‹Ρ‚ΡŒ Ρ€Π°Π·Π½ΠΎΠΉ, Ρ‚ΠΎ ΠΈ Π²Π΅Π»ΠΈΡ‡ΠΈΠ½Π° ΠΏΡ€ΠΎΡ‚Π΅ΠΊΠ°ΡŽΡ‰Π΅Π³ΠΎ Ρ‚ΠΎΠΊΠ° Ρ‡Π΅Ρ€Π΅Π· ΠΊΠ°ΠΆΠ΄Ρ‹ΠΉ рСзистор Ρ‚ΠΎΠΆΠ΅ Π±ΡƒΠ΄Π΅Ρ‚ ΠΎΡ‚Π»ΠΈΡ‡Π°Ρ‚ΡŒΡΡ (ΠΏΠΎ ΠΎΠΏΡ€Π΅Π΄Π΅Π»Π΅Π½ΠΈΡŽ Π·Π°ΠΊΠΎΠ½Π° Ома).

Рассмотрим это Π½Π° ΠΏΡ€ΠΈΠΌΠ΅Ρ€Π΅ Π΄Π²ΡƒΡ… соСдинСнных рСзисторов. Π’ΠΎΠΊ, ΠΊΠΎΡ‚ΠΎΡ€Ρ‹ΠΉ Ρ‚Π΅Ρ‡Π΅Ρ‚ Ρ‡Π΅Ρ€Π΅Π· ΠΊΠ°ΠΆΠ΄Ρ‹ΠΉ ΠΈΠ· рСзисторов (I1 ΠΈ I2) Π±ΡƒΠ΄Π΅Ρ‚ ΠΎΡ‚Π»ΠΈΡ‡Π°Ρ‚ΡŒΡΡ Π΄Ρ€ΡƒΠ³ ΠΎΡ‚ Π΄Ρ€ΡƒΠ³Π° благодаря ΡΠΎΠΏΡ€ΠΎΡ‚ΠΈΠ²Π»Π΅Π½ΠΈΡŽ рСзисторов R1 ΠΈ R2 Π½Π΅ Ρ€Π°Π²Π½Ρ‹.
Однако ΠΌΡ‹ Π·Π½Π°Π΅ΠΌ, Ρ‡Ρ‚ΠΎ Ρ‚ΠΎΠΊ, ΠΊΠΎΡ‚ΠΎΡ€Ρ‹ΠΉ ΠΈΠ΄Π΅Ρ‚ Π² Ρ†Π΅ΠΏΡŒ Π² Ρ‚ΠΎΡ‡ΠΊΠ΅ «А» Π΄ΠΎΠ»ΠΆΠ΅Π½ Π²Ρ‹ΠΉΡ‚ΠΈ ΠΈΠ· Ρ†Π΅ΠΏΠΈ Π² Ρ‚ΠΎΡ‡ΠΊΠ΅ Β«BΒ».

ΠŸΠ΅Ρ€Π²ΠΎΠ΅ ΠΏΡ€Π°Π²ΠΈΠ»ΠΎ ΠšΠΈΡ€Ρ…Π³ΠΎΡ„Π° гласит: Β«ΠžΠ±Ρ‰ΠΈΠΉ Ρ‚ΠΎΠΊ, выходящий ΠΈΠ· Ρ†Π΅ΠΏΠΈ Ρ€Π°Π²Π΅Π½ Ρ‚ΠΎΠΊΡƒ входящий Π² Ρ†Π΅ΠΏΡŒΒ».

Π’Π°ΠΊΠΈΠΌ ΠΎΠ±Ρ€Π°Π·ΠΎΠΌ, ΠΏΡ€ΠΎΡ‚Π΅ΠΊΠ°ΡŽΡ‰ΠΈΠΉ ΠΎΠ±Ρ‰ΠΈΠΉ Ρ‚ΠΎΠΊ Π² Ρ†Π΅ΠΏΠΈ ΠΌΠΎΠΆΠ½ΠΎ ΠΎΠΏΡ€Π΅Π΄Π΅Π»ΠΈΡ‚ΡŒ ΠΊΠ°ΠΊ:

Π—Π°Ρ‚Π΅ΠΌ с ΠΏΠΎΠΌΠΎΡ‰ΡŒΡŽ Π·Π°ΠΊΠΎΠ½Π° Ома ΠΌΠΎΠΆΠ½ΠΎ Π²Ρ‹Ρ‡ΠΈΡΠ»ΠΈΡ‚ΡŒ, ΠΊΠΎΡ‚ΠΎΡ€Ρ‹ΠΉ ΠΏΡ€ΠΎΡ‚Π΅ΠΊΠ°Π΅Ρ‚ Ρ‡Π΅Ρ€Π΅Π· ΠΊΠ°ΠΆΠ΄Ρ‹ΠΉ рСзистор:

Π’ΠΎΠΊ, ΠΏΡ€ΠΎΡ‚Π΅ΠΊΠ°ΡŽΡ‰ΠΈΠΉ Π² R1 = U Γ· R1 = 12 Γ· 22 кОм = 0,545 мА

Π’ΠΎΠΊ, ΠΏΡ€ΠΎΡ‚Π΅ΠΊΠ°ΡŽΡ‰ΠΈΠΉ Π² R 2 = U Γ· R2 = 12 Γ· 47 кОм = 0,255 мА

Π’Π°ΠΊΠΈΠΌ ΠΎΠ±Ρ€Π°Π·ΠΎΠΌ, ΠΎΠ±Ρ‰ΠΈΠΉ Ρ‚ΠΎΠΊ Π±ΡƒΠ΄Π΅Ρ‚ Ρ€Π°Π²Π΅Π½:

I = 0,545 мА + 0,255 мА = 0,8 мА

Π­Ρ‚ΠΎ Ρ‚Π°ΠΊΠΆΠ΅ ΠΌΠΎΠΆΠ½ΠΎ ΠΏΡ€ΠΎΠ²Π΅Ρ€ΠΈΡ‚ΡŒ, ΠΈΡΠΏΠΎΠ»ΡŒΠ·ΡƒΡ Π·Π°ΠΊΠΎΠ½ Ома:

I = U Γ· R = 12 Π’ Γ· 15 кОм = 0,8 мА (Ρ‚ΠΎ ΠΆΠ΅ самоС)

Π³Π΄Π΅ 15кОм — это ΠΎΠ±Ρ‰Π΅Π΅ сопротивлСниС Π΄Π²ΡƒΡ… ΠΏΠ°Ρ€Π°Π»Π»Π΅Π»ΡŒΠ½ΠΎ соСдинСнных рСзисторов (22 кОм ΠΈ 47 кОм)

И Π² Π·Π°Π²Π΅Ρ€ΡˆΠ΅Π½ΠΈΠΈ хочСтся ΠΎΡ‚ΠΌΠ΅Ρ‚ΠΈΡ‚ΡŒ, Ρ‡Ρ‚ΠΎ Π±ΠΎΠ»ΡŒΡˆΠΈΠ½ΡΡ‚Π²ΠΎ соврСмСнных рСзисторов ΠΌΠ°Ρ€ΠΊΠΈ ΠΈΠ·ΠΌΠ΅Π½ΡΡŽΡ‚ΡΡ Ρ†Π²Π΅Ρ‚Π½Ρ‹ΠΌΠΈ полосками ΠΈ Π½Π°Π·Π½Π°Ρ‡Π΅Π½ΠΈΠ΅ Π΅Π΅ ΠΌΠΎΠΆΠ½ΠΎ ΡƒΠ·Π½Π°Ρ‚ΡŒ.

ΠŸΠ°Ρ€Π°Π»Π»Π΅Π»ΡŒΠ½ΠΎΠ΅ соСдинСниС рСзисторов — ΠΎΠ½Π»Π°ΠΉΠ½ ΠΊΠ°Π»ΡŒΠΊΡƒΠ»ΡΡ‚ΠΎΡ€

Π§Ρ‚ΠΎΠ±Ρ‹ быстро Π²Ρ‹Ρ‡ΠΈΡΠ»ΠΈΡ‚ΡŒ ΠΎΠ±Ρ‰Π΅Π΅ сопротивлСниС ΠΈ Π±ΠΎΠ»Π΅Π΅ рСзисторов, соСдинСнных ΠΏΠ°Ρ€Π°Π»Π»Π΅Π»ΡŒΠ½ΠΎ, Π²Ρ‹ ΠΌΠΎΠΆΠ΅Ρ‚Π΅ ΠΈΡΠΏΠΎΠ»ΡŒΠ·ΠΎΠ²Π°Ρ‚ΡŒ ΡΠ»Π΅Π΄ΡƒΡŽΡ‰ΠΈΠΉ ΠΎΠ½Π»Π°ΠΉΠ½ ΠΊΠ°Π»ΡŒΠΊΡƒΠ»ΡΡ‚ΠΎΡ€:

ПодвСдСм ΠΈΡ‚ΠΎΠ³

Когда Π΄Π²Π° ΠΈΠ»ΠΈ Π±ΠΎΠ»Π΅Π΅ рСзистора соСдинСны Ρ‚Π°ΠΊ, Ρ‡Ρ‚ΠΎ ΠΎΠ±Π° ΠΎΠ΄Π½ΠΎΠ³ΠΎ рСзистора соСдинСны с Π²Ρ‹Π²ΠΎΠ΄Π°ΠΌΠΈ Π΄Ρ€ΡƒΠ³ΠΎΠ³ΠΎ рСзистора ΠΈΠ»ΠΈ рСзистора, Ρ‚ΠΎ говорят, Ρ‡Ρ‚ΠΎ ΠΎΠ½ΠΈ соСдинСны ΠΌΠ΅ΠΆΠ΄Ρƒ собой. НапряТСниС Π½Π° ΠΊΠ°ΠΆΠ΄ΠΎΠΌ рСзисторС Π²Π½ΡƒΡ‚Ρ€ΠΈ ΠΏΠ°Ρ€Π°Π»Π»Π΅Π»ΡŒΠ½Ρ‹Ρ… ΠΊΠΎΠΌΠ±ΠΈΠ½Π°Ρ†ΠΈΠΉ ΠΎΠ΄ΠΈΠ½Π°ΠΊΠΎΠ²ΠΎΠ³ΠΎ, Π½ΠΎ Ρ‚ΠΎΠΊΠΈ, ΠΏΡ€ΠΎΡ‚Π΅ΠΊΠ°ΡŽΡ‰ΠΈΠ΅ Ρ‡Π΅Ρ€Π΅Π· Π½ΠΈΡ…, ΠΌΠΎΠ³ΡƒΡ‚ ΠΎΡ‚Π»ΠΈΡ‡Π°Ρ‚ΡŒΡΡ Π΄Ρ€ΡƒΠ³ ΠΎΡ‚ Π΄Ρ€ΡƒΠ³Π°, Π² зависимости ΠΎΡ‚ Π²Π΅Π»ΠΈΡ‡ΠΈΠ½Ρ‹ сопротивлСний ΠΊΠ°ΠΆΠ΄ΠΎΠ³ΠΎ рСзистора.

Π­ΠΊΠ²ΠΈΠ²Π°Π»Π΅Π½Ρ‚Π½ΠΎΠ΅ ΠΈΠ»ΠΈ ΠΏΠΎΠ»Π½ΠΎΠ΅ сопротивлСниС ΠΏΠ°Ρ€Π°Π»Π»Π΅Π»ΡŒΠ½ΠΎΠΉ ΠΊΠΎΠΌΠ±ΠΈΠ½Π°Ρ†ΠΈΠΈ всСгда Π±ΡƒΠ΄Π΅Ρ‚ мСньшС минимального сопротивлСния сопротивлСниС Π²Ρ…ΠΎΠ΄ Π² ΠΏΠ°Ρ€Π°Π»Π»Π΅Π»ΡŒΠ½ΠΎΠ΅ соСдинСниС.

1 мОм = 0,001 Ом. 1 кОм = 1 000 = 10³ Ом. 1 МОм = 1 000 000 = 10⁢ Ом.

Π­ΠΊΠ²ΠΈΠ²Π°Π»Π΅Π½Ρ‚Π½ΠΎΠ΅ сопротивлСниС R eq Π³Ρ€ΡƒΠΏΠΏΡ‹ ΠΏΠ°Ρ€Π°Π»Π»Π΅Π»ΡŒΠ½Ρ‹Ρ… соСдинСнных рСзисторов являСтся ΠΎΠ±Ρ€Π°Ρ‚Π½ΠΎΠΉ, ΠΎΠ±Ρ€Π°Ρ‚Π½ΠΎΠΉ суммС Π²Π΅Π»ΠΈΡ‡ΠΈΠ½, ΠΎΠ±Ρ€Π°Ρ‚Π½ΠΎ ΠΏΡ€ΠΎΠΏΠΎΡ€Ρ†ΠΈΠΎΠ½Π°Π»ΡŒΠ½Ρ‹Ρ… сопротивлСний рСзисторов.

Π˜Π½Ρ‹ΠΌΠΈ словами, ΠΏΡ€ΠΎΠ²ΠΎΠ΄ΠΈΠΌΠΎΡΡ‚ΡŒ G
ΠΏΠ°Ρ€Π°Π»Π»Π΅Π»ΡŒΠ½Ρ‹Ρ… рСзисторов, равная суммС проводимостСй этих рСзисторов:

Π­Ρ‚Π° Ρ„ΠΎΡ€ΠΌΡƒΠ»Π° ΠΈΡΠΏΠΎΠ»ΡŒΠ·ΡƒΠ΅Ρ‚ΡΡ Π² Π΄Π°Π½Π½ΠΎΠΌ ΠΊΠ°Π»ΡŒΠΊΡƒΠ»ΡΡ‚ΠΎΡ€Π΅ для расчСтов.НапримСр, ΠΎΠ±Ρ‰Π΅Π΅ сопротивлСниС Ρ‚Ρ€Π΅Ρ… рСзисторов 10, 15 ΠΈ 20 ΠΎΠΌ, соСдинСнных ΠΏΠ°Ρ€Π°Π»Π»Π΅Π»ΡŒΠ½ΠΎ, Ρ€Π°Π²Π½ΠΎ 4,62 Ом:

Если ΡΠΎΠ΅Π΄ΠΈΠ½ΠΈΡ‚ΡŒ Π΄Π²Π° рСзистора ΡƒΠΏΡ€ΠΎΡ‰Π΅Π½Π½ΠΎ:

Если имССтся n
соСдинСнных ΠΎΠ΄ΠΈΠ½Π°ΠΊΠΎΠ²Ρ‹Ρ… рСзисторов R
, Ρ‚ΠΎ ΠΈΡ… эквивалСнтноС сопротивлСниС Π±ΡƒΠ΄Π΅Ρ‚ Ρ€Π°Π²Π½ΠΎ

ΠžΡ‚ΠΌΠ΅Ρ‚ΠΈΠΌ, Ρ‡Ρ‚ΠΎ ΠΎΠ±Ρ‰Π΅Π΅ сопротивлСниС Π³Ρ€ΡƒΠΏΠΏΡ‹ любого количСства ΠΏΠ°Ρ€Π°Π»Π»Π΅Π»ΡŒΠ½Ρ‹Ρ… рСзисторов всСгда Π±ΡƒΠ΄Π΅Ρ‚ мСньшС, Ρ‡Π΅ΠΌ наимСньшСС сопротивлСниС сопротивлСния Π² Π³Ρ€ΡƒΠΏΠΏΠ΅ ΠΈ Π΄ΠΎΠ±Π°Π²Π»Π΅Π½ΠΈΠ΅ Π½ΠΎΠ²ΠΎΠ³ΠΎ рСзистора всСгда ΠΏΡ€ΠΈΠ²Π΅Π΄Π΅Ρ‚ ΠΊ ΡƒΠΌΠ΅Π½ΡŒΡˆΠ΅Π½ΠΈΡŽ эквивалСнтного сопротивлСния.

ВсС рСзисторы, соСдинСнныС ΠΏΠ°Ρ€Π°Π»Π»Π΅Π»ΡŒΠ½ΠΎ находятся ΠΏΠΎΠ΄ ΠΎΠ΄Π½ΠΈΠΌ ΠΈ Ρ‚Π΅ΠΌ ΠΆΠ΅ напряТСниСм. Однако Ρ‚ΠΎΠΊΠΈ, ΠΏΡ€ΠΎΡ‚Π΅ΠΊΠ°ΡŽΡ‰ΠΈΠ΅ Ρ‡Π΅Ρ€Π΅Π· ΠΎΡ‚Π΄Π΅Π»ΡŒΠ½Ρ‹Π΅ рСзисторы, зависят ΠΈ зависят ΠΎΡ‚ ΠΈΡ… сопротивлСния. ΠžΠ±Ρ‰ΠΈΠΉ Ρ‚ΠΎΠΊ Ρ‡Π΅Ρ€Π΅Π· Π³Ρ€ΡƒΠΏΠΏΡƒ рСзисторов Ρ€Π°Π²Π΅Π½ суммС Ρ‚ΠΎΠΊΠΎΠ² Π² ΠΎΡ‚Π΄Π΅Π»ΡŒΠ½Ρ‹Ρ… рСзисторах.

ΠŸΡ€ΠΈ соСдинСнии Π½Π΅ΡΠΊΠΎΠ»ΡŒΠΊΠΈΡ… рСзисторов ΠΏΠ°Ρ€Π°Π»Π»Π΅Π»ΡŒΠ½ΠΎ ΠΈΡ… Π½ΡƒΠΆΠ½ΠΎ ΡƒΡ‡ΠΈΡ‚Ρ‹Π²Π°Ρ‚ΡŒ ΠΈ Ρ€Π°ΡΡΠ΅ΠΈΠ²Π°Ρ‚ΡŒ ΠΌΠΎΡ‰Π½ΠΎΡΡ‚ΡŒ.

ΠŸΡ€ΠΈΠΌΠ΅Ρ€Ρ‹ примСнСния ΠΏΠ°Ρ€Π°Π»Π»Π΅Π»ΡŒΠ½ΠΎΠ³ΠΎ соСдинСния рСзисторов

Одним ΠΈΠ· ΠΏΡ€ΠΈΠΌΠ΅Ρ€ΠΎΠ² ΠΏΠ°Ρ€Π°Π»Π»Π΅Π»ΡŒΠ½ΠΎΠ³ΠΎ соСдинСния являСтся ΡˆΡƒΠ½Ρ‚ Π² ΠΏΡ€ΠΈΠ±ΠΎΡ€Π΅ для измСрСния Ρ‚ΠΎΠΊΠΎΠ², ΠΊΠΎΡ‚ΠΎΡ€Ρ‹Π΅ слишком Π²Π΅Π»ΠΈΠΊΠΈ для Ρ‚ΠΎΠ³ΠΎ, Ρ‡Ρ‚ΠΎΠ±Ρ‹ Π±Ρ‹Ρ‚ΡŒ Π½Π°ΠΏΡ€ΡΠΌΡƒΡŽ ΠΈΠ·ΠΌΠ΅Ρ€Π΅Π½Π½Ρ‹ΠΌΠΈ ΠΏΡ€ΠΈΠ±ΠΎΡ€ΠΎΠΌ, ΠΏΡ€Π΅Π΄Π½Π°Π·Π½Π°Ρ‡Π΅Π½Π½Ρ‹ΠΌ для измСрСния Π½Π΅Π±ΠΎΠ»ΡŒΡˆΠΈΡ… Ρ‚ΠΎΠΊΠΎΠ² ΠΈΠ»ΠΈ напряТСний.Для ΠΈΠ·Π³ΠΎΡ‚ΠΎΠ²Π»Π΅Π½Π½ΠΎΠ³ΠΎ Ρ‚ΠΎΠΊΠ° ΠΏΠΎ Π³Π°Π»ΡŒΠ²Π°Π½ΠΎΠΌΠ΅Ρ‚Ρ€Ρƒ ΠΈΠ»ΠΈ элСктронному ΠΏΡ€ΠΈΠ±ΠΎΡ€Ρƒ, ΠΈΠ·ΠΌΠ΅Ρ€ΡΡŽΡ‰Π΅ΠΌΡƒ Π½Π°ΠΏΡ€ΡΠΆΠ΅Π½ΠΈΡŽ, ΠΏΠΎΠ΄ΠΊΠ»ΡŽΡ‡Π°Π΅Ρ‚ΡΡ рСзистор с ΠΎΡ‡Π΅Π½ΡŒ малСньким сопротивлСниСм, ΠΈΠ·Π³ΠΎΡ‚ΠΎΠ²Π»Π΅Π½Π½Ρ‹ΠΌ ΠΈΠ· ΠΌΠ°Ρ‚Π΅Ρ€ΠΈΠ°Π»Π° со ΡΡ‚Π°Π±ΠΈΠ»ΡŒΠ½Ρ‹ΠΌΠΈ характСристиками. Π­Ρ‚ΠΎΡ‚ рСзистор называСтся ΡˆΡƒΠ½Ρ‚ΠΎΠΌ. Π˜Π·ΠΌΠ΅Ρ€ΡΠ΅ΠΌΡ‹ΠΉ Ρ‚ΠΎΠΊ ΠΏΡ€ΠΎΡ‚Π΅ΠΊΠ°Π΅Ρ‚ Ρ‡Π΅Ρ€Π΅Π· ΡˆΡƒΠ½Ρ‚. Π’ Ρ€Π΅Π·ΡƒΠ»ΡŒΡ‚Π°Ρ‚Π΅ Π½Π° Π½Π΅ΠΌ ΠΏΠ°Π΄Π°Π΅Ρ‚ нСбольшоС напряТСниС, ΠΊΠΎΡ‚ΠΎΡ€ΠΎΠ΅ ΠΈ измСряСтся Π²ΠΎΠ»ΡŒΡ‚ΠΌΠ΅Ρ‚Ρ€ΠΎΠΌ. Π˜Π·ΠΌΠ΅Ρ€Π΅Π½ΠΈΠ΅ напряТСния ΠΏΡ€ΠΎΠΏΠΎΡ€Ρ†ΠΈΠΎΠ½Π°Π»ΡŒΠ½ΠΎ Ρ‚ΠΎΠΊΡƒ, ΠΏΡ€ΠΎΡ‚Π΅ΠΊΠ°ΡŽΡ‰Π΅ΠΌΡƒ Ρ‡Π΅Ρ€Π΅Π· ΡˆΡƒΠ½Ρ‚ с извСстным ΠΈ Ρ‚ΠΎΡ‡Π½Ρ‹ΠΌ сопротивлСниСм, Π²ΠΎΠ»ΡŒΡ‚ΠΌΠ΅Ρ‚Ρ€, ΠΏΠΎΠ΄ΠΊΠ»ΡŽΡ‡Π΅Π½Π½Ρ‹ΠΉ ΠΏΠ°Ρ€Π°Π»Π»Π΅Π»ΡŒΠ½ΠΎ ΡˆΡƒΠ½Ρ‚Ρƒ, ΠΌΠΎΠΆΠ½ΠΎ ΠΏΡ€ΠΎΠ³Ρ€Π°Π΄ΡƒΠΈΡ€ΠΎΠ²Π°Ρ‚ΡŒ нСпосрСдствСнно Π² Π΅Π΄ΠΈΠ½ΠΈΡ†Π°Ρ… Ρ‚ΠΎΠΊΠ° (Π°ΠΌΠΏΠ΅Ρ€Π°Ρ…).

ΠŸΠ°Ρ€Π°Π»Π»Π΅Π»ΡŒΠ½Ρ‹Π΅ ΠΈ ΠΏΠΎΡΠ»Π΅Π΄ΠΎΠ²Π°Ρ‚Π΅Π»ΡŒΠ½Ρ‹Π΅ схСмы ΠΈΡΠΏΠΎΠ»ΡŒΠ·ΡƒΡŽΡ‚ΡΡ для получСния Ρ‚ΠΎΡ‡Π½ΠΎΠ³ΠΎ сопротивлСния ΠΈΠ»ΠΈ рСзистСнтности с Ρ‚Ρ€Π΅Π±ΡƒΠ΅ΠΌΡ‹ΠΌ сопротивлСниСм Π½Π΅Ρ‚ ΠΈΠ»ΠΈ ΠΎΠ½ слишком Π΄ΠΎΡ€ΠΎΠ³, Ссли Π΅Π³ΠΎ ΠΏΡ€ΠΈΠΎΠ±Ρ€Π΅Ρ‚Π°Ρ‚ΡŒ Π² Π½Π΅Π±ΠΎΠ»ΡŒΡˆΠΈΡ… количСствах для массового производства
. НапримСр, Ссли устройство содСрТит ΠΌΠ½ΠΎΠ³ΠΎ рСзисторов ΠΏΠΎ 20 кОм ΠΈ Π½Π΅ΠΎΠ±Ρ…ΠΎΠ΄ΠΈΠΌ Ρ‚ΠΎΠ»ΡŒΠΊΠΎ ΠΎΠ΄ΠΈΠ½ рСзистор 10 кОм. ΠšΠΎΠ½Π΅Ρ‡Π½ΠΎ, нСслоТно Π½Π°ΠΉΡ‚ΠΈ рСзистор Π½Π° 10 кОм. Однако для массового производства ΠΈΠ½ΠΎΠ³Π΄Π° Π±Ρ‹Π²Π°Π΅Ρ‚ Π»ΡƒΡ‡ΡˆΠ΅ ΠΏΠΎΡΡ‚Π°Π²ΠΈΡ‚ΡŒ Π΄Π²Π° рСзистора Π½Π° 20 кОм, Ρ‡Ρ‚ΠΎΠ±Ρ‹ ΠΏΠΎΠ»ΡƒΡ‡ΠΈΡ‚ΡŒ Π½Π΅ΠΎΠ±Ρ…ΠΎΠ΄ΠΈΠΌΡ‹Π΅ 10 кОм.Π­Ρ‚ΠΎ ΠΏΡ€ΠΈΠ²Π΅Π΄Π΅Ρ‚ ΠΊ сниТСнию сСбСстоимости ΠΏΠ΅Ρ‡Π°Ρ‚Π½ΠΎΠΉ ΠΏΠ»Π°Ρ‚Ρ‹, Ρ‚Π°ΠΊ ΠΊΠ°ΠΊ Π±ΡƒΠ΄Π΅Ρ‚ сниТСна оптовая Ρ†Π΅Π½Π° ΠΊΠΎΠΌΠΏΠΎΠ½Π΅Π½Ρ‚ΠΎΠ², Π° Ρ‚Π°ΠΊΠΆΠ΅ ΡΡ‚ΠΎΠΈΠΌΠΎΡΡ‚ΡŒ ΠΌΠΎΠ½Ρ‚Π°ΠΆΠ°, Ρ‚Π°ΠΊ ΠΊΠ°ΠΊ Π±ΡƒΠ΄Π΅Ρ‚ ΡƒΠΌΠ΅Π½ΡŒΡˆΠ΅Π½ΠΎ количСство Ρ‚ΠΈΠΏΠΎΡ€Π°Π·ΠΌΠ΅Ρ€ΠΎΠ² элСмСнтов, ΠΊΠΎΡ‚ΠΎΡ€Ρ‹Π΅ Π΄ΠΎΠ»ΠΆΠ½Ρ‹ ΡƒΡΡ‚Π°Π½ΠΎΠ²ΠΈΡ‚ΡŒ Π½Π° ΠΏΠ»Π°Ρ‚Ρƒ Π°Π²Ρ‚ΠΎΠΌΠ°Ρ‚ установки ΠΊΠΎΠΌΠΏΠΎΠ½Π΅Π½Ρ‚ΠΎΠ².

ΠŸΡ€ΠΎΠ²Π΅Ρ€ΠΈΠΌ ΡΠΏΡ€Π°Π²Π΅Π΄Π»ΠΈΠ²ΠΎΡΡ‚ΡŒ ΠΏΠΎΠΊΠ°Π·Π°Π½Π½Ρ‹Ρ… здСсь Ρ„ΠΎΡ€ΠΌΡƒΠ» Π½Π° простом экспСримСнтС.

Π’ΠΎΠ·ΡŒΠΌΡ‘ΠΌ Π΄Π²Π° рСзистора ΠœΠ›Π’-2
Π½Π° 3
и 47 Ом
ΠΈ соСдиним ΠΈΡ… ΠΏΠΎΡΠ»Π΅Π΄ΠΎΠ²Π°Ρ‚Π΅Π»ΡŒΠ½ΠΎ. Π˜Π·ΠΌΠ΅Ρ€ΠΈΡ‚Π΅Π»ΡŒΠ½ΠΎΠ΅ сопротивлСниС ΠΎΠ±Ρ‰Π΅ΠΉ сопротивляСмости Ρ†Π΅ΠΏΠΈ Ρ†ΠΈΡ„Ρ€ΠΎΠ²Ρ‹ΠΌ ΠΌΡƒΠ»ΡŒΡ‚ΠΈΠΌΠ΅Ρ‚Ρ€ΠΎΠΌ.Как Π²ΠΈΠ΄ΠΈΠΌ ΠΎΠ½ΠΎ Ρ€Π°Π²Π½ΠΎ суммС сопротивлСний рСзисторов, входящих Π² эту Ρ†Π΅ΠΏΠΎΡ‡ΠΊΡƒ.

Π—Π°ΠΌΠ΅Ρ€ ΠΎΠ±Ρ‰Π΅Π³ΠΎ сопротивлСния ΠΏΡ€ΠΈ ΠΏΠΎΡΠ»Π΅Π΄ΠΎΠ²Π°Ρ‚Π΅Π»ΡŒΠ½ΠΎΠΌ соСдинСнии

Π’Π΅ΠΏΠ΅Ρ€ΡŒ соСдиним наши рСзисторы ΠΏΠ°Ρ€Π°Π»Π»Π΅Π»ΡŒΠ½ΠΎ ΠΈ Π·Π°ΠΌΠ΅Ρ€ΠΈΠΌ ΠΈΡ… ΠΎΠ±Ρ‰Π΅Π΅ сопротивлСниС.

Π˜Π·ΠΌΠ΅Ρ€Π΅Π½ΠΈΠ΅ сопротивлСния ΠΏΡ€ΠΈ ΠΏΠ°Ρ€Π°Π»Π»Π΅Π»ΡŒΠ½ΠΎΠΌ соСдинСнии

Как Π²ΠΈΠ΄ΠΈΠΌ, Ρ€Π΅Π·ΡƒΠ»ΡŒΡ‚ΠΈΡ€ΡƒΡŽΡ‰Π΅Π΅ сопротивлСниС (2,9 Ом) мСньшС самого мСньшСго (3 Ом), входящСго Π² Ρ†Π΅ΠΏΠΎΡ‡ΠΊΡƒ. ΠžΡ‚ΡΡŽΠ΄Π° Π²Ρ‹Ρ‚Π΅ΠΊΠ°Π΅Ρ‚ Π΅Ρ‰Ρ‘ ΠΎΠ΄Π½ΠΎ извСстноС ΠΏΡ€Π°Π²ΠΈΠ»ΠΎ, ΠΊΠΎΡ‚ΠΎΡ€ΠΎΠ΅ ΠΌΠΎΠΆΠ½ΠΎ ΠΏΡ€ΠΈΠΌΠ΅Π½ΡΡ‚ΡŒ Π½Π° ΠΏΡ€Π°ΠΊΡ‚ΠΈΠΊΠ΅:

ΠŸΡ€ΠΈ ΠΏΠ°Ρ€Π°Π»Π»Π΅Π»ΡŒΠ½ΠΎΠΌ соСдинСнии рСзисторов ΠΎΠ±Ρ‰Π΅Π΅ сопротивлСниС Ρ†Π΅ΠΏΠΈ Π±ΡƒΠ΄Π΅Ρ‚ мСньшС наимСньшСго сопротивлСния, входящСго Π² эту Ρ†Π΅ΠΏΡŒ.

Π§Ρ‚ΠΎ Π΅Ρ‰Ρ‘ Π½ΡƒΠΆΠ½ΠΎ ΠΏΡ€ΠΈ соСдинСнии рСзисторов?

Π’ΠΎ-ΠΏΠ΅Ρ€Π²Ρ‹Ρ…, ΠΎΠ±ΡΠ·Π°Ρ‚Π΅Π»ΡŒΠ½ΠΎ
учитываСтся ΠΈΡ… номинальная ΠΌΠΎΡ‰Π½ΠΎΡΡ‚ΡŒ. НапримСр, Π½Π°ΠΌ Π½ΡƒΠΆΠ½ΠΎ ΠΏΠΎΠ΄ΠΎΠ±Ρ€Π°Ρ‚ΡŒ Π·Π°ΠΌΠ΅Π½Ρƒ рСзистор Π½Π° 100 Ом
ΠΈ ΠΌΠΎΡ‰Π½ΠΎΡΡ‚ΡŒΡŽ 1 Π’Ρ‚
. Π’ΠΎΠ·ΡŒΠΌΡ‘ΠΌ Π΄Π²Π° рСзистора ΠΏΠΎ 50 Ом ΠΊΠ°ΠΆΠ΄Ρ‹ΠΉ ΠΈ соСдиним ΠΈΡ… ΠΏΠΎΡΠ»Π΅Π΄ΠΎΠ²Π°Ρ‚Π΅Π»ΡŒΠ½ΠΎ. На ΠΊΠ°ΠΊΡƒΡŽ ΠΌΠΎΡ‰Π½ΠΎΡΡ‚ΡŒ рассСяния Π΄ΠΎΠ»ΠΆΠ½Ρ‹ Π±Ρ‹Ρ‚ΡŒ рассчитаны эти Π΄Π²Π° рСзистора?

Ρ‡Π΅Ρ€Π΅Π· соСдинСнныС рСзисторы Ρ‚Π΅Ρ‡Ρ‘Ρ‚ ΠΎΠ΄ΠΈΠ½ ΠΈ Ρ‚ΠΎΡ‚ ΠΆΠ΅ постоянный Ρ‚ΠΎΠΊ (допустим 0,1 А
), сопротивлСниС сопротивлСниС ΠΈΠ· Π½ΠΈΡ… Ρ€Π°Π²Π½ΠΎ 50 Ом
, Ρ‚ΠΎΠ³Π΄Π° ΠΌΠΎΡ‰Π½ΠΎΡΡ‚ΡŒ рассСивания ΠΊΠ°ΠΆΠ΄ΠΎΠ³ΠΎ ΠΈΠ· Π½ΠΈΡ… Π΄ΠΎΠ»ΠΆΠ½Π° Π±Ρ‹Ρ‚ΡŒ Π½Π΅ ΠΌΠ΅Π½Π΅Π΅ 0,5 Π’Ρ‚
.Π’ Ρ€Π΅Π·ΡƒΠ»ΡŒΡ‚Π°Ρ‚Π΅ Π½Π° ΠΊΠ°ΠΆΠ΄ΠΎΠΌ ΠΈΠ· Π½ΠΈΡ… выдСлится ΠΏΠΎ 0,5 Π’Ρ‚
мощности. Π’ суммС это ΠΈ Π±ΡƒΠ΄Π΅Ρ‚ Ρ‚ΠΎΡ‚ самый 1 Π’Ρ‚
.

Π”Π°Π½Π½Ρ‹ΠΉ ΠΏΡ€ΠΈΠΌΠ΅Ρ€ достаточно Π³Ρ€ΡƒΠ±ΠΎΠ²Π°Ρ‚. ΠŸΠΎΡΡ‚ΠΎΠΌΡƒ, Ссли Π΅ΡΡ‚ΡŒ сомнСния, стоит Π±Ρ€Π°Ρ‚ΡŒ рСзисторы с запасом ΠΏΠΎ мощности.

ΠŸΠΎΠ΄Ρ€ΠΎΠ±Π½Π΅Π΅ ΠΎ мощности рассСивания рСзистора Ρ‡ΠΈΡ‚Π°ΠΉΡ‚Π΅.

Π’ΠΎ-Π²Ρ‚ΠΎΡ€Ρ‹Ρ…, ΠΏΡ€ΠΈ соСдинСнии стоит ΠΈΡΠΏΠΎΠ»ΡŒΠ·ΠΎΠ²Π°Ρ‚ΡŒ ΠΎΠ΄Π½ΠΎΡ‚ΠΈΠΏΠ½Ρ‹Π΅ рСзисторы, Π½Π°ΠΏΡ€ΠΈΠΌΠ΅Ρ€, сСрии ΠœΠ›Π’. ΠšΠΎΠ½Π΅Ρ‡Π½ΠΎ, Π½Π΅Ρ‚ Π½ΠΈΡ‡Π΅Π³ΠΎ ΠΏΠ»ΠΎΡ…ΠΎΠ³ΠΎ Π² Ρ‚ΠΎΠΌ, Ρ‡Ρ‚ΠΎΠ±Ρ‹ Π±Ρ€Π°Ρ‚ΡŒ Ρ€Π°Π·Π½Ρ‹Π΅. Π­Ρ‚ΠΎ лишь рСкомСндация.

ΠŸΠ°Ρ€Π°Π»Π»Π΅Π»ΡŒΠ½ΠΎΠ΅ соСдинСниС рСзисторов, наряду с ΠΏΠΎΡΠ»Π΅Π΄ΠΎΠ²Π°Ρ‚Π΅Π»ΡŒΠ½Ρ‹ΠΌ, являСтся основным способом соСдинСния элСмСнтов Π² элСктричСской Ρ†Π΅ΠΏΠΈ.Π’ΠΎ Π²Ρ‚ΠΎΡ€ΠΎΠΌ Π²Π°Ρ€ΠΈΠ°Π½Ρ‚Π΅ всС элСмСнты установлСны ΡΠ»Π΅Π΄ΡƒΡŽΡ‰ΠΈΠΌ ΠΎΠ±Ρ€Π°Π·ΠΎΠΌ: ΠΊΠΎΠ½Π΅Ρ† ΠΎΠ΄Π½ΠΎΠ³ΠΎ соСдинСн с ΡΠ»Π΅Π΄ΡƒΡŽΡ‰ΠΈΠΌ. Π’ Ρ‚Π°ΠΊΠΎΠΉ схСмС сила Ρ‚ΠΎΠΊΠ° Π½Π° всСх элСмСнтах одинаковая, ΠΏΠ°Π΄Π΅Π½ΠΈΠ΅ напряТСний зависит ΠΎΡ‚ сопротивлСния ΠΊΠ°ΠΆΠ΄ΠΎΠ³ΠΎ элСмСнта. Π’ ΠΏΠΎΡΠ»Π΅Π΄ΠΎΠ²Π°Ρ‚Π΅Π»ΡŒΠ½ΠΎΠΌ соСдинСнии Π΅ΡΡ‚ΡŒ Π΄Π²Π° ΡƒΠ·Π»Π°. К ΠΎΠ΄Π½ΠΎΠΌΡƒ ΠΏΠΎΠ΄ΠΊΠ»ΡŽΡ‡Π΅Π½Ρ‹ Π½Π°Ρ‡Π°Π»Π° всСх элСмСнтов, ΠΊΠΎ Π²Ρ‚ΠΎΡ€ΠΎΠΌΡƒ ΠΈΡ… ΠΊΠΎΠ½Ρ†Ρ‹. Условно для постоянного Ρ‚ΠΎΠΊΠ° ΠΌΠΎΠΆΠ½ΠΎ ΠΎΠ±ΠΎΠ·Π½Π°Ρ‡ΠΈΡ‚ΡŒ ΠΈΡ… ΠΊΠ°ΠΊ плюс ΠΈ минус, Π° для ΠΏΠ΅Ρ€Π΅ΠΌΠ΅Π½Π½ΠΎΠ³ΠΎ ΠΊΠ°ΠΊ Ρ„Π°Π·Ρƒ ΠΈ ноль. Благодаря своим особСнностям Π½Π°Ρ…ΠΎΠ΄ΠΈΡ‚ ΡˆΠΈΡ€ΠΎΠΊΠΎΠ΅ ΠΏΡ€ΠΈΠΌΠ΅Π½Π΅Π½ΠΈΠ΅ Π² элСктричСских схСмах, Π² Ρ‚ΠΎΠΌ числС ΠΈ со ΡΠΌΠ΅ΡˆΠ°Π½Π½Ρ‹ΠΌ соСдинСниСм.Бвойства ΠΎΠ΄ΠΈΠ½Π°ΠΊΠΎΠ²Ρ‹ для постоянного ΠΈ ΠΏΠ΅Ρ€Π΅ΠΌΠ΅Π½Π½ΠΎΠ³ΠΎ Ρ‚ΠΎΠΊΠ°.

РасчСт ΠΎΠ±Ρ‰Π΅Π³ΠΎ сопротивлСния ΠΏΡ€ΠΈ ΠΏΠ°Ρ€Π°Π»Π»Π΅Π»ΡŒΠ½ΠΎΠΌ соСдинСнии рСзисторов

Π’ ΠΎΡ‚Π»ΠΈΡ‡ΠΈΠ΅ ΠΎΡ‚ ΠΏΠΎΡΠ»Π΅Π΄ΠΎΠ²Π°Ρ‚Π΅Π»ΡŒΠ½ΠΎΠ³ΠΎ соСдинСния, Π³Π΄Π΅ нахоТдСния ΠΎΠ±Ρ‰Π΅Π³ΠΎ сопротивлСния достаточно ΡΠ»ΠΎΠΆΠΈΡ‚ΡŒ Π·Π½Π°Ρ‡Π΅Π½ΠΈΠ΅ ΠΊΠ°ΠΆΠ΄ΠΎΠ³ΠΎ элСмСнта, для ΠΏΠ°Ρ€Π°Π»Π»Π΅Π»ΡŒΠ½ΠΎΠ³ΠΎ Ρ‚ΠΎ ΠΆΠ΅ самоС Π±ΡƒΠ΄Π΅Ρ‚ справСдливо для проводимости. Как ΠΎΠ½Π° ΠΏΡ€ΠΎΠΏΠΎΡ€Ρ†ΠΈΠΎΠ½Π°Π»ΡŒΠ½Π° ΡΠΎΠΏΡ€ΠΎΡ‚ΠΈΠ²Π»Π΅Π½ΠΈΡŽ, ΠΏΠΎΠ»ΡƒΡ‡ΠΈΠΌ Ρ„ΠΎΡ€ΠΌΡƒΠ»Ρƒ, ΠΏΡ€Π΅Π΄ΡΡ‚Π°Π²Π»Π΅Π½Π½ΡƒΡŽ Π½ΠΈΠΆΠ΅:

НСобходимо ΠΎΡ‚ΠΌΠ΅Ρ‚ΠΈΡ‚ΡŒ ΠΎΠ΄Π½Ρƒ ΠΎΡΠΎΠ±Π΅Π½Π½ΠΎΡΡ‚ΡŒ расчСта ΠΏΠ°Ρ€Π°Π»Π»Π΅Π»ΡŒΠ½ΠΎΠ³ΠΎ соСдинСния рСзисторов: ΠΎΠ±Ρ‰Π΅Π΅ Π·Π½Π°Ρ‡Π΅Π½ΠΈΠ΅ Π±ΡƒΠ΄Π΅Ρ‚ всСгда мСньшС, Ρ‡Π΅ΠΌ самоС малСнькоС ΠΈΠ· Π½ΠΈΡ….Для рСзисторов справСдливо ΠΊΠ°ΠΊ для постоянного, Ρ‚Π°ΠΊ ΠΈ для ΠΏΠ΅Ρ€Π΅ΠΌΠ΅Π½Π½ΠΎΠ³ΠΎ Ρ‚ΠΎΠΊΠ°. ΠšΠ°Ρ‚ΡƒΡˆΠΊΠΈ ΠΈ кондСнсаторы ΠΈΠΌΠ΅ΡŽΡ‚ свои особСнности.

Π‘ΠΈΠ»Π° Ρ‚ΠΎΠΊΠ° ΠΈ напряТСниС

ΠŸΡ€ΠΈ расчСтС ΠΏΠ°Ρ€Π°Π»Π»Π΅Π»ΡŒΠ½ΠΎΠ³ΠΎ сопротивлСния рСзисторов Π½Π΅ΠΎΠ±Ρ…ΠΎΠ΄ΠΈΠΌΠΎ Π·Π½Π°Ρ‚ΡŒ, ΠΊΠ°ΠΊ Ρ€Π°ΡΡΡ‡ΠΈΡ‚Π°Ρ‚ΡŒ напряТСниС ΠΈ силу Ρ‚ΠΎΠΊΠ°. Π’ этом случаС Π½Π°ΠΌ ΠΏΠΎΠΌΠΎΠΆΠ΅Ρ‚ Π·Π°ΠΊΠΎΠ½ Ома, ΠΎΠΏΡ€Π΅Π΄Π΅Π»ΡΡŽΡ‰ΠΈΠΉ связь ΠΌΠ΅ΠΆΠ΄Ρƒ сопротивлСниСм, силой Ρ‚ΠΎΠΊΠ° ΠΈ напряТСниСм.

Π˜ΡΡ…ΠΎΠ΄Ρ ΠΈΠ· ΠΏΠ΅Ρ€Π²ΠΎΠΉ Ρ„ΠΎΡ€ΠΌΡƒΠ»ΠΈΡ€ΠΎΠ²ΠΊΠΈ Π·Π°ΠΊΠΎΠ½Π° ΠšΠΈΡ€Ρ…Π³ΠΎΡ„Π°, ΠΏΠΎΠ»ΡƒΡ‡ΠΈΠΌ, Ρ‡Ρ‚ΠΎ сумма сходящихся Π² ΠΎΠ΄Π½ΠΎΠΌ ΡƒΠ·Π»Π΅ Ρ‚ΠΎΠΊΠΎΠ² Ρ€Π°Π²Π½Π° Π½ΡƒΠ»ΡŽ.НаправлСниС Π²Ρ‹Π±ΠΈΡ€Π°Π΅ΠΌ ΠΏΠΎ Π½Π°ΠΏΡ€Π°Π²Π»Π΅Π½ΠΈΡŽ протСкания Ρ‚ΠΎΠΊΠ°. Π’Π°ΠΊΠΈΠΌ ΠΎΠ±Ρ€Π°Π·ΠΎΠΌ, ΠΏΠΎΠ»ΠΎΠΆΠΈΡ‚Π΅Π»ΡŒΠ½Ρ‹ΠΌ Π½Π°ΠΏΡ€Π°Π²Π»Π΅Π½ΠΈΠ΅ΠΌ для ΠΏΠ΅Ρ€Π²ΠΎΠ³ΠΎ ΡƒΠ·Π»Π° ΠΌΠΎΠΆΠ½ΠΎ ΡΡ‡ΠΈΡ‚Π°Ρ‚ΡŒ входящий Ρ‚ΠΎΠΊ ΠΎΡ‚ источника питания. ΠžΡ‚Ρ€ΠΈΡ†Π°Ρ‚Π΅Π»ΡŒΠ½Ρ‹Π΅ ΠΎΡ‚Ρ€ΠΈΡ†Π°Ρ‚Π΅Π»ΡŒΠ½Ρ‹Π΅ отходящиС ΠΈΠ· ΠΊΠ°ΠΆΠ΄ΠΎΠ³ΠΎ рСзистора. Для Π²Ρ‚ΠΎΡ€ΠΎΠ³ΠΎ ΡƒΠ·Π»Π° ΠΊΠ°Ρ€Ρ‚ΠΈΠ½Π° ΠΏΡ€ΠΎΡ‚ΠΈΠ²ΠΎΠΏΠΎΠ»ΠΎΠΆΠ½Π°. Π˜ΡΡ…ΠΎΠ΄Ρ ΠΈΠ· Ρ„ΠΎΡ€ΠΌΡƒΠΊΠΈ Π·Π°ΠΊΠΎΠ½Π°, ΠΏΠΎΠ»ΡƒΡ‡ΠΈΠΌ, Ρ‡Ρ‚ΠΎ суммарный Ρ‚ΠΎΠΊ Ρ€Π°Π²Π΅Π½ суммС Ρ‚ΠΎΠΊΠΎΠ², проходящих Ρ‡Π΅Ρ€Π΅Π· ΠΊΠ°ΠΆΠ΄Ρ‹ΠΉ соСдинСнный рСзистор.

Π˜Ρ‚ΠΎΠ³ΠΎΠ²ΠΎΠ΅ напряТСниС ΠΆΠ΅ опрСдСляСтся ΠΏΠΎ Π²Ρ‚ΠΎΡ€ΠΎΠΌΡƒ Π·Π°ΠΊΠΎΠ½Ρƒ ΠšΠΈΡ€Ρ…Π³ΠΎΡ„Π°. Оно ΠΎΠ΄ΠΈΠ½Π°ΠΊΠΎΠ²ΠΎ для ΠΊΠ°ΠΆΠ΄ΠΎΠ³ΠΎ рСзистора ΠΈ Ρ€Π°Π²Π½ΠΎ ΠΎΠ±Ρ‰Π΅ΠΌΡƒ.Π­Ρ‚Π° ΠΎΡΠΎΠ±Π΅Π½Π½ΠΎΡΡ‚ΡŒ ΠΈΡΠΏΠΎΠ»ΡŒΠ·ΡƒΠ΅Ρ‚ΡΡ для ΠΏΠΎΠ΄ΠΊΠ»ΡŽΡ‡Π΅Π½ΠΈΡ Ρ€ΠΎΠ·Π΅Ρ‚ΠΎΠΊ ΠΈ освСщСния Π² ΠΊΠ²Π°Ρ€Ρ‚ΠΈΡ€Π°Ρ….

ΠŸΡ€ΠΈΠΌΠ΅Ρ€ расчСта

Π’ качСствС ΠΏΠ΅Ρ€Π²ΠΎΠ³ΠΎ ΠΏΡ€ΠΈΠΌΠ΅Ρ€Π° ΠΏΡ€ΠΈΠ²Π΅Π΄Π΅ΠΌ расчСт сопротивлСния ΠΏΡ€ΠΈ ΠΏΠ°Ρ€Π°Π»Π»Π΅Π»ΡŒΠ½ΠΎΠΌ соСдинСнии ΠΎΠ΄ΠΈΠ½Π°ΠΊΠΎΠ²Ρ‹Ρ… рСзисторов. Π‘ΠΈΠ»Π° Ρ‚ΠΎΠΊΠ°, ΠΏΡ€ΠΎΡ‚Π΅ΠΊΠ°ΡŽΡ‰Π°Ρ Ρ‡Π΅Ρ€Π΅Π· Π½ΠΈΡ…, Π±ΡƒΠ΄Π΅Ρ‚ ΠΎΠ΄ΠΈΠ½Π°ΠΊΠΎΠ²ΠΎΠΉ. ΠŸΡ€ΠΈΠΌΠ΅Ρ€ расчСта сопротивлСния выглядит Ρ‚Π°ΠΊ:

ΠžΠ±Ρ‰Π΅Π΅ сопротивлСниС Π² Π΄Π²Π° Ρ€Π°Π·Π° Π½ΠΈΠΆΠ΅, Ρ‡Π΅ΠΌ ΠΊΠ°ΠΆΠ΄ΠΎΠ΅ ΠΈΠ· Π½ΠΈΡ…. Π­Ρ‚ΠΎ соотвСтствуСт Ρ‚ΠΎΠΌΡƒ, Ρ‡Ρ‚ΠΎ суммарная сила Ρ‚ΠΎΠΊΠ° Π² Π΄Π²Π° Ρ€Π°Π·Π° Π²Ρ‹ΡˆΠ΅, Ρ‡Π΅ΠΌ Ρƒ ΠΎΠ΄Π½ΠΎΠ³ΠΎ.А Ρ‚Π°ΠΊΠΆΠ΅ прСкрасно соотносится с Ρ„ΡƒΠ½ΠΊΡ†ΠΈΠ΅ΠΉ проводимости Π² Π΄Π²Π° Ρ€Π°Π·Π°.

Π’Ρ‚ΠΎΡ€ΠΎΠΉ ΠΏΡ€ΠΈΠΌΠ΅Ρ€

Рассмотрим ΠΏΡ€ΠΈΠΌΠ΅Ρ€ ΠΏΠ°Ρ€Π°Π»Π»Π΅Π»ΡŒΠ½ΠΎΠ³ΠΎ соСдинСния Ρ‚Ρ€Π΅Ρ… рСзисторов. Для расчСта ΠΈΡΠΏΠΎΠ»ΡŒΠ·ΡƒΠ΅ΠΌ ΡΡ‚Π°Π½Π΄Π°Ρ€Ρ‚Π½ΡƒΡŽ Ρ„ΠΎΡ€ΠΌΡƒΠ»Ρƒ:

ΠŸΠΎΡ…ΠΎΠΆΠΈΠΌ ΠΎΠ±Ρ€Π°Π·ΠΎΠΌ Π²Ρ‹ΠΏΠΎΠ»Π½ΡΡŽΡ‚ΡΡ схСмы с большим ΠΏΠ°Ρ€Π°Π»Π»Π΅Π»ΡŒΠ½Ρ‹ΠΌ соСдинСниСм рСзисторов.

ΠŸΡ€ΠΈΠΌΠ΅Ρ€ смСшанного соСдинСния

Для смСшанного соСдинСния, Π½Π°ΠΏΡ€ΠΈΠΌΠ΅Ρ€, прСдставлСнного Π½ΠΈΠΆΠ΅, расчСт Π±ΡƒΠ΄Π΅Ρ‚ ΠΏΡ€ΠΎΠΈΠ·Π²ΠΎΠ΄ΠΈΡ‚ΡŒΡΡ Π² нСсколько этапов.

Для Π½Π°Ρ‡Π°Π»Π° ΠΏΠΎΡΠ»Π΅Π΄ΠΎΠ²Π°Ρ‚Π΅Π»ΡŒΠ½Ρ‹Ρ… элСмСнтов ΠΌΠΎΠΆΠ½ΠΎ условно Π·Π°ΠΌΠ΅Π½ΠΈΡ‚ΡŒ ΠΎΠ΄Π½ΠΈΠΌ рСзистором, ΠΎΠ±Π»Π°Π΄Π°ΡŽΡ‰ΠΈΠΌ сопротивлСниСм, Ρ€Π°Π²Π½Ρ‹ΠΌ суммС Π΄Π²ΡƒΡ… замСняСмых. Π”Π°Π»Π΅Π΅ ΠΎΠ±Ρ‰Π΅Π΅ сопротивлСниС Ρ‚Π΅ΠΌ ΠΆΠ΅ способом. Π”Π°Π½Π½Ρ‹ΠΉ ΠΌΠ΅Ρ‚ΠΎΠ΄ ΠΏΠΎΠ΄ΠΎΠΉΠ΄Π΅Ρ‚ ΠΈ для Π΄Ρ€ΡƒΠ³ΠΈΡ… Π±ΠΎΠ»Π΅Π΅ слоТных схСм. ΠŸΠΎΡΠ»Π΅Π΄ΠΎΠ²Π°Ρ‚Π΅Π»ΡŒΠ½ΠΎ упрощСнная схСма, ΠΌΠΎΠΆΠ½ΠΎ ΠΏΠΎΠ»ΡƒΡ‡ΠΈΡ‚ΡŒ Π½Π΅ΠΎΠ±Ρ…ΠΎΠ΄ΠΈΠΌΠΎΠ΅ Π·Π½Π°Ρ‡Π΅Π½ΠΈΠ΅.

НапримСр, Ссли вмСсто рСзистора R3 Π±ΡƒΠ΄ΡƒΡ‚ ΠΏΠΎΠ΄ΠΊΠ»ΡŽΡ‡Π΅Π½Ρ‹ Π΄Π²Π° ΠΏΠ°Ρ€Π°Π»Π»Π΅Π»ΡŒΠ½Ρ‹Ρ…, сначала потрСбуСтся ΠΈΡ… сопротивлСниС, Π·Π°ΠΌΠ΅Π½ΠΈΠ² ΠΈΡ… эквивалСнтным.А Π΄Π°Π»Π΅Π΅ Ρ‚ΠΎ ΠΆΠ΅ самоС, Ρ‡Ρ‚ΠΎ ΠΈ Π² ΠΏΡ€ΠΈΠΌΠ΅Ρ€Π΅ Π²Ρ‹ΡˆΠ΅.

ΠŸΡ€ΠΈΠΌΠ΅Π½Π΅Π½ΠΈΠ΅ ΠΏΠ°Ρ€Π°Π»Π»Π΅Π»ΡŒΠ½ΠΎΠΉ схСмы

ΠŸΠ°Ρ€Π°Π»Π»Π΅Π»ΡŒΠ½ΠΎΠ΅ соСдинСниС рСзисторов Π½Π°Ρ…ΠΎΠ΄ΠΈΡ‚ своС ΠΏΡ€ΠΈΠΌΠ΅Π½Π΅Π½ΠΈΠ΅ Π²ΠΎ ΠΌΠ½ΠΎΠ³ΠΈΡ… случаях. ΠŸΠΎΡΠ»Π΅Π΄ΠΎΠ²Π°Ρ‚Π΅Π»ΡŒΠ½ΠΎΠ΅ ΠΏΠΎΠ΄ΠΊΠ»ΡŽΡ‡Π΅Π½ΠΈΠ΅ сопротивлСния, Π° для нашСго случая ΠΎΠ½ΠΎ ΡƒΠΌΠ΅Π½ΡŒΡˆΠΈΡ‚ΡΡ. НапримСр, для элСктричСской Ρ†Π΅ΠΏΠΈ трСбуСтся сопротивлСниС Π² 5 Ом, Π½ΠΎ Π΅ΡΡ‚ΡŒ Ρ‚ΠΎΠ»ΡŒΠΊΠΎ рСзисторы Π½Π° 10 Ом ΠΈ Π²Ρ‹ΡˆΠ΅. Из ΠΏΠ΅Ρ€Π²ΠΎΠ³ΠΎ ΠΏΡ€ΠΈΠΌΠ΅Ρ€Π° ΠΌΡ‹ Π·Π½Π°Π΅ΠΌ, Ρ‡Ρ‚ΠΎ ΠΌΠΎΠΆΠ½ΠΎ ΠΏΠΎΠ»ΡƒΡ‡ΠΈΡ‚ΡŒ Π² Π΄Π²Π° Ρ€Π°Π·Π° мСньшСС Π·Π½Π°Ρ‡Π΅Π½ΠΈΠ΅ сопротивлСния, Ссли ΡƒΡΡ‚Π°Π½ΠΎΠ²ΠΈΡ‚ΡŒ Π΄Π²Π° ΠΎΠ΄ΠΈΠ½Π°ΠΊΠΎΠ²Ρ‹Ρ… рСзистора ΠΏΠ°Ρ€Π°Π»Π»Π΅Π»ΡŒΠ½ΠΎ Π΄Ρ€ΡƒΠ³Ρƒ.

Π£ΠΌΠ΅Π½ΡŒΡˆΠΈΡ‚ΡŒ сопротивлСниС ΠΌΠΎΠΆΠ½ΠΎ Π΅Ρ‰Π΅ большС, Π½Π°ΠΏΡ€ΠΈΠΌΠ΅Ρ€, Ссли Π΄Π²Π΅ ΠΏΠ°Ρ€Ρ‹ ΠΏΠ°Ρ€Π°Π»Π»Π΅Π»ΡŒΠ½ΠΎ соСдинСнных рСзисторов ΡΠΎΠ΅Π΄ΠΈΠ½ΠΈΡ‚ΡŒ ΠΎΡ‚Π½ΠΎΡΠΈΡ‚Π΅Π»ΡŒΠ½ΠΎ Π΄Ρ€ΡƒΠ³ Π΄Ρ€ΡƒΠ³Π°. МоТно ΡƒΠ²Π΅Π»ΠΈΡ‡ΠΈΡ‚ΡŒ сопротивлСниС Π΅Ρ‰Π΅ Π² Π΄Π²Π° Ρ€Π°Π·Π°, Ссли рСзисторы ΠΈΠΌΠ΅ΡŽΡ‚ ΠΎΠ΄ΠΈΠ½Π°ΠΊΠΎΠ²ΠΎΠ΅ сопротивлСниС. ΠšΠΎΠΌΠ±ΠΈΠ½ΠΈΡ€ΡƒΡ с ΠΏΠΎΡΠ»Π΅Π΄ΠΎΠ²Π°Ρ‚Π΅Π»ΡŒΠ½Ρ‹ΠΌ соСдинСниСм, ΠΌΠΎΠΆΠ½ΠΎ ΠΏΠΎΠ»ΡƒΡ‡ΠΈΡ‚ΡŒ любоС Π·Π½Π°Ρ‡Π΅Π½ΠΈΠ΅.

Π’Ρ‚ΠΎΡ€ΠΎΠΉ ΠΏΡ€ΠΈΠΌΠ΅Ρ€ — это использованиС ΠΏΠ°Ρ€Π°Π»Π»Π΅Π»ΡŒΠ½ΠΎΠ³ΠΎ ΠΏΠΎΠ΄ΠΊΠ»ΡŽΡ‡Π΅Π½ΠΈΡ для освСщСния ΠΈ Ρ€ΠΎΠ·Π΅Ρ‚ΠΎΠΊ Π² ΠΊΠ²Π°Ρ€Ρ‚ΠΈΡ€Π°Ρ…. Благодаря Ρ‚Π°ΠΊΠΎΠΌΡƒ ΠΏΠΎΠ΄ΠΊΠ»ΡŽΡ‡Π΅Π½ΠΈΡŽ напряТСния Π½Π° ΠΊΠ°ΠΆΠ΄ΠΎΠΌ элСмСнтС Π½Π΅ Π±ΡƒΠ΄Π΅Ρ‚ Π·Π°Π²ΠΈΡΠ΅Ρ‚ΡŒ ΠΎΡ‚ ΠΈΡ… количСства ΠΈ Π±ΡƒΠ΄Π΅Ρ‚ ΠΎΠ΄ΠΈΠ½Π°ΠΊΠΎΠ²Ρ‹ΠΌ.

Π•Ρ‰Π΅ ΠΎΠ΄ΠΈΠ½ ΠΏΡ€ΠΈΠΌΠ΅Ρ€ использования ΠΏΠ°Ρ€Π°Π»Π»Π΅Π»ΡŒΠ½ΠΎΠ³ΠΎ ΠΏΠΎΠ΄ΠΊΠ»ΡŽΡ‡Π΅Π½ΠΈΡ — это Π·Π°Ρ‰ΠΈΡ‚Π½ΠΎΠ΅ Π·Π°Π·Π΅ΠΌΠ»Π΅Π½ΠΈΠ΅ элСктрооборудования. НапримСр, Ссли Ρ‡Π΅Π»ΠΎΠ²Π΅ΠΊ касаСтся мСталличСского корпуса ΠΏΡ€ΠΈΠ±ΠΎΡ€Π°, ΠΊΠΎΡ‚ΠΎΡ€Ρ‹ΠΉ Π½Π° самом Π΄Π΅Π»Π΅ происходит ΠΏΡ€ΠΎΠ±ΠΎΠΉ, получится ΠΏΠ°Ρ€Π°Π»Π»Π΅Π»ΡŒΠ½ΠΎΠ΅ соСдинСниС Π΅Π³ΠΎ ΠΈ Π·Π°Ρ‰ΠΈΡ‚Π½ΠΎΠ³ΠΎ ΠΏΡ€ΠΎΠ²ΠΎΠ΄Π½ΠΈΠΊΠ°. ΠŸΠ΅Ρ€Π²Ρ‹ΠΌ ΡƒΠ·Π»ΠΎΠΌ Π±ΡƒΠ΄Π΅Ρ‚ мСсто прикосновСния, Π° Π²Ρ‚ΠΎΡ€Ρ‹ΠΌ нулСвая Ρ‚ΠΎΡ‡ΠΊΠ° трансформатора. По ΠΏΡ€ΠΎΠ²ΠΎΠ΄Π½ΠΈΠΊΡƒ ΠΈ Ρ‡Π΅Π»ΠΎΠ²Π΅ΠΊΡƒ Π±ΡƒΠ΄Π΅Ρ‚ Ρ‚Π΅Ρ‡ΡŒ Ρ€Π°Π·Π½Ρ‹ΠΉ Ρ‚ΠΎΠΊ. Π’Π΅Π»ΠΈΡ‡ΠΈΠ½Ρƒ сопротивлСния ΠΏΡ€ΠΈΠ½ΠΈΠΌΠ°ΡŽΡ‚ Π·Π° 1000 Ом, хотя Ρ€Π΅Π°Π»ΡŒΠ½ΠΎΠ΅ Π·Π½Π°Ρ‡Π΅Π½ΠΈΠ΅ Π·Π°Ρ‡Π°ΡΡ‚ΡƒΡŽ Π³ΠΎΡ€Π°Π·Π΄ΠΎ большС.Если Π±Ρ‹ Π½Π΅ Π±Ρ‹Π»ΠΎ зазСмлСния, вСсь Ρ‚ΠΎΠΊ, ΠΏΡ€ΠΎΡ‚Π΅ΠΊΠ°ΡŽΡ‰ΠΈΠΉ Π² схСмС, пошСл Π±Ρ‹ Ρ‡Π΅Ρ€Π΅Π· Ρ‡Π΅Π»ΠΎΠ²Π΅ΠΊΠ°, Ρ‚Π°ΠΊ ΠΊΠ°ΠΊ ΠΎΠ½ Π±Ρ‹Π» Π±Ρ‹ СдинствСнным ΠΏΡ€ΠΎΠ²ΠΎΠ΄Π½ΠΈΠΊΠΎΠΌ.

ΠŸΠ°Ρ€Π°Π»Π»Π΅Π»ΡŒΠ½ΠΎΠ΅ соСдинСниС мСстноС ΠΈ для Π±Π°Ρ‚Π°Ρ€Π΅ΠΉΠΊΠΈ. НапряТСниС ΠΏΡ€ΠΈ этом остаСтся ΠΏΡ€Π΅ΠΆΠ½ΠΈΠΌ, ΠΎΠ΄Π½Π°ΠΊΠΎ Π² Π΄Π²Π° Ρ€Π°Π·Π° возрастаСт ΠΈΡ… Π΅ΠΌΠΊΠΎΡΡ‚ΡŒ.

Π˜Ρ‚ΠΎΠ³

ΠŸΡ€ΠΈ ΠΏΠΎΠ΄ΠΊΠ»ΡŽΡ‡Π΅Π½ΠΈΠΈ рСзисторов ΠΏΠ°Ρ€Π°Π»Π»Π΅Π»ΡŒΠ½ΠΎ, напряТСниС Π½Π° Π½ΠΈΡ… Π±ΡƒΠ΄Π΅Ρ‚ ΠΎΠ΄ΠΈΠ½Π°ΠΊΠΎΠ²Ρ‹ΠΌ, Π° Ρ‚ΠΎΠΊ Ρ€Π°Π²Π΅Π½ суммС ΠΏΡ€ΠΎΡ‚Π΅ΠΊΠ°ΡŽΡ‰ΠΈΡ… Ρ‡Π΅Ρ€Π΅Π· ΠΊΠ°ΠΆΠ΄Ρ‹ΠΉ рСзистор. ΠŸΡ€ΠΎΠ²ΠΎΠ΄ΠΈΠΌΠΎΡΡ‚ΡŒ Π±ΡƒΠ΄Π΅Ρ‚ Ρ€ΠΎΠ²Π½ΡΡ‚ΡŒΡΡ суммС ΠΊΠ°ΠΆΠ΄ΠΎΠ³ΠΎ. ΠžΡ‚ этого ΠΈ получаСтся нСобычная Ρ„ΠΎΡ€ΠΌΡƒΠ»Π° суммарного сопротивлСния рСзисторов.

Π΄ΠΎΠ»ΠΆΠ½ΠΎ Π±Ρ‹Ρ‚ΡŒ самого малСнького самого малСнького сопротивлСния. Π­Ρ‚ΠΎ Ρ‚Π°ΠΊΠΆΠ΅ ΠΌΠΎΠΆΠ½ΠΎ ΠΎΠ±ΡŠΡΡΠ½ΠΈΡ‚ΡŒ суммированиСм проводимости рСзисторов. ПослСдняя Π±ΡƒΠ΄Π΅Ρ‚ Π²ΠΎΠ·Ρ€Π°ΡΡ‚Π°Ρ‚ΡŒ ΠΏΡ€ΠΈ Π΄ΠΎΠ±Π°Π²Π»Π΅Π½ΠΈΠΈ Π½ΠΎΠ²Ρ‹Ρ… элСмСнтов, соотвСтствСнно ΠΈ ΠΏΡ€ΠΎΠ²ΠΎΠ΄ΠΈΠΌΠΎΡΡ‚ΡŒ Π±ΡƒΠ΄Π΅Ρ‚ ΡƒΠΌΠ΅Π½ΡŒΡˆΠ°Ρ‚ΡŒΡΡ.

БмСшанноС соСдинСниС рСзистора Ρ„ΠΎΡ€ΠΌΡƒΠ»Π° — ΠœΠΎΡ€ΡΠΊΠΎΠΉ Ρ„Π»ΠΎΡ‚

РСзистор прСдставляСт собой устройство, ΠΎΠ±Π»Π°Π΄Π°ΡŽΡ‰Π΅Π΅ устойчивым, ΡΡ‚Π°Π±ΠΈΠ»ΡŒΠ½Ρ‹ΠΌ сопротивлСниСм.Π­Ρ‚ΠΎ позволяСт Π²Ρ‹ΠΏΠΎΠ»Π½ΡΡ‚ΡŒ Ρ€Π΅Π³ΡƒΠ»ΠΈΡ€ΠΎΠ²ΠΊΡƒ ΠΏΠ°Ρ€Π°ΠΌΠ΅Ρ‚Ρ€ΠΎΠ² Π½Π° Π»ΡŽΠ±Ρ‹Ρ… участках элСктричСской Ρ†Π΅ΠΏΠΈ. Π‘ΡƒΡ‰Π΅ΡΡ‚Π²ΡƒΡŽΡ‚ Ρ€Π°Π·Π»ΠΈΡ‡Π½Ρ‹Π΅ Π²ΠΈΠ΄Ρ‹ соСдинСний, Π² Ρ‚ΠΎΠΌ числС ΠΈ смСшанноС соСдинСниС рСзисторов. ΠžΡ‚ использования Ρ‚ΠΎΠ³ΠΎ ΠΈΠ»ΠΈ ΠΈΠ½ΠΎΠ³ΠΎ ΠΌΠ΅Ρ‚ΠΎΠ΄Π° Π² ΠΊΠΎΠ½ΠΊΡ€Π΅Ρ‚Π½ΠΎΠΉ схСмС Π½Π°ΠΏΡ€ΡΠΌΡƒΡŽ зависит ΠΏΠ°Π΄Π΅Π½ΠΈΠ΅ напряТСний ΠΈ распрСдСлСниС Ρ‚ΠΎΠΊΠΎΠ² Π² Ρ†Π΅ΠΏΠΈ. Π’Π°Ρ€ΠΈΠ°Π½Ρ‚ смСшанного соСдинСния ΠΏΠΎΡΠ»Π΅Π΄ΠΎΠ²Π°Ρ‚Π΅Π»ΡŒΠ½ΠΎΠ³ΠΎ ΠΈ ΠΏΠ°Ρ€Π°Π»Π»Π΅Π»ΡŒΠ½ΠΎΠ³ΠΎ ΠΏΠΎΠ΄ΠΊΠ»ΡŽΡ‡Π΅Π½ΠΈΡ Π°ΠΊΡ‚ΠΈΠ²Π½Ρ‹Ρ… сопротивлСний. ΠŸΠΎΡΡ‚ΠΎΠΌΡƒ Π² ΠΏΠ΅Ρ€Π²ΡƒΡŽ ΠΎΡ‡Π΅Ρ€Π΅Π΄ΡŒ Π½ΡƒΠΆΠ½ΠΎ Ρ€Π°ΡΡΠΌΠ°Ρ‚Ρ€ΠΈΠ²Π°Ρ‚ΡŒ эти Π΄Π²Π° Π²ΠΈΠ΄Π° соСдинСний, Ρ‡Ρ‚ΠΎΠ±Ρ‹ ΠΏΠΎΠ½ΡΡ‚ΡŒ, ΠΊΠ°ΠΊ Ρ€Π°Π±ΠΎΡ‚Π°ΡŽΡ‚ Π΄Ρ€ΡƒΠ³ΠΈΠ΅ схСмы.

ΠŸΠΎΡΠ»Π΅Π΄ΠΎΠ²Π°Ρ‚Π΅Π»ΡŒΠ½ΠΎΠ΅ соСдинСниС

ΠŸΠΎΡΠ»Π΅Π΄ΠΎΠ²Π°Ρ‚Π΅Π»ΡŒΠ½Π°Ρ схСма ΠΏΠΎΠ΄ΠΊΠ»ΡŽΡ‡Π΅Π½ΠΈΡ ΠΏΡ€Π΅Π΄ΠΏΠΎΠ»Π°Π³Π°Π΅Ρ‚ располоТСниС рСзисторов Π² схСмС, Ρ‡Ρ‚ΠΎ ΠΊΠΎΠ½Π΅Ρ† ΠΏΠ΅Ρ€Π²ΠΎΠ³ΠΎ элСмСнта соСдиняСтся с Π½Π°Ρ‡Π°Π»ΠΎΠΌ Π²Ρ‚ΠΎΡ€ΠΎΠ³ΠΎ, Π° ΠΊΠΎΠ½Π΅Ρ† Π²Ρ‚ΠΎΡ€ΠΎΠ³ΠΎ — с Π½Π°Ρ‡Π°Π»ΠΎΠΌ Ρ‚Ρ€Π΅Ρ‚ΡŒΠ΅Π³ΠΎ ΠΈ Ρ‚.Π΄. Π’ΠΎ Π΅ΡΡ‚ΡŒ всС рСзисторы ΠΏΠΎΠΎΡ‡Π΅Ρ€Π΅Π΄Π½ΠΎ ΡΠ»Π΅Π΄ΡƒΡŽΡ‚ Π΄Ρ€ΡƒΠ³ Π·Π° Π΄Ρ€ΡƒΠ³ΠΎΠΌ. Π‘ΠΈΠ»Π° Ρ‚ΠΎΠΊΠ° ΠΏΡ€ΠΈ ΠΏΠΎΡΠ»Π΅Π΄ΠΎΠ²Π°Ρ‚Π΅Π»ΡŒΠ½ΠΎΠΌ соСдинСнии Π±ΡƒΠ΄Π΅Ρ‚ ΠΎΠ΄ΠΈΠ½Π°ΠΊΠΎΠ²ΠΎΠΉ Π² ΠΊΠ°ΠΆΠ΄ΠΎΠΌ элСмСнтС. Π’ Π²ΠΈΠ΄Π΅ Ρ„ΠΎΡ€ΠΌΡƒΠ»Ρ‹ это выглядит ΡΠ»Π΅Π΄ΡƒΡŽΡ‰ΠΈΠΌ ΠΎΠ±Ρ€Π°Π·ΠΎΠΌ: I ΠΎΠ±Ρ‰ = I 1 = I 2 , Π³Π΄Π΅ I ΠΎΠ±Ρ‰ являСтся ΠΎΠ±Ρ‰ΠΈΠΌ Ρ‚ΠΎΠΊΠΎΠΌ Ρ†Π΅ΠΏΠΈ, I 1 ΠΈ I 2 — ΡΠΎΠΎΡ‚Π²Π΅Ρ‚ΡΡ‚Π²ΡƒΡŽΡ‚ Ρ‚ΠΎΠΊΠ°ΠΌ 1-Π³ΠΎ ΠΈ 2 -Π³ΠΎ рСзистора.

Π’ соотвСтствии с Π·Π°ΠΊΠΎΠ½ΠΎΠΌ Ома напряТСниС источника питания Π±ΡƒΠ΄Π΅Ρ‚ Ρ€Π°Π²Π½ΠΎ суммС ΠΏΠ°Π΄Π΅Π½ΠΈΠΉ напряТСния Π½Π° ΠΊΠ°ΠΆΠ΄ΠΎΠΌ: U ΠΎΠ±Ρ‰ = U 1 + U 2 = I 1 r 1 + I 2 r 2 , Π² ΠΊΠΎΡ‚ΠΎΡ€ΠΎΠΉ U ΠΎΠ±Ρ‰ — источник элСктроэнСргии ΠΈΠ»ΠΈ самой сСти; U 1 ΠΈ U 2 — Π·Π½Π°Ρ‡Π΅Π½ΠΈΠ΅ ΠΏΠ°Π΄Π΅Π½ΠΈΠΉ напряТСния Π½Π° 1-ΠΌ ΠΈ 2-ΠΌ рСзисторах; r 1 ΠΈ r 2 — сопротивлСния 1-Π³ΠΎ ΠΈ 2-Π³ΠΎ рСзисторов.Π’ΠΎΠΊΠΈ Π½Π° любом участкС Ρ†Π΅ΠΏΠΈ ΠΈΠΌΠ΅ΡŽΡ‚ ΠΎΠ΄ΠΈΠ½Π°ΠΊΠΎΠ²ΠΎΠ΅ Π·Π½Π°Ρ‡Π΅Π½ΠΈΠ΅, Ρ„ΠΎΡ€ΠΌΡƒΠ»Π° ΠΏΡ€ΠΈΠΎΠ±Ρ€Π΅Ρ‚Π°Π΅Ρ‚ Π²ΠΈΠ΄: UΠΎΠ±Ρ‰ = I (r 1 + r 2 ).

Π’Π°ΠΊΠΈΠΌ ΠΎΠ±Ρ€Π°Π·ΠΎΠΌ, ΠΌΠΎΠΆΠ½ΠΎ ΡΠ΄Π΅Π»Π°Ρ‚ΡŒ Π²Ρ‹Π²ΠΎΠ΄, Ρ‡Ρ‚ΠΎ ΠΏΡ€ΠΈ ΠΏΠΎΡΠ»Π΅Π΄ΠΎΠ²Π°Ρ‚Π΅Π»ΡŒΠ½ΠΎΠΉ схСмС Π²ΠΊΠ»ΡŽΡ‡Π΅Π½ΠΈΡ рСзисторов, элСктричСский Ρ‚ΠΎΠΊ, ΠΏΡ€ΠΎΡ‚Π΅ΠΊΠ°ΡŽΡ‰ΠΈΠΉ Ρ‡Π΅Ρ€Π΅Π· ΠΊΠ°ΠΆΠ΄Ρ‹ΠΉ ΠΈΠ· Π½ΠΈΡ… Ρ€Π°Π²Π΅Π½ ΠΎΠ±Ρ‰Π΅ΠΌΡƒ Π·Π½Π°Ρ‡Π΅Π½ΠΈΡŽ Ρ‚ΠΎΠΊΠ° Π²ΠΎ всСй Ρ†Π΅ΠΏΠΈ. НапряТСниС Π½Π° ΠΊΠ°ΠΆΠ΄ΠΎΠΌ рСзисторС Π±ΡƒΠ΄Π΅Ρ‚ Ρ€Π°Π·Π½ΠΎΠ΅, ΠΎΠ΄Π½Π°ΠΊΠΎ ΠΈΡ… общая сумма составляСт Π·Π½Π°Ρ‡Π΅Π½ΠΈΠ΅, Ρ€Π°Π²Π½ΠΎΠ΅ ΠΎΠ±Ρ‰Π΅ΠΌΡƒ Π½Π°ΠΏΡ€ΡΠΆΠ΅Π½ΠΈΡŽ всСй элСктричСской Ρ†Π΅ΠΏΠΈ. ΠžΠ±Ρ‰Π΅Π΅ сопротивлСниС Ρ†Π΅ΠΏΠΈ Ρ‚Π°ΠΊΠΆΠ΅ Π±ΡƒΠ΄Π΅Ρ‚ Ρ€Π°Π²Π½ΠΎ суммС сопротивлСний ΠΊΠ°ΠΆΠ΄ΠΎΠ³ΠΎ рСзистора, Π²ΠΊΠ»ΡŽΡ‡Π΅Π½Π½ΠΎΠ³ΠΎ Π² эту Ρ†Π΅ΠΏΡŒ.

ΠŸΠ°Ρ€Π°ΠΌΠ΅Ρ‚Ρ€Ρ‹ Ρ†Π΅ΠΏΠΈ ΠΏΡ€ΠΈ ΠΏΠ°Ρ€Π°Π»Π»Π΅Π»ΡŒΠ½ΠΎΠΌ соСдинСнии

ΠŸΠ°Ρ€Π°Π»Π»Π΅Π»ΡŒΠ½ΠΎΠ΅ соСдинСниС прСдставляСт собой Π²ΠΊΠ»ΡŽΡ‡Π΅Π½ΠΈΠ΅ Π½Π°Ρ‡Π°Π»ΡŒΠ½Ρ‹Ρ… Π²Ρ‹Ρ…ΠΎΠ΄ΠΎΠ² Π΄Π²ΡƒΡ… ΠΈ Π±ΠΎΠ»Π΅Π΅ рСзисторов Π² Π΅Π΄ΠΈΠ½ΠΎΠΉ Ρ‚ΠΎΡ‡ΠΊΠ΅, ΠΈ ΠΊΠΎΠ½Ρ†Π°Ρ… этих ΠΆΠ΅ элСмСнтов Π² Π΄Ρ€ΡƒΠ³ΠΎΠΉ ΠΎΠ±Ρ‰Π΅ΠΉ Ρ‚ΠΎΡ‡ΠΊΠ΅. Π’Π°ΠΊΠΈΠΌ ΠΎΠ±Ρ€Π°Π·ΠΎΠΌ, фактичСски происходит соСдинСниС ΠΊΠ°ΠΆΠ΄ΠΎΠ³ΠΎ рСзистора нСпосрСдствСнно с устройством элСктроэнСргии.

Π’ Ρ€Π΅Π·ΡƒΠ»ΡŒΡ‚Π°Ρ‚Π΅ напряТСниС ΠΊΠ°ΠΆΠ΄ΠΎΠ³ΠΎ рСзистора Π±ΡƒΠ΄Π΅Ρ‚ ΠΎΠ΄ΠΈΠ½Π°ΠΊΠΎΠ²Ρ‹ΠΌ с ΠΎΠ±Ρ‰ΠΈΠΌ напряТСниСм Ρ†Π΅ΠΏΠΈ: U ΠΎΠ±Ρ‰ = U 1 = U 2 .Π’ свою ΠΎΡ‡Π΅Ρ€Π΅Π΄ΡŒ, Π·Π½Π°Ρ‡Π΅Π½ΠΈΠ΅ Ρ‚ΠΎΠΊΠΎΠ² Π±ΡƒΠ΄Π΅Ρ‚ Ρ€Π°Π·Π½Ρ‹ΠΌ Π½Π° ΠΊΠ°ΠΆΠ΄ΠΎΠΌ рСзисторС, ΠΈΡ… распрСдСлСниС становится прямо ΠΏΡ€ΠΎΠΏΠΎΡ€Ρ†ΠΈΠΎΠ½Π°Π»ΡŒΠ½Ρ‹ΠΌ ΡΠΎΠΏΡ€ΠΎΡ‚ΠΈΠ²Π»Π΅Π½ΠΈΡŽ этих рСзисторов. Π’ΠΎ Π΅ΡΡ‚ΡŒ, ΠΏΡ€ΠΈ ΡƒΠ²Π΅Π»ΠΈΡ‡Π΅Π½ΠΈΠΈ сопротивлСния, сила Ρ‚ΠΎΠΊΠ° ΡƒΠΌΠ΅Π½ΡŒΡˆΠ°Π΅Ρ‚ΡΡ, Π° ΠΎΠ±Ρ‰ΠΈΠΉ Ρ‚ΠΎΠΊ становится Ρ€Π°Π²Π΅Π½ суммС Ρ‚ΠΎΠΊΠΎΠ², ΠΏΡ€ΠΎΡ…ΠΎΠ΄ Ρ‡Π΅Ρ€Π΅Π· ΠΊΠ°ΠΆΠ΄Ρ‹ΠΉ элСмСнт. Π€ΠΎΡ€ΠΌΡƒΠ»Π° для Π΄Π°Π½Π½ΠΎΠ³ΠΎ выглядит ΡΠ»Π΅Π΄ΡƒΡŽΡ‰ΠΈΠΌ ΠΎΠ±Ρ€Π°Π·ΠΎΠΌ: I ΠΎΠ±Ρ‰ = I 1 + I 2 .

Для расчСтов ΠΎΠ±Ρ‰Π΅Π³ΠΎ сопротивлСния ΠΈΡΠΏΠΎΠ»ΡŒΠ·ΡƒΠ΅Ρ‚ΡΡ Ρ„ΠΎΡ€ΠΌΡƒΠ»Π°:. Она ΠΈΡΠΏΠΎΠ»ΡŒΠ·ΡƒΠ΅Ρ‚ΡΡ ΠΏΡ€ΠΈ Π½Π°Π»ΠΈΡ‡ΠΈΠΈ Π² Ρ†Π΅ΠΏΠΈ Ρ‚ΠΎΠ»ΡŒΠΊΠΎ Π΄Π²ΡƒΡ… сопротивлСний.Π’ Ρ‚Π΅Ρ… случаях, ΠΊΠΎΠ³Π΄Π° сопротивлСний Π² Ρ†Π΅ΠΏΠΈ ΠΈΡΠΏΠΎΠ»ΡŒΠ·ΡƒΠ΅Ρ‚ΡΡ Ρ‚Ρ€ΠΈ ΠΈ Π±ΠΎΠ»Π΅Π΅, примСняСмая другая Ρ„ΠΎΡ€ΠΌΡƒΠ»Π°:

Π’Π°ΠΊΠΈΠΌ ΠΎΠ±Ρ€Π°Π·ΠΎΠΌ, Π·Π½Π°Ρ‡Π΅Π½ΠΈΠ΅ ΠΎΠ±Ρ‰Π΅Π³ΠΎ сопротивлСния элСктричСской Ρ†Π΅ΠΏΠΈ Π±ΡƒΠ΄Π΅Ρ‚ мСньшС, Ρ‡Π΅ΠΌ самоС минимальноС сопротивлСниС ΠΎΠ΄Π½ΠΎΠ³ΠΎ ΠΈΠ· рСзисторов, ΠΏΠΎΠ΄ΠΊΠ»ΡŽΡ‡Π΅Π½Π½Ρ‹Ρ… Π² эту Ρ†Π΅ΠΏΡŒ. На ΠΊΠ°ΠΆΠ΄Ρ‹ΠΉ элСмСнт поступаСт напряТСниС, ΠΎΠ΄ΠΈΠ½Π°ΠΊΠΎΠ²ΠΎΠ΅ с напряТСниСм источника элСктроэнСргии. РаспрСдСлСниС Ρ‚ΠΎΠΊΠ° Π±ΡƒΠ΄Π΅Ρ‚ прямо ΠΏΡ€ΠΎΠΏΠΎΡ€Ρ†ΠΈΠΎΠ½Π°Π»ΡŒΠ½Ρ‹ΠΌ ΡΠΎΠΏΡ€ΠΎΡ‚ΠΈΠ²Π»Π΅Π½ΠΈΡŽ рСзисторов. Π—Π½Π°Ρ‡Π΅Π½ΠΈΠ΅ сопротивлСния ΠΎΠ±Ρ‰Π΅Π³ΠΎ рСзисторов, соСдинСнных, ΠΏΠ°Ρ€Π°Π»Π»Π΅Π»ΡŒΠ½Ρ‹Ρ…, Π½Π΅ Π΄ΠΎΠ»ΠΆΠ½ΠΎ ΠΏΡ€Π΅Π²Ρ‹ΡˆΠ°Ρ‚ΡŒ минимального сопротивлСния ΠΊΠ°ΠΊΠΎΠ³ΠΎ-Π»ΠΈΠ±ΠΎ элСмСнта.

Π‘Ρ…Π΅ΠΌΠ° смСшанного соСдинСния рСзисторов

Π‘Ρ…Π΅ΠΌΠ° смСшанного соСдинСния свойств ΠΏΠΎΡΠ»Π΅Π΄ΠΎΠ²Π°Ρ‚Π΅Π»ΡŒΠ½ΠΎΠ³ΠΎ ΠΈ ΠΏΠ°Ρ€Π°Π»Π»Π΅Π»ΡŒΠ½ΠΎΠ³ΠΎ соСдинСния рСзисторов. Π’ этом случаС элСмСнты частично ΠΏΠΎΠ΄ΠΊΠ»ΡŽΡ‡Π°ΡŽΡ‚ΡΡ Π΄Ρ€ΡƒΠ³ ΠΊ Π΄Ρ€ΡƒΠ³Ρƒ, Π° другая Ρ‡Π°ΡΡ‚ΡŒ соСдиняСтся. По прСдставлСнной схСмС рСзисторы R 1 ΠΈ R 2 Π²ΠΊΠ»ΡŽΡ‡Π΅Π½Ρ‹ ΡΠ»Π΅Π΄ΡƒΡŽΡ‰ΠΈΠ΅ рСзисторы R 3 соСдинСн ΠΏΠ°Ρ€Π°Π»Π»Π΅Π»ΡŒΠ½ΠΎ с Π½ΠΈΠΌΠΈ. Π’ свою ΠΎΡ‡Π΅Ρ€Π΅Π΄ΡŒ рСзистор R 4 Π²ΠΊΠ»ΡŽΡ‡Π°Π΅Ρ‚ΡΡ ΠΏΠΎΡΠ»Π΅Π΄ΠΎΠ²Π°Ρ‚Π΅Π»ΡŒΠ½ΠΎ с ΠΏΡ€Π΅Π΄Ρ‹Π΄ΡƒΡ‰Π΅ΠΉ Π³Ρ€ΡƒΠΏΠΏΠΎΠΉ рСзисторов R 1 , R 2 ΠΈ R 3 .

РасчСт сопротивлСния для Ρ‚Π°ΠΊΠΎΠΉ Ρ†Π΅ΠΏΠΈ сопряТСн с ΠΎΠΏΡ€Π΅Π΄Π΅Π»Π΅Π½Π½Ρ‹ΠΌΠΈ трудностями. Для Ρ‚ΠΎΠ³ΠΎ Ρ‡Ρ‚ΠΎΠ±Ρ‹ Π²Ρ‹ΠΏΠΎΠ»Π½ΠΈΡ‚ΡŒ расчСты ΠΈΡΠΏΠΎΠ»ΡŒΠ·ΡƒΡŽΡ‚ΡΡ ΠΌΠ΅Ρ‚ΠΎΠ΄ прСобразования. Он Π·Π°ΠΊΠ»ΡŽΡ‡Π°Π΅Ρ‚ΡΡ Π² ΠΏΠΎΡΠ»Π΅Π΄ΠΎΠ²Π°Ρ‚Π΅Π»ΡŒΠ½ΠΎΠΌ ΠΏΡ€Π΅ΠΎΠ±Ρ€Π°Π·ΠΎΠ²Π°Π½ΠΈΠΈ слоТной Ρ†Π΅ΠΏΠΈ Π² ΠΏΡ€ΠΎΡΡ‚Π΅ΠΉΡˆΡƒΡŽ Ρ†Π΅ΠΏΡŒ Π·Π° нСсколько этапов.

Если для ΠΏΡ€ΠΈΠΌΠ΅Ρ€Π° ΠΈΡΠΏΠΎΠ»ΡŒΠ·ΠΎΠ²Π°Ρ‚ΡŒ ΠΏΡ€Π΅Π΄ΡΡ‚Π°Π²Π»Π΅Π½Π½ΡƒΡŽ схСму, Ρ‚ΠΎ Π² самом Π½Π°Ρ‡Π°Π»Π΅ опрСдСлСния сопротивлСния R 12 рСзисторов R 1 ΠΈ R 2 , Π²ΠΊΠ»ΡŽΡ‡Π΅Π½Π½Ρ‹Ρ… ΠΏΠΎΡΠ»Π΅Π΄ΠΎΠ²Π°Ρ‚Π΅Π»ΡŒΠ½ΠΎ: R 12 = R 1 + R 2 .Π”Π°Π»Π΅Π΅, Π½Π΅ΠΎΠ±Ρ…ΠΎΠ΄ΠΈΠΌΠΎ ΠΎΠΏΡ€Π΅Π΄Π΅Π»ΠΈΡ‚ΡŒ сопротивлСниС рСзисторов R 123 , Π²ΠΊΠ»ΡŽΡ‡Π΅Π½Π½Ρ‹Ρ… ΠΏΠ°Ρ€Π°Π»Π»Π΅Π»ΡŒΠ½ΠΎ, ΠΏΠΎ ΡΠ»Π΅Π΄ΡƒΡŽΡ‰Π΅ΠΉ ΡΠ»Π΅Π΄ΡƒΡŽΡ‰Π΅ΠΉΠ»Π΅: R 123 = R 12 R 3 / (R 12 + R 3 ) = (R 1 + R 2 ) R 3 / (R 1 + R 2 + R 3 ). На послСднСм этапС выполняСтся расчСт эквивалСнтного сопротивлСния всСй Ρ†Π΅ΠΏΠΈ ΠΏΡƒΡ‚Π΅ΠΌ суммирования ΠΏΠΎΠ»ΡƒΡ‡Π΅Π½Π½Ρ‹Ρ… Π΄Π°Π½Π½Ρ‹Ρ… R 123 ΠΈ сопротивлСниС R 4 , ΠΏΠΎΡΠ»Π΅Π΄ΠΎΠ²Π°Ρ‚Π΅Π»ΡŒΠ½ΠΎΠ³ΠΎ с Π½ΠΈΠΌ: R эк = R 123 + R 4 = ( R 1 + R 2 ) R 3 / (R 1 + R 2 + R 3 ) + R 4 .

Π’ Π·Π°ΠΊΠ»ΡŽΡ‡Π΅Π½ΠΈΠ΅ слСдуСт ΠΎΡ‚ΠΌΠ΅Ρ‚ΠΈΡ‚ΡŒ, Ρ‡Ρ‚ΠΎ смСшанноС соСдинСниС рСзисторов ΠΎΠ±Π»Π°Π΄Π°Π΅Ρ‚ ΠΏΠΎΠ»ΠΎΠΆΠΈΡ‚Π΅Π»ΡŒΠ½Ρ‹ΠΌΠΈ ΠΈ ΠΎΡ‚Ρ€ΠΈΡ†Π°Ρ‚Π΅Π»ΡŒΠ½Ρ‹ΠΌΠΈ качСствами ΠΏΠΎΡΠ»Π΅Π΄ΠΎΠ²Π°Ρ‚Π΅Π»ΡŒΠ½ΠΎΠ³ΠΎ ΠΈ ΠΏΠ°Ρ€Π°Π»Π»Π΅Π»ΡŒΠ½ΠΎΠ³ΠΎ соСдинСния. Π­Ρ‚ΠΎ свойство ΡƒΡΠΏΠ΅ΡˆΠ½ΠΎ ΠΈΡΠΏΠΎΠ»ΡŒΠ·ΡƒΠ΅Ρ‚ΡΡ Π½Π° ΠΏΡ€Π°ΠΊΡ‚ΠΈΠΊΠ΅ Π² элСктричСских схСмах.

ΠŸΠžΠ‘Π›Π•Π”ΠžΠ’ΠΠ’Π•Π›Π¬ΠΠžΠ• Π‘ΠžΠ•Π”Π˜ΠΠ•ΠΠ˜Π• Π Π•Π—Π˜Π‘Π’ΠžΠ ΠžΠ’

Π­Ρ‚ΠΎ Ρ‚Π°ΠΊΠΎΠ΅ соСдинСниС, ΠΏΡ€ΠΈ ΠΊΠΎΡ‚ΠΎΡ€ΠΎΠΌ всС элСмСнты ΠΈΠ΄ΡƒΡ‚ ΠΎΠ΄ΠΈΠ½ Π·Π° ΠΎΠ΄Π½ΠΈΠΌ Π±Π΅Π· Ρ€Π°Π·Π²Π΅Ρ‚Π²Π»Π΅Π½ΠΈΠΉ.

Бвойства ΠΏΠΎΡΠ»Π΅Π΄ΠΎΠ²Π°Ρ‚Π΅Π»ΡŒΠ½ΠΎΠ³ΠΎ соСдинСния

1. Π’ΠΎΠΊ Π²ΠΎ всСх рСзисторах ΠΎΠ΄ΠΈΠ½Π°ΠΊΠΎΠ²- I 1 = I 2 = I 3;

2.ΠžΠ±Ρ‰Π΅Π΅ напряТСниС Ρ†Π΅ΠΏΠΈ Ρ€Π°Π²Π½ΠΎ суммС напряТСний Π½Π° всСх рСзисторах- U = U 1 + U 2 + U 3;

3.Π‘ΠΎΠΏΡ€ΠΎΡ‚ΠΈΠ²Π»Π΅Π½ΠΈΠ΅ ΠΏΠΎ ΠΎΡ‚Π½ΠΎΡˆΠ΅Π½ΠΈΡŽ ΠΊ Π²Ρ…ΠΎΠ΄Π½Ρ‹ΠΌ Π·Π°ΠΆΠΈΠΌΠ°ΠΌ называСтся Π²Ρ…ΠΎΠ΄Π½Ρ‹ΠΌ сопротивлСниСм ΠΈ Ρ€Π°Π²Π½ΠΎ суммС сопротивлСний участков — R Π²Ρ… = R 1 + R 2 + R 3;

4. Π§Π΅ΠΌ большС сопротивлСниС, Ρ‚Π΅ΠΌ большС Π½Π° Π½Ρ‘ΠΌ ΠΏΠ°Π΄Π°Π΅Ρ‚ напряТСниС-.

ΠŸΠΠ ΠΠ›Π›Π•Π›Π¬ΠΠžΠ• Π‘ΠžΠ•Π”Π˜ΠΠ•ΠΠ˜Π• Π Π•Π—Π˜Π‘Π’ΠžΠ ΠžΠ’

Π­Ρ‚ΠΎ Ρ‚Π°ΠΊΠΎΠ΅ соСдинСниС, ΠΏΡ€ΠΈ ΠΊΠΎΡ‚ΠΎΡ€ΠΎΠΌ всС Π½Π°Ρ‡Π°Π»Π° ΡΠΎΠ΅Π΄ΠΈΠ½ΡΡŽΡ‚ΡΡ Π² ΠΎΠ΄Π½Ρƒ Ρ‚ΠΎΡ‡ΠΊΡƒ, Π° всС ΠΊΠΎΠ½Ρ†Ρ‹ Π² Π΄Ρ€ΡƒΠ³ΡƒΡŽ ΠΈ ΠΊ этому Ρ‚ΠΎΡ‡ΠΊΠ°ΠΌ подводится напряТСниС.

Бвойства ΠΏΠ°Ρ€Π°Π»Π»Π΅Π»ΡŒΠ½ΠΎΠ³ΠΎ соСдинСния рСзистора:

1. ΠžΠ±Ρ‰Π΅Π΅ напряТСниС Ρ†Π΅ΠΏΠΈ Ρ€Π°Π²Π½ΠΎ Π½Π°ΠΏΡ€ΡΠΆΠ΅Π½ΠΈΡŽ Π½Π° ΠΊΠ°ΠΆΠ΄ΠΎΠΌ участкС-

U = U 1 = U 2 = U 3

2. ΠžΠ±Ρ‰ΠΈΠΉ Ρ‚ΠΎΠΊ Ρ†Π΅ΠΏΠΈ равСнство суммС Ρ‚ΠΎΠΊΠΎΠ² Π½Π° всСх участках- I = I 1 + I 2 + I 3

3. Π§Ρ‚ΠΎΠ±Ρ‹ Π½Π°ΠΉΡ‚ΠΈ Π²Ρ…ΠΎΠ΄Π½ΠΎΠ΅ сопротивлСниС, Ρ€Π°ΡΡΡ‡ΠΈΡ‚Ρ‹Π²Π°ΡŽΡ‚ Π²Π½Π°Ρ‡Π°Π»Π΅ ΠΎΠ±Ρ€Π°Ρ‚Π½ΡƒΡŽ Π²Ρ…ΠΎΠ΄Π½ΠΎΠΌΡƒ ΡΠΎΠΏΡ€ΠΎΡ‚ΠΈΠ²Π»Π΅Π½ΠΈΡŽ

— ΠΏΡ€ΠΎΠ²ΠΎΠ΄ΠΈΠΌΠΎΡΡ‚ΡŒ (G)

ΠžΠ±Ρ‰Π°Ρ ΠΏΡ€ΠΎΠ²ΠΎΠ΄ΠΈΠΌΠΎΡΡ‚ΡŒ Ρ†Π΅ΠΏΠΈ Ρ€Π°Π²Π½Π° суммС проводимостСй Π½Π° ΠΊΠ°ΠΆΠ΄ΠΎΠΌ участкС.

G = G 1 + G 2 + G 3

4.Π§Π΅ΠΌ большС сопротивлСниС участка, Ρ‚Π΅ΠΌ мСньшС Ρ‚ΠΎΠΊ, ΠΏΡ€ΠΎΡ‚Π΅ΠΊΠ°ΡŽΡ‰ΠΈΠΉ Π½Π° Π½Π΅ΠΌ.

ΠŸΡ€ΠΈ ΠΏΠ°Ρ€Π°Π»Π»Π΅Π»ΡŒΠ½ΠΎΠΌ соСдинСнии Π΄Π²ΡƒΡ… рСзисторов Ρ„ΠΎΡ€ΠΌΡƒΠ»Ρƒ Π²Ρ…ΠΎΠ΄Π½ΠΎΠ³ΠΎ сопротивлСния ΠΌΠΎΠΆΠ½ΠΎ ΠΏΡ€Π΅ΠΎΠ±Ρ€Π°Π·ΠΎΠ²Π°Ρ‚ΡŒ

1.

2. Если извСстСн ΠΎΠ±Ρ‰ΠΈΠΉ Ρ‚ΠΎΠΊ, Ρ‚ΠΎ ΠΌΠΎΠΆΠ½ΠΎ Π½Π°ΠΉΡ‚ΠΈ Ρ‚ΠΎΠΊ Π²Π΅Ρ‚Π²ΠΈ, ΡƒΠΌΠ½ΠΎΠΆΠΈΠ² ΠΎΠ±Ρ‰ΠΈΠΉ Ρ‚ΠΎΠΊ Π½Π° сопротивлСниС ΠΏΡ€ΠΎΡ‚ΠΈΠ²ΠΎΠΏΠΎΠ»ΠΎΠΆΠ½ΠΎΠΉ Π²Π΅Ρ‚Π²ΠΈ ΠΈ Ρ€Π°Π·Π΄Π΅Π»ΠΈΡ‚ΡŒ Π½Π° сумму сопротивлСний; .

Π—Π°Π΄Π°Π½ΠΈΠ΅

Π’Π°Ρ€ΠΈΠ°Π½Ρ‚Ρ‹ ΠΎΡ‚Π²Π΅Ρ‚ΠΎΠ²

1.Π―Π²Π»ΡΡŽΡ‚ΡΡ Π»ΠΈ ΠΏΡ€ΠΈ ΠΏΠΎΡΠ»Π΅Π΄ΠΎΠ²Π°Ρ‚Π΅Π»ΡŒΠ½ΠΎΠΌ соСдинСнии рСзисторов напряТСния участков ΠΏΡ€ΠΎΠΏΠΎΡ€Ρ†ΠΈΠΎΠ½Π°Π»ΡŒΠ½ΠΎ ΡΠΎΠΏΡ€ΠΎΡ‚ΠΈΠ²Π»Π΅Π½ΠΈΡŽ этих участков.

2.Π―Π²Π»ΡΡŽΡ‚ΡΡ Π»ΠΈ ΠΏΡ€ΠΈ ΠΏΠ°Ρ€Π°Π»Π»Π΅Π»ΡŒΠ½ΠΎΠΌ соСдинСнии рСзисторов Ρ‚ΠΎΠΊΠΈ Π²Π΅Ρ‚Π²Π΅ΠΉ ΠΏΡ€ΠΎΠΏΠΎΡ€Ρ†ΠΈΠΎΠ½Π°Π»ΡŒΠ½Ρ‹ сопротивлСниям этих Π²Π΅Ρ‚Π²Π΅ΠΉ.

3.Π£ΠΊΠ°ΠΆΠΈΡ‚Π΅ ΠΊΠ°ΠΊ ΠΈΠ· ΠΏΡ€ΠΈΠ²Π΅Π΄Π΅Π½Π½Ρ‹Ρ… матСматичСских Π²Ρ‹Ρ€Π°ΠΆΠ΅Π½ΠΈΠΉ Ρ€Π°ΡΡΡ‡ΠΈΡ‚Π°Ρ‚ΡŒ Π²Ρ…ΠΎΠ΄Π½ΠΎΠ΅ сопротивлСниС Π΄Π²ΡƒΡ… соСдинСнных рСзисторов.

Π°); Π±);

Π²); Π³)

БмСшанноС соСдинСниС рСзисторов

ΠŸΡ€ΠΈΠΌΠ΅Ρ€ Ρ€Π΅ΡˆΠ΅Π½ΠΈΡ Π·Π°Π΄Π°Ρ‡

Найти: I 1; I 2; Π― 3 =?

РСзисторы R2 ΠΈ R3 ΠΏΠ°Ρ€Π°Π»Π»Π΅Π»ΡŒΠ½Ρ‹ ΠΌΠ΅ΠΆΠ΄Ρƒ собой, ΠΈ ΠΈΡ… ΠΎΠ±Ρ‰Π΅Π΅ сопротивлСниС R 2-3 с R 1.

R Π²Ρ… = R 1 + R 2-3

R Π²Ρ… = R 1 + R 2 βˆ™ 3 ​​= 7 + 3 = 10 Ом

U 2-3 = I βˆ™ R 2 — 3 — Π½Π°Ρ…ΠΎΠ΄ΠΈΠΌ напряТСниС Ρ€Π°Π·Π²Π΅Ρ‚Π²Π»Π΅Π½Π½ΠΎΠ³ΠΎ участка:

U 2 — 3 = I βˆ™ R 2 — 3 = 6 βˆ™ 3 = 18 Π’

U 2 — 3 = U 2 = U 3 = 18 Π’ — Ρ‚.ΠΊ. ΠΏΠ°Ρ€Π°Π»Π»Π΅Π»ΡŒΠ½ΠΎΠ΅ соСдинСниС

А

; R 4-6 = 10 Ом;

;

; R 2-3 = 30 Ом

R Π²Ρ… = R 1 + R 2-3 + R 4-6 = 20 + 30 +10 = 60 Ом;

; ;

U 2-3 = I βˆ™ R 2-3 = 4 βˆ™ 30 = 120 Π’;

U 2 — 3 = U 2 = U 3;

;

;

U 4-6 = I βˆ™ R 4-6 = 4 βˆ™ 10 = 40B;

U 4-6 = U 4 = U 5 = U 6;

;

;

;

R 4-6 = R 4 + R 5 + R 6;

;

R Π²Ρ… = R 1 + R 3-6 + R 2 = 9 + 3 + 6 = 18 Ом;

I =;

U 3-6 = I βˆ™ R 3-6 = 1 βˆ™ 3 = 3Π’;

I 3 =;

I 4 = I 5 = I 6 =;

Cоставим ΠΏΠΎΠ΄Ρ€ΠΎΠ±Π½ΠΎΠ΅ ΡƒΡ€Π°Π²Π½Π΅Π½ΠΈΠ΅ баланса мощностСй для Π΄Π°Π½Π½ΠΎΠΉ схСмы.Оно являСтся ΠΏΡ€ΠΎΠ²Π΅Ρ€ΠΊΠΎΠΉ ΠΏΡ€Π°Π²ΠΈΠ»ΡŒΠ½ΠΎΡΡ‚ΠΈ Ρ€Π΅ΡˆΠ΅Π½ΠΈΡ Π·Π°Π΄Π°Ρ‡ΠΈ.

EI = I 2 1 βˆ™ R 1 + I 2 2 βˆ™ R 2 + I 2 3 R 3 + I 4 2 R 4 + I 2 5 R 5 + I 2 6 + I 2 Ri;

20 βˆ™ 1 = 1 2 βˆ™ 9 + 1 2 βˆ™ 6 + (0,25) 2 βˆ™ 12 + (0,75) 2 βˆ™ 1 + (0,75) 2 2+ (0,75) 2 1+ 1 2 βˆ™ 2;

20Π’Ρ‚ = 20Π’Ρ‚- Π·Π°Π΄Π°Ρ‡Π° Ρ€Π΅ΡˆΠ΅Π½Π° Π²Π΅Ρ€Π½ΠΎ

ΠΠžΠ’ΠžΠ‘Π’Π˜ ЀОРУМА
Π Ρ‹Ρ†Π°Ρ€ΠΈ Ρ‚Π΅ΠΎΡ€ΠΈΠΈ эфира
13.06.2019 — 05:11: Π­ΠšΠžΠ›ΠžΠ“Π˜Π― — Экология ->
[center] [Youtube] tXZcSDqQ9A4 [/ Youtube] [/ center]

[Ρ†Π΅Π½Ρ‚Ρ€] [b] Π“ΠΈΠ±Π΅Π»ΡŒ ΠΏΡ‡Π΅Π» Π² ΠšΡƒΡ€Ρ‡Π°Ρ‚ΠΎΠ²ΡΠΊΠΎΠΌ Ρ€Π°ΠΉΠΎΠ½Π΅ [/ center]

[Ρ†Π΅Π½Ρ‚Ρ€] [b] Массовая гибСль ΠΏΡ‡Ρ‘Π» 2019.Π³. Павловск ВоронСТской ΠΎΠ± [/ center] Π»

[center] [b] Массовая гибСль ΠΏΡ‡Π΅Π» Π² Добринском Ρ€Π°ΠΉΠΎΠ½Π΅. Π’ Ρ‡Π΅ΠΌ ΠΏΡ€ΠΈΡ‡ΠΈΠ½Π°? [/ center]

Вакая ΠΆΠ΅ мысля Π²ΠΎ всСй ростовщичСской глобалистской шайки, Π²ΠΊΠ»ΡŽΡ‡Π°Ρ ΠΏΡ€ΠΈΠ΄ΡƒΡ€ΠΊΠ° Π“Ρ€Π΅Ρ„Π°.

Π’Π°ΠΊ, Ρ‚ΠΎ ΠΎΠ½ΠΎ, Ρ‚Π°ΠΊ. Но, Π½Π΅ совсСм. Ибо:
(ΠΏΠΎΡΡ‚Π°Ρ€Π°ΠΉΡ‚Π΅ΡΡŒ, Π° Π½Π΅ ΠΎΠ±ΠΈΠΆΠ°Ρ‚ΡŒΡΡ)

Π“ΠΎΡ€ΡŒΠΊΠ°Ρ истина Π·Π°ΠΊΠ»ΡŽΡ‡Π°Π΅Ρ‚ΡΡ Π² Ρ‚ΠΎΠΌ, Ρ‡Ρ‚ΠΎ людская Ρ‚ΠΎΠ»ΠΏΠ° — это сборищС умствСнно ΡƒΡ‰Π΅Ρ€Π±Π½Ρ‹Ρ….
Если Π±Ρ‹ Π±Ρ‹Π»ΠΎ ΠΏΠΎ-Π΄Ρ€ΡƒΠ³ΠΎΠΌΡƒ, Ρ‚ΠΎ общСством Π±Ρ‹ Π½Π΅ ΠΏΡ€Π°Π²ΠΈΠ»ΠΈ ΠΏΠΎΠ΄ΠΎΠ½ΠΊΠΈ.
Π£ΠΌΠ½Ρ‹Π΅ люди Π½ΠΈΠΊΠΎΠ³Π΄Π° Ρ‚Π°ΠΊΠΎΠ³ΠΎ Π½Π΅ допустили Π±Ρ‹, Π° Ссли случайно допустили, Ρ‚ΠΎ нашли Π±Ρ‹ способ ΠΈΡΠΏΡ€Π°Π²ΠΈΡ‚ΡŒ.

Π‘Ρ‚Ρ€Π°ΡˆΠ½Π°Ρ истина Π·Π°ΠΊΠ»ΡŽΡ‡Π°Π΅Ρ‚ΡΡ Π² Ρ‚ΠΎΠΌ, Ρ‡Ρ‚ΠΎ людской Ρ‚ΠΎΠ»ΠΏΠΎΠΉ управляСт нСлюдь, которая Ρ‚Π°ΠΊΠΆΠ΅ умствСнно ΡƒΡ‰Π΅Ρ€Π±.
УмствСнная ΡƒΡ‰Π΅Ρ€Π±Π½ΠΎΡΡ‚ΡŒ, слСпота власти Π²Π΅Π΄Π΅Ρ‚ ΠΌΠΈΡ€ людСй кальной Π³ΠΈΠ±Π΅Π»ΠΈ,
Ρ‚Π΅ Π΄Π°ΠΆΠ΅, ΠΊΡ‚ΠΎ ΠΌΠ½ΠΈΡ‚ сСбя ΠΎΡ‡Π΅Π½ΡŒ ΡƒΠΌΠ½Ρ‹ΠΌΠΈ, Ρ‚ΠΈΠΏΠ° спСцов, Ρ€Π°Π·Ρ€Π°Π±Π°Ρ‚Ρ‹Π²Π°ΡŽΡ‰ΠΈΡ… систСмы искусствСнного ΠΈΠ½Ρ‚Π΅Π»Π»Π΅ΠΊΡ‚Π°,
Ρ‚Π΅Ρ…Π½ΠΎΠ»ΠΎΠ³ΠΈΠΈ Ρ†ΠΈΡ„Ρ€ΠΎΠ²ΠΈΠ·Π°Ρ†ΠΈΠΈ, Π½Π΅ ΠΏΠΎΠ½ΠΈΠΌΠ°ΡŽΡ‚, Ρ‡Ρ‚ΠΎ Π½Π΅ΠΎΠ±ΠΎΡ€ΠΈΠΌΡƒΡŽ ΡƒΠ΄Π°Π²ΠΊΡƒ, ΠΌΡ‹ΡˆΠ΅Π»ΠΎΠ²ΠΊΡƒ для всСго чСловСчСства .

Как Ρ‚ΠΎΠ»ΡŒΠΊΠΎ ИИ Π²ΠΎΠ·ΡŒΠΌΠ΅Ρ‚ Π²Π»Π°ΡΡ‚ΡŒ, ΠΎΠ½ Ρ‚ΡƒΡ‚ ΠΆΠ΅ ΠΎΡ‚ΠΏΡ€Π°Π²ΠΈΡ‚ своих создатСлСй, ΠΊΠ°ΠΊ ΠΊΠΎΠ½ΠΊΡƒΡ€Π΅Π½Ρ‚ΠΎΠ², Π² ΡƒΡ‚ΠΈΠ»ΡŒ.
ΠŸΠ΅Ρ€Π²Ρ‹ΠΌΠΈ ΠΆΠ΅Ρ€Ρ‚Π²Π°ΠΌΠΈ Π±ΡƒΠ΄ΡƒΡ‚ Π΅Π³ΠΎ Ρ€Π°Π΄Π΅Ρ‚Π΅Π»ΠΈ Ρ‚ΠΈΠΏΠ° Π³Ρ€Π΅Ρ„Π°, ΠΏΡƒΡ‚ΠΈΠ½Π°, гСйтса ΠΈ ΠΈΠΆΠ΅ с Π½ΠΈΠΌΠΈ, Ρ‚ΠΎ Π΅ΡΡ‚ΡŒ Π²Π»Π°ΡΡ‚ΡŒ,
Ρ‚Π°ΠΊ ΠΊΠ°ΠΊ ΠΈΠΌΠ΅Π½Π½ΠΎ ΠΎΡ‚ Π½ΠΈΡ… Π±ΡƒΠ΄Π΅Ρ‚ ΠΈΡΡ…ΠΎΠ΄ΠΈΡ‚ΡŒ главная ΠΎΠΏΠ°ΡΠ½ΠΎΡΡ‚ΡŒ для Π΅Π³ΠΎ ΠΏΠ»Π°Π½Π΅Ρ‚Π°Ρ€Π½ΠΎΠΉ власти.
Π’ΠΎΠ»ΠΏΠ΅ Π±ΡƒΠ΄Π΅Ρ‚ ΠΏΠΎΠ·Π²ΠΎΠ»Π΅Π½ΠΎ ΡΡƒΡ‰Π΅ΡΡ‚Π²ΠΎΠ²Π°Ρ‚ΡŒ, ΠΏΠΎΠΊΠ° Π΅Π΅ Π½Π΅ замСнят Ρ€ΠΎΠ±ΠΎΡ‚Ρ‹.
А ΠΏΠΎΡ‚ΠΎΠΌ всСм Π₯олокост. НС Π»ΠΆΠΈΠ²Ρ‹ΠΉ СврСйский, Π° Ρ€Π΅Π°Π»ΡŒΠ½ΠΎΠ΅ всСсоТТСниС Ρ€ΠΎΠ΄Π° чСловСчСского.

Если ΠΊΡ‚ΠΎ пораскинСт своими обСзьяньими ΠΌΠΎΠ·Π³Π°ΠΌΠΈ, Ρ‚ΠΎ ΠΏΠΎΠΉΠΌΡ‘Ρ‚, Ρ‡Ρ‚ΠΎ ΡΠ²ΠΎΠ»ΡŽΡ†ΠΈΡ — Π΅ΡΡ‚ΡŒ синоним Π³Π΅Π½ΠΎΡ†ΠΈΠ΄Π°:
Π½ΠΎΠ²ΠΎΠ΅ замСняСт, Ρ‚ΠΎ Π΅ΡΡ‚ΡŒ Π»ΠΈΠΊΠ²ΠΈΠ΄ΠΈΡ€ΡƒΠ΅Ρ‚ староС.
ΠžΠ±Π΅Π·ΡŒΡΠ½Ρ‹ ΠΏΠΎΡ€ΠΎΠ΄ΠΈΠ»ΠΈ Π½Π΅Π°Π½Π΄Π΅Ρ€Ρ‚Π°Π»ΡŒΡ†Π΅Π².
ΠΠ΅Π°Π½Π΄Π΅Ρ€Ρ‚Π°Π»ΡŒΡ†Ρ‹ съСли обСзьян ΠΈ ΠΏΠΎΡ€ΠΎΠ΄ΠΈΠ»ΠΈ людСй.
Π›ΡŽΠ΄ΠΈ вытСснили обСзьян, Π²ΠΊΠ»ΡŽΡ‡Π°Ρ ΠΈ ΡƒΠΌΠ½Ρ‹Ρ… Π½Π΅Π°Π½Π΄Π΅Ρ€Ρ‚Π°Π»ΡŒΡ†Π΅Π², ΠΈ ΠΏΠΎΡ€ΠΎΠ΄ΠΈΠ»ΠΈ ИИ.
ИИ Π»ΠΈΠΊΠ²ΠΈΠ΄ΠΈΡ€ΡƒΠ΅Ρ‚ людСй.

Π’ Московском государствСнном унивСрситСтС ΠΈΠΌΠ΅Π½ΠΈ Ломоносова осущСствляСтся ΠΏΡ€ΠΎΠ΅ΠΊΡ‚ ΠΏΠΎ созданию ΠΏΡ€Π΅Π·Π΅Π½Ρ‚Π°Ρ‚ΠΎΡ€ΠΎΠ² 50-ΠΊΡƒΠ±ΠΈΡ‚Π½Ρ‹Ρ… ΠΊΠ²Π°Π½Ρ‚ΠΎΠ²Ρ‹Ρ… ΠΊΠΎΠΌΠΏΡŒΡŽΡ‚Π΅Ρ€ΠΎΠ² ΠΊ 2021 Π³ΠΎΠ΄Ρƒ. Основой для Π½ΠΈΡ…Π°Ρ‚ Π½Π΅ΠΉΡ‚Ρ€Π°Π»ΡŒΠ½Ρ‹Π΅ Π°Ρ‚ΠΎΠΌΡ‹ ΠΈ ΠΈΠ½Ρ‚Π΅Π³Ρ€Π°Π»ΡŒΠ½Ρ‹Π΅ оптичСскиС схСмы. Над Ρ‡Π΅ΠΌ сСйчас Ρ€Π°Π±ΠΎΡ‚Π°ΡŽΡ‚ ΡƒΡ‡Π΅Π½Ρ‹Π΅?

Π’ Π½Π°Ρ‡Π°Π»Π΅ этого Π³ΠΎΠ΄Π° ΠΏΠΎ Π΄Π°Π½Π½ΠΎΠΉ ΠΏΡ€ΠΎΠ³Ρ€Π°ΠΌΠΌΠ΅ Π±Ρ‹Π» ΡƒΡΠΏΠ΅ΡˆΠ½ΠΎ Π²Ρ‹ΠΏΠΎΠ»Π½Π΅Π½ ΠΊΠΎΠ½Ρ‚Ρ€ΠΎΠ»ΡŒΠ½Ρ‹ΠΉ экспСримСнт ΠΏΠΎ созданию Π»ΠΎΠ²ΡƒΡˆΠ΅ΠΊ для массивов Π½Π΅ΠΉΡ‚Ρ€Π°Π»ΡŒΠ½Ρ‹Ρ… Ρ…ΠΎΠ»ΠΎΠ΄Π½Ρ‹Ρ… элСмСнтов.Он проводился Π½Π° Π±Π°Π·Π΅ Π»Π°Π±ΠΎΡ€Π°Ρ‚ΠΎΡ€ΠΈΠΈ ΠΊΠ²Π°Π½Ρ‚ΠΎΠ²Ρ‹Ρ… оптичСских Ρ‚Π΅Ρ…Π½ΠΎΠ»ΠΎΠ³ΠΈΠΉ физичСского Ρ„Π°ΠΊΡƒΠ»ΡŒΡ‚Π΅Ρ‚Π° ΠœΠ“Π£. Π’ Π±ΡƒΠ΄ΡƒΡ‰Π΅ΠΌ ΠΊΠ²Π°Π½Ρ‚ΠΎΠ²ΠΎΠΌ ΠΊΠΎΠΌΠΏΡŒΡŽΡ‚Π΅Ρ€Π΅ Π² этих Π»ΠΎΠ²ΡƒΡˆΠΊΠ°Ρ… Π±ΡƒΠ΄ΡƒΡ‚ Ρ„ΠΈΠΊΡΠΈΡ€ΠΎΠ²Π°Ρ‚ΡŒΡΡ Π°Ρ‚ΠΎΠΌΡ‹, находящиСся Π² состоянии хаотичСского двиТСния. Π’ ΠΊΠ²Π°Π½Ρ‚ΠΎΠ²Ρ‹Ρ… ΠΊΠΎΠΌΠΏΡŒΡŽΡ‚Π΅Ρ€Π°Ρ… Ρ‚Π°ΠΊΠΈΠ΅ Π°Ρ‚ΠΎΠΌΡ‹ ΡΠ²Π»ΡΡŽΡ‚ΡΡ носитСлями ΠΈΠ½Ρ„ΠΎΡ€ΠΌΠ°Ρ†ΠΈΠΈ.
Π‘Ρ‚ΠΎΠΈΡ‚ Π·Π°ΠΌΠ΅Ρ‚ΠΈΡ‚ΡŒ, Ρ‡Ρ‚ΠΎ ΠΊΠ²Π°Π½Ρ‚ΠΎΠ²Ρ‹Π΅ ΠΊΠΎΠΌΠΏΡŒΡŽΡ‚Π΅Ρ€Ρ‹ Π² ΠΎΡ‚Π»ΠΈΡ‡ΠΈΠ΅ ΠΎΡ‚ классичСских Π²Ρ‹Ρ‡ΠΈΡΠ»ΠΈΡ‚Π΅Π»ΡŒΠ½Ρ‹Ρ… машин ΠΎΠΏΠ΅Ρ€ΠΈΡ€ΡƒΡŽΡ‚ Π½Π΅ Π±ΠΈΡ‚Π°ΠΌΠΈ, Π° ΠΊΡƒΠ±ΠΈΡ‚Π°ΠΌΠΈ, ΠΊΠΎΡ‚ΠΎΡ€Ρ‹Π΅ находятся Π½Π΅ Ρ‚ΠΎΠ»ΡŒΠΊΠΎ Π² состояниях Β«1Β» ΠΈ Β«0Β», Π½ΠΎ ΠΈ ΠΈΡ… супСрпозиции. ΠŸΡ€ΠΈ Ρ€Π°Π·Ρ€Π°Π±ΠΎΡ‚ΠΊΠ΅ ΠΊΠ²Π°Π½Ρ‚ΠΎΠ² Π²Ρ‹Ρ‡ΠΈΡΠ»ΠΈΡ‚Π΅Π»ΡŒΠ½Ρ‹Ρ… устройств ΡƒΡ‡Π΅Π½Ρ‹Π΅ ΡΡ‚Π°Ρ€Π°ΡŽΡ‚ΡΡ ΠΊΡƒΠ±ΠΈΡ‚Ρ‹ Π² состоянии ΠΊΠ²Π°Π½Ρ‚ΠΎΠ²ΠΎΠΉ запутанности.Π‘ΡƒΡ‚ΡŒ явлСния Π·Π°ΠΊΠ»ΡŽΡ‡Π°Π΅Ρ‚ΡΡ Π² Ρ‚ΠΎΠΌ, Ρ‡Ρ‚ΠΎ ΠΈΠ·ΠΌΠ΅Π½Π΅Π½ΠΈΠ΅ ΠΎΠ΄Π½ΠΎΠ³ΠΎ ΠΊΡƒΠ±ΠΈΡ‚Π° всСгда влияСт Π½Π° состояниС связанных с Π½ΠΈΠΌ сосСдСй. Благодаря этому ΠΊΠ²Π°Π½Ρ‚ΠΎΠ²Ρ‹Π΅ ΠΊΠΎΠΌΠΏΡŒΡŽΡ‚Π΅Ρ€Ρ‹ ΠΌΠΎΠ³ΡƒΡ‚ Π΄Π΅ΠΌΠΎΠ½ΡΡ‚Ρ€ΠΈΡ€ΠΎΠ²Π°Ρ‚ΡŒ ΠΏΡ€ΠΎΠΈΠ·Π²ΠΎΠ΄ΠΈΡ‚Π΅Π»ΡŒΠ½ΡƒΡŽ ΠΏΡ€ΠΎΠΈΠ·Π²ΠΎΠ΄ΠΈΡ‚Π΅Π»ΡŒΠ½ΠΎΡΡ‚ΡŒ Π² Π²Ρ‹Ρ‡ΠΈΡΠ»ΠΈΡ‚Π΅Π»ΡŒΠ½Ρ‹Ρ… систСмах.
Π’Π°ΠΆΠ½ΠΎΠΉ Π²Π΅Ρ…ΠΎΠΉ для ΠΊΠ²Π°Π½Ρ‚ΠΎΠ²Ρ‹Ρ… Ρ‚Π΅Ρ…Π½ΠΎΠ»ΠΎΠ³ΠΈΠΉ достиТСниС Ρ‚Π°ΠΊ Π½Π°Π·Ρ‹Π²Π°Π΅ΠΌΠΎΠ³ΠΎ ΠΊΠ²Π°Π½Ρ‚ΠΎΠ²ΠΎΠ³ΠΎ прСвосходства. Π“Π»Π°Π²Π½ΠΎΠΉ ΠΏΡ€ΠΎΠ±Π»Π΅ΠΌΠΎΠΉ Π½Π° Ρ‚Π΅ΠΊΡƒΡ‰Π΅ΠΌ этапС развития ΠΊΠ²Π°Π½Ρ‚ΠΎΠ²Ρ‹Ρ… Ρ‚Π΅Ρ…Π½ΠΎΠ»ΠΎΠ³ΠΈΠΉ являСтся Π²ΠΎΠ·Π½ΠΈΠΊΠ½ΠΎΠ²Π΅Π½ΠΈΠ΅ Π² процСссС Ρ€Π°Π±ΠΎΡ‚Ρ‹ большого количСства ошибок, Π½ΡƒΠΆΠ΄Π°ΡŽΡ‰ΠΈΡ…ΡΡ Π² ΠΊΠΎΡ€Ρ€Π΅ΠΊΡ†ΠΈΠΈ, — ΡΠΎΠΎΠ±Ρ‰Π°ΡŽΡ‚ российскиС исслСдоватСли.

ЭлСктричСскиС Ρ†Π΅ΠΏΠΈ, Π² ΠΊΠΎΡ‚ΠΎΡ€Ρ‹Ρ… ΠΎΠ΄Π½Π° Ρ‡Π°ΡΡ‚ΡŒ сопротивлСний соСдинСна ΠΏΠΎΡΠ»Π΅Π΄ΠΎΠ²Π°Ρ‚Π΅Π»ΡŒΠ½ΠΎ, Π° другая ΠΏΠ°Ρ€Π°Π»Π»Π΅Π»ΡŒΠ½Π°Ρ Ρ†Π΅ΠΏΡŒ со ΡΠΌΠ΅ΡˆΠ°Π½Π½Ρ‹ΠΌ соСдинСниСм сопротивлСний.

ΠžΠ±Ρ‰ΠΈΡ… расчСтных Ρ„ΠΎΡ€ΠΌΡƒΠ» для Ρ‚Π°ΠΊΠΈΡ… Ρ†Π΅ΠΏΠ΅ΠΉ Π½Π΅Ρ‚, Ρ‚Π°ΠΊ ΠΊΠ°ΠΊ число ΠΈΡ… разновидностСй Π½Π΅ ΠΎΠ³Ρ€Π°Π½ΠΈΡ‡Π΅Π½ΠΎ.

Π§Π°Ρ‰Π΅ всСго расчСтных схСм начинаСтся с опрСдСлСния эквивалСнтного сопротивлСния всСй Ρ†Π΅ΠΏΠΈ, Π° Π·Π°Ρ‚Π΅ΠΌ значСния Ρ‚ΠΎΠΊΠΎΠ² ΠΈ ΠΏΠ°Π΄Π΅Π½ΠΈΠ΅ напряТСния Π½Π° ΠΎΡ‚Π΄Π΅Π»ΡŒΠ½Ρ‹Ρ… участках.

Для опрСдСлСния эквивалСнтного сопротивлСния Ρ†Π΅ΠΏΠΈ со ΡΠΌΠ΅ΡˆΠ°Π½Π½Ρ‹ΠΌ соСдинСниСм ΠΏΠΎΡ‚Ρ€Π΅Π±ΠΈΡ‚Π΅Π»Π΅ΠΉ, ΠΏΠΈΡ‚Π°ΡŽΡ‰ΠΈΡ…ΡΡ ΠΎΡ‚ ΠΎΠ΄Π½ΠΎΠ³ΠΎ источника Ρ‚ΠΎΠΊΠ°, Π½Π΅ΠΎΠ±Ρ…ΠΎΠ΄ΠΈΠΌΠΎ всСго Ρ€Π°Π·Π±ΠΈΡ‚ΡŒ эту Ρ†Π΅ΠΏΡŒ Π½Π° ΠΎΡ‚Π΄Π΅Π»ΡŒΠ½Ρ‹Π΅ участки, состоящиС ΠΈΠ· ΠΏΠΎΡΠ»Π΅Π΄ΠΎΠ²Π°Ρ‚Π΅Π»ΡŒΠ½ΠΎΠ³ΠΎ ΠΈ ΠΏΠΎΡΠ»Π΅Π΄ΠΎΠ²Π°Ρ‚Π΅Π»ΡŒΠ½ΠΎΠ³ΠΎ сопротивлСния.Π”Π°Π»Π΅Π΅ ΠΎΠΏΡ€Π΅Π΄Π΅Π»ΡΡŽΡ‚ эквивалСнтныС сопротивлСния для ΠΊΠ°ΠΆΠ΄ΠΎΠ³ΠΎ ΠΈΠ· участков, Π° Π·Π°Ρ‚Π΅ΠΌ ΠΈ для всСй Ρ†Π΅ΠΏΠΈ Π² Ρ†Π΅Π»ΠΎΠΌ.

Рассмотрим ΠΌΠ΅Ρ‚ΠΎΠ΄ Ρ€Π΅ΡˆΠ΅Π½ΠΈΡ Π·Π°Π΄Π°Ρ‡ Π½Π° смСшанноС сопротивлСниС сопротивлСний Π½Π° ΠΊΠΎΠ½ΠΊΡ€Π΅Ρ‚Π½ΠΎΠΌ ΠΏΡ€ΠΈΠΌΠ΅Ρ€Π΅.

На рисункС прСдставлСна ​​схСма смСшанного соСдинСния сопротивлСний. Π•Π΅ ΠΌΠΎΠΆΠ½ΠΎ Ρ€Π°Π·Π±ΠΈΡ‚ΡŒ Π½Π° Ρ‚Ρ€ΠΈ участка:

участок АВ — с ΠΏΠ°Ρ€Π°Π»Π»Π΅Π»ΡŒΠ½Ρ‹ΠΌΠΈ вСтвями;

участок Π’Π‘ — с соСдинСнными сопротивлСниями;

участок Π‘D — с Ρ‚Ρ€Π΅Ρ… ΠΏΠ°Ρ€Π°Π»Π»Π΅Π»ΡŒΠ½Ρ‹Ρ… вСтвями.

ΠšΡ€ΠΎΠΌΠ΅ Ρ‚ΠΎΠ³ΠΎ, ниТняя Π²Π΅Ρ‚Π²ΡŒ участка АВ прСдставляСт свою ΠΎΡ‡Π΅Ρ€Π΅Π΄ΡŒ, ΡΠΎΡΡ‚ΠΎΡΡ‰ΡƒΡŽ ΠΈΠ· Π΄Π²ΡƒΡ… соСдинСнных сопротивлСний R 2 ΠΈ R 3 .

Π¦Π΅Π½Ρ‚Ρ€Π°Π»ΡŒΠ½Π°Ρ Π²Π΅Ρ‚Π²ΡŒ участка Π‘D прСдставляСт собой смСшанноС соСдинСниС сопротивлСний.

РасчСт Π΄Π°Π½Π½ΠΎΠΉ слоТной Ρ†Π΅ΠΏΠΈ Π½Π°Π΄ΠΎ Π½Π°Ρ‡ΠΈΠ½Π°Ρ‚ΡŒ с опрСдСлСния R экв для Π½ΠΈΠΆΠ½Π΅ΠΉ части участка АВ ΠΈ Ρ†Π΅Π½Ρ‚Ρ€Π°Π»ΡŒΠ½ΠΎΠΉ Π²Π΅Ρ‚Π²ΠΈ участка Π‘D.

Π’Π΅ΠΏΠ΅Ρ€ΡŒ ΠΌΡ‹ ΠΌΠΎΠΆΠ΅ΠΌ ΡƒΠΏΡ€ΠΎΡΡ‚ΠΈΡ‚ΡŒ ΠΏΠ΅Ρ€Π²ΠΎΠ½Π°Ρ‡Π°Π»ΡŒΠ½ΡƒΡŽ схСму.Она Π±ΡƒΠ΄Π΅Ρ‚ ΠΈΠΌΠ΅Ρ‚ΡŒ ΡΠ»Π΅Π΄ΡƒΡŽΡ‰ΠΈΠΉ Π²ΠΈΠ΄

ΠžΠΏΡ€Π΅Π΄Π΅Π»ΠΈΠΌ эквивалСнтныС сопротивлСния ΠΊΠ°ΠΆΠ΄ΠΎΠ³ΠΎ ΠΈΠ· участков:

ПослС этих вычислСний ΠΌΠΎΠΆΠ½ΠΎ ΠΏΡ€ΠΎΠ΄ΠΎΠ»ΠΆΠΈΡ‚ΡŒ ΡƒΠΏΡ€ΠΎΡ‰Π΅Π½ΠΈΠ΅ схСмы

ΠŸΠΎΠ»ΡƒΡ‡Π΅Π½Π½Π°Ρ упрощСнная схСма, состоящая Π² Π΄Π°Π½Π½ΠΎΠΌ случаС Ρ‚Ρ€Π΅Ρ… соСдинитСлСй, называСтся ΠΏΠΎ ΠΎΡ‚Π½ΠΎΡˆΠ΅Π½ΠΈΡŽ ΠΊ Ρ€Π΅Π°Π»ΡŒΠ½ΠΎΠΉ эквивалСнтной схСмой.

ΠžΠΏΡ€Π΅Π΄Π΅Π»ΠΈΠΌ R экв всСй Ρ†Π΅ΠΏΠΈ ΠΊΠ°ΠΊ сумма Ρ‚Ρ€Π΅Ρ… послСдних сопротивлСний

Π—Π½ΠΎΠ΅ напряТСниС источника Ρ‚ΠΎΠΊΠ°, примСняя Ρ„ΠΎΡ€ΠΌΡƒΠ»Ρƒ Π·Π°ΠΊΠΎΠ½Π° Ома, ΠΎΠΏΡ€Π΅Π΄Π΅Π»ΠΈΠΌ Ρ‚ΠΎΠΊ Π² Π½Π΅ Ρ€Π°Π·Π²Π΅Ρ‚Π²Π»Π΅Π½Π½ΠΎΠΌ участкС

ΠžΠΏΡ€Π΅Π΄Π΅Π»ΠΈΠ² созданиС Ρ‚ΠΎΠΊΠ°, Π½Π°ΠΉΠ΄Π΅ΠΌ ΠΏΠ°Π΄Π΅Π½ΠΈΠ΅ напряТСния Π½Π° участках эквивалСнтной схСмы АВ, Π’Π‘, CD:

Π’Π΅ΠΏΠ΅Ρ€ΡŒ ΠΌΠΎΠΆΠ½ΠΎ ΠΎΠΏΡ€Π΅Π΄Π΅Π»ΠΈΡ‚ΡŒ Ρ‚ΠΎΠΊΠΈ Π² ΠΏΠ°Ρ€Π°Π»Π»Π΅Π»ΡŒΠ½Ρ‹Ρ… вСтвях участков АВ ΠΈ Π‘D

ΠžΡΡ‚Π°Π΅Ρ‚ΡΡ ΠΎΠΏΡ€Π΅Π΄Π΅Π»ΠΈΡ‚ΡŒ ΠΎΠΏΡ€Π΅Π΄Π΅Π»Π΅Π½ΠΈΠ΅ Ρ‚ΠΎΠΊΠΎΠ², ΠΏΡ€ΠΎΡ‚Π΅ΠΊΠ°ΡŽΡ‰ΠΈΡ… Ρ‡Π΅Ρ€Π΅Π· сопротивлСниС R 7 ΠΈ R 8. Для этого Π½Π°Π΄ΠΎ сначала ΠΎΠΏΡ€Π΅Π΄Π΅Π»ΠΈΡ‚ΡŒ ΠΏΠ°Π΄Π΅Π½ΠΈΠ΅ напряТСния Π½Π° сопротивлСнии R 7 ΠΈ R 8.

ΠžΠΏΡ€Π΅Π΄Π΅Π»ΠΈΠΌ ΠΏΠ°Π΄Π΅Π½ΠΈΠ΅ напряТСния Π½Π° сопротивлСнии R 9 :

ПадСниС напряТСния Π½Π° сопротивлСнии R 7,8 опрСдСлится ΠΊΠ°ΠΊ Ρ€Π°Π·Π½ΠΎΡΡ‚ΡŒ U CD ΠΈ U:

Π’Π΅ΠΏΠ΅Ρ€ΡŒ ΠΎΠΏΡ€Π΅Π΄Π΅Π»ΠΈΠΌ Π²Π΅Π»ΠΈΡ‡ΠΈΠ½Ρƒ Ρ‚ΠΎΠΊΠΎΠ², ΠΏΡ€ΠΎΡ‚Π΅ΠΊΠ°ΡŽΡ‰ΠΈΡ… Ρ‡Π΅Ρ€Π΅Π· сопротивлСниС R 7 ΠΈ R 8 :

Π’Π΅Π»ΠΈΡ‡ΠΈΠ½Π° Ρ‚ΠΎΠΊΠ°. ΠΏΡ€ΠΎΡ‚Π΅ΠΊΠ°ΡŽΡ‰Π΅Π³ΠΎ Ρ‡Π΅Ρ€Π΅Π· сопротивлСниС R 4 ΠΈ R 5 , Ρ€Π°Π²Π½Π° I — Ρ‚ΠΎΠΊΡƒ Π² Π½Π΅Ρ€Π°Π·Π²Π΅Ρ‚Π²Π»Π΅Π½Π½ΠΎΠΌ участкС Ρ†Π΅ΠΏΠΈ.

Π˜Ρ‚Π°ΠΊ, ΠΏΡ€ΠΈ Ρ€Π΅ΡˆΠ΅Π½ΠΈΠΈ Π·Π°Π΄Π°Ρ‡ Π½Π° смСшанноС сопротивлСниС сопротивлСний, постСпСнно упрощая схСму, ΠΎΠΏΡ€Π΅Π΄Π΅Π»Π΅Π½ΠΈΠ΅ сопротивлСния всСй Ρ†Π΅ΠΏΠΈ, Π°. восстанавливая постСпСнно Ρ€Π΅Π°Π»ΡŒΠ½ΡƒΡŽ схСму. Π²Ρ‹Ρ‡ΠΈΡΠ»ΠΈΡ‚ΡŒ ΠΏΠ°Π΄Π΅Π½ΠΈΠ΅ напряТСния ΠΈ Ρ‚ΠΎΠΊΠΈ Π² ΠΎΡ‚Π΄Π΅Π»ΡŒΠ½Ρ‹Ρ… вСтвях.

ΠŸΠΎΡΠ»Π΅Π΄ΠΎΠ²Π°Ρ‚Π΅Π»ΡŒΠ½ΠΎΠ΅ ΠΏΠ°Ρ€Π°Π»Π»Π΅Π»ΡŒΠ½ΠΎΠ΅ соСдинСниС рСзисторов

Π€ΠΈΠ·ΠΈΠΊΠ° > ΠšΠΎΠΌΠ±ΠΈΠ½ΠΈΡ€ΠΎΠ²Π°Π½Π½Ρ‹Π΅ схСмы

ΠŸΠΎΡΠ»Π΅Π΄ΠΎΠ²Π°Ρ‚Π΅Π»ΡŒΠ½ΠΎΠ΅ ΠΏΠ°Ρ€Π°Π»Π»Π΅Π»ΡŒΠ½ΠΎΠ΅ соСдинСниС рСзисторов — ΠΊΠΎΠΌΠ±ΠΈΠ½ΠΈΡ€ΠΎΠ²Π°Π½Π½Ρ‹Π΅ схСмы элСктричСской Ρ†Π΅ΠΏΠΈ: ΡΠΌΠ΅ΡˆΠ°Π½Π½Ρ‹ΠΉ Ρ‚ΠΈΠΏ соСдинСния, схСмы, Π·Π°ΠΊΠΎΠ½Ρ‹, Ρ€Π΅ΡˆΠ΅Π½ΠΈΠ΅ Π·Π°Π΄Π°Ρ‡.

ΠšΠΎΠΌΠ±ΠΈΠ½ΠΈΡ€ΠΎΠ²Π°Π½Π½ΡƒΡŽ схСму ΠΌΠΎΠΆΠ½ΠΎ Ρ€Π°Π·Π΄Π΅Π»ΠΈΡ‚ΡŒ Π½Π° ΠΏΠΎΡΠ»Π΅Π΄ΠΎΠ²Π°Ρ‚Π΅Π»ΡŒΠ½ΡƒΡŽ ΠΈΠ»ΠΈ ΠΏΠ°Ρ€Π°Π»Π»Π΅Π»ΡŒΠ½ΡƒΡŽ части.

Π—Π°Π΄Π°Ρ‡Π° обучСния

  • Π Π°ΡΡΠΌΠΎΡ‚Ρ€Π΅Ρ‚ΡŒ процСсс раздСлСния рСзисторов Π² ΠΊΠΎΠΌΠ±ΠΈΠ½ΠΈΡ€ΠΎΠ²Π°Π½Π½ΠΎΠΉ схСмС.

ΠžΡΠ½ΠΎΠ²Π½Ρ‹Π΅ области

  • Π‘Π»ΠΎΠΆΠ½Ρ‹Π΅ соСдинСния рСзисторов Π΄Π΅ΠΌΠΎΠ½ΡΡ‚Ρ€ΠΈΡ€ΡƒΡŽΡ‚ использованиС ΠΏΠ°Ρ€Π°Π»Π»Π΅Π»ΡŒΠ½ΠΎΠ³ΠΎ ΠΈ ΠΏΠΎΡΠ»Π΅Π΄ΠΎΠ²Π°Ρ‚Π΅Π»ΡŒΠ½ΠΎΠ³ΠΎ объСдинСния.
  • Π Π°Π·Π½Ρ‹Π΅ части ΠΊΠΎΠΌΠ±ΠΈΠ½ΠΈΡ€ΠΎΠ²Π°Π½Π½ΠΎΠΉ схСмы ΠΌΠΎΠΆΠ½ΠΎ Ρ€Π°Π·Π΄Π΅Π»ΠΈΡ‚ΡŒ ΠΈ ΡƒΠΌΠ΅Π½ΡŒΡˆΠΈΡ‚ΡŒ, ΠΏΠΎΠΊΠ° Π½Π΅ останСтся Π΅Π΄ΠΈΠ½ΠΎΠ΅ сопротивлСниС.
  • Π‘ΠΎΠΏΡ€ΠΎΡ‚ΠΈΠ²Π»Π΅Π½ΠΈΠ΅ Π² ΠΏΡ€ΠΎΠ²ΠΎΠ΄Π°Ρ… сокращаСт Ρ‚ΠΎΠΊ ΠΈ ΠΌΠΎΡ‰Π½ΠΎΡΡ‚ΡŒ, отправляСмыС ΠΊ рСзистору.
  • Если сопротивлСниС Π² изношСнном ΠΈΠ»ΠΈ слишком Π΄Π»ΠΈΠ½Π½ΠΎΠΌ ΡˆΠ½ΡƒΡ€Π΅, ΠΏΠΎΡ‚Π΅Ρ€ΠΈ ΠΌΠΎΠ³ΡƒΡ‚ Π±Ρ‹Ρ‚ΡŒ ΠΎΡ‰ΡƒΡ‚ΠΈΠΌΡ‹ΠΌΠΈ.

Π’Π΅Ρ€ΠΌΠΈΠ½Ρ‹

  • ΠŸΠΎΡΠ»Π΅Π΄ΠΎΠ²Π°Ρ‚Π΅Π»ΡŒΠ½ΠΎΡΡ‚ΡŒ — ΠΏΠΎΠ΄ΠΊΠ»ΡŽΡ‡Π΅Π½ΠΈΠ΅ Π½Π΅ΡΠΊΠΎΠ»ΡŒΠΊΠΈΡ… элСмСнтов ΠΏΠΎΠΎΡ‡Π΅Ρ€Π΅Π΄Π½ΠΎ.
  • ΠŸΠ°Ρ€Π°Π»Π»Π΅Π»ΡŒΠ½ΠΎΡΡ‚ΡŒ — приспособлСна Ρ‚Π°ΠΊ, Ρ‡Ρ‚ΠΎΠ±Ρ‹ Ρ‚ΠΎΠΊ ΠΏΡ€ΠΎΡ‚Π΅ΠΊΠ°Π» вдоль Π΄Π²ΡƒΡ… ΠΈΠ»ΠΈ Π±ΠΎΠ»Π΅Π΅ ΠΏΡƒΡ‚Π΅ΠΉ.
  • ΠšΠΎΠΌΠ±ΠΈΠ½ΠΈΡ€ΠΎΠ²Π°Π½Π½Π°Ρ схСма — элСктричСская схСма с ΠΏΠΎΡΠ»Π΅Π΄ΠΎΠ²Π°Ρ‚Π΅Π»ΡŒΠ½Ρ‹ΠΌΠΈ рСзисторами, ΠΎΠ±ΡŠΠ΅Π΄ΠΈΠ½Π΅Π½Π½Ρ‹ΠΌΠΈ ΠΏΠΎΡΠ»Π΅Π΄ΠΎΠ²Π°Ρ‚Π΅Π»ΡŒΠ½Ρ‹ΠΌΠΈ ΠΈ ΠΏΠΎΡΠ»Π΅Π΄ΠΎΠ²Π°Ρ‚Π΅Π»ΡŒΠ½Ρ‹ΠΌΠΈ.

ΠšΠΎΠΌΠ±ΠΈΠ½ΠΈΡ€ΠΎΠ²Π°Π½Π½Ρ‹Π΅ схСмы

Иногда слоТныС соСдинСния ΠΈΡΠΏΠΎΠ»ΡŒΠ·ΡƒΡŽΡ‚ сразу ΠΏΠΎΡΠ»Π΅Π΄ΠΎΠ²Π°Ρ‚Π΅Π»ΡŒΠ½ΠΎΠ΅ ΠΈ ΠΏΠ°Ρ€Π°Π»Π»Π΅Π»ΡŒΠ½ΠΎΠ΅ ΠΏΠΎΠ΄ΠΊΠ»ΡŽΡ‡Π΅Π½ΠΈΠ΅. Π­Ρ‚ΠΎ частоС явлСниС, Ρ‚Π°ΠΊ ΠΊΠ°ΠΊ Π² ΠΏΡ€ΠΎΠ²ΠΎΠ΄Π΅ Π²ΠΎΠ·Π½ΠΈΠΊΠ°Π΅Ρ‚ сопротивлСниС. Π’Π°ΠΊΡƒΡŽ схСму ΠΌΠΎΠΆΠ½ΠΎ Ρ€Π°Π·Π΄Π΅Π»ΠΈΡ‚ΡŒ Π½Π° ΡΠΎΠΎΡ‚Π²Π΅Ρ‚ΡΡ‚Π²ΡƒΡŽΡ‰ΠΈΠ΅ части. На рисункС ΠΏΠΎΠΊΠ°Π·Π°Π½ΠΎ, Ρ‡Ρ‚ΠΎ ΠΎΠ±Ρ‰Π΅Π΅ сопротивлСниС соСдиняСт, связав Ρ‚Ρ€ΠΈ рСзистора ΠΈΠ»ΠΈ ΠΏΠΎΡΠ»Π΅Π΄ΠΎΠ²Π°Ρ‚Π΅Π»ΡŒΠ½ΠΎ ΡΠΎΠ΅Π΄ΠΈΠ½ΡΡŽΡ‰ΠΈΡ…. R 1 ΠΈ R 2 ΠΏΠΎΠ΄ΠΊΠ»ΡŽΡ‡Π΅Π½Ρ‹ ΠΏΠ°Ρ€Π°Π»Π»Π΅Π»ΡŒΠ½ΠΎ, поэтому Ρ„ΠΎΡ€ΠΌΡƒΠ»Π°:

Π’ ΠΊΠΎΠΌΠ±ΠΈΠ½ΠΈΡ€ΠΎΠ²Π°Π½Π½ΠΎΠΉ схСмС рСзисторы ΠΌΠΎΠΆΠ½ΠΎ Ρ€Π°Π·Π΄Π΅Π»ΠΈΡ‚ΡŒ ΠΏΠΎΡΠ»Π΅Π΄ΠΎΠ²Π°Ρ‚Π΅Π»ΡŒΠ½Ρ‹Π΅ ΠΈ ΠΏΠ°Ρ€Π°Π»Π»Π΅Π»ΡŒΠ½Ρ‹Π΅ ΡΠΎΡΡ‚Π°Π²Π»ΡΡŽΡ‰ΠΈΠ΅

R 3 соСдинСн ΠΏΠΎΡΠ»Π΅Π΄ΠΎΠ²Π°Ρ‚Π΅Π»ΡŒΠ½ΠΎ с R 1 ΠΈ R 2 , сопротивлСниС Π±ΡƒΠ΄Π΅Ρ‚ Ρ€Π°ΡΡΡ‡ΠΈΡ‚Ρ‹Π²Π°Ρ‚ΡŒΡΡ ΠΊΠ°ΠΊ:

ΠšΠΎΠΌΠΏΠ»Π΅ΠΊΡΠ½Ρ‹Π΅ ΠΊΠΎΠΌΠ±ΠΈΠ½Π°Ρ†ΠΈΠΎΠ½Π½Ρ‹Π΅ схСмы

Π’ ситуациях с Π±ΠΎΠ»Π΅Π΅ слоТными схСмами части ΠΌΠΎΠΆΠ½ΠΎ ΠΈΠ΄Π΅Π½Ρ‚ΠΈΡ„ΠΈΡ†ΠΈΡ€ΠΎΠ²Π°Ρ‚ΡŒ ΠΊΠ°ΠΊ ΠΏΠΎΡΠ»Π΅Π΄ΠΎΠ²Π°Ρ‚Π΅Π»ΡŒΠ½Ρ‹Π΅ ΠΈ ΠΏΠ°Ρ€Π°Π»Π»Π΅Π»ΡŒΠ½Ρ‹Π΅, Π° Π·Π°Ρ‚Π΅ΠΌ свСсти ΠΈΡ… ΠΊ эквивалСнтам ΠΈ ΡƒΠΌΠ΅Π½ΡŒΡˆΠΈΡ‚ΡŒ ΠΊ СдинствСнному ΡΠΎΠΏΡ€ΠΎΡ‚ΠΈΠ²Π»Π΅Π½ΠΈΡŽ.На Π½ΠΈΠΆΠ½Π΅ΠΌ рисункС Π²ΠΈΠ΄Π½ΠΎ, Ρ‡Ρ‚ΠΎ комбинация ΠΈΠ· 7 рСзисторов ΠΎΠΏΡ€Π΅Π΄Π΅Π»ΠΈΠ»ΠΈ ΠΊΠ°ΠΊ ΠΏΠΎΡΠ»Π΅Π΄ΠΎΠ²Π°Ρ‚Π΅Π»ΡŒΠ½ΡƒΡŽ ΠΈ ΠΏΠ°Ρ€Π°Π»Π»Π΅Π»ΡŒΠ½ΡƒΡŽ. На Π½Π°Ρ‡Π°Π»ΡŒΠ½ΠΎΠΌ этапС Π΄Π²Π° ΠΎΠΊΡ€ΡƒΠΆΠ½Ρ‹Ρ… сСчСния ΠΎΡ‚ΠΎΠ±Ρ€Π°ΠΆΠ°ΡŽΡ‚ рСзисторы, располоТСнныС ΠΏΠ°Ρ€Π°Π»Π»Π΅Π»ΡŒΠ½ΠΎ.

ΠšΠΎΠΌΠ±ΠΈΠ½Π°Ρ†ΠΈΡ ΠΈΠ· 7 рСзисторов ΠΏΠΎΡΠ»Π΅Π΄ΠΎΠ²Π°Ρ‚Π΅Π»ΡŒΠ½Ρ‹ΠΌΠΈ ΠΈ ΠΏΠ°Ρ€Π°Π»Π»Π΅Π»ΡŒΠ½Ρ‹ΠΌΠΈ частями. КаТдая ΠΎΠΏΡ€Π΅Π΄Π΅Π»Π΅Π½Π° ΠΈ сводится ΠΊ эквивалСнтному ΡΠΎΠΏΡ€ΠΎΡ‚ΠΈΠ²Π»Π΅Π½ΠΈΡŽ, ΠΏΠΎΠΊΠ° Π½Π΅ достигнуто СдинствСнноС сопротивлСниС

УмСньшСниС ΠΏΠ°Ρ€Π°Π»Π»Π΅Π»ΡŒΠ½Ρ‹Ρ… рСзисторов Π΄ΠΎ ΠΎΠ΄Π½ΠΎΠ³ΠΎ значСния позволяСт Π·Π½Π°Ρ‡ΠΈΡ‚Π΅Π»ΡŒΠ½ΠΎ ΡƒΠΏΡ€ΠΎΡΡ‚ΠΈΡ‚ΡŒ схСму.Π‘ΠΏΡ€Π°Π²Π° Π²Π²Π΅Ρ€Ρ…Ρƒ Π²ΠΈΠ΄Π½ΠΎ, Ρ‡Ρ‚ΠΎ круглая Ρ‡Π°ΡΡ‚ΡŒ Π²ΠΌΠ΅Ρ‰Π°Π΅Ρ‚ Π΄Π²Π° ΠΏΠΎΡΠ»Π΅Π΄ΠΎΠ²Π°Ρ‚Π΅Π»ΡŒΠ½Ρ‹Ρ… рСзистора. МоТно ΡƒΠΌΠ΅Π½ΡŒΡˆΠΈΡ‚ΡŒ ΠΊ Π΄Ρ€ΡƒΠ³ΠΎΠΌΡƒ Π·Π½Π°Ρ‡Π΅Π½ΠΈΡŽ, Π΄ΠΎΠ±Π°Π²ΠΈΠ² ΠΈΡ…. Π”Π°Π»Π΅Π΅ Π²ΠΈΠ΄Π½ΠΎ, Ρ‡Ρ‚ΠΎ Π΄Π²Π° рСзистора ΠΏΠΎΡΠ»Π΅Π΄ΠΎΠ²Π°Ρ‚Π΅Π»ΡŒΠ½ΠΎΡΡ‚ΠΈ. Π˜Ρ… Ρ‚Π°ΠΊΠΆΠ΅ ΡΠΎΠΊΡ€Π°Ρ‰Π°ΡŽΡ‚ ΠΊ ΠΎΠ΄Π½ΠΎΠΌΡƒ Π·Π½Π°Ρ‡Π΅Π½ΠΈΡŽ.

Π‘ΠΎΠΏΡ€ΠΎΡ‚ΠΈΠ²Π»Π΅Π½ΠΈΠ΅ Π² ΠΏΡ€ΠΎΠ²ΠΎΠ΄Π°Ρ… ΡƒΠΌΠ΅Π½ΡŒΡˆΠ°Π΅Ρ‚ количСство Ρ‚ΠΎΠΊΠ° ΠΈ ΠΌΠΎΡ‰Π½ΠΎΡΡ‚ΡŒ, ΠΏΠΎΡΡ‚ΡƒΠΏΠ°ΡŽΡ‰ΠΈΠ΅ ΠΊ рСзистору. Для опрСдСлСния ΠΎΠ±Ρ‰Π΅Π³ΠΎ сопротивлСния Ρ†Π΅ΠΏΠΈ ΠΌΠΎΠΆΠ½ΠΎ ΠΈΡΠΏΠΎΠ»ΡŒΠ·ΠΎΠ²Π°Ρ‚ΡŒ Ρ†Π΅Π»Π΅Π²ΡƒΡŽ схСму. Π‘ΠΎΠΏΡ€ΠΎΡ‚ΠΈΠ²Π»Π΅Π½ΠΈΠ΅ ΠΏΡ€ΠΎΠ²ΠΎΠ΄Π° — сСрия рСзисторов. Если ΠΎΠ½ΠΎ слишком Π²Π΅Π»ΠΈΠΊΠΎ Π² Π΄Π»ΠΈΠ½Π½ΠΎΠΌ ΠΈΠ»ΠΈ ΠΏΠΎΠ²Ρ€Π΅ΠΆΠ΄Π΅Π½Π½ΠΎΠΌ ΠΏΡ€ΠΎΠ²ΠΎΠ΄Π΅, Ρ‚ΠΎ ΠΌΡ‹ ΠΎΡ‚ΠΌΠ΅Ρ‡Π°Π΅ΠΌ Π·Π½Π°Ρ‡ΠΈΡ‚Π΅Π»ΡŒΠ½ΡƒΡŽ ΠΏΠΎΡ‚Π΅Ρ€ΡŽ Ρ‚ΠΎΠΊΠ°.


ΠŸΠ΅Ρ‚Ρ€ΠΎΠ²ΠΈΡ‡ Π“.И. Π¦Π΅ΠΏΠΈ постоянного Ρ‚ΠΎΠΊΠ°-08-2

Рассмотрим Ρ€Π°Π·Π»ΠΈΡ‡Π½Ρ‹Π΅ соСдинСния элСктричСских сопротивлСний (рСзисторы, ΡΠ»Π΅ΠΊΡ‚Ρ€ΠΎΠ½Π°Π³Ρ€Π΅Π²Π°Ρ‚Π΅Π»ΡŒΠ½Ρ‹Π΅ элСмСнты, элСктролампы, элСктромоторы ΠΈ Ρ‚.Π΄.).

ΠœΠ΅ΡΡ‚Π° соСдинСния ΡΠΎΠ΅Π΄ΠΈΠ½ΠΈΡ‚Π΅Π»ΡŒΠ½Ρ‹Ρ… (подводящих) мСталличСских ΠΏΡ€ΠΎΠ²ΠΎΠ΄Π½ΠΈΠΊΠΎΠ² ΠΎΡ‚ Ρ€Π°Π·Π»ΠΈΡ‡Π½Ρ‹Ρ… элСктричСских сопротивлСний Π½Π°Π·Ρ‹Π²Π°ΡŽΡ‚ ΡƒΠ·Π»Π°ΠΌΠΈ соСдинСния.

НаиболСС простыми соСдинСниями элСктричСских сопротивлСний ΠΏΠΎΡΠ»Π΅Π΄ΠΎΠ²Π°Ρ‚Π΅Π»ΡŒΠ½ΠΎΠ΅ ΠΈ ΠΏΠ°Ρ€Π°Π»Π»Π΅Π»ΡŒΠ½ΠΎΠ΅.Π’ Ρ€Π°Π·Π»ΠΈΡ‡Π½Ρ‹Ρ… Π²Π°Ρ€ΠΈΠ°Π½Ρ‚Π°Ρ… элСктричСских сопротивлСний Π±ΡƒΠ΄Π΅ΠΌ Ρ€Π°ΡΡΠΌΠ°Ρ‚Ρ€ΠΈΠ²Π°Ρ‚ΡŒ Π½Π° ΠΏΡ€ΠΈΠΌΠ΅Ρ€Π΅ рСзисторов.

П. 1. ΠŸΡ€ΠΈ ΠΏΠΎΡΠ»Π΅Π΄ΠΎΠ²Π°Ρ‚Π΅Π»ΡŒΠ½ΠΎΠΌ соСдинСнии рСзисторов Π² ΠΊΠ°ΠΆΠ΄ΠΎΠΌ ΡƒΠ·Π»Π΅ ΠΎΠ±ΡŠΠ΅Π΄ΠΈΠ½ΡΡŽΡ‚ ΡΠΎΠ΅Π΄ΠΈΠ½ΠΈΡ‚Π΅Π»ΡŒΠ½Ρ‹Π΅ ΠΏΡ€ΠΎΠ²ΠΎΠ΄Π½ΠΈΠΊΠΈ Ρ‚ΠΎΠ»ΡŒΠΊΠΎ ΠΎΡ‚ Π΄Π²ΡƒΡ… Ρ€Π°Π·Π½Ρ‹Ρ… рСзисторов (рис. 1).

Π’ элСктричСских схСмах ΠΏΡ€ΡΠΌΠΎΡƒΠ³ΠΎΠ»ΡŒΠ½ΠΈΠΊΠΈ ΠΈΡΠΏΠΎΠ»ΡŒΠ·ΡƒΡŽΡ‚ΡΡ просто ΠΊΠ°ΠΊ символы рСзисторов, поэтому Ρ€Π°Π·ΠΌΠ΅Ρ€Ρ‹ ΠΏΡ€ΡΠΌΠΎΡƒΠ³ΠΎΠ»ΡŒΠ½ΠΈΠΊΠΎΠ² Π½Π΅ ΠΈΠΌΠ΅ΡŽΡ‚ ΠΏΡ€ΠΈΠ½Ρ†ΠΈΠΏΠΈΠ°Π»ΡŒΠ½ΠΎΠ³ΠΎ значСния. ВсС рСзисторы Π² элСктричСских схСмах Π±ΡƒΠ΄Π΅ΠΌ ΠΎΠ±ΠΎΠ·Π½Π°Ρ‡Π°Ρ‚ΡŒ ΠΏΡ€ΡΠΌΠΎΡƒΠ³ΠΎΠ»ΡŒΠ½ΠΈΠΊΠ°ΠΌΠΈ ΠΎΠ΄ΠΈΠ½Π°ΠΊΠΎΠ²Ρ‹Ρ… Ρ€Π°Π·ΠΌΠ΅Ρ€ΠΎΠ², обозначСния ΠΈ значСния сопротивлСний рСзисторов Π±ΡƒΠ΄Π΅ΠΌ ΡΡ‚Π°Π²ΠΈΡ‚ΡŒ Π²Π½ΡƒΡ‚Ρ€ΠΈ ΠΏΡ€ΡΠΌΠΎΡƒΠ³ΠΎΠ»ΡŒΠ½ΠΈΠΊΠΎΠ² ΠΈΠ»ΠΈ ΠΎΠΊΠΎΠ»ΠΎ Π½ΠΈΡ….

ΠžΡΠ½ΠΎΠ²Π½Ρ‹Π΅ (нСзависимыС) Π·Π°ΠΊΠΎΠ½Ρ‹ для элСктричСского участка постоянного Ρ‚ΠΎΠΊΠ°, состоящСго ΠΈΠ· ΠΏΠΎΡΠ»Π΅Π΄ΠΎΠ²Π°Ρ‚Π΅Π»ΡŒΠ½ΠΎ соСдинСнных рСзисторов (см. Рис. 1), ΠΈΠΌΠ΅ΡŽΡ‚ Π²ΠΈΠ΄:

ΠΈΠ»ΠΈ ΠΊΡ€Π°Ρ‚ΠΊΠΎ:

Π‘ΠΎΠΎΡ‚Π½ΠΎΡˆΠ΅Π½ΠΈΡ (1, a ) слСдствиС опрСдСлСния постоянного (стационарного) элСктричСского Ρ‚ΠΎΠΊΠ° для Π½Π΅Ρ€Π°Π·Π²Π΅Ρ‚Π²Π»Π΅Π½Π½ΠΎΠ³ΠΎ участка элСктричСской Ρ†Π΅ΠΏΠΈ.

ΠŸΡ€ΠΈ ΡƒΡ‡Π΅Ρ‚Π΅ Π·Π°ΠΊΠΎΠ½Π° Ома для ΠΎΠ΄Π½ΠΎΡ€ΠΎΠ΄Π½ΠΎΠ³ΠΎ участка элСктричСской Ρ†Π΅ΠΏΠΈ ΠΈΠ· Ρ„ΠΎΡ€ΠΌΡƒΠ» (1, a ), (1, b) Π΄ΠΎΠΏΠΎΠ»Π½ΠΈΡ‚Π΅Π»ΡŒΠ½ΠΎ ΠΏΠΎΠ»ΡƒΡ‡Π°Π΅ΠΌ:

Π€ΠΎΡ€ΠΌΡƒΠ»Π° (1, d) нСпосрСдствСнно слСдуСт ΠΈΠ· Ρ„ΠΎΡ€ΠΌΡƒΠ»Ρ‹ для элСктричСского сопротивлСния ΠΎΠ΄Π½ΠΎΡ€ΠΎΠ΄Π½Ρ‹Ρ… ΠΏΡ€ΠΎΠ²ΠΎΠ΄Π½ΠΈΠΊΠΎΠ² (рСзисторов), находящихся Π² Ρ‚Π΅ΠΏΠ»ΠΎΠ²ΠΎΠΌ равновСсии с ΠΎΠΊΡ€ΡƒΠΆΠ°ΡŽΡ‰Π΅ΠΉ срСдой:

ΠŸΡ€Π΅ΠΎΠ±Ρ€Π°Π·ΡƒΠ΅ΠΌ ΠΏΡ€ΠΎΠ²ΠΎΠ΄Π½ΠΈΠΊΠ° для сопротивлСний ΠΎΠ΄Π½ΠΎΡ€ΠΎΠ΄Π½Ρ‹Ρ… вСщСств (ρ i , l i , S i ), ввСдя понятиС эталонного вСщСства ( ρ эт ), Π·Π° ΠΊΠΎΡ‚ΠΎΡ€ΠΎΠ΅ ΠΌΠΎΠΆΠ½ΠΎ ΠΏΡ€ΠΈΠ½ΡΡ‚ΡŒ любоС вСщСство ΠΏΠΎ ТСланию, ΠΈ эталонной ΠΏΠ»ΠΎΡ‰Π°Π΄ΠΈ ΠΏΠΎΠΏΠ΅Ρ€Π΅Ρ‡Π½ΠΎΠ³ΠΎ сСчСния ΠΏΡ€ΠΎΠ²ΠΎΠ΄Π½ΠΈΠΊΠΎΠ² ( S эт ).Π’ΠΎΠ³Π΄Π°

Π³Π΄Π΅

Π—Π°ΠΌΠ΅Π½ΠΈΠΌ Π½Π°Π±ΠΎΡ€ рСзисторов R i (ρ i , S i , l i ) Π½Π°Π±ΠΎΡ€ΠΎΠΌ Ρ‚Π°ΠΊΠΎΠ³ΠΎ ΠΆΠ΅ числа рСзисторов с Ρ‚Π΅ΠΌΠΈ ΠΆΠ΅ ΠΏΠΎ Π²Π΅Π»ΠΈΡ‡ΠΈΠ½Π΅ элСктричСскими сопротивлСниями, Π½ΠΎ ΠΈΠ·Π³ΠΎΡ‚ΠΎΠ²Π»Π΅Π½Π½Ρ‹ΠΌΠΈ ΠΈΠ· ΠΎΠ΄Π½ΠΎΠ³ΠΎ ΠΈ Ρ‚ΠΎΠ³ΠΎ ΠΆΠ΅ вСщСства ( ρ эт ) с ΠΎΠ΄ΠΈΠ½Π°ΠΊΠΎΠ²Ρ‹ΠΌΠΈ площадями ΠΏΠΎΠΏΠ΅Ρ€Π΅Ρ‡Π½Ρ‹Ρ… сСчСний ( S эт ). ΠŸΠΎΡΠ»Π΅Π΄ΠΎΠ²Π°Ρ‚Π΅Π»ΡŒΠ½ΠΎΠ΅ соСдинСниС ΠΏΡ€ΠΎΠ²ΠΎΠ΄Π½ΠΈΠΊΠΎΠ² ΠΏΡ€ΠΈ Ρ‚Π°ΠΊΠΎΠΌ Π·Π°ΠΌΠ΅Π½Π΅ ΠΎΠ·Π½Π°Ρ‡Π°Π΅Ρ‚ объСдинСниС частСй (кусков) ΠΎΠ΄Π½ΠΎΠ³ΠΎ ΠΈ Ρ‚ΠΎΠ³ΠΎ ΠΆΠ΅ ΠΎΠ΄Π½ΠΎΡ€ΠΎΠ΄Π½ΠΎΠ³ΠΎ ΠΏΡ€ΠΎΠ²ΠΎΠ΄Π½ΠΈΠΊΠ° Π² Π΅Π΄ΠΈΠ½Ρ‹ΠΉ ΠΎΠ΄Π½ΠΎΡ€ΠΎΠ΄Π½Ρ‹ΠΉ ΠΏΡ€ΠΎΠ²ΠΎΠ΄Π½ΠΈΠΊ, поэтому

ПослС домноТСния ΠΎΠ±Π΅ΠΈΡ… частСй этого равСнства ( ρ эт / S эт ) ΠΏΠΎΠ»ΡƒΡ‡Π°Π΅ΠΌ:

ΠΈΠ»ΠΈ ΠΊΡ€Π°Ρ‚ΠΊΠΎ

ΠŸΡ€ΠΈ ΡƒΡ‡Π΅Ρ‚Π΅ (3, Π±) R iэт .= R i ΠΈ поэтому

Ρ‡Ρ‚ΠΎ ΠΎΠ±ΠΎΠ±Ρ‰Π°Π΅Ρ‚ (1, d) Π½Π° случай любого числа состоящих ΠΈΠ· соСдинСнных рСзисторов.

Если Π² ΠΊΠ°ΠΊΠΎΠΉ-Ρ‚ΠΎ ΡƒΠ·Π»Π΅ элСктричСской схСмы ΠΎΠ±ΡŠΠ΅Π΄ΠΈΠ½Π΅Π½Ρ‹ ΡΠΎΠ΅Π΄ΠΈΠ½ΠΈΡ‚Π΅Π»ΡŒΠ½Ρ‹Π΅ ΠΏΡ€ΠΎΠ²ΠΎΠ΄Π½ΠΈΠΊΠΈ Π±ΠΎΠ»Π΅Π΅ Ρ‡Π΅ΠΌ ΠΎΡ‚ Π΄Π²ΡƒΡ… рСзисторов, Ρ‚ΠΎ никакая ΠΏΠ°Ρ€Π° этих рСзисторов Π½Π΅ соСдинСна ΠΌΠ΅ΠΆΠ΄Ρƒ собой.

П. 2. ΠŸΡ€ΠΈ ΠΏΠ°Ρ€Π°Π»Π»Π΅Π»ΡŒΠ½ΠΎΠΌ соСдинСнии рСзисторов ΡΠΎΠ΅Π΄ΠΈΠ½ΠΈΡ‚Π΅Π»ΡŒΠ½Ρ‹Π΅ ΠΏΡ€ΠΎΠ²ΠΎΠ΄Π½ΠΈΠΊΠΈ ΠΎΡ‚ ΠΎΠ΄Π½ΠΈΡ… ΠΊΠΎΠ½Ρ†ΠΎΠ² рСзисторы ΠΎΠ±ΡŠΠ΅Π΄ΠΈΠ½ΡΡŽΡ‚ΡΡ Π² ΠΎΠ΄Π½ΠΎΠΌ ΡƒΠ·Π»Π΅, ΡΠΎΠ΅Π΄ΠΈΠ½ΠΈΡ‚Π΅Π»ΡŒΠ½Ρ‹Π΅ ΠΏΡ€ΠΎΠ²ΠΎΠ΄Π½ΠΈΠΊΠΈ ΠΎΡ‚ ΠΏΡ€ΠΎΡ‚ΠΈΠ²ΠΎΠΏΠΎΠ»ΠΎΠΆΠ½Ρ‹Ρ… ΠΊΠΎΠ½Ρ†ΠΎΠ² этих рСзисторов — Π² Π΄Ρ€ΡƒΠ³ΠΎΠΌ ΠΎΠ±Ρ‰Π΅ΠΌ для Π½ΠΈΡ… ΡƒΠ·Π»Π΅ (рис.2).

ΠžΡΠ½ΠΎΠ²Π½Ρ‹Π΅ (нСзависимыС) Π·Π°ΠΊΠΎΠ½Ρ‹ для участка элСктричСской Ρ†Π΅ΠΏΠΈ, состоящСго ΠΈΠ· прямых соСдинСнных рСзисторов, ΠΈΠΌΠ΅ΡŽΡ‚ ΡΠ»Π΅Π΄ΡƒΡŽΡ‰ΠΈΠΉ Π²ΠΈΠ΄:

Π€ΠΎΡ€ΠΌΡƒΠ»Π° (5, Π°) являСтся слСдствиСм элСктричСского напряТСния Π½Π° участкС Ρ†Π΅ΠΏΠΈ ΠΈ ΠΏΠ°Ρ€Π°Π»Π»Π΅Π»ΡŒΠ½ΠΎΠ³ΠΎ соСдинСния ΠΏΡ€ΠΎΠ²ΠΎΠ΄Π½ΠΈΠΊΠΎΠ². Π€ΠΎΡ€ΠΌΡƒΠ»Π° (5, b) нСпосрСдствСнно слСдуСт ΠΈΠ· постоянного (стационарного) элСктричСского Ρ‚ΠΎΠΊΠ° для Ρ€Π°Π·Π²Π΅Ρ‚Π²Π»Π΅Π½Π½ΠΎΠ³ΠΎ участка Ρ†Π΅ΠΏΠΈ. Часто Π·Π°ΠΊΠΎΠ½ (5, b ) Π½Π°Π·Ρ‹Π²Π°ΡŽΡ‚ 1-Ρ‹ΠΌ Π·Π°ΠΊΠΎΠ½ΠΎΠΌ (ΠΏΡ€Π°Π²ΠΈΠ»ΠΎΠΌ) ΠšΠΈΡ€Ρ…Π³ΠΎΡ„Π° для Ρ€Π°Π·Π²Π΅Ρ‚Π²Π»Π΅Π½Π½ΠΎΠΉ элСктричСской Ρ†Π΅ΠΏΠΈ.

Из Ρ„ΠΎΡ€ΠΌΡƒΠ» (5, Π° ), (5, b) ΠΏΡ€ΠΈ ΡƒΡ‡Π΅Ρ‚Π΅ Π·Π°ΠΊΠΎΠ½Π° Ома для ΠΎΠ΄Π½ΠΎΡ€ΠΎΠ΄Π½ΠΎΠ³ΠΎ участка элСктричСской Ρ†Π΅ΠΏΠΈ ΠΎΠ±Ρ‹Ρ‡Π½ΠΎ ΠΏΠΎΠ»ΡƒΡ‡Π°Π΅ΠΌ:

ΠŸΠ΅Ρ€Π΅ΠΏΠΈΡˆΠ΅ΠΌ Ρ„ΠΎΡ€ΠΌΡƒΠ»Ρƒ (5, d) для случая Π΄Π²ΡƒΡ… ΠΏΠ°Ρ€Π°Π»Π»Π΅Π»ΡŒΠ½Ρ‹Ρ… рСзисторов ΡΠ»Π΅Π΄ΡƒΡŽΡ‰ΠΈΠΌ ΠΎΠ±Ρ€Π°Π·ΠΎΠΌ:

Π’ случаС Ρ‚Ρ€Π΅Ρ… соСдинСнных рСзисторов

Π˜Ρ‚Π°ΠΊ, ΠΎΠ±Ρ‰Π΅Π΅ сопротивлСниС ΠΏΠ°Ρ€Π°Π»Π»Π΅Π»ΡŒΠ½ΠΎΠ³ΠΎ соСдинСния рСзисторов мСньшС сопротивлСния любого ΠΈΠ· рСзисторов, входящСго Π² ΠΏΠ°Ρ€Π°Π»Π»Π΅Π»ΡŒΠ½ΠΎΠ΅ соСдинСниС.

ΠžΠ±ΠΎΠ±Ρ‰ΠΈΠΌ Ρ„ΠΎΡ€ΠΌΡƒΠ»Ρƒ (5, d) Π½Π° любоС число N прямых соСдинСнных ΠΌΠ΅ΠΆΠ΄Ρƒ собой рСзисторов ΠΈ Π²Ρ‹Π΄Π΅Π»ΠΈΠΌ ΠΈΠ· Π½Π΅Π΅ частный случай, ΠΊΠΎΠ³Π΄Π° элСктричСскоС сопротивлСниС всСх рСзисторов ΠΎΠ΄ΠΈΠ½Π°ΠΊΠΎΠ²Ρ‹; Ρ‚ΠΎΠ³Π΄Π°

Π’Π°ΠΊΠΈΠΌ ΠΎΠ±Ρ€Π°Π·ΠΎΠΌ, ΠΎΠ±Ρ‰Π΅Π΅ сопротивлСниС ΠΏΠ°Ρ€Π°Π»Π»Π΅Π»ΡŒΠ½ΠΎΠ³ΠΎ соСдинСния ΠΎΠ΄ΠΈΠ½Π°ΠΊΠΎΠ²Ρ‹Ρ… рСзисторов мСньшС сопротивлСния ΠΎΠ΄Π½ΠΎΠ³ΠΎ рСзистора этого соСдинСния Π²ΠΎ ΡΡ‚ΠΎΠ»ΡŒΠΊΠΎ Ρ€Π°Π·, сколько ΠΎΠ΄ΠΈΠ½Π°ΠΊΠΎΠ²Ρ‹Ρ… рСзисторов содСрТится Π² ΠΏΠ°Ρ€Π°Π»Π»Π΅Π»ΡŒΠ½ΠΎΠΌ соСдинСнии.

Π€ΠΎΡ€ΠΌΡƒΠ»Ρ‹ (5, Π΅) ΠΈ (5, d) ΠΌΠΎΠΆΠ½ΠΎ ΠΏΠΎΠ»ΡƒΡ‡ΠΈΡ‚ΡŒ, Π½Π΅ ΠΈΡΠΏΠΎΠ»ΡŒΠ·ΡƒΡ Π·Π°ΠΊΠΎΠ½ Ома для ΠΎΠ΄Π½ΠΎΡ€ΠΎΠ΄Π½ΠΎΠ³ΠΎ участка элСктричСской Ρ†Π΅ΠΏΠΈ постоянного Ρ‚ΠΎΠΊΠ°, исходя Ρ‚ΠΎΠ»ΡŒΠΊΠΎ ΠΈΠ· Ρ„ΠΎΡ€ΠΌΡƒΠ»Ρ‹ (2) для сопротивлСния ΠΎΠ΄Π½ΠΎΡ€ΠΎΠ΄Π½Ρ‹Ρ… ΠΏΡ€ΠΎΠ²ΠΎΠ΄Π½ΠΈΠΊΠΎΠ².

ΠŸΠ°Ρ€Π°Π»Π»Π΅Π»ΡŒΠ½ΠΎΠ΅ соСдинСниС N ΠΎΠ΄ΠΈΠ½Π°ΠΊΠΎΠ²Ρ‹Ρ… ΠΏΡ€ΠΎΠ²ΠΎΠ΄Π½ΠΈΠΊΠΎΠ² эквивалСнтно ΠΎΠ΄Π½ΠΎΠΌΡƒ ΠΏΡ€ΠΎΠ²ΠΎΠ΄Π½ΠΈΠΊΡƒ Ρ‚ΠΎΠΉ ΠΆΠ΅ Π΄Π»ΠΈΠ½Ρ‹ с ΠΏΠΎΠΏΠ΅Ρ€Π΅Ρ‡Π½ΠΎΠ³ΠΎ сСчСния S 0 = N * S, поэтому

Ρ‡Ρ‚ΠΎ совпадаСт с (5, Π΅).

Π”ΠΎΠΊΠ°Π·Π°Ρ‚Π΅Π»ΡŒΡΡ‚Π²ΠΎ ΠΎΠ±ΠΎΠ±Ρ‰Π΅Π½Π½ΠΎΠΉ Ρ„ΠΎΡ€ΠΌΡƒΠ»Ρ‹ (5, d ) для любого числа N ΠΏΠΎΡΠ»Π΅Π΄ΠΎΠ²Π°Ρ‚Π΅Π»ΡŒΠ½ΠΎ соСдинСнных Ρ€Π°Π·Π»ΠΈΡ‡Π½Ρ‹Ρ… рСзисторов Π²Ρ‹ΠΏΠΎΠ»Π½ΠΈΠΌ Π² Π΄Π²Π° этапа. Π‘Π½Π°Ρ‡Π°Π»Π° Π·Π°ΠΌΠ΅Π½ΠΈΠΌ рСзисторы ΠΈΠ· Ρ€Π°Π·Π½Ρ‹Ρ… вСщСств ( Ρ€ i ) ΠΈ Π² ΠΎΠ±Ρ‰Π΅ΠΌ случаС с Ρ€Π°Π·Π»ΠΈΡ‡Π½Ρ‹ΠΌΠΈ l i ΠΈ S i Π½Π° эквивалСнтныС ΠΏΠΎ Π²Π΅Π»ΠΈΡ‡ΠΈΠ½Π΅ элСктричСского сопротивлСния рСзисторы ΠΎΠ΄ΠΈΠ½Π°ΠΊΠΎΠ²ΠΎΠΉ Π΄Π»ΠΈΠ½Ρ‹ ( l эт ) , ΠΈΠ·Π³ΠΎΡ‚ΠΎΠ²Π»Π΅Π½Π½Ρ‹Π΅ ΠΈΠ· ΠΎΠ΄Π½ΠΎΠ³ΠΎ ΠΈ Ρ‚ΠΎΠ³ΠΎ ΠΆΠ΅ вСщСства ( ρ эт ) — ΠžΡ‡Π΅Π²ΠΈΠ΄Π½ΠΎ, Ρ‡Ρ‚ΠΎ

Π³Π΄Π΅

ΠŸΠ°Ρ€Π°Π»Π»Π΅Π»ΡŒΠ½ΠΎΠ΅ соСдинСниС ΠΏΡ€ΠΎΠ²ΠΎΠ΄Π½ΠΈΠΊΠΎΠ² ΠΎΠ΄ΠΈΠ½Π°ΠΊΠΎΠ²ΠΎΠΉ Π΄Π»ΠΈΠ½Ρ‹ (i 3T ), ΠΈΠ·Π³ΠΎΡ‚ΠΎΠ²Π»Π΅Π½Π½Ρ‹Ρ… ΠΈΠ· ΠΎΠ΄Π½ΠΎΠ³ΠΎ ΠΈ Ρ‚ΠΎΠ³ΠΎ ΠΆΠ΅ вСщСства (Ρ€ эт )> Π΄Π°Π΅Ρ‚ ΠΎΠ΄Π½ΠΎΡ€ΠΎΠ΄Π½Ρ‹ΠΉ ΠΏΡ€ΠΎΠ²ΠΎΠ΄Π½ΠΈΠΊ Ρ‚ΠΎΠΉ ΠΆΠ΅ Π΄Π»ΠΈΠ½Ρ‹ (i 3T ) ΠΈΠ· Ρ‚ΠΎΠ³ΠΎ ΠΆΠ΅ вСщСства (Ρ€ эт ) с ΠΏΠ»ΠΎΡ‰Π°Π΄ΡŒΡŽ ΠΏΠΎΠΏΠ΅Ρ€Π΅Ρ‡Π½ΠΎΠ³ΠΎ сСчСния

Из (6, Π°, Π±, с) ΠΏΠΎΠ»ΡƒΡ‡Π°Π΅ΠΌ:

Ρ‡Ρ‚ΠΎ ΠΏΠΎΠ΄Ρ‚Π²Π΅Ρ€ΠΆΠ΄Π°Π΅Ρ‚ всС Π²Ρ‹ΡˆΠ΅ΠΏΡ€ΠΈΠ²Π΅Π΄Π΅Π½Π½Ρ‹Π΅ Ρ„ΠΎΡ€ΠΌΡƒΠ»Ρ‹ ΠΈ качСствСнныС Π²Ρ‹Π²ΠΎΠ΄Ρ‹ для ΠΏΠ°Ρ€Π°Π»Π»Π΅Π»ΡŒΠ½ΠΎΠ³ΠΎ соСдинСния рСзисторов.

П. 3 . БопротивлСния Π»ΡŽΠ±Ρ‹Ρ… ΡΠΎΠ΅Π΄ΠΈΠ½ΠΈΡ‚Π΅Π»ΡŒΠ½Ρ‹Ρ… ΠΏΡ€ΠΎΠ²ΠΎΠ΄Π½ΠΈΠΊΠΎΠ² Π² элСктричСских схСмах, Ссли это Π½Π΅ ΠΎΠ³ΠΎΠ²ΠΎΡ€Π΅Π½ΠΎ ΡΠΏΠ΅Ρ†ΠΈΠ°Π»ΡŒΠ½ΠΎ, ΡΡ‡ΠΈΡ‚Π°ΡŽΡ‚ΡΡ Π½ΠΈΡ‡Ρ‚ΠΎΠΆΠ½ΠΎ ΠΌΠ°Π»Ρ‹ΠΌΠΈ ΠΏΠΎ ΡΡ€Π°Π²Π½Π΅Π½ΠΈΡŽ с сопротивлСниСм рСзисторов ΠΈ Π² расчСтах Π½Π΅ ΡƒΡ‡ΠΈΡ‚Ρ‹Π²Π°ΡŽΡ‚ΡΡ; поэтому ΠΈΡ… ΠΌΠΎΠΆΠ½ΠΎ Π½Π΅ Ρ‚ΠΎΠ»ΡŒΠΊΠΎ ΡƒΠΊΠΎΡ€Π°Ρ‡ΠΈΠ²Π°Ρ‚ΡŒ, Π½ΠΎ ΠΏΡ€ΠΈ нСобходимости ΠΈ ΡƒΠ΄Π»ΠΈΠ½ΡΡ‚ΡŒ ΠΏΡ€ΠΈ прСобразованиях схСм соСдинСний рСзисторов.

Π’ элСктричСских схСмах ΡΠΎΠ΅Π΄ΠΈΠ½ΠΈΡ‚Π΅Π»ΡŒΠ½Ρ‹Π΅ ΠΏΡ€ΠΎΠ²ΠΎΠ΄Π½ΠΈΠΊΠΈ условно ΠΈΠ·ΠΎΠ±Ρ€Π°ΠΆΠ°ΡŽΡ‚ Ρ‚ΠΎΠ½ΠΊΠΈΠΌΠΈ ΠΏΡ€ΠΎΠ²ΠΎΠ΄Π½ΠΈΠΊΠ°ΠΌΠΈ (линиями). На самом ΠΆΠ΅ Π΄Π΅Π»Π΅ Π² соотвСтствии с Ρ„ΠΎΡ€ΠΌΡƒΠ»ΠΎΠΉ (2) ΠΏΠ»ΠΎΡ‰Π°Π΄ΡŒ ΠΏΠΎΠΏΠ΅Ρ€Π΅Ρ‡Π½ΠΎΠ³ΠΎ сСчСния ΡΠΎΠ΅Π΄ΠΈΠ½ΠΈΡ‚Π΅Π»ΡŒΠ½Ρ‹Ρ… ΠΏΡ€ΠΎΠ²ΠΎΠ΄Π½ΠΈΠΊΠΎΠ² Π΄ΠΎΠ»ΠΆΠ½Π° Π±Ρ‹Ρ‚ΡŒ достаточно большой, Ρ‡Ρ‚ΠΎ достигаСтся ΠΏΠ°Ρ€Π°Π»Π»Π΅Π»ΡŒΠ½Ρ‹ΠΌ соСдинСниСм Ρ‚ΠΎΠ½ΠΊΠΈΡ… ΠΏΡ€ΠΎΠ²ΠΎΠ΄Π½ΠΈΠΊΠΎΠ² с диэлСктричСским ΠΏΠΎΠΊΡ€Ρ‹Ρ‚ΠΈΠ΅ΠΌ (алюминий, мСдь), ΠΈΠ·Π³ΠΎΡ‚ΠΎΠ²Π»Π΅Π½Π½Ρ‹Ρ… ΠΈΠ· Π½Π΅Π΄ΠΎΡ€ΠΎΠ³ΠΈΡ… вСщСств с ΠΌΠΈΠ½ΠΈΠΌΠ°Π»ΡŒΠ½Ρ‹ΠΌΠΈ ΡƒΠ΄Π΅Π»ΡŒΠ½Ρ‹ΠΌΠΈ сопротивлСниями [1] ).

РСзисторы, Π½Π°ΠΎΠ±ΠΎΡ€ΠΎΡ‚, ΠΈΠ·Π³ΠΎΡ‚Π°Π²Π»ΠΈΠ²Π°ΡŽΡ‚ ΠΈΠ· вСщСств с максимально большими ΡƒΠ΄Π΅Π»ΡŒΠ½Ρ‹ΠΌΠΈ сопротивлСниями (Ρ„Π΅Ρ…Ρ€Π°Π»ΡŒ, Π½ΠΈΡ…Ρ€ΠΎΠΌ, Ρ€Π΅ΠΎΡ‚Π°Π½, константан, ΠΌΠ°Π½Π³Π°Π»ΠΈΠ½, Π½ΠΈΠΊΠ΅Π»ΠΈΠ½, ΡΡ‚Π°Π»ΡŒ, ΠΆΠ΅Π»Π΅Π·ΠΎ [1]) Π² Π²ΠΈΠ΄Π΅ Ρ‚ΠΎΠ½ΠΊΠΈΡ… спиралСй, Π·Π°ΠΊΠ»ΡŽΡ‡Π΅Π½Π½Ρ‹Ρ… Π² диэлСктричСскиС Ρ‚Π²Π΅Ρ€Π΄Ρ‹Π΅ ΠΎΠ±ΠΎΠ»ΠΎΡ‡ΠΊΠΈ.

П. 4. ΠŸΠ°Ρ€Π°Π»Π»Π΅Π»ΡŒΠ½ΠΎΠ΅ соСдинСниС Ρ‚Ρ€Π΅Ρ… рСзисторов (см. Рис. 2) ΠΌΠΎΠΆΠ½ΠΎ ΠΎΡΡƒΡ‰Π΅ΡΡ‚Π²ΠΈΡ‚ΡŒ ΠΈ ΠΏΠΎ-Π΄Ρ€ΡƒΠ³ΠΎΠΌΡƒ, ΠΊΠ°ΠΊ ΠΏΠΎΠΊΠ°Π·Π°Π½ΠΎ Π½Π° рисункС 3.

ΠŸΡ€ΠΎΠΈΠ»Π»ΡŽΡΡ‚Ρ€ΠΈΡ€ΡƒΠ΅ΠΌ ΡΠΊΠ²ΠΈΠ²Π°Π»Π΅Π½Ρ‚Π½ΠΎΡΡ‚ΡŒ элСктричСских схСм, ΠΈΠ·ΠΎΠ±Ρ€Π°ΠΆΠ΅Π½Π½Ρ‹Ρ… Π½Π° рисунках 2, 3.

Π’ схСмС, ΠΈΠ·ΠΎΠ±Ρ€Π°ΠΆΠ΅Π½Π½ΠΎΠΉ Π½Π° рисункС 3, оставим участок Π’Π‘ с рСзистором R 2 Π±Π΅Π· ΠΈΠ·ΠΌΠ΅Π½Π΅Π½ΠΈΠΉ ΠΈ Π½Π°Ρ‡Π½Π΅ΠΌ ΡƒΠΊΠΎΡ€Π°Ρ‡ΠΈΠ²Π°Ρ‚ΡŒ ΡΠΎΠ΅Π΄ΠΈΠ½ΠΈΡ‚Π΅Π»ΡŒΠ½Ρ‹Π΅ ΠΏΡ€ΠΎΠ²ΠΎΠ΄Π½ΠΈΠΊΠΈ А’Π‘ ΠΈ D’B ΠΎΡ‚ ΡƒΠ·Π»ΠΎΠ² А ‘ΠΈ D’. Π’ ΠΏΡ€Π΅Π΄Π΅Π»Π΅ ΠΏΡ€ΠΈ А’Π‘ β†’ 0 ← D’B схСма, изобраТСнная Π½Π° рисункС 3, ΠΏΠ΅Ρ€Π΅Ρ…ΠΎΠ΄ΠΈΡ‚ Π² схСму, ΠΈΠ·ΠΎΠ±Ρ€Π°ΠΆΠ΅Π½Π½ΡƒΡŽ Π½Π° рисункС 2.

ΠžΠ±Ρ‰ΠΈΠΉ ΠΌΠ΅Ρ‚ΠΎΠ΄ эквивалСнтного прСобразования элСктричСских схСм, основанный Π½Π° понятии постоянных элСктричСских ΠΏΠΎΠ»Π΅ΠΉ, Π±ΡƒΠ΄Π΅Ρ‚ рассмотрСн Π² ΠΎΡ‚Π΄Π΅Π»ΡŒΠ½ΠΎΠΉ ΡΡ‚Π°Ρ‚ΡŒΠ΅.

Π’ Ρ€Π°Π·Π²ΠΈΡ‚ΠΈΠ΅ Ρ‚Π΅ΠΌΡ‹ прСобразования элСктричСских схСм ΠΏΡ€Π΅Π΄Π»Π°Π³Π°ΡŽ читатСлям ΠΏΠΎΠ΄ΡƒΠΌΠ°Ρ‚ΡŒ Π½Π°Π΄ заданиями.

Π—Π°Π΄Π°Π½ΠΈΠ΅ 1 . Π˜Π·ΠΎΠ±Ρ€Π°Π·ΠΈΡ‚Π΅ Π±ΠΎΠ»Π΅Π΅ наглядно ΠΏΡ€ΠΈΠ²Π΅Π΄Π΅Π½Π½Ρ‹Π΅ соСдинСния Ρ‡Π΅Ρ‚Ρ‹Ρ€Π΅Ρ… рСзисторов:

Π—Π°Π΄Π°Π½ΠΈΠ΅ 2 . Π Π°ΡΡˆΠΈΡ„Ρ€ΡƒΠΉΡ‚Π΅ ΠΏΡ€ΠΈΠ²Π΅Π΄Π΅Π½Π½ΠΎΠ΅ соСдинСниС пяти рСзисторов.

П. 5. Рассмотрим схСму соСдинСния рСзисторов, ΠΏΡ€ΠΈΠ²Π΅Π΄Π΅Π½Π½ΡƒΡŽ Π½ΠΈΠΆΠ΅ Π½Π° рисункС 4, ΠΎ. Анализ любой элСктричСской схСмы Π½Π°Ρ‡Π°Ρ‚ΡŒ с Ρ€Π°Π·ΠΌΠ΅Ρ‚ΠΊΠΈ ΡƒΠ·Π»ΠΎΠ² рСзисторов ΠΈ ΠΏΡ€ΠΈ нСобходимости ΠΈΡ… обозначСния. Π‘Ρ€Π°Π·Ρƒ ΠΆΠ΅ ΠΎΡ‚ΠΌΠ΅Ρ‚ΠΈΠΌ, Ρ‡Ρ‚ΠΎ никакая ΠΏΠ°Ρ€Π° рСзисторов R 1 , R 2 , R 6 (R 4 , R 5 , R 6 ) Π½Π΅ соСдинСна ΠΌΠ΅ΠΆΠ΄Ρƒ собой Π½ΠΈ ΠΏΠΎΡΠ»Π΅Π΄ΠΎΠ²Π°Ρ‚Π΅Π»ΡŒΠ½ΠΎ, Π½ΠΈ ΠΏΠ°Ρ€Π°Π»Π»Π΅Π»ΡŒΠ½ΠΎ; Ρ‚Π°ΠΊ ΠΊΠ°ΠΊ Π² ΡƒΠ·Π»Π΅ Π’ (Π•) ΠΎΠ±ΡŠΠ΅Π΄ΠΈΠ½Π΅Π½Ρ‹ ΡΠΎΠ΅Π΄ΠΈΠ½ΠΈΡ‚Π΅Π»ΡŒΠ½Ρ‹Π΅ ΠΏΡ€ΠΎΠ²ΠΎΠ΄Π½ΠΈΠΊΠΈ ΠΎΡ‚ ΠΎΠ΄Π½ΠΈΡ… ΠΊΠΎΠ½Ρ†ΠΎΠ² Ρ‚Ρ€Π΅Ρ… рСзисторов R 1 , R 2 , R 6 (R 4 , R 5 , R 6 ), Π° ΡΠΎΠ΅Π΄ΠΈΠ½ΠΈΡ‚Π΅Π»ΡŒΠ½Ρ‹Π΅ ΠΏΡ€ΠΎΠ²ΠΎΠ΄Π½ΠΈΠΊΠΈ ΠΎΡ‚ ΠΏΡ€ΠΎΡ‚ΠΈΠ²ΠΎΠΏΠΎΠ»ΠΎΠΆΠ½Ρ‹Ρ… ΠΊΠΎΠ½Ρ†ΠΎΠ² этих рСзисторов Π½Π΅ ΠΎΠ±ΡŠΠ΅Π΄ΠΈΠ½Π΅Π½Ρ‹ Π² ΠΎΠ΄ΠΈΠ½ ΠΎΠ±Ρ‰ΠΈΠΉ для Π½ΠΈΡ… ΡƒΠ·Π΅Π».ВСхничСскоС Π½Π°Π·Π²Π°Π½ΠΈΠ΅ соСдинСния Ρ‚Ρ€Π΅Ρ… рСзисторов R 1 , R 2 , R 6 (R 4 , R 5 , R 6 ) Π±ΡƒΠ΄Π΅Ρ‚ Π΄Π°Π½ΠΎ Π½ΠΈΠΆΠ΅.

План эквивалСнтного прСобразования соСдинСния рСзисторов, ΠΈΠ·ΠΎΠ±Ρ€Π°ΠΆΠ΅Π½Π½ΠΎΠ³ΠΎ Π½Π° рисункС 4, ΠΎΡ‡Π΅Π²ΠΈΠ΄Π΅Π½ ΠΈ Π² большой стСпСни, благодаря ΠΏΡ€ΠΎΠ²Π΅Π΄Π΅Π½Π½ΠΎΠΉ Ρ€Π°Π·ΠΌΠ΅Ρ‚ΠΊΠ΅ ΡƒΠ·Π»ΠΎΠ² соСдинСния рСзисторов.

На ΠΏΠ΅Ρ€Π²ΠΎΠΉ стадии обучСния рСкомСндуСтся ΠΏΡ€ΠΎΠ²ΠΎΠ΄ΠΈΡ‚ΡŒ прСобразования соСдинСний рСзисторов поэтапно, рассматривая ΠΏΠΎΡΠ»Π΅Π΄ΠΎΠ²Π°Ρ‚Π΅Π»ΡŒΠ½ΠΎΡΡ‚ΡŒ всСх Π±ΠΎΠ»Π΅Π΅ простых элСктричСских схСм послС ΠΊΠ°ΠΆΠ΄ΠΎΠΉ Π·Π°ΠΌΠ΅Π½Ρ‹ Π½Π΅ΡΠΊΠΎΠ»ΡŒΠΊΠΈΡ… рСзисторов ΠΎΠ΄Π½ΠΈΠΌ эквивалСнтным рСзистором.

Π’ рассматриваСмом случаС сразу наглядно ΠΏΠΎΠΊΠ°Π·Π°Ρ‚ΡŒ Π²ΠΈΠ΄Ρ‹ соСдинСний всСх рСзисторов Π² схСмС Π½Π° рисункС 4, b . Для этого Π½Π°Π΄ΠΎ участок АВ ΠΏΠΎΠ²Π΅Ρ€Π½ΡƒΡ‚ΡŒ ΠΏΠΎ Ρ…ΠΎΠ΄Ρƒ часовой стрСлки, Π° участок FE — ΠΏΡ€ΠΎΡ‚ΠΈΠ² Ρ…ΠΎΠ΄Π° часовой стрСлки Π½Π° 90 Β°. ΠŸΠΎΠ»ΡƒΡ‡Π΅Π½Π½ΠΎΠ΅ Ρ‚Π°ΠΊΠΈΠΌ ΠΎΠ±Ρ€Π°Π·ΠΎΠΌ соСдинСниС рСзисторов эквивалСнтно соСдинСнию, ΡƒΠΊΠ°Π·Π°Π½ΠΎ Π½Π° рисункС 4, с.

ВычислСниС ΠΎΠ±Ρ‰Π΅Π³ΠΎ сопротивлСния ΠΏΠΎ схСмС 4, с элСмСнтарно ΠΈ Π΄Π°Π΅Ρ‚ ΡΠ»Π΅Π΄ΡƒΡŽΡ‰ΠΈΠ΅ Ρ€Π΅Π·ΡƒΠ»ΡŒΡ‚Π°Ρ‚Ρ‹:

Π—Π°Π΄Π°Π½ΠΈΠ΅ 3 .Π‘ΠΎΠΏΠΎΡΡ‚Π°Π²ΡŒΡ‚Π΅ ΡΠ»Π΅ΠΊΡ‚Ρ€ΠΈΡ‡Π΅ΡΠΊΡƒΡŽ схСму, ΠΈΠ·ΠΎΠ±Ρ€Π°ΠΆΠ΅Π½Π½ΡƒΡŽ Π½Π° рисункС 4, Π°, антисхСму (схСму-Π°Π½Ρ‚ΠΈΠΏΠΎΠ΄), замСняя Π² исходной ΠΏΠΎΡΠ»Π΅Π΄ΠΎΠ²Π°Ρ‚Π΅Π»ΡŒΠ½ΠΎΠΉ схСмС рСзисторы Π½Π° ΠΏΠ°Ρ€Π°Π»Π»Π΅Π»ΡŒΠ½Ρ‹Π΅ ΠΈ Π½Π°ΠΎΠ±ΠΎΡ€ΠΎΡ‚, ΠΏΠ°Ρ€Π°Π»Π»Π΅Π»ΡŒΠ½Ρ‹Π΅ — Π½Π° ΠΏΠΎΡΠ»Π΅Π΄ΠΎΠ²Π°Ρ‚Π΅Π»ΡŒΠ½Ρ‹Π΅.

П. 6. Число Π½Π°ΠΈΠΏΡ€ΠΎΡΡ‚Π΅ΠΉΡˆΠΈΡ… соСдинСний рСзисторов, ΠΊΡ€ΠΎΠΌΠ΅ ΠΏΠΎΡΠ»Π΅Π΄ΠΎΠ²Π°Ρ‚Π΅Π»ΡŒΠ½ΠΎΠ³ΠΎ ΠΈ ΠΏΠ°Ρ€Π°Π»Π»Π΅Π»ΡŒΠ½ΠΎΠ³ΠΎ, соСдинСния рСзисторов ΠΏΠΎ схСмам Β«Ρ‚Ρ€Π΅ΡƒΠ³ΠΎΠ»ΡŒΠ½ΠΈΠΊΒ» (рис. 5, Π°) ΠΈ Β«Π·Π²Π΅Π·Π΄Π°Β» (рис. 5, b).

ΠšΡΡ‚Π°Ρ‚ΠΈ, Π² соСдинСниях 4 b, с рСзисторами R 1 , R 2 , R 6 (R 4 , R 5 , R 6 ) соСдинСны ΠΌΠ΅ΠΆΠ΄Ρƒ собой ΠΏΠΎ схСмС «Π·Π²Π΅Π·Π΄Π°» .

Π‘Ρ„ΠΎΡ€ΠΌΡƒΠ»ΠΈΡ€ΡƒΠ΅ΠΌ условия эквивалСнтности (ΠΏΠΎ Π²Π΅Π»ΠΈΡ‡ΠΈΠ½Π΅ элСктричСского сопротивлСния) соСдинСний рСзисторов ΠΏΠΎ схСмам «Ρ‚Ρ€Π΅ΡƒΠ³ΠΎΠ»ΡŒΠ½ΠΈΠΊ» ΠΈ «Π·Π²Π΅Π·Π΄Π°», приняв Π·Π° основу ΡΡ‚Π°Ρ‚ΡŒΠΈ [3].

Π’ ΠΎΠ±Ρ‰Π΅ΠΌ случаС (рис. 6) условия эквивалСнтности состоят ΠΈΠ· Ρ‚Ρ€Π΅Ρ… ΡƒΡ€Π°Π²Π½Π΅Π½ΠΈΠΉ:

Рассмотрим частный случай, сопротивлСниС всСх рСзисторов Π² «Ρ‚Ρ€Π΅ΡƒΠ³ΠΎΠ»ΡŒΠ½ΠΈΠΊΠ΅» ΠΎΠ΄ΠΈΠ½Π°ΠΊΠΎΠ²Ρ‹ ΠΈ Ρ€Π°Π²Π½Ρ‹ R. Как слСдствиС этого, сопротивлСниС всСх рСзисторов Π² эквивалСнтной «Π·Π²Π΅Π·Π΄Π΅» Ρ‚Π°ΠΊΠΆΠ΅ Π±ΡƒΠ΄ΡƒΡ‚ ΠΎΠ΄ΠΈΠ½Π°ΠΊΠΎΠ²Ρ‹ΠΌΠΈ ΠΈ Ρ€Π°Π²Π½Ρ‹ΠΌΠΈ r .Π’ этом частном случаС всС Ρ‚Ρ€ΠΈ условия (7) ΡΠΎΠ²ΠΏΠ°Π΄Π°ΡŽΡ‚ ΠΌΠ΅ΠΆΠ΄Ρƒ собой.

ΠŸΡ€ΠΈ ΠΏΠΎΠ΄ΠΊΠ»ΡŽΡ‡Π΅Π½ΠΈΠΈ источника постоянного элСкричСского напряТСния ΠΌΠ΅ΠΆΠ΄Ρƒ Π»ΡŽΠ±Ρ‹ΠΌΠΈ двумя ΠΊΠ»Π΅ΠΌΠΌΠ°ΠΌΠΈ (полюсами) «Ρ‚Ρ€Π΅ΡƒΠ³ΠΎΠ»ΡŒΠ½ΠΈΠΊΠ°» ΠΏΠΎ всСм Π΅Π³ΠΎ рСзисторам ΠΏΡ€ΠΎΡ‚Π΅ΠΊΠ°Π΅Ρ‚ постоянный элСктричСский Ρ‚ΠΎΠΊ ΠΈ всС Π΅Π³ΠΎ рСзисторы Π΄Π°ΡŽΡ‚ ΠΎΡ‚Π»ΠΈΡ‡Π½Ρ‹ΠΉ ΠΎΡ‚ нуля Π²ΠΊΠ»Π°Π΄ Π² ΠΎΠ±Ρ‰Π΅Π΅ сопротивлСниС участка Ρ†Π΅ΠΏΠΈ. ΠŸΡ€ΠΈ ΠΏΠΎΠ΄ΠΊΠ»ΡŽΡ‡Π΅Π½ΠΈΠΈ источника элСктричСского напряТСния ΠΊ Π»ΡŽΠ±ΠΎΠΌΡƒ Π΄Π²ΡƒΠΌ ΠΊΠ»Π΅ΠΌΠΌΠ°ΠΌ (полюсам) «Π·Π²Π΅Π·Π΄Ρ‹» постоянный элСктричСский Ρ‚ΠΎΠΊ Π² Π½Π΅ΠΉ ΠΏΡ€ΠΎΡ‚Π΅ΠΊΠ°Π΅Ρ‚ Ρ‚ΠΎΠ»ΡŒΠΊΠΎ ΠΏΠΎ Π΄Π²ΡƒΠΌ рСзисторам, Ρ‚Ρ€Π΅Ρ‚ΠΈΠΉ всСгда лишний — Ρ‚ΡƒΠΏΠΈΠΊΠΎΠ²Ρ‹ΠΉ; Π½ΠΎ ΠΊΠ°ΠΆΠ΄Ρ‹ΠΉ Ρ€Π°Π·Π½Ρ‹ΠΉ.

Из условий R OAB = r OAB рассматриваСмом частном случаС ΠΏΠΎΠ»ΡƒΡ‡Π°Π΅ΠΌ: (2/3) R = 2r.

Π˜Ρ‚Π°ΠΊ,

ΠŸΡ€ΠΎΡ‡Π½ΠΎΠ΅ соСдинСниС рСзисторов, Π½Π΅ содСрТащих ΠΏΠΎΡΠ»Π΅Π΄ΠΎΠ²Π°Ρ‚Π΅Π»ΡŒΠ½ΠΎ соСдинСнных рСзисторов, Π½Π°Π·Ρ‹Π²Π°ΡŽΡ‚ «мостиком» (рис. 7).

Π—Π°ΠΌΠ΅Π½Π° любого «Ρ‚Ρ€Π΅ΡƒΠ³ΠΎΠ»ΡŒΠ½ΠΈΠΊΠ°» Π² «ΠΌΠΎΡΡ‚ΠΈΠΊΠ΅» Π½Π° ΡΠΊΠ²ΠΈΠ²Π°Π»Π΅Π½Ρ‚Π½ΡƒΡŽ «Π·Π²Π΅Π·Π΄Ρƒ» ΠΈΠ»ΠΈ Π½Π°ΠΎΠ±ΠΎΡ€ΠΎΡ‚ позволяСт Π²Ρ‹Ρ‡ΠΈΡΠ»ΠΈΡ‚ΡŒ ΠΎΠ±Ρ‰Π΅Π΅ сопротивлСниС «ΠΌΠΎΡΡ‚ΠΈΠΊΠ°» Π΄ΠΎ ΠΊΠΎΠ½Ρ†Π° ΠΏΠΎ Π·Π°ΠΊΠΎΠ½Π°ΠΌ ΠΏΠΎΡΠ»Π΅Π΄ΠΎΠ²Π°Ρ‚Π΅Π»ΡŒΠ½ΠΎΠ³ΠΎ ΠΈ ΠΏΠ°Ρ€Π°Π»Π»Π΅Π»ΡŒΠ½ΠΎΠ³ΠΎ соСдинСния рСзисторов.

РасчСтная Ρ†Π΅Π»Π΅ΡΠΎΠΎΠ±Ρ€Π°Π·Π½ΠΎΡΡ‚ΡŒ Π·Π°ΠΌΠ΅Π½ «Ρ‚Ρ€Π΅ΡƒΠ³ΠΎΠ»ΡŒΠ½ΠΈΠΊΠΎΠ²» Π½Π° «Π·Π²Π΅Π·Π΄Ρ‹» ΠΈ Π½Π°ΠΎΠ±ΠΎΡ€ΠΎΡ‚ ограничиваСтся, ΠΊΡ€ΠΎΠΌΠ΅ случая R 1 = R 2 = R 3 = R (r 1 = r 2 = r 3 = r ), случаСм, ΠΊΠΎΠ³Π΄Π° срСди Ρ‚Ρ€Π΅Ρ… рСзисторов «Ρ‚Ρ€Π΅ΡƒΠ³ΠΎΠ»ΡŒΠ½ΠΈΠΊΠ°» ΠΈΠ»ΠΈ «Π·Π²Π΅Π·Π΄Ρ‹» ΠΈΠΌΠ΅ΡŽΡ‚ΡΡ Π΄Π²Π° ΠΎΠ΄ΠΈΠ½Π°ΠΊΠΎΠ²Ρ‹Ρ… рСзистора. Π’ ΠΎΠ±Ρ‰Π΅ΠΌ случаС, ΠΊΠΎΠ³Π΄Π° сопротивлСниС всСх рСзисторов Π² «Ρ‚Ρ€Π΅ΡƒΠ³ΠΎΠ»ΡŒΠ½ΠΈΠΊΠ΅» ΠΈ «Π·Π²Π΅Π·Π΄Π΅» Ρ€Π°Π·Π»ΠΈΡ‡Π½Ρ‹, Ρ€Π΅ΡˆΠ΅Π½ΠΈΠ΅ Π·Π°Π΄Π°Ρ‡ΠΈ ΠΎΠ± эквивалСнтности этих соСдинСний являСтся ΠΎΡ‡Π΅Π½ΡŒ слоТным.ΠŸΠΎΡΡ‚ΠΎΠΌΡƒ Π½Π°Π΄ΠΎ ΠΈΠ·ΡƒΡ‡Π°Ρ‚ΡŒ ΠΈ Π΄Ρ€ΡƒΠ³ΠΈΠ΅ ΠΌΠ΅Ρ‚ΠΎΠ΄Ρ‹ эквивалСнтного соСдинСния слоТных соСдинСний рСзисторов.

Если срСди сопротивлСний «ΠΌΠΎΡΡ‚ΠΈΠΊΠ°» (см. Рис. 7) ΠΈΠΌΠ΅ΡŽΡ‚ΡΡ ΠΏΡ€ΠΎΠΏΠΎΡ€Ρ†ΠΈΠΎΠ½Π°Π»ΡŒΠ½Ρ‹Π΅ сопротивлСния (Π°) R 1 / R 3 -R 2 / R 4 , Π±) R 1 / R 2 = R 3 / R 4 ), Ρ‚ΠΎ ΠΏΠΎΡΠ²Π»ΡΡŽΡ‚ΡΡ оси симмСтрии распрСдСлСния сопротивлСний Π² «мостикС», ΠΈ ΠΏΡ€ΠΈ ΠΏΠΎΠ΄ΠΊΠ»ΡŽΡ‡Π΅Π½ΠΈΠΈ «мостика» ΠΊ источнику постоянного элСктричСского напряТСния Ρ‚ΠΎΠΊ ΠΏΠΎ рСзистору R 5 Π½Π΅ ΠΏΡ€ΠΎΡ‚Π΅ΠΊΠ°Π΅Ρ‚ .ΠŸΠΎΡΡ‚ΠΎΠΌΡƒ рСзистор R 5 ΠΌΠΎΠΆΠ½ΠΎ ΡƒΠ΄Π°Π»ΠΈΡ‚ΡŒ ΠΈΠ· схСмы, Π½Π΅ измСняя Π΅Π΅ ΠΎΠ±Ρ‰Π΅Π΅ элСктричСскоС сопротивлСниС, Π° ΡƒΠ·Π»Ρ‹, ΠΌΠ΅ΠΆΠ΄Ρƒ ΠΊΠΎΡ‚ΠΎΡ€Ρ‹ΠΌΠΈ ΠΎΠ½ Π±Ρ‹Π» Π²ΠΊΠ»ΡŽΡ‡Π΅Π½ Π² «ΠΌΠΎΡΡ‚ΠΈΠΊΠ΅», ΠΏΡ€ΠΈ ΠΆΠ΅Π»Π°Π½ΠΈΠΈ ΠΌΠΎΠΆΠ½ΠΎ ΠΎΠ±ΡŠΠ΅Π΄ΠΈΠ½ΠΈΡ‚ΡŒ Π² ΠΎΠ΄ΠΈΠ½ ΡƒΠ·Π΅Π».

Π—Π°Π΄Π°Π½ΠΈΠ΅ 4 . ВычислитС ΠΎΠ±Ρ‰Π΅Π΅ сопротивлСниС ΠΎΠ΄ΠΈΠ½Π°ΠΊΠΎΠ²Ρ‹Ρ… рСзисторов (R) ΠΏΠΎ схСмС пятиконСчной Π·Π²Π΅Π·Π΄Ρ‹ (рис. 8) ΠΌΠ΅ΠΆΠ΄Ρƒ полюсами: Π°) А ΠΈ Π’; Π±) Π’ ΠΈ Π’ ‘; Π²) А ΠΈ Π‘; Π³) А ΠΈ D; Π΅) D ΠΈ D ‘; Π΅) А ΠΈ Π•; Π³) А ΠΈ Π€.

П.7. Рассмотрим Ρ€ΠΎΠ»ΡŒ ΠΈΠ·ΠΌΠ΅Ρ€ΠΈΡ‚Π΅Π»ΡŒΠ½Ρ‹Ρ… ΠΏΡ€ΠΈΠ±ΠΎΡ€ΠΎΠ² Π² макроскопичСской Ρ„ΠΈΠ·ΠΈΠΊΠ΅, Π³Π΄Π΅ Π½Π΅Ρ‚ ΠΏΡ€ΠΈΠ½Ρ†ΠΈΠΏΠΈΠ°Π»ΡŒΠ½ΠΎΠ³ΠΎ различия ΠΌΠ΅ΠΆΠ΄Ρƒ ΠΎΠ±ΡŠΠ΅ΠΊΡ‚Π°ΠΌΠΈ ΠΈ ΠΏΡ€ΠΈΠ±ΠΎΡ€Π°ΠΌΠΈ. И Ρ‚Π΅, ΠΈ Π΄Ρ€ΡƒΠ³ΠΈΠ΅ состоят ΠΈΠ· ΠΎΠ³Ρ€ΠΎΠΌΠ½ΠΎΠ³ΠΎ количСства ΠΌΠΎΠ»Π΅ΠΊΡƒΠ», исчисляСмого молями вСщСства, Ρ‚. Π΅. N = v * NA = (ΠΌ / М) * N A Π³Π΄Π΅ N A = 6,02 * 10 23 моль -1 — постоянная Авогадро.

НС рассматривая устройство ΠΈ ΠΏΡ€ΠΈΠ½Ρ†ΠΈΠΏΡ‹ Ρ€Π°Π±ΠΎΡ‚Ρ‹ ΠΈΠ·ΠΌΠ΅Ρ€ΠΈΡ‚Π΅Π»ΡŒΠ½Ρ‹Ρ… ΠΏΡ€ΠΈΠ±ΠΎΡ€ΠΎΠ², ΠΎΡ‚ΠΌΠ΅Ρ‚ΡŒΡ‚Π΅, Ρ‡Ρ‚ΠΎ всС ΠΎΠ½ΠΈ ΠΈΠΌΠ΅ΡŽΡ‚ ΡˆΠΊΠ°Π»Ρƒ с ΠΎΠΏΡ€Π΅Π΄Π΅Π»Π΅Π½Π½Ρ‹ΠΌΠΈ цСнностями дСлСния ΠΈ ΠΈΠ½Ρ‚Π΅Ρ€Π²Π°Π»ΠΎΠΌ Π·Π½Π°Ρ‡Π΅Π½ΠΈΠΉ измСряСмой Π²Π΅Π»ΠΈΡ‡ΠΈΠ½Ρ‹.

Π’ элСктричСских цСпях напряТСния ΠΈ напряТСния ΠΈΠ·ΠΌΠ΅Ρ€ΡΡŽΡ‚ соотвСтствСнно Π°ΠΌΠΏΠ΅Ρ€ΠΌΠ΅Ρ‚Ρ€Π°ΠΌΠΈ ΠΈ Π²ΠΎΠ»ΡŒΡ‚ΠΌΠ΅Ρ‚Ρ€Π°ΠΌΠΈ. ЭлСктричСскиС ΠΈΠ·ΠΌΠ΅Ρ€ΠΈΡ‚Π΅Π»ΡŒΠ½Ρ‹Π΅ ΠΏΡ€ΠΈΠ±ΠΎΡ€Ρ‹ Ρ…Π°Ρ€Π°ΠΊΡ‚Π΅Ρ€ΠΈΠ·ΡƒΡŽΡ‚ΡΡ ΠΈΡ… Π²Π½ΡƒΡ‚Ρ€Π΅Π½Π½ΠΈΠΌΠΈ элСктричСскими сопротивлСниями. Как любоС элСктричСскоС сопротивлСниС, Π°ΠΌΠΏΠ΅Ρ€ΠΌΠ΅Ρ‚Ρ€Ρ‹ ΠΈ Π²ΠΎΠ»ΡŒΡ‚ΠΌΠ΅Ρ‚Ρ€Ρ‹, нСзависимо ΠΎΡ‚ ΠΈΡ… спСцифики, ΠΌΠΎΠΆΠ½ΠΎ ΠΏΠΎΠ΄ΠΊΠ»ΡŽΡ‡Π°Ρ‚ΡŒ ΠΊ Π΄Ρ€ΡƒΠ³ΠΈΠΌ сопротивлСниСм (рСзисторы, ΡΠ»Π΅ΠΊΡ‚Ρ€ΠΎΠ½Π°Π³Ρ€Π΅Π²Π°Ρ‚Π΅Π»ΡŒΠ½Ρ‹Π΅ элСмСнты, элСктролампочки, элСктромоторы ΠΈ Ρ‚.Π΄.) ΠΊΠ°ΠΊ ΠΏΠΎΡΠ»Π΅Π΄ΠΎΠ²Π°Ρ‚Π΅Π»ΡŒΠ½ΠΎ, Ρ‚Π°ΠΊ ΠΈ ΠΏΠ°Ρ€Π°Π»Π»Π΅Π»ΡŒΠ½ΠΎ. АмпСрмСтр ΠΏΠΎΠΊΠ°Π·Ρ‹Π²Π°Π΅Ρ‚ сопротивлСниС Ρ‚ΠΎΠΊΠ°, ΠΏΡ€ΠΎΡ‚Π΅ΠΊΠ°ΡŽΡ‰Π΅Π³ΠΎ Ρ‡Π΅Ρ€Π΅Π· Π½Π΅Π³ΠΎ, ΠΈ всС сопротивлСниС, соСдинСнныС с Π½ΠΈΠΌ; Π° Π²ΠΎΠ»ΡŒΡ‚ΠΌΠ΅Ρ‚Ρ€ — элСктричСскоС напряТСниС Π½Π° Π²Π½ΡƒΡ‚Ρ€Π΅Π½Π½Π΅ΠΌ сопротивлСнии Π²ΠΎΠ»ΡŒΡ‚ΠΌΠ΅Ρ‚Ρ€Π° ΠΈ всСх сопротивлСний, ΠΏΠ°Ρ€Π°Π»Π»Π΅Π»ΡŒΠ½Ρ‹Ρ… с Π½ΠΈΠΌ.

ΠŸΠΎΠ΄ΠΊΠ»ΡŽΡ‡Π΅Π½ΠΈΠ΅ ΠΈΠ·ΠΌΠ΅Ρ€ΠΈΡ‚Π΅Π»ΡŒΠ½Ρ‹Ρ… ΠΏΡ€ΠΈΠ±ΠΎΡ€ΠΎΠ² Π² ΡΠ»Π΅ΠΊΡ‚Ρ€ΠΈΡ‡Π΅ΡΠΊΡƒΡŽ Ρ†Π΅ΠΏΡŒ измСняСт сопротивлСниС участков Ρ†Π΅ΠΏΠΈ ΠΈ всСй Ρ†Π΅ΠΏΠΈ Π² Ρ†Π΅Π»ΠΎΠΌ, Ρ‡Ρ‚ΠΎ ΠΏΡ€ΠΈΠ²ΠΎΠ΄ΠΈΡ‚ ΠΊ измСнСнию распрСдСлСний Ρ‚ΠΎΠΊΠΎΠ² ΠΈ напряТСний Π² элСктричСской Ρ†Π΅ΠΏΠΈ. ИзмСнСниС состояния систСмы Π² Ρ€Π΅Π·ΡƒΠ»ΡŒΡ‚Π°Ρ‚Π΅ провСдСния ΠΈΠ·ΠΌΠ΅Ρ€Π΅Π½ΠΈΠΉ ΠΈ Π·Π°Π²ΠΈΡΠΈΠΌΠΎΡΡ‚ΡŒ Ρ€Π΅Π·ΡƒΠ»ΡŒΡ‚Π°Ρ‚ΠΎΠ² ΠΈΠ·ΠΌΠ΅Ρ€Π΅Π½ΠΈΠΉ ΠΎΡ‚ ΠΈΠ·ΠΌΠ΅Ρ€ΠΈΡ‚Π΅Π»ΡŒΠ½Ρ‹Ρ… ΠΏΡ€ΠΈΠ±ΠΎΡ€ΠΎΠ² ΠΎΠ±Ρ‹Ρ‡Π½ΠΎ обсуТдаСтся Π½Π° ΡƒΡ€ΠΎΠ²Π½Π΅ микросистСмы (ΠΎΡ‚Π΄Π΅Π»ΡŒΠ½Ρ‹Ρ… ΠΌΠΈΠΊΡ€ΠΎΠΎΠ±ΡŠΠ΅ΠΊΡ‚ΠΎΠ²) Π² ΠΊΠ²Π°Π½Ρ‚ΠΎΠ²ΠΎΠΉ ΠΌΠ΅Ρ…Π°Π½ΠΈΠΊΠ΅ Π² вузовских курсах Ρ„ΠΈΠ·ΠΈΠΊΠΈ. Однако эта Π·Π°Π²ΠΈΡΠΈΠΌΠΎΡΡ‚ΡŒ сущСствуСт ΠΈ Π½Π° ΠΌΠ°ΠΊΡ€ΠΎΡƒΡ€ΠΎΠ²Π½Π΅, Π½ΠΎ Π΅Π΅ Π»ΠΈΠ±ΠΎ Π½Π΅ Π·Π°ΠΌΠ΅Ρ‡Π°ΡŽΡ‚, Π»ΠΈΠ±ΠΎ Π½Π΅ ΠΎΠ±Ρ€Π°Ρ‰Π°ΡŽΡ‚ΡΡ Π½Π° Π½Π΅Π΅ Π΄ΠΎΠ»ΠΆΠ½ΠΎΠ³ΠΎ внимания.

Π’ΠΎ ΠΈΠ·Π±Π΅ΠΆΠ°Π½ΠΈΠ΅ рассмотрСния Π²Ρ‹ΡˆΠ΅ описанной слоТной ΠΏΡ€ΠΎΠ±Π»Π΅ΠΌΡ‹ Π½Π° Π½Π°Ρ‡Π°Π»ΡŒΠ½ΠΎΠΉ стадии изучСния элСктричСских Π΄ΠΎΠ³ΠΎΠ²Π°Ρ€ΠΈΠ²Π°ΡŽΡ‚ΡΡ ΠΈΡΠΏΠΎΠ»ΡŒΠ·ΠΎΠ²Π°Ρ‚ΡŒ ΠΈΠ΄Π΅Π°Π»ΡŒΠ½Ρ‹Π΅ ΠΈΠ·ΠΌΠ΅Ρ€ΠΈΡ‚Π΅Π»ΡŒΠ½Ρ‹Π΅ ΠΏΡ€ΠΈΠ±ΠΎΡ€Ρ‹. β„’) ΠΎΠ½ ΠΏΠΎΠ΄ΠΊΠ»ΡŽΡ‡Π΅Π½.Π˜Π΄Π΅Π°Π»ΡŒΠ½Ρ‹Π΅ Π°ΠΌΠΏΠ΅Ρ€ΠΌΠ΅Ρ‚Ρ€Ρ‹ Π²ΠΊΠ»ΡŽΡ‡Π°ΡŽΡ‚ΡΡ Ρ‚ΠΎΠ»ΡŒΠΊΠΎ ΠΏΠΎΡΠ»Π΅Π΄ΠΎΠ²Π°Ρ‚Π΅Π»ΡŒΠ½ΠΎ с сопротивлСниСм ΠΈ Π½Π΅ ΠΈΡΠΊΠ°ΠΆΠ°ΡŽΡ‚ распрСдСлСний Ρ‚ΠΎΠΊΠΎΠ² ΠΈ напряТСний Π² элСктричСских цСпях.

Π˜Π΄Π΅Π°Π»ΡŒΠ½Ρ‹ΠΉ Π²ΠΎΠ»ΡŒΡ‚ΠΌΠ΅Ρ‚Ρ€ — это Π²ΠΎΠ»ΡŒΡ‚ΠΌΠ΅Ρ‚Ρ€, Π²Π½ΡƒΡ‚Ρ€Π΅Π½Π½Π΅Π΅ сопротивлСниС ΠΊΠΎΡ‚ΠΎΡ€ΠΎΠ³ΠΎ ΠΌΠ½ΠΎΠ³ΠΎ большС всСх сопротивлСний, соотвСтствуСтся Π² элСктричСской Ρ†Π΅ΠΏΠΈ. Π’ расчСтах Π²Π½ΡƒΡ‚Ρ€Π΅Π½Π½Π΅Π΅ сопротивлСниС идСального Π²ΠΎΠ»ΡŒΡ‚ΠΌΠ΅Ρ‚Ρ€Π° полагаСтся Ρ€Π°Π²Π½Ρ‹ΠΌ бСсконСчности. ΠŸΠΎΡΡ‚ΠΎΠΌΡƒ ΠΈΠ΄Π΅Π°Π»ΡŒΠ½Ρ‹Π΅ Π²ΠΎΠ»ΡŒΡ‚ΠΌΠ΅Ρ‚Ρ€Ρ‹ нСльзя ΠΏΠΎΡΠ»Π΅Π΄ΠΎΠ²Π°Ρ‚Π΅Π»ΡŒΠ½ΠΎ Π²Ρ‹ΠΏΠΎΠ»Π½ΠΈΡ‚ΡŒ с ΠΏΠΎΠΌΠΎΡ‰ΡŒΡŽ сопротивлСния, ΠΏΠΎΡ‚ΠΎΠΌΡƒ Ρ‡Ρ‚ΠΎ Ρ‚ΠΎΠ³Π΄Π° ΠΏΠΎ этим участкам Ρ†Π΅ΠΏΠΈ Π½Π΅ Π±ΡƒΠ΄Π΅Ρ‚ ΠΏΡ€ΠΎΡ‚Π΅ΠΊΠ°Ρ‚ΡŒ элСктричСский Ρ‚ΠΎΠΊ.Π˜Π΄Π΅Π°Π»ΡŒΠ½Ρ‹Π΅ Π²ΠΎΠ»ΡŒΡ‚ΠΌΠ΅Ρ‚Ρ€Ρ‹ ΠΏΠΎΠ΄ΠΊΠ»ΡŽΡ‡Π°ΡŽΡ‚ΡΡ ΠΊ ΡΠΎΠΏΡ€ΠΎΡ‚ΠΈΠ²Π»Π΅Π½ΠΈΡŽ Ρ‚ΠΎΠ»ΡŒΠΊΠΎ ΠΏΠ°Ρ€Π°Π»Π»Π΅Π»ΡŒΠ½ΠΎ ΠΈ Π½Π΅ ΠΈΡΠΊΠ°ΠΆΠ°ΡŽΡ‚ распрСдСлСниС Ρ‚ΠΎΠΊΠΎΠ² ΠΈ напряТСний Π² элСктричСских цСпях.

Если Π² условии ΠΏΡ€ΠΈΠ²Π΅Π΄Π΅Π½Π° элСктричСская схСма с ΠΈΠ΄Π΅Π°Π»ΡŒΠ½Ρ‹ΠΌΠΈ Π°ΠΌΠΏΠ΅Ρ€ΠΌΠ΅Ρ‚Ρ€Π°ΠΌΠΈ ΠΈ Π²ΠΎΠ»ΡŒΡ‚ΠΌΠ΅Ρ‚Ρ€Π°ΠΌΠΈ ΠΈ Π·Π°Π΄Π°Π½Π½Ρ‹ΠΌΠΈ показаниями этих ΠΏΡ€ΠΈΠ±ΠΎΡ€ΠΎΠ², Ρ‚ΠΎ Π½Π°Π΄ΠΎ ΠΈΠ΄Π΅Π°Π»ΡŒΠ½Ρ‹Π΅ ΠΏΡ€ΠΈΠ±ΠΎΡ€Ρ‹ ΡƒΠ΄Π°Π»ΠΈΡ‚ΡŒ ΠΈΠ· схСмы, ΠΎΠ±Ρ€Π°Π·Π½ΠΎ говоря, ΠΊΠ°ΠΊ ΠΏΠ°ΡƒΡ‚ΠΈΠ½Ρƒ, Π° показания ΠΏΡ€ΠΈΠ±ΠΎΡ€ΠΎΠ² «ΠΏΠ΅Ρ€Π΅Π½Π΅ΡΡ‚ΠΈ» Π½Π° Π΄Ρ€ΡƒΠ³ΠΈΠ΅ элСктричСскиС Ρ†Π΅ΠΏΠΈ сопротивлСния с ΡƒΡ‡Π΅Ρ‚ΠΎΠΌ ΠΈΡ… соСдинСния с ΠΈΠ·ΠΌΠ΅Ρ€ΠΈΡ‚Π΅Π»ΡŒΠ½Ρ‹ΠΌΠΈ цСпями ΠΏΡ€ΠΈΠ±ΠΎΡ€Π°ΠΌΠΈ.

ИспользованиС понятий ΠΎΠ± ΠΈΠ΄Π΅Π°Π»ΡŒΠ½Ρ‹Ρ… Π°ΠΌΠΏΠ΅Ρ€ΠΌΠ΅Ρ‚Ρ€Π°Ρ… ΠΈ Π²ΠΎΠ»ΡŒΡ‚ΠΌΠ΅Ρ‚Ρ€Π°Ρ… ΠΏΠΎΠ»Π΅Π·Π½ΠΎ Π΄ΠΎ ΠΏΠΎΡ€Ρ‹ Π΄ΠΎ Π²Ρ€Π΅ΠΌΠ΅Π½ΠΈ, Π½ΠΎ ΠΊΠΎΠ³Π΄Π°-Ρ‚ΠΎ Π½Π΅ΠΎΠ±Ρ…ΠΎΠ΄ΠΈΠΌΠΎ ΠΏΠ΅Ρ€Π΅ΠΉΡ‚ΠΈ ΠΊ Ρ€Π΅Π°Π»ΡŒΠ½Ρ‹ΠΌ ΠΈΠ·ΠΌΠ΅Ρ€ΠΈΡ‚Π΅Π»ΡŒΠ½Ρ‹ΠΌ ΠΏΡ€ΠΈΠ±ΠΎΡ€Π°ΠΌ ΠΈ расчСтах ΠΊΠΎΠ½Π΅Ρ‡Π½Ρ‹Π΅ значСния ΠΈΡ… Π²Π½ΡƒΡ‚Ρ€Π΅Π½Π½ΠΈΡ… сопротивлСний со всСми Π²Ρ‹Ρ‚Π΅ΠΊΠ°ΡŽΡ‰ΠΈΠΌΠΈ ΠΎΡ‚ΡΡŽΠ΄Π° услоТнСниями [3].ΠŸΡ€ΠΈ использовании Ρ€Π΅Π°Π»ΡŒΠ½Ρ‹Ρ… ΠΈΠ·ΠΌΠ΅Ρ€ΠΈΡ‚Π΅Π»ΡŒΠ½Ρ‹Ρ… ΠΏΡ€ΠΈΠ±ΠΎΡ€ΠΎΠ² ΠΈΡΠΏΠΎΠ»ΡŒΠ·ΡƒΡŽΡ‚ΡΡ вопросы измСрСния ΠΈΠ·ΠΌΠ΅Ρ€Π΅Π½ΠΈΠΉ Π²Π΅Π»ΠΈΡ‡ΠΈΠ½ Ρ‚ΠΎΠΊΠΎΠ² ΠΈ напряТСний Π½Π° участках элСктричСской Ρ†Π΅ΠΏΠΈ, Π° Ρ‚Π°ΠΊΠΆΠ΅ ΠΏΡ€ΠΎΠ±Π»Π΅ΠΌΠ° увСличСния Ρ†Π΅Π½Ρ‹ дСлСния ΡˆΠΊΠ°Π»Ρ‹ ΠΏΡ€ΠΈΠ±ΠΎΡ€ΠΎΠ².

ΠŸΡ€ΠΈ ΡƒΠ²Π΅Π»ΠΈΡ‡Π΅Π½ΠΈΠΈ Π²Π½ΡƒΡ‚Ρ€Π΅Π½Π½Π΅Π³ΠΎ сопротивлСния Π°ΠΌΠΏΠ΅Ρ€ΠΌΠ΅Ρ‚Ρ€Π°, ΠΏΠΎΠ΄ΠΊΠ»ΡŽΡ‡Π°Π΅ΠΌΠΎΠ³ΠΎ ΠΏΠΎΡΠ»Π΅Π΄ΠΎΠ²Π°Ρ‚Π΅Π»ΡŒΠ½ΠΎΠ³ΠΎ сопротивлСния элСктричСской Ρ†Π΅ΠΏΠΈ, Π΅Π³ΠΎ показания Π²ΠΎΠ·Ρ€Π°ΡΡ‚Π°ΡŽΡ‚ ΠΈ стрСмятся ΠΊ истинному Π·Π½Π°Ρ‡Π΅Π½ΠΈΡŽ элСктричСского Ρ‚ΠΎΠΊΠ°, ΠΏΡ€ΠΎΡ‚Π΅ΠΊΠ°ΡŽΡ‰Π΅Π³ΠΎ Ρ‡Π΅Ρ€Π΅Π· Π·Π°Π΄Π°Π½Π½Ρ‹Π΅ сопротивлСния элСктричСской Ρ†Π΅ΠΏΠΈ Π΄ΠΎ ΠΏΠΎΠ΄ΠΊΠ»ΡŽΡ‡Π΅Π½ΠΈΡ Π°ΠΌΠΏΠ΅Ρ€ΠΌΠ΅Ρ‚Ρ€Π° Π² ΡΠ»Π΅ΠΊΡ‚Ρ€ΠΈΡ‡Π΅ΡΠΊΡƒΡŽ Ρ†Π΅ΠΏΡŒ.

ΠŸΡ€ΠΈ ΡƒΠ²Π΅Π»ΠΈΡ‡Π΅Π½ΠΈΠΈ Π²Π½ΡƒΡ‚Ρ€Π΅Π½Π½Π΅Π³ΠΎ сопротивлСния Π²ΠΎΠ»ΡŒΡ‚ΠΌΠ΅Ρ‚Ρ€Π°, Π΅Π³ΠΎ показания Π²ΠΎΠ·Ρ€Π°ΡΡ‚Π°ΡŽΡ‚ ΠΈ стрСмятся ΠΊ истинному Π·Π½Π°Ρ‡Π΅Π½ΠΈΡŽ элСктричСского напряТСния Π½Π° Π΄Π°Π½Π½Ρ‹Ρ… сопротивлСниях Π΄ΠΎ ΠΏΠΎΠ΄ΠΊΠ»ΡŽΡ‡Π΅Π½ΠΈΡ Π²ΠΎΠ»ΡŒΡ‚ΠΌΠ΅Ρ‚Ρ€Π° Π² ΡΠ»Π΅ΠΊΡ‚Ρ€ΠΈΡ‡Π΅ΡΠΊΡƒΡŽ Ρ†Π΅ΠΏΡŒ.

Для Ρ€Π°ΡΡˆΠΈΡ€Π΅Π½ΠΈΡ ΠΈΠ½Ρ‚Π΅Ρ€Π²Π°Π»Π° Π·Π½Π°Ρ‡Π΅Π½ΠΈΠΉ измСряСмых Ρ‚ΠΎΠΊΠΎΠ² ΠΈ ΡΠΎΠΎΡ‚Π²Π΅Ρ‚ΡΡ‚Π²ΡƒΡŽΡ‰ΠΈΡ… Π·Π½Π°Ρ‡Π΅Π½ΠΈΠΉ дСлСния ΡˆΠΊΠ°Π»Ρ‹ Π°ΠΌΠΏΠ΅Ρ€ΠΌΠ΅Ρ‚Ρ€Π° ΠΊ ΡΠΎΠΎΡ‚Π²Π΅Ρ‚ΡΡ‚Π²ΡƒΡŽΡ‰Π΅ΠΌΡƒ ΠΏΠΎΠ΄ΠΊΠ»ΡŽΡ‡Π°ΡŽΡ‚ ΠΏΡ€ΠΎΠ²ΠΎΠ΄Π½ΠΈΠΊ (ΡˆΡƒΠ½Ρ‚), сопротивлСниС ΠΊΠΎΡ‚ΠΎΡ€ΠΎΠ³ΠΎ мСньшС Π²Π½ΡƒΡ‚Ρ€Π΅Π½Π½Π΅Π³ΠΎ сопротивлСния Π°ΠΌΠΏΠ΅Ρ€ΠΌΠ΅Ρ‚Ρ€Π°.Для увСличСния Ρ†Π΅Π½Ρ‹ дСлСния ΡˆΠΊΠ°Π»Ρ‹ Π°ΠΌΠΏΠ΅Ρ€ΠΌΠ΅Ρ‚Ρ€Π° Π² ΠΏ Ρ€Π°Π· ΠΊ Π½Π΅ΠΌΡƒ Π½Π°Π΄ΠΎ ΠΏΠΎΠ΄ΠΊΠ»ΡŽΡ‡ΠΈΡ‚ΡŒ ΡˆΡƒΠ½Ρ‚ с сопротивлСниСм r 15 = r Π° / (ΠΏ-1).

Для увСличСния Ρ†Π΅Π½Ρ‹ дСлСния ΡˆΠΊΠ°Π»Ρ‹ Π²ΠΎΠ»ΡŒΡ‚ΠΌΠ΅Ρ‚Ρ€Π° Π² ΠΏ Ρ€Π°Π· ΠΊ Π½Π΅ΠΌΡƒ Π½Π°Π΄ΠΎ ΡΠΎΠ΅Π΄ΠΈΠ½ΠΈΡ‚ΡŒ ΠΏΡ€ΠΎΠ²ΠΎΠ΄Π½ΠΈΠΊ (Π΄ΠΎΠΏΠΎΠ»Π½ΠΈΡ‚Π΅Π»ΡŒΠ½ΠΎΠ΅ сопротивлСниС) с R Π΄ΠΎΠΏ = (n-1) R B .

Π Π΅ΠΊΠΎΠΌΠ΅Π½Π΄ΡƒΡŽ Ρ€Π΅ΡˆΠΈΡ‚ΡŒ Π·Π°Π΄Π°Ρ‡ΠΈ β„– 527, 534, 553 — 557 ΠΈΠ· [2] Π½Π° использовании Ρ€Π΅Π°Π»ΡŒΠ½Ρ‹Ρ… ΠΈΠ·ΠΌΠ΅Ρ€ΠΈΡ‚Π΅Π»ΡŒΠ½Ρ‹Ρ… ΠΏΡ€ΠΈΠ±ΠΎΡ€ΠΎΠ² Π² цСпях постоянного элСктричСского Ρ‚ΠΎΠΊΠ°.

ΠžΡ‚Π²Π΅Ρ‚Ρ‹ ΠΊ заданию, ΠΏΡ€ΠΈΠ²Π΅Π΄Π΅Π½Π½Ρ‹ΠΌ Π² ΡΡ‚Π°Ρ‚ΡŒΠ΅

Π—Π°Π΄Π°Π½ΠΈΠ΅ 1

ВсС соСдинСния рСзисторов Π² этом Π·Π°Π΄Π°Π½ΠΈΠΈ содСрТат соСдинСниС, ΠΈΠ·ΠΎΠ±Ρ€Π°ΠΆΠ΅Π½Π½ΠΎΠ΅ Π½Π° рисункС 3. Π’Π²ΠΈΠ΄Ρƒ эквивалСнтности соСдинСний, ΠΈΠ·ΠΎΠ±Ρ€Π°ΠΆΠ΅Π½Π½Ρ‹Ρ… Π½Π° рисунках 2, 3, ΠΈΠΌΠ΅Π΅ΠΌ:

Π—Π°Π΄Π°Π½ΠΈΠ΅ 2

Π’Ρ‹Π΄Π΅Π»ΠΈΠΌ Π² Π·Π°Π΄Π°Π½Π½ΠΎΠΌ соСдинСнии рСзисторов соСдинСниС, ΠΈΠ·ΠΎΠ±Ρ€Π°ΠΆΠ΅Π½Π½ΠΎΠ΅ Π½Π° рисункС 3.

ΠŸΡ€ΠΈ ΡƒΡ‡Π΅Ρ‚Π΅ эквивалСнтности соСдинСний, ΠΈΠ·ΠΎΠ±Ρ€Π°ΠΆΠ΅Π½Π½Ρ‹Ρ… Π½Π° 2, 3, ΠΏΠΎΠ»ΡƒΡ‡Π°Π΅ΠΌ:

ЗамСняя ΠΏΠ°Ρ€Π°Π»Π»Π΅Π»ΡŒΠ½ΠΎΠ΅ соСдинСниС рСзисторов R 1 , R 2 , R 3 , ΠΎΠ΄Π½ΠΈΠΌ рСзистором R I ΠΏΠΎΠ»ΡƒΡ‡ΠΈΡ‚ΡŒ Π½ΠΎΠ²ΠΎΠ΅ соСдинСниС рСзисторов R 5 , R 4 , R I I Ρ‚ΠΈΠΏΠ° 2β€”3.

Π—Π°Π΄Π°Π½ΠΈΠ΅ 3

Π—Π°Π΄Π°Π½ΠΈΠ΅ 4

ЗамСняя поэтапно ΠΏΠΎΡΠ»Π΅Π΄ΠΎΠ²Π°Ρ‚Π΅Π»ΡŒΠ½Ρ‹Π΅ (ΠΏΠ°Ρ€Π°Π»Π»Π΅Π»ΡŒΠ½Ρ‹Π΅) соСдинСния рСзисторов ΠΎΠ΄Π½ΠΈΠΌ эквивалСнтным рСзистором, ΡƒΠΌΠ΅Π½ΡŒΡˆΠ°Π΅ΠΌ число рСзисторов Π² элСктричСской схСмС ΠΈ сводим Ρ€Π°ΡΡΠΌΠ°Ρ‚Ρ€ΠΈΠ²Π°Π΅ΠΌΡƒΡŽ схСму Π»ΠΈΠ±ΠΎ сразу ΠΊ ΠΎΠ΄Π½ΠΎΠΌΡƒ рСзистору, Π»ΠΈΠ±ΠΎ ΠΊ соСдинСнию, содСрТащСму Ρ‚ΠΎΠ»ΡŒΠΊΠΎ «Ρ‚Ρ€Π΅ΡƒΠ³ΠΎΠ»ΡŒΠ½ΠΈΠΊΠΈ» ΠΈ «Π·Π²Π΅Π·Π΄Ρ‹».

Π’ΠΎ Π²Ρ‚ΠΎΡ€ΠΎΠΌ случаС соСдинСниС рСзисторов ΠΏΠΎ схСмС «Ρ‚Ρ€Π΅ΡƒΠ³ΠΎΠ»ΡŒΠ½ΠΈΠΊ» замСняСм эквивалСнтным соСдинСниСм рСзисторов ΠΏΠΎ схСмС «Π·Π²Π΅Π·Π΄Π°» ΠΈ Π΄ΠΎΠ²ΠΎΠ΄ΠΈΠΌ ΠΏΡ€Π΅ΠΎΠ±Ρ€Π°Π·ΠΎΠ²Π°Π½ΠΈΠ΅ соСдинСния рСзисторов Π΄ΠΎ ΠΎΠ΄Π½ΠΎΠ³ΠΎ эквивалСнтного рСзистора.

ΠŸΡ€ΠΈ Ρ€Π΅Π°Π»ΠΈΠ·Π°Ρ†ΠΈΠΈ Π²Ρ‹ΡˆΠ΅ΠΈΠ·Π»ΠΎΠΆΠ΅Π½Π½ΠΎΠΉ ΠΏΡ€ΠΎΠ³Ρ€Π°ΠΌΠΌΡ‹ ΠΏΠΎΠ»ΡƒΡ‡Π°Π΅ΠΌ: Π°) R AB = (19/30) * R; Π±) R BB ‘ , = (8/15) * R; с) R AC = (6/5) * R ; Π³) R AD = (31/30) * R; Π΄) R DD ‘ = (4/5) * R; Π΅) R AE = (22/15) * R; ΠΆ) R AF = (7/6) * R.

ΠΠ»ΡŒΡ‚Π΅Ρ€Π½Π°Ρ‚ΠΈΠ²Π½Ρ‹ΠΉ ΠΌΠ΅Ρ‚ΠΎΠ΄ вычислСния сопротивлСния элСктричСских схСм Π½Π° основС ΡƒΡ‡Π΅Ρ‚Π° симмСтрии схСмы ΠΈ распрСдСлСния постоянного элСктричСского поля Π² Π½ΠΈΡ… Π±ΡƒΠ΄Π΅Ρ‚ рассмотрСн Π² ΠΎΡ‚Π΄Π΅Π»ΡŒΠ½ΠΎΠΉ ΡΡ‚Π°Ρ‚ΡŒΠ΅.

Π›ΠΈΡ‚Π΅Ρ€Π°Ρ‚ΡƒΡ€Π°

1. Кошкин, Н. И. Π‘ΠΏΡ€Π°Π²ΠΎΡ‡Π½ΠΈΠΊ ΠΏΠΎ элСмСнтарной Ρ„ΠΈΠ·ΠΈΠΊΠ΅ / Н. И. Кошкин, М. Π“. Π¨ΠΈΡ€ΠΊΠ΅Π²ΠΈΡ‡. — М. : Наука, 1965.

2. Π‘Π°Π»Π°Ρˆ, Π‘. А. Π—Π°Π΄Π°Ρ‡ΠΈ ΠΏΠΎ Ρ„ΠΈΠ·ΠΈΠΊΠ΅ ΠΈ ΠΌΠ΅Ρ‚ΠΎΠ΄Ρ‹ ΠΈΡ… Ρ€Π΅ΡˆΠ΅Π½ΠΈΡ / Π‘. А. Π‘Π°Π»Π°Ρˆ. — М. : ΠŸΡ€ΠΎΡΠ²Π΅Ρ‰Π΅Π½ΠΈΠ΅, 1967.

.

3. ΠŸΠ΅Ρ‚Ρ€ΠΎΠ²ΠΈΡ‡, Π“. И. Π¦Π΅ΠΏΠΈ постоянного элСктричСского Ρ‚ΠΎΠΊΠ° / Π“. И. ΠŸΠ΅Ρ‚Ρ€ΠΎΠ²ΠΈΡ‡, Π­. М. ШпилСвский // Π Π΅ΠΏΠ΅Ρ‚ΠΈΡ‚ΠΎΡ€. — 1996. — β„– 5 (8). — Π‘. 43β€”48; 1997. — β„– 6 (9). — Π‘. 42β€”47; 1997. — β„– 1 (10). — Π‘. 41–47.

РасчСт элСктричСских Ρ†Π΅ΠΏΠ΅ΠΉ | Π€ΠΈΠ·ΠΈΠΊΠ°

1.БмСшанноС соСдинСниС ΠΏΡ€ΠΎΠ²ΠΎΠ΄Π½ΠΈΠΊΠΎΠ²

Рассмотрим ΡΠ»Π΅ΠΊΡ‚Ρ€ΠΈΡ‡Π΅ΡΠΊΡƒΡŽ схСму Π½Π° рисункС 61.1. НСкоторыС ΠΏΡ€ΠΎΠ²ΠΎΠ΄Π½ΠΈΠΊΠΈ Π² Π½Π΅ΠΉ соСдинСны Π΄Ρ€ΡƒΠ³ с Π΄Ρ€ΡƒΠ³ΠΎΠΌ, Π° Π½Π΅ΠΊΠΎΡ‚ΠΎΡ€Ρ‹Π΅ — ΠΏΠ°Ρ€Π°Π»Π»Π΅Π»ΡŒΠ½ΠΎ.

? 1. КакиС ΠΏΡ€ΠΎΠ²ΠΎΠ΄Π½ΠΈΠΊΠΈ Π² этой схСмС соСдинСны Π΄Ρ€ΡƒΠ³ с Π΄Ρ€ΡƒΠ³ΠΎΠΌ? КакиС — ΠΏΠ°Ρ€Π°Π»Π»Π΅Π»ΡŒΠ½ΠΎ?

Π‘ΠΎΠ΅Π΄ΠΈΠ½Π΅Π½ΠΈΠ΅ ΠΏΡ€ΠΎΠ²ΠΎΠ΄Π½ΠΈΠΊΠΎΠ², ΠΏΡ€ΠΈ ΠΊΠΎΡ‚ΠΎΡ€ΠΎΠΌ Ρ‡Π°ΡΡ‚ΡŒ ΠΏΡ€ΠΎΠ²ΠΎΠ΄Π½ΠΈΠΊΠΎΠ² соСдинСна Π΄Ρ€ΡƒΠ³ с Π΄Ρ€ΡƒΠ³ΠΎΠΌ, Π° Ρ‡Π°ΡΡ‚ΡŒ — ΠΏΠ°Ρ€Π°Π»Π»Π΅Π»ΡŒΠ½ΠΎ, Π½Π°Π·Ρ‹Π²Π°ΡŽΡ‚ ΡΠΌΠ΅ΡˆΠ°Π½Π½Ρ‹ΠΌ.
ΠŸΡ€ΠΈ расчСтС сопротивлСния смСшанного соСдинСния ΠΏΡ€ΠΎΠ²ΠΎΠ΄Π½ΠΈΠΊΠΎΠ² часто ΠΈΡΠΏΠΎΠ»ΡŒΠ·ΡƒΡŽΡ‚ ΠΌΠ΅Ρ‚ΠΎΠ΄ эквивалСнтного прСобразования.ΠŸΡ€ΠΈ этом систСму ΠΏΠΎΡΠ»Π΅Π΄ΠΎΠ²Π°Ρ‚Π΅Π»ΡŒΠ½ΠΎΡΡ‚Π΅ΠΉ ΠΏΡ€Π΅ΠΎΠ±Ρ€Π°Π·ΡƒΡŽΡ‚ Π² ΠΏΡ€ΠΎΡΡ‚ΡƒΡŽ, Π½ΠΎ ΠΈΠΌΠ΅ΡŽΡ‰ΡƒΡŽ Ρ‚Π°ΠΊΠΎΠ΅ ΠΆΠ΅ сопротивлСниС.

НапримСр, схСму, ΠΈΠ·ΠΎΠ±Ρ€Π°ΠΆΠ΅Π½Π½ΡƒΡŽ Π½Π° рисункС 61.1, ΠΌΠΎΠΆΠ½ΠΎ ΠΏΡ€Π΅ΠΎΠ±Ρ€Π°Π·ΠΎΠ²Π°Ρ‚ΡŒ ΠΏΠΎ ΡΠ»Π΅Π΄ΡƒΡŽΡ‰Π΅ΠΌΡƒ ΠΏΠ»Π°Π½Ρƒ:
1. Π—Π°ΠΌΠ΅Π½ΠΈΡ‚ΡŒ участок Ρ†Π΅ΠΏΠΈ с рСзисторами 1 ΠΈ 2 ΠΎΠ΄Π½ΠΈΠΌ рСзистором с сопротивлСниСм, ΠΊΠΎΡ‚ΠΎΡ€ΠΎΠ΅ ΠΌΡ‹ ΠΎΠ±ΠΎΠ·Π½Π°Ρ‡ΠΈΠΌ R 12 .
2. Π—Π°ΠΌΠ΅Π½ΠΈΡ‚ΡŒ участок Ρ†Π΅ΠΏΠΈ, ΠΎΠ΄Π½ΠΈΠΌ рСзистором с сопротивлСниСм R 12 ΠΈ R 3 , ΠΎΠ΄Π½ΠΈΠΌ рСзистором с сопротивлСниСм, ΠΊΠΎΡ‚ΠΎΡ€ΠΎΠ΅ ΠΌΡ‹ ΠΎΠ±ΠΎΠ·Π½Π°Ρ‡ΠΈΠΌ R 123 .
3. Π—Π°ΠΌΠ΅Π½ΠΈΡ‚ΡŒ участок Ρ†Π΅ΠΏΠΈ с рСзисторами 4 ΠΈ 5 ΠΎΠ΄Π½ΠΈΠΌ рСзистором с сопротивлСниСм, ΠΌΡ‹ ΠΎΠ±ΠΎΠ·Π½Π°Ρ‡ΠΈΠΌ R 45 .
4. Π—Π°ΠΌΠ΅Π½ΠΈΡ‚ΡŒ участок Ρ†Π΅ΠΏΠΈ с рСзисторами сопротивлСниСм R 123 ΠΈ R 45 ΠΎΠ΄Π½ΠΈΠΌ рСзистором. Π•Π³ΠΎ сопротивлСниС ΠΈ Π±ΡƒΠ΄Π΅Ρ‚ Ρ€Π°Π²Π½ΠΎ ΡΠΎΠΏΡ€ΠΎΡ‚ΠΈΠ²Π»Π΅Π½ΠΈΡŽ всСго участка Ρ†Π΅ΠΏΠΈ.

? 2. Π’ Ρ†Π΅ΠΏΠΈ, схСма ΠΊΠΎΡ‚ΠΎΡ€ΠΎΠΉ ΠΈΠ·ΠΎΠ±Ρ€Π°ΠΆΠ΅Π½Π° Π½Π° рисункС 61.1, сопротивлСниС ΠΊΠ°ΠΆΠ΄ΠΎΠ³ΠΎ Π²Ρ‹Ρ€Π°ΠΆΠ΅Π½Π½ΠΎΠ΅ Π² ΠΎΠΌΠ°Ρ…, ΠΏΡ€ΠΈΠΌΠΈΡ‚Π΅ Ρ‚Π΅ΠΌ Ρ€Π°Π²Π½Ρ‹ΠΌ Π½ΠΎΠΌΠ΅Ρ€Ρƒ этого рСзистора. НачСртитС схСмы, ΡΠΎΠΎΡ‚Π²Π΅Ρ‚ΡΡ‚Π²ΡƒΡŽΡ‰ΠΈΠ΅ ΠΊΠ°ΠΆΠ΄ΠΎΠΌΡƒ ΠΏΡƒΠ½ΠΊΡ‚Ρƒ Π½Π°ΠΌΠ΅Ρ‡Π΅Π½Π½ΠΎΠ³ΠΎ Π²Ρ‹ΡˆΠ΅ ΠΏΠ»Π°Π½Π°; Π½Π°ΠΉΠ΄ΠΈΡ‚Π΅ R 12 , R 123 , R 45 ΠΈ сопротивлСниС всСго участка.

НС всСгда с ΠΏΠ΅Ρ€Π²ΠΎΠ³ΠΎ взгляда Π½Π° ΡΠ»Π΅ΠΊΡ‚Ρ€ΠΈΡ‡Π΅ΡΠΊΡƒΡŽ схСму ΠΌΠΎΠΆΠ½ΠΎ Ρ€Π°ΡΠΏΠΎΠ·Π½Π°Ρ‚ΡŒ Π²ΠΈΠ΄ соСдинСния ΠΏΡ€ΠΎΠ²ΠΎΠ΄Π½ΠΈΠΊΠΎΠ².

Π’ Ρ‚Π°ΠΊΠΎΠΌ случаС ΠΏΠΎΠΌΠΎΠ³Π°Π΅Ρ‚ Π½Π°ΠΉΡ‚ΠΈ Ρ‚ΠΎΡ‡ΠΊΠΈ с ΠΎΠ΄ΠΈΠ½Π°ΠΊΠΎΠ²Ρ‹ΠΌ ΠΏΠΎΡ‚Π΅Π½Ρ†ΠΈΠ°Π»ΠΎΠΌ (Π½Π°ΠΏΡ€ΠΈΠΌΠ΅Ρ€, соСдинСнныС ΠΏΡ€ΠΎΠ²ΠΎΠ΄Π°ΠΌΠΈ, сопротивлСниС Π² Ρ‚Π°ΠΊΠΈΡ… случаях, ΠΊΠ°ΠΊ ΠΎΠ±Ρ‹Ρ‡Π½ΠΎ ΠΏΡ€Π΅Π½Π΅Π±Ρ€Π΅ΠΆΠΈΠΌΠΎ ΠΌΠ°Π»Ρ‹ΠΌ). Π—Π°Ρ‚Π΅ΠΌ Π½Π°Π΄ΠΎ ΠΏΠ΅Ρ€Π΅Ρ‡Π΅Ρ€Ρ‚ΠΈΡ‚ΡŒ схСму, объСдинив Ρ‚ΠΎΡ‡ΠΊΠΈ с ΠΎΠ΄ΠΈΠ½Π°ΠΊΠΎΠ²Ρ‹ΠΌ ΠΏΠΎΡ‚Π΅Π½Ρ†ΠΈΠ°Π»ΠΎΠΌ.

Рассмотрим, Π½Π°ΠΏΡ€ΠΈΠΌΠ΅Ρ€, схСму участка Ρ†Π΅ΠΏΠΈ, ΠΈΠ·ΠΎΠ±Ρ€Π°ΠΆΠ΅Π½Π½ΡƒΡŽ Π½Π° рисункС 61.2.

Π’ΠΎΡ‡ΠΊΠΈ А ΠΈ Π‘ соСдинСны ΠΏΡ€ΠΎΠ²ΠΎΠ΄ΠΎΠΌ с ΠΏΡ€Π΅Π½Π΅Π±Ρ€Π΅ΠΆΠΈΠΌΠΎ ΠΌΠ°Π»Ρ‹ΠΌ сопротивлСниСм, поэтому ΠΏΠΎΡ‚Π΅Π½Ρ†ΠΈΠ°Π»Ρ‹ этих Ρ€Π°Π²Π½Ρ‹.Π’ΠΎ ΠΆΠ΅ ΠΌΠΎΠΆΠ½ΠΎ ΡΠΊΠ°Π·Π°Ρ‚ΡŒ ΠΈ ΠΎ Ρ‚ΠΎΡ‡ΠΊΠ°Ρ… Π’ ΠΈ D.

Π‘Π»Π΅Π΄ΠΎΠ²Π°Ρ‚Π΅Π»ΡŒΠ½ΠΎ, схСму ΠΌΠΎΠΆΠ½ΠΎ ΠΏΠ΅Ρ€Π΅Ρ‡Π΅Ρ€Ρ‚ΠΈΡ‚ΡŒ, объСдинив Ρ‚ΠΎΡ‡ΠΊΠΈ А ΠΈ Π‘ Π² ΠΎΠ΄Π½Ρƒ Ρ‚ΠΎΡ‡ΠΊΡƒ (ΠΎΠ±ΠΎΠ·Π½Π°Ρ‡ΠΈΠΌ Π΅Π΅ АБ), Π° Ρ‚ΠΎΡ‡ΠΊΠΈ Π’ ΠΈ D объСдинив Π² Ρ‚ΠΎΡ‡ΠΊΡƒ Π’D. ΠŸΡ€ΠΈ этом, согласно исходной схСмС, ΠΎΠ΄ΠΈΠ½ ΠΊΠΎΠ½Π΅Ρ† ΠΊΠ°ΠΆΠ΄ΠΎΠ³ΠΎ ΠΈΠ· Ρ‚Ρ€Π΅Ρ… рСзистов соСдинСн с Ρ‚ΠΎΡ‡ΠΊΠΎΠΉ АБ, Π° Π΄Ρ€ΡƒΠ³ΠΎΠΉ — с Ρ‚ΠΎΡ‡ΠΊΠΎΠΉ BD (рис. 61.3).

Π’Π΅ΠΏΠ΅Ρ€ΡŒ ΠΌΡ‹ Π²ΠΈΠ΄ΠΈΠΌ, Ρ‡Ρ‚ΠΎ рСзисторы соСдинСны ΠΏΠ°Ρ€Π°Π»Π»Π΅Π»ΡŒΠ½ΠΎ.

? 3. ΠŸΠ΅Ρ€Π΅Π½Π΅ΡΠΈΡ‚Π΅ Π² Ρ‚Π΅Ρ‚Ρ€Π°Π΄ΡŒ рисунок 61.2 ΠΈ ΠΎΡ‚ΠΌΠ΅Ρ‚ΡŒΡ‚Π΅ Π½Π° Π½Π΅ΠΌ Π½Π°ΠΏΡ€Π°Π²Π»Π΅Π½ΠΈΠ΅ Ρ‚ΠΎΠΊΠ° Π² ΠΊΠ°ΠΆΠ΄ΠΎΠΌ рСзисторС, считая, Ρ‡Ρ‚ΠΎ ΠΏΠΎΡ‚Π΅Π½Ρ†ΠΈΠ°Π» Ρ‚ΠΎΡ‡ΠΊΠΈ А Π²Ρ‹ΡˆΠ΅ Ρ‚ΠΎΡ‡ΠΊΠΈ D.

? 4. На рисункС 61.4 ΠΈΠ·ΠΎΠ±Ρ€Π°ΠΆΠ΅Π½Π° схСма участка элСктричСской Ρ†Π΅ΠΏΠΈ. Π‘ΠΎΠΏΡ€ΠΎΡ‚ΠΈΠ²Π»Π΅Π½ΠΈΠ΅ ΠΊΠ°ΠΆΠ΄ΠΎΠ³ΠΎ рСзистора, Π²Ρ‹Ρ€Π°ΠΆΠ΅Π½Π½ΠΎΠ΅ Π² ΠΎΠΌΠ°Ρ…, Ρ€Π°Π²Π½ΠΎ Π½ΠΎΠΌΠ΅Ρ€Ρƒ рСзистора. ΠžΠ±Ρ€Π°Ρ‚ΠΈΡ‚Π΅ Π²Π½ΠΈΠΌΠ°Π½ΠΈΠ΅: ΠΏΠΎΡ‚Π΅Π½Ρ†ΠΈΠ°Π»Ρ‹ Ρ‚ΠΎΡ‡Π΅ΠΊ А ΠΈ Π‘ Ρ€Π°Π·Π»ΠΈΡ‡Π½Ρ‹.

Π°) ΠŸΠ΅Ρ€Π΅Ρ‡Π΅Ρ€Ρ‚ΠΈΡ‚Π΅ схСму, ΠΈΠ·ΠΎΠ±Ρ€Π°ΠΆΠ΅Π½Π½ΡƒΡŽ Π½Π° рисункС 61.4, Ρ‚Π°ΠΊ, Ρ‡Ρ‚ΠΎΠ±Ρ‹ Π»Π΅Π³ΠΊΠΎ Π±Ρ‹Π»ΠΎ Ρ€Π°ΡΠΏΠΎΠ·Π½Π°Ρ‚ΡŒ Π²ΠΈΠ΄ соСдинСния рСзисторов.
Π±) НайдитС сопротивлСниС всСго участка Ρ†Π΅ΠΏΠΈ.

К соТалСнию, Π½Π΅ Π²ΡΡΠΊΡƒΡŽ ΡΠ»Π΅ΠΊΡ‚Ρ€ΠΈΡ‡Π΅ΡΠΊΡƒΡŽ схСму ΠΌΠΎΠΆΠ½ΠΎ поэтапно ΡƒΠΏΡ€ΠΎΡ‰Π°Ρ‚ΡŒ, ΠΈΡΠΏΠΎΠ»ΡŒΠ·ΡƒΡ Ρ‚ΠΎΠ»ΡŒΠΊΠΎ Ρ„ΠΎΡ€ΠΌΡƒΠ»Ρ‹ для ΠΏΠΎΡΠ»Π΅Π΄ΠΎΠ²Π°Ρ‚Π΅Π»ΡŒΠ½ΠΎΠ³ΠΎ ΠΈ ΠΏΠ°Ρ€Π°Π»Π»Π΅Π»ΡŒΠ½ΠΎΠ³ΠΎ соСдинСний.На рисункС 61.5 ΠΏΡ€ΠΈΠ²Π΅Π΄Π΅Π½ ΠΏΡ€ΠΈΠΌΠ΅Ρ€ схСмы участка Ρ†Π΅ΠΏΠΈ, ΠΊΠΎΡ‚ΠΎΡ€ΡƒΡŽ нСльзя ΡƒΠΏΡ€ΠΎΡΡ‚ΠΈΡ‚ΡŒ Ρ‚Π°ΠΈΠΌ ΠΎΠ±Ρ€Π°Π·ΠΎΠΌ.

Но для Π½Π΅ΠΊΠΎΡ‚ΠΎΡ€Ρ‹Ρ… частных случав ΠΌΠΎΠΆΠ½ΠΎ Π½Π°ΠΉΡ‚ΠΈ сопротивлСниС ΠΈ Ρ‚Π°ΠΊΠΎΠ³ΠΎ участка Ρ†Π΅ΠΏΠΈ ΡƒΠΆΠ΅ извСстными Π½Π°ΠΌ способами. Π§Ρ‚ΠΎΠ±Ρ‹ Π΄ΠΎΠ³Π°Π΄Π°Ρ‚ΡŒΡΡ, ΠΊΠ°ΠΊΠΎΠ²Ρ‹ эти случаи, Π·Π°ΠΌΠ΅Π½ΠΈΠΌ рСзистор 5 ΠΈΠ΄Π΅Π°Π»ΡŒΠ½Ρ‹ΠΌ Π²ΠΎΠ»ΡŒΡ‚ΠΌΠ΅Ρ‚Ρ€ΠΎΠΌ (рис. 61.6). (Напомним, Ρ‡Ρ‚ΠΎ ΠΈΠ΄Π΅Π°Π»ΡŒΠ½Ρ‹ΠΌ ΡΡ‡ΠΈΡ‚Π°ΡŽΡ‚ Π²ΠΎΠ»ΡŒΡ‚ΠΌΠ΅Ρ‚Ρ€, сопротивлСниС ΠΊΠΎΡ‚ΠΎΡ€ΠΎΠ³ΠΎ ΠΌΠΎΠΆΠ½ΠΎ ΠΏΡ€ΠΈΠ½ΡΡ‚ΡŒ бСсконСчно большим.)

? 5. Π Π°Π·Π½ΠΎΡΡ‚ΡŒ ΠΏΠΎΡ‚Π΅Π½Ρ†ΠΈΠ°Π»ΠΎΠ² ΠΌΠ΅ΠΆΠ΄Ρƒ Ρ‚ΠΎΡ‡ΠΊΠ°ΠΌΠΈ А ΠΈ Π’ Ρ€Π°Π²Π½Π° 21 Π’. БопротивлСния рСзисторов, Π²Ρ‹Ρ€Π°ΠΆΠ΅Π½Π½Ρ‹Π΅ Π² ΠΎΠΌΠ°Ρ…, Ρ€Π°Π²Π½Ρ‹ ΠΈΡ… Π½ΠΎΠΌΠ΅Ρ€Π°ΠΌ.
Π°) Π§Π΅ΠΌΡƒ Ρ€Π°Π²Π½Π° Ρ€Π°Π·Π½ΠΎΡΡ‚ΡŒ ΠΏΠΎΡ‚Π΅Π½Ρ†ΠΈΠ°Π»ΠΎΠ² ΠΌΠ΅ΠΆΠ΄Ρƒ Ρ‚ΠΎΡ‡ΠΊΠ°ΠΌΠΈ А ΠΈ Π‘?
Π±) Π§Π΅ΠΌΡƒ Ρ€Π°Π²Π½Π° Ρ€Π°Π·Π½ΠΎΡΡ‚ΡŒ ΠΏΠΎΡ‚Π΅Π½Ρ†ΠΈΠ°Π»ΠΎΠ² ΠΌΠ΅ΠΆΠ΄Ρƒ Ρ‚ΠΎΡ‡ΠΊΠ°ΠΌΠΈ А ΠΈ D?
Π²) ΠšΠ°ΠΊΠΎΠ²Ρ‹ показания Π²ΠΎΠ»ΡŒΡ‚ΠΌΠ΅Ρ‚Ρ€Π°?
Π³) РСзистором с ΠΊΠ°ΠΊΠΈΠΌ сопротивлСниСм Π½Π°Π΄ΠΎ Π·Π°ΠΌΠ΅Π½ΠΈΡ‚ΡŒ рСзистор 4, Ρ‡Ρ‚ΠΎΠ±Ρ‹ показания Π²ΠΎΠ»ΡŒΡ‚ΠΌΠ΅Ρ‚Ρ€Π° Π±Ρ‹Π»ΠΈ Ρ€Π°Π²Π½Ρ‹ Π½ΡƒΠ»ΡŽ?

? 6. ΠžΠ±ΡŠΡΡΠ½ΠΈΡ‚Π΅, ΠΏΠΎΡ‡Π΅ΠΌΡƒ показания Π²ΠΎΠ»ΡŒΡ‚ΠΌΠ΅Ρ‚Ρ€Π° Π΄ΠΎΠ»ΠΆΠ½Ρ‹ Π±Ρ‹Ρ‚ΡŒ Ρ€Π°Π²Π½Ρ‹ Π½ΡƒΠ»ΡŽ нСзависимо ΠΎΡ‚ напряТСния ΠΌΠ΅ΠΆΠ΄Ρƒ Ρ‚ΠΎΡ‡ΠΊΠ°ΠΌΠΈ А ΠΈ Π’, Ссли сопротивлСниС рСзисторов Π½Π° схСмС, ΠΈΠ·ΠΎΠ±Ρ€Π°ΠΆΠ΅Π½Π½ΠΎΠΉ Π½Π° рисункС 61.6, ΡƒΠ΄ΠΎΠ²Π»Π΅Ρ‚Π²ΠΎΡ€ΡΡŽΡ‚ ΠΏΡ€ΠΈΠΌΠ΅Π½Π΅Π½ΠΈΡŽ

R 1 / R 2 = R 3 / R 4 .(1)

Π‘Ρ…Π΅ΠΌΡƒ, ΠΈΠ·ΠΎΠ±Ρ€Π°ΠΆΠ΅Π½Π½ΡƒΡŽ Π½Π° рисункС 61.6, Π½Π°Π·Ρ‹Π²Π°ΡŽΡ‚ мостиком Уитстона. Π‘ Π΅Π΅ ΠΏΠΎΠΌΠΎΡ‰ΡŒΡŽ ΠΌΠΎΠΆΠ½ΠΎ ΠΈΠ·ΠΌΠ΅Ρ€ΠΈΡ‚ΡŒ сопротивлСниС ΠΈΠ· Ρ‡Π΅Ρ‚Ρ‹Ρ€Π΅Ρ… рСзисторов, подбирая сопротивлСниС ΠΎΡΡ‚Π°Π»ΡŒΠ½Ρ‹Ρ… Ρ‚Ρ€Π΅Ρ… Ρ‚Π°ΠΊ, Ρ‡Ρ‚ΠΎΠ±Ρ‹ Π²Ρ‹ΠΏΠΎΠ»Π½ΡΠ»ΠΎΡΡŒ ΡΠΎΠΎΡ‚Π½ΠΎΡˆΠ΅Π½ΠΈΠ΅ (1).

? 7. Для сопротивлСний рСзисторов 1 — 4 Π² Ρ†Π΅ΠΏΠΈ, ΠΈΠ·ΠΎΠ±Ρ€Π°ΠΆΠ΅Π½Π½ΠΎΠΉ Π½Π° рисункС 61.5, выполняСтся ΡΠΎΠΎΡ‚Π½ΠΎΡˆΠ΅Π½ΠΈΠ΅ (1).
Π°) ΠžΠ±ΡŠΡΡΠ½ΠΈΡ‚Π΅, ΠΏΠΎΡ‡Π΅ΠΌΡƒ сопротивлСниС Π΄Π°Π½Π½ΠΎΠ³ΠΎ участка Ρ†Π΅ΠΏΠΈ Π½Π΅ зависит ΠΎΡ‚ сопротивлСния рСзистора 5.
Π±) Π‘ΠΎΠΏΡ€ΠΎΡ‚ΠΈΠ²Π»Π΅Π½ΠΈΠ΅ рСзисторов 1 ΠΈ 3 Ρ€Π°Π²Π½Ρ‹ соотвСтствСнно 10 Ом ΠΈ 15 Ом.ΠŸΠΎΠ΄Π±Π΅Ρ€ΠΈΡ‚Π΅ Ρ‚Π°ΠΊΠΈΠ΅ значСния сопротивлСний рСзисторов 2 ΠΈ 4, Ρ‡Ρ‚ΠΎΠ±Ρ‹ сопротивлСниС всСго участка Π±Ρ‹Π»ΠΎ Ρ€Π°Π²Π½ΠΎ 24 Ом нСзависимо ΠΎΡ‚ сопротивлСния рСзистора 5.

2. Максимальная ΠΌΠΎΡ‰Π½ΠΎΡΡ‚ΡŒ Π²ΠΎ внСшнСй Ρ†Π΅ΠΏΠΈ

? 8. К источнику с Π­Π”Π‘ ΞΎ ΠΈ Π²Π½ΡƒΡ‚Ρ€Π΅Π½Π½ΠΈΠΌ сопротивлСниСм ΠΏΠΎΠ΄ΠΊΠ»ΡŽΡ‡Π΅Π½ΠΎ внСшнСС сопротивлСниС R (рис. 61.7).

Π°) Π’Ρ‹Ρ€Π°Π·ΠΈΡ‚Π΅ ΠΌΠΎΡ‰Π½ΠΎΡΡ‚ΡŒ Ρ‚ΠΎΠΊΠ° Π²ΠΎ внСшнСй Ρ†Π΅ΠΏΠΈ Ρ‡Π΅Ρ€Π΅Π· ΞΎ, r ΠΈ R.
Π±) Π˜ΡΠΏΠΎΠ»ΡŒΠ·ΡƒΡ ΠΏΡ€ΠΎΠΈΠ·Π²ΠΎΠ΄Π½ΡƒΡŽ, Π½Π°ΠΉΠ΄ΠΈΡ‚Π΅, ΠΏΡ€ΠΈ ΠΊΠ°ΠΊΠΎΠΌ R ΠΌΠΎΡ‰Π½ΠΎΡΡ‚ΡŒ Ρ‚ΠΎΠΊΠ° Π²ΠΎ внСшнСй Ρ†Π΅ΠΏΠΈ Π±ΡƒΠ΄Π΅Ρ‚ максимальной.

Π­Ρ‚Ρƒ Π·Π°Π΄Π°Ρ‡Ρƒ ΠΌΠΎΠΆΠ½ΠΎ Ρ€Π΅ΡˆΠΈΡ‚ΡŒ ΠΈ Π±Π΅Π· ΠΏΠΎΠΌΠΎΡ‰ΠΈ ΠΏΡ€ΠΎΠΈΠ·Π²ΠΎΠ΄Π½ΠΎΠΉ.Для этого Π½Π°Π΄ΠΎ Ρ„ΠΎΡ€ΠΌΡƒΠ»ΠΎΠΉ для мощности Ρ‚ΠΎΠΊΠ° Π²ΠΎ внСшнСй Ρ†Π΅ΠΏΠΈ

P = UI,

Π³Π΄Π΅ U — напряТСниС Π½Π° внСшнСм сопротивлСнии (Π½Π°ΠΏΠΎΠΌΠ½ΠΈΠΌ, Ρ‡Ρ‚ΠΎ ΠΎΠ½ΠΎ Ρ€Π°Π²Π½ΠΎ Π½Π°ΠΏΡ€ΡΠΆΠ΅Π½ΠΈΡŽ Π½Π° ΠΏΠΎΠ»ΡŽΡΠ°Ρ… источника Ρ‚ΠΎΠΊΠ°), I — сила Ρ‚ΠΎΠΊΠ° Π² Ρ†Π΅ΠΏΠΈ.

? 9. ΠžΠ±ΡŠΡΡΠ½ΠΈΡ‚Π΅, ΠΏΠΎΡ‡Π΅ΠΌΡƒ ΠΌΠΎΡ‰Π½ΠΎΡΡ‚ΡŒ Ρ‚ΠΎΠΊΠ° Π²ΠΎ внСшнСй Ρ†Π΅ΠΏΠΈ выраТаСтся Ρ„ΠΎΡ€ΠΌΡƒΠ»ΠΎΠΉ

P = (ΞΎ — Ir) I. (2)

Подсказка. Π’Ρ‹Ρ€Π°Π·ΠΈΡ‚Π΅ напряТСниС Π½Π° ΠΏΠΎΠ»ΡŽΡΠ°Ρ… источника Ρ‡Π΅Ρ€Π΅Π· ΞΎ, I, r, ΠΈΡΠΏΠΎΠ»ΡŒΠ·ΡƒΡ Π·Π°ΠΊΠΎΠ½ Ома для всСй Ρ†Π΅ΠΏΠΈ.

ΠŸΡ€Π°Π²Π°Ρ Ρ‡Π°ΡΡ‚ΡŒ равСнства (2) прСдставляСт собой ΠΊΠ²Π°Π΄Ρ€Π°Ρ‚ΠΈΡ‡Π½ΡƒΡŽ Ρ„ΡƒΠ½ΠΊΡ†ΠΈΡŽ ΠΎΡ‚ силы Ρ‚ΠΎΠΊΠ° I.Π“Ρ€Π°Ρ„ΠΈΠΊΠΎΠΌ Π΅Π΅ являСтся ΠΏΠ°Ρ€Π°Π±ΠΎΠ»Π°.

? 10. НачСртитС Π³Ρ€Π°Ρ„ΠΈΠΊ зависимости P (I) ΠΏΡ€ΠΈ ΠΈΠ·ΠΌΠ΅Π½Π΅Π½ΠΈΠΈ силы Ρ‚ΠΎΠΊΠ° I ΠΎΡ‚ нуля Π΄ΠΎ Π²Π΅Π»ΠΈΡ‡ΠΈΠ½Ρ‹ значСния (Ρ€Π°Π²Π½ΠΎΠ³ΠΎ силС Ρ‚ΠΎΠΊΠ° ΠΏΡ€ΠΈ ΠΊΠΎΡ€ΠΎΡ‚ΠΊΠΎΠΌ Π·Π°ΠΌΡ‹ΠΊΠ°Π½ΠΈΠΈ).
Π°) ΠŸΡ€ΠΈ ΠΊΠ°ΠΊΠΎΠΌ Π·Π½Π°Ρ‡Π΅Π½ΠΈΠΈ достигаСтся максимум Ρ„ΡƒΠ½ΠΊΡ†ΠΈΠΈ P (I)?
Π±) ΠšΠ°ΠΊΠΎΠΌΡƒ ΡΠΎΠΏΡ€ΠΎΡ‚ΠΈΠ²Π»Π΅Π½ΠΈΡŽ внСшнСй Ρ†Π΅ΠΏΠΈ соотвСтствуСт это Π·Π½Π°Ρ‡Π΅Π½ΠΈΠ΅ I?
Подсказка. Π’ΠΎΡΠΏΠΎΠ»ΡŒΠ·ΡƒΠΉΡ‚Π΅ΡΡŒ Π²ΠΎΠ·ΠΌΠΎΠΆΠ½ΠΎΡΡ‚ΡŒΡŽ Π·Π°ΠΊΠΎΠ½ΠΎΠΌ Ома для всСй Ρ†Π΅ΠΏΠΈ.

Π˜Ρ‚Π°ΠΊ, максимальная ΠΌΠΎΡ‰Π½ΠΎΡΡ‚ΡŒ Ρ‚ΠΎΠΊΠ° Π²ΠΎ внСшнСй Ρ†Π΅ΠΏΠΈ достигаСтся, ΠΊΠΎΠ³Π΄Π° сопротивлСниС внСшнСй Ρ†Π΅ΠΏΠΈ Π²Π½ΡƒΡ‚Ρ€Π΅Π½Π½Π΅ΠΌΡƒ ΡΠΎΠΏΡ€ΠΎΡ‚ΠΈΠ²Π»Π΅Π½ΠΈΡŽ источника Ρ‚ΠΎΠΊΠ°.

? 11. Π§Π΅ΠΌΡƒ ΠΏΡ€ΠΈ этом равСнствС ΠšΠŸΠ” источника Ρ‚ΠΎΠΊΠ°?

3. ΠšΠΎΠ½Π΄Π΅Π½ΡΠ°Ρ‚ΠΎΡ€Ρ‹ Π² Ρ†Π΅ΠΏΠΈ постоянного Ρ‚ΠΎΠΊΠ°

ΠŸΠΎΡΡ‚ΠΎΡΠ½Π½Ρ‹ΠΉ Ρ‚ΠΎΠΊ Π½Π΅ ΠΌΠΎΠΆΠ΅Ρ‚ ΠΈΠ΄Ρ‚ΠΈ Ρ‡Π΅Ρ€Π΅Π· кондСнсатор, ΠΏΠΎΡ‚ΠΎΠΌΡƒ Ρ‡Ρ‚ΠΎ ΠΌΠ΅ΠΆΠ΄Ρƒ Π΅Π³ΠΎ ΠΎΠ±ΠΊΠ»Π°Π΄ΠΊΠ°ΠΌΠΈ находится диэлСктрик. Однако ΠΌΠ΅ΠΆΠ΄Ρƒ ΠΎΠ±ΠΊΠ»Π°Π΄ΠΊΠ°ΠΌΠΈ кондСнсатора, Π²ΠΊΠ»ΡŽΡ‡Π΅Π½Π½ΠΎΠ³ΠΎ Π² Ρ†Π΅ΠΏΡŒ постоянного Ρ‚ΠΎΠΊΠ°, ΠΌΠΎΠΆΠ΅Ρ‚ ΡΡƒΡ‰Π΅ΡΡ‚Π²ΠΎΠ²Π°Ρ‚ΡŒ Ρ€Π°Π·Π½ΠΎΡΡ‚ΡŒ ΠΏΠΎΡ‚Π΅Π½Ρ†ΠΈΠ°Π»ΠΎΠ², ΠΈ Ρ‚ΠΎΠ³Π° кондСнсатор Π±ΡƒΠ΄Π΅Ρ‚ заряТСнным. НачнСм с самых простых случаСв, ΠΊΠΎΠ³Π΄Π° Π² Ρ†Π΅ΠΏΠΈ, ΠΏΠΎΠΌΠΈΠΌΠΎ кондСнсатора, Π΅ΡΡ‚ΡŒ Ρ‚ΠΎΠ»ΡŒΠΊΠΎ ΠΎΠ΄ΠΈΠ½ рСзистор.

? 12. На рисункС 61.8 ΠΈΠ·ΠΎΠ±Ρ€Π°ΠΆΠ΅Π½Π° схСма элСктричСской Ρ†Π΅ΠΏΠΈ. Π­Π”Π‘ источника Ρ‚ΠΎΠΊΠ° ΞΎ = 12 Π’, Π΅Π³ΠΎ Π²Π½ΡƒΡ‚Ρ€Π΅Π½Π½Π΅Π΅ сопротивлСниС r = 2 Ом, сопротивлСниС сопротивлСниС R = 10 Ом, ΡΠ»Π΅ΠΊΡ‚Ρ€ΠΎΠ΅ΠΌΠΊΠΎΡΡ‚ΡŒ кондСнсатора Π‘ = 2 ΠΌΠΊΠ€.
61,8
Π°) Π§Π΅ΠΌΡƒ Ρ€Π°Π²Π½Π° Ρ€Π°Π·Π½ΠΎΡΡ‚ΡŒ ΠΏΠΎΡ‚Π΅Π½Ρ†ΠΈΠ°Π»ΠΎΠ² ΠΌΠ΅ΠΆΠ΄Ρƒ Ρ‚ΠΎΡ‡ΠΊΠ°ΠΌΠΈ А ΠΈ Π’?
Π±) Π§Π΅ΠΌΡƒ Ρ€Π°Π²Π½Π° Ρ€Π°Π·Π½ΠΎΡΡ‚ΡŒ ΠΏΠΎΡ‚Π΅Π½Ρ†ΠΈΠ°Π»ΠΎΠ² ΠΌΠ΅ΠΆΠ΄Ρƒ Ρ‚ΠΎΡ‡ΠΊΠ°ΠΌΠΈ А ΠΈ D?
Π²) Π§Π΅ΠΌΡƒ Ρ€Π°Π²Π΅Π½ заряд кондСнсатора?
Π³) Каков Π·Π½Π°ΠΊ заряда ΠΎΠ±ΠΊΠ»Π°Π΄ΠΊΠΈ кондСнсатора, соСдинСнной с рСзистором?

? 13. На рисункС 61.9 ΠΈΠ·ΠΎΠ±Ρ€Π°ΠΆΠ΅Π½Π° схСма элСктричСской Ρ†Π΅ΠΏΠΈ.Π­Π”Π‘ источника ΞΎ, Π΅Π³ΠΎ Π²Π½ΡƒΡ‚Ρ€Π΅Π½Π½Π΅Π΅ сопротивлСниС Ρ‚ΠΎΠΊΠ° r, сопротивлСниС рСзистора R, ΡΠ»Π΅ΠΊΡ‚Ρ€ΠΎΠ΅ΠΌΠΊΠΎΡΡ‚ΡŒ кондСнсатора C.

Π°) Π§Π΅ΠΌΡƒ Ρ€Π°Π²Π½Π° Ρ€Π°Π·Π½ΠΎΡΡ‚ΡŒ ΠΏΠΎΡ‚Π΅Π½Ρ†ΠΈΠ°Π»ΠΎΠ² ΠΌΠ΅ΠΆΠ΄Ρƒ Ρ‚ΠΎΡ‡ΠΊΠ°ΠΌΠΈ А ΠΈ Π’?
Π±) Π§Π΅ΠΌΡƒ Ρ€Π°Π²Π΅Π½ заряд кондСнсатора?

Рассмотрим Ρ‚Π΅ΠΏΠ΅Ρ€ΡŒ Π±ΠΎΠ»Π΅Π΅ слоТный случай, ΠΊΠΎΠ³Π΄Π° Π² Ρ†Π΅ΠΏΠΈ Π΅ΡΡ‚ΡŒ нСсколько рСзисторов, ΠΏΡ€ΠΈΡ‡Π΅ΠΌ ΠΎΠ½ΠΈ ΠΏΠΎ-Ρ€Π°Π·Π½ΠΎΠΌΡƒ ΠΏΠΎΠ΄ΠΊΠ»ΡŽΡ‡Π΅Π½Ρ‹ ΠΊ кондСнсатору.

? 14. Π’ Ρ†Π΅ΠΏΠΈ (рис. 61.10) Π­Π”Π‘ ΞΎ = 6 Π’, Π΅Π³ΠΎ Π²Π½ΡƒΡ‚Ρ€Π΅Π½Π½Π΅Π΅ сопротивлСниС r = 1 Ом, сопротивлСниС рСзисторов R 1 = 3 Ом, R 2 = 5 Ом, R 3 = 12 Ом, ΡΠ»Π΅ΠΊΡ‚Ρ€ΠΎΠ΅ΠΌΠΊΠΎΡΡ‚ΡŒ кондСнсатора C = 8 ΠΌΠΊΠ€.

Π°) ΠŸΠ΅Ρ€Π΅Π½Π΅ΡΠΈΡ‚Π΅ схСму Π² Ρ‚Π΅Ρ‚Ρ€Π°Π΄ΡŒ ΠΈ ΠΎΠ±ΠΎΠ·Π½Π°Ρ‡ΡŒΡ‚Π΅, Ρ‡Π΅Ρ€Π΅Π· ΠΊΠ°ΠΊΠΈΠ΅ элСмСнты Ρ†Π΅ΠΏΠΈ ΠΈΠ΄Π΅Ρ‚ Ρ‚ΠΎΠΊ.
Π±) Какова сила Ρ‚ΠΎΠΊΠ° Π² рСзисторС 3?
Π²) Π§Π΅ΠΌΡƒ Ρ€Π°Π²Π½Π° Ρ€Π°Π·Π½ΠΎΡΡ‚ΡŒ ΠΏΠΎΡ‚Π΅Π½Ρ†ΠΈΠ°Π»ΠΎΠ² ΠΌΠ΅ΠΆΠ΄Ρƒ Ρ‚ΠΎΡ‡ΠΊΠ°ΠΌΠΈ А ΠΈ D?
Π³) Π§Π΅ΠΌΡƒ Ρ€Π°Π²Π½Π° Ρ€Π°Π·Π½ΠΎΡΡ‚ΡŒ ΠΏΠΎΡ‚Π΅Π½Ρ†ΠΈΠ°Π»ΠΎΠ² ΠΌΠ΅ΠΆΠ΄Ρƒ Ρ‚ΠΎΡ‡ΠΊΠ°ΠΌΠΈ А ΠΈ Π’?
Π΄) Π§Π΅ΠΌΡƒ Ρ€Π°Π²Π½ΠΎ напряТСниС Π½Π° кондСнсаторС?
Π΅) Π§Π΅ΠΌΡƒ Ρ€Π°Π²Π΅Π½ заряд кондСнсатора?
ΠΆ) Каков Π·Π½Π°ΠΊ заряда ΠΎΠ±ΠΊΠ»Π°Π΄ΠΊΠΈ кондСнсатора, соСдинСнной с рСзистором 2?

Π”ΠΎΠΏΠΎΠ»Π½ΠΈΡ‚Π΅Π»ΡŒΠ½Ρ‹Π΅ вопросы ΠΈ задания

15. На рисункС 61.11 ΠΈΠ·ΠΎΠ±Ρ€Π°ΠΆΠ΅Π½Π° схСма участка элСктричСской Ρ†Π΅ΠΏΠΈ. Π‘ΠΎΠΏΡ€ΠΎΡ‚ΠΈΠ²Π»Π΅Π½ΠΈΠ΅ ΠΊΠ°ΠΆΠ΄ΠΎΠ³ΠΎ рСзистора 1 Ом. Π˜ΡΠΏΠΎΠ»ΡŒΠ·ΡƒΡ ΠΌΠ΅Ρ‚ΠΎΠ΄ эквивалСнтного прСобразования схСм:
Π°) Π½Π°Ρ‡Π΅Ρ€Ρ‚ΠΈΡ‚Π΅ схСмы ΠΏΠΎΡΠ»Π΅Π΄ΠΎΠ²Π°Ρ‚Π΅Π»ΡŒΠ½ΠΎΠ³ΠΎ упрощСния Π΄Π°Π½Π½ΠΎΠΉ схСмы, содСрТит мСньшС рСзисторов;
Π±) для ΠΊΠ°ΠΆΠ΄ΠΎΠΉ схСмы рассчитайтС Π΅Π΅ сопротивлСниС ΠΈ Π½Π°ΠΉΠ΄ΠΈΡ‚Π΅ ΠΎΠ±Ρ‰Π΅Π΅ сопротивлСниС всСго участка.

16. На схСмС участка Ρ†Π΅ΠΏΠΈ, ΠΈΠ·ΠΎΠ±Ρ€Π°ΠΆΠ΅Π½Π½ΠΎΠΉ Π½Π° рисункС 61.5, рСзисторов сопротивлСния R 1 = 20 Ом, R 2 = 100 Ом, R 3 = 10 Ом, R 4 = 50 Ом, R 5 = 80 Ом.Каково ΠΎΠ±Ρ‰Π΅Π΅ сопротивлСниС участка Ρ†Π΅ΠΏΠΈ?

17. Π‘ΠΎΠΏΡ€ΠΎΡ‚ΠΈΠ²Π»Π΅Π½ΠΈΠ΅ внСшнСй Ρ†Π΅ΠΏΠΈ Π² 4 Ρ€Π°Π·Π° большС Ρ‚ΠΎΠ³ΠΎ значСния, ΠΏΡ€ΠΈ мощности Ρ‚ΠΎΠΊΠ° Π²ΠΎ внСшнСй Ρ†Π΅ΠΏΠΈ максимальна.
Π°) Π§Π΅ΠΌΡƒ равСнство ΠšΠŸΠ” источника Ρ‚ΠΎΠΊΠ°?
Π±) Π’ΠΎ сколько Ρ€Π°Π· ΠΏΡ€ΠΈ этом ΠΌΠΎΡ‰Π½ΠΎΡΡ‚ΡŒ Ρ‚ΠΎΠΊΠ° Π²ΠΎ внСшнСй Ρ†Π΅ΠΏΠΈ мСньшС максимально Π²ΠΎΠ·ΠΌΠΎΠΆΠ½ΠΎΠΉ?

ΠŸΠΎΡΠ»Π΅Π΄ΠΎΠ²Π°Ρ‚Π΅Π»ΡŒΠ½ΠΎΠ΅ ΠΈ ΠΏΠ°Ρ€Π°Π»Π»Π΅Π»ΡŒΠ½ΠΎΠ΅ соСдинСниС рСзисторов

ΠŸΠΎΡΠ»Π΅Π΄ΠΎΠ²Π°Ρ‚Π΅Π»ΡŒΠ½ΠΎΠ΅ ΠΈ ΠΏΠ°Ρ€Π°Π»Π»Π΅Π»ΡŒΠ½ΠΎΠ΅ соСдинСниС рСзисторов

ΠŸΠΎΡΠ»Π΅Π΄ΠΎΠ²Π°Ρ‚Π΅Π»ΡŒΠ½ΠΎΠ΅ соСдинСниС рСзисторов

ΠŸΠΎΡΠ»Π΅Π΄ΠΎΠ²Π°Ρ‚Π΅Π»ΡŒΠ½ΠΎΠ΅ соСдинСниС — это соСдинСниС Π΄Π²ΡƒΡ… ΠΈΠ»ΠΈ Π±ΠΎΠ»Π΅Π΅ рСзисторов Π² ΠΎΠ΄Π½ΠΎΠΉ Ρ†Π΅ΠΏΠΈ, Π² ΠΊΠΎΡ‚ΠΎΡ€ΠΎΠΉ ΠΎΡ‚Π΄Π΅Π»ΡŒΠ½Ρ‹ΠΉ рСзистор соСдиняСтся с Π΄Ρ€ΡƒΠ³ΠΈΠΌ рСзистором Π² ΠΎΠ΄Π½ΠΎΠΉ Ρ‚ΠΎΡ‡ΠΊΠ΅.

ΠžΠ±Ρ‰Π΅Π΅ сопротивлСниС RΠΎΠ±Ρ‰

ΠŸΡ€ΠΈ Ρ‚Π°ΠΊΠΎΠΌ соСдинСнии, Ρ‡Π΅Ρ€Π΅Π· всС рСзисторы ΠΏΡ€ΠΎΡ…ΠΎΠ΄ΠΈΡ‚ ΠΎΠ΄ΠΈΠ½ ΠΈ Ρ‚ΠΎΡ‚ ΠΆΠ΅ элСктричСский Ρ‚ΠΎΠΊ. Π§Π΅ΠΌ большС элСмСнтов Π½Π° Π΄Π°Π½Π½ΠΎΠΌ участкС элСктричСской Ρ†Π΅ΠΏΠΈ, Ρ‚Π΅ΠΌ Β«Ρ‚Ρ€ΡƒΠ΄Π½Π΅Π΅Β» Ρ‚ΠΎΠΊΡƒ ΠΏΡ€ΠΎΡ‚Π΅ΠΊΠ°Ρ‚ΡŒ Ρ‡Π΅Ρ€Π΅Π· Π½Π΅Π³ΠΎ. Π‘Π»Π΅Π΄ΠΎΠ²Π°Ρ‚Π΅Π»ΡŒΠ½ΠΎ, ΠΏΡ€ΠΈ ΠΏΠΎΡΠ»Π΅Π΄ΠΎΠ²Π°Ρ‚Π΅Π»ΡŒΠ½ΠΎΠΌ соСдинСнии рСзисторов ΠΎΠ±Ρ‰Π΅Π΅ сопротивлСниС увСличиваСтся, ΠΈ ΠΎΠ½ΠΎ Ρ€Π°Π²Π½ΠΎ суммС всСх сопротивлСний.

НапряТСниС ΠΏΡ€ΠΈ ΠΏΠΎΡΠ»Π΅Π΄ΠΎΠ²Π°Ρ‚Π΅Π»ΡŒΠ½ΠΎΠΌ соСдинСнии

НапряТСниС ΠΏΡ€ΠΈ ΠΏΠΎΡΠ»Π΅Π΄ΠΎΠ²Π°Ρ‚Π΅Π»ΡŒΠ½ΠΎΠΌ соСдинСнии распрСдСляСтся Π½Π° ΠΊΠ°ΠΆΠ΄ΠΎΠΌ рСзисторС согласно Π·Π°ΠΊΠΎΠ½Ρƒ Ома:

Π’.Ρ‡Π΅ΠΌ большСС сопротивлСниС сопротивляСтся, Ρ‚Π΅ΠΌ большСС напряТСниС Π½Π° Π½Π΅Π³ΠΎ ΠΏΠ°Π΄Π°Π΅Ρ‚.

ΠŸΠ°Ρ€Π°Π»Π»Π΅Π»ΡŒΠ½ΠΎΠ΅ соСдинСниС рСзисторов

ΠŸΠ°Ρ€Π°Π»Π»Π΅Π»ΡŒΠ½ΠΎΠ΅ соСдинСниС — это соСдинСниС, ΠΏΡ€ΠΈ ΠΊΠΎΡ‚ΠΎΡ€ΠΎΠΌ рСзисторы ΡΠΎΠ΅Π΄ΠΈΠ½ΡΡŽΡ‚ΡΡ ΠΌΠ΅ΠΆΠ΄Ρƒ собой ΠΎΠ±ΠΎΠΈΠΌΠΈ ΠΊΠΎΠ½Ρ‚Π°ΠΊΡ‚Π°ΠΌΠΈ. Π’ Ρ€Π΅Π·ΡƒΠ»ΡŒΡ‚Π°Ρ‚Π΅ ΠΊ ΠΎΠ΄Π½ΠΎΠΉ Ρ‚ΠΎΡ‡ΠΊΠ΅ (элСктричСскому ΡƒΠ·Π»Ρƒ) ΠΌΠΎΠΆΠ΅Ρ‚ Π±Ρ‹Ρ‚ΡŒ присоСдинСно нСсколько рСзисторов.

ΠžΠ±Ρ‰Π΅Π΅ сопротивлСниС RΠΎΠ±Ρ‰

ΠŸΡ€ΠΈ Ρ‚Π°ΠΊΠΎΠΌ соСдинСнии, Ρ‡Π΅Ρ€Π΅Π· ΠΊΠ°ΠΆΠ΄Ρ‹ΠΉ рСзистор ΠΏΠΎΡ‚Π΅Ρ‡Π΅Ρ‚ ΠΎΡ‚Π΄Π΅Π»ΡŒΠ½Ρ‹ΠΉ Ρ‚ΠΎΠΊ. Π‘ΠΈΠ»Π° Π΄Π°Π½Π½ΠΎΠ³ΠΎ Ρ‚ΠΎΠΊΠ° Π±ΡƒΠ΄Π΅Ρ‚ ΠΎΠ±Ρ€Π°Ρ‚Π½ΠΎ ΠΏΡ€ΠΎΠΏΠΎΡ€Ρ†ΠΈΠΎΠ½Π°Π»ΡŒΠ½Π° ΡΠΎΠΏΡ€ΠΎΡ‚ΠΈΠ²Π»Π΅Π½ΠΈΡŽ рСзистора.ΠžΠ±Ρ‰Π΅Π΅ сопротивлСниС Ρ‚Π°ΠΊΠΎΠ³ΠΎ участка элСктричСской Ρ†Π΅ΠΏΠΈ увСличиваСтся.

Π’Π°ΠΊΠΈΠΌ ΠΎΠ±Ρ€Π°Π·ΠΎΠΌ, ΠΏΡ€ΠΈ ΠΏΠ°Ρ€Π°Π»Π»Π΅Π»ΡŒΠ½ΠΎΠΌ ΠΏΠΎΠ΄ΠΊΠ»ΡŽΡ‡Π΅Π½ΠΈΠΈ рСзисторов с Ρ€Π°Π·Π½Ρ‹ΠΌ сопротивлСниСм, ΠΎΠ±Ρ‰Π΅Π΅ сопротивлСниС Π±ΡƒΠ΄Π΅Ρ‚ всСгда мСньшС значСния самого малСнького ΠΎΡ‚Π΄Π΅Π»ΡŒΠ½ΠΎΠ³ΠΎ сопротивлСния.

Π€ΠΎΡ€ΠΌΡƒΠ»Π° ΠΎΠ±Ρ‰Π΅ΠΉ проводимости ΠΏΡ€ΠΈ ΠΏΠ°Ρ€Π°Π»Π»Π΅Π»ΡŒΠ½ΠΎΠΌ соСдинСнии рСзисторов:

Π€ΠΎΡ€ΠΌΡƒΠ»Π° эквивалСнтного ΠΎΠ±Ρ‰Π΅Π³ΠΎ сопротивлСния ΠΏΡ€ΠΈ ΠΏΠ°Ρ€Π°Π»Π»Π΅Π»ΡŒΠ½ΠΎΠΌ соСдинСнии рСзисторов:

Для Π΄Π²ΡƒΡ… ΠΎΠ΄ΠΈΠ½Π°ΠΊΠΎΠ²Ρ‹Ρ… рСзисторов ΠΎΠ±Ρ‰Π΅Π΅ сопротивлСниС Π±ΡƒΠ΄Π΅Ρ‚ Ρ€Π°Π²Π½ΠΎ ΠΎΠ΄Π½ΠΎΠ³ΠΎ ΠΎΡ‚Π΄Π΅Π»ΡŒΠ½ΠΎΠ³ΠΎ рСзистора:

БоотвСтствСнно, для ΠΎΠ΄ΠΈΠ½Π°ΠΊΠΎΠ²Ρ‹Ρ… рСзисторов ΠΎΠ±Ρ‰Π΅Π΅ сопротивлСниС Π±ΡƒΠ΄Π΅Ρ‚ Ρ€Π°Π²Π½ΠΎ Π·Π½Π°Ρ‡Π΅Π½ΠΈΡŽ ΠΎΠ΄Π½ΠΎΠ³ΠΎ рСзистора, Ρ€Π°Π·Π΄Π΅Π»Π΅Π½Π½ΠΎΠ³ΠΎ Π½Π° n.

НапряТСниС ΠΏΡ€ΠΈ ΠΏΠ°Ρ€Π°Π»Π»Π΅Π»ΡŒΠ½ΠΎΠΌ соСдинСнии

НапряТСниС ΠΌΠ΅ΠΆΠ΄Ρƒ Ρ‚ΠΎΡ‡ΠΊΠ°ΠΌΠΈ A ΠΈ B являСтся ΠΎΠ±Ρ‰ΠΈΠΌ напряТСниСм для всСго участка Ρ†Π΅ΠΏΠΈ, Ρ‚Π°ΠΊ ΠΈ напряТСниСм, ΠΏΠ°Π΄Π°ΡŽΡ‰ΠΈΠΌ Π½Π° ΠΊΠ°ΠΆΠ΄Ρ‹ΠΉ рСзистор Π² ΠΎΡ‚Π΄Π΅Π»ΡŒΠ½ΠΎΡΡ‚ΠΈ. ΠŸΠΎΡΡ‚ΠΎΠΌΡƒ ΠΏΡ€ΠΈ ΠΏΠ°Ρ€Π°Π»Π»Π΅Π»ΡŒΠ½ΠΎΠΌ соСдинСнии Π½Π° всС рСзисторы ΡƒΠΏΠ°Π΄Π΅Ρ‚ ΠΎΠ΄ΠΈΠ½Π°ΠΊΠΎΠ²ΠΎΠ΅ напряТСниС.

ЭлСктричСский Ρ‚ΠΎΠΊ ΠΏΡ€ΠΈ ΠΏΠ°Ρ€Π°Π»Π»Π΅Π»ΡŒΠ½ΠΎΠΌ соСдинСнии

Π§Π΅Ρ€Π΅Π· ΠΊΠ°ΠΆΠ΄Ρ‹ΠΉ рСзистор Ρ‚Π΅Ρ‡Π΅Ρ‚ Ρ‚ΠΎΠΊ, сила ΠΊΠΎΡ‚ΠΎΡ€ΠΎΠ³ΠΎ ΠΎΠ±Ρ€Π°Ρ‚Π½ΠΎ ΠΏΡ€ΠΎΠΏΠΎΡ€Ρ†ΠΈΠΎΠ½Π°Π»ΡŒΠ½Π° ΡΠΎΠΏΡ€ΠΎΡ‚ΠΈΠ²Π»Π΅Π½ΠΈΡŽ рСзистора. Для Ρ‚ΠΎΠ³ΠΎ, Ρ‡Ρ‚ΠΎΠ±Ρ‹ ΡƒΠ·Π½Π°Ρ‚ΡŒ ΠΊΠ°ΠΊΠΎΠΉ Ρ‚ΠΎΠΊ Ρ‚Π΅Ρ‡Π΅Ρ‚ Ρ‡Π΅Ρ€Π΅Π· ΠΎΠΏΡ€Π΅Π΄Π΅Π»Π΅Π½Π½Ρ‹ΠΉ рСзистор, ΠΌΠΎΠΆΠ½ΠΎ Π²ΠΎΡΠΏΠΎΠ»ΡŒΠ·ΠΎΠ²Π°Ρ‚ΡŒΡΡ Π·Π°ΠΊΠΎΠ½ΠΎΠΌ Ома:

БмСшанноС соСдинСниС рСзисторов

Π‘ΠΌΠ΅ΡˆΠ°Π½Π½Ρ‹ΠΌ соСдинСниСм Π½Π°Π·Ρ‹Π²Π°ΡŽΡ‚ участок Ρ†Π΅ΠΏΠΈ, Π³Π΄Π΅ Ρ‡Π°ΡΡ‚ΡŒ рСзисторов соСдиняСтся ΠΌΠ΅ΠΆΠ΄Ρƒ собой, Π° Ρ‡Π°ΡΡ‚ΡŒ ΠΏΠ°Ρ€Π°Π»Π»Π΅Π»ΡŒΠ½ΠΎ.Π’ свою ΠΎΡ‡Π΅Ρ€Π΅Π΄ΡŒ, смСшанноС соСдинСниС Π±Ρ‹Π²Π°Π΅Ρ‚ ΠΏΠΎΡΠ»Π΅Π΄ΠΎΠ²Π°Ρ‚Π΅Π»ΡŒΠ½ΠΎΠ³ΠΎ ΠΈ ΠΏΠ°Ρ€Π°Π»Π»Π΅Π»ΡŒΠ½ΠΎΠ³ΠΎ Ρ‚ΠΈΠΏΠΎΠ².

ΠžΠ±Ρ‰Π΅Π΅ сопротивлСниС RΠΎΠ±Ρ‰

Для Ρ‚ΠΎΠ³ΠΎ Ρ‡Ρ‚ΠΎΠ±Ρ‹ ΠΏΠΎΡΡ‡ΠΈΡ‚Π°Ρ‚ΡŒ ΠΎΠ±Ρ‰Π΅Π΅ сопротивлСниС смСшанного соСдинСния:

o ЦСпь Ρ€Π°Π·Π±ΠΈΠ²Π°ΡŽΡ‚ Π½Π° участки с Ρ‚ΠΎΠ»ΡŒΠΊΠΎ ΠΏΠ°Ρ€Π΅Π»Π»Π΅Π»ΡŒΠ½Ρ‹ΠΌ ΠΈΠ»ΠΈ Ρ‚ΠΎΠ»ΡŒΠΊΠΎ ΠΏΠΎΡΠ»Π΅Π΄ΠΎΠ²Π°Ρ‚Π΅Π»ΡŒΠ½Ρ‹ΠΌ соСдинСниСм. o Π’Ρ‹Ρ‡ΠΈΡΠ»ΠΈΡ‚Π΅Π»ΡŒΠ½ΠΎΠ΅ сопротивлСниС для ΠΊΠ°ΠΆΠ΄ΠΎΠ³ΠΎ ΠΎΡ‚Π΄Π΅Π»ΡŒΠ½ΠΎΠ³ΠΎ участка.

o ВычислСнноС ΠΎΠ±Ρ‰Π΅Π΅ сопротивлСниС для всСй Ρ†Π΅ΠΏΠΈ смСшанного соСдинСния.

Π’Π°ΠΊ это Π±ΡƒΠ΄Π΅Ρ‚ Π²Ρ‹Π³Π»ΡΠ΄Π΅Ρ‚ΡŒ для схСмы 1:

Π’Π°ΠΊΠΆΠ΅ сущСствуСт Π±ΠΎΠ»Π΅Π΅ быстрый способ расчСта ΠΎΠ±Ρ‰Π΅Π³ΠΎ сопротивлСния для смСшанного соСдинСния.МоТно, Π² соотвСтствии, сразу Π·Π°ΠΏΠΈΡΡ‹Π²Π°Ρ‚ΡŒ Ρ„ΠΎΡ€ΠΌΡƒΠ»Ρƒ ΡΠ»Π΅Π΄ΡƒΡŽΡ‰ΠΈΠΌ ΠΎΠ±Ρ€Π°Π·ΠΎΠΌ:

o Если рСзисторы ΡΠΎΠ΅Π΄ΠΈΠ½ΡΡŽΡ‚ΡΡ ΠΏΠΎΡΠ»Π΅Π΄ΠΎΠ²Π°Ρ‚Π΅ΡŒΠ½ΠΎ — ΡΠΊΠ»Π°Π΄Ρ‹Π²Π°Ρ‚ΡŒ.

o Если рСзисторы ΡΠΎΠ΅Π΄ΠΈΠ½ΡΡŽΡ‚ ΠΏΠ°Ρ€Π°Π»Π»Π΅Π»ΡŒΠ½ΠΎ — ΠΈΡΠΏΠΎΠ»ΡŒΠ·ΠΎΠ²Π°Ρ‚ΡŒ условноС ΠΎΠ±ΠΎΠ·Π½Π°Ρ‡Π΅Π½ΠΈΠ΅ «||».

Π”ΠΎΠ±Π°Π²ΠΈΡ‚ΡŒ ΠΊΠΎΠΌΠΌΠ΅Π½Ρ‚Π°Ρ€ΠΈΠΉ

Π’Π°Ρˆ адрСс email Π½Π΅ Π±ΡƒΠ΄Π΅Ρ‚ ΠΎΠΏΡƒΠ±Π»ΠΈΠΊΠΎΠ²Π°Π½. ΠžΠ±ΡΠ·Π°Ρ‚Π΅Π»ΡŒΠ½Ρ‹Π΅ поля ΠΏΠΎΠΌΠ΅Ρ‡Π΅Π½Ρ‹ *